You are on page 1of 752

tr

PREVIOUS YEARS’ SOLVED QUESTION PAPERS Also Helpful for GAIL, BARC, HPCL, BHEL, ONGC, SAIL, DRDO & Other PSU’s

ish
ELECTRICAL ENGINEERING

na
G R AD UAT E A PT I T UD E T E S T I N E NG I N EE R I N G
SERIES
2019

’s
This book is a one-stop solution for GATE aspirants to crack the exam. The book includes previous years’ GATE questions
segregated topic-wise along with exam analysis at the beginning of every unit. Detailed step-wise solutions are

PREVIOUS YEARS’ SOLVED


provided for all the questions. It will help the students to get an idea about the pattern and weightage of questions
appeared in GATE exam.

WHAT THE FACULTIES SAY ABOUT THIS BOOK...

QUESTION PAPERS
The approach used in this book is direct and application oriented which helps students to learn problem-solving skills for
GATE exam. The language used is simple and appropriate. The pedagogy used in this book is simple as it is based on
problem-solving skill based on theoretical foundations. This makes it a useful book for faculty as well as student.
Dr. Madan Mohan Tripathi, Associate Professor, Delhi Technological University, New Delhi
The book is very well written and language used is simple. The analysis given at beginning of every unit for GATE
previous year questions will be helpful for students to get an idea about the GATE pattern. The solutions given for
GATE questions are detailed and self explanatory. PREVIOUS YEARS’
Dr. S.K. Mydhili, Assistant Professor (Senior Grade), SVS College of Engineering, Coimbatore,Tamil Nadu SOLVED QUESTION PAPERS

ELECTRICAL
The language used is simple and easy to understand and the approach used is excellent. The book will be extremely
useful for the GATE aspirants and faculties.
Dr. Umesh Kumar, Associate Professor, IIT Delhi, Hauz Khas, New Delhi

ENGINEERING

ELECTRICAL
ENGINEERING
2019

C r ack th e
crack the gate series
HIGHLIGHTS
Includes more than 28 years’ GATE questions
arranged chapter-wise
Detailed solutions for better understanding
Includes latest GATE solved question papers with detailed analysis

in.pearson.com
2019
About Pearson
Pearson is the world’s learning company, with presence across 70 countries
worldwide. Our unique insights and world-class expertise comes from a long
history of working closely with renowned teachers, authors and thought
leaders, as a result of which, we have emerged as the preferred choice for
millions of teachers and learners across the world.
We believe learning opens up opportunities, creates fulfilling careers and
hence better lives. We hence collaborate with the best of minds to deliver you
class-leading products, spread across the Higher Education and K12 spectrum.
Superior learning experience and improved outcomes are at the heart of
everything we do. This product is the result of one such effort.
Your feedback plays a critical role in the evolution of our products and you
can contact us - reachus@pearson.com. We look forward to it.

F01_Gate_EE_978-93-325-7605-6_FM.indd 1 6/19/2017 7:22:36 PM


This page is intentionally left blank.

F01_Gate_EE_978-93-325-7605-6_FM.indd 2 6/19/2017 7:22:36 PM


GATE
Previous Years’
Solved Question Papers
Electrical Engineering

F01_Gate_EE_978-93-325-7605-6_FM.indd 3 6/19/2017 7:22:36 PM


Copyright © 2018 Pearson India Education Services Pvt. Ltd

Published by Pearson India Education Services Pvt. Ltd, CIN: U72200TN2005PTC057128.

No part of this eBook may be used or reproduced in any manner whatsoever without the publisher’s
prior written consent.

This eBook may or may not include all assets that were part of the print version. The publisher
reserves the right to remove any material in this eBook at any time.

ISBN: 978-93-530-6129-6
eISBN
Head Office: 15th Floor, Tower-B, World Trade Tower, Plot No. 1, Block-C, Sector 16, Noida 201 301,
Uttar Pradesh, India.
Registered Office: 4th Floor, Software Block, Elnet Software City, TS-140, Block 2 & 9, Rajiv
Gandhi Salai, Taramani, Chennai 600 113, Tamil Nadu, India.
Fax: 080-30461003, Phone: 080-30461060
Website: in.pearson.com, Email: companysecretary.india@pearson.com

F01_Gate_EE_978-93-325-7605-6_FM.indd 4 6/19/2017 7:22:36 PM


Contents
Preface ix

Syllabus: Electrical Engineering xi

Important Tips for GATE Preparation xiii

Detailed Analysis of GATE 2018 Papers xiv-a

Solved Papers 2018 xiv-b

Detailed Analysis of GATE 2017 Papers xv

Solved Papers 2017 xvi

Detailed Analysis of GATE 2016 Papers lxxix

Solved Papers 2016 lxxxiii

Detailed Analysis of GATE 2015 Papers cxxv

Solved Papers 2015 cxxviii

Unit 1:  Electric Circuits and Fields 1.1


Chapter 1:  Network Elements 1.3
Chapter 2:  Network Graph 1.22
Chapter 3:  Network Theorems 1.24
Chapter 4:  Transient Response 1.33
Chapter 5:  Sinusoidal Steady State Analysis 1.44
Chapter 6:  Three-phase Circuits 1.60
Chapter 7:  Two Port Networks 1.64
Chapter 8:  Miscellaneous 1.69
Chapter 9:  Fields 1.76

Unit 2:  Signals and Systems 2.1


Chapter 1:  CT and DT Signals 2.3
Chapter 2:  LTI Systems (CT and DT)  2.7
Chapter 3:  Periodic Signal: Fourier Series 2.24
Chapter 4:  Signal Fourier Transform 2.32
Chapter 5:  Sampling Theorem 2.35
Chapter 6:  Signal: Laplace Transform 2.37
Chapter 7:  DT Signal: Z-Transform 2.40
Chapter 8:  Miscellaneous 2.42

F01_Gate_EE_978-93-325-7605-6_FM.indd 5 6/19/2017 7:22:36 PM


vi | Contents

Unit 3:  Electric Machines 3.1


Chapter 1:  DC Machines 3.3
Chapter 2:  Transformers 3.18
Chapter 3:  Induction Motors 3.44
Chapter 4:  Synchronous Machines 3.67

Unit 4:  Power Systems 4.1


Chapter 1:  Transmission and Distribution 4.3
Chapter 2:  Economics of Power Generation 4.28
Chapter 3:  Symmetrical Components and Faults 4.34
Chapter 4:  Power System Stability 4.48
Chapter 5:  Protection 4.58
Chapter 6:  Circuit Breaker 4.65
Chapter 7:  Generating Stations 4.68
Chapter 8:  Load Flows 4.70
Chapter 9:  HVDC 4.78
Chapter 10:  Per Unit System 4.80

Unit 5:  Control Systems 5.1


Chapter 1:  Basic Control Systems 5.3
Chapter 2:  Signal Flow Graph and Block Diagram 5.6
Chapter 3:  Stability 5.9
Chapter 4:  Time Response Analysis 5.14
Chapter 5:  Root Locus Diagram 5.26
Chapter 6:  Frequency Response Analysis 5.32
Chapter 7:  Controllers (or) Compensators 5.48
Chapter 8:  State Space Analysis 5.51

Unit 6:  Electrical and Electronics Measurements 6.1


Chapter 1:  Fundamentals and Error Analysis 6.3
Chapter 2:  Indicating Instruments 6.5
Chapter 3:  Measurement of Voltage and Current 6.7
Chapter 4:  Measurement of Resistance 6.15
Chapter 5:  Bridge Measurement, Measurement of Induction and Capacitance 6.17
Chapter 6:  Extension of Instrument Ranges 6.21

F01_Gate_EE_978-93-325-7605-6_FM.indd 6 6/19/2017 7:22:37 PM


Contents | vii

Chapter 7:  Potentiometer 6.22


Chapter 8:  Measurement of Power and Energy 6.23
Chapter 9:  Measurement of Phase and Frequency 6.32
Chapter 10:  Potentiometric Recorders and Q-Meter 6.34
Chapter 11:  Oscilloscope 6.36
Chapter 12:  Electronic Multimeters and Digital Voltmeters 6.41
Chapter 13:  Instrument Transformer 6.43

Unit 7:  Digital Electronics and Microprocessors 7.1


Chapter 1:  Number Systems and Code Conversions 7.3
Chapter 2:  Boolean Algebra 7.4
Chapter 3:  Logic Gates 7.5
Chapter 4:  Combinational Digital Circuits 7.9
Chapter 5:  Sequential Digital Circuits 7.14
Chapter 6:  Semiconductor Memories 7.20
Chapter 7:  Logic Gate Families 7.21
Chapter 8:  A/D and D/A Converters 7.23
Chapter 9:  Microprocessors 7.26

Unit 8:  Analog Electronics 8.1


Chapter 1:  Diode Circuit 8.3
Chapter 2:  BJT and FET Biasing  8.13
Chapter 3:  Small Signal Modeling and Analysis 8.23
Chapter 4:  Frequency Response 8.28
Chapter 5:  Operational Amplifiers and Applications  8.29
Chapter 6:  Feedback Amplifiers and Oscillators  8.48
Chapter 7:  Function Generator and 555 Timer  8.54

Unit 9:  Power Electronics and Drives 9.1


Chapter 1:  Basics and Power Semiconductor Devices 9.3
Chapter 2:  AC–DC Converters 9.12
Chapter 3:  DC–DC Converters 9.25
Chapter 4:  DC–AC Converters 9.35
Chapter 5:  AC–AC Converters 9.42
Chapter 6:  Fundamentals of Drives 9.45

F01_Gate_EE_978-93-325-7605-6_FM.indd 7 6/19/2017 7:22:37 PM


This page is intentionally left blank.

F01_Gate_EE_978-93-325-7605-6_FM.indd 8 6/19/2017 7:22:37 PM


Preface
Graduate Aptitude Test in Engineering (GATE) is one of the primarily tests for various undergraduate subjects—
Engineering/Technology/Architecture and postgraduate level for Science. The GATE examination pattern has
undergone several changes over the years—sometimes apparent and sometimes subtle. It is bound to continue to
do so with changing technological environment.
GATE Previous Years’ Solved Question Papers for Electrical Engineering acts as a practice material for GATE
aspirants to strengthen their conceptual understanding and application skills. The book includes more than 23
years, GATE questions segregated topic-wise along with exam analysis which is provided at the beginning of
every unit. This book helps the GATE aspirants to get an idea about the pattern and weightage of questions asked
in GATE examination.
Owing to multifaceted opportunities open to any good performer, the number of aspirants appearing for the
GATE examination is increasing significantly every year. Apart from giving the aspirant a chance to pursue an
M.Tech. from institutions such as the IITs /NITs, a good GATE score can be highly instrumental in landing
the candidate a plush public sector job, as many PSUs are recruiting graduate engineers on the basis of their
performance in GATE.

Salient Features
☞☞ Includes more than 23 years’ GATE questions arranged chapter-wise.
☞☞ Detailed solutions for better understanding.
☞☞ Includes latest GATE solved question papers with detailed analysis.
☞☞ Includes free online mock test based on GATE examination pattern for practice.
Despite of our best efforts, some errors may have inadvertently crept into the book. Constructive comments and
suggestions to further improve the book are welcome and shall be acknowledged gratefully.

F01_Gate_EE_978-93-325-7605-6_FM.indd 9 6/19/2017 7:22:37 PM


This page is intentionally left blank.

F01_Gate_EE_978-93-325-7605-6_FM.indd 10 6/19/2017 7:22:37 PM


Syllabus: Electrical Engineering
Electric Circuits
Network graph, KCL, KVL, Node and Mesh analysis, Transient response of dc and ac networks, Sinusoidal
steady state analysis, Resonance, Passive filters, Ideal current and voltage sources, Thevenin’s theorem, Norton’s
theorem, Superposition theorem, Maximum power transfer theorem, Twoport networks, Three phase circuits,
Power and power factor in ac circuits.

Electromagnetic Fields
Coulomb’s Law, Electric Field Intensity, Electric Flux Density, Gauss’s Law, Divergence, Electric field and potential
due to point, line, plane and spherical charge distributions, Effect of dielectric medium, Capacitance of simple
configurations, Biot Savart’s law, Ampere’s law, Curl, Faraday’s law, Lorentz force, Inductance, Magnetomotive
force, Reluctance, Magnetic circuits, Self and Mutual inductance of simple configurations.

Signals and Systems


Representation of continuous and discrete time signals, Shifting and scaling operations, Linear Time Invariant and
Causal systems, Fourier series representation of continuous periodic signals, Sampling theorem, Applications of
Fourier Transform, Laplace Transform and z-Transform.

Electrical Machines
Single phase transformer: equivalent circuit, phasor diagram, open circuit and short circuit tests, regulation
and efficiency; Three phase transformers: connections, parallel operation; Auto transformer, Electromechanical
energy conversion principles, DC machines: separately excited, series and shunt, motoring and generating mode
of operation and their characteristics, starting and speed control of dc motors; Three phase induction motors:
principle of operation, types, performance, torque-speed characteristics, no-load and blocked rotor tests, equivalent
circuit, starting and speed control; Operating principle of single phase induction motors; Synchronous machines:
cylindrical and salient pole machines, performance, regulation and parallel operation of generators, starting of
synchronous motor, characteristics; Types of losses and efficiency calculations of electric machines.

Power Systems
Power generation concepts, ac and dc transmission concepts, Models and performance of transmission lines and
cables, Series and shunt compensation, Electric field distribution and insulators, Distribution systems, Perunit
quantities, Bus admittance matrix, GaussSeidel and Newton-Raphson load flow methods, Voltage and Frequency
control, Power factor correction, Symmetrical components, Symmetrical and unsymmetrical fault analysis,
Principles of over current, differential and distance protection; Circuit breakers, System stability concepts, Equal
area criterion.

Control Systems
Mathematical modeling and representation of systems, Feedback principle, transfer function, Block diagrams
and Signal flow graphs, Transient and Steady state analysis of linear time invariant systems, Routh-Hurwitz and
Nyquist criteria, Bode plots, Root loci, Stability analysis, Lag, Lead and Lead Lag compensators; P, PI and PID
controllers; State space model, State transition matrix.

F01_Gate_EE_978-93-325-7605-6_FM.indd 11 6/19/2017 7:22:37 PM


xii | Syllabus: Electrical Engineering

Electrical and Electronic Measurements


Bridges and Potentiometers, Measurement of voltage, current, power, energy and power factor; Instrument
transformers, Digital voltmeters and multimeters, Phase, Time and Frequency measurement; Oscilloscopes, Error
analysis.

Analog and Digital Electronics


Characteristics of diodes, BJT, MOSFET; Simple diode circuits: clipping, clamping, rectifiers; Amplifiers:
Biasing, Equivalent circuit and Frequency response; Oscillators and Feedback amplifiers; Operational amplifiers:
Characteristics and applications; Simple active filters, VCOs and Timers, Combinational and Sequential logic
circuits, Multiplexer, Demultiplexer, Schmitt trigger, Sample and hold circuits, A/D and D/A converters,
8085Microprocessor: Architecture, Programming and Interfacing.

Power Electronics
Characteristics of semiconductor power devices: Diode, Thyristor, Triac, GTO, MOSFET, IGBT; DC to DC
conversion: Buck, Boost and Buck-Boost converters; Single and three phase configuration of uncontrolled
rectifiers, Line commutated thyristor based converters, Bidirectional ac to dc voltage source converters, Issues
of line current harmonics, Power factor, Distortion factor of ac to dc converters, Single phase and three phase
inverters, Sinusoidal pulse width modulation.

F01_Gate_EE_978-93-325-7605-6_FM.indd 12 6/19/2017 7:22:37 PM


Important Tips for GATE
Preparation
The followings are some important tips which would be helpful for students to prepare for GATE exam
1. Go through the pattern (using previous years’ GATE paper) and syllabus of the exam and start preparing
accordingly.
2. Preparation time for GATE depends on many factors, such as, individual’s aptitude, attitude, fundamentals,
concentration level etc., Generally rigorous preparation for 4 to 6 months is considered good but it may vary
from student to student.
3. Make a list of books which cover complete syllabus, contains solved previous years’ questions and mock
tests for practice based on latest GATE pattern. Purchase these books and start your preparation.
4. Make a list of topics which needs to be studied and make priority list for study of every topic based upon the
marks for which that particular topic is asked in GATE exam. Find out the topics which fetch more marks
and give more importance to those topics. Make a timetable for study of topics and follow the timetable
strictly.
5. An effective way to brush up your knowledge about technical topics is group study with your friends.
During group study you can explore new techniques and procedures.
6. While preparing any subject highlight important points (key definitions, equations, derivations, theorems
and laws) which can be revised during last minute preparation.
7. Pay equal attention to both theory and numerical problems. Solve questions (numerical) based on latest
exam pattern as much as possible, keeping weightage of that topic in mind. Whatever topics you decide to
study, make sure that you know everything about it.
8. Try to use short-cut methods to solve problems instead of traditional lengthy and time consuming methods.
9. Go through previous year papers (say last ten years), to check your knowledge and note the distribution
of different topics. Also analyze the topics in which you are weak and concentrate more on those topics.
Always try to solve papers in given time, to obtain an idea how many questions you are able to solve in the
given time limit.
10. Finish the detail study of topics one and a half month before your exam. During last month, revise all the
topics once again and clear leftover doubts.

F01_Gate_EE_978-93-325-7605-6_FM.indd 13 6/19/2017 7:22:37 PM


Detailed Analysis of GATE 2018 Papers
GATE EE Solved 2018 Paper (Set 1) Detailed Analysis
1 Mark 2 Mark Total
Subject Questions Questions Marks

General Aptitude 5 5 15
Engineering Maths 5 3 11
Electric Circuits and Fields 4 6 16
Signals and Systems 1 5 11
Electrical Machines 3 4 11
Power Systems 4 3 10
Control Systems 2 2 6
Electrical and Electronic Measurements 1 1 3
Analog and Digital Electronics 2 3 8
Power Electronics and Drives 3 3 9
Total 100

F01_Gate_EE_FM.indd 1 6/18/2018 5:11:43 PM


GATE 2018 Solved Paper
Electrical Engineering
Set – 1
Number of Questions: 65 Total Marks:100.0

Wrong answer for MCQ will result in negative marks, (-1/3) for 1 Mark Questions and (-2/3) for 2 Marks Questions.

General Aptitude
Number of Questions: 10  Section Marks: 15.0

Q.1 to Q.5 carry 1 mark each and Q.6 to Q.10 carry (A) 2 (B) 4
2 marks each. (C) 6 (D) 36
Question Number: 1 Question Type: MCQ Solution:  F(a, b) = (a – b)2,
The three roots of the equation f (x) = 0 are x = {–2, 0, 3}. F(1, 3) = (3 - 1)2= 4
What are the three values of x for which f (x – 3) = 0? G(a, b) = a − b
(A) –5, –3, 0 (B) –2, 0, 3
(C) 0, 6, 8 (D) 1, 3, 6 G(1, 3) = 2
∴ G(4,2) = 2.
Solution:  f (x) = 0 for x = –2, 0 and 3.
Hence, the correct option is (A)
∴ f (x – 3) = 0
⇒ x – 3 = –2, 0 or 3 Question Number: 4 Question Type: MCQ
⇒ x = 1, 3 or 6. “Since you have gone off the _____, the _____ sand is
likely to damage the car.” The words that best fill the blanks
Hence, the correct option is (D)
in the above sentence are
Question Number: 2 Question Type: MCQ (A) course, coarse (B) course, course

For what values of k given below is


(k + 2)2 an integer? (C) coarse, course (D) coarse, coarse
k −3 Solution: 
(A) 4, 8, 18 (B) 4, 10, 16 Hence, the correct option is (A)
(C) 4, 8, 28 (D) 8, 26, 28
Question Number: 5 Question Type: MCQ
( k + 2) 2 k 2 + 4k + 4 “A common misconception among writers is that sentence
Solution:  =
k −3 k −3 structure mirrors though; the more ______ the structure,
the more complicated the ideas.”
k 2 − 3k + 7k − 21 + 25 (A) detailed (B) simple
=
k −3 (C) clear (D) convoluted
25 Solution: 
=k+7+
k −3
Hence, the correct option is (D)
– 3 has to be a factor of 25.
K
Question Number: 6 Question Type: MCQ
∴ K – 3 = 1, 5, 25 or –1, –5, –25
A class of twelve children has two more boys than girls. A
i.e., K = 4, 8, 28 or 2, –2, –22. Among the options only 4, 8,
group of three children are randomly picked from this class
28 occur.
to accompany the teacher on a field trip. What is the prob-
Hence, the correct option is (C) ability that the group accompanying the teacher contains
Question Number: 3 Question Type: MCQ more girls than boys?
325
Functions F(a, b) and G(a, b) are defined as follows: (A) 0 (B)
864
F(a, b) – (a – b)2 and G(a, b) = |a – b|, where |x| repre-
525 5
sents the absolute value of x. What would be the value of (C) (D)
G(F(1, 3), G(1, 3))? 864 12

M01_GATE EE 2018 paper 1.indd 2 6/18/2018 5:13:01 PM


GATE 2018 Solved Paper Electrical Engineering: Set – I  |  xiv-c

Solution:  Let the number of boys be b and number of girls (A) 35.75 (B) 40.25
be g, (C) 43.75 (D) 46.25
As per problem Solution:  If we assume the total number of marbles be
b = g + 2 (1) 100n. Then the number of blue, black, red, yellow marbles
and also will be 40n, 25n, 20n, 15n.
5
b + g = 12 (2) The price of each marble increased by 25% (to its origi-
4
Solving (1) and (2) we nal value.) Therefore, the number of marbles has to reduce
∴ Number of boys b = 7 4
to so that the cost remains unchanged. It has to be 80n,
  Number of girls g = 5 5
7 i.e., it has to reduce by 20n. As the number reduced for all
probability of selecting a boy Pb = the colors are equal, the number in each color has to reduce
12
by 5n.
5
probability of selecting a girl Pg= The number of blue, black, red, yellow marbles in the new
12 design are 35n, 20n, 15n, 10n. The percentage of blue mar-
Assume that three students are selected randomly one after bles in this new design is
another with replacement. The favorable cases that the
35/35 + 20 + 15 + 10, i.e., 7/16, which is 43.75%
group consists girls more than boys is
Hence, the correct option is (C)
(i) all are girls
(ii) Two girls and one boy Question Number: 8 Question Type: MCQ
5 5 5 125 P, Q, R and S crossed a lake in a boat that can hold a maxi-
Case I: The probability that all are girls is . . = mum of two persons, with only one set of oars. The follow-
12 12 12 1728
ing additional facts are available.
Case II: The probability that two girls and one boy in the
  (i) The boat held two persons on each of the three forward
5 5 7
group is x x trips across the lake and one person on each of the two
12 12 12 trips.
5 5 7 (ii) P is unable to row when someone else is in the boat.
∴ The probability = 3 x x x
12 12 12 (iii) Q is unable to row with anyone else except R.
525    (iv) Each person rowed for at least one trip.
=
1728   (v) Only one person can row during a trip.
125 525 Who rowed twice?
Required probability = +
1728 1728 (A) P (B) Q
650 325 (C) R (D) S
= =
1728 864 Solution:  On the first trip Q and R will travel, with Q row-
Hence, the correct option is (B) ing the boat. R will return alone and take P along with him.
R will row the boat this time as P can not row when come
Question Number: 7 Question Type: MCQ one is with him. P alone will come back and take S along
A designer uses marbles of four different colours for his with him. S will row the boat this time. Only R rowed the
designs. The cost of each marble is the same irrespective of boat twice.
the colour. The table below shows the percentage of mar- Hence, the correct option is (C)
bles of each colour used in the current design. The cost
of each marble increased by 25%. Therefore, the designer Question Number: 9 Question Type: MCQ
decided to reduce equal number of marbles of each colour An e – mail password must contain three characters. The
to keep the total cost unchanged. What is the percentage of password has to contain one numeral from 0 to 9, one
blue marbles in the new design? upper case and one lower case character from the English
Blue Black Red Yellow ­alphabet. How many distinct passwords are possible?
(A) 6,760 (B) 13,520
40% 25% 20% 15%
(C) 40,560 (D) 1,05,456

M01_GATE EE 2018 paper 1.indd 3 6/18/2018 5:13:03 PM


A M C F

+4 +4 +4 +4

xiv-d  |  GATE 2018 Solved Paper Electrical Engineering: Set – I E Q G J

Solution: 
N K U F
∴ Number of passwords = 10(26) (26) (6)
 = 40560. +4 +4 +4 +4
Hence, the correct option is (C)
Question Number: 10 Question Type: MCQ R O Y J

In a certain code. AMCF is written as EQGJ and NKUF is So the code for the given word will be:
written as ROYJ. How will DHLP be written in that code?
D H L P
(A) RSTN (B) TLPH
(C) HLPT (D) XSVR
+4 +4 +4 +4
Solution:  The code for the given words will be
A M C F H L P T

+4 +4 +4 +4
Hence, the correct option is (C)

E Q G J

N K U F

+4 +4 +4 Electrical Engineering
+4
Number of Questions: 55 Section Marks: 85.0
R 1 mark
Q.11 to Q.25 carry O eachYand Q.26
J to Q.65 carry Solution:  We know that
2 marks each. Ev v2 ⎡ 1 1⎤
P= Sinδ + ⎢ − ⎥ sin (2δ )
Question Number: 11 Question Type: MCQ XS 2 ⎢⎣ xq xd ⎦⎥
A single – phase 100 kVA, 1000 V/100 V, 50 Hz trans-
v2 ⎡1 1⎤
former has a voltage drop of 5% across its series imped- ∴ Preluctance = ⎢ − ⎥ sin (2δ )
ance at full load. Of this, 3% is due to resistance. The 2 ⎢⎣ xq xd ⎦⎥
percentage regulation of the transformer at full load with
I f δ = 45° then Preluctance is maximum.
0.8 lagging power factor is
(A) 4.8 (B) 6.8 Hence, the correct option is (B)
(C) 8.8 (D) 10.8 Question Number: 13 Question Type: MCQ
Solution:  A single phase fully controlled rectifier is supplying a load
∴% X = (% Z ) − (% R) 2 = 52 − 32 = 4% with an anti – parallel diode as shown in the figure. All
2

switches and diodes are ideal. Which one of the following


∴ % voltage regulation
is true for instantaneous load voltage and current?
= % R(cos f2) + % x (sin f2)
 (for lagging P.F) io +
L
= 3 × 0.8 + 4 × 0.6 O
A
D
= 4.8% –

Hence, the correct option is (A)


v0 ≤ 0 & i0 < 0
(A) (B) v0 < & i0 < 0
Question Number: 12 Question Type: MCQ v0 ≥ 0 & i0 ≥ 0
(C) (D) v0 < 0 & i0 ≥ 0
In a salient pole synchronous motor, the developed reluc- Solution:  No negative ripple appear in the output because
tance torque attains the maximum value when the load the freewheeling diode is connected at output section,
angle in electrical degrees is therefore
(A) 0 (B) 45
∴ V0 ≥ 0
(C) 60 (D) 90

M01_GATE EE 2018 paper 1.indd 4 6/18/2018 5:13:05 PM


GATE 2018 Solved Paper Electrical Engineering: Set – I  |  xiv-e

urrent flows only from Anode to Cathode because the


C are the series impedance and reactance of the line respec-
given bridge is Thyristor based bridge. tively. The steady – state stability limit for the transmission
∴ i0 ≥ 0 line will be
Hence, the correct option is (C) VV VV
(A) greater than 1 2 (B) less than 1 2
X X
Question Number: 14 Question Type: MCQ
Four power semiconductor devices are shown in the figure V1V2 V1V2
(C) equal to (D) equal to
along with their relevant terminals. The device(s) that can X X 13.5
carry dc current continuously in the direction shown when
Solution: 
gated appropriately is (are)
A MT1 A D
⎧⎪ Vs Vr A 2
⎫⎪
Pr= ⎨ cos ( β − δ) − Vr cos ( β − α)⎬ MW(1)
G ⎪⎩ B B ⎪⎭
G K G K
G MT2
S I f Resistance of the transmission line = 0
I I I I
β = 90°
Thyristor Triac GTO MOSFET
Vs Vr
(A) Triac only (B) Triac and MOSFET Pr = sinδ,
X
(C) Triac and GTO (D) Thyristor and Triac
Vs Vr
Solution:  only TRIAC allow bidirectional current flow. Prmax = (2)
X
Hence, the correct option is (A)
( 1) is always less than (2).
Question Number: 15 Question Type: MCQ Hence, the correct option is (B)
Two wattmeter method is used for measurement of power
in a balanced three – phase load supplied from a balanced Question Number: 17 Question Type: MCQ
three – phase system– If one of the wattmeters reads half The graph of a network has 8 nodes and 5 independent
of the other (both positive), then the power factor of the loops. The number of branches of the graph is
load is (A) 11 (B) 12
(A) 0.532 (B) 0.632 (C) 13 (D) 14
(C) 0.707 (D) 0.866 Solution: 
Solution:  Given, ℓ=b–n+1
w2 = w1/2 5=b–8+1
power factor b = 12
P.F = cos ϕ Hence, the correct option is (B)
⎛ 3 ( w1 − w2 ) ⎞ Question Number: 18 Question Type: MCQ
ϕ = tan −1 ⎜ ⎟
⎝ w1 + w2 ⎠ In the figure the voltages are v1(t) = 100 cos(ω t), v2(t) = 100
cos(ω t + p /18) and v3(t) = 100 cos(ω t + p /36). The circuit
⎛ ⎛ w ⎞⎞ is in sinusoidal steady state, and P < < ω L. P1, P2 and P3
3 ⎜ w1 − 1 ⎟
⎜ ⎝ 2 ⎠⎟ are the average power outputs. Which one of the following
= tan −1 ⎜ ⎟
⎜ w1 + w1 ⎟ statement is true?
⎝ 2 ⎠
R L L R
⎛ 1 ⎞
= tan -1
⎜⎝ ⎟⎠ = 30°
3 P2 P3
P1


P.F = cos 30° = 0.866 + + +
V1(t) V2(t) V3(t)
Hence, the correct option is (D) – – –

Question Number: 16 Question Type: MCQ


Consider a lossy transmission line with V1 and V2 as the
sending and receiving end voltages respectively. Z and X

M01_GATE EE 2018 paper 1.indd 5 6/18/2018 5:13:07 PM


xiv-f  |  GATE 2018 Solved Paper Electrical Engineering: Set – I

P1 = P2 = P3 = 0
(A) (B) P1 < 0, P2 > 0, P3 > 0 Transfer function Nature of system
P1 < 0, P2 > 0, P3 < 0 (D) P1 > 0, P2 < 0, P3 > 0
(C)
25
Q.  2 II. Critically damped
Solution:  As we know that s + 10s + 25
V1(t) = 100 cosω t
35
R. III. Under damped
⎛ π⎞ s 2 + 18s + 35
V2(t) = 100 cos ⎜ ω t + ⎟
⎝ 18 ⎠
(A) P – I, Q – II, R - III (B) P – II, Q – I, R – III
⎛ π⎞
V3(t) = 100 cos ⎜ ω t + ⎟ (C) P – III, Q – II, R – I (D) P – III, Q – I, R – II
⎝ 36 ⎠
15
R L L R Solution:  P:
S 2 + 5S + 15

P3 5
P1 P2 ξ= = 0.2581 → underdamped system
+ + 2 15
V1(t) V2(t) +
– V3(t)
– – 25
Q:
S 2 + 5S + 25
10
ξ= = 1 → Critically damped system.
f rom the given data 2 15
V1 = Vm ∠ 0 35
R: = 1.521 → over damped system
V2 = Vm ∠10° S + 18S + 35
2

V3 = Vm ∠5° P – III
So V2 leads V1 and V3 Q – II
P2 > 0, P1 and P3 < 0. R – I.
Hence, the correct option is (C) Hence, the correct option is (C)
Question Number: 19 Question Type: MCQ Question Number: 20 Question Type: MCQ
Match the transfer functions of the second – order systems A positive charge of 1 nC is placed at (0, 0, 0.2) where all
with the nature of the systems given below dimensions are in metres. Consider the x – y plane to be a
conducting ground plane. Take ∈0 = 8.85 × 10–12 F/– The z
Transfer function Nature of system
component of the E field at (0, 0, 0.1) closest to
P.
15
I. Over damped
(A) 899.18 V/m (B) –899.18 V/m
s 2 + 5s + 15 (C) 999.09 V/m (D) –999.09 V/m

Solution: 
Z Z
0.2
0.2 1 nC point
1 nC point
charge charge
0.1
0.1
Conducting
Grounded X –Y X –Y
Plate 1 nC
(image)
–0.2

M01_GATE EE 2018 paper 1.indd 6 6/18/2018 5:13:10 PM


GATE 2018 Solved Paper Electrical Engineering: Set – I  |  xiv-g

Q Gradient of the given function


Electric field intensity due to point charge E = aˆ R
4π ∈0 R 2 ∇φ = ( y 2 + 2 zx ) i + ( 2 xy + z 2 ) j + ( 2 yz + x 2 ) k

The Z – component of E at P(0, 0, 0.1) due to point charge
∇φ at ( 2, −1, 1) = 5 i − 3 j + 2 k
(+1nC) and due to its image (–1 nC) is given by
E = E z aˆ Z We know that P = i + 2 j + 2 k
10 −9 ( − aˆ Z ) 10 −9 ( − aˆ Z ) P i + 2 j + 2k
= −12
+ ∴ n̂ = =
4π × 8.85 × 10 × (0.1) 2
4π × 8.85 × 10 −12 × (0.3) 2 P 12 + 22 + 22

z ≈ - 999.09 v/m
E
i + 2 j + 2k
Hence, the correct option is (D) =
3
Question Number: 21 Question Type: MCQ
1 2 2
Let f be a real-valued function of a real variable defined as ∴ n̂ = i + j + k
3 3 3
f (x) = x2 for x ≤ 0, and f (x) = –x2 for x < 0. Which one of the
following statements is true? directional derivative of ϕ (x, y, z) in the direction of the
(A) f (x) is discontinuous at x = 0 vector P is ∇ϕ ⋅ n̂
(B) f (x) is continuous but not differentiable at x = 0 ⎛1 2 2 ⎞
(C)  f (x) is differentiable but its first derivative is not = (5i − 3 j + 2k ) ⋅ ⎜ i + j + k ⎟
⎝3 3 3 ⎠
continuous at x = 0
(D)  f (x) is differentiable but its first derivative is not 5 4
= −2+ = 1
differentiable at x = 0 3 3
Solution:  The given function is Hence, the correct option is (A)

⎪⎧ x
2
for x ≥ 0 Question Number: 23 Question Type: MCQ
f (x) = ⎨ 2 z +1
⎪⎩ − x for x < 0 The value of the integral
C z 2
−4 ∫
dz in a counter clock-
function f (x) is continuous at x = 0 wise direction around a circle C of radius 1 with centre at
⎧2 x for x ≥ 0 the point z = –2 is
f ′( x ) = ⎨
⎩ −2 x for x < 0 πi
(A) (B) 2p i
Also at x = 0; LHD = RHD for f (x) 2
πi
Therefore function f (x) is differentiable at x = 0 as well as (C) − (D) – 2p i
2
f ′( x ) is continuous at x = 0
Solution:  The given integral is
⎧2 for x ≥ 0
f ′′( x ) = ⎨ Z +1
⎩ −2 for x < 0 ∫
I=
c Z2 − 4
dZ
At x = 0; f ′′ (0 − ) ≠ f ′′(0 + )
Z +1
so, f ′( x ) is not differentiable at x = 0 Z = ± 2 are the singularities of , of which Z = – 2
Z2 − 4
Hence, the correct option is (D) lies inside C and Z = 2 lies outside C.
Question Number: 22 Question Type: MCQ
The value of the directional derivative of the function Φ (x,
y, z) = xy2 + yz2 + zx2 at the point (2, –1, 1) in the direction
of the vector p = I + 2j + 2k is
(A) 1 (B) 0.84
(C) 0.93 (D) 0.9
Solution:  The given function is Z +1
∴ ∫
I=
Z2 − 4
dZ
ϕ (x, y, z) = xy 2 + yz 2 + zx 2 c

M01_GATE EE 2018 paper 1.indd 7 6/18/2018 5:13:16 PM


xiv-h  |  GATE 2018 Solved Paper Electrical Engineering: Set – I

(Z + 1Z − 2) dZ R R
−Z 2
Z 1 (B)
(A)
= ∫
c
Z +2
R2 R1
R1
(C) −Z
Z (D)
⎛ Z +1⎞ R1 + R2
= 2πi ⎜
⎝ Z − 2 ⎟⎠ at Z = − 2
Solution:  Consider the circuit given below
⎛ −1 ⎞
= 2πi ⎜ ⎟ Z
⎝ −4 ⎠
πi Iin
=
2 +
Hence, the correct option is (A) – V0
+ R1
Vin
Question Number: 24 Question Type: MCQ –
In the logic circuit shown in the figure, Y is given by
A R2
B
Y

C Input current
D
Vin − V0
(A) Y = ABCD I in =
z
(B) Y = (A + B)(C + D)
(C) Y=A+B+C+D Vout R2
= Vin
(D) Y = AB + CD R1 + R2
Solution:  Consider the logic GATE given below The output voltage will be
A AB Vin ( R1 + R2 )
B Vout =
R2
Y

C CD 1⎡ ⎛ R1 ⎞ ⎤
D
Iin = ⎢Vin − Vin ⎜1 + ⎟ ⎥
z⎣ ⎝ R2 ⎠ ⎦

Y = AB ⋅ CD = AB + CD 1 ⎡ −Vin R1 ⎤
Iin = ⎢ ⎥
Hence, the correct option is (D) z ⎣ R2 ⎦

Question Number: 25 Question Type: MCQ Vin − z R2


Rin = =
To op – amp shown in the figure is ideal. The input imped- I in R1
v
ance in is given by Hence, the correct option is (B)
iin
Question Number: 26 Question Type: MCQ
Z
A continuous – time input signal x(t) is an eigen function of
Iin an LTI system, if the output is
+ (A) kx(t), where k is an eigen value
V0
(B)  kejω tx(t), where k is an eigen value and ejω t is a

+ R1 complex exponential signal
Vin
– (C) x(t)ejω t, where ejω t is a complex exponential s­ ignal.
(D)  kH(ω), where k is an eigenvalue and H(ω) is a
frequency response of the system
R2
Solution: 
Hence, the correct option is (A)

M01_GATE EE 2018 paper 1.indd 8 6/18/2018 5:13:20 PM


GATE 2018 Solved Paper Electrical Engineering: Set – I  |  xiv-i

Question Number: 27 Question Type: NAT


Consider a non-singular 2 × 2 square matrix A. If trace(A)
= 4 and trace (A2) = 5, the determinant of the matrix A is
_____ (up to 1 decimal place).
Solution:  Given A is a 2 × 2 non-singular matrix.
Let λ1 and λ2 be the eigen values of A.
⇒ λ12 and λ22 will be the eigen values of A2.
Trace (A) = 4 ⇒ λ1 + λ2 = 4
Solution:  Consider the circuit diagram given below
Trace (A2) = 5 ⇒ λ12 + λ22 = 5 2I1

Now (λ1 + λ2) = λ + λ + 2 λ1 λ2


2 2
1
2
2

⇒ 42 = 5 + 2λ1λ2
11 1Ω 1Ω
⇒ λ1λ2 = = 5.5 + +
2
Hence, the correct answer is 5.5
I1
Question Number: 28 Question Type: NAT V1 1Ω V2

Let f be a real-valued function of a real variable defined as


f (x) = x – [x], where [x] denotes the largest integer less than
1.25 – –
or equal to x. The value of ∫
0.25
f ( x ) dx is ______ (up to 2
h11 = ?
decimal places).
V1
Solution:  Real-valued function is given as h11 = at V2 = 0
I1
f (x) = x – [x]
pplying source transform to given network and short
A
Integrating both sides we get ­circuiting the second port.
1.25 1.25

( x − [ x ])dx
2I1

0.25
∫ f ( x )dx = ∫
0.25 +
I1 1Ω Vx 1Ω
– +
1.25 1.25

= ∫
0.25
xdx −
0.25
∫ [ x ] dx
V1
1Ω
1.25
x ⎤ 2
⎛ 1 1.25

= ⎥ - ⎜ ∫ 0 dx + ∫ 1dx ⎟
2 ⎦ 0.25 ⎝ 0.25 1 ⎠

= ⎡3 ⎤
1.25
VX − V1 VX VX + 2 I1
⎢⎣ 4 − x ⎥⎦ + + =0
1
1 1 1
3 1 1 − I1 + V X + V X + 2 I1 = 0
= − = = 0.5
4 4 2 2VX + I1 = 0
Hence, the correct answer is 0.5 I1
VX = −
Question Number: 29 Question Type: NAT 2
In the two – port network shown, the h11 parameter (where, But
V V1 − VX
h11 = 1 , when V2 = 0) in ohms is _____ (up to 2 decimal = I1
I1 1
places). V1 + 0.5 I1 = I1
V1 = 0.5 I1

M01_GATE EE 2018 paper 1.indd 9 6/18/2018 5:13:24 PM


–I1 + Vx + Vx + 2I1 = 0
2Vx + I1 = 0
on Type: NAT
−I
ameter (where, Vx = 1
2
up to 2 decimal V1 − Vx
But | GATE 2018 Solved Paper
xiv-j = I1 Electrical Engineering: Set – I
1
V1 + 0.5I1 = I1
V1 Question Number: 32 Question Type: NAT
=V1h11= =0.5I
0.51 Ω
I1 A 1000 × 1000 bus admittance matrix for an electric power
V1 system has 8000 non – zero elements. The minimum num-
= h11 = 0.5 Ω
I1 is 0.5
Hence, the correct answer ber of branches (transmission lines and transformers) in
Question Number: 30 Question this system are ______ (up to 2 decimal points)
20 QuestionType: Type:NAT NAT
The series
series impedance
impedancematrix
matrixofofa ashort
short three – phase trans-
three-phase trans- Solution: Number of transmission lines
⎡ Z s Z sZ m Z mZ m ⎤Z m  ⎛ Number of non zero off diagonal elements ⎞
⎢ Z  Z Z ⎥ . If the = ⎜ ⎟⎠
mission line
lineininphase
phasecoordinates
coordinates ⎢ m Z m s Z s m ⎥Z m  ⋅ If
is is ⎝ 2
⎢⎣ Z m Z mZ m Z mZ s ⎥⎦Z s 
Number of non-zero off diagonal elements
positive sequence
the positive impedance
sequence impedance is (1is (1 + j10)Ω,Ω,and
+ j10) andthe thezero
zero
8000 − 1000
j31) Ω,then
sequence is (4 + j31)Ω, then the
the imaginary
imaginary part (inΩ)
of ZZmm(in
part of Ω) = = 3500.
is _____
______(upto
(upto2 2decimal
decimalplaces).
places). 2
Hence, the correct answer is 3500.
Solution: Zs – Zm = 1 + j10
Question Number: 33 Question Type: NAT
Zs + 2Zm = 4 + j31
The waveform of the current drawn by a semi – converter
2Zs – 2Zm = 2 + j20 from a sinusoidal AC voltage source is shown in the figure.
+ Zs + 2Zm = 4 + j31 If I0 = 20A, the rms value of fundamental component of the
3Zs = 6 + j51 current is _____ A (up to 2 decimal places)
V2
Zs = (2 + j17) Ω
2 + j17 – 1 – j10 = Zm
Zm = j7 + 1
– Imaginary part is 7.
Imaginary part is 7.
Hence, the
Hence, the correct
correct answer
answer isis 77 to
to 7.
7.
Question Number:
Question Number: 31 21 QuestionType:
Question Type:NATNAT Solution: From the give figure we get
The positive,
positive, negative
negative andand zero
zero sequence
sequence impedances
impedancesof ofaa 4I α
The IS1 = s cos
125 MVA,
125 MVA, three-phase
three – phase 15.5
15.5kV,kV,
start-grounded, 50 Hz
start – grounded, 50 gen-
Hz π 2
erator are j0.1 pu, j0.05 and j0.01 pu, respectively
generator are j0.1 pu, j0.05 and j0.01 pu respectively on the on the
2 2 x 20
machine rating
machine rating base.
base. TheThe machine
machineisisunloaded
unloadedandandworking
working IS1r = cos 15 = 17.40 A
k and short the at the rated terminal voltage. If the grounding impedance π
at the rated terminal voltage. If the grounding impedance
of the
of the generator
generator is is j0.01 pu, then
j0.01 pu, then the
the magnitude
magnitude ofof fault
fault Hence, the correct answer is 17.40.
current for a b – phase to ground fault (in kA) is _____ (up Question Number: 34 Question Type: NAT
to 2 decimal places)
A separately excited dc motor has an armature resistance
Solution: Ra = 0.05 Ω The field excitation is kept constant. At an
Fault current will be armature voltage of 100 V, the motor produces a torque of
3 Ea1 500 Nm at zero speed. Neglecting all mechanical losses,
If = pu the no16:44:41
– load speed of the motor (in radian/s) for an arma-
Z1 + Z 2 + Z 0 + 3Z n 15-03-2018
ture voltage of 150 V is _____ (up to 2 decimal places)
Base current
125 Solution: For separately excited d.c. motor
Ibase = × 103 = 4656.050 (A)
3 × 15.5 ∴ vt1 = Eb1 + I a1 Ra (1)
Now we have Eb1 = ka ϕ w1
If (KA) = If (pu) × Ibase Eb1 = 0 (∵ ω1 = 0)
3 × 1 × 4656.050 ∴ From eqn 1,
If =
0.1 + 0.05 + 0.01 + 3(0.01) ∴ 100 = I a × 0.05
1
= 73.5236 (KA)
⇒ I a1 = 2000 A
Hence, the correct answer is 73.5236.

M01_GATE EE 2018 paper 1.indd 10 6/18/2018 5:13:28 PM


pu T1 = 500 Nm at speed, N = 0 rpm.
+ 3Z n
\ vt1 = Eb1 + I a1 Ra (1)
Eb1 = ka φω1
3 Eb1 = 0 ( w1 = 0)
\ From Equation (1), GATE 2018 Solved Paper Electrical Engineering: Set – I | xiv-k
100 = I a1 × 0.05
656.050 ⇒ I a1 =condition,
Under no-load 2000 A no-load voltage drop is very
μVr 2 N P2 ω μIr 2 N P N s ω
small.
Under no-load condition, no-load voltage drop is very small. (A) (B)
01 + 3 (0.01) I V
\ Eb ≅ vt ⇒ vt2 = Eb2
μVr N P ω
2 2
μIr N P2 ω
2
\ Eb2 = 150 V. (C) (D)
on Type: NAT 2I 2V
Tem1 = ka φI a1
electric power 1 Solution:
imum number 500 = (kaf) × 2000 ⇒ (kaf) = (2)
4 MMF
rmers) in this Flux(f) =
Eb2 Reluctance
Eb2 = ka φ ⋅ ω2 ⇒ w2 =
Kaφ MMF
∴ Reluctance =
\ w = 150 × 4 = 600 rad/sec. Flux(ϕ )
2
ments  Hence, the correct answer is (600). dϕ
 600. ∴ In General, EMF induced E = N
 dt
Question
Question Number:
Number: 2535 QuestionType:
Question Type:NATNAT
t
Consider
Consider aa unity
unity feedback
feedback system
system with
with forward
forward transfer
transfer 1
N ∫0
∴ f= E dt
function given by
function given by
1 1⋅ t
G(s) = G(s) = 1
Np ∫0
on Type: NAT ( s + 1) ( s( s++21)) ( s + 2) ∴ f= E dt
emi-converter
steady – state
The steady-state error
error in the
in the outputoutput of system
of the the system
for afor a
unit-
n in the figure. As per Lenz law E = -Vp
unit input
step – stepisinput is ______
______ (up to 2(up to 2 decimal
decimal places).places)
mponent of the 1
t

Np ∫0
Solution: transfer function is given as ∴ f= −Vpdt
1
Solution: G (s) = 1
G(S) = ( S + 1) ( S + 2)
( S + 1) ( S + 2) 1
t

Np ∫0
∴ f= v cos ω t dt
1 1
= = S
R (S)
R(S)
S v
e
Steady – state error in thessoutput=
A
of the system for a unit – ⇒ f= sin ω t
ω Np
step input will be 1 + KP
Np.I
essK= = ALt G ( s) = 1 ∴ Reluctance = ∵(IP=I sin ωt )
P
1+SK→P0 2 V
1 21 ωNp
KPe=ss =Lt G(S) =
15 = 17.40 A
S →0 1 32 Np 2 . ω I
1+ ⇒ Reluctance =
ess = 1 22 V
=
= 0.6666
1 3 2R
1+ Reluctance in terms of radius ⇒ 2
Q.26 to Q.55 carry two marks each. 2 μr
on Type: VAT
= 0.6666
ure resistance Question Number: 26 Question Type: MCQ 2R Np 2 . ωI
onstant. At an Hence, the correct answer is 0.6666. ∴ =
A transformer with toroidal core of permeability m is shown μr 2
V
es a torque of Question
in Number:
the figure. Assuming36 uniform flux Question Type:the
density across NAT
cir-
hanical losses, Np 2 . ωI μr 2
cular core cross-section of radius r << R, and
A transformer with toroidal core of permeability μ is shown neglecting ∴ R=
or an armature any leakage 2V
in the figure. flux, the best
Assuming estimate
uniform flux for the mean
density acrossradius
the cir-R
places). is:
cular core cross – section of radius r << R, and neglecting Hence, the correct option is (D)
any leakage flux, the best estimate for the mean radius R is Question Number: 37 Question Type: MCQ
A 0-1 Ampere moving iron ammeter has an internal resist-
ance of 50 mΩ And inductance of 0.1 mH. A shunt coil is
connected to extend its range to 0-10 Ampere for all oper-
ating frequencies. The time constant in milliseconds and
resistance
15-03-2018 16:44:44in mΩ of the shunt coil respectively are

(A) 2, 5.55 (B) 2, 1


(C) 2.18, 0.55 (D) 11.1, 2

M01_GATE EE 2018 paper 1.indd 11 6/18/2018 5:13:34 PM


xiv-l  |  GATE 2018 Solved Paper Electrical Engineering: Set – I

Solution: 
⎛ 1− K ⎞
Zf = ⎜ ( Z + Z2 )
0 – 1A Rm Lm ⎝ K ⎟⎠ 1
I
Hence, the correct option is (A)

(I – Im) Question Number: 39 Question Type: MCQ


Consider the two bus power system network with given
loads as shown in the figure. All the values shown in the
Lsh
Rsh figure are in per unit. The reactive power supplied by gen-
erator G1 and G2 are QG1 and QG2 respectively. The per unit
e know that meter time constant should be equal to shunt
W values of QG1 and QG2, and line reactive power loss (Qloss)
coil time constant in order to make the meter independent respectively are.
of frequency,.
0.1
τ m = τ sh ⇒ τ sh = =2
50 × 10 −3
Lm
= τ sh
Rm
(A) 5.00, 12.68, 2.68 (B) 6.34, 10.00, 1.34
To extend meter range to 10 A,
(C) 6.34, 11.34, 2.68 (D) 5.00, 11.34, 1.34
Im Rm = (I – Im)Rsh
⇒ 1 × 50 × 10–3 = 9 × Rsh Vs Vr
Solution:     Ps = sinδ
Rsh = 5.55 X
Hence, the correct option is (A) 5 = 10 sinδ
δ = 30°
Question Number: 38 Question Type: MCQ
⎧⎪ V 2 V V ⎫⎪
The positive, negative and zero sequence impedances of a QS = ⎨ S sin 90° − s r cos δ ⎬
three phase generator are Z1, Z2 and Z0 respectively. For a X X
⎩⎪ ⎪⎭
line – to – line fault with fault impedance Zf , the fault cur-
rent is If1 = kIf, where If is the fault current with zero fault QS = 10 – 10 cos30° = 1.33974
impedance. The relation between Zf and k is QR = –1.33974
( z1 + z2 )(1− k ) ( z1 + z2 )(1+ k )  (Receiving end supplies)
(A) Zf (B) Zf QLine = QLoss = QS – QR
k k
( z1 + z 2 ) k ( z1 + z 2 ) k = 2.68 pu
(C) Zf (D) Zf
1− k 1+ k QG1 = QLoad + QS
Solution:  fault current without fault impedance = 6.34 pu
3 Ea1 QG2 = 11.34 pu.
I f1 =
Z1 + Z 2 Hence, the correct option is (C)
fault current with fault impedance
Question Number: 40 Question Type: MCQ
3 Ea1 The pre – unit power output of a salient – pole genera-
I f1 =
Z1 + Z 2 + Z f tor which is connected to an infinite bus, is given by the
expression, P = 1.4 sin δ + 0.15 sin 2 δ , where δ is the
I f1 = K I f load angle. Newton – Raphson method is used to calculate
1 K the vaue of δ for P = 0.8 pu. If the initial guess is 30°, then
= its value (in degree) at the end of the first iteration is
Z1 + Z 2 + Z f Z1 + Z 2
(A) 15° (B) 27.48°
Z1 + Z2 = K (Z1 + Z2) + KZf (C) 28.74° (D) 31.20°
Z1 + Z2 – K (Z1 + Z2) = KZf Solution:  P = 1.4 sinδ + 0.15 sin2δ
(Z1 + Z2) (1 – K) = Zf K δo = 30°

M01_GATE EE 2018 paper 1.indd 12 6/18/2018 5:13:38 PM


GATE 2018 Solved Paper Electrical Engineering: Set – I  |  xiv-m

∂P 1
t t

C ∫0
= 1.4cosδ + 0.30cos2δ
Vc (t ) = idt = ∫ 5 sin tdt 
∂δ 0
⎡ Δ P ⎤ ⎡ J1 J 2 ⎤ ⎡ Δ δ ⎤ Vc (t ) = 5[1 − cos t ]V (ii)
⎢Δ Q⎥ = ⎢ J J ⎥ ⎢Δ V ⎥
⎣ ⎦ ⎣ 3 4⎦ ⎣ ⎦ From (i) and (ii)
∆P = J1 ∆δ i2(t) + (Vc(t) – 5)2 = (5 sin t)2 + (–5 cos t)2
∆δ = [ J1 ] ΔP
−1
(Vc(t) – 5)2 + i2(t) = 52
δ = δo – ∆δ = 28.79° It representing a circle with centre (5,0) and r = 5.
Question Number: 41 Question Type: MCQ Hence, the correct option is (B)
A DC voltage source is connected to a series L – C circuit Question Number: 42 Question Type: MCQ
by turning on the switch S at time t = 0 as shown in the fig- The equivalent impedance Zeq for the infinite ladder circuit
ure. Assume i(0) = 0, v(0) = 0. Which one of the following shown in the figure is
circular loci represents the plot of i(t) versus v(t)?
j9 Ω j9 Ω
i(t)
S

t=0 L = 1H
j5 Ω j5 Ω
Zeq ...
+ +
5V C = 1F v(t)
– –j1 Ω –j1 Ω

(A)
j12Ω (B) –j12Ω
(C)
j13Ω (D) 13 Ω
(A) i(t) (B)
i(t)

v(t)
Solution:  infinite ladder circuit is shown below
(0, –5)

j9 Ω j9 Ω
(5, 0) v(t)

(C) i(t) (D) i(t)


j5 Ω j5 Ω

(0, 5)
(–5, 0) v(t)
–j1 Ω –j1 Ω
v(t)
Solution:  As per problem i(0) = 0, V(0) = 0
Redrawing the circuit in S domain
Zeq
S I(S)
Assume Z1 = j9Ω and Z2 = j5 – j1 = j4Ω
Approximate equivalent circuit is shown below
Z1 Z1
+ + 1
5/S V(S)
– – S

Z2 Z2

5
S5
I (S ) = =
1 S2 +1
S+
S
Zeq Zeq
i(t) = 5 sint Amp (i)

M01_GATE EE 2018 paper 1.indd 13 6/18/2018 5:13:42 PM


xiv-n  |  GATE 2018 Solved Paper Electrical Engineering: Set – I

Zeq = Z1 + {Z2 || Zeq} x1 (t ) = (0.5 + 0.5e −2t ) x1 (0) +


Z 2 ⋅ Z eq
Zeq = Z1 + (0.5 − 0.5e −2t ) x 2 (0)
Z 2 + Z eq
Similarly,
Zeq {Z2 + Zeq} = Z1Z2 + Z1Zeq + Z2Zeq
x1 f = Lt x1 (t ) = 0.5 x1 (0) + 0.5 x2 (0)
sub Z1 = j9Ω and Z2 = j4Ω t →∞

x2 f = Lt x2 (t ) = 0.5 x1 (0) + 0.5 x2 (0)


Z eq2 + j 4 Z eq = –36 + j9 Zeq + j4Zeq t →∞

Here x1f and x2f are equal and as per the given data x1(0) <
Z eq2 – j9Zeq + 36 = 0
x2(0) < ∞, means
from the given options
x1f = x2f < ∞
Zeq = j12Ω
Hence, the correct option is (C)
Hence, the correct option is (A)
Question Number: 44 Question Type: MCQ
Question Number: 43 Question Type: MCQ
The number of roots of the polynomial. s7 + s6 + 7s5 + 14s4
Consider a system governed by the following equation + 31 s3 + 73s2 + 25s + 200, in the open left half of the com-
dx1 (t ) plex plane is
= x2 (t ) − x1 (t ) (A) 3 (B) 4
dt
(C) 5 (D) 6
dx2 (t )
= x1 (t ) − x2 (t ) Solution: 
dt
The initial conditions are such that x1(0) < x2(0) < ∞ . Let C. E = S7 + 36 + 7S5 + 14S4 + 31S3 + 73S2 + 25S + 200 = 0
x1f = lim x1 (t ) and x2f = lim x2 (t ) . Which one of the fol- + S7 1 7 31 25
t →∞ t →∞
lowing is true? + S6 1 14 73 200
(A) x1f < x2f < ∞ (B)
x2f < x1f < ∞
– S5 –7 –42 –175 0
(C) x1f = x2f < ∞ (D)
x1f = x2f = ∞
+ S4 8 48 200
dx 1 (t ) + S3 0 (32) 0 (96) 0
Solution:    = x2 (t ) − x1 (t ) (1)
dt + S2 24 200
dx 2 (t ) −512
= x1 (t ) − x2 (t ) (2) – S1
dt 3
Applying laplace transform to both
+ S0 200
Sx1 ( s) − x1 (0) = x2 ( s) − x1 ( s) (3)

A. E = 8S4 + 48S2 + 200
Sx2 ( s) − x2 (0) = x1 ( s) − x2 ( s) (4)
dAE
Where x1(0) and x2(0) are initial conditions = 32S3 + 96S
dS
x1 ( s ) x2 ( 0 ) Number of sign changes below A. E = 2
from (4) x2(s) = + (5)
s +1 s +1 ∴ jω
substituting (5) in (3)
⎡ 1 ⎤ x2 ( 0 )
x1 ( s) ⎢ s + 1 − = + x1 (0)
⎣ s + 1⎥⎦ s + 1
⎡ 1 ⎤ ( s + 1)
x1 ( s) = ⎢ ⎥ x2 ( 0 ) + x1 (0)
⎣ s( s + 2) ⎦ s( s + 2)
⎡1 1 ⎤
x1 ( s) = ⎢ − x2 ( 0 ) +
⎣ 2 s 2( s + 2) ⎥⎦ σ

⎡1 1 ⎤
⎢ 2 s + 2( s + 2) ⎥ x1 (0)
⎣ ⎦ 

M01_GATE EE 2018 paper 1.indd 14 6/18/2018 5:13:47 PM


GATE 2018 Solved Paper Electrical Engineering: Set – I  |  xiv-o

And number of sign changes above auxiliary equation are 2. ∴ Res f ( z ) = 4 z = 1  (2)
Total number of RHP = 4
⎡d ⎤
Total number of jω poles = 0

Res f ( z ) = Lt ⎢
z→2
⎣ dz
(
( z − 2) 2 f ( z ) ⎥

)
Total number of LHP = 3. z=2

Lt ⎢⎡ d ⎛ z ⎞⎤
Hence, the correct option is (A) 2
= z→2 ⎜ 2 ⎟⎥
Question Number: 45 Question Type: MCQ ⎣ dz ⎝ ( z − 1) ⎠ ⎦
z2
Lt ⎡ 2 z ( z − 1) − 2 z ( z − 1) ⎤
2 2
If C is a circle |z| = 4 and f(z) = Z f 2 , then = z→2
( z − 3 z + 2) 2 ⎢ ⎥
⎣ ( z − 1) 4 ⎦
∫ f ( z ) dz is
Lt ⎡ 2 z ( z − 2 z + 1) − 2 z + 2 z ⎤
C 2 3 2

(A) 1 (B) 0 = z→2 ⎢ ⎥


⎣ ( z − 1) 4 ⎦
(C) –1 (D) –2
Lt ⎡ 2 z − 2 z ⎤
2
Z2 = z→2 ⎢ 4 ⎥
Solution:    f ( Z ) = 2 ⎣ ( z − 1) ⎦
( Z − 3Z + 2) 2
Z2 ∴ Res f ( z ) = −4 (3)
= z=2 
(( Z − 1) ( Z − 2))2
Substituting (2) and (3) in (1), we have
Z2
∴ f(Z) =
( Z − 1) 2 ( Z − 2) 2 ∫ f ( z )dz = 2 πi [4 − 4] = 0
c

Hence, the correct option is (B)


Question Number: 46 Question Type: MCQ
Which one of the following statements is true about the
digital circuit shown in the figure.

z = 1 and z = 2 are the singularities of f(z) and both of them


lie inside C.
∴By residue theorem,
(A) It can be sued for dividing the input frequency by e.
⎡ Re s f ( z ) + Re s ( f ( z )) ⎤ (B) It can be used for dividing the input frequency by 5.
∫ f ( z )dz = 2π i ⎢
⎣z = 1 z=2 ⎥

(1)
(C) It can be used for dividing the input frequency by 7.
C
(D) It cannot be reliably used as a frequency divider
⎡d ⎤
Res f ( z )
z =1
= Lt ⎢
z→1
⎣ dz
(
( z − 1) 2 f ( z ) ⎥

) due to disjoint internal cycles.
Solution: 
Lt ⎡⎢ d ⎛ z ⎞⎤
2
= z→1 ⎜ 2 ⎟⎥
⎣ dz ⎝ ( z − 2) ⎠ ⎦

Lt ⎡ 2 z ( z − 2) − 2 z ( z − 2) ⎤
2 2
= z→1 ⎢ ⎥
⎣ ( z − 2) 4 ⎦

Lt ⎡ 2 z ( z − 4 z + 4) − 2 z + 4 z ) ⎤
2 3 2
= z→1 ⎢ ⎥
⎣ ( z − 2) 4 ⎦ CIK A B C
0 0 0 0
Lt ⎡ 8 z − 4 z ⎤
2
= z→1 ⎢ 4 ⎥ 1 1 0 0
⎣ ( z − 2) ⎦

M01_GATE EE 2018 paper 1.indd 15 6/18/2018 5:13:54 PM


xiv-p | GATE 2018 Solved Paper Electrical Engineering: Set – I

CIK A B C Solution: Plot for x1(t) and x2(t) are shown below
2 1 1 0
3 1 1 1
4 0 1 1
5 0 0 1
6 1 0 0
7 1 1 0

the modulus of he given counter is 5 so it is used to divide Sampled Versions x1[n] and x2[n] can be shown as
the input frequency by 5.
Hence, the correct option is (B)
Question Number: 47 Question Type: MCQ
Digital input signals A, B, C with A as the MSB and C as
the LSB are used to realized the Boolean function F = m0
+ m2 + m3 + m5 + m7, where m1 denotes the ith minterm– In
addition, F has a don’t care for m1. The simplified expres-
X1[n] = δ [n + 1] + 0.75 δ [n + 0.75] + 0.5 δ [n + 0.5]
sion for F is given by
+ 0.25 δ [n + 0.25] + 0.25 δ [n – 0.25]
(A) AB + BC + AC (B) A + C
+ 0.5 δ [n – 0.5] + 0.75 δ [n – 0.75] + δ [n – 1]
(C) C + A (D) AC + BC + AC
1
Solution: Energy in X1[n] is ∑
n = −1
x12 [n]

Ex1[n] = 3.75
Sampled Version of x2[n] is

F = I + II where I = C
F = A + C II = A X2[n] = 0.25 δ [n + 0.75] + 0.5 δ [n + 0.5]
Hence, the correct option is (B) + 0.75 δ [n + 0.25] + δ [n] + 0.75 δ [n – 0.25]
Question Number: 48 Question Type: MCQ + 0.5 δ [n – 0.5] + 0.25 δ [n – 0.75]
1
Consider the two continuous – time signals defined below:
⎧ t , −1 ≤ t ≤ 1
Energy in X2[n] is ∑
n = −1
x22 [n]
x1(t) = ⎨
⎩0, otherwise Ex2[n] = 2.75
⎧1 − t , −1 ≤ t ≤ 1 Energy of x1[n] is greater than energy of x2[n]
x2(t) = ⎨ Hence, the correct option is (A)
⎩ 0, otherwise
These signals are sampled with a sampling period of T = Question Number: 49 Question Type: MCQ
0.25 seconds to obtain discrete time signals x1[n] and x2[n], The signal energy of the continuous – time signal
respectively. Which one of the following statements is true? x(t) = [(t – 1) u(t – 1)] – [(t – 2) u(t – 2)]
(A) The energy of x1[n] is greater than the energy
of x2[n]. – [(t – 3) u(t – 3)] + [(t – 4) u(t – 4)] is
(B) The energy of x2[n] is greater than the energy (A) 11/3 (B) 7/3
of x1[n] (C) 1/3 (D) 5/3
(C) x1[n] and x2[n] have equal energies. Solution: Consider the figure given below
(D) Neither x1[n] nor x2[n] is a finite – energy signal.

M01_GATE EE 2018 paper 1.indd 16 6/18/2018 5:13:58 PM


ing statements – [(t – 3) u(t – 3)] + [(t – 4) u(t – 4)] is
(A) 11/3 (B) 7/3
the energy of (C) 1/3 (D) 5/3
Solution: Given
the energy of x(t) = [(t – 1) u(t – 1)] – [(t – 2) u(t – 2)]
GATE 2018 Solved Paper Electrical Engineering: Set – I | xiv-q
– [(t – 3) u(t – 3)] + [(t – 4) u(t – 4)]
ergy signal. x(t)
x(t) ∴ X(t) = te–10t
Now, In x(t ) = ln t e −10 t

= ln t − 10 In e
11
at t = 1
tt = In1 − 10 = 10
11 22 33 44
Hence, the correct answer is 10.
1 ≤ 1t ≤≤ 2:
t ≤x(t) = (t =– (t1)– 1)
2: x(t) Question Number: 51 Question Type: NAT
wn as
2 ≤ 2t ≤≤ 3:
t ≤x(t) = 1= 1
3: x(t) In the circuit shown in the figure, the bipolar junction tran-
3 ≤ 3t ≤≤ 4:
t ≤x(t) = (4=–(4t) – t)
4: x(t) sistor (BJT) has a current gain β = 100. The base – emitter
Energy of x(t) is given by, voltage drop is a constant. VBE = 0.7 V. The value of the
∞ Thevenin equivalent resistance RTh (in Ω) as shown in the
∫ ( x (t ) )
2 figure is _____ (up to 2 decimal places.
E = dt
−∞

2 3 4

∫ (t − 1) dt + ∫ 1 dt + ∫ (4 − 5) dt
2 2 2
+ 0.5δ[n + 0.5]
1 2 3
+ 0.5δ[n – 0.5]
2 3 4
.75] + δ[n – 1]
∫ (t − 2t + 1) dt + ∫ 1dt + ∫ (16 + t − 8t ) dt
2 2

1 2 3 Solution: Consider the circuit below

1/3 + 1 + 1/3 = 5/3 15 V

Hence, the correct option is (D).


Hence, the correct option is (D)
10 Ω
Question Number: 40 Question Type: NAT
Question Number: 50 Question Type: NAT
The Fourier transform of a continuous-time single x(t) is
The Fourier transform of a continuous – time single x(t) is 10 Ω
1
given by X(w) = 1 , −∞ < ω < ∞, where j = −1
given by X(ω) = (10 + jω)22 , −∞ < ω < ∞ , where j = −1
(10 + j ω) +
and w denotes frequency. Then the value of |? In x(t)| at t = 1 10.7
and
is ω denotes
______ frequency.
(up to 1 decimalThen the (In
place). value of |?Inthe
denotes x(t)| at t = 1
logarithm –
1 kΩ
is _____
to base e)(up to 1 decimal place). (In denotes the logarithm RTh
to base e)
0.5] 1 Emitter current
Solution: Given
Solution: We know thatx(w) = , −∞ < ω < ∞
.75δ[n – 0.25]
1 (10 + jω) 2 10.7 − 0.7
.25δ[n – 0.75] x(ω) = ,–∞<ω<∞ IE = = 9.17mA
lxxii | GATE 2018(10 + j ω)Paper Set – 1
Solved 10
2
1+
101
F.T
x(t ) ← 
→ x(ω) Collector current I
⇒ gm = C = 0.35
VT
F.T dx(ω) ⎛ β ⎞
− jt x(t ) ← 
→ IC = ⎜
dω β ⎝ β + 1⎟⎠
15-03-2018 16:44:59 rp = = 0.285 k Ω
gm
1 IE = 9.08 mA
e −10t ←

10 + jω a.c equivalent circuit is as shown below
I
⇒ gm = C = 0.35
−j VT
− jt e −10t ←

(10 + jω) 2 β β ib
10 kΩ
rp =0.285 kΩ
= 0.285 kΩ
1 gm 10 kΩ
⇒ te −10t ←
F.T

→ 2
(10 + jω) a.c equivalent circuit of given circuit will be
–10t
\ X(t) = te
1 kΩ
Now, In x(t ) = In te −10t
= In t − 10 In e RTh
M01_GATE EE 2018 paper 1.indd 17 6/18/2018 5:14:02 PM
at t=1
xiv-r  |  GATE 2018 Solved Paper Electrical Engineering: Set – I

∫ ⎡⎣d ( xy ) + d ( x 2 y ) ⎤⎦
2
=
C
0.285 kΩ β ib
( 0 ,3)

( )
10 kΩ 10 kΩ
= ∫
( 3, 0 )
⎡ d xy 2 + x 2 y ⎤
⎣ ⎦
( 0 ,3)

1 kΩ
= xy + x y ⎦⎤2 2
=0
( 3, 0 )

RTh Hence, the correct answer is 0.


Question Number: 53 Question Type: NAT
Thevenin equivalent resistance Let f (x) = 3x – 7x + 5x + 6. The maximum value of f (x)
3 2

⎛ 10 0.285 ⎞ over the interval [0, 2] is _____ (up to 1 decimal place).


Rth =1kΩ || ⎜ +
⎝ 101 101 ⎟⎠ Solution:  The function is
Rth = 1 kΩ || 0.1 kΩ f (x) = 3 x 3 − 7 x 2 + 5 x + 6
Rth = 91 Ω the derivative of above function will be
Hence, the correct answer is 91. ⇒ f ′ ( x ) = 9 x 2 − 14 x + 5
Question Number: 52 Question Type: NAT f ′ ( x ) = 0 ⇒ 9 x 2 − 14 x + 5 = 0
As shown in the figure C is the arc from the point (3, 0) to
5
the point (0, 3) on the circle x2 + y2 = 9. The value of the ⇒ (9x – 5)(x – 1) = 0 ⇒ x = ; x = 1
9
( )
integral ∫ ( y 2 + 2 yx )dx + 2 xy + x 2 dy is ______ (up to 2
∴The maximum value of f (x) in [0,2]
C
decimal places). ⎧ 5 ⎫
= Max. ⎨ f (0) , f ( 2), f ( ), f (1)⎬
⎩ 9 ⎭
= Max. {6,12, 7.1317, 7}
= 12
Hence, the correct answer is 12.
Question Number: 54 Question Type: NAT
⎡ 1 0 −1⎤
Solution:  We have to evaluate
Let A = ⎢ −1 2 0 ⎥⎥ and B = A3 – A2 – 4A + 5I, where I

∫(y + 2 yx )dx + ( 2 xy + x 2 )dy
2
⎢⎣ 0 0 −2⎥⎦
c
is the 3 × 3 identity matrix. The determinant of B is ______
a long the boundary of the circle x2 + y2 = 9 from (3,0) to
(up to 1 decimal place)
(0,3)
⎡ 1 0 −1⎤
Solution:  Matrix A = ⎢ −1 2 0 ⎥⎥

⎢⎣ 0 0 −2⎥⎦
Characteristic equation of matrix A will be
A − λΙ = 0

1− λ 0 −1
∴ ∫(y ) (
+ 2 yx dx + 2 xy + x 2 dy )
2
⇒ −1 2 − λ 0 =0
C 
0 0 −2 − λ
= ∫ ⎡⎣ y 2 dx + 2 yxdx + 2 xydy + x 2 dy ⎤⎦
C ⇒ (–2–λ)(2–λ)(1–λ) = 0
= ∫ ⎡⎣( y dx + 2 xydy ) + (2 yxdx + x dy )⎤⎦ ⇒ λ = 1, 2, –2
2 2

M01_GATE EE 2018 paper 1.indd 18 6/18/2018 5:14:07 PM


GATE 2018 Solved Paper Electrical Engineering: Set – I  |  xiv-s

eigen values of A are 1, 2 and –2 we know that shown in the figure. The value of K is ______ (up to 2 deci-
B = A3 –A2 – 4A + 5I mal places)
Eigen Values of A    Eigen Values of B

∴ The eigen values of B are 1,1 and 1.


So, the determinant of B = 1 × 1 × 1 = 1
Hence, the correct answer is 1. K
Solution:  CLTF =
Question Number: 55 Question Type: NAT ( S + 1) ( S + 2) + K
2

The capacitance of an air – filled parallel – plate capacitor ⎛ 1⎞ ⎧ K ⎫


is 60 pF. When a dielectric slab whose thickness, is half 0.8 = Lt ⎜ ⎟ S ⎨ ⎬
S →0 ⎝ S ⎠
⎩ ( S + 1) ( S + 2) + K ⎭
2
the distance between the plates, is placed on one of the
plates covering it entirely, the capacitance becomes 86 pF. K
0.8 =
Neglecting the fringing effects, the relative permittivity of 2+ K
the dielectric is ______ (up to 2 decimal places) ⇒ 1.6 + 0.8K = K
Solution:  Parallel plate capacitor with air gap ⇒ 0.2K = 1.6
⇒ K = 8.
Hence, the correct answer is 8.
Question Number: 57 Question Type: NAT
A three – phase load is connected to a three – phase bal-
Parallel plate capacitor with dielectric is given below anced supply as shown in the figure. If Van = 100∠0° V, Vbn
= 100∠–120° V and Vcn = 100 ∠–240° V(angles are con-
sider positive in the anti – clockwise direction), the value
of R for zero current in the neutral wire is ____ Ω

Capacitance of parallel plate capacitor is


ε0 A
C0 = = 60 pF
d
Now we have
C1 = 2C0
Solution:
C2 = 2C0 ∈r We know that
C1C2 120 ∈r IN = IR + IY + IB = 0
Ceq = = = 86 pF
C1 + C2 1+ ∈r V ∠0°
IR =
∈r = 2.53 R
Hence, the correct answer is 2.53. V ∠ − 120°
 Iy =
j ωL
Question Number: 56 Question Type: NAT
The unit step response y(t) of a unity feedback system with V ∠ − 240°
IB =
K ⎛ 1 ⎞
open loop transfer function G(s)H(s) = is ⎜⎝ j ωc ⎟⎠
( s + 1) 2 ( s + 2)

M01_GATE EE 2018 paper 1.indd 19 6/18/2018 5:14:11 PM


xiv-t | GATE 2018 Solved Paper Electrical Engineering: Set – I
Question Nu
V(t) = 5 – 5 cos wt + 5 3 sin wt
The figure sh
1 1 ∠ − 120° i(t) = 5 + X cos wt Question Nu
+ + jω c ∠–240° = 0 V(t) = 5 – 5 cos wt + 5 3 sin wt lel. The comm
R j ωL P = V(t) ⋅ i(t) The figure sh
i(t) = 5 + X cos wt value of 100
1 1 cos120° + jω c cos 240° = 0 (1) P = [5 – 5 cos wt + 5 3 sin wt] [5 + X cos wt] lel. The comm
value. The lo
cos 0° + P = V(t) ⋅ i(t) value of 100
R j ωL = 25 + 5X cos wt – 25 cos wt negligible rip
P = [5 – 5 cos wt + 5 3 sin wt] [5 +2 X cos wt] value. The loa
conduction m
1 1 – 5X cos wt + ... etc
sin 0° + sin(–120°) + jω c sin(–240°) = 0 (2) = 25 + 5X cos wt – 25 cos wt negligible ripp
switch contro
R j ωL 5X conduction m
P = 25 − {1+ cos 2wt} – 5X + 5X 2
wt + ... etc
coscoswt the steady sta
1 2 switch contro
equally, the a
Solve equation (2) ⇒ ω = put in equation (1) then 5X – 25 wt + ... etc.
coscoswt
P = 25 − {1+ cos 2wt} + 5X the
(in steady
Ampere),stat
LC 2 – 2.5X cos wt + 5X cos wt ... etc
P = 25 – 2.5X equally, the a
1 L – 25 cos wt + ... etc. (in Ampere),
R= given Pavg = 0
3 C P = 25 – 2.5X – 2.5X cos wt + 5X cos wt ... etc
So, 25 – 2.5X = 0 ⇒ X = 10
1 given Pavg = 0
R= 100 = 5.77 (Ω) Hence,
So, 25the
Question correct
Number:
– 2.5X = 0answer
⇒ X49 = is1010. Question Type: NAT
3 A phase controlled single phase rectifier, supplied by NAT
an AC
Question
Question Number:
Number: 59 Question Type:
Hence, the correct answer is 5.77. source, feeds power 49 to an R-L-E load as Question
shown in Type: NAT
the figure.
A
A phase
phase controlled
controlled single phase rectifier, supplied by an
The rectifier outputsingle phase
voltage hasrectifier,
an averagesupplied
valueby an AC
given by
Question Number: 58 Question Type: NAT AC source, feeds power to an R – L – E load
source,Vfeeds power to an R-L-E load as shown in the figure. as shown in
m
The voltage across the circuit in figure, and the current V0 fi=rectifier
the
The gure. The cos a),
+rectifi
(3 output er output
where
voltage voltage
has has volts
Vman=average
80p anvalue
average avalue
andgiven is the
by Solution:
2π V
through it, are given by the following expressions: given Vby V = m
(3 + cos α ), where V = 80 p volts and
firing
0 =
angle.
m If cos
the power delivered to the lossless battery
V(t) = 5 – 10 cos(ω t + 60°)V
V (3
0 +
2πW, a in2degreeπ a), where Vm = 80p mvolts and a is the Solution:
is 1600 is ______ (up to 2 decimal places).
α is the
firing firingIfangle.
angle. If thedelivered
the power power delivered to the lossless
to the lossless battery
I(t) = 5 + X cos(ω t) A
battery is 1600 W, α in degree is
is 1600 W, a in degree is ______ (up to 2 decimal____ (up to 2 places).
decimal
Where ω = 100 p radian/s. If the average power delivered to places).
the circuit is zero, then the value of X (in ampere) is _____
(up to 2 decimal places)
i(t) Hence, the co
+ Question
Hence, theNu
co
A three-phas
Question Nu
former has pr
V(t) Electrical Aof three-phase
0.3 Ω and
circuit Solution: Given former
phase of has0.02
pr
V of
full0.3 Ω%
load ande
Solution:
Solution: We V0know
Given = mthat(3 + cos α)
2π phase
powerof 0.02
factor
– Vm full load % e
V
V0m == 80p(3 + cos α)
2π power factorGii
Solution:
Solution: 80 π
⇒ VVm0 ==80p (3 + cos α) = 40 (3 + cos α) Solution: Gi
i(t) 2π H.V. side: R1/
+ 80 π

Current V0 =
through (3 + cos α) = 40 (3 + cos α)
battery,
2π H.V. side: R1/
V0 − E L.V. side: R2/
Current through I0 =
battery, (1)
R
Electrical V0 − E L.V. side: R2/p
V(t) Also given, 0 I = (1) Total iron los
circuit R
Pbattery = 80 × I0
Also as per problem Pbattery = 80 × I0 = 1600
given, Total iron loss
= 1600 Primary curre
Pbattery = 80 × I0 I = 20A
– I0 = 20 A 0
= 1600 Primary curre
P = V⋅I Equation (1) 40(3 + cos α ) − 80
(1) ⇒ 2020
I0 = =A cos α = 0
V(t) = 5 –10 {cosω t ⋅ cos 60° – sin ωt ⋅ sin 60°} 40 (3 + cos1α) − 80

Equation (1) 20 = cos α = 0
α =1 90°
V(t) = 5 – 5 cosω t + 5 3 sin ωt 40 (3 + cos α) − 80

Hence, 20a =
the correct = 90°
answer is 90°. cos α = 0
1 Secondary cu
Hence, the correct answer is a = 90°.
a = 90° Secondary cu
Hence, the correct answer is a = 90°.
M01_GATE EE 2018 paper 1.indd 20 6/18/2018 5:14:17 PM
Question Nu
900 × 103
= = 300 A A dc to dc co
3 × 3 × 103 tery bank. B2
\ Total Cu losses in a transformer battery bank w
= 3I12 /ph R1/ph + 3I 22 /ph R2 /ph the inductor,
GATE 2018 Solved Paper 4 – I | xiv-u with a switch
= 3 ×Electrical
104 × 0.3Engineering:
+ 3 × 9 × 10Set × 0.02
0.4. Once the
= 9000 + 5400 = 14400 W. the diode D to
Question Number: 60 Question Type: NAT ∴ Total Cu losses in a transformer
\ h at F.L. (in Watt) is __
The figure shown two buck converters connected in paral- = 3 I12 / ph R1/ph + 3 I 22 /ph R2/ph.
(U.P.F)
lel. The common input dc voltage for the converters has a = 3 × 104 × 0.3(1+×3900
× 9××10 103 4××1)0.02
value of 100 V. The converters have inductors of identical =
= 9000
(1 × + × 10=3 14400
5400
900 w. × 103 + 14, 400)
× 1) + (10
value. The load resistance is 1 Ω. The capacitor voltage has
negligible ripple. Both converters operate in the continu- ∴ η at F.L, = 0. 9736
ous conduction mode. The switching frequency in 1 kHz (U.P.F)
\%h = 97.36%
and the switch control signals are as shown. The circuit (1 × 900 × 103 × 1)
Hence, the correct
= answer is 97.36.
operates in the steady state. Assuming that the converters Solution: Gi
Question Number:(1 ×52
900 × 103 × 1) + (10 × 103 + 14
Question , 400)NAT
Type:
share the load equally, the average value of is1, the current
of switch S1 (in Ampere), is _____ (up to 2 decimal places) A 200 V DC series = 0. motor,
9736 when operating from rated voltage
Switch ON:
S1 while
∴ % η = 97.36%load, draws 10 A current and runs at
driving a certain
+ 1000 r.p.m. The total
Hence, the correct seriesis resistance
answer 97.36. is 1 Ω. the magnetic
S1 L
100 V C 1Ω
Switch control signals
circuits is assumed to be linear. At the same supply volt-
– Question Number:
age, the load torque 62 Question
is increased by 44%. The Type:
speed NAT
of the
S1 motor in r.p.m. (rounded to the nearest integer) is ______.
t A 200 V DC series motor, when operating from rated volt-
S2 L age while driving a certain load, draws 10 A current and
S2 Solution: Given
t runs at 1000 r.p.m. The total series resistance is 1 Ω. the 50 V
0 0.5 ms 1 ms D.C. series motor,is assumed to be linear. At the same sup-
magnetic circuits
ply voltage, the = 200torque
Vt load V is increased by 44%. The speed
Solution: Voltage Vs = 100 V
I
of the motor ain = 10 A, N1 = 1000tor.p.m.
r.p.m. (rounded the nearest integer) is
Current ID = 100/1 = 100 A 1
Total
_____.series resistance,
We know that input power = output power R = 1 Ω.
Solution: (Total) Inductor char
Pin = Pout Given, TL2 = 1.44TL1
Vs ⋅ Is = V0 ⋅ I0
For,
For, D.C.
D.C. series motor, f
series motor, faα IIa
a
100 × Is = 100 × 100 \ Tem α I a2
Is = 100 A TL1  Ia 
2

Is1 = 50 A = 1 
1.44TL  I a2 
Hence, the correct answer is 50.
I a1 1
Question Number: 61 Question Type: NAT \ = ⇒ I a2 = 12 A
I a2 1.2
A 3 – phase 900 kVA, 3kV/ 3 kV(∆/Y), 50 Hz trans-
former has primary (high voltage side) resistance per phase Eb1 = Vt − I a1 150 V
of 0.3 Ω and secondary (low voltage side) resistance per Ra = 200 – (10 × 1) = 190 V
phase of 0.02 Ω Iron loss of the transformer is 10 kW. The Eb2 = Vt − I a2
full load % efficiency of the transformer operated at unity Ra = 200 – (12 × 1) = 188 V
power factor is ______ (up to 2 decimal places). Eb = Ka f w IL
Solution: Eb a IaN ( f a Ia)
Primary current, I1/Ph Eb1 I a1 N1
= ×
I1 (line) Eb2 I a2 N2 0
=
3
190 1 1000 Power tra
1 900 × 103 ⇒ = ×
= × = 100A 188 1.2 N 2
3 3 × 3 × 103 \ N2 = 824.561 rpm
Secondary current, I2 /ph Hence, the correct answer is 824.56.
= I2(line) Question Number: 63 Question Type: NAT
900 × 10 3
A dc to dc converter shown in the figure is charging a bat-
= = 300A
3 × 3 × 103 tery bank. B2 whose voltage is constant 150 V. B1 is another
battery bank whose voltage is constant at 50 V. The value of
GATE_EEE-2018.indd 76

M01_GATE EE 2018 paper 1.indd 21 6/18/2018 5:14:20 PM


xiv-v  |  GATE 2018 Solved Paper Electrical Engineering: Set – I

the inductor, L is 5 mH and the ideal switch, S is operated Question Number: 64 Question Type: NAT
with a switching frequency of 5 kHz with a duty ratio of The equivalent circuit of a single phase induction motor
0.4. Once the circuit has attained steady state and assuming is shown in the figure, where the parameters are R1 = R21
the diode D to be ideal, the power transferred from B1 to B2 = Xl1 = X l12 = 12 Ω, XM = 240 Ω. And s is the slip. At no
(in Watt) is ______ (up to 2 decimal places) – load, the motor speed can be approximated to be the syn-
chronous speed. The no – load lagging power factor of the
motor is ______ (up to 3 decimal places.)

Solution: 
Switch ON:
VL
+ 5 mH –

I
50 V 150 V

Solution: 
VL = 50 V At no-load, Nr = Ns
Inductor charges linearly. Ns - Nr −
∴ S= =0
LdI Ns
VL =
dt ∴ The equivalent circuit becomes,
TON
VL
∫ dI = L ∫ dt
0

V
Δ I = L TON [TON = DT ]
L
VB
2

150 V

IL
0.8

0 t
0.4T 0.6T T

Power transferred to B2 is given by


0.6T
1
T 0.∫4T
P= 150 × I L dt

1 0.8T
× 0.2T × =
× 150 = 12W
T 2
Hence, the correct answer is 12.

M01_GATE EE 2018 paper 1.indd 22 6/18/2018 5:14:23 PM


GATE 2018 Solved Paper Electrical Engineering: Set – I  |  xiv-w

V ∠0°
∴ No load current, I0 =
Z ∠θ


Z eq = 138.56 ∠83.9 Ω at Resonance
Zab = (Zab)real
∴ No load P.F = cos(83.9)
= 0.106 lagging Imag {Zab} = 0
Hence, the correct answer is 106. ⎛ 1 ⎞ Set – 1
lxxviii | GATE
Z ab =2018
jω L Solved
+ ⎜100 Paper

⎝ SC ⎠
Question Number: 65 Question Type: NAT
The voltage v(t) across the terminal a and b as shown in 1
100 × Z
the figure, is a sinusoidal voltage having a frequency ω = Zab = j ωL + SC
100  radian/s. When the inductor current i(t) is in phase 1
100 +
with the voltage v(t), the magnitude of the impedance Z (in SC Z
Ω) seen between the terminals a and b is ______ (up to 2 100
decimal places). Zab = j ωL +
I + 100 SC At resonance,
100 Z
= j ωL +
lxxviii | GATE 2018 Solved Paper Set – 1 1 + j100 × 100 × 10 −6 × 100

1 100
100 × Zab = jωL +
Zab = j ωL + SC 1 + j1
1
100 + 100{1 − j}
SC Zab = jωL +
1 + (1) 2
100
Zab = jωL +
I + 100 SC At resonance,
Solution:  Consider the circuit given below
100 100
= j ωL + Zab = Zreal = = 50 Ω
2
1 + j100 × 100 × 10 −6 × 100
Hence, the correct answer is 50.

M01_GATE EE 2018 paper 1.indd 23 6/18/2018 5:14:25 PM


This page is intentionally left blank.

F01_Gate_EE_978-93-325-7605-6_FM.indd 14 6/19/2017 7:22:37 PM


Detailed Analysis of GATE 2017 Papers
GATE EE Solved 2017 Paper (Set 1) Detailed Analysis
1 Mark 2 Mark Total
Subject Questions Questions Marks

General Aptitude 5 5 15
Engineering Mathematics 5 4 13
Networks 2 3 8
Control Systems 3 4 11
Digital Electronics 1 2 5
Signals and Systems 2 2 6
E.M.F. 1 1 3
Analog Electronics 3 2 7
Electrical Machines 2 5 12
Power Systems 2 3 8
Power Electronics 2 2 6
Measurements 2 2 6
Total 100

GATE EE Solved 2017 Paper (Set 2) Detailed Analysis


1 Mark 2 Mark Total
Subject Questions Questions Marks
General Aptitude 5 5 15
Engineering Mathematics 5 5 15
Networks 0 4 8
Signals and Systems 2 2 6
Analog Electronics 0 3 6
Digital Electronics 1 1 3
Power Electronics 4 3 10
Electrical & Electronic Measurements 2 1 4
Electrical Machines 2 4 10
Electromagnetic Theory 2 1 4
Control Systems 4 3 10
Power System 3 3 9
Total 100

F01_Gate_EE_978-93-325-7605-6_FM.indd 15 6/19/2017 7:22:37 PM


GATE 2017 Solved Paper
Electrical Engineering
Set - 1
Number of Questions: 65 Total Marks: 100.0

Wrong answer for MCQ will result in negative marks, (-1/3) for 1 Mark Questions and (-2/3) for 2 Marks
Questions.

General Aptitude
Number of Questions: 10 Section Marks: 15.0

Q. 1 to Q. 5 carry 1 mark each and Q. 6 to Q. 10 carry In both the cases, we get the minimum of x and y, the
2 marks each correct option is (b).
Question Number: 1 Question Type: MCQ Hence, the correct option is (B).
( x + y )− | x − y | Question Number: 2 Question Type: MCQ
The expression is equal to
2 “The hold of the nationalist imagination on our colo-
(A) The maximum of x and y nial past is such that anything inadequately or improp-
(B) The minimum of x and y erly nationalist is just not history.”
(C) 1 Which of the following statements best reflects the
(D) None of the above author’s opinion?
Solution: (A) Nationalists are highly imaginative.
(B) History is viewed through the filter of nation-
As per question expression is
alism.
( x + y )− | x − y | (C) Our colonial past never happened.
2 (D) Nationalism has to be both adequately and
Also, we know modulus of any number should be a properly imagined.
positive value. Solution:  Hence, the correct option is (B).
Case (1): x > y (here, y is minimum) then |x – y| = (x – y) Question Number: 3 Question Type: MCQ
positive value then
A contour line joins locations having the same height
( x + y) − ( x − y) ( x + y) − ( x − y) above the mean sea level. The following is a contour
=
2 2 plot of a geographical region. Contour lines are shown
2y at 25 m intervals in this plot. If in a flood, the water
=
2 level rises to 525 m. Which of the villages P, Q, R, S,
= y ( minimum of x and y ) T get submerged?
Case (2): y > x (here, x is minimum) then |x – y| 425 Q
= (y – x) (positive value) then 450
R
550
( x + y )− | x − y | ( x + y ) − ( y − x )
=
2 2 P
2x 550
= T
  2 500
450
= x ( minimum of x and y ) 500 S

M01_GATE_9789332576063_ME_SET1.indd 16 6/20/2017 7:34:28 AM


GATE 2017 Solved Paper Electrical Engineering: Set – 1 | xvii

(A) P, Q (B) P, Q, T (A) 21 (B) 18


(C) R, S, T (D) Q, R, S (C) 16 (D) 14
Solution:  Solution: 
Height above mean sea level for Colour – Red, Pink, Blue, White
P ⇒ H P = 575 m Arun ← Red
Q ⇒ H Q = 525 m
R ⇒ H R = 475 m Shweta ← White

S ⇒ H S = 475 m
T ⇒ H T = 500 m Case 1: Arun chooses pink shirt then Shweta will have
two options Red and blue so number of ways
if water level in a flood is 252 m then R,S,T will be n1 = 1C1 ⋅2 C1 ⋅2 C1 ⋅1 C1 = 4
submerged.
Case 2: Arun chooses bule shirt, Shweta will have two
Hence, the correct option is (C).
options Red and Pink, so
Question Number: 4 Question Type: MCQ n2 = 1C1 ⋅2 C1 ⋅2 C1 ⋅1 C1 = 4
Six people are seated around a circular table. There are
Case 3: Arun chooses white, then Shweta will have
at least two men and two women. There are at least three
three options, so
right-handed persons. Every woman has a left-handed
person to her immediate right. None of the women are n3 = 1C1 ⋅3 C1 ⋅2 C1 ⋅1 C1 = 6
right-handed. The number of women at the table is Total number of ways = 4 + 4 + 6 = 14.
(A) 2
Hence, the correct option is (D).
(B) 3
(C) 4 Question Number: 6 Question Type: MCQ
(D) Cannot be determined The probability that a k-digit number does NOT con-
Solution:  tain the digits 0, 5, or 9 is
(A) 0.3k (B) 0.06k
Total perosns–6
(C) 0.7 k
(D) 0.9k
Conditions:
Solution:
1. Atleast two men and two women
k-digit number
2. Atleast 3 right-handed persons
3.  Every women has a left-handed person to her
immediate right and all women are left handed. digit 1 2 3 4 k−1 k

Let us choose at least two women (minimum) then total Excluding digits 0, 5 or 9
left-handed persons = 2 + 1 (1 man is immediate right
Probability
of one woman when both woman are sitting together) =
3. Remaining three will be right-handed. C1 ⋅7 C1 ⋅7 C1  7C1
7
( k -times)
P=
Hence, the correct option is (A).
10
C1 ⋅10 C1 ⋅10 C1  10C1 ( k -times)
7.7.7… 7 ( k -times)
Question Number: 5 Question Type: MCQ =
10.10.10 …10 ( k -times)
Arun, Gulab, Neel, and Shweta must choose one shift
each from a pile of four shirt coloured red, pink, blue, or   P = (0.7)k.
and white, respectively. Arun dislike the colour red and Hence, the correct option is (C).
Shweta dislikes the colour white. Gulab and Neel like
all the colours. In how many different ways can they Question Number: 7 Question Type: MCQ
choose the shirts, so that no one has a shirt with a col- Rahul, Murali, Srinivas, and Arul are seated around
our he or she dislikes? a square table. Rahul is sitting to the left of Murali.

M01_GATE_9789332576063_ME_SET1.indd 17 6/20/2017 7:34:29 AM


xviii | GATE 2017 Solved Paper Electrical Engineering: Set – 1

Srinivas is sitting to the right of Arul. Which of the fol- Solution:


lowing pairs are seated opposite each other? Beside → “next to”
(A) Rahul and Murali
Besides → “Except”
(B) Srinivas and Arul
(C) Srinivas and Murali Hence, the correct option is (D).
(D) Srinvas and Rahul Question Number: 9 Question Type: MCQ
Solution: After Rajendra Chola returned from his voyage to
Srinivas Indonesia, he _____ to visit the temple in Thanjavur.
(A) Was wishing (B) Is wishing
(C) Wished (D) Had wished
Rahul Arul Solution:  Hence, the correct option is (C).
Question Number: 10 Question Type: MCQ
Murali Find the smallest number y such that y × 162 is a per-
fect cube
Hence, the correct option is (C). (A) 24 (B) 27 (C) 32 (D) 36
Question Number: 8 Question Type: MCQ Solution:
Research in the workplace reveals that people work for y × 162 as perfect cube
many reasons _____ 162 = 2 × 3 × 3 × 3 × 3
(A) Money beside to make it perfect cube y = 2 × 2 × 3 × 3
(B) Beside money
(C) Money besides or y = 36
(D) Besides money Hence, the correct option is (D).

Electrical Engineering
Number of Questions: 55 Section Marks: 85.0

Q. 11 to Q. 35 carry 1 mark each and Q. 36 to Q. 65 for n = 1,


carry 2 marks each y[1] = (1 + ( −1)1 ) x[1] = 0
Question Number: 11 Question Type: MCQ for n = 2,
Consider the system with following input-output   y[2] = (1 + ( −1) 2 ) x[2] = 0
relation
for n = 3,
y[n] = (1 + ( −1) n ) x[n]
  y[3] = (1 + ( −1)3 ) x[3] = 0
Where, x[n] is the input and y[n] is the output. The sys-
tem is Therefore, for odd values of “n” output will always be
(A) Invertible and time invariant. zero, so system is non-invertible.
(B) Invertible and time varying. To check time invariance
(C) Non-invertible and time invariant. For delayed input,
(D) Non-invertible and time varying. y[n1n0 ] = (1 + ( −1) n ) x[n − n0 ] (1)
Solution:
For delayed response,
The input-output relation is given as
y[n1n0 ] = (1 + ( −1) n − n0 ) x[n − n0 ] (2)
y[n] = (1 + ( −1) ) x[n]
n
For time invariant system output for delayed input
We know that, if there is a one-to-one correspondence should be equal to delayed response.
between its input and output signals then the system is Hence, this system is time varying.
said to be invertible Hence, the correct option is (D).

M01_GATE_9789332576063_ME_SET1.indd 18 6/20/2017 7:34:31 AM


e − j 2π nt 1 e − j 2π nt
=t⋅ 1
−∫ 1⋅ dt
− j 2π nt
0 0 − j 2π nt
−1 1
= − 2 2 2 (ee − j 2π n − 1)
− j 2π n − j 4π n
GATE 2017 Solved Paper−1Electrical Engineering: Set – 1 | xix
Gn =
j 2π n
Question Number: 12 Question Type: NAT −1
Consider g(t) = Gn =
j 4π
t − t , t≥o 1
 , where t ∈ R G2 = = 0.0796
t − t , otherwise 4π
Here, t represents the largest integer less than or Hence, the correct answer is (0.0796).
equal to t and t denotes the smallest integer greater
than or equal to t. The coefficient of the second har- Question Number: 13 Question Type: MCQ
monic component of the Fourier series representing The Boolean expression AB + AC + BC simplifies to
g(t) is _____
BC + AC (B)
(A) AB + AC + B
Solution:
(C) AB + AC (D) AB + BC
t − t , t≥o
g (t ) =  Solution: 
t − t , otherwise
The Boolean expression is
g(t)
f = AB + AC + BC
drawing k-map for above expression, we get
1
BC
A B
00 01 11 10
−3 −2 −1 0 1 2 3 t 0 1

A 1 1 1 1

C
T =1
2π From the above figure, we get
ω0 = = 2π
T
g (t ) = t 0 ≤ t ≤1 f = AC + BC

Hence, the correct option is (A).
g (t ) = ∑Ge
n =−∞
n
− jnω0 t

Question Number: 14 Question Type: MCQ


1 T2
T ∫−T 2
Gn = g (t )e − jnω0 t dt Let z(t) = x(t)*y(t), where “*” denotes convolution.
Let  c be a positive real-valued constant. Choose the
1 1
= ∫ (t )e − j 2π nt dt correct expression for z(ct)
1 0 (A) c.x(ct) * y(ct) (B) x(ct) * y(ct)
1
= ∫ (t )e − j 2π nt dt (C) c.x(t) * y(ct) (D) c.x(ct) * y(t)
0
1 Solution:
Gn = ∫ te − j 2π nt dt
0
Given that
e − j 2π nt 1 e − j 2π nt z (t ) = x (t ) * y (t )
=t⋅ 1
−∫ 1⋅ dt
− j 2π nt
0 0 − j 2π nt
taking Fourier transform, we get
−1 1
= − (ee − j 2π n − 1) Z ( jω ) = X ( jω ) ⋅ Y ( jω ) (1)
− j 2π n − j 2 4π 2 n2
−1 1  jω 
Gn = z (t ) → Z  (2)
j 2π n c  c 
−1
Gn =
j 4π
1
G2 = = 0.0796

M01_GATE_9789332576063_ME_SET1.indd 19 6/20/2017 7:34:34 AM
xx | GATE 2017 Solved Paper Electrical Engineering: Set – 1

Also, by using Eq. (1) fault current does not exceed 3.75 pu if a single line to
 jω   jω   jω  ground fault occurs at the terminals is _____ (assume
Z  = X  c  ⋅Y  c  fault impedance to be zero). (Give the answer up to one
 c      decimal place).
1  jω  1  jω   jω 
∴ Z = X ⋅Y Solution: 
c  c  c  c   c 
Given: Positive-sequence reactance X1 = 0.2 pu
Multiplying and dividing RHS by c, we get Negative-sequence reactance   X2 = 0.2 pu
1  jω   1  jω  1  jω   Zero-sequence reactance X0 = 0.1 pu
Z  = ⋅X ⋅ Y  
c  c  c  c  c  c  Fault current if = 3.75 pu
z (t ) = c ⋅ x(ct ) * y(ct ) For single line to ground fault, the relation for fault
Hence, the correct option is (A). current is

Question Number: 15 Question Type: NAT 3E


if =
In the converter circuit shown in the figure, the X1 + X 2 + X 0 + 3 X n
switches are controlled such that the load voltage v0(t)
is a 400 Hz square wave. In above expression, Xn is the reactance connected
between neutral and ground.
Therefore, for E = 1 pu
S1 S3
3E
if = = 3.75
X1 + X 2 + X 0 + 3 X n
220 V + LOAD

+ V0(t) −
Substituting the values and simplifying the above
S4 S2
expression for Xn, we get
X n = 0.1 pu
The RMS value of the fundamental component of v0(t)
in volts is _____ Hence, the correct answer is (0.1 pu).
Solution: Question Number: 17 Question Type: MCQ
We know that for single phase full bridge inverter, Two passive two-port networks are connected in cas-
cade as shown in Figure. A voltage source is connected
n
4Vs at port 1.
v0 = ∑
n =1, 3, 5… nπ
sin n ωt Volts
I1 I2 I3
+ +
V1 ∼ Two port Two port +
Also the RMS value of fundamental component will be − network 1 − V2 V
network 2 − 3
4Vs 1 4 × 220 1 Part 1 Part 2 Part 3
× = ×
π 2 π 2
Given:
= 198.069 Volts.
V1 = AV
1 2 + B1 I 2
Hence, the correct answer is (198.069).
I1 = C1V2 + D1 I 2
Question Number: 16 Question Type: NAT V2 = A2V3 + B2 I 3
The positive-, negative-, and zero-sequence reactances I 2 = C2V3 + D2 I 3
of a wye-connected synchronous generator are 0.2 pu,
0.2 pu and 0.1 pu, respectively. The generator is on A1, B1, C1, D1, A2, B2, C2, and D2 are the generalized cir-
open circuit with a terminal voltage of 1 pu. The mini- cuit constants. If the Thevenin equivalent circuit at port
mum value of the inductive reactance, in pu, required 3 consists of a voltage source VT and an impedance ZT,
to be connected between neutral and ground so that the connected in series, then

M01_GATE_9789332576063_ME_SET1.indd 20 6/20/2017 7:34:36 AM


GATE 2017 Solved Paper Electrical Engineering: Set – 1 | xxi

V1 A B + B1 D2 Question Number: 18 Question Type: NAT


VT =
(A) , ZT = 1 2
A1 A2 A1 A2 + B1C2 A 220 V DC series motor runs drawing a current of
30 A from the supply. Armature and field circuit resist-
V1 A B + B1 D2 ances are 0.4 Ω and 0.1 Ω, respectively. The load
VT =
(B) , ZT = 1 2
A1 A2 + B1C2 A1 A2 torque varies as the square of the speed. The flux in the
V1 A B + B1 D2 motor may be taken as being proportional to the arma-
VT =
(C) , ZT = 1 2 ture current. To reduce the speed of the motor by 50%,
A1 A2 A1 A2
the resistance in ohms the should be added in series
V1 A B + B1 D2 with the armature is _____. (Give the answer up to two
VT =
(D) , ZT = 1 2
A1 A2 + B1C2 A1 A2 + B1C2 decimal places).
Solution:  Solution: 
Equivalent ABCD parameters for cascaded network are
30 Α
given by,
+
 A B   A1 B1   A2 B2  Rf = 0.1 Ω
C D  = C D1  C2 D2 
   1

 A A + B1C2 A1 B2 + B1 D2  Ra = 0.4 Ω
 =  1 2
C1 A2 + D1C2 C1 B2 + D1 D2 
-
Therefore,

V1   A B  V3  Back E.m.f. E1 = 220 − 30(0.1 + 0.4)


 =   = 205V
 I 2  C D   I 3 
Torque, τ = φ Ia
Now, open circuit voltage VT = V3 when I3 = 0.
V1 = AV3 {Where, φ = flux, Ia = armature current}
So,
= AVT τ = I a2 (1)
V1
⇒ VT = As in series motor φ ∝ I a
A
V1 Also, τ ∝ N 2 (2)
⇒ VT =
A1 A2 + B1C2
Where N is the speed of motor
Voltage-source is short circuited to calculate Thevenin Using Eqs. (1) and (2)
equivalent impedance ZT. So, for voltage source of V3
feeding current (–I3) I a2 ∝ N 2 or I a ∝ N (3)

V3 Therefore, to reduce speed by 50%, Ia will reduce to


ZT =
−I3 50%, i.e., 15 A.
0 = AV3 + BI 3 Now back emf will change to
B E2 = 220 − 15( R + 0.1 + 0.4)
⇒ ZT =
A
A1 B2 + B1 D2 Where R is the external resistance added in series with
ZT =
A1 A2 + B1C2 armature.
Hence, the correct option is (D). Since, E ∝φ⋅N

M01_GATE_9789332576063_ME_SET1.indd 21 6/20/2017 7:34:39 AM


xxii | GATE 2017 Solved Paper Electrical Engineering: Set – 1

So, E1 ∝ φ1 ⋅ N1 Question Number: 20 Question Type: MCQ


E 2 ∝ φ2 ⋅ N 2 A function f(x) is defined as

φ  ex , x < 1
φ2 = 1 f ( x) =  ,
1nx + ax + bx x ≥ 1
2
2
N1 Where x ∈ R.
N2 =
2 Which one of the following statement is TRUE?
Thus, (A)  f(x) is NOT differentiable at x = 1 for any val-
205 = φ1 N1 (4) ues of a and b.
(B)  f(x) is differentiable at x = 1 for the unique
220 − 15( R + 0.5) = φ2 N 2 (5)
value of a and b.
Dividing Eq. (5) by Eq. (4), (C)  f(x) is differentiable at x = 1 for all values of
220 − 15( R + 0.5) φ2 N 2 (φ1 / 2) ⋅ ( N1 / 2) a and b such that a + b = e.
= = (D)  f(x) is differentiable at x = 1 for all values of
205 φ1 N1 φ1 ⋅ N1
a and b
220 − 15( R + 0.5) 1
= Solution: 
205 4
On solving,  ex , x < 1
f ( x) =  
1n x + ax + bx x ≥ 1
2
R = 10.75 Ω
Hence, the correct answer is (10.75 Ω). f ( x ) − f (1)
Left-hand derivative = lim
Question Number: 19 Question Type: MCQ x →1 x −1
The logical gate implemented using the circuit shown Where,  f(1) = ln (1) + a(1)2 + b (1) = a + b
in the figure where, V1 and V2 are inputs (with 0 V as
So,
digital 0 and 5 V as digital 1) and VOUT is the output, is
5V e x − ( a + b)
Left-hand derivatives (LHD) = lim
x →1 x −1
1 kΩ This limit exists if
VOUT
V1 1 kΩ e = a + b(1)
Q1 1 kΩ
V2
Q2 Then
ex
LHD = lim =e
x →1 1
(A) NOT (B) NOR
(C) NAND (D) XOR f ( x ) − f (1)
Right-hand derivative (RHD) = lim
Solution: 
x →1 x −1
1n x + ax 2 + bx − ( a + b)
From the given circuit it can be deduced that Q1 will be = lim
x →1 x −1
ON when V1 is high, Q2 is ON when V2 is high
Truth Table This limit exist if
V1 Q1 V2 Q2 VOUT 1 + 2a + b = a+b  or  a = -1(2)
High ON High ON Low
High ON Low OFF Low
Hence,
Low OFF High ON Low 1
Low OFF Low OFF High + 2ax + b
This is the truth table of NOR gate. RHD = lim x = 1 + 2a + b
x →1 1
Hence, the correct option is (B).

M01_GATE_9789332576063_ME_SET1.indd 22 6/20/2017 7:34:43 AM


GATE 2017 Solved Paper Electrical Engineering: Set – 1 | xxiii

LHD will be equal to RHD if a = –1 and b = e + 1. −


t
Hence iL (t ) = 2.5 e 1/ 4
Hence, f(x) is differentiable at x = 1 for unique values
of a and b. or iL (t ) = 2.5 e −4 t
Hence, the correct option is (B). Hence, the correct option is (A).
Question Number: 21 Question Type: MCQ Question Number: 22 Question Type: NAT
The switch in the figure was closed for a long time. It A three-phase, three winding Δ/Δ/Y(1.1 kV/6.6
is opened at t = 0. The current in the inductor of 2 H kV/400 V) transformer is energized from AC mains at
for t ≥ 0, is the 1.1 kV side. It supplies 900 kVA load at 0.8 power
6Ω 8Ω factor lag from the 6.6 kV winding and 300 kVA load
at 0.6 power factor lag from the 400 V winding. The
RMS line current in ampere drawn by the 1.1 kV wind-

+ 8Ω 32 Ω ing from the mains is _____. (Give the answer up to


− 50 V 2H
32

one decimal place).


Solution: 
(A) 2.5e–4t (B) 5e–4t Given
(C) 2.5e –0.25t
(D) 5e–0.25t Δ Δ (1.1 kV/6.6 kV 400 V) Tr.
Primary Secondary a' Tertiary a"
Solution: 
400 0
For t = 0– circuit can be represented as shown in the A 3
C n"
figure
B b"
b
6Ω 8Ω c
c"

50 V + 8Ω 2H Let us assume Va“n” as reference,



400
∴ Va “ n ” ∠0°
3
Inductor can be taken as short circuit at steady state. ⇒
lag Load at tertiary → 300 kVA, 0.6 Pf
So, current in inductor at t = 0– will be
50 8 ∴ I a′′ = 433.01∠ − 53.13 A
iL (0 − ) = ×
6 + (8118) (8 + 8) V AB 1.1× 103
⇒ iL = 2.5 A ∵ = = 4.763
V a“ n ” 400 / 3
On opening of switch at t = 0, iL can be given by iL (t) = [V AB ← Va“ n ” are in phase]
iL(0) e–t/τ where τ = L/R
R is Requivalent across L to calculate Requivalent, ∴ I AB (corresponding to I a" )

6Ω = 90.91∠ − 53.13 A

We know,

8Ω 32 Ω L=2H
I A = I AB − I CA
32

Req = 8 Ω = 90.91∠ − 53.13 − 90.91∠66.86


= 3 × 90.91× ∠( −53.13 − 30)
L 2 1
τ= = =
R 8 4 = 157.46 ∠ − 83.13 A

M01_GATE_9789332576063_ME_SET1.indd 23 6/20/2017 7:34:45 AM


xxiv | GATE 2017 Solved Paper Electrical Engineering: Set – 1

Now, Load at secondary 900 kVA, .8 pF lag the line c is a straight line passing through origin shown
in the figure and its equation is given by y = x
V AB 1.1 1
= =
Va‘b’ 6.6 6 y

i.e., V a‘b’ = 6.6∠0° kV (1, i)


i
∴ I a‘b’ = 45.45 ∠ − 36.87 A
I AB = (corresponding to I a ’b ’ ) = 272.73 ∠− 36.87 C
∵ I A = I AB − I AC
∴ I A (corresponding to I a‘b’ ) = 3 × 272.73∠−36.87 − 30
= 472.38 ∠ − 66.87 A 1 X
∴ Total line current from supply.
substituting y = x in given integral, we get
I L = I A (corresponding to load ) + I A (corresponding
I = ∫ ( x 2 + iy 2 )( dx + idx )
∆ load )
c
= 157.46∠ − 83.13 + 472.38∠ − 66.877
= ∫ x 2 (1 + i )(1 + i )dx
= 625.01∠ − 70.91 c

∴ RMS line current = 625.01 A


= ∫ x 2 (1 − 1 + 2i )dx
Hence, the correct answer is (625.1 A). c

Question Number: 23 Question Type: MCQ = ∫ 2i x 2 dx


Consider the line integral c

I = ∫ ( x + iy )dz,
2 2 1
= 2i ∫ x 2 dx
c 0

Where z = x + iy. The line C is shown in the Figure. 1


 x3 
= 2i  
y  2 0
(1, i)
i 1 0 
= 2i  − 
3 3
C 2i
=
3
Hence, the correct option is (B).
(0, 0) 1 Question Number: 24
X Question Type: NAT
The figure shows the single line diagram of a power
The value of I is system with a double circuit transmission line. The
1 2 expression for electrical power is 1.5 sin δ, where δ
(A) i (B) i
2 3 is the rotor angle. The system is operating at the sta-
3 4 ble equilibrium point with mechanical power equal to
(C) i (D) i 1 pu. If one of the transmission line circuits is removed,
4 5
the maximum value of δ, as the rotor swings, is 1.221
Solution:  radian. If the expression for electrical power with one
Given integral transmission line circuit removed is Pmax sin δ, the
I = ∫ ( x + iy )dz , z = x + iy
2 2 value of Pmax, in pu is _____ (Give the answer up to
c three decimal places.)

M01_GATE_9789332576063_ME_SET1.indd 24 6/20/2017 7:34:47 AM


GATE 2017 Solved Paper Electrical Engineering: Set – 1 | xxv

VSS
Vin
+

Solution:  D
- VSS
With double circuit transmission line VO

PeI = 1.5 sin δ


R
with single line
PeII = Pmax sin δ

1.5 PeI = 1.5 sin δ (A) VO

Pmax PeII = Pmax sin δ


A2
Pm =1pu
A1
Vin
δ δ δ
12m (B) VO

Here,

 1 
δ1 = sin −1   = 41.81° or 0.7297 radian
 1.5  Vin
δ m = 1.221 radian or 69.96°
(C) VO

For stability
A1 = A2
δ2 δm
∫ δ1
(1 − Pmax sin δ )dδ = ∫ (1 − Pmax sin δ − 1)dδ
δ2
Vin

δ2 δ2 δm δm (D) VO
δ δ1 − Pmax ( − cos δ ) δ1 = Pmax ( − cos δ ) δ2 −δ δ2

Pmax = (cos δ m − cos δ1 ) = δ1 − δ m

Substituting values and solving for Pmax Vin


Pmax (cos 69.96° – cos 41.81°) = 41.81°– 69.96.°
Solution: 
Pmax = 1.220 pu For Vin < 0
Output of operational amplifier will be negative hence
Hence, the correct answer is (1.220 pu).
due to presence of diode (reversed biased) in this case
Question Number: 25 Question Type: MCQ output will be 0
The approximate transfer characteristic for the circuit For Vin > 0
shown in the figure with an ideal operational amplifier Diode will be forward biased so Vin = V0
and diode will be Thus transfer characteristics

M01_GATE_9789332576063_ME_SET1.indd 25 6/20/2017 7:34:50 AM


xxvi | GATE 2017 Solved Paper Electrical Engineering: Set – 1

VO Solution: 
Ia = 150 A
+

Rf
145 V = Vt
Vin Ra = 0.1 Ω

Hence, the correct option is (A).


-
Question Number: 26 Question Type: MCQ
A load is supplied by a 230 V, 50 Hz source. The active N = 800 rpm (speed of generator)
power P and the reactive power Q consumed by the Since, back emf E = Vt + I a Ra (1)
load are such that 1 kW ≤ P ≤ 2 kW and 1 kVAR ≤
Q ≤ 2 kVAR. A capacitor connected across the load And E ∝ fN(2)
for power factor correction generates 1 kVAR reactive For separately excited generator f remains
power. The worst case power factor after power factor Constant so E ∝ N (3)
correction is
For N = 800 rpm
(A) 0.447 lag
(B) 0.707 lag E1 = 145 + 150 × 0.1 = 160 V
(C) 0.894 lag Using Eq. (3)
(D) 1 E1 ∝ N1
Solution:  or 160 ∝ 800 (4)
For worst case power factor For N = 1000 rpm
E2 ∝ 1000 (5)
P = 1 kW,
On solving Eqs. (4) and (5)
Q = 2 kVAR.
E2 = 200 V
After addition of capacitor for power factor correction
Thus 200 = 145 + I a × 0.1
Q becomes 2–1 = 1 kVAR new
200 − 145
Ia =
 Q 0.1
Pf = cos  tan −1 
 P or I a = 550 A

 1 Hence, the correct Answer is (550 A).


= cos  tan −1 
 1 Question Number: 28 Question Type: MCQ
dy
= cos 45° Consider the differential equation (t 2 − 81) + 5ty = sin(t )
dt
or Pf = 0.707 lag sin(t) with y(1) = 2π. There exists a unique solution for
this differential equation, when t belongs to the
Hence, the correct option is (B). interval
(A) (–2, 2) (B) (–10, 10)
Question Number: 27 Question Type: NAT (C) (–10, 2) (D) (0, 10)
A separately excited DC generator supplies 150 A to a
145 V DC grid. The generator is running at 800 RPM. Solution:
The armature resistance of the generator is 0.1 Ω. If Given differential equation is
the speed of the generator is increased to 1000 RPM, dy
the current in amperes supplied by the generator to the (t 2 − 81) + 5ty = sin t (1)
DC grid is _____. (Give the answer up to one decimal dt
place). Initial condition y(1) = 2π

M01_GATE_9789332576063_ME_SET1.indd 26 6/20/2017 7:34:52 AM


GATE 2017 Solved Paper Electrical Engineering: Set – 1 | xxvii

Converting the given equation into standard form 2 3 2 3


(A) ≤ D ≤ ⋅ (B) ≤D≤ ⋅
dy  5t  sin t 5 5 3 4
+ y= 2 (2)
dt  t 2 − 81  t − 81 1 2
0 ≤ D ≤ 1 ⋅ (D)
(C) ≤D≤ ⋅
This is of the form 3 3
dy Solution: 
+ py = Q
dt For buck-boost converter
5t sin t α
Where P= , Q= V0 = Vs
t 2 − 81 t 2 − 81 1−α

We know integrating factor (IF) = e ∫


pdt
where α is duty cycle of converter
5t
∫ t 2 −81dt Vs = supply voltage
=e
V0 = output voltage
5  2t 
∫  
=e 2  t 2 − 81 
For Vs = 32 V and V0 = 48 V
5 5
1n ( t 2 − 81) 2
=e 2
[∵ e1n x = x ] α
48 = × 32
5 1−α
IF = (t 2 − 81) 2
3
y(IF) = ∫ Q IF dt + c ⇒ α=
5
5 For Vs = 72 V and V0 = 48 V
sin t 2
y(t 2 − 81)5 2 = ∫ (t 2 − 81) dt + c α
(t − 81)
2
48 = × 72
5 1−α
sin t (t 2 − 81) 2 −5
2
y=∫ 5
dt + c(t 2 − 81) 2
α=
(t − 81)
2 2
5
−5 2 3
y = ∫ sin t (t 2 − 81) −1 dt + c(t 2 − 81) 2
≤α ≤
5 5
and solving from the options by verifying initial condi-
Hence, the correct option is (A).
tion, we get unique solution If t = ±9 then solution is
not unique hence range (–10, 10), (–10, 2), (0, 10) can Question Number: 30 Question Type: MCQ
be eliminated, then left option is (–2, 2). The load shown in the figure is supplied by a 400 V
Hence, the correct option is (A). (line-to-line), 3-phase source (RYB sequence). The
load is balanced and inductive, drawing 3464 VA.
Question Number: 29 Question Type: MCQ
When the switch S is in position N, the three wattme-
The input voltage VDC for the buck-boost converter ters W1, W2 and W3 read 577.35 W each. If the switch
shown in the figure varies from 32 V to 72 V. Assume is moved to position Y, the readings of the wattmeters
that all components are ideal, inductor current is con- in watts will be:
tinuous and output voltage is ripple free. The range of
duty ratio D of the convector for which the magnitude W1
R
400 v, 3-phase source

of the steady-state output voltage remains constant at


48 V is W2
S1 Y
LOAD

- W3
B
VDC + L C R Y
VO
-
+
S N
N

M01_GATE_9789332576063_ME_SET1.indd 27 6/20/2017 7:34:56 AM


xxviii | GATE 2017 Solved Paper Electrical Engineering: Set – 1

(A) W1 = 1732 and W2 = W3 = 0 Reactance


(B) W1 = 0, W2 = 1732 and W3 = 0 is 0.05 pu
(C) W1 = 866, W2 = 0, W3 = 866 Susceptance Susceptance
(D) W1 = W2 = 0 and W3 = 1732 is 0.05 pu is 0.05 pu

Solution: 
If this transmission line is removed from service, what
Apparent power = 3464 VA is the modified bus admittance matrix?
Real power = 3 × 577.35 W = 1732.05 Watts  − j19.9 j 20 0 
When switch is moved to position Y  − j 39.9 j 20  pu
(A)  j 20
⇒ Voltage across potential coil of watt-meter two is  0 j 20 − j19.9 
zero so W2 = 0
 − j 39.95 j 20 0 
For RYB phase sequence  j 20
(B)
 − j 39. 9 j 20  pu
VRY  0 j 20 − j 39.9 
VR
 − j19.95 j 20 0 
IR
 − j 39.9 j 20  pu
−VY
30°  j 20
(C)
φ  0 j 20 − j19.95
VBY
30°  − j19.95 j 20 0 
IB  − j 39.9 j 20  pu
φ  j 20
(D)
 j 20 j 20 − j19.95
VY
VB
Solution: 
It is given that y-bus
Voltage across potential coil of wattmeter one is VRY.
 − j 39.9 j 20 j 20 
Voltage across potential coil of wattmeter two is VBY. 
[Y ]3×3 =  j 20 − j 39.9 j 20 
So,
 j 20 j 20 − j 39.9 
W1 = VRY ⋅ I R ⋅ cos(30 + φ )
Converting the given transmission line parameters into
W2 = VBY ⋅ I B ⋅ cos(30° − φ )
Y parameters, we get
For Pf = 0.5; φ = cos −1 (0.5) = 60° Bus 1
y13 = - j20
Bus 3
Thus
W1 = VRY ⋅ I R ⋅ cos(30 + 60°) = 0
W2 = VBY ⋅ I B ⋅ cos(30 − 60°) = 1732 W
Hence, the correct option is (D). y11 = j0.05 y33 = j0.05
Question Number: 31 Question Type: MCQ
The bus admittance matrix for a power system network
is The parameters Y11, Y13, Y31, Y33 will get affected when-
 − j 39.9 j 20 j 20  ever we remove the transmission line between Bus 1
 j 20 − j 39 .9 j 20  pu and Bus 3

 j 20 j 20 − j 39.9  Y11= –j39.9 – y11 – y13
= –j39.9 – (j0.05) – (–j20)
There is a transmission line, connected between buses
1 and 3, which is represented by the circuit shown in = –j39.9 – j0.05 + j20
figure = –j19.95

M01_GATE_9789332576063_ME_SET1.indd 28 6/20/2017 7:34:58 AM


TF =
∑P ∆k k


1 1 1 1 k
P1 = ⋅ ⋅ k1 ⋅ ⋅ = 1 2
GATE 2017 Solved Paper Electrical
L s Engineering:
J s JLs Set – 1 | xxix
∆1 = 1
Y13 = j20 + y13
 − R   −k1k2 
= j20 – j20 = 0 ∆ = 1 −  + 2 
 Ls   JLs 
Y31 = j20 + y13
R −k1k2
= j20 – j20 = 0 = 1+ +
Ls JLs 2
Y33 = –j39.9 – y33 – y31
JLs 2 + JRs + k1k2
= –j39.9 – j0.05 – (–j20) ∆=
JLs 2
= – j19.95 k1
∴ New Y bus matrix JLs 2
TF =
 − j19.95 j 20 0  JLs + JRs + k1k2
2

 j 20 
[Y ]3×3 =  j 20 − j 39.9 JLs 2
 0 j 20 − j19.95 k1
TF =
JLs 2 + JRs + k1k2
Hence, the correct option is (C).
Question Number: 32 Question Type: MCQ Hence, the correct option is (A).
In the system, whose signal flow graph is shown in the Question Number: 33 Question Type: NAT
figure, U1(s) and U2(s) are inputs. The transfer function
Y ( s) The circuit shown in the figure uses matched transistors
is with a thermal voltage VT = 25 mV. The base currents
U1 ( s )
of the transistors are negligible. The value of the resist-
−R ance R in kΩ that is required to provide 1 μA bias cur-
U2
rent for the differential amplifier block shown in _____.
1 −1 1 (Give the answer up to one decimal place).
U1 y
1/L 1/s k1 1/J 1/s
12 V

−k2
Differential
k1 Amplifier
(A) 2
JLs + JRs + k1k2
1 μA
k1 1 mA
(B) 2
JLs − JRs − k1k2
k1 − U 2 ( R + sL)
(C) 2
JLs + ( JR − U 2 L) s + k1k2 − U 2 R
R
k1 − U 2 ( sL − R)
(D) 2
JLs − ( JR + U 2 L) s − k1k2 + U 2 R
−12 V

Solution: 
Solution:
From mason’s gain formula, we get
Given data VT = 25 mV
TF =
∑P ∆ k k

∆ given I B1 = I B2 ≈ 0 A
1 1 1 1 k
P1 = ⋅ ⋅ k1 ⋅ ⋅ = 1 2 I C1 = 1 mA
L s J s JLs
∆1 = 1 I C2 = 1 mA ( Bias current)
 − R   −k1k2 
∆ = 1 −  + 2 
 Ls   JLs 
R −k1k2
= 1+ +
Ls JLs 2
M01_GATE_9789332576063_ME_SET1.indd 29 2 6/20/2017 8:07:07 AM
xxx | GATE 2017 Solved Paper Electrical Engineering: Set – 1

Applying the KVL for the given circuit, we get Solution: 


+12 V
−s + 1
G ( s) =
s +1
DA
IµA =IC2 For unit step input
IC =1 mA
1
1
+ + R( s) =
VBE1 VBE s
− −2 IE2
So output
R
1 ( − s + 1)
y( s) = R( s) ⋅ G ( s) = ⋅
s s +1
−12 V
 −1  −1  1 
−VBE1 + VBE2 + I E2 R = 0 y(t ) = L−1   + L  s( s + 1) 
 s +1  
I E 2 R = VBE1 − VBE2 (1)
t
∵ I B2 = 0 we get = −e − t + ∫ e − t dt
0

I E2 = I C2 = 1 µA (2) −t
= −e + ( −e − t ) + 1
substituting Eq. (2) in Eq. (1), we get y (t ) = 1 − 2 e − t
(1µ ) R = VBE1 − VBE2 (3)
t = 1.5 sec
 IC 
VBE1 = VT ln  1  (4) at y(1.5) = 1 − 2e −1.5
 Is 
= 0.5537
 IC 
VBE2 = VT ln  2  (5) or y(1.5) = 0.554
 Is 
Substituting Eqs. (4) and (5) in Eq. (3), we get Hence, the correct Answer is (0.554).

 IC   I C2  Question Number: 35 Question Type: NAT


(1µ ) R = VT ln  1  − VT ln   Two parallel connected, three-phase, 50 Hz, 11 kV, star-
 Is   Is 
connected synchronous machines A and B are operat-
 IC  ing as synchronous condensers. They together supply
VT ln  1 
 IC  50 MVAR to a 11 kV grid. Current supplied by both
R=  2
the machines are equal. Synchronous reactances of
(1µ )
machine A and machine B are 1Ω and 3Ω, respectively.
 1m  Assuming the magnetic circuit to be linear, the ratio of
25m ln  
=  1µ  excitation current of machine A to that of machine B
(1µ ) is _____. (Give the answer up to two decimal places).
Solution: 
R = 172.69 kΩ
Hence, the correct Answer is (172.69 kΩ). As the machines works at same current and same volt-
age, so they supply same reactive power.
Question Number: 34 Question Type: NAT
As machines are operating as synchronous condenser
−s + 1 so they will work as overexcited synchronous motor.
For a system having transfer function G ( s) = ,a
s +1 Total current,
unit step input is applied at time t = 0. The value of the
50 MVA
response of the system at t = 1.5 sec (rounded off to IT =
three decimal places) is _____ 3 × 11 KV

M01_GATE_9789332576063_ME_SET1.indd 30 6/20/2017 7:35:04 AM


GATE 2017 Solved Paper Electrical Engineering: Set – 1 | xxxi

= 2.624 kA The Fourier series representation of the output is given


IT as
I=
A I=
B (A) 4000 + 4000cos(2000πt) + 4000cos(4000πt)
2
(B) 2000 + 2000cos(2000πt) + 2000cos(4000πt)
2.624 kA (C) 4000cos(2000πt)
= = 1.312 kA
2 (D) A2000cos(2000πt)
Current taken by motor will be leading because the
motor is working as synchronous condenser, Solution: 
IA We know that function in time domain is

 k 
x (t ) = ∑ (−1) δ  t − 2000 
k =−∞
k

This function looks like f(t – T) delayed by time T.


 k 
Here, δ  t −  is compared with δ(t – kT)
 2000 
1
90˚ Where T =
2000
Vt
   The values of x(t) for k = 0,1,2, . . . . are
Also, E A = V T − jI a X s k = –1,–2, –3,....
11 kV
= − j (1.312∠90° kA )(1) x (t ) = δ (t ) for k = 0
3
 1 
11 x(t ) = ( −1)δ  t −  for k = 1
= + 1.312 × 1 = 7.662 kV  2000 
3
 11 kV  1 
Similarly EB = − j (1.312∠90° kA )(3) x(t ) = ( −1) −1 δ  t −  for k = −1
3  2000 
11
= + 1.312 × 3 = 10.286 kV Drawing the function x(t) for various values of k, we
3 get
I A EA 7.662
Hence, ∝ = = 0.74 δ (t)
I B E B 10.286
Hence, the correct answer is (0.74).
Question Number: 36 Question Type: MCQ t = −T t=T
Let the signal k = −1 k=1
+∞
 k 
x (t ) = ∑ (−1) δ  t − 2000 
k =−∞
k
k = −2 k=0 k=2
t = −2T T=0 t = 2T
be passed through an LTI system with frequency To
response H(ω), as given in the figure.
H (ω )

(
−δ t+ 1
2000 ) (
−δ t− 1
2000 )

ω
The figure shown above passess even half wave sym-
−5000π 5000π metry with time period

M01_GATE_9789332576063_ME_SET1.indd 31 6/20/2017 7:35:06 AM


xxxii | GATE 2017 Solved Paper Electrical Engineering: Set – 1

2 1 BD + BCD
(A) (B) BD + AB
T0 = 2T = =
2000 2000 (C) BD + ABC (D) BD + ABC
2π 2π
ω0 = = = 2000π Solution: 
T  1 
 1000  The Karnaugh map is given as
  C
CD
In the case of even half wave symmetry bn = 0 and con- AB
00 01 11 10
sists of only odd harmonics of an.
00 0 0 0 0
The frequency components are ω0, 3ω0…
i.e., 2000π, 6000π…… 01 1 0 0 1
and 2000π is the only frequency available in the above B
range or –5000 π to 5000 π 11 1 0 1 1
T0 / 2 A

∴ an =
T ∫
−T0 / 2
f (t ) cos nω0 t dt 10 0 0 0 0

T0 / 2 D
4
=
T0 ∫
0
f (t ) cos nω0 t dt
f = BD + ABC
T0 / 2
4 Hence, the correct option is (D).
a2 =
T0 ∫ δ (t ) cos
0
2ω0 (0)dt
Question Number: 38 Question Type: MCQ
T0 / 2
4 The transfer function of the system Y(s)/U(s) whose
=
T0 ∫ δ (t ) dt
0
state-space equations are given below is:
4  x1 (t )  1 2   x1 (t )  1 
= (1) = 4000  =   +   u (t )
T0  x2 (t )   2 0   x2 (t )   2 
∴ The output  x1 (t ) 
y(t ) = [1 0]  
y(t) = 4000 cos ω0 + 4000 cos (3 ω0 t)+....  x2 (t ) 
= 4000 cos2000πt + 4000 cos6000πt + .... ( s + 2) ( s − 2)
(A) 2 (B)
Hence, 4000 cos2000πt is in the range of –5000π to ( s + 2 s − 2) ( s + s − 4)
2

5000π.
( s − 4) ( s + 4)
Hence, the correct option is (C). (C) 2 (D)
( s + s − 4) ( s − s − 4)
2

Question Number: 37 Question Type: MCQ Solution: 


The output expression for the Karnaugh map shown in Transfer function TF = C(sI – A)–1B
the figure is
 s − 1 −2 
sI − A = 
CD  −2 s 
AB
00 01 11 10 sI − A = s( s − 1) − 4 = s 2 − s − 4
00 +2 
0 0 0 0 1 s
[ sI − A]−1 =  +2 s − 1
01
s −s−4 
2

1 0 0 1
1  s +2  1 
[ sI − A]−1 B =   
11 1 0 1 1 s − s − 4  2 s − 1  2 
2

1 s + 4
10 0 0 0 0 =  
s − s − 4  2s 
2

1 s + 4
C [ sI − A]−1 B = [1 0]  
s2 − s − 4  25 
s+4
T .F . = 2
s −s−4
M01_GATE_9789332576063_ME_SET1.indd 32 6/20/2017 7:35:10 AM
1 s +2 
[ sI − A]−1 =  +2 s − 1
s −s−4 
2

1  s +2  1 
[ sI − A]−1 B =   
s − s − 4  2 s − 1  2 
2

1 s + 4 GATE 2017 Solved Paper Electrical Engineering: Set – 1 | xxxiii


=  
s − s − 4  2s 
2

s + 4 In case of regular hexagon


1
C [ sI − A]−1 B = [1 0]   a =1
s −s−4
2
 25 
s+4
T .F . = 2 ρ
s −s−4
Hence, the correct option is (D). a =1 a =1

Question Number: 39 Question Type: NAT


The magnitude of magnetic flux density (b) in micro
Teslas (μT) , at the center of a loop of wire wound as a
regular hexagon of side length 1 m carrying a current
(I = 1 A) and placed in vacuum as shown in the figure
is _____. (Give the answer up to two decimal places).
a2
ρ = a2 − where a = 1
4
1 3
= 1− = m
4 2
Using formula in eq-(i) magnetic field intensity at cen-
tre due to one side of regular Hexagon
1
H′ = [cos 60° − cos 120°]αφ
 3
4π  
 2 
Solution:  = 0.091888 H/m
Magnetic field due to finite length of current carrying Magnetic field intensity due to all six sides of regular
conductor is given by hexagon will be
H = 6 × H′
I
=H (cos a 2 − cos a 1 )aˆf  (1) = 6 × 0.091888
4pr
= 0.551329 H/m
Magnetic flux density is given by relation
α2 B = μH
I
In vacuum
B = μ0H
ρ = 4π × 10−7 × 0.551329
= 6.9282 × 10–7 Tesla
or B = 0.6928 µT

α1
B = 0.69 µT upto two decimal places
Hence, the correct answer is (0.69).
Question Number: 40 Question Type: NAT
The figure shows an uncontrolled diode bride rectifier
α 2 = 60°
supplied from a 220 V, 50 Hz, 1-phase ac source. The
α1 = 120° load draws a constant current I0 = 14 A. The conduction

M01_GATE_9789332576063_ME_SET1.indd 33 6/20/2017 7:35:13 AM


xxxiv | GATE 2017 Solved Paper Electrical Engineering: Set – 1

angle of the diode D1 in degrees (rounded off to two 3Ω


decimal places) is _____
5Ω
-+
D1 D3
LS =10 mH
6V
R
220 V, IO =14 A 5V +
-
5Ω 2A
50 HZ

D1 D2
Solution:
Solution:  If VTh is the Thevenin’s voltage across “R”
For single phase controlled bridge rectifier effect of RTh is the Thevenin’s resistance across “R”
source inducfance will modify the average output volt- Maximum power Pmax across R will be
age as,
VTh2
V Pmax =
V0 = m [cos α + cos(α + µ )] 4 RTh
π
where μ is overlap angle Calculating Resistance Rth
But, for diode (uncontrolled) bridge, α = 0 By short circuiting all voltage sources and open circuit-
Vm ing all current sources, the circuit reduces to
So, V0 = [1 + cos µ ]  (1)
π
3Ω
Also 5Ω
2Vm 2ω Ls
V0 = − I 0 (2)
π π
R 5Ω 5Ω 5Ω R
In above expression Ls = source inductance
From Eq. (1) and Eq. (2).
2Vm 2ω Ls V
− I 0 = m [1 + cos µ] Hence, RTh = 5 Ω||5 Ω = 2.5 Ω
π π π
Substituting all the values in above equation Calculating resistance Vth
−3
2 × 220 × 2 4π × 50 × 10 × 10 × 14 Using superposition theorem

π π Taking 5 V source only
220 × 2 Circuit reduces to
= [1 + cos µ ]
π
5Ω
Solving for cos μ
cos μ = 0.7173
⇒ µ = 44.17°
+
+ V1 5Ω
Conduction angle for diode will be 180° + μ 5V

Hence, conduction angle γ = 180° + μ = 180° + 44.17°
γ = 224.17° upto two decimal places. −

Hence, the correct answer is (224.17°).


From the above circuit, we get
Question Number: 41 Question Type: NAT
In the circuit shown in the figure, the maximum power 5
V1= = 2.5
transferred to the resistor R is_____W 2

M01_GATE_9789332576063_ME_SET1.indd 34 6/20/2017 7:35:18 AM


GATE 2017 Solved Paper Electrical Engineering: Set – 1 | xxxv

Taking 6 V source only, circuit reduces to For maximum Torque φ = 90° between currents of
3Ω auxiliary winding and mains winding.

−6 V + 230
IM =
+ (12.50 + j15.75)
5Ω V2 5Ω

 15.75 
φM = − tan −1  
From the above circuit we get  12.50 
V2 = –3 V
Taking Xc as reactance of capacitor Cs
Taking 2 A current source only,
5Ω 230
IA =
( 24.50 + j12.75 − jX c )
+
V3 5 Ω 2A  12.75 − X C 
φM = − tan −1  
 24.50 
Form the above circuit, we get Taking φm + 90° = φ A
V3 = –5 V
 15.75   12.75 − X c 
VTh = V1 + V2 + V3 = 2.5 – 3 – 5 = – 5.5 V tan −1   + 90° = tan −1  
 12.50   24.50 
(5.5) 2
 Pmax =  15.75   12.75 − X c 
4 × 2.5 tan −1   − tan −1   = 90°
 12.5   24.50 
or Pmax = 3.025 W
Taking tan on both sides
Hence, the correct Answer is (3.025 W).
 15.75   12.75 − X c 
Question Number: 42 Question Type: NAT  12.5  −  24.50 
    = tan 90° = ∞
A 375 W, 230 V, 50 Hz, capacitor start single-phase
 15.75   12 . 75 − X c 
induction motor has the following constants for the 1+  − 
main and auxiliary windings (at starting):  12.5   24.50 
Zm = (12.50 + j15.75)Ω (main winding),  15.75   12.75 − X c 
∴1 +   =0
Za = (24.50 + j12.75)Ω (auxiliary winding).  12.5   24.50 
Neglecting the magnetizing branch, the value of the
capacitance (in μF) to be added in series with the aux- Solving for Xc
iliary winding to obtain maximum torque at starting is Xc = 32.194
_____
1
Solution:  Capacitor start single phase induction motor Also Xc =
ωC s
Switch

1 1
RM CS ⇒ Cs = =
V Rotor ω X c 2π × 100 × 32.194
XM

or Cs = 98.87 µF
RA XA
IA Hence, the correct Answer is (98.87mF ).
ϕ V
I Question Number: 43 Question Type: NAT
Consider a causal and stable LTI system with rational
IM transfer function H(z), whose corresponding impulse

M01_GATE_9789332576063_ME_SET1.indd 35 6/20/2017 7:35:21 AM


xxxvi | GATE 2017 Solved Paper Electrical Engineering: Set – 1

5 K ⋅Z4
response begins at n = 0. Furthermore, H(1) = . The H ( z) =
4 ( Z − P1 )( Z − P2 )( Z − P3 )( Z − P4 )
 ( 2k − 1)π 
1
poles of H(z) are pk = exp  j  for k = 1, 2,
2  4  K ⋅Z4
=
3, 4. The zeros of H(z) are all at z = 0. Let g[n] = jnh[n].   1 + j    −1 + j     −1 − j 
Z −  2   Z −  2   Z −  2 
The value of g[8] equals _____. (Give the answer up to         
three decimal places).
  1 − j 
Solution:   Z −  2 
  
Given that
KZ 4
1  ( 2k − 1)π  H ( z) =
Pk = exp  j 1
4 , Z4 +
2   4
k = 1, 2, 3, 4 5
H (1) =
1 jπ 4
P1 = e 4

2 K 5
=
1 4
1  π   π  1+
= cos   + j sin  4   4
2 4  
4 5
(1 + j ) K=
= 5 4
2
25
1 j 3π
K=
P2 = e 4 16
2
25 Z4
1   3π   3π   H ( z) =
= cos   + j sin  4   16 1
2  4    Z4 +
4
( −1 + j ) 25  1 4 1 −8 
= H ( z) = 1 − Z + Z + .....
2 16  4 16 
j 5π
1 25  1 1 
P3 = e 4
h[n] = δ ( n) − δ ( n − 4) + δ ( n − 8).....
2 16  4 16 
1   5π   5π   25  1 1 
= cos  4  + j sin  4   h[8] = δ (8) − δ ( 4) + δ (0).....
2     
16  4 16 
( −1 − j ) 25  1 1 
= = 0 − × 0 + × 1 + .....0 
2 16  4 16 
j 7π
1
P4 = e 4
25 1 25
2 h[8] = × = = 0.098
16 16 256
1   7π   7π  
= cos   + j sin  4   g[n] = j n h[n]
2  4   
(1 − j ) g[8] = j 8 h[8] = h[8] = 0.098
=
2
g [8] = 0.098
System is causal so order of numerator can not be
greater than order of denominator. Therefore, Hence, the correct answer is (0.098).

M01_GATE_9789332576063_ME_SET1.indd 36 6/20/2017 7:35:23 AM


GATE 2017 Solved Paper Electrical Engineering: Set – 1 | xxxvii

Question Number: 44 Question Type: MCQ Therefore


Let a causal LTI system be characterized by the follow- | 2 −1 |
< 0.001
ing differential equation, with initial rest condition 2n
d2 y dy dx(t ) ⇒ 2n > 1000
+ 7 + 10 y(t ) = 4 x(t ) + 5
dt 2 dt dt ⇒ n = 10
where, x(t) and y(t) are the input and output respec-
Hence, the correct answer is (10).
tively. The impulse response of the system is [u(t) is the
unit step function] Question Number: 46 Question Type: NAT
(A) 2e–2tu(t) – 7e–5tu(t) The equivalent resistance between the terminals A and
(B) –2e–2tu(t) + 7e–5tu(t) B is _______ Ω.
(C) 7e–2tu(t) – 2e–5tu(t) 1Ω 2Ω 1Ω
(D) –7e–2tu(t) + 2e–5tu(t) A
6Ω
Solution:
The give differential equation is
6Ω 1Ω
3Ω
d2 y dy dx(t )
+ 7 + 10 y(t ) = 4 x(t ) + 5
dt 2 dt dt 3Ω

Taking Laplace on both sides (initial rest condition) B


we get 0.8 Ω

s2Y(s) + 7sY(s) + 10Y(s) = 4X(s) + 5s X(s) Solution:


Y ( s) 5s + 4 Simplifying the given circuit, we get
H ( s) = = 1Ω 2Ω
X ( s) s 2 7 s + 10 A
Impulse response h(t) = L-1 H(s)
 5s + 4 
h(t ) = L−1  
 ( s + 2)( s + 5)  3Ω 6Ω 3Ω 6Ω 1Ω 2Ω

 −2 7 
= L−1  +
 s + 2 s + 5 
B
h(t ) = −2e −2t 4(t ) + 7e −5t 4(t ) 0.8 Ω

Hence, the correct option is (B). Combining parallel resistances 3 Ω, 6 Ω, and 2 Ω, we


Question Number: 45 Question Type: NAT get the equivalent as
Only one of the real roots of f (x) = x6 – x – 1 lies in 3 Ω || 6 Ω || 2 Ω = 1 Ω
the interval 1 ≤ x ≤ 2 and bisection method is used to Consider the circuit given below
find its value. For achieving an accuracy of 0.001, the 1Ω 1Ω
required minimum number of iterations is _____ A

Solution: 
6
In bisection method, the minimum number of iterations 3Ω 6Ω 3Ω Ω
5
|b−a|
is given by <ε
2n B
0.8 Ω 0.8 Ω
Where
6 15
a is the lower limit of interval Req = 1 + + 0.8 = Ω
5 5
b is the upper limit of interval
ε is the error in approximation or Req = 3 Ω
n is the number of iteration Hence, the correct answer is (3 Ω).

M01_GATE_9789332576063_ME_SET1.indd 37 6/20/2017 7:35:26 AM


xxxviii | GATE 2017 Solved Paper Electrical Engineering: Set – 1

Question Number: 47 Question Type: MCQ (C) 150 ns has to be inserted into both x and y
Consider an electron a neutron and a proton initially at channels
rest and placed along a straight line such that the neu- (D) 100 ns has to be inserted into both x and y
tron is exactly at the center of the line joining the elec- channels
tron and proton. At t = 0, the particles are released but Solution:
are constrained to move along the same straight line. In a CRO the beam moves left to right across the CRT
Which of these will collide first? during the sweep time, the beam quickly moves to the
(A) The particles will never collide left side of the CRT screen during the retrace time as
(B) All will collide together shown in figure given below.
(C) Proton and neutron
(D) Electron and neutron Volts
Solutions: ts = sweep time
e− n p tr = retrace time

d d

Mass of electron, me = 9.1094 × 10–31 Kg


Mass of proton, mp = 1.6726 × 10–27 Kg
Electrostatic force will exist between electron and pro- ts tr ts tr t
ton only. If the force is “F” then by relation
F = ma
Where m is mass of particle and a is acceleration. Slope and level detector has delay time (td) = 100 ns
We know that mass of electron is lesser than proton, so Response time (tre) = 50 ns
acceleration of electron will be more than proton. Total time taken for one sweep cycle of x-plate
Using second kinematics equation = (t re + t d + t s + t r )
= 150 ns + (t s + t r )
1
s = ut + at 2 In order to display correctly signal to y-channel has to
2
s = distance travelled be applied after a delay of 150 ns.
u = inital speed Hence, the correct option is (A).
Question Number: 49 Question Type: NAT
As u = 0 The following measurements are obtained on a single
1 2 phase load V = 220 V ± 1%. I = 50A ± 1% and W =
s= at 555 W ± 2%. If the power factor is calculated using
2
these measurements the worst case error in the calcu-
To travel distance “d” electron will take lesser time so lated power factor in percent is _____(Give answer up
electron will collide with neutron first. to one decimal place)
Hence, the correct option is (D). Solution: 
Question Number: 48 Question Type: MCQ Given data
The slope and level detector circuit in a CRO has a V = 220 V ± 1%
delay of 100 ns. The start-stop sweep generator has a I = 5 A ± 1%
response time of 50 ns. In order to display correctly a W = 555 W ± 2%
delay line of
(A) 150 ns has to be inserted into the y-channel Now we know that
(B) 150 ns has to be inserted into the x-channel W = VIcosΦ

M01_GATE_9789332576063_ME_SET1.indd 38 6/20/2017 7:35:27 AM


GATE 2017 Solved Paper Electrical Engineering: Set – 1 | xxxix

δW  δ V ∆I δ (cos Φ )  Question Number: 51 Question Type: MCQ


So, = ± + +
W  V I cos Φ  3
0
1
2 2
 δ ( p. f .)   
0.02 = ±  0.01 + 0.01 +  The matrix A =  0 −1 0  has three distinct eigen-
 p. f 
1 3
In worst case  0 
2 2 1 
δ p. f .  
= 0.02 + 0.01 + 0.01 values and one of its eigenvectors is 0  . Which one of
p. f . 1 
= 0.04 the following can be another eigenvector of A?
 = 4% or 4.0% 0  −1
  0
Hence, the correct answer is (4%).
(A)  0  (B)  
 −1  0 
Question Number: 50 Question Type: MCQ 1 1
A closed loop system has the characteristic equation    −1
 0  (D)
(C)  
given by s3 + Ks2 + (K + 2)s + 3 = 0. For this system to  −1  1 
be stable, which one of the following conditions should
be satisfied? Solution:
(A) 0 < K < 0.5 The given matrix is
(B) 0.5 < K < 1
(C) 0 < K < 1 3 1
2 0
(D) K>1 2
 
A = 0 −1 0 
Solution:  1 3
 0 
Using Routh Hurwitz Criteria 2 2

s3 1 k +2 We know that to calculate eigen values of A,


s2 k 3 det ( A − λ I 3 ) = 0
k ( k + 2) − 3
s1 3 1
k −λ 0
2 2
s0 3
0 −1 − λ 0 =0
We know that for stability 1 3
0 −λ
2 2
k >0  (i)
k ( k + 2) − 3 3  3 
(ii)  2 − λ  ( −1 − λ )  2 − λ  
k    
1 1 
( k − 1)( k + 3) + 0 − ( −1 − λ )  = 0
⇒ >0 2 2 
k
⇒ Either k > 1; k > −3  3  1
2

( −1 − λ )  − λ  −  = 0
or k < 1; k < −3  (iii)  2  4 

From equation (i) and (iii), we get k > 1. −(1 + λ )[λ 2 − 3λ + 2] = 0


Hence, the correct option is (D). or −(1 + λ )(λ − 1)(λ − 2) = 0

M01_GATE_9789332576063_ME_SET1.indd 39 6/20/2017 7:35:31 AM


xl | GATE 2017 Solved Paper Electrical Engineering: Set – 1

The eigen values of A are -1, 1 and 2. Total number of buses (N) = 10
If X be an eigen vector of A associated to λ, then Number of PV buses (x1) = 2 (i.e., G3 and G4)
AX = λ X Number of voltage controlled buses (x2) = 2
3 1 (i.e., L3 and L4)
2 0
2  1  1  The total number of equations to be solved
  
0 −1 0  0  = λ 0  = [2 N − 2 − ( x1 + x2 )]
So,  1 3  1  1  = [2(10) − 2 − ( 2 + 2)]
 0 
2 2 = 20 − 2 − 4
= 14
Simplifying, we get The size of the jacobian matrix
λ=2
= [2 N − 2 − ( x1 + x2 )] × [2 N − 2 − ( x1 + x2 )]
x = 14 × 14
Thus for λ = +1 by taking X =  y 
  Hence, the correct answer is (14).
 z 
Question Number: 53 Question Type: MCQ
[ A + 1I 3 ] X = 0 A 3-bus power system is shown in the figure below,
where the diagonal elements of Y bus matrix are: Y11
1 1 = –j12 pu, Y22 = –j15 pu and Y33 = –j7 pu
x+ z =0
2 2
Bus-1 Bus-2
−2 y = 0 jq

1 1
x+ z =0
2 2
jr jp
1 1
or x+ z =0
2 2
Bus-3
y=0
Hence option (c) satisfies.
Hence, the correct option is (C). The per unit values of the line reactances p, q and r
shown in the figure are
Question Number: 52 Question Type: NAT
(A) p = –0.2, q = –0.1, r = –0.5
A 10-bus power system consists of four generator buses (B) p = 0.2, q = 0.1, r = 0.5
indexed as G1, G2, G3, G4 and six load buses indexed as (C) p = –5, q = –10, r = –2
L1, L2, L3, L4, L5, L6. The generator-bus G1 is considered (D) p = 5, q = 10, r = 2
as slack bus and the load buses L3 and L4 are voltage
controlled buses. The generator at bus G2 cannot sup- Solution: 
ply the required reactive power demand and hence it From the bus diagram given in problem, we get
is operating at its maximum reactive power limit. The
number of non-linear equations required for solving 1
y=
12 y=
21
the load flow problem using Newton-Raphson method jq
in polar form is _____. 1
y= y=
Solution:  13 31
jr
Number of slack buses = 1 (i.e., G1)
1
Number of load buses = 5 y=
23 y=
32
jp

M01_GATE_9789332576063_ME_SET1.indd 40 6/20/2017 7:35:35 AM


GATE 2017 Solved Paper Electrical Engineering: Set – 1 | xli

Since diagonal elements Question Number: 55 Question Type: NAT


Y11 = y11 + y12 + y13 Consider the unity feedback control system shown. The
value of K that results in a phase margin of the system
1 1
So, + = − j12 to be 30° is _____. (Give the answer up to two decimal
jq jr places.)
1 1
+ = 12  (i)
q r U(s) + Ke−s Y(s)
s
Similary for Y22 = –j15 −

1 1
+ = − j15
jq jp
1 1
or + = 15  (ii) Solution: 
q p
For unity feedback system with
For Y33 = –j7
1 1 Ke − s
+ = − j7 G ( jω ) H ( jω ) =
jr jp s

1 1 Phase margin is given by


or + =7  (iii)
r p P . M . = 180° + φ
Simplifying equations (i), (ii) and (iii) Where,
1 1 1 φ =| G ( jω ).H ( jω ) |ω =ωgc
= 5=
; 10; =2
p q r
and
Hence,
| G ( jω ).H ( jω ) |ω =ωgc = 1
p = 0.2, q = 0.1, r = 0.5
Hence, the correct option is (B). Since,
Question Number: 54 Question Type: MCQ | e − jω |= 1
For the power semiconductor devices IGBT, MOSFET, So,
Diode and Thyristor, which one of the following state-
k
ment is TRUE? = 1 at ω = ω gc
(A) All the four are majority carrier devices ω
(B) All the four are minority carrier devices So,
(C) IGBT and MOSFET are majority carrier de- ω gc = k
vices, whereas Diode and Thyristor are mi-
nority carrier devices Now,
(D) MOSFET is majority carrier device, whereas φ = G ( jω ) H ( jω ) = −90° − 57.3ω
IGBT, Diode, Thyristor are minority carrier
devices at
Solution:  ω gc = k
MOSFET is the only majority carrier device among φ = −90° − 57.3ω
MOSFET, DIODE, Thyristor and IGBT. In majority PM = 180° + φ = 30°
carrier devices conduction is only because of majority
carriers whereas in minority carrier devices conduction ⇒ φ = −150°
is due to both majority and minority carriers. ⇒ −90° − 57.3k = −150°
Hence, the correct option is (D). ⇒ k = 1.047

M01_GATE_9789332576063_ME_SET1.indd 41 6/20/2017 7:35:40 AM


xlii | GATE 2017 Solved Paper Electrical Engineering: Set – 1

Upto two decimal places The DC components of voltages v1 and v2, respectively
k = 1.05 are
Hence, the correct answer is (1.05). (A) 0 V and 1 V (B) –0.5 V and 0.5 V
(C) 1 V and 0.5 V (D) 1 V and 1V
Question Number: 56 Question Type: MCQ Solution: 
The transfer function of a system is given by During positive half cycle
V0 ( s) 1 − s Diode D1 is ON but diodes D2 and D3 will be OFF
=
Vi ( s) 1 + s
During negative half cycle
Let the output of the system be v0 (t ) = Vm sin(ωt + φ ) Diodes D2 and D3 are ON but diode D1 is OFF
for the input, vi (t ) = Vm sin(ωt ) . Then the minimum
and maximum values of f (in radians) are respectively π sin (100πt)
V(t)
π π π
(A) − and ⋅ (B) − and 0 ⋅
2 2 2
π
(C) 0 and ⋅ (D) −π and 0 ⋅
2
Solution:  π sin (100πt)
V1 2

We know that for transfer function


V0 ( s) 1 − s
=
Vi ( s) 1 + s
π sin (100πt)
V0 ( jω )
=1
Vi ( jω ) π sin (100πt)
V2
V0 ( jω )
2

= −2 tan −1 ω
Vi ( jω )
Here,
vi (t ) = Vm sin(ωt )
v0 (t ) = Vm sin(ωt + φ )
So, for ω = 0 to ω = ∞
1  ππ
2π  ∫0 2
V1( avg) = sin(100π t )d (ωt ) +
−2 tan −1 ω varies from –180° to 0°
Hence, the correct option is (D).
∫π π sin(100π t )d (ωt ) 

Question Number: 57 Question Type: MCQ


1 1
For the circuit shown in the figure below, assume that = [π − 2π ] = − = −0.5 V
diodes D1, D2 and D3 are ideal. 2π 2
D1
1  ππ 
2π  ∫0 2
V2( avg ) = sin(100π t )d (ωt ) 
R 
+ V1 − 1 π 
= ( − cos π + cos 0) 
D2 +
2π  2 
v(t) = π sin (100πt) V D3 R V2 1
− = = 0.5
2

Hence, the correct option is (B).

M01_GATE_9789332576063_ME_SET1.indd 42 6/20/2017 7:35:44 AM


GATE 2017 Solved Paper Electrical Engineering: Set – 1 | xliii

Question Number: 58 Question Type: NAT For unity P.f. for synchronous motor.
The power supplied by the 25 V source in the figure I V
shown below is _____W δ =30°
R1
I

+ 17 V − + jIXS
+ R2 0.4I
25 V − 14 A

Ef
Solution:
R1 From above phasor diagram,
I a
E f cos δ = V
+ 17 V − +
+
or,
14 A R2 0.4I
25 V −
V
Ef =
cos δ
6.6 kV
Using Kirchoffs current law at node “a” we get =
cos 30°
I + 0.4 I = 14 = 7.62 kV
⇒ I = 10 A
Hence,
Power supplied by 25 V source will be
7.62 × 6.6
P = 25 V × 10 A P= sin 30° MW
30
P = 250 watt = 0.8383 MW
Hence, the correct answer is (250). or,
P = 838.3 kW
Question Number: 59 Question Type: NAT
A three-phase, 50 Hz, star-connected cylindrical- Hence, the correct answer is (838.3 kW).
rotor synchronous machine is running as a motor. The Question Number: 60 Question Type: MCQ
machine is operated from a 6.6 kV grid and draws
A source is supplying a load through a 2-phase, 3-wire
current at unity power factor (UPF). The synchronous
transmission system as shown in figure below. The
reactance of the motor is 30 W per phase. The load
instantaneous voltage and current in phase-a are van =
angle is 30°. The power deliver to the motor in kW is
220sin(100πt)V and ia = 10sin(100πt)A , respectively.
_____. (Give the answer up to one decimal place).
Similarly for phase-b, the instantaneous voltage and cur-
rent are Vbn = 220cos(100πt)V and ib = 10cos(100πt)A
Solution:
Given a ia a′
V = 6.6 kV
+
δ = 30° Van

P. f . = 1(UPF ) b ib b′
Source Load
+
Synchronous reactance is ( X s ) = 30 Ω
Vbn
n n′
VE f
P= sin δ
Xs

M01_GATE_9789332576063_ME_SET1.indd 43 6/20/2017 7:35:47 AM


xliv | GATE 2017 Solved Paper Electrical Engineering: Set – 1

The total instantaneous power flowing from the source Y


to the load is 10
(A) 2200 W
(B) 2200 sin2 (100πt)W
(C) 4400 W
(D) 2200 sin(100πt) cos(100πt)W
2
Solution: R
We know that Instantaneous power is given by relation X
1 5
P = v ⋅i
  Solution: 
P = van ⋅ ia + vbn ⋅ ib
I = c ∫ ∫ xy 2 dx dy
= 220 sin(100π t ) ⋅10 sin(100π t ) R

+ 220 cos(100π t ) ⋅10 cos(100π t ) Y


10
= 2200 sin (100π t ) + 2200 cos (100π t )
2 2

P = 2200 W
Hence, the correct option is (A).
Question Number: 61 Question Type: MCQ
For a complex number z, 2
R
z2 +1
lim 3
z → i z + 2 z − i ( z 2 + 2)
1 5 X
(A) –2i (B) –i
Region R is bounded by y = 0 and y = 2x
(C) i (D) 2i
I = c  ∫ ∫ xy 2 dy dx 
5 2x
Solution: 
 1 0 
We know that
 5  xy 3  2x

z2 +1 = c ∫   dx 
lim  1  3 0 
z →i z 3 + 2 z − i( z 2 + 2)  
 58 
0 = c  ∫ x 4 dx 
1 3
The above limit is form, so on differentiating both  
0
8 5
 8 5 
numerator and denominator, we get = c  x5  = c  5 −1  ( )
15 1
 15 
z +1
2
lim
z →i 3 z 2 + 2 − 2 zi 8 
= 6 × 10 −4 ×  (55 − 1) 
 15 
2i 2i
= = = 2i
3(i ) + 2 − 2(i )i −3 + 2 + 2
2
I = 0.99 (upto two decimal places)
Hence, the correct option is (D). Hence, the correct answer is (0.99).

Question Number: 62 Question Type: NAT Question Number: 63 Question Type: MCQ
Let I = c ∫ ∫ xy dxdy , where R is the region shown in
2
A 4 pole induction machine is working as an induction
R generator. The generator supply frequency is 60 Hz.
the figure and c = 6 × 10 −4. The value of I equals _____. The rotor current frequency is 5 Hz. The mechanical
(Give the answer up to two decimal places). speed of the rotor in RPM is

M01_GATE_9789332576063_ME_SET1.indd 44 6/20/2017 7:35:50 AM


GATE 2017 Solved Paper Electrical Engineering: Set – 1 | xlv

(A) 1350 (B) 1650 Solution:


(C) 1950 (D) 2250 In 120° device canduction mode and star connected
Solution:  load:
For 4 pole, 60 Hz induction machine synchronous At any instant only 2 IGBTs will conduct so, when
speed; IGBT 1 and 6 are conducting in 0–60° cycle, equiva-
120 × f lent ckt can be given as
Ns =
P
120 × 60
=
4 1 3 5

= 1800 r.p.m. +

fr
s= Vs = 600 V
fs
4 6 2
5
=
60
For induction generator slip is negative,
So, b
a
Ns − Nr 5
=− R
Ns 60 R

0
1800 − N r −5
⇒ =
1800 60 R
−5
⇒ 1800 − N r = × 1800
60 C
a
5
⇒ N r = 1800 + × 1800 VS
600 R
2
+
N r = 1950 r.p.m. Vs = 600 V − 0
VS
Hence, the correct option is (C). R
2
b
Question Number: 64 Question Type: NAT
A 3-phase voltage source inverter is supplied from a So power,
600 V DC source as shown in the figure below. For a 2
 Vs 
star connected resistive load of 20 Ω per phase, the load 2
power for 120° device conduction, in kW, is _____ P =   ×2
R
2
V
= s
2R
20 Ω (600) 2
600 V P=
+ 2 × 20
Ω

− = 9000 watt
20

20
Ω

or, = 9 kW
Hence, the correct answer is (9 wK).

M01_GATE_9789332576063_ME_SET1.indd 45 6/20/2017 7:35:52 AM


xlvi | GATE 2017 Solved Paper Electrical Engineering: Set – 1

Question Number: 65 Question Type: MCQ (C) Remain uniform as before


A solid iron cylinder is placed in a region containing (D) Cease to exist inside the cycliner
a uniform magnetic field such that the cylinder axis is Solution:
parallel to the magnetic field direction. The magnetic
field lines inside the cylinder will The magnetic field lines will bend closer to the cylinder
(A) Bend closer to the cylinder axis axis to find a minimum reluctance path.
(B) Bend farther away from the axis Hence, the correct option is (A).

M01_GATE_9789332576063_ME_SET1.indd 46 6/20/2017 7:35:52 AM


GATE 2017 Solved Paper
Electrical Engineering
Set - 2
Number of Questions: 65 Total Marks: 100.0

Wrong answer for MCQ will result in negative marks, (-1/3) for 1 Mark Questions and (-2/3) for 2 Marks
Questions.

General Aptitude
Number of Questions: 10 Section Marks: 15.0

Q. 1 to Q. 5 carry 1 mark each and Q. 6 to Q. 10 carry Solution: 


2 marks each
has has has
Question Number: 1 Question Type: MCQ Apples Oranges Both

“We lived in a culture that denied any merit to liter- Oranges Apples Apples
ary works, considering them important only when or or or
Label:
they were handmaidens to something seemingly more Both Both Oranges
urgent—namely ideology. This was a country where
all gestures, even the most private, were interpreted in Let us choose a box labelled “apples”. If an orange
political terms. comes out, it can be either be having oranges or both.
Hence, can’t say.
The author’s belief that ideology is not as important as
literature is revealed by the word: Similarly, if we choose box labelled “oranges”, the
(A) “culture” same scenario will occur.
(B) “seemingly” Now, if we choose a box labelled “Both”, then
(C) “urgent” (i)  If an orange comes out, then the box labelled
(D) “political” “Apples” has both and box labelled “oranges” has
Solution:  Apple.
As literature is revealed by the word “seemingly” the (ii) If an apple comes out, then box labelled “apples”
author’s belief that ideology is not as important. has oranges and box labelled “oranges” has both.
Hence, option (B).
Hence, the correct option is (C).
Hence, the correct option is (B).
Question Number: 2 Question Type: MCQ
Question Number: 3 Question Type: MCQ
There are three boxes shown in the figure. One con-
X is a 30 digit number starting with the digit 4 followed
tains apples, another contains oranges, and the last one
by the digit 7. Then the number X3 will have:
contains both apples and oranges. All three are known
(A) 90 digits (B) 91 digits
to be incorrectly labelled. If you are permitted to open
(C) 92 digits (D) 93 digits
just one box and then pull out and inspect only one
fruit, which box would you open to determine the con- Solution: 
tents of all three boxes? Given X is a 30 digit
(A) The box labelled “Apples”
Starts with 4 followed by 7
(B) The box labelled “Apples and Oranges”
(C) The box labelled “Oranges” Let the number be X = (47 X28 X27 X26 X25... X0)
(D) Cannot be determined ∴ The number lies between,

M01_GATE_9789332576063_ME_SET1.indd 47 6/20/2017 7:44:34 AM


xlviii | GATE 2017 Solved Paper Electrical Engineering: Set – 2

47 × 1028 < X < 48 × 1028 0.65 R


Now, X3 will be in between, 0.1 0.9
(47 × 1028)3 < X3 < (48 × 1028)3 S

0.95
0.
(47)3 × 1084 < X3 < (48)3 × 1084

95
P
103823 × 1084 < X3 < 110592 × 1084 0.9
0.8
∴ A total of (6 + 84) = 90 digits Q
∴ The number X3 will have 90 digits. 0.8 0.75
Hence, the correct option is (A).
B 1 2 km
Question Number: 4 Question Type: MCQ
If the possibility of a thunderstorm is given by how fast
The number of roots of ex + 0.5x2 – 2 = 0 in the range air pressure rises or drops over a region, which of the
[–5, 5] is: following regions is most likely to have a thunderstorm?
(A) 0 (B) 1 (A) P (B) Q
(C) 2 (D) 3 (C) R (D) S
Solution: Solution: 
The given equation is We know that there should be pressure difference
ex + 0.5x2 – 2 = 0 between the land and sea in order to have the rain. The
1 2 region which is having low pressure is most likely to
ex + x −2 = 0 have heavy rainfall/thunderstorm. Region R has lowest
2
pressure hence most likely to have thunderstorm in this
1
ex = 2 − x2 region.
2 Hence, the correct option is (C).
Question Number: 6 Question Type: MCQ
There are five buildings called V, W, X, Y and Z in a row
(not necessarily in that order). V is to the West of W. Z
is to the East of X and the West of V. W is to the West of
2
Y. Which is the building in the middle?
1 (A) V (B) W
(C) X (D) Y
Solution: 
−2 2
As given, V is to west of W
\ V ← W (1)
Z is east of X and west of Y
X → Z ← V (2)
No. of solutions = 2 W is to the west of  Y
W ← Y (3)
Hence, no. of roots = 2.
From Eqs. (1), (2), and (3), we have the order of direc-
Hence, the correct option is (C). tion as
Question Number: 5 Question Type: MCQ West East
An air-pressure contour line joins locations in a region X Z V W Y
having the same atmospheric pressure. The following is
Hence, the middle building is V.
an air-pressure contour plot of a geographical region.
Contour lines are shown at 0.05 bar intervals in this plot. Hence, the correct option is (A).

M01_GATE_9789332576063_ME_SET1.indd 48 6/20/2017 7:44:36 AM


GATE 2017 Solved Paper Electrical Engineering: Set – 2 | xlix

Question Number: 7 Question Type: MCQ Question Number: 9 Question Type: MCQ
A test has twenty questions worth 100 marks in total. Saturn is _________ to be seen on a clear night with
There are two types of questions. Multiple choice ques- the naked eye.
tions are worth 3 marks each and essay questions are (A) Enough bright (B) Bright enough
worth 11 marks each. How many multiple choice ques- (C) As enough bright (D) Bright as enough
tions does the exam have? Solution: 
(A) 12 (B) 15
The correct sentence is
(C) 18 (D) 19
“Saturn is Bright enough to be seen on a clear night
Solution:  with the naked eye.”
If “e” is the number of essay questions and “m” is the Hence, the correct option is (B).
number of multiple choice questions
Test has 20 questions for 100 marks Question Number: 10 Question Type: MCQ
Each essay (e) = 11 marks There are 3 red socks, 4 green socks and 3 blue socks.
Each multiple choice (m) = 3 marks You choose 2 socks. The probability that they are of the
Therefore, same colour is:
e + m = 20 (1) (A) 1/5 (B) 7/30
and (C) 1/4 (D) 4/15
3m + 11e = 100 (2) Solution:
Solving Eqs. (1) and (2), we get We are given that we have 3 red socks, 4 green socks,
m = 15. 3 blue socks When 2 out of all picked, probability that
Hence, no. of multiple choice questions = 15. they are of same colour
Hence, the correct option is (B). No. of ways in which
Question Number: 8 Question Type: MCQ 2 same colour socks
Choose the option with words that are not synonyms. cann be selected
P (same colour) =
(A) Aversion, dislike Total no. of ways in
(B) Luminous, radiant which 2 out of total
(C) Plunder, loot
(D) Yielding, resistant can be selected
3
C2 + 4 C2 + 3 C2
Solution:  = 10
C2
We know that “yielding” means giving up and “resist-
ant” means the one who resists. Thus both of these 3+6 +3 4
= =
words are not synonyms . 45 45
Hence, the correct option is (D). Hence, the correct option is (D).

Electrical Engineering
Number of Questions: 55 Section Marks: 85.0
Q. 11 to Q. 35 carry 1 mark each and Q. 36 to Q. 65 a
carry 2 marks each (A) 0 (B)
b
Question Number: 11 Question Type: MCQ
a+K a−K
When a unit ramp input is applied to the unity feedback (C) (D)
b b
system having closed loop transfer function
C ( s) K
= 2 s + b ,( a > 0, b > 0, K > 0) , the steady
R( s) s + as + b
state error will be

M01_GATE_9789332576063_ME_SET1.indd 49 6/20/2017 7:44:37 AM


l | GATE 2017 Solved Paper Electrical Engineering: Set – 2

Solution:  Mean square value of the given periodic function is


Closed loop transfer function is
0.7 2.7
C ( s) K 0+ ∫ ( 4) 2 dt + ∫ (−2) dt + 0
2
= 2 s+b
R( s) s + as + b = −0.3 0.7

Time period
And is closed loop unity feedback system.
0.7 2.7
Error, cal be calculated as
∫ 16 dt + ∫ 4dt
E ( s) = R( s) − C ( s) = −0.3 0.7

3.7 − ( −0.3)
 K 
= R( s) 1 − 2 s + b 
 s + as + b  16(0.7 + 0.3) + 4( 2.7 − 0.7)
=
4
Steady state error, can be calculated as
16 + 8
= =6
 s 2 + as + b − ks − b  4
ess = lim  
 s + as + b 
x →∞ 3 2
Hence, the correct answer is (6).
 s 2 + as − ks  0
= lim  3  … Question Number: 13 Question Type: MCQ
x →∞ s + as 2 + bs 0
 
In the circuit shown in the figure, the diodes are ideal,
the inductance is small and I ≠ 0. Which one of the
Using L-hospital method, we get
following statements is TRUE?
2s + a − k a−k
lim =
x →∞ 3s 2 + 2 sa + b b D1

Hence, the correct option is (D).

Question Number: 12 Question Type: NAT


D2 Io
The mean square value of the given periodic waveform
f(t) is _____

f(t) (A) 
D1 conducts for greater than 180° and D2 con-
4 ducts for greater than 180°.
(B) 
D2 conducts for more than 180° and D1 con-
t ducts for 180°.
−3.3 −1.3 −0.3 0.7 2.7 3.7 4.7 (C) 
D1 conducts for 180° and D2 conducts for
180°.
2
(D) 
D1 conducts for more than 180° and D2 con-
ducts for 180°.
Solution:
Solution: 
4 Diode D1 is forward-biased during the positive half
cycle 0° to 180° and D2 is reverse-biased during this
−1.3 0.7 2.7 4.7 period. After this period current through D1 starts
−3.3 −0.3 −3.7 decaying and current through D2 starts rising in order
to maintain load current I0 constant and waveforms of
the same are shown in the figure.

M01_GATE_9789332576063_ME_SET1.indd 50 6/20/2017 7:44:38 AM


GATE 2017 Solved Paper Electrical Engineering: Set – 2 | li

Vs(Supply voltage) From the phasor diagram shown in the figure, it is


clearly obvious that Ef is greater them Vt in magnitude.
Hence, the correct option is (B).

Question Number: 15 Question Type: MCQ


A 3-phase, 4-pole, 400 V, 50 Hz squirrel-cadge induc-
2π ωt
π 3π tion motor is operating at a slip of 0.02. The speed of
Vo(Load voltage)

the rotor flux in mechanical rad/sec, sensed by a sta-


tionary observer is closest to
(A) 1500 (B) 1470
(C) 157 (D) 154
Solution: 
ωt
Given a 3-phase, 4P, 50 Hz, 400 V Squirrel-cage induc-
tion motor.
current)
Io(Load

Slip = 0.02
ID ID Speed of rotor flux with respect to stationary observer
1 2

in mech radians per second will be


ωt
D1 2
D1 D2 D1 ωsm = × 2π f
D2 D2 P
2
Thus, both the diodes conduct for more than 180°. = × 2π × 50
4
Hence, the correct option is (A). = 50π
= 157 mech rad/ sec
Question Number: 14 Question Type: MCQ
If a synchronous motor is running at a leading power Hence, the correct option is (C).
factor, its excitation induced voltage (Ef ) is Question Number: 16 Question Type: MCQ
(A) Equal to terminal voltage Vt
(B) Higher than the terminal voltage Vt The output y(t) of the following system is to be sam-
(C) Less than terminal voltage Vt pled, so as to reconstruct it from its samples uniquely.
(D) Dependent upon supply voltage Vt The required minimum sampling rate is

Solution:  X(ω )
ω
Synchronous motor running at leading pf, i.e., Ia lead- −1000π 1000π y(t)
ing Vt. h(t) −
sin(1500xt)
x(t) X(ω ) xt

P = VIacosϕ = const cos(1000π t)

Ia (A) 1000 samples/s (B) 1500 samples/s


(C) 2000 samples/s (D) 3000 samples/s
ϕ
Vt
δ
Solution: 
−jIaXs
z(t) sin(1500xt)
x(t) h(t) − y(t)
Ef xt
1 x(ω )
Efsinδ = const cos(1000π t)
−1000π 1000π

M01_GATE_9789332576063_ME_SET1.indd 51 6/20/2017 7:44:39 AM


lii | GATE 2017 Solved Paper Electrical Engineering: Set – 2

From the block diagram, shown in the figure, we get Question Number: 17 Question Type: NAT
z (t ) = x(t ).cos 1000π t For the synchronous sequential circuit shown in the fig-
Using modulation property of Fourier Transform, ure, the output Z is 0 for the initial conditions
we get QA QB QC = QA′ QB′ QC′ = 100

1
Z (ω ) = [ X (ω + 1000π ) + X (ω − 1000π )]
2 D Q D Q D Q

X(ω +1000π ) Q Q Q
clock QA QB QC
[MSB]
Z
1/2
[MSB] QA QB QC
D Q D Q D Q
ω Q Q Q
−2000 π

X(ω -1000π )
The minimum number of clock cycles after which the
output Z would again become 0 is _____
1/2 Solution:
Given:
ω QA QB QC = 100
2000π
QA′ QB′ QC′ = 100
Now,
The output Z for the given circuit is given by
Z(ω )
Z = (QA ⊕ QA′ ) + (QB ⊕ QB′ ) + (QC ⊕ QC′ ) (i)
Now, tabulating the values of outputs of flip-flops and
Z as shown in the table.
1/2
Clock Q A QB Q C QA QB QC Z

0 1 0 0 1 0 0 0 initial
ω
−2000π 2000π
1 0 1 0 1 1 0 1
Now,
sin1500π 2 0 0 1 1 1 1 1
h(t) =
πt
3 1 0 0 0 1 1 1

H(ω ) 4 0 1 0 0 0 1 1

1 0 0 0 1
5 0 0 1

6 1 0 0 1 0 0 0 after
6 clock
ω pulse
−1500π 1500π
Hence, the output Z is “0” after 6 clock pulses.
Therefore, H(ω) is a low pass filter and it will pass fre-
Hence, the correct answer is (6).
quency component of Z(ω) upto 1500 π rad/sec.
Hence, required minimum sample rate is 1500 samples/ Question Number: 18 Question Type: MCQ
sec. For the circuit shown in the figure, assume that the
Hence, the correct option is (B). OPAMP is ideal.

M01_GATE_9789332576063_ME_SET1.indd 52 6/20/2017 7:44:42 AM


GATE 2017 Solved Paper Electrical Engineering: Set – 2 | liii

R V3
I3 =
(5)
R
R R From Eqs. (3) and (5), we get
R V2
− I2 =
,
R
+ V0 V
I3 = 3
2R R
V3 Substituting in Eq. (4), we get
2R
V 
V3 = V2 +  2 R
Which one of the following is TRUE? R 
(A) v0 = vs (B) v0 = 1.5 vs = 2V2
(C) v0 = 2.5 vs (D) v0 = 5 vs
= 2V1 (∵V1 = V2 )
Solution: 
2V1 = 2 × Vs / 2 …(from eq.1)
Redrawing the given circuit as shown in the figure,
V3 = Vs (6 )
R V3 R I0 I0 = I3 + I 2
I2 I3 R V3 V2
R I2 = +
R R

V2 V V
V1
+ I0 = s + 2 (from eq. (7))
Vs V0 R R
2R 2R
Vs Vs
= + (∵V1 = V2 )
R 2R
 1
= Vs 1 + 
 2
We know that for an ideal op-amp V 
= 1.5  s  (8)
v1 = v2 R
Also, Applying KVL at output, we get
 2R  −V0 + I 0 R + V3 = 0
v1 = V  
 2R + 2R  ⇒ V3 = V3 + I 0 R
 2R  ⇒ V0 = V3 + I 0 R
= Vs   (1)
 4R  Vs
V V0 = Vs + (1.5) R
= s R
2 [from eq. (7)]
Applying KCL at node v3, we get V0 = Vs + Vs (1.5)
I 0 = I 3 + I 2 (2) V0 = Vs (1 + 1.5)
V2 = I 2 R (3) V0 = 2.5Vs
Applying KVL at the input terminals of op-amp and Hence, the correct option is (C).
node V3, we get
Question Number: 19 Question Type: MCQ
−V3 + I 2 R + V2 = 0
The figure shows a half-bridge voltage source inverter
I 2 R = V3 − V2 (4)  0.3 
supplying an RL-load with R = 40 W and L =   H.
V3 = V2 + I 2 R  π 

M01_GATE_9789332576063_ME_SET1.indd 53 6/20/2017 7:44:44 AM


2
 0.3 
= ( 40) 2 +  2π × 50 ×
 π 
= ( 40) 2 + (30) 2
= 50Ω
liv | GATE 2017 Solved Paper Electrical Engineering: Set – 2 40
cos φ =
50
The desired fundamental frequency of the load voltage = 0.8
is 50 Hz. The switch control signals of the converter are
φ = cos −1 (0.8)
generated using sinusoidal pulse width modulation
with modulation index M = 0.6. At 50 Hz, the RL-load = 36.86
draws an active power of 1.44 kW. The value of DC PL = V01 I 01 cos φ
source voltage VDC in volts is (V01 )
1.44 K = V01 cos φ
Z1
S1 2
VDC +  0.3VDC  1
− 1440 =   × 0.8
R L  2  50
1440 × 2 × 50
= VDC
2

VDC + (0.3) 2 × 0.8


− S2
VDC = 1414.21 V
= 1000 2 V
(A) 300 2 (B) 500 Hence, the correct option is (D).
(C) 500 (D) 1000 2
Question Number: 20 Question Type: NAT
Solution: 
Consider the system described by the following state
Given resistance and inductance is space representation
 0.3 
R = 40 Ω, L =  H
 π   x1 (t )   0 1   x1 (t )   0 
 x (t )  =    +   u (t )
Modulation index (M)  2   0 −2   x2 (t )   1 
= 0.6  x (t ) 
PL = 1.44 kW y(t ) = [1 0]  1 
 x2 (t ) 
V01( Peak )
M=
VDC  x (t )   1 
If u(t) is a unit step input and  1  =   , the value
2  x2 (t )   0 
VDC of output y(t) at t = 1 sec (rounded off to three decimal
V01( Peak ) = ×M
2 places) is______
V Solution:
V01( rms ) = DC × M
2 2 0 1 
V A= 
= DC × 0.6 0 −2 
2 2
0.3VDC 0 
= B= 
2 1 

Z1 = R 2 + (ω L) 2 C = [1 0]
2
 0.3 
= ( 40) 2 +  2π × 50 ×  x ( 0)   1 
 π  Initial values  1  =  
 x2 ( 0 )   0 
= ( 40) 2 + (30) 2
= 50Ω 1
u (t ) = 1 ⇒ u ( s ) =
40 s
cos φ = State equation x (t ) = A X (t ) + Bu ( t )
50
= 0.8
φ = cos −1 (0.8)
= 36.86
PL = V01 I 01 cos φ
M01_GATE_9789332576063_ME_SET1.indd 54 6/20/2017 7:44:46 AM
Y ( s) = C X ( s) + Du ( s )
 1 1 
 + 2 
s s ( s + 2)  
= [10]  
 
Engineering:
1
GATE 2017 Solved Paper Electrical
 s  Set – 2 | lv
   
Converting into Laplace domain [∵ D = [0]]
s X ( s) − X (0) = A X ( s) + Bu ( s ) 1 1
= + 2
[ sI − A] X ( s ) = X ( 0 ) + Bu ( s ) s s ( s + 2)
X ( s) = [ sI − A]−1{ X (0) + Bu ( s ) } (1) 1 1 1 1
= + 2− +
−1 s 2s 4 s 4( s + 2)
  s 0  0 1  
[ sI − A]−1 =   −  3 1 1 
 0 s  0 −2   y(t ) = L−1  + 2 + 
 4 s 2s 4( s + 2) 
−1
 s −1  3 t 1
=  y(t ) = + + e −2t, at t = 1
0 s + 2  4 2 4
1  s + 2 1 We get
=
s( s + 2)  0 s 
3 1 1 −2
1 1  y (t ) = + + e = 1.2838 ∼ 1.284
 s s( s + 2)  4 2 4
=  Hence, the correct answer is (1.284).
 1 
 0 ( s + 2)  Question Number: 21 Question Type: MCQ
 
0  1 The range of K for which all the roots of the equation
Bu ( s ) =   s3 + 3s2 + 2s + K = 0 are in the left half of the complex
1  s s-plane is
0  (A) 0 < K < 6
= 1  (B) 0 < K < 16
 
 s  (C) 6 < K < 36
(D) 6 < K < 16
Substituting the values of [sI – A]–1, Bu(s), X(0) in
Eq. (1), we get Solution: 
The given equation is
1 1 
s  0 
s + 2   1     f ( x ) = s3 + 2 s 2 + 2s + K = 0
X ( s) =    + 1
0 1   0     For all roots in left half of s-plane, using Routh criteria
  s  

 ( s + 2)  
s3 1 2
1 1 
s2 3 K
s 1 
s ( s + 2)   
=  1 6−K
 1   s1 0
0  s  3
 ( s + 2) 
6−K  
 1 1  
3 K −0
 + 2  s0    0
s s ( s + 2)    6−K  
=    3  
 1     
 
 s 
6−K
Y ( s) = C X ( s) + Du ( s ) ∴ > 0 and K > 0
3
 1 1  ⇒6−K > 0
 + 2  ⇒ K < 6 and K > 0
s s ( s + 2 )
= [10]   
 1  ∴0 < K < 6
 s 
    Hence, the correct option is (A).
[∵ D = [0]]
1 1
= + 2
s s ( s + 2)
1 1 1 1
= + 2− +
M01_GATE_9789332576063_ME_SET1.indd 55
s 2s 4 s 4( s + 2) 6/20/2017 7:44:48 AM
lvi | GATE 2017 Solved Paper Electrical Engineering: Set – 2

Question Number: 22 Question Type: MCQ (A) x(n) = {1, 2, 1, 1}


The roots locus of the feedback control system having (B) x(n) = {1, 1, 2, 2}
the characteristic equation s2 + 6Ks + 2s + 5 = 0, where (C) x(n) = {1, 1, 1, 1}
K > 0, enters into the real axis at (D) x(n) = {1, 2, 2, 1}
(A) s = −1 (B) s=− 5 Solution: 
(C) s = −5 (D) s= 5
Solution:  h1(n) h2(n)
x(n) y(n)
The characteristic equation is
s 2 + 6 ks + 2 s + 5 = 0
6 ks h(n)
1+ 2 = 0 (2)
s + 2s + 5 Now, h( n) = overall system impulse
Comparing the Eq. (i) with 1 + G(s) = 0, We get response
6 sk = h1 ( n) ∗ h2 ( n)
G ( s) =
s 2 + 2s + 5 = {1, −1} ∗ {1,1}
In order to find the point at which the root locus enters Convolution by tabulation method,
the real axis we have to find the break away/break-in
h1(n)
point. In order find break away/break-in point,
dk 1 −1
=0 h2(n)
1
ds 1 −1

dk [6( s 2 + 2 s + 5) − 6 s( 2 s + 2)]
i.e., =− =0 1 1 −1
ds (6 s ) 2
[6( s 2 + 2 s + 5) − 6 s( 2 s + 2)]
⇒− =0
(6 s ) 2

⇒ 6( s 2 + 2 s + 5) = 6 s( 2 s + 2) h(n) = {1, o, −1}

⇒ s 2 + 2 s + 5 − 2s 2 − 2 s = 0
⇒ − s2 + 5 = 0 We know that,

⇒ s2 = 5 H ( z) = ∑ h(n) ⋅ z −n

n =−∞
∴ s=± 5
= 1 ⋅ z −0 + 0 ⋅ z −1 − 1 ⋅ z −2 = 1 − z −2
If s = 5 root locus plot becomes unstable. As we know,
Hence, s = − 5 is the break-away/break-in point. y( z )
H ( z) =
A cascade system having the impulse responses h1(n) = x( z )
{1, 1} and h2 (n) = {1, 1} is shown in the figure, where ∞

symbol ↑ denotes the time origin. y( z ) = ∑ y(n) ⋅ z


n =−∞
−n

x(n) h1(n) h2(n) y(n)


= 1 + 2 z −1 + z −2 − z −3 − 2 z −4 − z −5
Hence, the correct option is (B). y( z )
x( z ) =
H ( z)
Question Number: 23 Question Type: MCQ
The input sequence x(n) for which the cascade system 1 + 2 z −1 + z −2 − z −3 − 2 z −4 − z −5
=
produces an output sequence y(n) = {1, 2, 1, –1, –2, 1 − z −2
–1} is ⇒ x( z ) = 1 + 2 z −1 + 2 z −2 + z −3

M01_GATE_9789332576063_ME_SET1.indd 56 6/20/2017 7:44:51 AM


GATE 2017 Solved Paper Electrical Engineering: Set – 2 | lvii

x( n) = {1, 2, 2,1} Question Number: 25 Question Type: NAT


↑ Two generating units rated 300 MW and 400 MW have
governor speed regulation of 6% and 4%, respectively
Hence, the correct option is (D). from no load to full load. Both the generating units
Question Number: 24 Question Type: MCQ are operating in parallel to share a load of 600 MW.
Assuming free governor action, the load shared by the
A person decides to toss a fair coin repeatedly until he
larger unit is _______ MW.
gets a head. He will make at most 3 losses. Let the ran-
dom variable Y denote the number of heads. The value Solution:
of var(Y ). Where var(.) denotes the variance, equals: For first generator
7 49 Rating = 300 MW
(A) (B)
8 64 Governor speed regulation = 6%
7 105
(C) (D) \ Frequency droop coefficient
64 64
 6 
− ×f
Solution: 
 100 01 
Maximum no. of losses = 3 K p1 =
300
The various combinations of three tosses = 8
{ f 01 = no load frequency of Gen. 1}
The random variable Y denotes no. of heads
The various combinations are, f1 = K P1 × ( P1 ) + f 01 (3)

H H H → (‘0’ heads) For second generator


H T H Rating = 400 MW
H T T Governor speed regulation = 4%
T H T \ Frequency droop coefficient
T T H  4 
− × f 02 
100
H H T K p2 =  
T H H 300
{ f 02 = no load frequency of Gen. 2}
T T T
f 2 = K P2 × ( P2 ) + f 02 (4)
From the above combinations,
1 Given f 01 = f 02 = f 0
The probability for “0” heads =
8 Since, both machine operate parallel so
7 f1 = f 2
The probability for atleast one head =
8 −0.06 f 0 −0.04 f 0
⇒ P1 = P2
∴ E (Y ) = y1 P( y1 ) + y2 P ( y2 ) 300 400
1 7 7 ⇒ 2 P1 = P2 (5)
= 0 × + 1× =
8 8 8 and P1 + P2 = 600 MW (6 )
 1  7 7
E (Y 2 ) = 0 2 ×   + 12 ×   = Using Eqs. (3) and (4)
8 8 8 P1 = 200 MW
∴ Variance(Y ) = E (Y 2 ) − [ E(Y )]2
And
2
7 7 7 49 P2 = 400 MW
=− = −
8  8  8 64
\ Load shared by larger unit
56 − 49 7
= = i.e., P2 = 400 MW
64 644
Hence, the correct answer is (400 MW).
Hence, the correct option is (C).

M01_GATE_9789332576063_ME_SET1.indd 57 6/20/2017 7:44:54 AM


lviii | GATE 2017 Solved Paper Electrical Engineering: Set – 2

Question Number: 26 Question Type: MCQ S

The eigenvalues of the matrix given below are:



0 1 0 
0 0 1  50 V
0.6 75 V 5Ω
  mH
0 −3 −4 
+

(A) (0, –1, –3) (B) (0, –2, –3)


(C) (0, 2, 3) (D) (0, 1, 3)
Solution:
Solution:  The given ckt is a Buck Boost converter
The given matrix is Given data,

0 1 0  Vs = 50 V
A = 0 0 1  L = 0.6 mH
0 −3 −4  V= V= 75 V
C 0

Now, f = 10 kHz
(A− λI ) = 0 duty cycle D = 0.6

0 1 0  λ 0 0  The output voltage of Buck Boost converter is given by,


( A − λ I ) = 0 0 1  −  0 λ 0  DVs
0 −3 −4   0 0 λ  V0 =
(1 − D )
λ 1 0  0.6Vs
=
=  0 −λ 1  (1 − 0, 6 )
 0 −3 ( −4 − λ )  0.6
= Vs
0.4
On solving,
V0 6 3
= =
⇒ ( −λ ) ( −4 − λ ) − ( −3) − 1[0] + 0 = 0 Vs 4 2

⇒ λ 2 ( 4 + λ ) + 3λ = 0 3
⇒ V0 = × 50 = 75 V
2
⇒ λ 3 + 4λ 2 + 3λ = 0
(
⇒ λ λ 2 + 4λ + 3 = 0 ) Load current
V0 75
∴ λ = ( 0, −1, −3) ( I 0=
=
) = 15 A
R 5
Hence, the correct option is (A). Source current is given by

Question Number: 27 Question Type: NAT (I ) = D I


s (1 − D ) 0
In the circuit shown all elements in the figure, are ideal 3
and the switch S is operated at 10 kHz and 60% duty = × 15
  2
ratio. The capacitor is large enough so that the ripple
across it is negligible and at steady state acquires a volt- = 22.5 A
age as shown. The peak current in amperes drawn from The wave form of current through the inductor shows
the 50V DC source is _________. (Give the answer up load current is sum of source current and capacitor
to one decimal place) current,

M01_GATE_9789332576063_ME_SET1.indd 58 6/20/2017 7:44:57 AM


GATE 2017 Solved Paper Electrical Engineering: Set – 2 | lix

I L (avg) = ( I s )avg + ( I 0 )avg RB


For this circuit, the value of is
= 22.5 + 15 R
(A) 43 (B) 92
= 37.5
(C) 121 (D) 129
IL (Inductor current)
Solution:

VCC = 10 V
IL (max)
Intput loop
IL (avg)
4R
IL (min) RB

C Output loop

DVs VBE _ E
Current ripple, ∆I L =
fL R
0.6 × 50
=
10 × 103 × 0.6 × 10 −3
=5A
∆I
( I L ) max = ( I L )avg + L Given:
2
VCC
5 VCE =
= 37.5 + 2
2
β = 29
= 40 A
VBE = 0.7 V
Peak current I peak = ( I L ) max = 40 A
Applying KVL at input, we get,
Hence, the correct answer is (40).

Question Number: 28 Question Type: MCQ −10 + I B (1 + β )4 R + I B RB + VBE + (1 + β ) I B C


For the circuit shown in the figure, it is given that −10 + I B (1 + 29)4 R + I B RB + 0.7 + (30) I B C
V 9.3 = 150 I B R + I B RB (1)
VCE = CC . The transistor has b = 29 and VBE = 0.7 V
2
when the B-E junction is forward-biased. Applying KVL at output, we get

VCC = 10 V −10 + 4 R(1 + β ) I B + VCE + (1 + β ) I B R = 0


VCC
4R 10 = 4 R(1 + 29) I B + (1 + 29) RI B + =0
RB 2
C 10 = 150 I B R + 5
B 5 = 150 I B R ( 2)
β = 29

E
Substituting Eq. (2) in Eq. (1), we get
R
9.3 = 5 + I B RB
I B RB = 4.3 (3)

M01_GATE_9789332576063_ME_SET1.indd 59 6/20/2017 7:44:59 AM


lx | GATE 2017 Solved Paper Electrical Engineering: Set – 2

From Eq. (1), we get We known in star connection,

 R  I L = I ph
9.3 = I B R 150 + B 
 R V ph
∴ Z=
5  R  I ph
= 150 + B 
150  R VL / 3
[From eq. (2)] =
I ph
9.3 × 150 R
= 150 + B 208/ 3
5 R =
1.9985
93 × 15 R
− 150 = B = 60.08 Ω
5 R
∴ Approximate value of Z
RB
⇒ = 129 = 60.08∠ − cos −1 (0.6)
R
Z = 60.08∠ − 53.13°
Hence, the correct answer is (D).
Hence, the correct option is (C).
Question Number: 29 Question Type: MCQ
Question Number: 30 Question Type: NAT
For the balanced Y-Y connected 3-phase circuit shown
A 3-phase, 2-pole, 50 Hz, synchronous generator has a
in the figure, the line-line voltage is 208V rms and the
rating of 250 MVA, 0.8 pf lagging. The kinetic energy
total power absorbed by the load is 432W at a power
of the machine at synchronous speed is 1000 MJ. The
factor of 0.6 leading.
machine is running steadily at synchronous speed and
a A delivering 60 MW power at a power angle of 10 electri-
cal degrees. If the load is suddenly removed, assuming
Van Z
the acceleration is constant for 10 cycles, the value of
the power angle after 5 cycles is____________ electri-
N cal degrees.
Vcn
Vbn Z Z Solution:
c b C B No of poles (P) = 2
No. of phases = 3
f = 50 Hz (frequency)
The approximate value of the impedance Z is: S = 250 MVA (rating)
(A) 33∠ –53.1ºW (B) 60∠ 53.1ºW
(C) 60∠ –53.1ºW (D) 180∠ –53.1ºW cos φ = 0.8 lag
Solution:  Kinetic energy = 1000 MJ
RMS Line to line voltage (VL) = 208 V
Pe = 60 MW
Total power absorbed by load (P) = 432 W
δ 0 = 10°
Operating power factor = 0.6 leading
When load is removed Pe = 0, then
Power absorbed by 3 f load,
P = 3VL I L cos φ Pa = Pm – Pe
= Pm – 0
432 = 3 ( 208)( I L )(0.6)
Pa = Pm
432
IL = i.e., Pm = 60 MW
3 × 208 × 0.6
= 1.9985 A 50 cycles in 1 sec,

M01_GATE_9789332576063_ME_SET1.indd 60 6/20/2017 7:45:00 AM


GATE 2017 Solved Paper Electrical Engineering: Set – 2 | lxi

10 cycle in? We know, PR = 3VR I R cos φ


10 × 1
t= = 0.2 sec
50 VS = 17.87 kV
VR = 17.32 kV
We know inertia constant (M) G R = 0.25 Ω

KE 1000 50 Hz X = j3.935 WΩ
= = = 0.111 MJ/deg- Hz
180 f 180 × 50 L
O
We know, PR= 3 MW A
d 2δ D
Pa = M 2
dt
Integrating on both sides, we get
t2 PR
Pa =δM IR = (1)
2 3VR cos φ
P t2
δ= a  VS VR 
2M  −  = I R ( R cos φ + X L sin φ )
 3 3
(60 M ) × (0.1) 2
δ= (Vs − VR ) PR
2(0.11) M = ( R cos φ + X L sin φ )
3 3VR cos φ
60 × (0.1) 2
δ= [From eq. (1)]
2( 0.11)
δ = 2.7° PR
VS − VR = ( R cos φ + X L sin φ )
\  The value of power angle after 5 cycles VR cos φ
PR
= (δ + δ 0 ) (5) (VS − VR ) = ( R + X L tan φ )
VR
= ( 2.7 + 10)
VR
= 12.7° (VS − VR ) − R = X L tan φ
PR
Hence, the correct answer is (12.7°).
VR R
Question Number: 31 Question Type: NAT tan φ = (VS − VR ) −
PR × L XL
A 3-phase, 50 Hz generator supplies power of 3 MW at
17.32 kV to a balanced 3-phase inductive load through 17.32 × 103 0.25
= (17.87 × 103 − 17.32 × 103 ) −
an overhead line. The per phase line resistance and 3 × 10 × 3.925
6
3.925
reactance are 0.25 Ω and 3.925 Ω, respectively. If the tan φ = 0.7453
voltage at the generator terminal is 17.87 kV, the power φ = tan −1 (0.7453)
factor of the load is________.
φ = 36.697°
Solution:
∴ cos φ = cos 36.697
f = 50 Hz
= 0.8018
PR = 3 MW
The power factor of the load is 0.8018.
(receiving end power)
Hence, the correct answer is (0.8018).
VR = 17.32 kV(line to line)
Question Number: 32 Question Type: NAT
R = 0.25 Ω /phase
A thin soap bubble of radius R = 1 cm, and thickness a
X L = 3.925 Ω /phase
= 3.3 μm(a << R), is at a potential of 1V with respect to
VS = 17.87 kV (line to line) a reference point at infinity. The bubble bursts and
cos φ = ? becomes a single spherical drop of soap (assuming all

M01_GATE_9789332576063_ME_SET1.indd 61 6/20/2017 7:45:03 AM


lxii | GATE 2017 Solved Paper Electrical Engineering: Set – 2

the soap is contained in the drop) of radius r. The vol- Question Number: 33 Question Type: NAT
ume of the soap in the thin bubble is 4πR2a and that of A 25 kVA, 400 V, ∆-connected, 3-phase, cylindrical
4 rotor synchronous generator requires a field current
the drop is π r 3. The
⋅ potential in volts, of the resulting
3 of 5 A to maintain the rated armature current under
single spherical drop with respect to the same reference short-circuit condition. For the same field current, the
point at infinity is _______. (Give the answer up to two open-circuit voltage is 360 V. Neglecting the armature
decimal places.) resistance and magnetic saturation, its voltage regula-
a tion (in percentage with respect to terminal voltage),
when the generator delivers the rated load at 0.8 pf
leading, at rated terminal voltage is ___________.
R Bursts Solution:
Soap drop
of radius ‘r’
Given data, S = 25 kVA
V = 400 V(l − l )
Soap Bubble I 0 = 5 A (field current)
Solution: V0 = 360 V(OC voltage)
a = 3.3 µm
Ra = 0
25 × 103
I rated =
3V
25000
=
3 × 400
R = 1cm
drop of = 36.08 A (line-line)
radius ‘r’ 36.08
I rated/phase = (for ∆ winding) = 20.833 A
3
V
V bubble =1v then, X s = 0/Phase
I rated/Phase
Using volume conservation, 360
= [∵V0 /( l − l ) = V0/Phase for ∆]
4 20.833
4π R2 a = π r 3 We know voltage regulation can be calculated by cal-
3
culating E.
⇒ 3R a = r 3
2

⇒ r = 0.00096655 m E = [(V cos φ + I a Ra ) 2 + (V sin φ − I a X s ) 2 ]1/ 2


1 Q 1
Vbubble = [( 400)(0.8) + (0)20.83]2 2
4πε 0 R = 2
 +[400 × 0.6 − (17.28)( 20.83)] 
= 1V (1) 1/ 2
= ( 400 × 0.8) 2 + (( 400 × 0.6 − (17.28)( 20.83)) 2 
1 Q
Vdropp = (2) 1
4πε 0 r = [(320) 2 + ( 240 − 359.9424) 2 ] 2
1/ 2
From Eqs. (1) and (2), we get = (320) 2 + (119.9424) 2  = 341.739 V

Vbubble r E −V
= ∴  % Voltage regulation  = × 100
Vdrop R V
1 0.000996655 341.739 − 400
⇒ = = × 100
Vdrop (0.01) 400
⇒ Vdrop = 10.03 V = −0.145 × 100 = −14.50%

Hence, the correct answer is (10.03 V). Hence, the correct answer is (–14.50).

M01_GATE_9789332576063_ME_SET1.indd 62 6/20/2017 7:45:06 AM


GATE 2017 Solved Paper Electrical Engineering: Set – 2 | lxiii

Question Number: 34 Question Type: NAT Question Number: 35 Question Type: MCQ
A star-connected, 12.5 kW, 208 V (line), 3-phase, 60 Hz Which of the following systems has maximum peak
squirrel cage induction motor has following equivalent overshoot due to a unit step input?
circuit parameters per phase referred to the stator. R1 = 100 100
0.3 W, R2 = 0.3 W, X1 = 0.41 W, X2 = 0.41 W. Neglect (A) 2 (B)
s + 10 s + 100 s 2 + 15s + 100
shunt branch in the equivalent circuit. The starting cur-
rent (in Ampere) for this motor when connected to an 100 100
80 V (line), 20 Hz, 3-phase AC source is __________. (C) 2 (D)
s + 5s + 100 s + 20 s + 100
2

Solution:
Solution: 
Given data, V = 208 (line-line)
As we know that damping ratio (x)
P = 12.5 kW
f = 60 Hz ξ ↑= M P ↓
R1 = 0.3 Ω Checking value of x for all options,
R2 = 0.3 Ω 100
(A) 2 ⇒ ωn = 10
X 1 = 0.41 Ω at 60 Hz s + 10 s + 100
X 2 = 0.41 Ω at 60 Hz 2ξωn = 10
Vst = 80V (line-line) ⇒ 2ξ = 1
20 ⇒ ξ = 0.5
at 20 Hz X1 = × 0.41 = 0.1366 Ω
60
20 100
X2 = × 0.41 = 0.1366 Ω (B) 2 ⇒ ωn = 10
60 s + 15s + 100
2ξωn = 15
at starting s = 1.
20ξ = 15
1 
Hence, R2  − 1 = 0 Ω. 3
s  ξ=
4
Ist X1= j0.1366 Ω X2= j0.1366 Ω = 0.75

R1= 0.3 Ω R2= 0.3 Ω 100


80 = V (C) 2 ⇒ ωn = 10
R3 ( 1s −1) = 0.3 Ω s + 5s + 100
√3
= (short) 2ξωn = 5
5
Starting current, ξ=
20
Vst = 0.25
I st =
( R1 + R2 ) + ( X 1 + X 2 ) 2
2
100
(D) 2 ⇒ ωn = 10
80 / 3 s + 20 s + 100
= 2ξωn = 20
(0.3 + 0.3) + (0.1366 + 0.1366) 2
2

20
80 / 3 ξ=
= 20
(0.6 2 + (0.273) 2 =1
80
= = 70.06 A Hence, option (C)
3 × 0.6591
MP will be highest at x = 0.25.
Hence, the correct answer is (70.06 A). Hence, the correct option is (C).

M01_GATE_9789332576063_ME_SET1.indd 63 6/20/2017 7:45:09 AM


lxiv | GATE 2017 Solved Paper Electrical Engineering: Set – 2

Question Number: 36 Question Type: MCQ Question Number: 37 Question Type: MCQ
A 220 V, 10 kW, 900 rpm separately excited DC motor The value of the contour integral in the complex-plane
has an armature resistance Ra = 0.02 Ω. When the
zs − 2z + 3
motor operates at rated speed and with rated terminal
voltage, the electromagnetic torque developed by the
∫ z−2
dz

motor is 70 Nm. Neglecting the rotational losses of along with the contour |z| = 3, taken counter-clockwise
the machine, the current drawn by the motor from the is:
220 V supply is: (A) −18πi (B) 0
(A) 34.2 A (B) 30 A (C) 14πi (D) 48πi
(C) 22 A (D) 4.84 A
Solution:  Solution: 
Rated torque = 70 Nm. Given contour Integral is
Irated = ?
z3 − 2z + 3 z3 − 2z + 3
Ia
∫ z−2
dz where f ( z ) =
( z − 2)

Ra= 0.2 Ω and the contour is |z| = 3 in counter-clockwise.


ϕ = const. Eb 220 V, The pole z = 2 lies inside the contour |z| = 3.
10 kW
Using residue theorem, we get

 z3 − 2z + 3 
In separately excited motor, field flux remains constant. Res( f ( z )) = lim( z − 2)  
x →∞
 ( z − 2) 
Now V = E + I a Ra = 8 − 2( 2) + 3
⇒ 220 = E + I a (0.02) (1) =7
NPφ Using Cauchy’s residue theorem, we get
and E= ( 2)
60 A
ZPφ I a z3 − 2z + 3
T=
2π A
(3) ∫ ( z − 2)
dz = 2π i[Res( f ( z ))]

From Eqs. (2) and (3), we get = 2π i(7)


E ( N )( 2π ) = 14π i
=
T 60 I a Hence, the correct option is (C).
(900)( 2π )(70)
⇒ EI a = Question Number: 28 Question Type: MCQ
60
= 6597.344 (4) For the network given in figure, the Thevenin’s ­voltage
Vab is:
From Eqs. (1) and (4), we get
10 Ω 10 Ω
6597.344
220 = + I a (0.02)
Ia
a
⇒ (0.02) I a2 − 220 I a + 6597.344 = 0 6A 10 Ω
+
16 V
5Ω −
b
⇒ I a = 10969.92, 30.07 A

Discarding excessively high armature current value, we


get rated current value 30.07 A ≈ 30 A. (A) –1.5 V (B) –0.5 V
Hence, the correct option is (B). (C) 0.5 V (D) 1.5 V

M01_GATE_9789332576063_ME_SET1.indd 64 6/20/2017 7:45:11 AM


GATE 2017 Solved Paper Electrical Engineering: Set – 2 | lxv

Solution:  IL

10 Ω 10 Ω Ish Ra
P Q Ia
= 0.8 Ω

240 Ω Eb
a 120 V
6A 5Ω 10 Ω +
− 16 V
b

As the voltage across shunt is constant (120 V), Ish


remains constant,
Applying KCL at points P and Q,
At P: 120
I sh =A = 0.5 A
240
Vp V p − VQ for No load, I 0 = 2 A
6+ + =0
5 10 and I 0 = I a 0 + I sh
⇒ VQ − 3VP = 60
2 = I a 0 + 0.5
At Q :
VQ VQ − 16 VQ − VP ⇒ I a 0 = 1.5 A
+ + =0
10 10 10 Now, vt 0 = E0 + I a 0 Ra
⇒ 3VQ − VP = 16 120 = E0 + (1.5)(0.8)
⇒ E0 = 118.8 V
Solving Eqs. (1) and (2), we get
At full load,
3
VQ = − I L = I a + I sh
2
= −1.5 V 7 = I a + 0..5
⇒ I a = 6.5 A
As VQ is Vab,
Now, Vt = Eb + I a Ra
Hence Vab = –1.5 V.
Hence, the correct option is (A). ⇒ 120 = Eb + (6.5)(0.8)

Question Number: 39 Question Type: NAT ⇒ Eb = 114.8 V


A 120 V DC shunt motor takes 2 A at no load. It takes Now,
7 A on full load while running at 1200 rpm. The arma-
as     E ∝ φ N
ture resistance is 0.8 Ω, and the shunt field resistance
is 240 Ω. The no load speed, in rpm, is __________. and φ = constant for DC shunt motor.

Solution: ∴ E∝N
DC shunt motor, Eb N
Hence =
In No load: E0 N 0
I0 = 2 A 114.8
=
N0 = ? 118.8
114.8
In Full load: ⇒ N0 = × 1200
118.8
IL = 7 A N 0 = 1241.811 rpm.
N = 1200 rpm Hence, the correct answer is (1241.811).

M01_GATE_9789332576063_ME_SET1.indd 65 6/20/2017 7:45:14 AM


lxvi | GATE 2017 Solved Paper Electrical Engineering: Set – 2

Question Number: 40 Question Type: NAT Input power factor


In the circuit shown in the figure, the diode used is Power output at load
ideal. The input power factor is ________. (Give the =
Power input at source
answer up to two decimal places)
V ×I
input p. f = L rms L rms (1)
VS rms × I S rms
100 sin (100π t)V 10 Ω VL rms = R × I L rms
Vm Vm
= R× =
2R 2
Solution: Vm
VL rms
Diode input p. f = = 2
VS rms Vm
2
V = 100 sin (100πt) 10 Ω 1
=
2
input p. f = 0.707
In the above half-wave diode rectifier,
Vin
Hence, the correct answer is (0.707).
Vm Question Number: 41 Question Type: NAT
π 2π 1
ω2 A 10 digit timer counter possesses a base clock of
2
frequency 100 MHz. When measuring a particular
Vo input, the reading obtained is the same in:
Vm (i) Frequency mode of operation with a gating time of
π 2π one second and
ω2
(ii) Period mode of operation (in the x10 ns scale).
The frequency of the unknown input (reading obtained)
V in Hz is________.
V0 = m
π Solution:
 Vm  Frequency of base clock (n) = 100 MHz
π 
∴ I0 =   Gate time (t) = 1 sec.
R Given the reading obtained in frequency mode and
2
π
1  Vm sin ωt  period mode is same
2π ∫0   d (ωt )
I Srms =
R  1. In frequency mode

Vm2 π 2
2π R 2 ∫0
I Srms = sin ωt d (ωt ) n
f =
t
Vm2 π
 1 − cos 2ωt  100 × 106
f =
=
2π R 2 ∫ 
0
2 d (ωt )
 1
f = 108
V 
= m 
 2R  2. In period mode, we know period P of input signal
I Srms = I Lrms is the inverse of its frequency

M01_GATE_9789332576063_ME_SET1.indd 66 6/20/2017 7:45:16 AM


GATE 2017 Solved Paper Electrical Engineering: Set – 2 | lxvii

1 I PL 1 ∵ I PLine = I PPhase 
P= =  
f I SPh 5  in star connection 
1
P= 8 ⇒ I SPh = 4.373 × 5 = 21.86 A
10
= 10 −8 Now I SL = 3 I SPh
= 10 × 10 −9 = 3 × 21.86
(Converting into n-sec scale) = 37.8787 A
P = 10 ns
Hence, the correct answer is (37.8787).
1 Question Number: 43 Question Type: NAT
The frequency and period displayed on 10 digit
2 Consider an overhead transmission line with 3-phase,
1
scale. We know in 10 digit scale the most significant 50 Hz balanced system with conductors located at the
2
bit shows only (0, 1) and remaining digit display from vertices of an equilateral triangle of length Dab = Dbc
(0 to 9). = Dca = 1 m as shown in figure. The resistances of the
conductors are neglected. The geometric mean radius
Thus, frequency is displayed as 100000000.00 Hz. (GMR) of each conductor is 0.01 m. Neglecting the
effect of ground the magnitude of positive sequence
Period is displayed as 100000000.00  n sec. reactance in Ω/km (rounded off to three decimal place)
is _________.
f = 100000000.00 Hz c

Hence, the correct answer is (100000000.00).

Question Number: 42 Question Type: NAT Dca Dbc


If the primary line voltage rating is 3.3 kV (Y side) of
a 25 kVA. Y-∆ transformer (the per phase turns ratio is
5:1), then the line current rating of the secondary side
a b
(in Ampere) is _________. Dab

Solution:
Solution:
c

3.3 KV
Dca Dbc
5:1

Transformer rating = 25 kVA a b


Dab
Finding primary line current,

25 × 103 frequency
I PL =
3 × 3.3 × 10 3
f = 50 Hz
= 4.373 A D= D= Dca = 1 m
ab bc

Now, transforming phase currents GMR = 0.01 m

M01_GATE_9789332576063_ME_SET1.indd 67 6/20/2017 7:45:18 AM


lxviii | GATE 2017 Solved Paper Electrical Engineering: Set – 2

 GMD  Calculating ZL,


we know, L = 2 × 10 −7 ln   …(1)
 GMR  R /jω C
Z L = jω L +
Since, it is equilateral spacing therefore, 1
R+
∴ GMDA = GMDB = GMDC = GMD = 0.01 m jωC
1
 1  ⇒ Z L = j (100)(5 × 10 −3 ) +
∴ L = 2 × 10 −7 ln   j (100C ) + 1
 0.01 
1 − j100C
= 2 × 10 −7 ln (100 ) = j (0.5) +
1 + 10000C 2
= 9.2103 × 10 −7 H
Positive sequence reactance Equating real parts of ZS and ZL
(XL) 1
0.5 =
= 2π fL 1 + 10000C 2
= 2π × 50 × 9.2103 × 10 −7 ⇒ C2 =
1
= 2.8935 × 10 −4 Ω /m 10000
⇒ C = 10 −2
= 0.28935 Ω /km
⇒ C = 10 mF
≈ 0.289 Ω /kkm
Hence, the correct answer is (0.289). Hence, the correct option is (D).
Question Number: 44 Question Type: MCQ Question Number: 45 Question Type: MCQ
In the circuit shown in the figure, the value of capacitor Let
C required for maximum power to be transferred to the
load is  − x x ≤ 1 1 − x x≤0
g( x) =  and f ( x) =  2 .
RS = 0.5 Ω
 x + 1 x ≥ 1  x x≥0

5 mH Consider the composition of f and g., i.e., (f  °g)(x) =


v (t) = 10 sin (100t) f(g(x)). The number of discontinuities in (f  °g)(x) pre-
1Ω C sent in the interval (− ∞, 0) is:
(A) 0 (B) 1
(C) 2 (D) 4
Load
Solution: 
(A) 1 nF (B) 1 μF
(C) 1 mF (D) 10 mF  − x, x ≤1
g( x) = 
Solution:   x + 1, x ≥1
RS = 0.5 Ω 1 − x, x≤0
f ( x) =  2
 x , x≥0
5 mH
1 − g ( x ), g ( x ) ≤ 0
v (t) = 10 sin (100t)
ZL f ° g ( x ) = f ( g ( x )) = 
 ( g ( x )), g ( x ) ≥ 0
2
1Ω C
1 − ( − x ), − x ≤ 0, x ≤ 1
1 − ( x + 1), x + 1 ≤ 0, x ≥ 1
Load 
=
 ( − x ), − x > 0, x ≤ 1
2
For max power transfer theorem,
 ( x + 1) 2 , x + 1 > 0, x ≥ 1
Z L = Z S* 
1 + x 0 ≤ x ≤1
 2
= x x<0
( x + 1) 2 x > 1

x2 x<0
M01_GATE_9789332576063_ME_SET1.indd 68  6/20/2017 7:45:21 AM
1− − g ( x ), g ( x ) ≤ 0
f ° g ( x ) = f ( g ( x )) = 
 ( g ( x )), g ( x ) ≥ 0
2

1 − ( − x ), − x ≤ 0, x ≤ 1
1 − ( x + 1), x + 1 ≤ 0, x ≥ 1

=
 ( − x ), − x > 0, x ≤ 1
2
GATE 2017 Solved Paper Electrical Engineering: Set – 2 | lxix
 ( x + 1) 2 , x + 1 > 0, x ≥ 1

1 + x 0 ≤ x ≤1 Addition of transformer will affect the existing sym-
 metry of the ZBUS and YBUS and thereby making both of
= x2 x<0
(ZBUS) and (YBUS) unsymmetric.
( x + 1) 2 x >1

Hence, the correct option is (D).
x2 x<0
 Question Number: 47 Question Type: MCQ
f ( g ( x )) =  x + 1 0 ≤ x ≤1
( x + 1) 2 Consider a solid sphere of radius 5 cm made of a per-
 x >1
fect electric conductor. If one million electrons are
added to this sphere, these electrons will be distributed
f ° g ( x ) ⋅ is continuous at every point in ((−∞, 0).
(A)  Uniformly over the entire volume of the
⇒ Number of points of discontinuities = 0. sphere
Hence, the correct option is (A). (B) Uniformly over the outer surface of the sphere
(C) Concentrated around the centre of the sphere
Question Number: 46 Question Type: MCQ (D) Along a straight line passing through the cen-
The figure shows the per-phase representation of a tre of the sphere
phase-shifting transformer connected between buses 1
and 2, where α is a complex number with non-zero real Solution: 
and imaginary parts. The electric field inside a perfect electric conductor is
Ideal 0, hence all the charge added to the sphere will be dis-
transformer tributed uniformly over the surface of the sphere, so
Z that at every point on the sphere, the electric field from
Bus 1 Bus 2
the surface of the sphere is radial in direction.
Hence, the correct option is (B).
Question Number: 48 Question Type: MCQ
The figures show diagrammatic representations of vec-
  
tor fields,  X , Y and Z , respectively. Which one of the
For the given circuit, Ybus and Zbus are bus admittance following choices is TRUE?
matrix and bus impedance matrix, respectively, each of
size 2 × 2. Which one of the following statements is X y Z
TRUE?
(A) Both Ybus and Zbus are symmetric
(B) Ybus is symmetric and Zbus is unsymmetric
(C) Ybus is unsymmetric and Zbus is symmetric
(D) Both Ybus and Zbus are unsymmetric
  
Solution:  ∇. X
(A) = 0, ∇ × Y ≠ 0, ∇ × Z = 0
Consider the figure given below   
∇. X
(B) ≠ 0, ∇ × Y = 0, ∇ × Z ≠ 0
Ideal
transformer
  
∇. X
(C) ≠ 0, ∇ × Y ≠ 0, ∇ × Z ≠ 0
  
Bus 1 Bus 2 ∇. X
(D) = 0, ∇ × Y = 0, ∇ × Z = 0
Z

Solution: 
From the given figures, we can observe that
Fig (1): X is diverging field hence its divergence of X
i.e., v • X * 0
Fig (2): Y is circularly rotating field hence its curl of Y
i.e., (V x Y) * 0.

M01_GATE_9789332576063_ME_SET1.indd 69 6/20/2017 7:45:24 AM


lxx | GATE 2017 Solved Paper Electrical Engineering: Set – 2

Fig (3): Z is also a circularly rotating field hence its curl Z


of Z, i.e., (V x Z) * 0
Hence, option (C) satisfies the above three conditions.
Hence, the correct option is (C). Y Y
2 2
Question Number: 49 Question Type: MCQ
A stationary closed Lissajous pattern on an oscillo-
scope has 3 horizontal tangencies and 2 vertical tan-
gencies for a horizontal input with frequency 3 kHz.
The frequency of the vertical input is Comparing this network with given Network, we get
(A) 1.5 kHz (B) 2 kHz
(C) 3 kHz (D) 4.5 kHz Y 1
=
2 3300
Solution:  2
Y =
f y No. of horizontal tangencies 3300
= 1
fx No. of vertical tangenciess Y = Ω
1650
fy 3 z = 100 Ω
=
3 kHz 2 The magnitude of characteristic impedance for a trans-
mission line is
f y = 4.5 kHz
Z
(Z ) =
Hence, the correct option is (D). c Y
100
Question Number: 50 Question Type: NAT =
(1/1650)
The nominal-π circuit of a transmission line is shown
in the figure = 1650 × 100
= 165000
Z
∴  Characteristic impedance
ZC = 406.20 Ω .
X X
Hence, the correct answer is (406.20 Ω).
Question Number: 51 Question Type: NAT
Impedance Z = 100∠80 Ω and reactance X = 3300 Ω. The figure shows the circuit diagram of a controlled
rectifier supplied from a 230 V, 50 Hz, 1-phase voltage
The magnitude of the characteristic impedance of the source and a 10:1 ideal transformer. Assume that all
transmission line in Ω⋅, is _____. (Give the answer up devices are ideal. The firing angles of the thyristors T1
to one decimal place). and T2 are 90° and 270°, respectively.
Solution:
T1 T2
The impedance and reactance is
230 V, 10:1 DS R
Z = 100∠80 50 Hz

X = 3300 Ω iDS
D1 D2
The general structure of nominal Π network is:

M01_GATE_9789332576063_ME_SET1.indd 70 6/20/2017 7:45:27 AM


GATE 2017 Solved Paper Electrical Engineering: Set – 2 | lxxi

The RMS value of the current through diode D3 in Solution:


amperes is _________. Let time of arrival be a random variable x.
Solution: 1
In the given circuit diagram the load is a resistance and  , 0≤ x≤5
f ( x) =  5
hence, there is no stored energy in the resistor (which will  0, else where
make the freewheeling diode to conduct) and hence, there
is no current in the freewheeling diode D3 and the same Assume waiting time be g(x), a function of arrival time.
can be shown in the waveforms shown in the figure.  0, 0 ≤ x ≤ 2 (Greenlight)
g( x) = 
Vs(supply 5 − x , 2 ≤ x ≤ 5 (Redlight)
voltage) Average waiting time = E(g(x))
5
= ∫ g ( x ) f ( x )dx
0
2 1 5 1
2π = ∫ 0 × dx + ∫ (5 − x ) × dx
π
ωt 0 5 2 5
5
 x2 
Vo(load = x − 
voltage)  10 
2
9
ωt = = 0.9
α = 90° α = 270° 100
Io(load Hence, the correct answer is (0.9).
current)
Question Number: 53 Question Type: NAT
ωt Let y − 2 y + 1 = x and
2
x + y = 5. The value of

T1D1 T2D2 x + y equals_____. (Give the answer up to three


decimal paces).
ID3(free wheeling Solution:
diode current)
ωt Given:
y 2 − 2 y + 1 = x (1)
Whenever, current flow in load is continuous, free-
and
wheeling diode comes into conduction. Here current in
load is discontinuous therefore the RMS value of cur- x + y = 5 (2)
rent through diode D3 is 0 amperes. Using Eqs. (1) and (2), we get
Hence, the correct answer is (0). y 2 − 2 y + 1 = (5 − y ) 2
Question Number: 52 Question Type: NAT ⇒ y 2 − 2 y + 1 = 25 + y 2 − 10 y
Assume that in a traffic junction, the cycle of the traf- ⇒ 8 y = 24
fic signal lights is 2 minutes of green (vehicle does not ⇒ y = 3 and x = 4
stop) and 3 minutes of red (vehicle stops). Consider
that the arrival time of vehicles at the junction is uni- ∴ x+ y = 4+ 3
formly distributed over 5 minute cycle. The expected = 4 + 1.732 = 5.732
waiting time (in minutes) for the vehicle at the junction
is _________. Hence, the correct answer is (5.732).

M01_GATE_9789332576063_ME_SET1.indd 71 6/20/2017 7:45:29 AM


lxxii | GATE 2017 Solved Paper Electrical Engineering: Set – 2

Question Number: 54 Question Type: MCQ Phase lead occurs between ω1 = 0.1 and ω2 = 1. The
The transfer function C(s) of a compensator is given frequency at which maximum phase lead occurs also
below, lie in between ω = 0.1 and 1 and its value is
1 1
 s  s  ωm = = = 0.316 rad/s
1 + 0.1  1 + 100  β ×T 2
T1 β
C ( s) =   . 1

 s  Hence, the correct option is (A).


(1 + s) 1 + 
 10 
Question Number: 55 Question Type: MCQ
The frequency range in which the phase (lead) intro- Two resistors with nominal resistance values R1 and R2
duced by the compensator reaches the maximum is have additive uncertainties ∆R1 and ∆R2, respectively.
(A) 0.1 < ω < 1 (B) 1 < ω < 10 When these resistances are connected in parallel, the
(C) 10 < ω < 100 (D) ω > 100 standard deviation of the error in the equivalent resist-
ance R is
Solution:
2 2
The transfer function given is a lead-lag compensator  ∂R   ∂R 
(A)
±  ∆R1  +  ∆R2 
 s  s   ∂R1   ∂R2 
1 + 0.1  1 + 100 
C ( s) =   
 ∂R   ∂R 
2 2

 s  (B)
±  ∆R1  +  ∆R2 
(1 + s) 1 +  ∂R ∂R
 10   2   1 
2 2
Comparing with the standard transfer function of lead (C)  ∂R   ∂R 
±   ∆R2 +   ∆R1
compensator  ∂R1   ∂R2 
(1 + T1 s)(1 + T2 s)
C ( s) =  ∂R 
2
 ∂R 
2
(1 + β T1 s)(1 + αβ T2 s) (D)
±   ∆R1 +   ∆R2
∂R
 1  ∂R2 
1 1
T1 = , T2 = ⇒ ω1 = 0.1ω4 = 100
0.1 100 Solution: 
1 Since, resistors R1 and R2 are connected in parallel,
β T1 = 1, α T2 = ⇒ ω2 = 1 ω3 = 10
10 uncertainties are
β = 0.1, α = 10 w R1 = ∆R1
The location of zeros at ω = 0.1,100 w R2 = ∆R2
The location of poles at ω = 1, 10 The deviation of error is given by standard formula,
Now drawing the bode plot for the given pole zero
locations is shown in the figure. 2
 ∂R  2  ∂R  2
2

wR = ±   w R1 +   w R2
Magnitude  ∂R1   ∂R2 
2 2
 ∂R   ∂R  2
=±   ∆R1 +   ∆2
2

 ∂R1   ∂R2 

Hence, the correct option is (A).

× ω Question Number: 56 Question Type: NAT


ω m = 0.316 rad/s ω 1 = 0.1 ω 2 = 1 ω 3 = 10 ω 4 = 10 In a load flow problem solved by Newton-Raphson
Lead Lag method with polar coordinates, the size of the Jacobian

M01_GATE_9789332576063_ME_SET1.indd 72 6/20/2017 7:45:32 AM


GATE 2017 Solved Paper Electrical Engineering: Set – 2 | lxxiii

is 100 × 100. If there are 20 PV buses in addition to PQ Question Number: 58 Question Type: NAT
buses and a slack bus, the total number of buses in the The initial charge in the 1F capacitor present in the circuit
system is. shown in the figure is 0. The energy in joules transferred
Solution: from the DC source until steady state condition is reached
Size of Jacobian [J] = 100 equals. (Give the answer up to one decimal place).
No. of PV bases [NPV] = 20 1F

One slack bus,


Let total no. of buses = N 5Ω 5Ω

Using J = 2 N − N PV − 2
+
⇒ 100 = 2 N − 20 − 2 10 V


⇒ 2 N = 122 5Ω 5Ω
⇒ N = 61

Hence, the correct option is (61).

Question Number: 57 Question Type: NAT Solution:


Let x and y be integers satisfying the following 1F
equations
5Ω 5Ω
2 x 2 + y 2 = 34 +
10 V

x + 2 y = 11 5Ω 5Ω

The value of (x + y) is

Solution:
The given equations are In the given CKT, the bridge is balanced, hence the
equivalent resistance would be 5 Ω.
2 x 2 + y 2 = 34 (1)
     
1F
and x + 2 y = 11 (2)
Solving Eqs. (1) and (2), we get 10 V + 5Ω

2
 11 − x 
2x2 +   = 34
 2 
The current in the CKT will decay according to i = i0e−t/RC
⇒ 8 x 2 + 121 + x 2 − 22 x = 136
till the steady state, when the CKT will be open and
⇒ 9 x 2 − 22 x − 15 = 0 capacitor will be charged upto +10 V in opposition of
Now using Sridharacharya formula, we get the voltage source. Energy transferred from voltage
source will be
22 ± 32 −10
x= = 3,
18 18 ∞
10 = ∫ E dt
i
Discarding − as a root, as it is given that 18x and y 0
18
∞  10 
= ∫ (10)   e −t /RC dt
are integers.
∴ y=4 0  5
Hence, value of (x + y) = 3 + 4 = 7 ∞
= 20 ∫ e −t /RC dt
Hence, the correct answer is (7). 0
= 20( −et /RC ⋅ RC ) ∞
0
= 20 RC = 20 × 5 × 1
= 100 J
M01_GATE_9789332576063_ME_SET1.indd 73 6/20/2017 7:45:35 AM

= ∫ E dt
i
0
∞  10 
= ∫ (10)   e −t /RC dt
0  5

lxxiv | GATE 2017 e −t /RC
= 20 ∫Solved Paper
dt Electrical Engineering: Set – 2
0
Fundamental rms of phase voltage will be
= 20( −et /RC ⋅ RC ) ∞
0
6Vdc
= 20 RC = 20 × 5 × 1
= π
= 100 J 3
2Vdc
Hence, the correct answer is (100). =
π
Question Number: 59 Question Type: MCQ Peak of fundamental of phase voltage will be
A three-phase voltage source inverter with ideal
devices operating in 180° conduction mode is feeding a  2Vdc  2V
=  ( 2 ) = dc
balanced star-connected resistive load. The DC voltage  π π
 
input is Vdc. The peak of the fundamental component of
the phase voltage is Hence, the correct option is (B).

V 2V Question Number: 60 Question Type: MCQ


(A) dc ⋅ (B) dc ⋅
A phase-controlled, single-phase, full-bridge converter
π π is supplying a highly inductive DC load. The converter
is fed from a 230 V, 50 Hz, AC source. The fundamental
3V 4V
(C) dc ⋅ (D) dc ⋅ frequency in Hz of the voltage ripple on the DC side is
π π (A) 25 (B) 50
(C) 100 (D) 300
Solution: 
Solution:
Consider the figure given
Single phase, full bridge converter supplying highly
inductive DC load.
Vdc iO

Vz = 230 V R Inductive
50 Hz DC
3 ϕ Star
L Load
Load

VS
50 Hz
Rms value of line voltage will be
Wt
4V  nπ  α
VLn = dc cos  
2nπ  6 
For fundamental, n = 1, rms line voltage. Therefore, 100 Hz

4Vdc π  Wt
VL1 = cos  
2π 6
4Vdc 3 From the figure, we conclude that
=
2π 2 f v0 = 2 f vs
2Vdc 3 = 2(50)
=
π
= 100 Hz
6Vdc
= Hence, the correct option is (C).
π

M01_GATE_9789332576063_ME_SET1.indd 74 6/20/2017 7:45:38 AM


GATE 2017 Solved Paper Electrical Engineering: Set – 2 | lxxv

Question Number: 61 Question Type: MCQ


Which one of the followings is TRUE about the fre-
quency selectivity of these systems?
An urn contains 5 red balls and 5 black balls. In the
(A) All three are high-pass filters
first draw, one ball is picked at random and discarded
(B) All three are band-pass filters
without noticing its colour. The probability to get a red
(C) All three are low-pass filters
ball in the second draw is
(D)  P is low-pass filter, Q is a band-pass filter and
1 4 R is a high-pass filter
(A) (B)
2 9
Solution: 
5 6 Im(z)
(C) (D)
9 9 P
2 poles
Solution: 
5 Red balls and 5 Black balls.
One ball discarded in first draw
× Re(z)

Discarded Unit
ball 4 circle
9 R

5
= 1 k ( z − 1)( z + 1)
10 2 R
5 B ∴ H ( z) =
9 ( z )( z )
5
9 R k ( z 2 − 1)
5
= 1
B =
10 2 z2
At low frequency,
4 B
9
z =1
k ( 0)
H (1) =
The probability of favourable branches from above 1
­figure will be =0
At high frequency,
1 4 1 5
 2× 9+ 2×9 z = −1
   
k ( 0)
4 5 H ( −1) =
= + 1
18 18
=0
1
= It is a band pass filter as output is 0 at both high and
2 low frequencies.
Hence, the correct option is (A). Im(z)
Q
Question Number: 62 Question Type: MCQ
The pole-zero plots of three discrete-time systems P, Q × 0.5
and R on the z-plane are shown in the figure
Re(z)
P Im(z) Q Im(z) R Im(z)
2 poles × 0.5
Re(z) 0.5 Re(z) Re(z) Unit
0.5 circle
Unit Unit Unit
circle circle circle Poles at Z = 0.5i, −0.5i

M01_GATE_9789332576063_ME_SET1.indd 75 6/20/2017 7:45:40 AM


lxxvi | GATE 2017 Solved Paper Electrical Engineering: Set – 2

Zeroes at At high frequency


z = 1, − 1
z = −1
k ( z − 1)( z + 1)
∴ H ( z) = k ( 0)
( z − 0 ⋅ 5i )( z + 0 ⋅ 5i ) H ( −1) =
2
k ( z 2 − 1) =0
=
z 2 + 0.25
It is a band pass filter as output is 0 at both low and high
At low frequency frequencies.
Hence, all the three are band pass filters.
z =1
Hence, the correct option is (B).
k ( 0)
H (1) =
1 + 0.25 Question Number: 63 Question Type: NAT
=0
For the given 2-port network, the value of transfer
impedance z21 in ohms is _________
It is a band pass filter as output is 0 at both low and high
frequencies

Im(z)
R
×
1 2
4Ω 2Ω

Re(z) 2Ω

1′ 2′
Unit
circle ×
Poles at Z = i, −i
Solution:
Zeroes at Z = 1, −1 Given circuit diagram is

k ( z − 1)( z + 1) 2Ω
∴ H ( z) =
( z − i )( z + i )
k ( z 2 − 1) 1 2
H ( z) =
( z 2 + 1) 4Ω 2Ω

2Ω
At low frequency
1′ 2′
z =1
k ( 0)
H (1) =
2 Converting the above circuit diagram into standard
=0 T-network by using Y−D transformation, we get

M01_GATE_9789332576063_ME_SET1.indd 76 6/20/2017 7:45:42 AM


GATE 2017 Solved Paper Electrical Engineering: Set – 2 | lxxvii


The value of Z21 in ohms is 3 Ω .⋅
R1 R2
Hence, the correct Answer is (3).

R3 Question Number: 64 Question Type: MCQ


1 2 For a 3-input logic circuit shown in the figure, the out-
4Ω 2Ω put Z can be expressed as

P
1′ 2′
Q Z

4×2 8
R1 = = =1Ω
2+4+2 8 R
2× 2 4
R2 = = = 0.5 Ω
2+4+2 8
Q + R (B)
(A) PQ + R
4×2 8
R3 = = =1Ω
2+4+2 8 Q + R (D)
(C) P+Q+R

Redrawing the given T-network with the above values Solution: 


Given ckt diagram is,

I1 1Ω 0.5 Ω I2
P
1

Q 3 Z

V1 V2 2
R
I1 I2

Output of NAND gate 1 is = PQ


Output of NAND gate 2 is = QR

applying KVL at the input and output loops, we get ( ) ( )


Output of NAND gate 3 is = PQ (Q ) QR

= PQ + (Q ) + QR
V1 = 4 I1 + 3I 2
V2 = 3I1 + 3.5 I 2 = PQ + Q + QR
We know = Q( P + 1) + QR
V2 [∵ (1 + P ) = 1]
Z 21 =
I2 I2 −0 = Q + QR
V2 A + AB = A + B]
∴ Z 21 = =3Ω Hence, the correct option is[∵
(C).
I1
= (Q )

M01_GATE_9789332576063_ME_SET1.indd 77 6/20/2017 7:45:45 AM


lxxviii | GATE 2017 Solved Paper Electrical Engineering: Set – 2

Question Number: 65 Question Type: NAT ∂f


To find:
Consider a function f (x, y, z) given by ∂x x = 2 , y =1, z = 3

f ( x, y, z ) = ( x2 + y 2 − 2 z 2 )( y 2 + z 2 ) f ( x, y, z ) = x2 y 2 + y 4 − 2 y 2 z 2 + x 2 z 2 + y 2 z 2 − 2 z 4

The partial derivative of this function with respect to x ∂f


= 2 xy 2 + 0 − 0 + 2 xz 2 + 0 − 0
at the point, x = 2, y = 1 and z = 3 is _____ ∂x
= 2( 2)(1) 2 + 2( 2)(3) 2
Solution:
= 4 + 36 = 40
f ( x, y, z ) = ( x2 + y 2 − 2 z 2 )( y 2 + z 2 ) Hence, the correct answer is (40).

M01_GATE_9789332576063_ME_SET1.indd 78 6/20/2017 7:45:46 AM


Detailed Analysis of GATE 2016 Papers
GATE EE Solved 2016 Paper (Set 1) Detailed Analysis
Subject Topic 1 Mark 2 Marks Total Total
Questions Questions Questions Marks
General Aptitude Numerical Ability 4 3 6 10
Verbal Ability 3 1 4 5
Total Marks 15
Engineering Maths Calculus 1 — 1 1
Linear Algebra 1 2 3 5
Transform Theory 1 — 1 1
Differential Equations 1 — 1 1
Complex Variables 1 1 1 1
Probability — 1 1 2
Numerical Methods 2
Total Marks 13
Machines Induction Machine 1 0 1 1
Synchronous Machine 1 1 2 3
DC Machine 1 1 2 3
Transformers 1 3 4 7
Total Marks 14
Power Systems Load Flow 1 0 1 1
Fault Analysis 1 1 2 3
Cables 0 1 1 2
Trasmission Line Parameters 0 1 1 2
Protection (Relays) 0 0 0 0
Total Marks 8
Power Electronics Choppers 1 0 1 1
Semi Conductor Devices 1 1 2 3
Rectifiers 1 1 2
Inverters 2 2 4
Total marks 10
Electrical Bridges 0 1 1 2
Measurements
Meters 0 1 1 2
Energy Meter
Total Marks 4
Control Systems Frequency Analysis 2 2 4 6
State Estimation 0 1 1 2
Stability 0 1 1 2
Control System Basics 0 0 0 0

F01_Gate_EE_2016 Analise Page.indd 79 6/19/2017 7:09:17 PM


lxxx | Detailed Analysis of GATE 2016 Papers

Lead Lag Networks 0 0 0 0


Time Domain Analysis
Root Locus
Basics
Total Marks 10
Networks Basic Elements 2 0 2 2
Nodal Analysis 1 1 2 3
Transient Analysis 0 1 1 2
Resonant Circuits 0 1 1 2
AC Circuit Analysis 0 0 0 0
Two Port Networks 0 0 0 0
Total Marks 9
Analog Zener Diode 1 0 1 1
Filters 0 0 0 0
Op amp
Total Marks 1
Digital Multiplexer 1 0 1 1
Analog to Digital Converter 1 1 2 3
Sequential Circuits 0 1 1 2
Karnagh Maps 0 0 0 0
Boolean Algebra 0 0 0 0
Total Marks 6
Electro Magnetic Static Electric Fields 1 1 2 3
Fields
Magnetic Fields 1 0 1 1
Total Marks 4
Signals and Continuous System 1 1 2 3
Systems
Continuous Signals 1 0 1 1
Fourier Transforms 0 1 1 2
Convolution 0 0 0 0
Total Marks 6

GATE EE Solved 2016 Paper (Set 2) Detailed Analysis


Subject Topic 1 Mark 2 Marks Total Total
Questions Questions Questions Marks
General Aptitude Numerical Ability 2 4 6 10
Verbal Ability 3 2 5 5
Total Marks 15
Engineering Maths Complex Variables 1 1 1
Differential Equations 1 1 1
Linear Algebra 1 1 2 3

F01_Gate_EE_2016 Analise Page.indd 80 6/19/2017 7:09:17 PM


Detailed Analysis of GATE 2016 Papers | lxxxi

Calculus 2 1 3 4
Laplace Transform 2 2 4
Probability 1 1 2
Total Marks 15
Machines Induction Machine 0 1 1 2
Synchronous Machine 1 1 2 3
dc Machine 0 1 1 2
Transformers 1 2 3 5
Total Marks 12
Power Systems Load Flow 1 0 1 1
Fault Analysis 1 2 3 5
Cables 1 0 1 1
Trasmission Line Parameters 0 1 1 2
Protection (Relays) 1 0 1 1
Total Marks 10
Power Electronics Choppers 1 1 2 3
Semi Conductor Devices 0 0 0 0
Rectifiers 1 2 3 5
Inverters 0 1 1 2
Total Marks 10
Electrical Bridges 0 0 0 0
Measurements
Meters 0 0 0 0
Energy Meter 1 1 2
Total Marks 2
Control Systems Frequency Analysis 1 1 1 1
State Estimation 0 0 0 0
Stability 0 0 1 2
Control System Basics 1 1 1 1
Lead Lag Networks 1 1 1 1
Time Domain Analysis 1 1 2
Root Locus 1 1 2
Basics 1 1 2
Total Marks 11
Networks Basic Elements 0 0 0 0
Nodal Analysis 2 1 3 4
Transient Analysis 0 1 1 2
Resonant Circuits 0 0 0 0
AC Circuit Analysis 2 1 3 4
Two Port Networks 0 1 1 2
Total Marks 12

F01_Gate_EE_2016 Analise Page.indd 81 6/19/2017 7:09:18 PM


lxxxii | Detailed Analysis of GATE 2016 Papers

Analog Zener Diode 0 0 0 0


Filters 1 0 1 1
OP amp 1 1 2
Total Marks 3
Digital Multiplexer 0 0 0 0
Analog to Digital Converter 0 0 0 0
Sequential Circuits 0 0 0 0
Karnagh Maps 1 0 1 1
Boolean Algebra 0 1 1 2
Total Marks 3
Electro Magnetic Static Electric Fields 2 1 3 4
Fields
Magnetic Fields 0 0 0 0
Total Marks 4
Signals and Continious System 0 0 0 0
Systems
Continious Signals 0 0 0 0
Fourier Transforms 0 1 1 2
Convolution 1 0 1 1
Total Marks 3

F01_Gate_EE_2016 Analise Page.indd 82 6/19/2017 7:09:18 PM


GATE 2016 Solved Paper
Electrical Engineering
Set - 1
Number of Questions: 65 Total Marks: 100.0

Wrong answer for MCQ will result in negative marks, (-1/3) for 1 Mark Questions and (-2/3) for 2 Marks
Questions.

General Aptitude
Number of Questions: 10 Section Marks: 15.0

Q. 1 to Q. 5 carry 1 mark each and Q. 6 to Q. 10 carry Question Number: 4 Question Type: MCQ
2 marks each Pick the odd one from the following options.
Question Number: 1 Question Type: MCQ (A) CADBE (B) JHKIL
(C) XVYWZ (D) ONPMQ
The man who is now Municipal Commissioner worked
as ________. Solution:  The arrangement within the group is similar
in options (A), (B) and (C), but different in option (D).
(A) the security guard at a university
Hence, option (D) is the odd one.
(B) a security guard at the university
(C) a security guard at university Hence, the correct option is (D).
(D) the security guard at the university Question Number: 5 Question Type: MCQ
Solution:  The reference is to a particular person who In a quadratic function, the value of the product of the
worked as a security guard. α n + βn
roots (a, b) is 4. Find the value of − n :
Hence, the correct option is (B). α + β− n
Question Number: 2 Question Type: MCQ (A) n4 (B) 4n
(C) 2 2n-1 (D) 4n-1
Nobody knows how the Indian cricket team is going
to cope with the difficult and seamer-friendly wickets Solution:  Let the quadratic equation be ax2 + bx + c = 0.
in Australia. Choose the option which is closest in If the roots of quadratic equation are a, b
meaning to the underlined phrase in the above sentence. b
\ a + b =
(A) put up with (B) put in with a
(C) put down to (D) put up against c
and ab =
Solution: To cope with something or someone is to a
bear with something or someone. Given ab = 4
Hence, the correct option is (A). α n + βn α n + βn n n
−n −n
= n n
α β
Question Number: 3 Question Type: MCQ α + β α + β
Find the odd one in the following group of words. = (ab)n = 4n
mock, deride, praise, jeer: Hence, the correct option is (B).
(A) mock (B) deride
Question Number: 6 Question Type: MCQ
(C) praise (D) jeer
Among 150 faculty members in an institute, 55 are
Solution: The words mock, deride, and jeer convey
connected with each other through Facebook® and
the same meaning.
85 are connected through WhatsApp®. Thirty faculty
Hence, the correct option is (C). members do not have Facebook® or WhatsApp®

F02_Gate_EE_2016_978-93-325-7605-6_P-I.indd 83 6/19/2017 7:10:44 PM


lxxxiv | GATE 2016 Solved Paper Electrical Engineering: Set – 2

accounts. The number of faculty members connected Question Number: 8 Question Type: MCQ
only through Facebook® accounts is ________. All hills-stations have a lake. Ooty has two lakes.
(A) 35 (B) 45 (i) Ooty is not a hill-station.
(C) 65 (D) 90
(ii) No hill-station can have more than one lake.
Solution: Consider the Venn diagram given below. (A) (i) only (B) (ii) only
Here F is Facebook and W is whatsApp (C) both (i) and (ii) (D) neither (i) nor (ii)
F(55) Solution: Neither statement (i) nor statement (ii) is
W(85)
logically valid.
Hence, the correct option is (D).
35 20 65
Question Number: 9 Question Type: MCQ
In a 2 × 4 rectangle grid shown below, each cell is a
30 rectangle. How many rectangles can be observed in the
grid?
As 30 faculty members have neither account, 120 have
accounts. As 55 have a Facebook account and 85 have
a WhatsApp account.
The members who have have either account will be
= 55 + 85 - 120, i.e., 20 members have both accounts.
The number of faculty members who have only a
Facebook account is = 55 - 20, i.e., 35.
Hence, the correct option is (A). (A) 21 (B) 27
Question Number: 7 Question Type: MCQ (C) 30 (D) 36
Computers were invented for performing only high-end Solution: To select a rectangle from the grid, from
useful computations. However, it is no understatement the 5 vertical lines we have to select 2 and from the
that they have taken over our world today. The internet, 3 horizontal lines we have to select 2. This number
for example, is ubiquitous. Many believe that the of ways in which this can be done is 5C2 3C2 = 10(3)
internet itself is an unintended consequence of the = 30 ways.
original invention. With the advent of mobile computing Hence, the correct option is (C).
on our phones, a whole new dimension is now enabled.
One is left wondering if all these developments are Question Number: 10 Question Type: MCQ
good or, more importantly, required. Choose the correct expression for f (x) given in the
Which of the statement(s) below is/are logically valid graph.
and can be inferred from the above paragraph?
(i) The author believes that computers are not f(x)
good for us.
(ii) Mobile computers and the internet are both
intended inventions
(A) (i) only
(B) (ii) only x
(C) both (i) and (ii)
(D) neither (i) nor (ii)
Solution: The author is not concluding that the
computers are not good. Therefore, statement (i) is not
valid. As per author statement (ii) is not valid.
Hence, the correct option is (D).

F02_Gate_EE_2016_978-93-325-7605-6_P-I.indd 84 6/19/2017 7:10:46 PM


GATE 2016 Solved Paper Electrical Engineering: Set – 2 | lxxxv

(A) f (x) = 1 - | x - 1 | (B)


f (x) = 1 + | x + 1 | a negative sign [we reject option (B) and (D)] at the
(C) f (x) = 2 - | x - 1 | (D)
f (x) = 2 + | x - 1 | vertex of the V, i.e., at x = 1, y is 2. We accept option (C)
Solution:  V which opens downwards is shown in graph and reject option (A).
given above. The mod expression has to be preceded by Hence, the correct option is (C).

Electrical Engineering
Number of Questions: 55 Section Marks: 85.0

Q. 11 to Q. 35 carry 1 mark each and Q. 36 to Q. 65 Solution: Given


carry 2 marks each f (t) = e2t sin (5t) u (t)
Question Number: 11 Question Type: NAT We know that
The maximum value attained by the function f (x) 5
= x (x - 1) (x - 2) in the interval [1, 2] is _________. L [e2t sin (5t)] = 2
  ( s − 2 ) + 52
Solution: Function
f (x) = x (x - 1) (x - 2) Let g (t) = e2t sin (5t)
We known that f (1) = f (2) = 0 in [1, 2] \ L [g (t)] = L [e2t sin (5t)]
and f (x) < 0 for 1 < x < 2 5
Therefore, the maximum value of f (x) in interval = 2 2
= g ( s)
( s − 2 ) + 5
[1, 2] is 0.
Hence, the correct Answer is (0). From the second shifting theorem, we know that
Question Number: 12 Question Type: NAT L[g(t - a) u(t - a)] = g ( s)e − as
Consider a 3 × 3 matrix with every element being equal Here a = 0
to 1. Its only non-zero eigenvalue is ________.
\ L [  f (t)] = L [e2t sin (5t) u (t)]
Solution:  Consider 3 × 3 matrix A given below:
= L [g (t) u (t)]
1 1 1
A = 1 1 1 = L [g(t - 0) u (t - 0)]
1 1 1
  = g ( s)e − os

The characteristic equation of A is 5
|A - l| = 0 =
( s − 2) 2 + 52
1− λ 1 1
⇒ 1 1− λ 1 =0 5
1 1 1− λ = 2
s − 4 s + 29
⇒ -l3 + 3l2 = 0 ⇒ l2 (l - 3) = 0 Hence, the correct option is (A).
⇒ l = 0; l = 0 and l = 3
\ The only non zero eigen value of A is 3. Question Number: 14 Question Type: MCQ
Hence, the correct Answer is (3). A function y (t) such that y (0) = 1 and y (1) = 3e-1
Question Number: 13 Question Type: MCQ d2 y
is a solution of the differential equation +
The Laplace Transform of f (t) = e2t sin (5t) u(t) is dt 2
________. dy
2 + y = 0 then y (2) is ________.
5 5 dt  
(A) 2 (B) 2
s − 4 s + 29 s +2
(A) 5e-1 (B) 5e-2
s−2 5
(C) 2 (D) (C) 7e-1 (D) 7e-2
s − 4 s + 29 s+5

F02_Gate_EE_2016_978-93-325-7605-6_P-I.indd 85 6/19/2017 7:10:49 PM


lxxxvi | GATE 2016 Solved Paper Electrical Engineering: Set – 2

Solution:  The differential equation given is Im


2
d y dy
2
+2 + y = 0 (1)
dt dt
Also y (0) = 1 and y (1) = 3e-1 (2)
The auxiliary equation for (1) is 0 1/2 Re
D2 + 2D + 1 = 0 C

⇒ (D + 1)2 = 0
⇒ D = -1; -1
The general solution of Equation (1) is
y = (c1 + c2 t)e-t (3) where C is | z | = 1
Given y (0) = 1 2z + 5
Let f (z) =

From Equation (3),
y = 1 at t = 0  1 2
(
 z − 2  z − 4 z + 5 )

1 = (c1 + c2 × 0) e-0 The singularities of f (z)
⇒ c1 = 1 1
Also given y (1) = 3e-1 at y = 3e-1 at t = 1 are z = ;z = 2 ± i
2
From Equation (3),
3e-1 = (c1 + c2 × 1) e-1 1
of these, only z = lies inside C.
3e-1 = (1 + c2) e-1 2
3e-1 = e-1 + c2 e-1 2z + 5
\ I = ∫  dz
c2 = 2
Substituting the values of c1 and c2 in Equation (3),
C
1 2
(
 z − 2  z − 4 z + 5 )
We get ( 2 z + 5)
y = (1 + 2t) e-t (4) ( z 2 − 4 z + 5)
y (2) = yat t=2 = (1 + 2 × 2) e-2 = ∫  1
dz
C
 z −
= 5e-2
2 
Hence, the correct option is (B). 2z + 5
2pi ⋅ g (a), where g (z) = 2
Question Number: 15 Question Type: MCQ        z − 4z + 5
The value of the integral 1
2z + 5 and a =
∫  1  2 dz 2
(
 z − 2  z − 4 z + 5 ) (By Cauchy’s Integral Formula)
24 48πi
Over the contour | z | = 1, taken in the anti-clockwise = 2πi × =
13 13
direction, would be
24 πi 48πi Hence, the correct option is (B).
=
(A) (B) =
13 13 Question Number: 16 Question Type: MCQ
24 12 Y ( s) s
(C) (D) The transfer function of a system is = the
13 13 R( s) S + 2
Solution: steady-state output y(t) is A cos (2t + f) for the input
2z + 5 cos (2t) the values of A and f, respectively are:
Let I = ∫  dz 1
C
(
1 2
)
 z − 2  z − 4 z + 5 (A)
2
, −45°

F02_Gate_EE_2016_978-93-325-7605-6_P-I.indd 86 6/19/2017 7:10:53 PM


GATE 2016 Solved Paper Electrical Engineering: Set – 2 | lxxxvii

1 y1 (t) = ay (t) and x3 (t) = x1 (t) cos t


(B) , +45° y3 (t) = [x1 (t) x2 (t)]
2
= x1 (t) cos t + x2 (t) cos t
(C) 2 , −45° y3 (t) = y1 (t) + y2 (t)
(D) 2 , +45° Thus, the given system is Linear system y (t) = x (t) cos t
y1 (t) x1 (t) cos t
Solution:
y2 (t) x2 (t) cos t
s
x2 (t) = x1 (t - t0) cos t
R(s) V(s)
s+2
y2 (t) = x1 (t - t0) cos t
y1 (t - t0) = x1 (t - t0) cos (t - t0)
Now we have y2 (t) = y1 (t - t0)
Y (t) = A cos (2t + f  ) Thus given system is time-variant system.
and Hence, the correct option is (C).
Y ( s) s
A = = Question Number: 19 Question Type: MCQ
R( s) s + 2 ω=2 +∞ − t

2 1
The value of ∫ −∞
e δ( 2t − 2)dt , where d (t) is the

= =
8 2 Dirac delta function, is:

1 2
 ω (A) (B)
and f = 90 − tan −1   = 45° 2e e
 2 ω=2
1 1
(C) 2 (D)
Hence, the correct option is (B). e 2e 2
Question Number: 17 Question Type: NAT Solution:
The phase cross-over frequency of the transfer function   b  1
δ a  t −   = δ(t − b/a)
G(S) =
100
in rad/s is.   a  |a|

( S + 1)3 ∞
1−t
Solution:  Phase of system = -180° ∫e ⋅
⇒ 
2
⋅ δ (t − 1) ⋅ dt
−∞
-3 tan-1 (W) = -180°
1 1
W = tan 60 = {e − t }t =1 =
2 2e
= tan 60 = 3
Hence, the correct option is (A).
Hence, the correct Answer is ( 3 ).
Question Number: 20 Question Type: MCQ
Question Number: 18 Question Type: MCQ A temperature in the range of −40°C to 55°C is to be
Consider a continuous-time system with input x (t) and measured with a resolution of 0.1°C. The minimum
output y (t) given by y (t) = x (t) cos (t). This system is: number of ADC bits required to get a matching dynamic
(A) linear and time-invariant range of the temperature sensor is:
(B) non-linear and time-invariant (A) 8 (B) 10
(C) 12 (D) 14
(C) linear and time-varying
(D) non-linear and time-varying Solution:  Resolution can be calculated using relation
Analog output
Solution:     y  (t) = x (t) cos t R =
No. of steps
x1 (t) = ax (t) cos t
y1 (t) = x1 (t) cos t 55 − ( −40)
=
0.1°C
= ax1 (t) cos t 2n − 1

F02_Gate_EE_2016_978-93-325-7605-6_P-I.indd 87 6/19/2017 7:10:58 PM


lxxxviii | GATE 2016 Solved Paper Electrical Engineering: Set – 2

95 Solution:  Consider the figure given below:


2n - 1 =
0.1
+10 V
2n - 1 = 950
2n = 951 I1 220Ω
n = 10 4.7 K

Hence, the correct option is (B).


Question Number: 21 Question Type: MCQ
+
Consider the following circuit which uses a 2-to-1 0.5 mA 0.6V
multiplexer as shown in the figure below. The Boolean + +
expression for output F in terms of A and B is: 5.3 V 470Ω
− −
0
IE
Y F

1 S

Zener diode breakdown voltage = 5.3 V


A
B 10 − 5.3
I1 = = 1 mA
4.7 KΩ
A+ B
(A) A ⊕ B (B)
The base current will be
(C) A + B (D) A⊕ B IB = I1 - 0.5 mA = 0.5 mA
Solution: The Boolean expression for output F in Base to emitter voltage drop = 0.6 V, therefore
terms of A and B is
5.3 V - 0.6 - 470IE = 0
F = S ⋅ I 0 + S ⋅ I1
470IE = 4.7 V
= B⋅ A+ B⋅ A
The emitter current can be calculates as
= A B = A⊕ B
4.7
IE = A
Hence, the correct option is (D). 470
IE = 10 mA
Question Number: 22 Question Type: NAT
The current gain can be calculated using the relation
A transistor circuit is given below. The Zener diode
breakdown voltage is 5.3 V as shown. Take base IE = (1 + b) · 0.5 mA
to emitter voltage drop to be 0.6 V. The value of the (1 + b) = 20
current gain b is _________. b = 19
Hence, the correct Answer is (18.0 to 20.0).
10 V
Question Number: 23 Question Type: MCQ
4.7 kΩ In cylindrical coordinate system, the potential produced
220Ω by a uniform ring charge is given by j = f (r, z), where f

is continuous function of r and z. Let E be the resulting

5 electric field. Then the magnitude of ∇ × E.
(A) increase with r (B) is 0
(C) is 3 (D) decrease with z
5.3 V 470Ω
Solution: We know that potential F and the electric

field E are functions of r, and z alone and they are
independent of time.

F02_Gate_EE_2016_978-93-325-7605-6_P-I.indd 88 6/19/2017 7:11:01 PM


GATE 2016 Solved Paper Electrical Engineering: Set – 2 | lxxxix

Now from Maxwell’s equation ∇ for static e-fields, Which one of the following statements is TRUE?
(A) RA = RB (B) RA = RB = 0
∇ × E = 0
(C) RA < RB (D) RB = RA/(1 + RA)
Hence, the correct option is (B).
Solution:
Question Number: 24 Question Type: NAT RA
A soft-iron toroid is concentric with a long straight RA = +2
1 + RA
conductor carrying a direct current I. If the relative
permeability mr of soft-iron is 100, the ratio of the RA2 - 2RA - 2 = 0
magnetic flux densities at two adjacent points located RA = 1 ± 3
just inside and just outside the toroid, is ________. \ RA = 2.73W
Solution:  Consider the figure given below in which a
is the radius of toroid.
R 1Ω RA

From the above figure we conclude that RA and 1 ohm


I resistance are in parallel with each other and their
a
equivalent resistance is RB, therefore
RA
RB =
1 + RA
Hence, the correct option is (D).
Let M be the flux density, magnetic field B of a toroid Question Number: 26 Question Type: MCQ
which is concentric with a conductor (N = 1) can be
calculated using In a constant V/f induction motor drive, the slip at the
maximum torque:
µNI Bin
B = (A) is directly proportional to the synchronous
2πρ Bout speed.
= mr = 100 (B) remains constant with respect to the
synchronous speed.
Hence, the correct Answer is (100).
(C) has an inverse relation with the synchronous
Question Number: 25 Question Type: MCQ speed.
RA and RB are the input resistances of circuits as shown (D) has no relation with the synchronous speed.
below. The circuits extend infinitely in the direction Solution: At maximum to square
shown. r
S = 2
XL
2Ω 2Ω 2Ω
2
V   3  v2
RA Tmax a   ; Tmax =  
1Ω 1Ω 1Ω  f  ω s  2 x2

We know that, wsaf, X2af
1
2Ω 2Ω 2Ω ⇒ Sα
ωs
RB
1Ω 1Ω 1Ω 1Ω Thus, the slip at the maximum torque has an inverse
relation with the synchronous speed.
Hence, the correct option is (C).

F02_Gate_EE_2016_978-93-325-7605-6_P-I.indd 89 6/19/2017 7:11:04 PM


xc | GATE 2016 Solved Paper Electrical Engineering: Set – 2

Question Number: 27 Question Type: NAT and the current supplied by the battery, in ampere can
In the portion of a circuit shown, if the heat generated be calculated using relation given below:
in 5W resistance is 10 calories per second, then heat I1 = I2 + I2 = 2I2
generated by the 4W resistance, in calories per second, V 
= 2  x  = Vx
is ________. 2

4Ω 6Ω 1 = I1 (1) + Vx
⇒ 2v = 1
⇒ Vx = 0.5 V
I1 = 0.5 A
5Ω Hence, the correct Answer is (0.5 A).
Question Number: 29 Question Type: MCQ
Solution: If I is the total current entering into the circuit
In a 100 bus power system, there are 10 generators.
10
I5 = I × = 2i/3 In a particular iteration of Newton Raphson load flow
15 technique (in polar coordinates), two of the PV buses
5 are converted to PQ type. In this iteration:
I4 = I 6 = I × = I /3
15 (A) the number of unknown voltage angles
increases by two and the number of unknown
We know that
3
voltage magnitudes increases by two.
 2I  (B) the number of unknown voltage angles
 3  × 5 = 10 remains unchanged and the number of

unknown voltage magnitudes increases by
I = 3 two.
2
(C) the number of unknown voltage angles
2
 3   increases by two and the number of unknown
  
2  voltage magnitudes decreases by two.
Heat generated in 4W =   ×4 (D) the number of unknown voltage angles
 3 
  remains unchanged and the number of
unknown voltage magnitudes decreases by
1 two.
= × 4 = 2 cal/sec
2 Solution:  Load buses = Total buses - Gen. buses
Hence, the correct Answer is (2). = 100 - 10
= 90
Question Number: 28 Question Type: NAT
We know that at every load bus → | v |, d must be
In the given circuit, the current supplied by the battery,
calculated and at every gent bus → Q, d must be
in ampere, is ________.
calculated, i.e., at all buses, (d) must be calculated. So,
voltage phase angle calculations will not change during
Vx
iteration, two of the gen. buses changes to load buses.
I1 I2
1Ω 1Ω Total load buses = 92.
1V 1Ω Therefore, voltage magnitude calculation will increase
I2 by two.
Hence, the correct option is (B).
Question Number: 30 Question Type: NAT
Solution:  From the given figure in problem
The magnitude of three-phase fault currents at buses
Vx
I2 = A and B of a power system are 10 pu and 8 pu,
2 respectively. Neglect all resistances in the system and

F02_Gate_EE_2016_978-93-325-7605-6_P-I.indd 90 6/19/2017 7:11:06 PM


GATE 2016 Solved Paper Electrical Engineering: Set – 2 | xci

consider the pre-fault system to be unloaded. The pre- From Equation (2)
fault voltage at all buses in the system is 1.0 pu. The 1 1
voltage magnitude at bus B during a three-phase fault =
8 + (4)
zB 4 zB + z A
at bus A is 0.8 pu. The voltage magnitude at bus A
during a three-phase fault at bus B, in pu, is ________. 32zB2 + 8zAzB = 5zB + zA
Solution:  Consider the figure given below: From Equation (1)
50zA zB + zA
I1
From Equation (3) and (4)
~ 1Pu ~ 1Pu zA = 0.76zB
ZA If bus (A) voltage is VA
ZB
1 1
=
8 +
Z zB z + z A
A B
1 1
= +
zB 4 zB + z A

zB = 0.151
Figure 1
⇒ zA = 114, z = 0.604
For fault at bus ‘A’ From Figure 2
1 1
=
10 + (1) 1 VA
z A z + zB =
8 +
zB z
Voltage magnitude at bus B is 0.8 Pu, therefore VA
1
0.8 1 ⇒ 8 = +
\ I2 = = 0.151 0.604
z z + zB
VA = 0.832 pu
⇒ 0.8 (z + zB) = z
Hence, the correct Answer is (0.832).
⇒ z = 4zB (2)
Question Number: 31 Question Type: MCQ
Consider a system consisting of a synchronous gen-
I1 erator working at a lagging power factor, a synchro-
~ 1.0PU ~ 1.0PU
nous motor working at an overexcited condition and a
directly grid-connected induction generator. Consider
ZA ZB
capacitive VAr to be a source and inductive VAr to be
a sink of reactive power. Which one of the following
Z statements is TRUE?
A I2 B (A) Synchronous motor and synchronous genera-
tor are sources and induction generator is a
sink of reactive power.
(B) Synchronous motor and induction generator
Figure 2 are sources and synchronous generator is a
sink of reactive power.
For fault at bus B:
(C) Synchronous motor is a source and induction
8 = Ix + Iy
generator and synchronous generator are
1 1 sinks of reactive power.
8 = + (3)
zB z + z A (D) All are sources of reactive power.

F02_Gate_EE_2016_978-93-325-7605-6_P-I.indd 91 6/19/2017 7:11:08 PM


xcii | GATE 2016 Solved Paper Electrical Engineering: Set – 2

Solution: Solution:  TON period:


VL = Vg - Vo = 30 V (1)
Q1
TOFF period:
Q2 VL = -Vo­ = -20 V (2)
Q3
G
Solving Equations (1) and (2), we get
~ ~ M ~ I.G Vg - 20 = 30
Vg = 30 + 20 = 50 V
and
Vo = DVg
Form the figure we can observe that given synchronous 20 = D (50)
generator operates at lagging pf, i.e., an inductive load 2
should be connected so, machine will generate reac- D = = 0.4
5
tive power to lagging load. Synchronous motor is over
excited, i.e., it is acting like a Synchronous condenser. Hence, the correct Answer is (0.4).
Induction generator will absorb reactive power of the Question Number: 33 Question Type: NAT
development of magnetic field.
A steady dc current of 100 A is flowing through a
Hence, the correct option is (A). power module (S, D) as shown in figure (a). The V-I
Question Number: 32 Question Type: NAT characteristics of the IGBT (S) and the diode (D)
are shown in figures (b) and (c), respectively, y. The
A buck converter, as shown in figure (a) below, is working
conduction power loss in the power module (S, D), in
in steady-state. The output voltage and the inductor
watts, is ________.
current can be assumed to be ripple free figure (b)
shows the indicator voltage VL during a complete IS(A)
switching interval. Assuming all devices are ideal, the
duty cycle of the buck converter is ________.
M L
+ VL – +

dV/dI = 0.02Ω
C R
D VC
S Vo = 1 V
D


100 A VS(Volt)
(a)
VL V-I characteristic of IGBT
(a) (b)
30 V

ID(A)

T ON T OFF

0 dV/dI = 0.01Ω
t
Vo = 0.7 V

VD(Volt)
–20 V Ts V-I characteristic of diode
(b) (c)

F02_Gate_EE_2016_978-93-325-7605-6_P-I.indd 92 6/19/2017 7:11:10 PM


GATE 2016 Solved Paper Electrical Engineering: Set – 2 | xciii

Solution:  For diode to be conducting Solution: Number of poles


dv P = 4, A = 4
= 0.01
dI Steady current Ia = 40 A
V = 0.01 × 100 = 1 V Speed N = 600 rpm
Conduction losses will be Now we have
= 1 × 100 + 0.7 × 100 I 40
= 100 + 70 = 170 W i = a = = 10 A
A 4
Hence, the correct Answer is (170 W).
Question Number: 34 Question Type: MCQ S
A 4-pole, lap-connected, separately excited dc motor
is drawing a steady current of 40 A while running at
600 rpm. A good approximation for the wave shape N ⊗ N
of the current in an armature conductor of the motor
is given by:

S i = 10 A
out of plane
I
10 A
(A) Case (i)
t
S

40 A
I N N
(B)

t
S
i=0

I Case (ii)
10 A
(C)
T = 25 ms S
t
X
T = 25 ms
–10 A
N N

I
10 A S

(D) T = 25 ms
t Case (iii)
T = 25 ms i = 10 A
–10 A Hence, the correct option is (C).

F02_Gate_EE_2016_978-93-325-7605-6_P-I.indd 93 6/19/2017 7:11:12 PM


xciv | GATE 2016 Solved Paper Electrical Engineering: Set – 2

Question Number: 35 Question Type: MCQ Given S = 2a


If an ideal transformer has an inductive load element ∞

at port 2 as shown in the figure below, the equivalent ⇒ ∑ nα n = 2a


n= 0
inductance at port 1 is:
⇒ 1 + a + 2a2 + 3a3 + ... + ∞ = 2a
n:1 α
⇒ = 2a
(1 − α ) 2
L
(\ 1 + a + 2a2 + 3a3 + ... +∞ is an AGP with a = 1;
r = a and d = 1)
Port 1 Port 2 1
⇒ = 2
(1 − α ) 2
n2L
(A) nL (B)
1
n2/L
(C) n/L (D) ⇒ (1 - a)2 =
2
Solution:  If the load element was shifted to primary 1
side, the effective equivalent inductance will be ⇒ 1 - a = ±
2
Lprim = n2L.
Hence, the correct option is (B). 1 −1
⇒ 1 - a = ,1 − α =
2 2
Question Number: 36 Question Type: NAT
1 1
Candidates were asked to come to an interview with ⇒ a = 1 − ,α = 1 +
3 pens each. Black, blue, green and red were the 2 2
permitted pen colours that the candidate could bring. ⇒ a = 1.7071; a = 0.2929
The probability that a candidate comes with all 3 pens ⇒ a = 0.2929 (∵ | a | < 1)
having the same colour is ________.
Hence, the correct Answer is (0.2929).
Solution:  All 3 pens are same colour = 4 ways.
Two pens are same colour and third pen different colour Question Number: 38 Question Type: MCQ
= 4 × 3 = 12. Let the eigenvalues of a 2 × 2 matrix A be 1, -2 with
All three are of different colour = 4C3 = 4. eigenvalues and eigenvectors x1 and x2, respectively.
Then the eigenvectors of the matrix A2 - 3A + 4I would
\ Total number of ways of selecting three pens from respectively be:
four colours pens = 4 + 12 + 4 = 20. (A) 2, 14; x1, x2
Favourable cases = 4. (B) 2, 14; x1 + x2, x1 - x2
4 1 (C) 2, 0; x1, x2
\ Required probability = = = 0.2
20 5 (D) 2, 0; x1 + x2, x1 - x2
Hence, the correct Answer is (0.2). Solution:  Given that A is a 2 × 2 matrix.
And the eigenvalues of A are 1, -2.
Question Number: 37 Question Type: NAT
∞ Let l1 = 1 and l2 = 2.
Let S = ∑ nα n , where | a | < 1. The value of a in the The eigenvectors of A are x1 and x2 corresponding to
n= 0
the eigenvalues l1 = 1 and l2 = 2, respectively.
range 0 < a < 1, such that S = 2a is ________.
\ The eigenvalues of A2 - 3A + 4I are
Solution:
λ12 − 3λ1 + 4 and λ 22 − 3λ 2 + 4

Given S = ∑ nα n ; where | a | < 1 i.e., 12 - 3 × 1 + 4 and (-2)2 - 3 (-2) + 4 = 2 and 14.
n= 0

F02_Gate_EE_2016_978-93-325-7605-6_P-I.indd 94 6/19/2017 7:11:15 PM


GATE 2016 Solved Paper Electrical Engineering: Set – 2 | xcv

Also, we know that, A and a matrix polynomial f (A) 2S


will have the same eigenvectors. (A)
(1 + 0.5S )(1 + 0.25S ) 2
\ The eigenvalues and the corresponding eigenvectors
of A2 - 3A + 4I are 2, 14, x1 and x2. 4(1 + 0.5S )
(B)
Hence, the correct option is (A). S (1 + 0.25S )
2S
Question Number: 39 Question Type: MCQ (C)
(1 + 2S )(1 + 4 S )
Let A be a 4 × 3 real matrix with rank 2. Which one of
the following statement is TRUE? 4S
(D)
(A) Rank of ATA is less than 2. (1 + 2S )(1 + 4 S ) 2
(B) Rank of ATA is equal to 2. Solution:  Using graph given in problem we observe
(C) Rank of ATA is greater than 2. +20 dB/dec slope and 0 dB/dec and -40 dB/dec, which
(D) Rank of ATA can be any number between say that there exist ‘1’ zero and ‘3’ poles.
1 and 3. KS
G(s) H(s) =
Solution: Given that A is a 4 × 3 real matrix with (1 + ST1 )(1 + ST1 ) 2

rank 2 i.e.,
|1G(s) H(s)| = 20 log k + 20 log w|w = 0.5
r (A) = 2 (1)
T = 0 dB
We know that r (A) = r (A ) = 2 2S
⇒ k = 2
we have r (ATA) ≤ min. [r (AT), r (A)] (2) (1 + ST1 )(1 + ST2 ) 2
[∵ r (AB) ≤ min. [r (A), r (B)] = G (s) H (s)
\ r (ATA) ≤ 2 (3) 0 dB/dec slope occurs at w1 which should be greater
Also, ATA is a 3 × 3 real symmetric matrix with 2 than 0.5 and -10 dB/dec slope occurs at w2, which
linearly independent rows/columns should be less than 8.
r (ATA) ≥ 2 (4) Hence, the correct option is (A).
From Equations (3) and (4), Question Number: 41 Question Type: NAT
r (ATA) = 2 Consider the following state-space representation of a
Hence, the correct option is (B). linear time-invariant system.
1 0  1
Question Number: 40 Question Type: MCQ x (t) =  x(t ), y (t) = cT x (t), c =   and x (0)
 0 2  1
Consider the following asymptotic Bode magnitude        
plot (w is in rad/s). 1
=   the value of y (t) for t = loge2 is ________.
1  
Solution:  We know that
Magnitude (dB)

Y(s) = C (SI - A)-1B + D


L −1
  s 0  1 0   1
12 dB = [1 1]   − 
  0 s   0 2   1
–40 dB/dec
0 dB −1
s − 1 0  1
ω = [1 1] 
0.5 8
 0 s − 2  1
1 1
Y(s) = + = et + e 2t
s −1 s − 2
Which one of the following transfer functions is best
represented by the above Bode magnitude plot? For t = loge2,

F02_Gate_EE_2016_978-93-325-7605-6_P-I.indd 95 6/19/2017 7:11:18 PM


xcvi | GATE 2016 Solved Paper Electrical Engineering: Set – 2

2 2 Put (s - 1) in place of s in the Equation (1)


Y (  f  ) = e loge + e 2 loge
(s - 1)3 + 5.5 (s - 1)2 + 8.5 (s - 1) + 3 = 0
= 2 + 22 = 6
(s - 1)2 (s + 4.5) + 8.5s - 5.5 = 0
Hence, the correct Answer is (6).
[s2 + 1 - 2s] [s + 4.5] + 8.5s - 5.5 = 0
Question Number: 42 Question Type: MCQ s3 + 4.5s2 + s + 4.5 - 2s2 - 9s + 8.5s - 5.5 = 0
Loop transfer function of a feedback system is s3 + 2.5s2 + 0.5s - 1 = 0
s+3
G (s) H (s) = 3 . Take the Nyquist contour in As per RH criteria
    s ( s − 3)
the clockwise direction. Then, the Nyquist plot of G (s) s3 1 0.5
H (s) encircles -1 + j0. 2
s 2.5 –1
(A) once in clockwise direction
1
(B) twice in clockwise direction s 1.25 + 1
(C) once in anticlockwise direction 2.5
(D) twice in anticlockwise direction
s0 –1
Solution:  Loop transfer function
s+3
G (s) H (s) = 3
    s ( s − 3) One sign change
RH criteria, And 2 roots on left of s = -1
s3 - 3s2 + s + 3 = 0 Hence, the correct Answer is (2).
s3 1 1 Question Number: 44 Question Type: MCQ
s2 –3 3 Suppose x1 (t) and x2 (t) have the Fourier transforms as
shown below:
s1 2

s0
3 X1(jω)

2 sign changes
1
2 poles lie right half of s - plane
No. of Encirclement (N) = z - p 0.5
= 2 - 1 = 1 (+ve)
0.3
Hence, one clockwise encirclement.
Hence, the correct option is (A). –1 0 1 2 ω

Question Number: 43 Question Type: NAT


Given the following polynomial equation S 3 + 5.5S 2
X1(jω)
+ 8.5S + 3 = 0, the number of roots of the polynomial,
which have real parts strictly less than -1 is ________.
Solution:  Given polynomial equation is
0.5
S3 + 5.5S2 + 8.5S + 3 = 0 (1)
1

0.3
s = –1
–2 –1 0 1 ω

F02_Gate_EE_2016_978-93-325-7605-6_P-I.indd 96 6/19/2017 7:11:20 PM


GATE 2016 Solved Paper Electrical Engineering: Set – 2 | xcvii

Which one of the following statements is TRUE? (A) 00 (B) 01


(A) x1 (t) and x2 (t) are complex and x1 (t) x2 (t) is (C) 11 (D) 10
also complex with nonzero imaginary part. Solution:  Consider the table given below:
(B) x1 (t) and x2 (t) are real and x1 (t) x2 (t) is also
real. CLK JA KA JB KB QA QB
(C) x1 (t) and x2 (t) are complex but x1 (t) x2 (t) is
real. - 1 1 - - 0 0
(D) x1 (t) and x2 (t) are imaginary but x1 (t) x2 (t) is 1 1 1 1 1 1 1
real.
Solution:  Using the figures for fourier transforms in → 11
00
question we get \ The next state of the QAQB is 11.
x1 (  jw) = x1 (−jw) Hence, the correct option is (C).
Hence, the correct option is (C).
Question Number: 47 Question Type: MCQ
Question Number: 45 Question Type: MCQ A 2-bit flash Analog to Digital Converter (ADC)
The output of a continuous-time, linear time-invariant is given below. The input is 0 ≤ VIN ≤ 3 Volts. The
system is denoted by T {x (t)} where x (t) is the input expression for the LSB of the output B0 as a Boolean
signal. A signal z (t) is called eigen-signal of the system function of X2, X1, and X0 is:
T, when T {x(t)} = xz (t), where x (t) is a complex 3V
number, in general, and is called an eigenvalue of T.
Suppose the impulse response of the system T is real
100Ω
and even. Which of the following statements is TRUE?
(A) cos (t) is an eigen-signal but sin (t) is not. + X2
(B) cos (t) and sin (t) are both eigen-signals but –
200Ω
with different eigenvalues. B1
(C) sin (t) is an eigen-signal but cos (t) is not.
+ X1 B0
(D) cos (t) and sin (t) are both eigen-signals but
300Ω –
with identical eigenvalues.
Solution:
+ X0
Hence, the correct option is (D). 100Ω

Question Number: 46 Question Type: MCQ
The current state QAQB of a two JK flip-flop system VIN
is 00. Assume that the clock rise-time is much smaller
than the delay of the JK flip-flop. The next state of the (A) X 0 [ X 2 ⊕ X1 ] (B)
X 0 [ X 2 ⊕ X1 ]
system is ________.
(C) X 0 [ X 2 ⊕ X1 ] (D)
X 0 [ X 2 ⊕ X1 ]
5
QA QB Solution:  Consider the table given below:
QA
J J
X2 X1 X0 B1 B0
0 0 0 0 0

QA
K K 0 0 1 0 1
0 1 1 1 0
CLK 1 1 1 1 1

F02_Gate_EE_2016_978-93-325-7605-6_P-I.indd 97 6/19/2017 7:11:22 PM


xcviii | GATE 2016 Solved Paper Electrical Engineering: Set – 2

Solution:  When time t < 0, the current across inductor


B0 = X 2 X1 X 0 + X 2 X1 X 0
will be
B0 = X 0 ( X1  X 2 ) 3
IL = = 1.5 A
B0 = X 0 ( X 2 ⊕ X1 ) 2
When time t > 0, under steady-state
Hence, the correct option is (A).
 3 3
Question Number: 48 Question Type: MCQ ILf =  +  = 4.5 A
 1 2
Two electric charges z and −2z are placed at (0, 0) −t
and (6, 0) on the x-y plane. The equation of the zero IL = 4.5 + (1.5 − 4.5)e
( 32)
equipotential curve in the x-y plane is ________.
IL( f ) = 4.5 - 3e-2t/3
(A) x = -2 (B) y=2 +
t t = 0 , the rate of change of current through the
A
2 2
(C) x + y = 2 (D) (x + 2)2 + y2 = 16 inductor, in amperes per second
Solution:  x = -2 and y = 2, represent the planes dI L( t )  −2  −2( t 3 )
= −3   e
dt t =0  3

y = 2 A/s
Hence, the correct Answer is (2).
Question Number: 50 Question Type: NAT
0. P A three-phase cable is supplying 800 kW and
(0, 2) –24
+ 600 kVAr to an inductive load. It is intended to supply
–6 2 x 6
an additional resistive load of 100 kW through the
same cable without increasing the heat dissipation in
the cable, by providing a three-phase bank of capacitors
connected in star across the load. Given the line voltage
is 3.3 kV, 50 Hz, the capacitance per phase of the bank,
expressed in microfarads, is ________.
Solution: Power
The potential at point P will be, P = 800 kW
9 × 109 × 4 2 g × 9 × 109 Quality factor Q = 600 kVAr
V = − =0
2 4 Voltage VL = 3.3 kV
Hence, the correct option is (D). For the given load,
Q
Question Number: 49 Question Type: NAT tan q = = 0.75
P
In the circuit shown, switch S2 has been closed for a
long time. At time t = 0 switch S1 is closed. At t = 0+, ⇒ cos f = 0.8 lag
the rate of change of current through the inductor, in Current flowing through cable can be calculated using
amperes per second, is ________. relation
P = 3VL I L cos φ
S1 1Ω S2
800 × 103
IL =
3 × 3.3 × 103 × 0.8
2Ω
3V
1H
= 174.95 A

3V
Now, new load of 100 kW is added if the heating in
cable should be same as before, the current should not
alter IL = 174.95 A.

F02_Gate_EE_2016_978-93-325-7605-6_P-I.indd 98 6/19/2017 7:11:25 PM


GATE 2016 Solved Paper Electrical Engineering: Set – 2 | xcix

But power P = 900 kW, therefore (0.15)(0.05 + 0.3 + 3 zn )


P = 3VL I L cos φ = 900 × 103 0.15 +
0.15 + 0.05 + 0.3 + 3 zn
900 × 103 1
cos f = =
3 × 3.3 × 103 × 174.95 3.4
= 0.90 ⇒ zn = 1.08
The new power factor requirement is 0.9 lag Hence, the correct answer is 1.08.
⇒ sin f = 0.435
The new reactive power from supply should be Question Number: 52 Question Type: MCQ
Q3 = 3 × 3.3 × 103 × 174.95 × 0.435 If the star side of the star-delta transformer shown in
= 434.98 kVAr the figure is excited by a negative sequence voltage,
But load requires 600 kVAr. then
So, the capacitor bank must supply A a
QC = 600 - 434.98
= 165 kVAr B N
165 b
Each phase = = 55 kVAr
3 C c
2
V ph
= 55 kVAr (A) VAB leads Vab by 60°
Xc (B) VAB lags Vab by 60°

⇒ Xc = 65.99 (C) VAB leads Vab by 30°
⇒ C = 48.23 mF (D) VAB lags Vab by 30°
Hence, the correct Answer is (48.23).
Solution:  Negative sequence on primary:
Question Number: 51 Question Type: MCQ
A 30 MVA, 3-phase, 50 Hz, 13.8 kV, star-connected VAB
synchronous generator has positive, negative and zero
VA
sequence reactances, 15%, 15% and 5%, respectively.
A reactance (Xn) is connected between the neutral of –VB
the generator and ground. A double line to ground 30°
fault takes place involving phases ‘b’ and ‘c’, with a
fault impedance of j0.1 p.u. The value of Xn (in p.u.)
that will limit the positive sequence generator current
to 4270 A is ________. VC
Solution: VB

30 × 106
Ibare = n secondary side:
3 * 13.8 × 103
Ia (p.u.) = 4270 A = 1255.1 A φ
1
4270 A
Ia1 (p.u.) = = 3.4 p.u.
1255 30°

Ef
w.r.t. Ia1 =
z 2 ( z0 + 3 z f + 3 z n ) Va =
z1 + VAB
z 2 + z0 + 3 z f + 3 z n
z 2 ( z0 + 3 z f + 3 z n ) 1.0 VAB lags Vab by 30°.
z1 + =
z 2 + z0 + 3 z f + 3 z n 3.4
Hence, the correct option is (D).

F02_Gate_EE_2016_978-93-325-7605-6_P-I.indd 99 6/19/2017 7:11:28 PM


c | GATE 2016 Solved Paper Electrical Engineering: Set – 2

Question Number: 53 Question Type: MCQ Solution:


A single-phase thyristor-bridge rectifier is fed from a Vdc
(V01)max = ( M .I .)
230 V, 50 Hz, single-phase AC mains. If it is delivering 2
a constant DC current of 10 A, at firing angle of 30°, = 0.8 × 250 = 200 V
then value of the power factor at AC mains is:
At fundamental frequency, the reactance will be
(A) 0.87 (B) 0.9
(C) 0.78 (D) 0.45 z = R 2 + X 2 = 122 + 16 2
Solution:  We know that supply power factor = 20W

= (Distortion factor) × (cos f1)
(V01 ) max 200
2 2 (I01)max = =
= × cos 30 = 0.7796  0.78 |z| 20
Π
Hence, the correct option is (C). = 10 A
Hence, the correct Answer is (10).
Question Number: 54 Question Type: NAT
The switches T1 and T2 in figure (a) are switched Question Number: 55 Question Type: NAT
in a complementary fashion with sinusoidal pulse The voltage (Vs) across and the current (Is) through
width modulation technique. The modulating voltage a semiconductor switch during a turn-ON transition
nm (t) = 0.8 sin (200pt) V and the triangular carrier voltage are shown in figure. The energy sissipated during the
(nc) are as shown in figure (b). The carrier frequency is turn-ON transition, in mJ is ________.
5 kHz. The peak value of the 100 Hz component of the
load current (iL), in ampere, is ________.
+
vs 600 V
Vdc/2 = 250 V
T1
0 t
IL
50 A
– is

+ XL = 16Ω at 100 A
R = 12Ω
100 Hz T2 0
t
T1 = 1µ T2 = 1µ

Vdc/2 = 250 V
Solution:
1µ sec 1µ sec

(a)
E = ∫ v1 (t )i1 (t )dt + ∫ v2 (t )i2 (t )dt
0 0
νc 1µ sec
+ ∫ v2 (t )i2 (t )dt
0
1
0.8 600 × 150 10 −12
E = × + 600 × 100 × 10 −6
10 −6 2
t
600 × 100 10 −12
− ×
10 −6 2
νm = 45 + 60 - 30 = 75 mJ
Hence, the correct Answer is (75).
(b)

F02_Gate_EE_2016_978-93-325-7605-6_P-I.indd 100 6/19/2017 7:11:31 PM


GATE 2016 Solved Paper Electrical Engineering: Set – 2 | ci

Question Number: 56 Question Type: NAT IL


A single-phase 400 V, 50 Hz transformer has an iron If Ia
loss of 5000 W at the rated condition. When operated
at 200 V, 25 Hz, the iron loss is 2000 W. When operated
at 416 V, 52 Hz, the value of the hysteretic loss divided
by the eddy current loss is ________. rf Vt

Solution:
V1 400
= =8
f1 50

v
= constant Hence, the correct Answer is (86.84).
f
Question Number: 58 Question Type: NAT
V2 200
= =8 A three-phase, 50 Hz salient-pole synchronous motor
f2 25
has a per-phase direct-axis reactance (Xd) of 0.8 pu and
We af 2 a per-phase quadrature-axis reactance (Xq) of 0.6 pu.
Whaf Resistance of the machine is negligible. It is drawing
full-load current at 0.8 pf (leading). When the terminal
Wi = Af + Bf 2
voltage is 1 pu, per-phase induced voltage, in pu, is
5000 = A (50) + B (50)2 ________.
2000 = A (25) + B (25)2 Solution:
4000 = A (50) + B2 (23)2 V sin ∅ + I a X a
tan y =
Wn = 60 (52) + 0.8 (52) V cos ∅
1000 = B (50)2 - B2 (25)2 y = 56.3°
1000 y = ∅ + d
B = 2
50 − 2( 25) 2 d = 56.3 - 36.86 = 19.7°
B = 0.8 E = V cos d + Id Xd
A = 60 Id = Ia sin ∅ = 0.831
60(52) E = (1) · cos (19.7) + (0.831) (0.8)
Ratio = = 1.44
(0.8)(52) 2 = 1.606 V
Hence, the correct Answer is (1.44). Hence, the correct Answer is (1.606).
Question Number: 57 Question Type: NAT Question Number: 59 Question Type: MCQ
A DC shunt generator delivers 45 A at a terminal A single-phase, 22 kVA, 2200 V/220 V, 50 Hz,
voltage of 220 V. The armature and the shunt field distribution transformer is to be connected as an
resistances are 0.01W and 44W, respectively. The stray auto-transformer to get an output voltage of 2420 V.
losses are 375W. The percentage efficiency of the DC Its maximum kVA rating as an auto-transformer is
generator is ________. ________.
Solution: (A) 22 (B) 24.2
Eg = 220 + (50) (0.01) (C) 242 (D) 2420
Eg · Ia = 220.5 × 50 Solution:
= 220.5 22 × 1000
I1 = = 10 A
= 11025 + 375 2200
Pin = 11025 + 375 = 11400 22 × 10000
I2 = = 100 A
h = 86.84% 220

F02_Gate_EE_2016_978-93-325-7605-6_P-I.indd 101 6/19/2017 7:11:33 PM


cii | GATE 2016 Solved Paper Electrical Engineering: Set – 2

An auto transformer will give maximum VA rating if it that of the inductance per conductor of the single-phase
is operated with a voltage ratio near to unity. system. The distance D, in meters, is ________.
Two connections are possible: Solution:  When a 1-ph system is available:
(1) 2200/2420 V (2) 220/2420 V
But connection (1) is suitable for maximum VA rating
I1
1m
220
 D 
L1-ph = 2 × 10 −7 ln   H/m
(I1 + I2)  D3 
2420
D → 1 mt
2200 Ds → 0.7788r
 1 
L1-ph = 2 × 10 −7 ln   H/m
I2  0.7788 × 10 × 10 −3 
= 2 × 10-7 × 4.85 H/m
VA rating = (2420) (I2) = (2420) (100)
= 242 kVA Now a 3-ph system:
Hence, the correct option is (C).
Question Number: 60 Question Type: MCQ
D D
A single-phase full-bridge voltage source inverter (VSI)
is fed from a 300 V battery. A pulse of 120° duration
is used to trigger the appropriate devices in each half-
cycle. The rms value of the fundamental component of 1m
the output voltage, in volts, is:
(A) 234 (B) 245 (1.05) ( 2 × 10 −7
× 4.85 )
(C) 300 (D) 331  3 D12 ⋅ D13 ⋅ D23 
−7
Solution:  Fundamental output voltage, = 2 × 10 ln   hfont
 0.7788r 
4V nπ
v01 = dc sin nd × sin × sin nωt (1.05) (4.85)
nπ 2
3 D12 ⋅ D13 ⋅ D23
2 2Vdc =
(v01)rms = × sin d 0.7788r
π
D = 1.42 m
2 2 × 300 Hence, the correct Answer is (1.42).
= × sin 60 = 234 V
π
Question Number: 62 Question Type: NAT
Hence, the correct option is (A).
In the circuit shown below, the supply voltage is 10 sin
Question Number: 61 Question Type: NAT (1000t) volts. The peak value of the steady-state current
A single-phase transmission line has two conductors through the 1W resistor, in amperes, is ________.
each of 10 mm radius. These are fixed at a center-to- 2 µF
center distance of 1 m in a horizontal plane. This is 4Ω
now converted to a three-phase transmission line by
introducing a third conductor of the same radius. This 250 µF 500 mH
1Ω
conductor is fixed at an equal distance D from the two
single-phase conductors. The three-phase line is fully 5Ω
0.02 mH
transposed. The positive sequence inductance per
phase of the three-phase system is to be 5% more than

F02_Gate_EE_2016_978-93-325-7605-6_P-I.indd 102 6/19/2017 7:11:36 PM


GATE 2016 Solved Paper Electrical Engineering: Set – 2 | ciii

Solution:  For steady-state, for inductive and capaci- The resistance will be
tive element branches impedance becomes infinity, 1 2R
therefore they act as open circuit = 2
R R + 200
10 sin(1000t )
I = 2R2 = R2 + 200
4 +1+ 5
R2 - 200
I = 1 sin (1000t)
Ip = 1 Amp R = 14.14W
Hence, the correct Answer is (1). Hence, the correct Answer is (14.14).

Question Number: 63 Question Type: NAT Question Number: 65 Question Type: NAT
A dc voltage with ripple is given by n (t) = [100 + 10 In the circuit shown below, the node voltage VA is
sin w (t) − 5 sin {3w (t)}] volts. Measurements of this ________V.
voltage n (t), made by moving-coil and moving-iron
I1 5Ω
voltmeters, show readings of V1 and V2, respectively. A
The value of V1 - V2, in volts, is ________. •

Solution: 5Ω
Moving coil, V1 = 100 V 5Ω 5A 5Ω

10 2 52
Moving iron, V2 = 100 2 + + 10I1
10 V
2 2 +

= 100.312 V
V2 - V1 = 0.312 V
Hence, the correct Answer is (0.312). Solution:  Applying KVL at node A, we get
VA V + 10 I1 VA − 10
Question Number: 64 Question Type: NAT −5+ A +
5 5 10
The circuit below is excited by a sinusoidal source.
The value of R, in W, for which the admittance of the =0

circuit becomes a pure conductance at all frequencies VA − 10
is ________. I1 =
10
100 µF R VA V V − 10 VA − 10
−5+ A + A +
5 5 5 10
=0

0.02 H R 1 1 1 1 
VA =  + + + 
 5 5 5 10 
=5+2+1

Solution:  The impedance will be 8 8
VA = =
L  R  3 1 6 1
R 2 + + j  ωRL − + +
C  ωC  5 10 10 10
z = y
 1  80
2 R + j  ωL −
 ωC  =
7
volts

1 2R = 11.428 volts

= =
2 L Hence, the correct Answer is (11.428).
R2 +
C

F02_Gate_EE_2016_978-93-325-7605-6_P-I.indd 103 6/19/2017 7:11:39 PM


GATE 2016 Solved Paper
Electrical Engineering
Set - 2
Number of Questions: 65 Total Marks: 100.0

Wrong answer for MCQ will result in negative marks, (-1/3) for 1 Mark Questions and (-2/3) for 2 Marks
Questions.

General Aptitude
Number of Questions: 10 Section Marks: 15.0

Q. 1 to Q. 5 carry 1 mark each and Q. 6 to Q. 10 carry (B) R2D2 is the only robot which can repair
2 marks each aeroplane
(C) R2D2 is a robot which can repair only
Question Number: 1 Question Type: MCQ
aeroplane
The chairman requested the aggrieved shareholders to (D) Only R2D2 is a robot
_______ him.
Solution:  As per option (A), R2D2 cannot do any-
(A) bare with (B) bore with
thing to an aeroplane apart from repairing. This is not
(C) bear with (D) bare in the scope of the given statements.
Solution:  To bear with someone or something is to be As per option (C), R2D2 cannot repair any other thing
patient with somebody or something. except aeroplanes. This is also not in the scope of the
Hence, the correct option is (C). given statements.
Option (D) contradicts the given statements.
Question Number: 2 Question Type: MCQ
From the given statements it is clear that R2D2 is the
Identify the correct spelling out of the given options: only which can repair aeroplanes. Hence, option (B)
(A) Managable (B) Manageable can be inferred.
(C) Mangaeble (D) Managible
Hence, the correct option is (B).
Solution:
Question Number: 5 Question Type: MCQ
Hence, the correct option is (B). 2
If | 9y – 6 | = 3, then y – 4y/3 is ______.
Question Number: 3 Question Type: MCQ (A) 0 (B) +1/3
Pick the odd one out in the following: (C) –1/3 (D) undefined
13, 23, 33, 43, 53 Solution:  We are given that
(A) 23 (B) 33
| 9y – 6 | = 3
(C) 43 (D) 53 1
9y – 6 = –3 ⇒ y =
Solution:  We know that except 33, rest are all prime 3
numbers. 9y – 6 = 3 ⇒ y = 1
Hence, the correct option is (B). 4 y 1 4 −1 4 −1
\ y2 – = − = or 1 − =
Question Number: 4 Question Type: MCQ 3 9 9 3 3 3
\ In either case,
R2D2 is a robot, R2D2 can repair aeroplanes. No other
4 y −1
robot can repair aeroplanes. y2 – =
(A) R2D2 is a robot which can only repair 3 3
aeroplanes Hence, the correct option is (C).

F03_Gate_EE_2016_978-93-325-7605-6_P-II.indd 104 6/19/2017 7:13:26 PM


GATE 2016 Solved Paper Electrical Engineering: Set – 2 | cv

Question Number: 6 Question Type: NAT Solution:  Option (A) cannot be inferred because the
The following graph represents the installed capacity given passage brings out the opinion of students with
for cement production (in tonnes) and the actual respect to mechanical engineering only.
production (in tonnes) of nine cement plants of a Option (B) cannot be inferred because the statement
cement company. Capacity utilization of a plant is does not make a specific reference to the opinions of
defined as ratio of actual production of cement to men with advanced degrees in mechanical engineering.
installed capacity. A plant with installed capacity of at Option (C) can be inferred because the research result
least 200 tonnes is called a large plant and a plant with indicates that nearly all the women with masters
lesser capacity is called a small plant. The difference or higher degrees in mechanical engineering were
between total production of large plants and small successful.
plants in tonnes is ______
Option (D) cannot be inferred because the passage
250 220
250
230
does not provide any information about the number
190
200 200
190 of women pursuing higher degrees in mechanical
200 180 190
160
150 160 160
150 140
engineering.
150 120 120
100 Hence, the correct option is (C).
100
50 Question Number: 8 Question Type: MCQ
0 Sourya committee had proposed the establishment of
1 2 3 4 5 6 7 8 9
Sourya Institutes of Technology (SITs) in line with
Solution:  As we know that plants 1, 4, 8, 9 are large Indian Institutes of Technology (IITs) to cater to the
while 2, 3, 5, 6, 7 are small. technological and industrial needs of a developing
The total production of the large plants is country.
160 + 190 + 230 + 190 = 770 Which of the following can be logically inferred form
The total production of the small plants is the above sentence?
150 + 160 + 120 + 100 + 120 = 650 Based on the proposal,
The difference is 770 - 650 = 120 tonnes. (i) In the initial years, SIT students will get
Hence, the correct Answer is (120). degrees from IIT
Question Number: 7 Question Type: MCQ (ii) SITs will have a distinct national objective
A poll of students appearing for masters in engineering (iii) SIT like institutions can only be established in
indicated that 60% of the students believed that consolation with IIT
mechanical engineering is a profession unsuitable (iv) SITs will serve technological needs of a
for women. A research study on women with masters developing country.
or higher degrees in mechanical engineering found (A) (iii) and (iv) only (B) (i) and (iv) only
that 99% of such women were successful in their (C) (ii) and (iv) only (D) (ii) and (iii) only
professions.
Solution:  Neither (i) nor (iii) is in the scope of the pas-
Which of the following can be logically inferred from
sage.
the above paragraph?
(A) Many students have misconceptions regarding Since, SITs are being established with a specific
various engineering disciplines purpose, (iii) can be inferred.
(B) Men with advanced degrees in mechanical (iv) is a direct extract of the given passage. Hence, only
engineering believe women are well suited to (ii) and (iv) can be inferred.
be mechanical engineers Hence, the correct option is (C).
(C) Mechanical engineering is a profession well
suited for women with masters or higher Question Number: 9 Question Type: MCQ
degrees in mechanical engineering Shaquille O’ Neal is a 60% career free throw shooter,
(D) The number of women pursuing higher meaning that he successfully makes 60 free throws out
degrees in mechanical engineering is small of 100 attempts on average. What is the probability

F03_Gate_EE_2016_978-93-325-7605-6_P-II.indd 105 6/19/2017 7:13:26 PM


cvi | GATE 2016 Solved Paper Electrical Engineering: Set – 2

that he will successfully make exactly 6 free throws in (0.0256) (0.046656)


10 attempts? = 0.2508
(A) 0.2508 (B) 0.2816 Hence, the correct option is (A).
(C) 0.2934 (D) 0.6000
Question Number: 10 Question Type: NAT
Solution:  The probability of exactly 6 successful
The numeral in the units position of 211870
throws is + 146127 × 3424 is ______.
10C (0.4)4 (0.6)6
6
Solution:  The units digit of the given expression is
10 (9) (8) (7) 256 46656
= = 210 1 + (6) (1) = 7
2 (3) ( 4) 10 4 106
Hence, the correct Answer is (7).

Electrical Engineering
Number of Questions: 55 Section Marks: 85.0

Q. 11 to Q. 35 carry 1 mark each and Q. 36 to Q. 65 Solution:  The circuit given in question is a practical
carry 2 marks each integrator therefore it will act like a low pass filter.
Question Number: 11 Question Type: MCQ Hence, the correct option is (A).
The output expression for the Karnaugh map shown Question Number: 13 Question Type: NAT
below is: The following figure shows the connection of an ideal
transformer with primary to secondary turns ratio of
  BC 1 : 100. The applied primary voltage is 100 V (rms),
A 00 01 11 10 50 Hz, AC. The rms value of the current I, in ampere,
     0 1 0 0 1 is ________.
     1 1 1 1 1
XL = 10 R = 80 kΩ
1:100
(A) A + B (B)
A+C
I
(C) A + C (D)
A+C 100 V
XC = 40 kΩ
Solution:  The output expression for the Karnaugh
map given is Solution:
f = A + C
XL = 10Ω
Hence, the correct option is (B). R = 80 kΩ

Question Number: 12 Question Type: MCQ


1:100
The circuit shown below is an example of a:
100 V
I XC = 40 kΩ
R2

On transferring the secondary parameters to primary side.


R1 +15 V 2
Vin Vout V 
R1 = R  1 
 V2 
–15 V 2
 100  80 × 1000
= (80 K)   =
(A) low pass filter (B) band pass filter  100 × 100  100 × 100
(C) high pass filter (D) notch filter = 8W

F03_Gate_EE_2016_978-93-325-7605-6_P-II.indd 106 6/19/2017 7:13:29 PM


GATE 2016 Solved Paper Electrical Engineering: Set – 2 | cvii

 1 
2  54 54 
X C1 = 40 K  (s) = 
Y  −
 = 4Ω 1 1
 100  s + s+ 
 6 3
Y (t) = 54e−t/6 u (t) − 54e−t/3 u (t)
j10Ω 8Ω –j4Ω Hence, the correct option is (D).
Question Number: 15 Question Type: NAT
100 V Suppose the maximum frequency in a band-limited
I
signal x (t) is 5 kHz. Then, the maximum frequency in
⇒I x (t) cos (2000pt), in kHz, is ________.
Solution:  The highest frequency component of
100 x (t) → f = 5 kHz {x (t) cos (2p × 1000t)}
I =
8 + j (10 − 4) 1
= [ x ( f − 1000) + x ( f + 1000) ]
100 2
I | =
|  = 10 A \ The maximum frequency in the product signal is,
10
f + 100 = 5000 + 1000 = 6 kHz
Hence, the correct Answer is (10). Hence, the correct Answer is (6).
Question Number: 14 Question Type: MCQ Question Number: 16 Question Type: MCQ
Consider a causal LTI system characterized by Consider the function f (z) = z + z* where z is a complex
dy ( t ) 1 variable and z* denotes its complex conjugate. Which
differential equation + y ( t ) = 3x (t). The
dt 6 one of the following is TRUE?
t
− (A) f (z) is both continuous and analytic
response of the system to the input x (t) 3e 3 u (t ) , (B) f (z) is continuous but not analytic
where u (t) denotes the unit step function, is _______. (C) f (z) is not continuous but is analytic

t (D) f (z) is neither continuous nor analytic
(A) 9e 3 u (t )
Solution:  Given

t f (z) = z + z*
(B) 9e 6 u (t ) Let z = x + iy
t t
⇒ z* = x – iy
− − \ f (z) = z + z*
(C) 9e 3 u (t ) − 6e 6 u (t )
= (x + iy) + (x – iy)
t t ⇒ f (z) = 2x
− −
(D) 54e 6 u (t ) − 54e 3 u (t )
Clearly f (z) is continuous
Solution:  Taking Laplace on both sides Let f (z) = 2x + i0 = u + iv
\ u = 2x; v = 0
 1 ∂u ∂u ∂v ∂u
 s + 6  Y (s) = 3X (s) = 2; = 0; = 0 and =2
    ∂x ∂y ∂x ∂x

Y ( s) ∂u ∂v
=
3 =2≠ (= 0)
X ( s) 1 ∂x ∂y
s+
6 \ The real and imaginary parts of f (z) fails to satisfy
 3  3  the Cauchy–Riemann equations.
Y (s) = 
1  1 Hence f (z) is not analytic.
 s +  s + 
   6  3 Hence, the correct option is (B).

F03_Gate_EE_2016_978-93-325-7605-6_P-II.indd 107 6/19/2017 7:13:34 PM


cviii | GATE 2016 Solved Paper Electrical Engineering: Set – 2

Question Number: 17 Question Type: MCQ Applying the inverse Laplace transform on both sides,
A 3 × 3 matrix P is such that, P3 = P. Then the −1  1 
L−1 [ y ] = L  2
 ( s + 1) 
eigenvalues of P are:
(A) 1, 1, –1
(B) 1, 0.5 + j0.866, 0.5 – j0.866  1 
⇒ y = L−1 
(C) 1, –0.5 + j0.866, –0.5, –j0.866 2
(D) 0, 1, –1  ( s − ( −1) ) 

Solution:  Given that P is a 3 × 3 matrix such that = te −t = te −t u (t )
P3 = P. \ The solution of the given differential equation is
Let l be an eigenvalue of P. y = te–t u (t)
As P is a 3 × 3 matrix and Hence, the correct option is (A).
P3 = P Question Number: 19 Question Type: MCQ
We have The value of the line integral
⇒ l3 – l = 0
∫ (2 xy dx + 2 x
2 2
⇒ l (l2 – 1) = 0 ydy + dz )
⇒ l = 0; l2 – 1 = 0 c
⇒ l = 0; l = ±1 along a path joining the origin (0, 0, 0) and the point
\ The eigenvalues of P are 0, 1 and –1. (1, 1, 1) is:
(A) 0 (B) 2
Hence, the correct option is (D).
(C) 4 (D) 6
Question Number: 18 Question Type: MCQ
Solution:  We have to evaluate the line integral
The solution of the differential equation, for t > 0,
∫ (2 xy dx + 2 x
2 2
y′′ (t) 2y′ (t) + y (t) = 0 with initial conditions y (0) = 0 ydy + dz )
and y′ (0) = 1, is (u (t) denotes the unit step function), c
(A) te–t u (t)
along a path joining the origin (0, 0, 0) and the point
(B) (e–t – te–t) u (t)
(1, 1, 1).
(C) (–e–t + te–t) u (t)
Equation of the line joining (0, 0, 0) and (1, 1, 1) is
(D) e–t u (t)
x−0 y−0 z−0
Solution:  Given differential equations is = = = t, (say)
1− 0 1− 0 1− 0
y″ + 2y′ + y = 0 (1) ⇒ x = y = z = t
with the initial conditions ⇒ x = t, y = t
y  (0) = 0 and y′ (0) = 1 (2) and z =t
Applying Laplace transform on both sides of Equa- ⇒ dx = dt, dy = dt and dz = dt
tion (1), and t varies from t = 0 to t = 1
We have
∫ [2 xy
2
dx + 2 x 2 ydy + dz ]
L [ y ′′] + 2 L [ y ′] + L [ y ] = 0 c
⇒ s y − sy(0) − y ′(0) + 2 ( sy − y(0) ) + y
2 1

= 0
= ∫ [2(t ) (t 2 ) dt + 2 (t 2 )(t ) dt + dt ]
t =0
where y = L [y]
2 1
⇒ s y − s × 0 − 1 + 2 sy − 2 × 0 + y
= ∫ [4t 3 + 1] dt
= 0 [from Equation (2)] t =0
⇒ ( s 2 + 2 s + 1) y = 1 1
= t4 + t 
1 1  t =0
⇒ y= =
2 2
( s + 2 s + 1) ( s + 1) Hence, the correct option is (B).

F03_Gate_EE_2016_978-93-325-7605-6_P-II.indd 108 6/19/2017 7:13:38 PM


GATE 2016 Solved Paper Electrical Engineering: Set – 2 | cix

Question Number: 20 Question Type: MCQ From the above figure, we get
Let f (x) be a real, periodic function satisfying Vs2 = (VR + VC cos f)2 + (VC sin f)2
f (–x) = –f (x). The general from of its Fourier series Vs2 = VR2 + VC2 + 2VRVC cos φ
representation would be
∞ V 2 − VR2 − VC2
(A) f (x) = a0 ∑ ak cos ( kx ) cos f = S
2VRVC
k =1
∞ From the given options, option (D) gives f = 90°.
(B) f (x) = ∑ bk sin (kx) Hence, the correct option is (D).
k =1
∞ Question Number: 22 Question Type: NAT
(C) f (x) = a0 + ∑ a2k cos (kx) The voltage (V) and current (A) across a load are as
k =1
∞ follows.
(D) f (x) = ∑ a2k + 1 sin ( 2k + 1) x n (t) = 100 sin (wt),
k = 0 +1 I (t) = 10 sin (wt – 60°) + 2 sin (3wt)
Solution:  Given + 5 sin (5wt)
f (–x) = –f (x) The average power consumed by the load, in W is
∞ ________.
∑ a2k +1 sin (2k + 1) is an odd function. Solution:
k =1
\ The Fourier series expansion of f (x) consists of only 1000
Vrms =
sine terms. 2

10
i.e., f (x) = ∑ bk sin ( kx ) Irms =
2
k =1
Hence, the correct option is (B). f = 60
Average power consumed
Question Number: 21 Question Type: MCQ
P = Vrms ⋅ Irms ⋅ cos f
A resistance and a coil are connected in series and
supplied form a single phase, 100 V, 50 Hz ac source as 1000 10
= ×
× cos 60° = 250 watts
shown in the figure below. The rms values of possible 2 2
voltages across the resistance (VR) and coil (VC) Hence, the correct Answer is (250).
respectively, in volts, are:
Question Number: 23 Question Type: MCQ
A power system with two generators is shown in
VR
the figure below. The system (generators, buses and
VS VC transmission lines) is protected by six over current
relays R1 to R6. Assuming a mix of directional and non-
directional relays at appropriate locations, the remote
backup relays for R4 are:
(A) 65, 35 (B) 50, 50 R1 R2
S1 R5 R6 S2
(C) 60, 90 (D) 60, 80
R3 R4
Solution:  Consider the figure given below:

VS (A) R1, R2
VC (B) R2, R6
VC sin φ
φ (C) R2, R5
VR VC cos φ (D) R1, R6

F03_Gate_EE_2016_978-93-325-7605-6_P-II.indd 109 6/19/2017 7:13:41 PM


cx | GATE 2016 Solved Paper Electrical Engineering: Set – 2

Solution:  Consider the figure given below:


( 400 × 103 ) 2
Load impedance (ZL) =
R1 R2 300 × 106
S1 R5 R6 S2 = 533.33W
The receiving end voltage will rise more than sending
end voltage because the load impedance is more than
R3 R4 surge impedance,
\ VR VS ⇒ VR > 400 kV
An inductive compensation is needed to reduce the
If a fault was taken place b/w (3) and (4) R3 and R4
voltage to 400 kV.
must operate. If R4 is failed to operate, then to limit
I1 → R1 must operate and to limit I2 → R6 must operate. Hence, the correct option is (B).
Hence, the correct option is (D). Question Number: 26 Question Type: MCQ
Question Number: 24 Question Type: MCQ A parallel plate capacitor filled with two dielectrics is
shown in the figure below. If the electric field in the
A power system has 100 buses including 10 generator
region A is 4 kV/cm, the electric field in the region B,
buses, for the load flow analysis using Newton–
in kV/cm, is ____.
Raphson, method in polar coordinates, the size of the
Jacobian is: A B
(A) 189 × 189 (B) 100 × 100
∈r = 1 ∈r = 4 2 cm
(C) 90 × 90 (D) 180 × 180
Solution:  Load buses 11
= 100 – 10 = 90 → (m) (A) 1 (B) 2
Gen buses = 10 → n (C) 4 (D) 16
Size of Jacobian matrix will be
Solution:  For parallel plate capacitor electric field is
= [2m + (n − 1)] [2m – (n − 1)]
same for region A and region B, therefore the electric
= [2 × 90 + (10 − 1)] [2 × 90 + (10 − 1)] field in the region B, is 4 kV/cm.
= 189 × 189
Hence, the correct option is (C).
Hence, the correct option is (A).
Question Number: 27 Question Type: NAT
Question Number: 25 Question Type: MCQ
A 50 mVA, 10 kV, 50 Hz, star-connected, unloaded three-
The inductance and capacitance of a 400 kV, three- phase alternator has a synchronous reactance of 1 p.u
phases 50 Hz lossless transmission line are 1.6 mH/km/ and a sub-transient reactance of 0.2 p.u. If a 3-phase
phase and 10 nF/km/phase respectively. The sending short circuit occurs close to the generator terminals,
end voltage is maintained at 400 kV. To maintain a the ratio of initial and final values of the sinusoidal
voltage of 400 kV at the receiving end, when the line component of the short circuit current is ________.
is delivering 300 mW load, the shunt compensation
required is: Solution:  Immediately after the short ckt (During-
(A) capacitive (B) inductive transient period)
(C) resistive (D) zero
ZS11
Solution:
L Eg ISC
Surge impedance (ZS) =
C

1.6 × 10 −3
= = 400Ω 1.0
10 × 10 −9 ⇒ ISC = = 1 p.u.
1.0

F03_Gate_EE_2016_978-93-325-7605-6_P-II.indd 110 6/19/2017 7:13:43 PM


GATE 2016 Solved Paper Electrical Engineering: Set – 2 | cxi

At steady state I0
=
3
ZS
100
Eg = = 57.735 A
3
ISC
Hence, the correct Answer is (57.735).
Question Number: 30 Question Type: NAT
1.0 A buck-boost DC–DC converter, shown in the figure
⇒ ISC = = 1 p.u.
1.0 below, is used to convert 24 V battery voltage to 36 V
Initial 5 DC voltage to feed a load of 72 W. it is operated at
= =5 20 kHz with an inductor of 2 mH and output capacitor
Final 1.0
of 1000 mF. All devices are considered to be ideal. The
Hence, the correct Answer is (5). peak voltage across the solid-state switch (S), in volt is
Question Number: 28 Question Type: NAT _______.
Consider a linear time-invariant system with transfer
function
1 S Load
H (S) = + –

( S + 1) 24 V 2 mH +
36 V

If the input is cos (t) and the steady state output is A cos
(t + a), then the value of A is _________.
Solution:  The given transfer function is Solution:  Load power,
1 P0 =72 W
H (S) =

( S + 1) P0 = V0 I0
Now we have V0 I0 = 72 W
1 1 1 72
A = H ( s) = = = I0 = =2A
s +1 36
2
ω +1 2

D
a = −tan−1 (w) = −tan−1 (1) = −45° V0 = × Vdc
1 − D
1
A = D
2 ⇒ 36 = × 24
Hence, the correct Answer is (0.707). 1− D
Ripple voltage, can be calculates using
Question Number: 29 Question Type: NAT I
A three-phase diode bridge rectifier is feeding a DV = 0 ⋅ TON
constant DC current of 100 A to a highly inductive load. C
If three-phase, 415 V, 50 Hz AC source is supplying to I D 2 × 0.6
∆v = 0 = = 0.06 V

this bridge rectifier then the rms value of the current in

Cf 10 × ( 20 × 103 )
3
each diode, in ampere, is ________.
The peak voltage will be
Solution:  Diode current rms can be calculated using ∆V 0.06
relation, Vpeak = (Vo)avg + = 36 + = 36.03 V
2 2
2 Peak voltage across switch
I0
IS 3 (VS)peak = 24 + 36.03 V = 60.03 V
[ID]rms = =
2 2 Hence, the correct Answer is (60.03).

F03_Gate_EE_2016_978-93-325-7605-6_P-II.indd 111 6/19/2017 7:13:47 PM


cxii | GATE 2016 Solved Paper Electrical Engineering: Set – 2

Question Number: 31 Question Type: NAT 2Ω


For the network shown in the figure below, the
frequency (in rad/s) at which the maximum phase lag
occurs, is ________.
10 V 5A Vout
9Ω

1Ω
vin v0

2 Ff Solution:  From the figure given above, we get


Vout = 5[2] + 10 = 20 V
Hence, the correct option is (D).
Solution:
Question Number: 34 Question Type: MCQ
 1
1 + s  The graph associated with an electrical network has
V0 ( s)   s +1 
= =  7 branches and 5 nodes. The number of independent
Vin ( s)
10 +
1  10 s + 1  KCL equations and the number of independent KVL
s equations, respectively, are:
T = 1 (A) 2 and 5 (B) 5 and 2
aT = 10   a = 10 (C) 3 and 4 (D) 4 and 3
1 1 Solution:  The number of independent KVL equation
w= w=
T a 10 =b−n+1
= 0.316 rad/sec = 7 − 5 + 1 = 3
The number of independent KCL equation
Hence, the correct Answer is (0.316).
= (n − 1) = 5 − 1 = 4
Question Number: 32 Question Type: MCQ Hence, the correct option is (D).
The direction of rotation of a single-phase capacitor
run induction motor is reversed by: Question Number: 35 Question Type: MCQ
(A) interchanging the terminals of the AC supply Two electrodes, whose cross-sectional view is shown
(B) interchanging the terminals of the capacitor in the figure below, are at the same potential. The
(C) interchanging the terminals of the auxiliary maximum electric field will be at the point:
winding
(D) interchanging the terminals of both the
A D C
windings.
Solution:  By interchanging the terminals of auxil-
iary winding rotating magnetic field directions can be B
changed
(A) A (B) B
Hence, the correct option is (C). (C) C (D) D
Question Number: 33 Question Type: MCQ Solution:
In the circuit shown on next column, the voltage and Hence, the correct option is (A).
current sources are ideal. The voltage (Vout) across the
current source, in volts, is ____. Question Number: 36 Question Type: MCQ
(A) 0 The Boolean expression ( a + b + c + d ) + (b + c )
(B ) 5 simplifies to:
(C) 10 (A) 1 (B) a ⋅ b
(D) 20 (C) a ⋅ b (D) 0

F03_Gate_EE_2016_978-93-325-7605-6_P-II.indd 112 6/19/2017 7:13:48 PM


GATE 2016 Solved Paper Electrical Engineering: Set – 2 | cxiii

Solution: Question Number: 38 Question Type: MCQ


f = ( a + b + cd ) + (b + c ) Let x1 (t) ↔ X1 (w) and x2 (t) ↔ X2(w) be two signals

f = ( a + b + c + d ) ⋅ (b + c ) whose Fourier Transforms are as shown in the figure
below. In the figure h (t) = e–2 | t | denotes the impulse
f = ( a ⋅ b ⋅ c ⋅ d ) (b ⋅ c) = 0
response.
Hence, the correct option is (D). For the system shown below, the minimum sampling
Question Number: 37 Question Type: MCQ rate required to sample y (t), so that y (t) can be uniquely
reconstructed form its samples, is
For the circuit shown below, taking the opamp as ideal,
the output voltage Vout in terms of the input voltages X1(ω) X2(ω)
V1, V2, and V3 is ______.
9Ω

ω ω
–B1 –B1 B1 B1 –B2 B2
1Ω VCC 2 2
V3
1Ω x1(t)
V1 Vout
VSS y(t)
h(t) = e–2(t)
4Ω
V2
x2(t)

(A) 1.8V1 + 7.2V2 – V3 (B) 2V1 + 8V2 – 9V3 (A) 2B1 (B) 2 (B1 + B2)
(C) 7.2V1 + 1.8V2 – V3 (D) 8V1 + 2V2 – 9V3 (C) 4 (B1 + B2) (D) ∞

Solution: Solution:  The product signal x1(t) x2(t) has the highest
frequency component of B1 + B2.
9Ω
\ As per the Nyquist principle,
fs = 2 [B1 + B2]
1Ω VCC Hence, the correct option is (B).
V3
1Ω
Question Number: 39 Question Type: MCQ
V1 Vout

 sin 2πt 
4Ω
VSS The value of the integral 2 ∫
 πt  dt is equal to:
−∞  
V2
(A) 0 (B) 0.5
(C) 1 (D) 2
By applying virtual GND concept, we get
Vx − V1 Vx − V2 Solution:
+ =0 ∞
1 4 sin 2πt
Let I=2∫ dt
5Vx = 4V1 + V2 (1) πt
Vx − V3 Vx − Vout −∞
+ =0
1 9  ∞ sin 2πt 
10Vx = 9V3 + Vout (2) = 2  2∫ dt 
 πt 
Solving Equations (1) and (2), we have  0 
8V1 + 2V2 = 9V3 + Vout
 sin 2πt 
Vout = 8V1 + 2V2 − 9V3 ∵ πt is an even function 
Hence, the correct option is (D).  

F03_Gate_EE_2016_978-93-325-7605-6_P-II.indd 113 6/19/2017 7:13:51 PM


cxiv | GATE 2016 Solved Paper Electrical Engineering: Set – 2

∞ Question Number: 40 Question Type: NAT


4 sin 2πt
\ I=
π ∫ t
dt (1) Let y (x) be the solution of the differential equation
0 d2 y dy
sin 2πt −4 + 4 y = 0 with initial conditions y (0) = 0
Consider the Laplace transform of = 2 dx
dx
t dy
∞ and = 1. Then the value of y (1) is ________.
 sin 2πt 
i.e.,  t  = ∫ L [ sin 2πt ] ds dx x =0
  s Solution:  Given differential equation is
 ∞ 
 f (t )  d2 y dy
∵ L 
 = ∫ L [ f ( t ) ] ds 
 2
−4
dx
+ 4 y = 0 (1)
  t  s  dx

∞ With the initial conditions



= ∫ s2 + ds
y (0) = 0 and
dy
=1
s ( 2π )2
dx x = 0
(2)


 s  Applying Laplace transform on both sides of Equa-
= tan −1  
 2π  s tion (1),

 s   d2 y   dy 
= tan −1 ∞ − tan −1   L  − 4L   + 4L [ y ] = 0
 2π   dx 2   dx 

2
π  s  ⇒ s y − sy (0) − y ′(0) − 4 ( sy − y (0)) + 4 y = 0
= − tan −1  
2  2π  where y = L [y]
 sin 2πt  −1  s  ⇒ s 2 y − s × 0 − 1 − 4 sy + 0 + 4 y = 0
\ L  = cot  2π 
 t    ⇒ ( s 2 − 4 s + 4) y = 1

− st  sin 2πt  1 1
⇒ ∫e  t  dt ⇒ y=
2
=
2
0   s − 4 s + 4 ( s − 2)
 s 
= cot −1   Applying inverse Laplace transform on both sides
 2π 
L−1 [ y ] = L−1  
1
Taking s = 0 on both sides,
2
∞  ( s − 2 ) 
 sin 2πt 
− oxt −1  0 
∫e  t

 dt = cot 


 2π  ⇒ y = xe2x
0
The solution of Equation (1) is

 sin 2πt  −1
⇒ ∫   dt = cot (0) y = xe2x
 t 
0 Now y (1) = yat x = 1 = 1 × e2 × 1 = e2

 sin 2πt  = π
⇒ ∫   dt 2 (2) \ y (1) = 7.389
 t 
0 Hence, the correct Answer is (7.389).
Substituting Equation (2) in (1), we get
Question Number: 41 Question Type: NAT
4 π
I= × The line integral of the vector field
π 2

sin 2πt F = 5 xziˆ + (3 x 2 + 2 y ) ˆj + x 2 zkˆ
i.e., 2∫ dt = 2
πt
−∞ along a path from (0, 0, 0) to (1, 1, 1) parameterized by
Hence, the correct option is (D). (t, t2, t) is _________.

F03_Gate_EE_2016_978-93-325-7605-6_P-II.indd 114 6/19/2017 7:13:59 PM


GATE 2016 Solved Paper Electrical Engineering: Set – 2 | cxv

Solution:  We have to find the line integral of the vec- 1


tor field (C) an ellipse with major axis along  
2 2 1
F = 5 xzi + (3 x + 2 y ) j + x zk
along a path from (0, 0, 0) to (1, 1, 1) parameterized 1
(D) an ellipse with minor axis along  
by (t, t2, t) 1
i.e., along the path
Solution:
x = t, y = t2 and z = t
3 1
⇒ dx = dt, dy = 2t dt and dz = dt Given P=
1 3 
and t varies from t = 0 to t = 1 
\ The required line integral is Given a2 + b2 =1 (1)
(1,1,1) (1,1,1) a  x  3 1  x   a 
and  b  = P  y  = 1 3   y   b 
∫ F ⋅ dr = ∫ (5 xzi + (3 x 2 + 2 y ) j
       
( 0, 0, 0) ( 0, 0, 0 )
2  3x + y 
+ x zk ) ⋅ ( dxi + dy j + dzk ) =
 x + 3 y 
(1,1,1)
= ∫ [5 xzdx + (3 x 2 + 2 y ) dy + x 2 zdz ] ⇒ a = 3x + y and b = x+ 3y
( 0, 0, 0 ) From Equation (1),

1 a2 + b2 = 1
= ∫ [5 (t ) (t ) dt + (3(t )2 + 2(t 2 )) 2tdt ⇒ (3x + y)2 + (x + 3y)2 =1
t =0
9x2 + 6xy + y2 + x2 + 6xy + 9y2 = 1
+ (t ) 2 (t ) dt ]
10x2 + 12xy + 10y2 = 1
1
⇒ 8(x + y )2 + 2(x – y)2 = 1
= ∫ [5t 2 + 10t 3 + t 3 ] dt
t =0 ( x + y )2 ( x − y )2
1 + =1
1/8 1/ 2
= ∫ [5t 2 + 11t 3 ] dt
Which represents an ellipse a < b
t =0
\ Major axis is x + y = 0 and minor axis is x – y = 0.
1
5 11 
= t3 + t 4  Hence, the correct option is (D).
3 4 t = 0
Question Number: 43 Question Type: MCQ
5 11
= + Let the probability density function of a random
3 4
variable, X, be given as:
53
= = 4.4167 3
12 fx (x) = e −3 x u ( x ) + ae 4 x ( − x )
2
Hence, the correct Answer is (4.4167). where u (x) is the unit step function.
Question Number: 42 Question Type: MCQ Then the value of ‘a’ and probe {X ≤ 0}, respectively,
are:
3 1
Let P =  ⋅ Consider the set S of all vectors 1 1
 1 3  (A) 2, (B) 4,
2 2
 x  such that a2 + b2 = 1 where  a  = P  x  ⋅ Then 1 1
 y b  y (C) 2, (D) 4,
      4 4
S is ________. Solution:  Given the probability density function of a
(A) a circle of radius 10 random variable X, is
1 3
(B) a circle of radius fX (x) = e −3 x u( x ) + ae 4 x u( − x )
10 2

F03_Gate_EE_2016_978-93-325-7605-6_P-II.indd 115 6/19/2017 7:14:05 PM


cxvi | GATE 2016 Solved Paper Electrical Engineering: Set – 2

 ae 4 x V = − V1 + VS
; −∞ < x < 0
i.e., f X ( x ) =  3 −3 x −6V + 3VS − 24V1 = 0
; 0≤x<∞
 2 e 3Vs = 30V1
Vs = 10V1
we know that Vs = 10 [2IS]

V
∫ f X ( x ) dx = 1 Zin = S = 20Ω
IS
−∞
0 ∞ Hence, the correct Answer is (20).
3 −3 x
⇒ ∫ ae 4 x dx + ∫ 2
e dx = 1
Question Number: 45 Question Type: NAT
−∞ 0
The Z-parameters of the two port network shown in
0 ∞
a 4x  3 e −3 x  the figure are Z11 = 40W, Z12 = 60W, Z21 = 80W, and
⇒ e  +  =1 Z22 = 100W. The average power delivered to RL = 20W,
4  −∞ 2 −3  0
in watts, is _________.
a 1 10Ω
⇒ + =1 I1 I2
4 2 + +
⇒ a = 2
0 +
20 V – V1 [Z] V2 RL
and P (X < 0) = ∫ f X ( x ) dx
−∞ – –
0
= ∫ ae 4 x dx Solution:
−∞
V1   Z11I1 + Z12 I 2 
0  = 
e 4x 1 V2   Z12 I1 + Z 22 I 2 
=2  =
4  2 V2 = −I2 [R2] = −20I2 (1)
−∞
= I2 [10] + V1
20
Hence, the correct option is (A).
= I2 (10) + 40 I1 + 60I1
20
Question Number: 44 Question Type: NAT
= 50I1 + 60I2 (a)
20
The driving point input impedance seen from the source
Vs of the circuit shown below, in W is_________. V2 = 80I1 + 100 I2 (2)
From Equation (1)
V1 2Ω
−20I2 = 80I1 + 100I1
Is
2Ω −120I2 = 80I1
Vs 3Ω 4V1
3
4Ω I1 = − I 2 (3)
2
Substitute Equation (3) in (a)
Solution:  From the given figure in question
 −2 
V = 50I1 + 60  I1 
20
IS = 1  3 
2
Applying KVL at 3W resistor, we get 20 = 50I1 − 40I1
V − VS V V 10I1 = 20
+ − 4V1 + = 0
2 3 6 I1 = 2
3V − 3VS + 2V − 24V1 + V = 0 I2 = −4/3

F03_Gate_EE_2016_978-93-325-7605-6_P-II.indd 116 6/19/2017 7:14:08 PM


GATE 2016 Solved Paper Electrical Engineering: Set – 2 | cxvii

2 Question Number: 47 Question Type: NAT


4 16
PR =   × 20 = × 20 In the circuit shown below, the initial capacitor voltage

C
 3  9
is 4 V. switch S1 is closed at t = 0. The charge (in mC)
= 35.55 W lost by the capacitor form t = 25 mS to t = 100 mS is
Hence, the correct Answer is (35.55). ________.
S1
Question Number: 46 Question Type: NAT
In the balanced 3-phase, 50 Hz, circuit shown below,
the value of inductance (L) is 10 mH. The value of the
capacitance (C) for which all the line currents are zero,
in millifarads, is ________. 4V
5Ω
5 µF

L C L

C C Solution:
L VC (t) = Vf + (Vi − Vf) e−t/T
−6
VC (t) = 0 + (4 − 0) e −t/25×10
Solution:  Consider the simplified circuit shown be- 4
VC (t) = 4e −t ×10 ×4
low:
Q = cvc (t)
= (5 × 10−6) [4e−(t × 4 × 104)]
C
Charge lost (Q) from t = 25 ms to t = 100 ms can be
L/3 calculated as
Q = 5 × 10−6 × 4 [e−1 − e−4]
= 6.99 mC
Hence, the correct Answer is (6.99).

L/3
L/3 Question Number: 48 Question Type: NAT
The single line diagram of a balanced power system
C C
is shown in the figure. The voltage magnitude at the
generator internal bus is constant and 1.0 p.u. the
p.u reactances of different components in the system
are also shown in the figure. The infinite bus voltage
magnitude is 1.0 p.u. A three phase fault occurs at the
Given, IL = 0 ⇒ capacitor must supply inductor current
middle of line 2.
V ph
IC = IL  Vph ( jwc) = Generator infinite bus
jω ( L/3) Infinite bus
j0.1
3
wc = j0.5
ωL Line 1
3
⇒ w2 = j0.2
LC
3
⇒ C = = 3 mF j0.5
−3
10 × 10 Line 2
Hence, the correct Answer is (3). j0.1

F03_Gate_EE_2016_978-93-325-7605-6_P-II.indd 117 6/19/2017 7:14:11 PM


cxviii | GATE 2016 Solved Paper Electrical Engineering: Set – 2

The ratio of the maximum real power that can be 1.0 1.15
transferred during the pre-fault condition to the 1.0 PU
maximum real power that can be transferred under the
faulted condition is ______ 0.6656 0.221
Solution:  For pre-fault condition:

j0.6 1.0 PU
1.0 PU
The maximum power can be calculated using
j0.2
(1.0) (1.0)
Pmax = = 0.869 PU
j0.6 1.15
The maximum power can be calculated using The ratio of the maximum real power that can be
(1.0) (1.0) (1.0) (1.0) transferred during the pre-fault condition to the
Pmax = = = 2 PU maximum real power that can be transferred under the
Xq j 0.5
faulted condition is
During the fault: Ppre-fault 2
= = 2.3
Convert ∆ to Y Pduring fault 0. 869

Hence, the correct Answer is (2.3).
Question Number: 49 Question Type: MCQ
j0.6 1.0 PU
The open loop transfer function of a unity feedback
1.0 PU
control system is given by
j0.2 K ( S + 1)
G  (S) = , K > 0, T > 0
j0.35 j0.25 S (1 + Ts ) (1 + 2S )

The closed loop system will be stable if,
4 ( K + 1)
(A) 0 < T <
K −1
The above circuit can be drawn as follows.
4 (T + 2 )
(B) 0 < K <
1.0 j0.175 j0.125 T −2
1.0 PU
T +2
j0.2 (C) 0 < K <
T −2
j0.072
8 ( K + 1)
(D) 0 < T <
K −1

Solution:
1 + G (S) = 0
Convert to Y ∆
S (1 + TS) (1 + 2S) + KS + K = 0
(2T) S 3 + (2 + T) S 2 + (1 + K) S + K
j0.1 RH criteria
1.0 j0.375 j0.125
S 3 2T 1+K
S 2 2+T K
j0.072 ( 2 + T ) (1 + K ) − 2TK
S1 0
2+T
S 0 K 0

F03_Gate_EE_2016_978-93-325-7605-6_P-II.indd 118 6/19/2017 7:14:15 PM


GATE 2016 Solved Paper Electrical Engineering: Set – 2 | cxix

For the system is to be stable, first column elements Question Number: 51 Question Type: NAT
should be positive The power consumption of an industry is 500 kVA, at
Given T > 0, hence 2 + T > 0 0.8 p.f. lagging. A synchronous motor is added to raise
2TK the power factor of the industry to unity. If the power
(1 + K) >
2+T intake of the motor is 100 kW, the p.f. of the motor is
________.
K > 0
 2T  Solution:
1>K − 1
2 + T  PL = (500) (0.8) = 400 kW

QL = (500) (0.6) = 300 kW
T − 2 
1 < K   \ Qmotor = 300 kVAR
T + 2 
Qmotor = 100 kW
T +2 Pmotor
\ 0 <K <
T −2 P.f. of the motor =
P 2 + Qm2
Hence, the correct option is (C). m
Question Number: 50 Question Type: NAT 100
=
At no load condition, a 3-phase, 50 Hz, lossless power 100 + 300 2
2

transmission line has sending-end and receiving-end = 0.316 lead


voltages of 400 kV and 420 kV respectively. Assuming Hence, the correct Answer is (0.316).
the velocity of travelling wave to be the velocity of
light, the length of the line, in km, is ________. Question Number: 52 Question Type: NAT
The flux linkage (l) and current (i) relation for an
Solution:
VS electromagnetic system is l = ( i ) /g. When i = 2 A
cos hnL = A = and g (air-gap length) = 10 cm, the magnitude of
VR no load mechanical force on the moving part, in N, is ________.

YZ 400 Solution:  The mechanical force will be
1+ =
2 420 F = (i3/2)/ 2g2 = 23/2/(2 × 0.1 × 0.1) = 141.4 N
( jωc) ( jωc) 400 Hence, the correct Answer is (141.4 N).
1+ =
2 420 Question Number: 53 Question Type: NAT
20 ω2 LC The starting line current of a 415 V, 3-phase, delta
= connected induction motor is 120 A, when the rated
420 2
voltage is applied to its stator winding. The starting
1 ω2 l 2 line current at a reduced voltage of 110 V, in ampere
= is ________.
21 2U 2
21 2 2 Solution:  The starting current of a 3-phase IM is
u2 = ω l V
2 Ist =
Rq + X q2
2
21
u = ωl
2 Ist a V
I1 V1
3 × 108 =
l= = 294.847 mm I 2 V2
21
314 ×
2 V 
⇒ I2 =  1  I1
V
Hence, the correct Answer is (294.847).  2

F03_Gate_EE_2016_978-93-325-7605-6_P-II.indd 119 6/19/2017 7:14:19 PM


cxx | GATE 2016 Solved Paper Electrical Engineering: Set – 2

 110  Question Number: 55 Question Type: NAT


=  (120)
 415  A full-bridge converter supplying an RLE load is shown

in figure. The firing angle of the bridge converter is
= 31.80 A 120°. The supply voltage nm (t) = 200p sin (100pt) V,
Hence, the correct answer is 31.8. R = 20W, E = 800 V. The inductor L is large enough
Question Number: 54 Question Type: NAT to make the output current IL a smooth dc current.
Switches are lossless. The real power feedback to the
A single-phase, 2 kVA, 100/200 V transformer is source, in kW, is __________.
reconnected as an auto-transformer such that its kVA
rating is maximum. The new rating, in kVA is _______. Load
IL
Solution:
L
I1 I2

R = 20Ω
T1 T3
V1 V2
vm Bridge

E = 800 V

T4 T2
+

Primary voltage V1 = 100 V,


Secondary voltage
V2 = 200 V
Now primary current will be
2 × 1000
I1 = = 20 A Solution:
100
Given E = 800 V
and secondary current will be
R = 20W
2 × 1000
I2 = = 10 A Output voltage, can be calculated using relation
200 V0 = −E + I0 R
If it is connected as an auto transformer V0 = −800 + I0 × 20 (1)
We also know that output voltage can be expressed as
I1
2V
V0 = m cos α
π
100
2 × 200π
= cos 120
300 π
(I1 + I2)
= −200 V (2)
200 From Equations (1) and (2), we get
−200 = −800 + I0 × 20
I2
I0 = 30 A
Power feedback to the source,
VA rating = (300) (20)
P0 = V0 I0 = 200 × 30
= 6000
= 6 kVA = 6 kW
Hence, the correct Answer is (6). Hence, the correct Answer is (6).

F03_Gate_EE_2016_978-93-325-7605-6_P-II.indd 120 6/19/2017 7:14:22 PM


GATE 2016 Solved Paper Electrical Engineering: Set – 2 | cxxi

Question Number: 56 Question Type: NAT V


V0 = dc
A three-phase Voltage Source Inverter (VSI) as shown 1− D
in the figure is feeding a delta connected resistive 360
load of 30W/phase. If it is fed from a 600 V battery, 400 =
1 −D
with 180° conduction of solid-state devices, the power
consumed by the load, in kW, is ________. D = 0.1
Output power can be expressed as
+ P0 = V0 I0
4000 = 400 × I0
I0 = 10 A

30
30
Average current across the inductor


600 V
30Ω I 10 10
(IL)avg = 0 = = A
1 − D 1 − 0.1 0.9
RMS current across the inductor
2
– (IL)rms = ( I L )avg + ( I L ripple) 2rms

Solution: ∆I
(IL)ripple rms = L
Vdc = 600 V 2 3
R = 10 Ohms By neglecting ripple current,
Output power, Po = VL IL cos f IL(ripple rms) = 0
2
VL2 2Vdc 2 × (600)2 (IL)rms = (IL)avg =
10
A
== =
3R 3 × 10 0.9
R
= 24 kW (ISW)rms = D ( I L ) rms
Hence, the correct Answer is (24). 10
= 0.1 × = 3.513 A
Question Number: 57 Question Type: NAT 0.9
A DC-DC boost converter, as shown in the figure below, Hence, the correct Answer is (3.51).
is used to boost 360 V to 400 V, at a power of 4 kW.
Question Number: 58 Question Type: NAT
All devices are ideal. Considering continuous inductor
current, the rms current in the solid state switch (S), in A single-phase bi-directional voltage source converter
ampere is _______. (VSC) is shown in the figure given on next page. All
devices are ideal. It is used to charge a battery at 400 V
10 mH with power of 5 kW from a source Vs = 220 V (rms),
+
50 Hz sinusoidal AC mains at unity p.f. If its AC side
interfacing inductor is 5 mH and the switches are
Load + operated at 20 kHz, then the phase shift (d) between
360 V
S 1 mF 400 V AC mains voltage (Vs) and fundamental AC rms VSC
– voltage (Vc1), in degree, is ________.
Solution:

DC power PDC = 5 kW = 5000 W
AC power PAC = Vsr1 Isr1 cos q
Solution: = 220 × Isr1 × 1
Output voltage, V0 = 400 V Now we know that
DC input voltage PAC = PDC
Vdc = 360 V
5000
Output power P0 = 4 kW = 4000 W Isr = = 22.73 A
220

F03_Gate_EE_2016_978-93-325-7605-6_P-II.indd 121 6/19/2017 7:14:24 PM


cxxii | GATE 2016 Solved Paper Electrical Engineering: Set – 2

 λ1t 0 
α 
X (t) eAt x (0) =  e λ 2t   
  0
0 e 
5 mH
  
X  (t) = e λ1t α
1 mF +
IS

400 V
Hence, the correct option is (A).

220 V

AC Question Number: 60 Question Type: MCQ


A second-order real system has the following pro-
perties:
(a) the damping ratio d = 0.5 and undamped
natural frequency wn = 10 rad/s
(b) the steady state value of the output, to a unit
IS
VS step input, is 1.02.
δ
The transfer function of the system is
VC1
ISXS 1.02 1.02
(A) (B)
2 2
S + 5S + 100 S + 10 S + 100
Xs = wL = 100p × 5 × 10–3
100 102
I x (C) (D)
sin d = sr s 2 S 2 + 5S + 100
Vs S + 10 S + 100

sin d = ( 22.73) (100p × 5 × 10–3)/Vs Solution:  Second order characteristic equation is
d = 9.21° S2 + 2dwnS + w2n = 0
Hence, the correct Answer is (9.21). Now we have d = 0.5 and wn = 10 rad/s. Substituting
Question Number: 59 Question Type: MCQ the values, we get the characteristic equation as
S2 + 10S + 100 = 0.
Consider a linear time invariant system x = Ax, with
initial condition x (0) at t = 0. Suppose a and b are 102
DC gain = = 1.02
eigenvectors of (2 × 2) matrix A corresponding to 100
distinct eigenvalues l1 and l2, respectively. Then the
Hence, the correct option is (B).
response x (t) of the system due to initial condition
x (0) = a is: Question Number: 61 Question Type: MCQ
(A) e λ1t α (B) e λ 2t β Three single-phase transformers are connected to from
(C) e λ 2t α (D) e λ1t α + e λ 2t β a delta-star three-phase transformer of 110 kV/11 kV.
The transformer supplies at 11 kV as load of 8 mW
Solution: at 0.8 p.f. lagging to a near by plant. Neglect the
x (t) eAt x (0) transformer losses. The ratio of phase currents in delta
side to star side is ________.
λ 0
A =  1  (A) 1 : 10 3 (B) 10 3 : 1
0 λ
 2
(C) 1 : 10 (D) 3 : 10
and x (0) = a
EAt = L–1 (SI – A) – 1 Solution:  D/Y connection [110 kV/11 kV]
Load on Y side
 λ1 1 
=  e λ2  11 kV, 8 mV, 0.8 p.f. log
 
0 e  ⇒ D side MVA rating is also 10 MVA

F03_Gate_EE_2016_978-93-325-7605-6_P-II.indd 122 6/19/2017 7:14:28 PM


GATE 2016 Solved Paper Electrical Engineering: Set – 2 | cxxiii

D side line current will be ( 2 − 1) ( 2 − 4)


10 mVA = 3 * 110 * 103 * IL
K=− =1
2
10 × 106 Hence, the correct option is (A).
IL = = 52.48 A
3 × 110 × 103 Question Number: 63 Question Type: NAT
52.48 Two identical unloaded generators are connected in
⇒ Iph = = 30.30 A parallel as shown in the figure. Both the generators are
3
having positive, negative and zero sequence impedances
Y side: of j0.4 pu., j0.3 pu., and j0.15 pu., respectively If the
10 MVA = 3 * 11 × 103 * IL pre-fault voltage is 1 pu., for a line-to-ground (L – G)
fault at the terminals of the generators, the fault current,
10 × 106 in pu., is _________.
⇒ IL = = 524.86 A
3
3 ∗ 11 × 10
⇒ Iph = IL = 524.86 A
I ph ( ∆) 30.30
The ratio = = = 0.0577
I ph (Y ) 524.86

1
=
10 3
Hence, the correct option is (A). Solution:  Two generators in parallel

Question Number: 62 Question Type: MCQ


The gain at the breakaway point of the root locus of a Z1 Z2
unity feedback system with open loop transfer function
KS
G (S) = is:
( S − 1) ( S − 4 ) G1 G2

(A) 1 (B) 2
(C) 5 (D) 9
* Equivalent positive sequence impedance.
Solution:  Open loop transfer function
(0.4)(0.4)
KS Z1eq = = 0.2 PU
G  (S) = 0.8
( S − 1) ( S − 4)

Characteristic equation is
1 + G (S) = 0 Z2 Z2
⇒ (S − 1) (S − 4) KS = 0
( S − 1) ( S − 4)
K=−
S * Equation negative sequence impedance.
Break away point occurs at a point (0.3)(0.3)
Z2eq = = 0.15 PU
dK 0.6
=0
dS
dK  S ( 2S − 5) − ( S 2 − 5S + 4) 
= −  Z0
dS  S2  Z0

2S2 – 5S = S2 – 5S + 4
S = ±2

F03_Gate_EE_2016_978-93-325-7605-6_P-II.indd 123 6/19/2017 7:14:32 PM


cxxiv | GATE 2016 Solved Paper Electrical Engineering: Set – 2

* Equation zero sequence impedance is Conductor is parallel to and at 1 m distance from the
Z0eq = 0.15 PU z-axis. The speed of the conductor in r.p.m. required to
( First generator neutral is disconnected form ground)
\ induce a voltage of 1 V across it, should be ________.
Eg z
\ Ia1 =
Z + Z 2eq + Z0eq
1eq
B
1.0
= = 2 PU
0.2 + 0.15 + 0.15 1m

If = 3Ia1 ( L – G fault)
\
1m
= (3) (2) = 6 PU
Hence, the correct Answer is (6).
Question Number: 64 Question Type: NAT Solution:
An energy meter, having meter constant of 1200 Flux density (B) = 1 Tesla
revolutions/kWh, makes 20 revolutions in 30 seconds Length l = 1 m
for a constant load. The load, in kW, is ________. Distance r = 1 m
Solution:  Energy meter constant, Now using the relation
rev E = Blv
K = 1200 1 =1×1×v
kWh
v = 1 m/s
Load power (in kW) v = rw
20 × 60 × 60 1 rw
P = = 2 kW
1200 × 30 w = 1

Hence, the correct Answer is (2). 2πN
= =1
Question Number: 65 Question Type: NAT 60
A rotating conductor of 1 m length is placed in a radially N = (60/2p) rpm
outward (about the z-axis) magnetic flux density (B) = 9.54 rpm
of 1 Tesla as shown in figure given on next column. Hence, the correct Answer is (9.54).

F03_Gate_EE_2016_978-93-325-7605-6_P-II.indd 124 6/19/2017 7:14:34 PM


Detailed Analysis of GATE 2015 Papers
GATE EE Solved 2015 Paper (Set 1) Detailed Analysis
Subject Topic 1 Mark 2 Marks Total Total
Questions Questions Questions Marks
General Aptitude Numerical Ability 2 4 6 10
Verbal Ability 3 1 4 5
Total Marks 15
Engineering Maths Linear Algebra 1 0 1 1
Probability and Distribution 0 1 1 2
Calculus 2 1 3 4
Differential Equation 0 1 1 2
Laplace Transform 0 1 1 2
Total Marks 11
Network Network Solution and Methodology 0 1 1 2
Sinusoidal Steady State Analysis 0 2 2 4
Laplace Transforms 0 1 1 2
Transient/Steady State Analysis of 1 0 1 1
RLC Circuit to DC Input
Total Marks 9
Signal & System Linear Time invariant (LTI) System 0 1 1 2
Fourier Representation of Signal 0 1 1 2
Z-Transform 0 1 1 2
Frequency Response of LTI System 0 1 1 2
Total Marks 8
Control System Time Domain Analysis 1 1 2 3
Root Locus Technique 0 1 1 2
Frequency Response Analysis 1 0 1 1
Nyquist Plot
State Variable Analysis 0 1 1 2
Total Marks 8
Analog Circuit Diode–Circuit–Analysis & Application 1 0 1 1
DC Biasing-BJTs 2 0 2 2
Feedback Oscillator Circuit 0 1 1 2
Operational Amplifier and Its 2 0 2 2
Application
Total Marks 7
Digital Electronics Boolean Algebra & KMap 1 0 1 1
Combinational Digital Circuit 0 1 1 2
Sequential Circuits 0 1 1 2
Total Marks 5

F04_Gate_EE_2015_978-93-325-7605-6_Detailed Analysis.indd 125 6/19/2017 7:15:25 PM


cxxvi | Detailed Analysis of GATE 2015 Papers

Electrical Machine Transformers 1 4 5 9


DC Machine 2 1 3 4
Induction Machines 1 1 2 3
Synchronous Machine 0 1 1 2
Total Marks 18
Power System Transmission and Distribution 0 1 1 2
Economics and Power Generation 2 2 4 6
Symmetrical Components & Fault
Calculation
Transient and Over Voltage 1 0 1 1
Total Marks 9
Power Electronics Phase Control Rectifier 0 1 1 2
Choppers 0 2 2 4
Total Marks 6
Electrical and Measurement of Basic Electrical 1 0 1 1
Electronic Quantities-1
Measurements
Measurement of Basic Electrical 1 1 2 3
Quantities-2
Total Marks 4

GATE EE Solved 2015 Paper (Set 2) Detailed Analysis


Subject Topic 1 Mark 2 Marks Total Total
Questions Questions Questions Marks

General Aptitude Numerical Ability 1 3 4 7

Verbal Ability 4 2 6 8

Total Marks 15

Engineering Maths Linear Algebra 1 1 2 3

Probability and Distribution 1 1 2 3

Calculus 0 1 1 2

Differential Equation 0 1 1 2

Total Marks 10

EMT Electric & Magnetic Field 2 1 3 4

Total Marks 4

Network Network Solution and Methodology 2 1 3 4

Sinusoidal Steady State Analysis 0 1 1 2

Laplace Transforms 1 0 1 1

Two-port Network 0 1 1 2

Total Marks 9

F04_Gate_EE_2015_978-93-325-7605-6_Detailed Analysis.indd 126 6/19/2017 7:15:25 PM


Detailed Analysis of GATE 2015 Papers | cxxvii

Signal & System Introduction to S&S 0 1 1 2

Linear Time Invariant (LTI) System 1 0 1 1

Z-Transform 0 1 1 2

Total Marks 5

Control System Basic of Control System 2 1 3 4

Time Domain Analysis 0 1 1 2

Root Locus Technique 0 1 1 2

Frequency Response Analysis 1 0 1 1


using Bode Plot

State Variable Analysis 0 1 1 2

Total Marks 11

Analog Circuit DC Biasing-BJTs 1 0 1 1

Operational Amplifier and its 2 1 3 4


Application

Total Marks 5

Digital Electronics Boolean Algebra & KMap 0 1 1 2

Combinational Digital Circuit 1 0 1 1

Sequential Circuits 0 1 1 2

AD/DA Convertor 0 1 1 2

Total Marks 7

Electrical Machine Transformers 2 2 4 6

DC Machine 1 2 3 5

Total Marks 11

Power System Transmission and Distribution 1 0 1 1

Economics and Power Generation 0 2 2 4


Symmetrical Components & Fault
Calculation

Power System and Stability 0 1 1 2

Generating Station 1 0 1 1

Transient and over voltage 0 1 2 2

Total Marks 10

Power Electronics Choppers 1 3 4 7

Application of Power Electronics 0 1 1 2

Total Marks 9

Electrical and Measurement of Basic Electrical 2 1 3 4


Electronic Quantities-1
Measurements

Total Marks 4

F04_Gate_EE_2015_978-93-325-7605-6_Detailed Analysis.indd 127 6/19/2017 7:15:26 PM


GATE 2015 Solved Paper
Electrical Engineering
Set – 1
Number of Questions: 65 Total marks: 100.0

Wrong answer for MCQ will result in negative marks, (-1/3) for 1 Mark Questions and (-2/3) for 2 Marks
Questions.

General Aptitude
Number of Questions: 10 Section Marks: 15.0

Q. 1 to Q. 5 carry 1 mark each and Q. 6 to Q. 10 carry (q + r) qr


2 marks each (A) (B)
qr (q + r)
Question Number: 1 Question Type: MCQ
(q + r )2
Out of the following four sentences, select the most (C) (q 2
)
+ r 2 (D)
qr
suitable sentence with respect to grammar and usage:
(A) Since the report lacked needed information, it Solution: Area of triangle
was of no use to them. PQR = Area of triangle PQS
(B) The report was useless to them because there + Area of triangle PSR
were no needed information in it.
Area of triangle PQR
(C) Since the report did not contain the needed
1
information, it was not real useful to them. = ( r )( q) sin ∠P
(D) Since the report lacked needed information, it 2
would not had been useful to them. 1
= ( r )( q) sin ( 2∠QPS )
Solution: Option (A) is free of all errors. Though, the 2
article ‘the’ before ‘needed’ would render it correct. rq
But, of the given options, option (A) is correct. The = sin 120°
2
use of ‘there were’ in option (B) is incorrect. ‘Real
useful’ in ungrammatical in option (C). ‘Not had been’ (∵ PS is the angle bisector of ∠QPR)
is ungrammatical in option (D). 1
\ = ( rq sin 120°)
Hence, the correct option is (A). 2
Question Number: 2 Question Type: MCQ 1 1
= [r ( PS ) sin 60°] + [q( PS ) sin 60°]
In a triangle PQR, PS is the angle bisector of ∠QPR 2 2
and ∠QPS = 60°. What is the length of PS ?  3 3
rq   = ( PS ) 2 ( r + q)
 2 
rq
PS =
r+q
Hence, the correct option is (B).
Question Number: 3 Question Type: NAT
If p, q, r, s are distinct integers such that:
f (p, q, r, s) = max (p, q, r, s)
g (p, q, r, s) = min (p, q, r, s)

F05_Gate_EE_2015_978-93-325-7605-6_P-I.indd 128 6/19/2017 7:16:39 PM


GATE 2015 Solved Paper Electrical Engineering: Set – 1 | cxxix

h (p, q, r, s) = remainder of (p × q)/(r × s) Question Number: 5 Question Type: MCQ


if (p × q) > (r × s) Four branches of a company are located at M, N, O and
or remainder of P. M is north of N at a distance of 4 km; P is south of
(r × s)/(p × q) if (r × s) > (p × q). O at a distance of 2 km; N is southeast of O by 1 km.
Also a function What is the distance between M and P in km?
fgh (p, q, r, s) = f (p, q, r, s) × g (p, q, r, s) (A) 5.34
(B) 6.74
× h (p, q, r, s). (C) 28.5
Also the same operations are valid with two variable (D) 45.49
functions of the form f (p, q). Solution: Line diagram:
What is the value of fg [h(2, 5, 7, 3), 4, 6, 8]?
Solution:
 21
h (2, 5, 7, 3) = remainder of   = 1
 10 
[∵ ( r × s) > ( p × q)]
fg [h (2, 5, 7, 3), 4, 6, 8]
= fg (1, 4, 6, 8)
= f (1, 4, 6, 8) × g (1, 4, 6, 8)
= max (1, 4, 6, 8) × min (1, 4, 6, 8)
=8×1=8 So, MP = (5) 2 + (1) 2 = 25 + 1 = 26
Hence, the correct Answer is (8). = 5.34 kms
Question Number: 4 Question Type: MCQ Hence, the correct option is (A).
If the list of letters, P, R, S, T, U is an arithmetic Question Number: 6 Question Type: MCQ
sequence, which of the following are also in arithmetic
sequence? We _______ our friend’s birthday and we _______ how
I. 2P, 2R, 2S, 2T, 2U to make it up to him.
II. P-3, R-3, S-3, T-3, U-3 (A) completely forgot—don’t just know
III. P2, R2, S2, T 2, U 2 (B) forgot completely—don’t just know
(A) I only (B) I and II (C) completely forgot—just don’t know
(C) II and III (D) I and III (D) forgot completely—just don’t know
Solution: P, R, S, T, U is an arithmetic sequence Solution: The correct answer is option (C). Here,
\ R – P = S – R = T – S = U – T. ‘completely’ modifies ‘forgot’ which is to say that an
Let each of these equal values be k. action was missed out on. The same rule applies to
I. 2 (R – P) = 2 (S – R) the second blank as well. ‘Don’t know’ come together,
= 2 (T – S) showing a misinformation and ‘just’ modifies it,
= 2 (U – T) showing an extent.
= 2k Hence, the correct option is (C).
\ 2P, 2R, 2S, 2T, 2U is an arithmetic sequence. Question Number: 7 Question Type: MCQ
II. R – 3 – (P – 3) = S – 3 – (R – 3)
Choose the statement where underlined word is used
= T – 3 – (S – 3) correctly.
= U – 3 – (T – 3) = k (A) The industrialist has a personnel jet.
\ P – 3, R – 3, S – 3, T – 3, U – 3 is an arithmetic (B) I write my experience in my personnel dairy.
sequence. (C) All personnel are being given the day off.
Hence, the correct option is (B). (D) Being religious is a personnel aspect.

F05_Gate_EE_2015_978-93-325-7605-6_P-I.indd 129 6/19/2017 7:16:40 PM


cxxx | GATE 2015 Solved Paper Electrical Engineering: Set – 1

Solution: The word ‘personnel’ means a group of (A) Statement I alone is not sufficient.
people who work for a company or an organization. (B) Statement II alone is not sufficient.
Option (C) uses the word correctly. The rest of the (C) Either statement I or II alone is sufficient.
options should use ‘personal’. (D) Both statements I and II together are not
Hence, the correct option is (C). sufficient.
Solution:
Question Number: 8 Question Type: MCQ
I. Weight of the pole = 20 kg
A generic term that includes various items of clothing
Total weight of 10 poles = (20) (10) kg
such as a skirt, a pair of trousers and a shirt is:
Statement I is sufficient.
(A) fabric (B) textile
(C) fibre (D) apparel II. Total weight of 8 poles = 160 kg
Total weight of 10 poles = (160) kg
Solution: The correct answer is option (D) apparel.
Statement II is sufficient.
Option (A) refers to the type of apparel. Option (B)
refers to the business that makes apparels. Option (C) Hence, the correct option is (C).
is again a material used to make an apparel. Question Number: 10 Question Type: MCQ
Consider a function f (x) = 1 – | x | on –1 ≤ x ≤ 1. The
Hence, the correct option is (D).
value of x at which the function attains a maximum and
Question Number: 9 Question Type: MCQ the maximum value of the function are:
Based on the given statements, select the most (A) 0, –1 (B) –1, 0
appropriate option to solve the given question. What (C) 0, 1 (D) –1, 2
will be the total weight of 10 poles each of same weight? Solution: f (x) is maximum when | x | is minimum, i.e.,
Statements: when | x | is zero i.e., when x is zero.
I. One fourth of the weight of a pole is 5 kg. Maximum value of f (x) = 1 – 0 = 1. Which occurs at
II. The total weight of these poles is 160 kg more x = 0.
than the total weight of two poles. Hence, the correct option is (C).

Electrical Engineering
Number of Questions: 55 Section Marks: 85.0

Q. 11 to Q. 35 carry 1 mark each and Q. 36 to Q. 65 Solution:


carry 2 marks each 1  1 
Zin = − jX c + 2 
jX L + 2 R 
Question Number: 11 Question Type: MCQ b  a 
Find the transformer ratios a and b such that the −j 1
impedance (Zin) is resistive and equals 2.5W when the Zin = +
(5000 × 10 × 10 −6 ) b2
network is excited with a sine wave voltage of angular
frequency of 5000 rad/s.  −6 1 
 j (5000)(10 × 10 ) + 2 ( 2.5) 
 a 
1  2.5 
Zin = − j 20 + 2 
j5 + 2 
b  a 
For Zin = 2.5W
= 2.5W (pure resistive)
j5
(A) a = 0.5, b = 2.0 − j 20 + 2 = 0 (reactance = 0)
(B) a = 2.0, b = 0.5 b
(C) a = 1.0, b = 1.0 1
b2 = and b = 0.5
(D) a = 4.0, b = 0.5 4

F05_Gate_EE_2015_978-93-325-7605-6_P-I.indd 130 6/19/2017 7:16:42 PM


GATE 2015 Solved Paper Electrical Engineering: Set – 1 | cxxxi

2.5 R2 (1S )
Zin = 2.5 Ω = 2 2
3I 22
a b = S
⇒ a2b2 = 1 ω
1 (0.05)(0.95)
a = = 2 3( 205.09) 2 ×
b = 0.05
2π × 2850
Hence, the correct option is (B). 60

Question Number: 12 Question Type: NAT Tg = 401.86 Nm
A shunt-connected DC motor operates at its rated Hence, the correct Answer is (400 to 403).
terminal voltage. Its no-load speed is 200 radian/second.
At its torque of 500 Nm, its speed is 180 radian/second. Question Number: 14 Question Type: MCQ
The motor is used to directly drive a load whose load A 4-pole, separately excited, wave wound DC machine
torque TL depends on its rotational speed wr (in radian/ with negligible armature resistance is rated for 230 V
second), such that TL = 2.78 × wr. Neglecting rotational and 5 kW at a speed of 1200 rpm. If the same armature
losses, the steady-state speed (in radian/second) of the coils are reconnected to form a lap winding, what is the
motor, when it drives this load, is ________. rated voltage (in volts) and power (in kW), respectively
Solution: At rated torque of 500 Nm with the relation at 1200 rpm of the reconnected machine if the field
TL = 2.78wr circuit is left unchanged?
T 500 (A) 230 and 5 (B) 115 and 5
wr = L = = 179.856 rad/sec (C) 115 and 2.5 (D) 230 and 2.5
2.78 2.78
Solution:
Hence, the correct Answer is (177 to 183).
ϕZN P
Question Number: 13 Question Type: NAT Bode emf Eb = ×
60 A
The figure shows the per-phase equivalent circuit of
a two-pole three-phase induction motor operating at 1
Eb ∝
50 Hz. The ‘air-gap’ voltage, Vg across the magnetizing A
inductance, is 210 V rms, and the slip, s, is 0.05. The Eb2 A1
torque (in Nm) produced by the motor is ________. =
Eb1 A2

Eb2
2( wave winding)
⇒ =
230 4(lap winding)
Eb2 = 115 V
In wave winding P = VI = 5 kW
ince number of parallel paths are doubled current also
S
Solution: Rotor current I2 doubles in lap winding
Vg V 
= P =   ( 2 I ) = 5 kW
2 2
 R2  2
 S  × X 2 Hence, the correct option is (B).

210 Question Number: 15 Question Type: NAT
= An open loop control system results in a response
12 + 0.222
of e–2t (sin 5t + cos 5t) for a unit impulse input. The
= 205.09 A DC gain of the control system is ________.
Gross torque Solution: Response
= Tg c (t) = e–2t (sin 5t + cos 5t)

F05_Gate_EE_2015_978-93-325-7605-6_P-I.indd 131 6/19/2017 7:16:45 PM


cxxxii | GATE 2015 Solved Paper Electrical Engineering: Set – 1

For unit impulse, gain


C ( s)
G (s) =
R( s)
(C)
5 ( S + 2)
= 2 2
+
( S + 2) + 5 ( S + 2) 2 + 5 2

S+7
G (s) =
( S + 2) 2 + 25

DC gain
S+7 (D)
lt G ( s) = s →
lt
s →0 0 ( S + 2) 2 + 25

7 7
= = = 0.241
4 + 25 29 Solution: Nuquist product of G1 (s) and G2 (s)
Hence, the correct Answer is (0.23 to 0.25). 1
= ×S =1
S
Question Number: 16 Question Type: MCQ
Nyquist plot
Nyquist plots of two functions G1 (s) and G2 (s) are
shown in figure.

Hence, the correct option is (B).


Question Number: 17 Question Type: NAT
The volume enclosed by the surface f (x, y) = ex over
the triangle bounded by the lines x = y; x = 0; y = 1 in
the xy plane is ______.
Solution: The volume enclosed by the surface
f (x, y) = ex
(A) over the triangle OAB is

(B)

F05_Gate_EE_2015_978-93-325-7605-6_P-I.indd 132 6/19/2017 7:16:48 PM


GATE 2015 Solved Paper Electrical Engineering: Set – 1 | cxxxiii

⇒ rs = 0.28 (1)
V= ∫∫ f ( x, y ) dx dy
P (TRTS) = P (TR) P (TS) = 0.18
O AB
⇒ (1 – r) (1 – s) = 0.18 (2)
1 y
P (HRTS) = P (HR) P (TS) = 0.30
= ∫ ∫ e x dx dy
⇒ r (1 – s) = 0.30 (3)
y =0 x =0
P (TRHS) = P (TR) P (HS) = 0.24
 y 1  ⇒ (1 – r) s = 0.24 (4)
∫  ∫
 x 
=e dx dy ⇒ s – rs = 0.24

y = 0  x = 0  ⇒ s – 0.28 = 0.24 ⇒ s = 0.52
 1
y From Equation (3),
= ∫ e  dy x r – rs = 0.30
 ⇒ r – 0.28 = 0.30 ⇒ r = 0.58

y =0  x =0
Substituting the values of r and s in Equation (2), we
1
∫ [ e − e ] dy
y 0 have
=
(1 – 0.58) (1 – 0.52) = 0.18
y =0
⇒ (0.42) (0.48) = 0.18
1 0.2016 = 0.18 which is a absurd.
= ∫ (e y − 1) dy Hence, the coin tosses are dependent.
y =0
Hence, the correct option is (D).
1 Question Number: 19 Question Type: NAT
= e y − y  0
di
1 0 A differential equation − 0.2i = 0 is applicable
= (e − 1) − (e − 0) dt
=e–2 over -10 < t ± 10. If i (4) = 10, then i (-5) is ______.
= 2.718 – 2 Solution: Given differential equation is
= 0.718 di
- 0.2i = 0 (1)
Hence, the correct Answer is (0.70 to 0.76). dt
Question Number: 18 Question Type: MCQ i (4) = 10 (2)
Two coins R and S are tossed. The 4 joint events HRHS, From Equation (1),
TRTS, HRTS, TRHS have probabilities 0.28, 0.18, 0.30, di 1
= 0.2i ⇒ di = 0.2dt
0.24, respectively, where H represents head and T dt   i
represents tail. Which one of the following is TRUE? Integrating on both sides
(A) The coin tosses are independent. 1
= ∫ di = ∫ 0.2dt
(B) R is fair, S is not. i
(C) S is fair, R is not. ⇒ ln i = 0.2t + c ⇒ i = e0.2t+c
(D) The coin tosses are dependent. ⇒ i = c1e0.2t where c1 = ec
Solution: When two coins R and S are tossed, given Given i (4) = 10 ⇒ 10 = c1e0.2 × 4
that the probabilities of the four joint events are ⇒ c1e0.8 = 10 ⇒ c1 = 10e-0.8
P (HRHS) = 0.28, P (TRTS) = 0.18, P (HRTS) = 0.30 and Substituting the value of c1 in Equation (3), we get
P (TRHS) = 0.24. i = 10e–0.8 e0.2t
Clearly, the coin tosses can’t be independent i = 10e0.2t – 0.8
For, let P (HR) = r and P (HS) = s Now i (–5) = 10e0.2 × (–5) – 0.8 = 10e–1.8
⇒ P (TR) = 1 – r and P (TS) = 1 – s 10
= = 1.653
If the coin tosses are independent, then e1.8
P (HRHS) = P (HR) P (HS) = 0.28 Hence, the correct Answer is (1.6 to 1.7).

F05_Gate_EE_2015_978-93-325-7605-6_P-I.indd 133 6/19/2017 7:16:51 PM


cxxxiv | GATE 2015 Solved Paper Electrical Engineering: Set – 1

Question Number: 20 Question Type: MCQ


Consider a signal defined by
 j10t for t ≤ 1
x (t) =  e
 0 for t > 1
Its Fourier Transform is:
2 sin (ω − 10) sin (ω − 10)
(A) (B) 2e j10 Solution:
ω − 10 ω − 10
Wattmeter error due to pressure coil
2 sin ω 2 sin ω
(C) (D) e j10ω V2 30 2
ω − 10 ω = = = 0.9
R 1000
Solution:
Wattmeter reading = VI cos q
 j10t for t ≤ 1
Given x (t) =  e = 30 × 25 × 0.8 = 600 W
 0 for t > 1 0.9
% error = × 100 = 0.15%
⇒ The Fourier Transform of x (t) is 600
∞ 1 Hence, the correct Answer is (0.14 to 0.16).
F [x (t)] = ∫ x (t ) e − j ωt dt = ∫e
j10t − j ωt
e dt Question Number: 22 Question Type: NAT
−∞ −1 A buck converter feeding a variable resistive load is
1 shown in the figure. The switching frequency of the
e(10 − ω) jt 
1
(10 − ω) jt
= ∫e dt =
(10 − ω) j 
 switch S is 100 kHz and the duty ratio is 0.6. The output
voltage VO is 36 V. Assume that all the components are
−1  −1
ideal, and that the output voltage is ripple-free. The
e(10 − ω) j e −(10 − ω) j value of R (in Ohm) that will make the inductor current
= − (iL) just continuous is _______.
(10 − ω) j (10 − ω) j

2  e(10 − ω) j − e −(10 − ω) j 
=  
(10 − ω)  2j 

−2  e( ω −10) j − e( ω −10) j 
=  
(ω − 10)  2j 

2  e( ω −10) j − e −( ω −10) j 
=   Solution: The inductor ripple current,
(ω − 10)  2j 
V (V − Vo )
∆I = o s
2 fLVs
= sin (ω − 10).
(ω − 10) 36 (60 − 36)
∆I =
3 3
Hence, the correct option is (A). 100 × 10 × 5 × 10 × 60
Question Number: 21 Question Type: NAT ∆I = 0.0288 A
The coils of a wattmeter have resistances 0.01Ω and The average inductor current,
1000Ω; their inductances may be neglected. The ∆I
IL = = 0.0144 A
wattmeter is connected as shown in the figure, to 2
measure the power consumed by a load, which draws
V 36
25 A at power factor 0.8. The voltage across the load IL = o = ;  R = 2500Ω
terminals is 30 V. The percentage error on the wattmeter R R
reading is ______. Hence, the correct Answer is (2480 to 2520).

F05_Gate_EE_2015_978-93-325-7605-6_P-I.indd 134 6/19/2017 7:16:55 PM


GATE 2015 Solved Paper Electrical Engineering: Set – 1 | cxxxv

Question Number: 23 Question Type: NAT Solution: Average output voltage, (Vo)avg
For the switching converter shown in the following π
1
2π ∫ m
figure, assume steady-state operation. Also assume that = V sin ωt d ωt
the components are ideal, the inductor current is always ∝

positive and continuous and switching period is Ts. If 1
the voltage VL is as shown, the duty cycle of the switch
+
2π ∫ −Vm sin ωt d ωt
π
S is _____.
V V
(Vo)avg = m [1 + cos α] − m [cos π − cos 2π]
2π 2π
V 100
(Vo)avg = m (3 + cos α) = [3 + cos 30]
2 π 2π
(Vo)avg = 61.56 V
Hence, the correct Answer is (61 to 62).
Question Number: 25 Question Type: MCQ
For linear time invariant systems, that are Bounded
Input Bounded Output stable, which one of the
following statements is TRUE?
(A) The impulse response will be integrable, but
may not be absolutely integrable.
(B) The unit impulse response will have finite
Solution: For continuous conduction, the average support.
inductor voltage (C) The unit step response will be absolutely
VL = 0 integrable.
T T (D) The unit step response will be bounded.
VL = 15 × ON + ( −45) OFF = 0
TS TS Solution: Unit step response will not be absolutely
15TON = 45TOFF integrable but it will be bounded.
Hence, the correct option is (D).
TOFF 1 T + TON 4
=⇒ OFF = Question Number: 26 Question Type: MCQ
TON 3 TON 3
The z-transform of a sequence x [n] is given as
TON
Duty ratio, d= = 0.75 X (z) = 2z + 4 – 4/z + 3/z2. If y [n] is the first difference
T of x [n], then Y (z) is given by
Hence, the correct Answer is (0.75). (A) 2z + 2 – 8/z + 7/z2 – 3/z3
Question Number: 24 Question Type: NAT (B) –2z + 2 – 6/z + 1/z2 + 3/z3
In the given rectifier, the delay angle of the thyristor T1 (C) –2z – 2 + 8/z – 7/z2 + 3/z3
measured from the positive going zero crossing of Vs (D) 4z – 2 – 8/z – 1/z2 + 3/z3
is 30°. If the input voltage Vs is 100 sin (100πt) V, the Solution:
average voltage across R (in Volt) under steady-state is
y (n) is first difference of
_____.
X (n) = x (n) – x (n – 1)
Y (z) = X (z) – Z –1 x (z)
Y (z) = [2z + 4 – 4Z–1 + 3Z –2]
– [2 + 4z–1 – 4z–2 – 3z–3]
8 7 3
Y (z) = 2z + 2 − + −
z z 2 z3
Hence, the correct option is (A).

F05_Gate_EE_2015_978-93-325-7605-6_P-I.indd 135 6/19/2017 7:16:58 PM


cxxxvi | GATE 2015 Solved Paper Electrical Engineering: Set – 1

Question Number: 27 Question Type: MCQ


Two semi-infinite conducting sheets are placed at right
angles to each other as shown in the figure. A point
charge of +Q is placed at a distance of d from both
Q2 K
sheets. The net force on the charge is where
4π∈0 d 2
K is given by:

(A) 00 (B) 01
(C) 10 (D) 11
Solution: Johnson counter (MOD-4), number of cycles
completed
333
= = 83
4
Remainder clock is 1.

J1 (Q0) K1 (Q0 ) J 0 (Q1 ) K 0 (Q1 ) Q1 Q0


(A) 0 — — — — 0 0
1 1 0 1 1 0 0 1
(B) − i − j
4 4 1 0 1 0 1 1
1 1 1 0 0 1 1 0
(C) − i − j
8 8 0 1 0 1 0 0
1− 2 2 1− 2 2 o, initial output at Q1 Q0 – 00, the next output is 01.
S
(D) i+ j
8 2 8 2 Hence, the correct option is (B).
Solution: The net force on the charge is Question Number: 29 Question Type: MCQ
F = F1 A Boolean function f (A, B, C, D) = ∏ (1, 5, 12, 15)
1 Q2 is to be implemented using an 8 × 1 multiplexer
F= (A is MSB). The inputs ABC are connected to the select
4 π ∈o ( 2d )2
inputs S2 S1 S0 of the multiplexer, respectively.
 1 
 −2dax − 2day + ( 2dax + 2day ) 
 2 2 
1 Q2 1 − 2 2 1− 2 2 
F=  ax + ay 
4 π ∈o d 2  8 2 8 2 

1 Q2
F= ⋅ (k )
4 π ∈o d 2

Hence, the correct option is (D).
Which one of the following options gives the correct
Question Number: 28 Question Type: MCQ inputs to pins 0, 1, 2, 3, 4, 5, 6, 7 in order?
In the following sequential circuit, the initial state (A) D, 0, D, 0, 0, 0, D, D
(before the first clock pulse) of the circuit is Q1 Q0 = 00. (B) D, 1, D, 1, 1, 1, D, D
The state (Q1Q0), immediately after the 333rd clock (C) D, 1, D, 1, 1, 1, D, D
pulse is ________. (D) D, 0, D, 0, 0, 0, D, D

F05_Gate_EE_2015_978-93-325-7605-6_P-I.indd 136 6/19/2017 7:17:01 PM


GATE 2015 Solved Paper Electrical Engineering: Set – 1 | cxxxvii

Solution: Boolean function Question Number: 31 Question Type: MCQ


f (A, B, C, D) = π (1, 5, 12, 15) Given f (z) = g (z) + h (z), where f, g, h are complex
The min term function valued functions of a complex variable z. Which one of
f (A, B, C, D) = Σm (0, 2, 3, 4, 6, 7, the following statements is TRUE?
8, 9, 10, 11, 13, 14) (A) If f (z) is differentiable at z0, then g (z) and h (z)
are also differentiable at z0.
D I0 I1 I2 I3 I4 I5 I6 I7 (B) If g (z) and h (z) are differentiable at z0, then
0 0 2 4 6 8 10 12 14 f (z) is also differentiable at z0.
(C) If f (z) is continuous at z0, then it is differenti-
1 1 3 5 7 9 11 3 15
able at z0.
D 1 D 1 1 1 D D (D) If f (z) is differentiable at z0, then so are its real
and imaginary parts.
Hence, the correct option is (B).
Solution: We know that every continuous function
Question Number: 30 Question Type: MCQ need NOT be differentiable
The saturation voltage of the ideal op-amp shown \ Option (C) is NOT TRUE
below is ±10 V. The output voltage v0 of the following
Counter Example for option (A):
circuit in the steady-state is:
Let g (z) = 2x + i3y and h (z) = 3x + i2y
⇒ f (z) = g (z) + h (z)
= (2x + i3y) + (3x + i2y)
= 5x + i5y
= 5 (x + iy)
= 5z, where z = x + iy
It can be easily observed that g (z) and h (z) does not
satisfy Cauchy-Riemann equations.
But f (z) is differentiable.
So, option (A) is NOT TRUE.
Option (D) is also NOT TRUE.
We know that the sum of two differentiable functions is
(A) square wave of period 0.55 ms. always differentiable.
(B) triangular wave of period 0.55 ms. Hence, the correct option (B) is TRUE.
(C) square wave of period 0.25 ms.
Question Number: 32 Question Type: MCQ
(D) triangular wave of period 0.25 ms.
We have a set of 3 linear equations in 3 unknowns.
Solution:
‘X ≡ Y’ means X and Y are equivalent statements and
R2 2 ‘X ≡/ Y ’ means X and Y are not equivalent statements.
b= = = 0.5
R1 + R2 4 P. There is a unique solution.

Time period, Q. The equations are linearly independent.
R. All eigen values of the coefficient matrix are
1 + β  non-zero.
T = 2RC log  
1 − β  S. The determinant of the coefficient matrix is
non-zero.
 1 + 0.5 
T = 2 × 1 × 103 × 0.25 × 10–6 log   Which one of the following is TRUE?
 1 − 0.5 
(A) P ≡ Q ≡ R ≡ S (B) P ≡ R ≡/ Q ≡ S
T = 0.55 msec (C) P ≡ Q ≡/ R ≡ S (D) P ≡/ Q ≡/ R ≡/ S
Astable multivibrator producer square wave with time Solution: All the four statements P, Q, R and S are
period 0.55 msec. equivalent.
Hence, the correct option is (A). Hence, the correct option is (A).

F05_Gate_EE_2015_978-93-325-7605-6_P-I.indd 137 6/19/2017 7:17:02 PM


cxxxviii | GATE 2015 Solved Paper Electrical Engineering: Set – 1

Question Number: 33 Question Type: MCQ Question Number: 35 Question Type: MCQ
Match the following: Match the following:
P. Stoke’s Theorem ∫∫ D ⋅ ds = Q
 1.
Instrument Type Used for
Q. Gauss’s Theorem 2. ∫ f ( z ) dz = 0 P. Permanent magnet moving coil 1. DC only

Q. 
Moving iron connected through 2. AC only
R. Divergence Theorem 3. ∫∫∫ (∇ ⋅ A) dv
current transformer
= ∫∫ A ⋅ ds R. Rectifier 3. AC and
DC
S. Cauchy’s Integral 4. ∫∫ (∇ × A) ⋅ ds
S. Electrodynamometer
Theorem
     ∫
= A ⋅ dl (A) P-1  Q-2  R-1  S-3

(A) P–2  Q–1  R–1  S–3 (B) P-1  Q-3  R-1  S-2
(B) P–4  Q–1  R–3  S–2 (C) P-1  Q-2  R-3  S-3
(C) P–4  Q–3  R–1  S–2 (D) P-3  Q-1  R-2  S-1
(D) P–3  Q–4  R–2  S–1 Solution: PMMC meter can show deflection for only
Solution: Correct matching is P–4, Q–1, R–3, S–2. dc and for ac average torque produced will be zero.
Hence, the correct option is (B). Moving iron instrument along with CT can be used
Question Number: 34 Question Type: MCQ only for AC. Rectifier instrument converter AC to DC
and can be used for both AC and DC.
The Laplace transform of f (t) = 2 t/π is s-3/2. The
Hence, the correct option is (C).
Laplace transform of g (t) = 1/πt is:
(A) 3s–5/2/2 Question Number: 36 Question Type: MCQ
(B) s-1/2 A three-phase balanced load which has a power factor
(C) s1/2 of 0.707 is connected to a balanced supply. The power
(D) s3/2 consumed by the load is 5 kW. The power is measured
by the two-wattmeter method. The readings of the two
Solution: wattmeters are:
−3 (A) 3.94 kW and 1.06 kW
t
Given f (t) = 2 and L [ f (t )] = s 2
(B) 2.50 kW and 2.50 kW
π
(C) 5.00 kW and 0.00 kW
1 1 1 1 (D) 2.96 kW and 2.04 kW
⇒ f  1 (t) = 2 ⋅ ⋅ = =
2 t π πt πt Solution:
\ f  1(t) = g (t) otal power consumed
T
We know that W1 + W2 = 5 kW (1)
L [  f  1(t)] = sL [ f (t)] – f  (0) Power factor = cos f = cos 45° = 0.707
 1  1 π  3 (W1 − W2 ) 
\ L [g (t)] = L   = L [ f (t )] = = φ = tan −1  
 π t  4  W1 + W2 

( )−2
−3
5
W1 – W2 = (2)
0 3
=s s2
π
Solving Equations (1) and (2)
−1 −1
W1 = 3.94 kW
= s2 −0= s2
and W2 = 1.06 kW
Hence, the correct option is (B). Hence, the correct option is (A).

F05_Gate_EE_2015_978-93-325-7605-6_P-I.indd 138 6/19/2017 7:17:06 PM


GATE 2015 Solved Paper Electrical Engineering: Set – 1 | cxxxix

Question Number: 37 Question Type: NAT


A capacitive voltage divider is used to measure the
bus voltage Vbus in a high-voltage 50 Hz AC system
as shown in the figure. The measurement capacitors
C1 and C2 have tolerances of ±10% on their nominal
capacitance values. If the bus voltage Vbus is 100 kV
rms, the maximum rms output voltage Vout (in kV),
considering the capacitor tolerance, is ______.
During negative half cycle of the supply under
steady-state.

Solution:
C1 Average voltage across resistance |(VR)avg |
Vout = Vbus ×
C1 + C2 = 200 V
For maximum Hence, the correct option is (C).
(1 + 10%) Question Number: 39 Question Type: MCQ
Vout = 100 ×
(1 + 10%) + (9 − 10%) Two semi-infinite dielectric regions are separated by

a plane boundary at y = 0. The dielectric constants
1.1 of region 1 (y < 0) and region 2 (y > 0) are 2 and 5,
= 100 × = 11.95 kV
(1 . 1 + 8.1) respectively.
→ Region 1 has uniform electric field
Hence, the correct Answer is (11.75 to 12.25). E = 3aˆ y + 2aˆ z , where aˆ x , aˆ y and âz are unit vectors
Question Number: 38 Question Type: MCQ along the x, y and z axes, respectively. The electric field
in region 2 is:
In the following circuit, the input voltage Vin is
100 sin (100π t). For 100π RC = 50, the average voltage (A) 3aˆ x + 1.6 aˆ y + 2aˆ z
across R (in Volts) under steady-state is nearest to: (B) 1.2 aˆ x + 4 aˆ y + 2aˆ z
(C) 1.2 aˆ x + 4 aˆ y + 0.8aˆ z
(D) 3 aˆ x + 10 aˆ y + 0.8aˆ z
Solution: The electric field in region 1 is
E1 = 3ax + 4ay + 2az
The electric field in region 2 is
2
E2 = 3ax + ( 4 a y ) + 2az
4
E2 = 3ax + 1.6ay + 2az
(A) 100
(B) 31.8 Hence, the correct option is (A).
(C) 200 Question Number: 40 Question Type: NAT
(D) 63.6 A circular turn of radius 1 m revolves at 60 rpm about
Solution: During positive half cycle of the supply its diameter aligned with the x-axis as shown in the
under steady-state. figure. The value of µ0 is 4π × 10–7 in SI unit. If a

F05_Gate_EE_2015_978-93-325-7605-6_P-I.indd 139 6/19/2017 7:17:09 PM


cxl | GATE 2015 Solved Paper Electrical Engineering: Set – 1


uniform magnetic field intensity H = 107 zˆ A/m is (1 × 103 )
1
applied, then the peak value of the induced voltage, (0.1 × 10 −6 S )
Vturn (in Volts), is______. Z2 =
1
103 +
(0.1 × 10 −6 S )

103
=
3
1 + j 0.1 × 10 ω
for ω = 2π × 20ω rad/sec
103
Z2 =
1 + j 0.4 π
103 103
Z2 | =
and |  =
Solution: Magnetic flux density by, 1.6
1 + (0.4 π)2
B = µo H = 4π × 10-7 × 107 âz
= 4π âz  103 
 
Flux, f = BA = 4π × πr2 sin ωt  1.6 
2
Vo | = 2 ×
|  = ≅ 1.25
3 1.6
−d φ 1 × 10
V= = 4π2 r2 (–cos ωt) ω
dt Hence, the correct Answer is (1.1 to 1.4).
2πN Question Number: 42 Question Type: NAT
|V| = 4 π2 r 2 ω = 4 π2 r 2
60 The following circuit, the transistor is in active mode
2π × 60 1
and VC = 2 V. To get VC = 4 V, we replace RC with RC .
= 4 π2 × 12 ×
60 Then the ratio RC1/RC is ________.
= 8π2 = 248.05 V
Hence, the correct Answer is (246 to 250).
Question Number: 41 Question Type: NAT
The operational amplifier shown in the figure is ideal.
The input voltage (in Volt) is Vi = 2 sin (2π × 2000t).
The amplitude of the output voltage Vo (in Volt) is
________.

Solution:
10 − Vc 10 − 2
For Vc = 2 V, Rc = = (1)
Ic Ic

10 − Vc 10 − 4
For Vc = 4 V, Rc1 = = (2)
Ic Ic
Solution: Output voltage of inverting amplifier
( −Z2 ) ( 2)
=
Rc16
= = 0.75
Vo = Vi ×
Z1 (1) Rc 8

Z1 = 1 × 102 Ω Hence, the correct Answer is (0.74 to 0.76).

F05_Gate_EE_2015_978-93-325-7605-6_P-I.indd 140 6/19/2017 7:17:13 PM


GATE 2015 Solved Paper Electrical Engineering: Set – 1 | cxli

Question Number: 43 Question Type: MCQ


Consider the following sum of products expression, F.
F = ABC + ABC + ABC + ABC + ABC
The equivalent product of sums expression is
(A) F = ( A + B + C ) ( A + B + C ) ( A + B + C )
(B) F = ( A + B + C ) ( A + B + C ) ( A + B + C )
(C) F = ( A + B + C ) ( A + B + C ) ( A + B + C )
(D) F = ( A + B + C ) ( A + B + C ) ( A + B + C ) Hence, the correct option is (B).
Solution: K-map for the expression Question Number: 45 Question Type: MCQ
F = ABC + ABC + ABC + ABC + ABC When a bipolar junction transistor is operating in the
The equivalent POS form is saturation mode, which one of the following statements
= ( A + B + C) ( A + B + C) ( A + B + C) is TRUE about the state of its collector-base (CB) and
the base-emitter (BE) junctions?
BC (A) The CB junction is forward biased and the BE
A junction is reverse biased.
00 01 10 11
(B) The CB junction is reverse biased and the BE
0 1 1 0 1 junction is forward biased.
1 0 1 0 1 (C) Both the CB and BE junctions are forward
biased.
Hence, the correct option is (A).
(D) Both the CB and BE junctions are reverse
Question Number: 44 Question Type: MCQ biased.
The filters F1 and F2 having characteristics as shown
Solution: BJT operates in saturation mode when both
in Figures (a) and (b) are connected as shown in Fig-
the junctions are in forward biased mode, operates in
ure (c).
cut-off mode when both the junctions are in reverse
biased mode and active mode when one of the junction
in forward biased mode and other one in reverse biased
mode.
Hence, the correct option is (C).
Question Number: 46 Question Type: MCQ
The synchronous generator shown in the figure is
supplying active power to an infinite bus via two short,
lossless transmission lines, and is initially is steady-
state. The mechanical power input to the generator and
the voltage magnitude E are constant. If one line is
The cut-off frequencies of F1 and F2 are f1 and f2, tripped at time t1 by opening the circuit breakers at the
respectively. If f1 < f2, the resultant circuit exhibits the two ends (although there is no fault), then it is seen that
characteristics of a: the generator undergoes a stable transient. Which one
(A) band-pass filter of the following waveforms of the rotor angle δ shows
(B) band-stop filter the transient correctly?
(C) all pass filter
(D) high-Q filter
Solution: Filter F1 is a low pass filter and filter F2 is
high pass filter and addition of two filters gives band
stop filter ( f1 < f2).

F05_Gate_EE_2015_978-93-325-7605-6_P-I.indd 141 6/19/2017 7:17:15 PM


cxlii | GATE 2015 Solved Paper Electrical Engineering: Set – 1

y33 = –j8 = y30 + y31 + y32


–j8 = y30 + (–j4) + (–j5)
y30 = j1
(A) j1
Compensation of y30 = = j 0.5
2
y33(new) = j0.5 – j4 – j5 = –j8.5
Hence, the correct option is (B).
Question Number: 48 Question Type: MCQ
(B) A series RL circuit is excited at t = 0 by closing a
switch as shown in the figure. Assuming zero initial
d 2t
conditions, the value of at t = 0+ is
2
dt

(C)

V −V
(A) (B)
(D) L R
−RV
(C) 0 (D) 2
L
Solution: For alternator rotor angle is positive, after Solution:
fault occurring this rotor angle increases. di
By applying KVL: V = iR + L
Hence, the correct option is (A). dt
Question Number: 47 Question Type: MCQ By solving above equation:
A 3-bus power system network consists of 3 transmis- V
i (t) = (1 − e −t/τ )
sion lines. The bus admittance matrix of the uncompen- R
 − j6 j3 j 4  Differentially above equation,
sated system is  j 3 − j 7 j 5  pu. di (t ) V 1 −t/τ V −t/τ
= × e = e
 j 4 j 5 − j8  dt R τ L

If the shunt capacitance of all transmission lines is and 2nd order differentially,
50% compensated, the imaginary part of the 3rd row di 2 (t )
−V 1 −t/τ
3rd column element (in pu) of the bus admittance = × e 2
matrix after compensation is: dt L τ
(A) –j7.0 (B) –j8.5 −VR −t/τ
= e
(C) –j7.5 (D) –j9.0 2
L
Solution:
d 2 i (t ) −VR
 y11 − y12 − y13  =
2


YBus =  − y21 y22 − y23 
 dt t = 0 L2

− y − y32 y33 
 31 Hence, the correct option is (D).

F05_Gate_EE_2015_978-93-325-7605-6_P-I.indd 142 6/19/2017 7:17:19 PM


GATE 2015 Solved Paper Electrical Engineering: Set – 1 | cxliii

Question Number: 49 Question Type: NAT For the total cost minimization
The current i (Ampere) in the 2Ω resistor of the given dC dC2
l= 1 =
network is ______. dP1 dP2

0.2P1 + 50 = 0.24P2 + 40 (2)
0.2 (250 – P2) + 50 = 0.24P2 + 40
P2 = 136.36 MW
Hence, the correct Answer is (135 to 137).
Question Number: 51 Question Type: NAT
A composite conductor consists of three conductors of
radius R each. The conductors are arranged as shown
Solution: The network can be redrawn as below. The geometric mean radius (GMR) (in cm)
of the composite conductor is kR. The value of k is
______.

For balanced bridge i = 0 A.


Question Number: 50 Question Type: NAT
The incremental costs (in Rupees/MW/h) of operating
two generating units are functions of their positive
powers P1 and P2 in MW, and are given by
dC1
= 0.2P1 + 50 Solution: For symmetrical spacing,
dP1
GMR = GMR1 GMR 2 GMR 3
dC2
= 0.24P2 + 40 = GMR1 = 3 0.7788 R × 3R × 3R
GMR
dP2
where GMR = KR = ( 3 0.7788 × 3 × 3 ) R
20 MW ≤ P1 ≤ 150 MW K = 1.913
20 MW ≤ P2 ≤ 150 MW. Hence, the correct Answer is (1.85 to 1.95).
For a certain load demand, P1 and P2 have been Question Number: 52 Question Type: MCQ
chosen such that dC1/dP1 = 76 Rupees/MW/h and A 3-k phase transformer rated for 33 kV/11 kV is
dC2/dP2 = 68.8 Rupees/MW/h. If the generations connected in delta/star as shown in figure. The current
are rescheduled to minimize the total cost, then P2 is transformers (CTs) on low and high voltage sides have
______. a ratio of 500/5. Find the currents i1 and i2, if the fault
Solution: The incremental fuel cost equations are current is 300 A as shown in figure.
dc
1
= 76 = 0.2P1 + 50 ⇒ P1 = 120
dp
1
dC2
= 68.8 = 0.24P2 + 40
dP2

⇒ P2 = 130
and P1 + P2 = 250 (1)

F05_Gate_EE_2015_978-93-325-7605-6_P-I.indd 143 6/19/2017 7:17:21 PM


cxliv | GATE 2015 Solved Paper Electrical Engineering: Set – 1

(A) i1 = 1/ 3 A, i2 = 0 A 3 × 230 × I p = 3 × 115 × 100


(B) i1 = 0 A, i2 = 0 A
(C) i1 = 0 A, i2 = 1/ 3 A Ip = 50 A
(D) i1 = 1 3 A, i2 = 1/ 3 A Line current star primary leads line current of ∆ by 30°.
Solution: Since fault occurred on secondary side and Hence, the correct option is (A).
outside of the CT connected winding, i2 = 0 A. Question Number: 54 Question Type: MCQ
Current through secondary of the transforms In the given network V1 = 100 ∠0° V, V2 = 100 ∠–120° V,
5 V3 = 100 ∠+120° V. The phasor current i (in Ampere)
IS = 300 × =3A is:
500
Current through primary of the transformer
11
= IP = 3 × =1A
33
Ip
Phase current Iph =
3
1
i1 = I ph = A
3
(A) 173.2 ∠–60° (B) 173.2 ∠120°
Hence, the correct option is (A). (C) 100.0 ∠–60° (D) 100.0 ∠120°
Question Number: 53 Question Type: MCQ V3 − V2 V3 − V1
A balanced (positive sequence) three-phase AC voltage Solution: Applying KCL: i + + 0
j1 − j1
source is connected to a balanced, star connected load
through a star-delta transformer as shown in the figure. V2 − V3 V1 − V3
i= +
The line-to-line voltage rating is 230 V on the star side, j1 −j

and 115 V on the delta side. If the magnetizing current
is neglected and I s = 100 ∠0° A, then what is the value
of I p in Ampere?

(100 ∠−120°) − (100 ∠120°)


i=
j1

(100 ∠0°) − (100 ∠120°)
+
− j1
i = 173.2 ∠120°
(A) 50 ∠30° (B) 50 ∠–30°
Hence, the correct option is (B).
(C) 50 3 ∠30° (D) 200 ∠30°
Question Number: 55 Question Type: NAT
Solution: In transformer
(KVA)primary = (KVA)secondary A symmetrical square wave of 50% duty cycle has
amplitude of ±15 V and time period of 0.4π ms. This
3V p I p = 3Vs I s square wave is applied across a series RLC circuit with

F05_Gate_EE_2015_978-93-325-7605-6_P-I.indd 144 6/19/2017 7:17:25 PM


GATE 2015 Solved Paper Electrical Engineering: Set – 1 | cxlv

R = 5Ω, L = 10 mH, and C = 4 µF. The amplitude of Solution: Primary current


the 5000 rad/s component of the capacitor voltage = I1 = I 2 + I 3
(in Volt) is _______. 50 50
Solution: I1 = 2 × ∠30° + 2 × ∠150°
100 100
I1 = 1 ∠30° + 1 ∠150° = 1 ∠90°

Hence, the correct option is (A).
Question Number: 58 Question Type: NAT
With an armature voltage of 100 V and rated field
winding voltage, the speed of a separately excited
DC motor driving a fan is 1000 rpm, and its armature
current s 10 A. The armature resistance is 1Ω. The load
Hence, the correct Answer is (190 to 192 V.)
torque of the fan load is proportional to the square of
Question Number: 56 Question Type: NAT the rotor speed. Neglecting rotational losses, the value
Two identical coils each having inductance L are placed of the armature voltage (in Volt) which will reduce the
together on the same core. If an overall inductance of rotor speed to 500 rpm is ______.
αL is obtained by interconnecting these two coils, the Solution:
minimum value of α is _____. Back emf Eb1 = V – R⋅R = 100 – 10 × 1 = 90 V
Solution: Torque, T a N 2, hence Io a N 2
Overall inductance = Leq = L1 + L2 ± 2M 2
I2  N2 
    = L + L ± 2k LL = 
I1  N1 
Leq is minimum value when k = 1 and two coils are
opposing. 2
 500 
Leq = L + L – 2L = 0 ⇒ I2 = 10   = 2.5 A
 1000 
Hence, the correct Answer is (0).
Back emf Eb a N
Question Number: 57 Question Type: MCQ Eb
2 =
N2
A three-winding transformer is connected to an ⇒
Eb N1
AC voltage source as shown in the figure. The number 1

of turns are as follows: N1 = 100, N2 = 50, N3 = 50. If  500 


the magnetizing current is neglected, and the currents in ⇒ Eb = 90 ×  
two windings are I 2 = 2 ∠30° A and I 3 = 2 ∠150° A,
2
 1000 
then what is the value of the current in Ampere? Eb = 45 V
2

V = Eb + I 2 R = 45 + 2.5 × 1
2

= 47.5 V
Hence, the correct Answer is (47.5 V).
Question Number: 59 Question Type: NAT
A three-phase, 11 kV, 50 Hz, 2 pole, stars connected,
cylindrical rotor synchronous motor is connected to
an 11 kV, 50 Hz source, its synchronous reactance is
50Ω per phase, and its stator resistance is negligible.
The motor has a constant field excitation. At a particular
load torque, its stator current is 100 A at unity power
factor. If the load torque is increased so that the stator
(A) 1 ∠90° (B) 1 ∠270° current is 120 A, then the load angle (in degrees) at this
(C) 4 ∠90° (D) 4 ∠270° load is ________.

F05_Gate_EE_2015_978-93-325-7605-6_P-I.indd 145 6/19/2017 7:17:29 PM


cxlvi | GATE 2015 Solved Paper Electrical Engineering: Set – 1

Solution:
Find excitation
11
Ef  = V − I a × R × 103 − j (100 × 50)
3
|Ef | = 8082.23 V

(IaR)2 = E 2f + V 2 − 2 E f V cos δ
(120 × 50)2 = (8082.23)2 + (6350)2
– 2 (8082.23) (6350) cos d
d = -47.270
Hence, the correct Answer is (–48 to –46°).
Question Number: 60 Question Type: NAT The phase difference (in degree) between voltages V1
A 220 V, 3-phase, 4-pole, 50 Hz induction motor of and V2 is ______.
wound rotor type is supplied at rated voltage and Solution: The transformer secondaries are connected
frequency. The stator resistance, magnetizing reactance, in Y and ∆ hence phase angle difference between them
and core loss are negligible. The maximum torque is 30°.
produced by the rotor is 225% of full load torque and
Hence, the correct Answer is (30).
it occurs at 15% slip. The actual rotor resistance is
0.03Ω/phase. The value of external resistance (in Ohm) Question Number: 62 Question Type: MCQ
which must be inserted in a rotor phase if the maximum The following discrete-time equations result from the
torque is to occur at start is ________. numerical integration of the differential equations of
Solution: Slip compounding to maximum rotor torque, an un-damped simple harmonic oscillator with state
variable x and y. The integration time step is h.
r xk +1 − xk
Sm = 2
= yx
X2 h

0.08 yk +1 − yk
0.15 = = –xk
X2 h
X2 = 0.2Ω For this discrete-time system, which one of the
For Textra = Te max following statements is TRUE?
(A) The system is not stable for h > 0
Textra 2
= =1 1
Te max 1 (B) The system is stable for h >
+ Sm π
Sm
1
(C) The system is stable for 0 < h < h <
r1 2π
Sm = 2 = 1
X2 1 1
(D) The system is stable for <h<
1
r2 = X 2 = 0.2Ω 2π π

External resistance Solution:
= 0.2 – 0.03 = 0.17Ω Hence, the correct option is (A).
Hence, the correct Answer is (0.16 to 0.18). Question Number: 63 Question Type: MCQ
Question Number: 61 Question Type: NAT The unit step response of a system with the transfer
Two three-phase transformers are realized using single- 1 − 2s
function G (s) = is given by which one of the
phase transformers as shown in the figure given on next 1+ s
column. following waveforms?

F05_Gate_EE_2015_978-93-325-7605-6_P-I.indd 146 6/19/2017 7:17:32 PM


GATE 2015 Solved Paper Electrical Engineering: Set – 1 | cxlvii

(C) –0.42 and –1.58


(D) none of the above
Solution:
(A) Characteristic equation
1 + G (s) H (s) = 0
S (S + 1) (S + 2) = K = 0
S [S 2 + 3S + 2] + K = 0
–K = S 3 + 3S 2 + 2S
(B) dk
− = 3S 2 + 6S + 2 = 0
ds
S = –0.42 and –1.58
For k > 0, S = –0.42
Hence, the correct option is (A).
(C) Question Number: 65 Question Type: MCQ
For the system governed by the set of equations:
dx1/dt = 2x1 + x2 + u
dx2/dt = –2x1 + u
y = 3x1
the transfer function Y (s)/U (s) is given by:
(D) (A) 3 (s + 1)/(s2 – 2s + 2)
(B) 3 (2s + 1)/(s2 – 2s + 1)
(C) (s + 1)/(s2 – 2s + 1)
(D) 3 (2s + 1)/(s2 – 2s + 2)
Solution: Transfer function
Solution:
1 − 2S
G (s) =  x1   2 1  x1  1
1+ S  x  =   + [ 4]
 2   −2 0   x2  1
1  1 − 2S 
Y (s) = unit step response =  1 + 2S 
S   x 
[4] = [ 3 0 ]  1 
1 −3  x2 
= +
S 1 +S
Transfer function
y (t) = u (t) – 3e-t u (t)
Y ( s)
y (t) = (1 – 3e-t) u (t) = C [SI – A]–1 B
U ( s)
Hence, the correct option is (A).
−1
Question Number: 64 Question Type: MCQ   S 0   2 1  1
= [3 0 ]   −  1
An open loop transfer function G (s) of a system is:
  0 S   −2 0    
K S 1  1
G (s) = = [3 0 ] 
s ( s + 1) ( s + 2)  
 −2 S − 2  1
For a unity feedback system, the breakaway point of the
3 ( S + 1)
root loci on the real axis occurs at, =
2
(A) –0.42 S − 2S + 1
(B) –1.58 Hence, the correct option is (A).

F05_Gate_EE_2015_978-93-325-7605-6_P-I.indd 147 6/19/2017 7:17:36 PM


GATE 2015 Solved Paper
Electrical Engineering
Set - 2
Number of Questions: 65 Total Marks: 100.0

Wrong answer for MCQ will result in negative marks, (-1/3) for 1 Mark Questions and (-2/3) for 2 Marks
Questions.

General Aptitude
Number of Questions: 10 Section Marks: 15.0

Q. 1 to Q. 5 carry 1 mark each and Q. 6 to Q. 10 carry (B) Statements I and III follow.
2 marks each (C) Statements II and III follow.
(D) All statements follow.
Question Number: 1 Question Type: MCQ
Solution:  When there is a significant drop in the wa-
Select the alternative meaning of the underlined part of ter level in the lakes supplying water in the city. The
the sentence. plausible course of action has to be the ones which are
The chain snatchers took to their heels when the police practically possible.
party arrived. Among the three given courses of action, only I and II
(A) took shelter in a thick jungle are practically possible. III speaks about banning the
(B) open indiscriminate fire water supply in lower areas.
(C) took to flight This is not an appropriate solution to the existing
(D) unconditionally surrendered problem. And stopping or banning water in the lower
Solution:  Took to their heels’ means to run away. This areas for proper supply in the city is unethical as well.
supports option (C). The other answer options are in- Hence, the correct option is (A).
correct. Question Number: 3 Question Type: NAT
Hence, the correct option is (C). The pie chart below has the breakup of the number of
Question Number: 2 Question Type: MCQ students from different departments in an engineering
The given statement is followed by some courses of college for the year 2012. The proportion of male to
action. Assuming the statement to be true, decide the female students in each department is 5:4. There are
correct option. 40 males in Electrical Engineering. What is the
difference between the numbers of female students in
Statement: There has been significant drop in the the Civil department and the female students in the
water level in the lakes supplying water to the city. Mechanical department?
Course of action:
I. The water supply authority should impose a
partial cut in supply to tackle the situation.
II. The government should appeal to all the
residents through mass media for minimal use
of water.
III. The government should ban the water supply
in lower areas.
(A) Statements I and II follow.

F06_Gate_EE_2015_978-93-325-7605-6_P-II.indd 148 6/19/2017 7:19:15 PM


GATE 2015 Solved Paper Electrical Engineering: Set – 2 | cxlix

Solution:  Number of students in the Electrical Engi- p + m + c = a + b + c + 2 (d + e + f ) + 3g


neering department = (a + b + c + d + e + f + g)
9 + (d + e + f + 2g)
= 40   = 72.
5 75 40 20 27
= + + =
Number of students in the Civil department 100 100 100 20
30 ⇒ I is true and II is not true.
= (72) = 108
20 (p) (m) (c) = probability (The student passing in all the
Number of students in the Mechanical department three subjects)

10 50 40 10 1
= (72) = 36 = − = =
100 100 100 10
20
Hence, I and III are true.
Number of female students in the Civil and the
4 4 Hence, the correct option is (D).
Mechanical departments are 108 -   and 36   ,
 
9 9 Question Number: 5 Question Type: MCQ
respectively.
The number of students in a class who have answered
i.e., 48 and 16, respectively. correctly, wrongly, or not attempted each question in an
Difference is 48 - 16, i.e., 32. exam, are listed in the table below. The marks for each
Hence, the correct answer is 32. question are also listed. There is no negative or partial
marking.
Question Number: 4 Question Type: MCQ
The probabilities that a student passes in Mathematics, Q. Marks Answered Answered Not
Physics and Chemistry are m, p and c, respectively. Of No. Correctly Wrongly Attempted
these subjects, the student has 75% chance of passing
in at least one, a 50% chance of passing in at least two 1 2 21 17 6
and a 40% chance of passing in exactly two. Following 2 3 15 27 2
relations are drawn in m, p, c:
3 1 11 29 4
I. p + m + c = 27/20
II. p + m + c = 13/20 4 2 23 18 3
III. (p) × (m) × (c) = 1/10
5 5 31 12 1
(A) Only relation I is true.
(B) Only relation II is true. What is the average of the marks obtained by the class
(C) Relations II and III are true. in the examination?
(D) Relations I and III are true. (A) 2.290 (B) 2.970
(C) 6.795 (D) 8.795
Solution:  VD for probabilities
Solution:  Average of the marks obtained by the class
2 ( 21) + 3 (15) + 1 (11) + 2 ( 23) + 5 (31)
=
Total number of students
299
= = 6.795
44
Hence, the correct option is (C).
Question Number: 6 Question Type: MCQ
Didn’t you buy _________ when you went shopping?
(A) any paper (B) much paper
(C) no paper (D) a few paper

F06_Gate_EE_2015_978-93-325-7605-6_P-II.indd 149 6/19/2017 7:19:17 PM


cl | GATE 2015 Solved Paper Electrical Engineering: Set – 2

Solution: (C) Both statements together are sufficient, but


Hence, the correct option is (A). neither statement alone is sufficient.
(D) Statements I and II together are not satisfied.
Question Number: 7 Question Type: MCQ
Solution:  The distance between the 2 floors of the
Which of the following options is the closest in meaning
building is 9 feet. And we are asked to find the number
to the sentence below?
steps from first floor to second floor.
She enjoyed herself immensely at the party.
From I: If each step is 3/4th foot.
(A) She had a terrible time at the party.
(B) She had a horrible time at the party. Let there be a total of n steps from first floor to record
(C) She had a terrific time at the party floor.
(D) She had a terrifying time at the party. 3 4×9
× n = 9;  n = = 12
4 3
Solution:
Hence, the correct option is (C). So, I alone gives this answer
II speaks about the width of each step. From width
Question Number: 8 Question Type: MCQ
we cannot get the number of steps between 1st and
Which one of the following combinations is incorrect? 2nd floors.
(A) Acquiescence—Submission
Hence, the correct option is (A).
(B) Wheedle—Roundabout
(C) Flippancy—Lightness Question Number: 10 Question Type: MCQ
(D) Profligate—Extravagant Given Set A = {2, 3, 4, 5} and Set B = {11, 12, 13,
Solution: 14, 15}, two numbers are randomly selected, one from
each set. What is the probability that the sum of the two
Hence, the correct option is (B).
numbers equals 16?
Question Number: 9 Question Type: MCQ (A) 0.20 (B) 0.25
Based on the given statements, select the most (C) 0.30 (D) 0.33
appropriate option to solve the given question. Solution:  Let the numbers randomly selected from
If two floors in a certain building are 9 feet apart, how set A and set B be a and b, respectively.
many steps are there in a set of stairs that extends from The number of (a, b) that can be formed taking a from
the first floor to the second floor of the building? A and b from B is 4 × 5 = 20. And the (a, b) for which
Statements: a + b = 16 are (2, 14), (3, 13), (4, 12), (5, 11).
I. Each step is 3/4th foot high. Number of favourable selections = 4
II. Each step is 1 foot wide. 4
(A) Statement I alone is sufficient but statement II Required probability = = 0.2
20
alone is not sufficient.
(B) Statement II alone is sufficient, but statement I Hence, the correct option is (A).
alone is not sufficient.

Electrical Engineering
Number of Questions: 55 Section Marks: 85.0

Q. 11 to Q. 35 carry 1 mark each and Q. 36 to Q. 65 (B) potential transformer


carry 2 marks each (C) current transformer
(D) distribution transformer
Question Number: 11 Question Type: MCQ
The primary mmf is least affected by the secondary Solution:  In current transformer least number of sec-
terminal conditions in a ondary turns therefore the effect on primary mmf is least.
(A) power transformer Hence, the correct option is (C).

F06_Gate_EE_2015_978-93-325-7605-6_P-II.indd 150 6/19/2017 7:19:18 PM


GATE 2015 Solved Paper Electrical Engineering: Set – 2 | cli

Question Number: 12 Question Type: MCQ


A Bode magnitude plot for the transfer function G (s)
of a plant is shown in the figure. Which one of the
following transfer functions best describes the plant?

Solution:
Y ( s)
The forward path given for is G2.
X 2 (S ) X1 ( S ) = 0

By using masons gain formula


1000 ( s + 10) 10 ( s + 10)
(A) (B) The transfer function
s + 1000 ( s + 1000)
s G2 G2
s + 1000 s + 1000 = =
(C) (D) 1 + G1G2 + G2 1 + G2 (1 + G1 )

10 s ( s + 10) 10 ( s + 10)
Hence, the correct option is (B).
Solution:  From the given bode plot one pole at -10 Question Number: 14 Question Type: MCQ
and one zero at 1000. The general transfer function is
The maximum value of ‘a’ such that the matrix
 S   −3 0 −2 
k 1 +
 1000   
 1 −1 0  has three linearly independent real
G (s) =  0 a −2 
 S   
 1 + 10  eigenvectors is:
 
2 1
Finding k ÷ The initial slope 20 log k = 20 (A) (B)
⇒ K = 10 3 3 3 3
1+ 2 3 1+ 3
 S  (C) (D)
10  1 +  3 3
 1000  S + 1000 3 3
G (s) = =
S 10 ( S + 10) Solution:  Let the given matrix be
1+
10
 −3 0 −2 
Hence, the correct option is (D). A =  1 −1 0  ⋅
Question Number: 13 Question Type: MCQ  0 a −2 

For the signal-flow graph shown in the figure in the
The characteristic equation of A is
next column, which one of the following expressions is
|A - lI| = 0
Y ( s)
equal to the transfer function ? −3 − λ 0 −2
X 2 ( s) X ( s ) = 0
1
1 −1 − λ 0
G1 G2 0 a −2 − λ
(A) (B)
1 + G2 (1 + G1 ) 1 + G1 (1 + G2 )
=0
G1 G2 ⇒ (-3 - l) (-1 - l) (-2 - l) - 2a = 0
(C) (D)
1 + G1G2 1 + G1G2 ⇒  (l + 1) (l + 2) (l + 3) + 2a = 0 (1)

F06_Gate_EE_2015_978-93-325-7605-6_P-II.indd 151 6/19/2017 7:19:22 PM


clii | GATE 2015 Solved Paper Electrical Engineering: Set – 2

We know that if A has three district Eigen values then A \ f (x) has a maximum at
has three linearly independent eigen vectors
−6 − 3
Let f (l) = (l + 1) (l + 2) (l + 3) x = and a minimum at
3
\ Equation (1) becomes,
−6 + 3
f (l) + 2a = 0 x =
3
⇒ f (l) = -2a (2)
The maximum value of
Consider f (x) = (x + 1) (x + 2) (x + 3)
2 −6 − 3
The graph of f (x) is as shown in the figure below: f (x) = y2 = f (x) = at x =
3 3 3
The minimum value of
f (x) = y1 = f (x)
−6 − 3 −2
at x = =
3 3 3
From Equation (4)
−2 2
=≤ f ( x ) ≤
3 3 3 3
−2 2
We know that the number of distinct real roots of an ⇒ ≤ - 2a ≤ [from Equation (3)]
equation F (x) = k (k is real) is same as that of the 3 3 3 3
number of points of intersection of the curve y = F (x) 1 −1 −1 1
⇒ ≥a≥ ⇒ ≤a≤
and the line y = k. 3 3 3 3 3 3 3 3

The curve y = f (x) intersects at three points with a line \ The maximum value of ‘a’ such that the matrix A has
y = y0 only when y1 ≤ y0 ≤ y2
1
three real linearly independent eigen vectors is ⋅
i.e., for f (x) + 2a = 0 (OR) f (x) = -2a, three distinct 3 3
real roots exist for
Hence, the correct option is (B).
y1 ≤ -2a ≤ y2 (3)
i.e. y1 ≤ f (x) ≤ y2 (4) Question Number: 15 Question Type: NAT
[from Equation (2)] A solution of the ordinary differential equation

Now we will find y1 and y2 [i.e. the minimum and d2 y dy


+5 + 6y = 0
2 dt
maximum values of f (x)] dt
f (x) = (x + 1) (x + 2) (x + 3) 1 − 3e
is such that y (0) = 2 and y (1) = − ⋅
= x3 + 6x2 + 11x + 6 e3
⇒ f 1 (x) = 3x2 + 12x + 11 dy
The value of (0) is ____.
f 1 (x) = 0 dt
2
⇒ 3x + 12x + 11 = 0 Solution:  Given differential equation is

−6 ± 3 d2 y dy
⇒ x= +5 + 6 y = 0 (1)
2 dt
3 dt

And 11 
f (x) = 6x + 12 1 − 3e
where y (0) = 2 and y (1) = − (2)
−6 + 3 11 −6 − 3 e3
At x = ; f (x) = 2 3 > 0 and at x = ;
3 3 The auxiliary equation of (1) is
f 11 (x) = -2 3 < 0. D2 + 5D + 6 = 0 ⇒ D = -2, -3

F06_Gate_EE_2015_978-93-325-7605-6_P-II.indd 152 6/19/2017 7:19:27 PM


GATE 2015 Solved Paper Electrical Engineering: Set – 2 | cliii

The general solution of Equation (1) is


y = c1 e-2t + c2 e-3t
From Equation (2),
y (0) = 2 ⇒ c1 + c2 = 2 (3)
1 − 3e
and y (1) = −
e3
⇒ c1e + c2e = 3e - e-3 (4)
-2 -3 -2

Solving Equations (3) and (4)


⇒ c1 = 3 and c2 = -1
Question Number: 17 Question Type: MCQ
\ The solution of given differential equation is
Two players, A and B, alternately keep rolling a fair
y = 3e-2t - e-3t dice. The person to get a six first wins the game. Given
dy that player A starts the game, the probability that A
⇒ = -6e-2t + 3e-3t
dt wins the game is:
dy dy (A) 5/11 (B) 1/2
\ (0) = at t = 0 = -6 + 3 = -3. (C) 7/13 (D) 6/11
dt dt
Hence, the correct Answer is (-3). Solution:  Let A and B denote the events of player A
getting six on the die and player B getting six on the
Question Number: 16 Question Type: MCQ die, respectively
The signum function is given by 1 1 5
\ P (A) = , P ( B) = , P ( A) =
 x 6 6 6
 ; x≠ 0
sgn (x) =  x 5
 and P ( B ) =
 0; x = 0 6
The Fourier series expansion of sgn [cos (t)] has Probability of A winning the game
(A) only sine terms with all harmonics.
(B) only cosine terms with all harmonics. = P ( A) + P ( A ∩ B ∩ A)
(C) only sine terms with even numbered harmon- + P ( A ∩ B ∩ A ∩ B ∩ A) + ...
ics.
(D) only cosine terms with odd numbered har- = P ( A) + P ( A) P ( B ) P ( A)
monics. + P ( A) P ( B ) P ( A) P ( B ) P ( A) + ... ∞
Solution: 1 5 5 1 5 5
= + × × + ×
 x 6 6 6 6 6 6
 ; x≠ 0
Given sgn (x) =  x 5 5 1
 0; x = 0 × × × + ... ∞
 6 6 6
2 4
 cos (t ) π 1 5 1 5 1
 cos t ; t ≠ 2 = +   × +   × + ... ∞
\ sgn [cos (t)] =  6  6  6  6  6
 0; π

t= which is a geometric series with a = 1 and
2  6
Its wave form is shown in figure given in next column. 2
5 25 
The function is half wave symmetric. So, its Fourier r=  =
series consists of only cosine terms with odd numbered 6 36 
harmonics. a
\ S∞ =
Hence, the correct option is (D). 1 − r

F06_Gate_EE_2015_978-93-325-7605-6_P-II.indd 153 6/19/2017 7:19:31 PM


cliv | GATE 2015 Solved Paper Electrical Engineering: Set – 2

1  1+ x 1 
= E −
= 6
 P + 1 + x 1 + P 
 25 
1−  For the maximum Vo w.r.t
 36 
dV
6 P= o =0
= ⋅ dP
11
d   1+ x 1 
\ Hence, probability of A winning the game  E 1 + x + P − 1 + P   = 0
6
dP   
=
11 1 1+ x
=
Hence, the correct option is (D).

(1 + P ) 2

(1 + x + P ) 2
Question Number: 18 Question Type: MCQ
An unbalanced DC Wheatstone bridge is shown in the 1 1+ x
=
figure. At what value of p will the magnitude of V0 be 1+ P 1+ x + P
maximum?
1 + x + P = (1 + P ) 1 + x

1 + x + P = (1 + x + P ( 1 + x )

P (1 − 1 + x ) = (1 + x (1 − 1 + x )

P = 1 + x
Hence, the correct option is (A).
Question Number: 19 Question Type: NAT
The circuit shown is meant to supply a resistive load RL
from two separate DC voltage sources. The switches S1
(A) (1 + x ) and S2 are controlled so that only one of them is ON at
(B) (1 + x) any instant. S1 is turned on for 0.2 ms and S2 is turned
on for 0.3 ms in a 0.5 ms switching cycle time period.
(C) 1/ (1 + x ) Assuming continuous conduction of the inductor
current and negligible ripple on the capacitor voltage,
(D) (1 − x )
the output voltage Vo (in Volt) across RL is ______.
Solution: Given

Solution:  The average output voltage


10 × 0.2 + 5 × 0.3
Vo = =7V
ER (1 + x ) R 0.5
V= −E
PR + R (1 + x ) PR + R Hence, the correct Answer is (7).

F06_Gate_EE_2015_978-93-325-7605-6_P-II.indd 154 6/19/2017 7:19:35 PM


GATE 2015 Solved Paper Electrical Engineering: Set – 2 | clv

Question Number: 20 Question Type: MCQ Question Number: 22 Question Type: NAT
A self commutating switch SW, operated at duty cycle A 3-phase 50 Hz square wave (6-step) VSI feeds a
d is used to control the load voltage as shown in the 3-phase, 4 pole induction motor. The VSI line voltage
figure. has a dominant 5th harmonic component. If the
operating slip of the motor with respect to fundamental
component voltage is 0.04, the slip of the motor with
respect to 5th harmonic component of voltage is
______.

Under steady-state operating conditions, the average


voltage across the inductor and the capacitor
respectively, are:
1 Solution:
(A) VL = 0 and VC = V 120 × 50
1 − δ dc Synchronous speed Ns = = 1500 rpm
δ 1 4
(B) VL = Vdc and VC = V 5th harmonic speed = Ns/5 = 300 rpm
2 1 − δ dc
δ N + N s /5
(C) VL = 0 and VC = V Slip at 5th harmonic = s
1 − δ dc Ns

δ δ 1500 + 300
(D) VL = Vdc and VC = V = = 1.2
2 1 − δ dc 1500
Hence, the correct Answer is (1.2).
Solution:  The circuit represents step up chopper. So,
1 Question Number: 23 Question Type: MCQ
Vc = Vdc
1− δ Consider a discrete time signal given by
x [n] = (–0.25)n u [n] + (0.5)n u [–n - 1]
Hence, the average output voltage of inductor is zero.
The region of convergence of its Z-transform would be:
Hence, the correct option is (A).
(A) the region inside the circle of radius 0.5 and
Question Number: 21 Question Type: NAT centered at origin.
The single-phase full-bridge voltage source inverter (B) the region outside the circle of radius 0.25 and
(VSI), shown in figure, has an output frequency of centered at origin.
50 Hz. It uses unipolar pulse width modulation with (C) the annular region between the two circles,
switching frequency of 50 kHz and modulation index both centered origin and having radii 0.25 and
of 0.7. For Vin = 100 V DC, L = 9.55 mH, C = 63.66 µF, 0.5.
and R = 5Ω, the amplitude of the fundamental compo- (D) the entire Z plane.
nent in the output voltage Vo (in Volts) under steady- Solution:
state is ___________. Given x (n) = (-0.25)n u (n) + (0.5)n u (–n - 1)
Solution: (-0.25)n u (n) is similar to (-1)n u (n)
2 2Vs So, ROC is
Vo = modulation index ×
π |Z| > |-0.25| and for (0.5)n
u (-n - 1) the ROC is |Z| < |0.5|
2 2 × 100
= 0.7 × = 63.05 V ∴ ROC is
3.14 0.25 < |Z| < 0.5
Hence, the correct Answer is (63.05). Hence, the correct option is (C).

F06_Gate_EE_2015_978-93-325-7605-6_P-II.indd 155 6/19/2017 7:19:37 PM


clvi | GATE 2015 Solved Paper Electrical Engineering: Set – 2

Question Number: 24 Question Type: NAT Solution:  By observing the circuit, it is Asynchro-
A parallel plate capacitor is partially filled with glass of nous, as well as synchronous circuits.
dielectric constant 4.0 as shown below. The dielectric The output of Qo is connected as Clk pulse to Q1, Q0
strengths of air and glass are 30 kV/cm and 300 kV/cm, flip-flops synchronously.
respectively. The maximum voltage (in kilovolts), For Q0 flip-flop, Jo = Ko = 1, so it works as toggle
which can be applied across the capacitor without any switch Qn+1 = Qn , for every clk pulse it will change
breakdown, is _____. its state.
Q1 Q2 work as synchronous circuit with Q0 as clk
pulse negative edge Clk (1 → 0)

J2
Clk Q2 Q1 Q0 K J (Q ) K1 J0 K0
(Q1) 2 1 2
0 0 0 0 0 1 1 1 1 1
Solution: 1 0 0 1 0 1 1 1 1 1
ε r εo A εo A 2 0 1 0 1 1 1 1 1 1
C1 = 1
= 3 0 1 1 1 1 1 1 1 1
d d
4 1 0 0 0 1 0 1 1 1
ε r εo A 4ε o A 5 1 0 1 0 1 0 1 1 1
C2 = 2
=
d d 6 0 0 0 0 1 1 1 1 1
C1C2 4 Aεo 7 0 0 1 0 1 1 1 1 1
Ceq = =
C + C2 5d Hence, the correct Answer is (6).
1
Electric field Question Number: 26 Question Type: MCQ
V f (A, B, C, D) = ∏M (0, 1, 3, 4, 5, 7, 9, 11, 12, 13, 14,
E=
d 150 is a maxterm representation of a Boolean function
Ceq ⋅ V f (A, B, C, D) where A is the MSB and D is the LSB. The
D ρ Q CV
⇒ E= n = s = = = equivalent minimized representation of this function is:
εo εo A × εo Aεo Aεo (A) ( A + C + D ) ( A + B + D )

4 × A × εo V (B) ACD + ABD
30 × 105 = ×
−3 Aεo (C) ACD + ABCD + ABCD
5 × 5 × 10
(D) ( B + C + D ) ( A + B + C + D ) ( A + B + C + D )
75
⇒ V= × 103 = 18.75 kV Solution:
4
F (A, B, C, D) = πM (0, 1, 3, 4, 5, 7,
Hence, the correct Answer is (17 to 20).
9, 11, 12, 13, 14, 15)
Question Number: 25 Question Type: NAT
The figure shows a digital circuit constructed using
negative edge triggered J-K flip-flops. Assume a starting
state of Q2 Q1 Q0 = 000. This state Q2 Q1 Q0 = 000
will repeat after ______ number of cycles of the clock
CLK.

D ( A + B) ( A + C )

F06_Gate_EE_2015_978-93-325-7605-6_P-II.indd 156 6/19/2017 7:19:41 PM


GATE 2015 Solved Paper Electrical Engineering: Set – 2 | clvii

−Vo
V1 =
1000
IR = IC

 Vs − V1  d
  = C ⋅ (V1 − V0 )
 R  dt

 V0 
 Vs + 
 1000  = C d  −V0 − V 
 0
 1000  dt  1000 

10 −6 d
ACD + ABD 10-3 Vs + 10-6 Vo = − [1001Vo ]
1000 dt
Hence, the correct option is (C).
10-3 × 10-3 + 10-6 Vo
Question Number: 27 Question Type: MCQ dVo
The op-amp shown in the figure has finite gain = -1.001 × 10-6
dt
A = 1000 and an infinite input resistance. A step-
voltage Vi = 1 mV is applied at the input at time t = 0 dVo
1 + Vo = -1.001
as shown. Assuming that the operational amplifier is dt
not saturated, the time constant (in millisecond) of the dVo
output voltage Vo is: ⇒ 1.001 + Vo = -1
dt
−1
⇒ Vo (S) =
1 . 001S +1
Time constant τ = 1.001 sec
T = 1001 msec
Hence, the correct option is (A).
Question Number: 28 Question Type: NAT
A random variable X has probability density function
f (x) as given below:
(A) 1001 (B) 101  a + bx; for 0 < x < 1
f (x) = 
(C) 11 (D) 1
 0; otherwise
Solution:
If the expected value E [X] = 2/3, then Pr [X < 0.5] is
_______.
Solution:  Given the probability density function of a
random variable X is
 a + bx for 0 < x < 1
f (x) = 
 0 otherwise

We know that for any probability density function f (x)


of a random variable X,

V2 = 0;  Vo = A (V2 - V1) ∫ f ( x ) dx = 1
Vo = -1000V1 −∞

F06_Gate_EE_2015_978-93-325-7605-6_P-II.indd 157 6/19/2017 7:19:44 PM


clviii | GATE 2015 Solved Paper Electrical Engineering: Set – 2

= x 2 ]0
∞ 1 ∞ 0.5
⇒ ∫ 0 dx + ∫ ( a + bx ) dx + ∫ 0 dx = 1
−∞ 0 1 = (0.5)2
1 = 0.25.
⇒ ∫ (a + bx)dx =1 Hence, the correct Answer is (0.25).
0 Question Number: 29 Question Type: MCQ
1
bx 2  If a continuous function f (x) does not have a root in
⇒ ax +  =1 the interval [a, b], then which one of the following
2 0
statements is TRUE?
b (A) f  (a) ⋅ f (b) = 0 (B) f (a) ⋅ f (b) < 0
⇒ a+ =1
2 (C) f (a) ⋅ f (b) > 0 (D) f (a)/f (b) ≤ 0
⇒ 2a + b = 2 (1) Solution:  As f (x) is continuous in [a, b] and f (x) has
Given the expected value of no root in [a, b], f (x) does not cut x-axis for all x in
2 [a, b]
X = E (X) =
3 ⇒ f (x) is either above x-axis or below x-axis for both
∞ x = a and x = b
2
i.e., ∫
xf ( x )dx =
3 ⇒ f (a) and f (b) are both positive or both negative
−∞ ⇒ f (a) ⋅ f (b) > 0
0 1 ∞ Hence, the correct option is (C).
⇒ ∫ x × 0 dx + ∫ x × ( a + bx ) dx + ∫ x × 0dx Question Number: 30 Question Type: NAT
−∞ 0 1
2 If the sum of the diagonal elements of a 2 × 2 matrix is
= –6, then the maximum possible value of determinant of
3
the matrix is _______.
1
2 Solution:  Let A be a 2 × 2 matrix with the sum of the
∫ (ax + bx
2
⇒ ) dx =
3 diagonal elements as -6
0
1 Let l1 and l2 be the eigen values of A
2 2 3
ax bx
⇒ +  = \ The sum of the diagonal elements of A = -6
2 3  3
0 ⇒ l1 + l2 = -6 (1)

Det of A = |A| = l1l2
a b 2
⇒ + = Now we have to find the maximum value of l1l2
2 3 3
Let f = l1l2
⇒ 3a + 2b = 4 (2) = l1 (-6 - l1)
Solving Equations (1) and (2), we get a = 0 and b = 2 [from Equation (1)]
\ f (x) becomes \ f = -6l1 - λ12

 2 x for 0 < x < 1 ⇒ f 1 = -6 -2l1


(x) = 
f 
0 otherwise For f to have maximum,

f 1 = 0
0.5 ⇒ -6 - 2l1 = 0
Now P (x < 0.5) = ∫ f ( x ) dx ⇒ l1 = -3
−∞ Now f 11 = -2 < 0
0 0.5 \ f has a maximum at l1 = -3
= ∫ 0 dx + ∫ 2 xdx From Equation (1),
−∞ 0 l1 + l2 = -6

F06_Gate_EE_2015_978-93-325-7605-6_P-II.indd 158 6/19/2017 7:19:48 PM


GATE 2015 Solved Paper Electrical Engineering: Set – 2 | clix

⇒ -3 + l2 = -6 Solution:
⇒ l2 = -3
The maximum value of the determinant of A = l1l2
= (-3) × (-3) = 9.
Hence, the correct Answer is (9).
Question Number: 31 Question Type: MCQ

1
Consider a function f = rˆ, where r is the distance
r2
from the origin and r̂ is the unit vector in the radial
direction. The divergence of this function over a sphere Given R1 = 50Ω
of radius R, which includes the origin, is: R2 = 65Ω
(A) 0 (B) 2π R3 = 100Ω
(C) 4π (D) Rπ R3 tolerance ± 5%
Then R3 = 95 ⇒ 105
Solution:
When bridge is balanced
∂ 1 ∂
∇f = ( r 2 f r ) + ( f sin θ) R1 Rx = R2 R3
∂r r sin θ ∂θ θ When R3 = 95 ⇒ 50Rx = 65 × 95
1 ∂ 1 ∂ 65 × 95
+ ( f sin θ) + (f ) Rx = = 123.5Ω
r sin θ ∂θ θ r sin θ ∂θ θ 50
1 ∂  2 1  R3 = 105 ⇒ 50 Rx = 65 × 105
When
∇f = r × =0
2 ∂r  65 × 105
r  r2  Rx = = 136.5Ω
50
Hence, the correct option is (A).
Hence, the correct option is (A).
Question Number: 32 Question Type: MCQ
Question Number: 33 Question Type: NAT
When the Wheatstone bridge shown in the figure is A (0-50 A) moving coil ammeter has a voltage drop
used to find the value of resistor Rx, the galvanometer of 0.1 V across its terminals at full scale deflection.
G indicates zero current when R1 = 50Ω, R2 = 65Ω The external shunt resistance (in milliohms) needed to
and R3 = 100Ω. If R3 is known with ±5% tolerance on extend its range to (0-500 A) is ________.
its nominal value of 100Ω, what is the range of Rx in
Ohms? Solution:  Voltage drop of (0-50 A) ammeter is
= 0.1 V
External shunt resistance
Rsh = ?
Extend range–(0-500 A)
I = 50 A
Im = 500 A
Voltage drop IRm = 0.1
0.1
Rm = = 2 mΩ
50
(A) [123.50, 136.50] Rm
Shunt resistance Rsh =
(B) [125.89, 134.12]
 I m  
(C) [117.00, 143.00]   − 1
I
(D) [120.25, 139.75]   

F06_Gate_EE_2015_978-93-325-7605-6_P-II.indd 159 6/19/2017 7:19:50 PM


clx | GATE 2015 Solved Paper Electrical Engineering: Set – 2

Question Number: 36 Question Type: NAT


2 × 10 −3
= A moving average function is given by
  500  
  50  − 1
t
1
  T ∫
  y  (t) = u ( τ) d τ.
–4 t − T
= 2.22 × 10
1
= 0.22 mΩ If the input u is a sinusoidal signal of frequency Hz,
2T
Hence, the correct Answer is (0.22 to 0.23).
then in steady-state, the output y will lag u (in degree)
Question Number: 34 Question Type: MCQ by ______.
Of the four characteristics given below, which are the Solution:
major requirements for an instrumentation amplifier?
u  (τ) = sin wτ
P. High common mode rejection ratio
t −τ
Q. High input impedance 1
t
cos ( ωτ )
R. High linearity (t) =
y  ∫ sin ( ωτ ) d τ =
T ωτ t
S. High output impedance t −T
(A) P, Q and R only (B) P and R only
(C) P, Q and S only (D) Q, R and S only 1
= [cos ω (t - τ) - cos ωt]
π
Solution:  Instrumentation amplifier has to amplify
small changes in the bridge circuit. So, for ideal 2 2
(t) = − cos ωt =
y  sin (90 + ωt)
instrumentation amplifier high CMMR, high, input π π
impedance, high, linearity and low output impedance
required. y (t) will lag u by 90°
Hence, the correct option is (A). Hence, the correct Answer is (90°).
Question Number: 35 Question Type: NAT Question Number: 37 Question Type: MCQ
In the following chopper, the duty ratio of switch is The impulse response g (t) of a system G, is as shown
0.4. If the inductor and capacitor are sufficiently large in Figure (a). What is the maximum value attained by
to ensure continuous inductor current and ripple free the impulse response of two cascaded blocks of G as
capacitor voltage, the charging current (in Ampere) of shown in Figure (b)?
the 5 V battery, under steady-state is ______.

Solution:  Given data duty ratio d = 0.4


Input voltage Vi = 20 V
Battery voltage E = 5 V
Resistance R = 3Ω 2 3
(A) (B)
Output voltage Vo = dVi 3 4
Vo = 0.4 × 20 = 8 V 4
Charging current (C) (D) 1
Vo − E 3 5
I= = =1 A
R 3 Solution:  Given
Hence, the correct Answer is (1). h (t) = g (t) × g (t)

F06_Gate_EE_2015_978-93-325-7605-6_P-II.indd 160 6/19/2017 7:19:53 PM


GATE 2015 Solved Paper Electrical Engineering: Set – 2 | clxi

The h (t) waveform Question Number: 39 Question Type: MCQ


A steady current I is flowing in the x-direction through
L
each of two infinitely long wires at y = ± as shown
2
in

the figure. The permeability of the medium is µ0. The
B - field at (0, L, 0) is:

So, maximum value of cascaded response is 1.


Hence, the correct option is (D).
Question Number: 38 Question Type: NAT
Consider a one-turn rectangular loop of wire placed in
a uniform magnetic field as shown in the figure. The
plane of the loop is perpendicular to the field lines.
The resistance of the loop is 0.4Ω, and its inductance 4µ 0 t 4µ0 I
(A) − Ẑ (B) + Ẑ
is negligible. The magnetic flux density (in Tesla) is a 3πL 3πL
function of time, and is given by B (t) = 0.25 sin ωt, 3µ0 I
where ω = 2π × 50 radian/second. The power absorbed (C) 0 (D) − Ẑ
(in Watt) by the loop from the magnetic field is _____. 4 πL
Solution:
H = H1 + H2
I I
= ( − az ) +
2π ( L/2 )  3L 
2π  
 2 
4I
= (-az); B = µoH
3πL
Hence, the correct option is (A).
Question Number: 40 Question Type: MCQ
Solution:  Resistance of loop Consider the circuit shown in the figure. In this circuit
R = 0.4Ω;  B (t) = 0.25 sin ωt R = 1 kΩ, and C = 1 µF. The input voltage is sinusoidal
ω = 2π × 50 radian/second with a frequency of 50 Hz, represented as a phasor with
1 magnitude Vi and phase angle 0 radian as shown in the
flux f = ∫ B ⋅ ds = sin ωt
800 figure. The output voltage is represented as a phasor
with magnitude Vo and phase angle δ radian. What is
d φ −1
V = − = π cos ωt the value of the output phase angle δ (in radian) relative
dt 8 to the phase angle of the input voltage?
V2 π2
P = = cos 2 ωt
R 64 R
π2  1 + cos 2ωt 
P =  
0.4 × 64  2 
π2
Pavg = = 0.193
2 × 0.4 × 64
Hence, the correct Answer is (0.193).

F06_Gate_EE_2015_978-93-325-7605-6_P-II.indd 161 6/19/2017 7:19:56 PM


clxii | GATE 2015 Solved Paper Electrical Engineering: Set – 2

(A) 0 (B) π ⇒ 15 - 6.7 = IB RB


(C) p/2 (D) -p/2 ⇒ IB RB = 8.3 (2)
Solution: ( 2 ) RB 8.3 × 76
= = 105.133

(1) RC 6
Hence, the correct Answer is (105.133).
Question Number: 42 Question Type: MCQ
In the 4 × 1 multiplexer, the output F is given by
F = A + B. Find the required input ‘I3 I2 I1 I0’.

By considering virtual grounding concept the nodal


equation at node (1) is
0 − Vi ∠0o 0 − Vo ∠δ
+ =0
XC R

Vo ∠ δ = –jωRC Vi ∠0°
δ = –90°
Hence, the correct option is (D).
(A) 1010 (B) 0110
Question Number: 41 Question Type: NAT (C) 1000 (D) 1110
In the given circuit, the silicon transistor has β = 75 and
Solution:
a collector voltage VC = 9 V. Then the ratio of RB and
RC is ______. F = A + B = AB + AB
S1 S0
AB − 0 − I 0
AB − 1 − I1
AB − 1 − I 2
AB − 0 − I 3
Hence, the correct option is (B).
Question Number: 43 Question Type: MCQ
Consider a HVDC link which uses thyristor based line-
commutated converters as shown in the figure. For a
power flow of 750 MW from system 1 to system 2,
the voltages at the two ends, and the current, are
given by: V1 = 500 kV, V2 = 485 kV and I = 1.5 kA.
If the direction of power flow is to be reversed (that
Solution: is, from system 2 to system 1) without changing the
Given b = 75; VC = 9 V electrical connections, then which one of the following
Collector equation combinations is feasible?
⇒ 15 - 9 - (IC + IB) RC = 0
76IB RC = 6 (1)
Base ⇒ 15 - 76IB RC - IB RB - 0.7
=0

F06_Gate_EE_2015_978-93-325-7605-6_P-II.indd 162 6/19/2017 7:19:58 PM


GATE 2015 Solved Paper Electrical Engineering: Set – 2 | clxiii

(A) 5
(B) 7
(C) 10
(D) 14

(A) V1 = –500 kV, V2 = –485 kV and I = 1.5 kA Solution:


(B) V1 = –485 kV, V2 = –500 kV and I = 1.5 kA 6
I3 = = 2
Given
(C) V1 = 500 kV, V2 = 485 kV and I = –1.5 kA 3
(D) V1 = –500 kV, V2 = –485 kV and I = –1.5 kA 2
I2 = = 1
Solution:  For reversing power flow in HVDC system 2
the direction of current same.
V −V
I = 1 2
R
Option (A): V1 = -500 kV, V2 = –485 kV
I = –1.5 kA
Option (B): V1 = -485 kV, V2 = -500 kV
I = 1.5 kA
Hence, the correct option is (B).
I + I2 = I3
Question Number: 44 Question Type: MCQ I + 1 = 2 ⇒ 1 A
Base load power plants are: Power of voltage source
P: wind farms. P = VI = 5 W
Q: run-of-river plants. Hence, the correct option is (A).
R: nuclear power plants.
S: diesel power plants. Question Number: 46 Question Type: NAT
(A) P, Q and S only (B) P, R and S only For the given circuit, the Thevenin equivalent is to be
(C) P, Q and R only (D) Q and R only determined. The Thevenin voltage, Vth (in Volt), seen
from terminal AB is ______.
Solution:  Diesel power plants are peak load plants and
wind plants are base load plants because once wind
plants comes into operation it used for all times be-
cause no fuel cost.
Hence, the correct option is (C).
Question Number: 45 Question Type: MCQ
The voltages developed across the 3Ω and 2Ω resistors
shown in the figure are 6 V and 2 V respectively, with
the polarity as marked. What is the power (in Watt)
Solution:  Given
delivered by the 5 V voltage source?

Vth = 2i1

F06_Gate_EE_2015_978-93-325-7605-6_P-II.indd 163 6/19/2017 7:19:59 PM


clxiv | GATE 2015 Solved Paper Electrical Engineering: Set – 2

= 1 (i + i1) + i
2  ωL 
= 2i + i1 (1)
2 Z= j 
 1 − ω2 LC 
21 For different frequencies the curves similar to
21i = 2i1 ⇒ i1 = i (2)
2 option (B).
Solving Equation (1) and (2), Question Number: 48 Question Type: MCQ
i1 = 1.68 A
A separately excited DC generator has an armature
Vth = 2i1 = 2 × 1.68 = 3.36 V
resistance of 0.1Ω and negligible armature inductance.
Hence, the correct Answer is (3.36 V). At rated field current and rated rotor speed, its open-
Question Number: 47 Question Type: MCQ circuited voltage is 200 V. When this generator is
An inductor is connected in parallel with a capacitor as operated at half the rated speed, with half the rated field
shown in the figure. current, an uncharged 1000 µF capacitor is suddenly
connected across the armature terminals. Assume that
the speed remains unchanged during the transient.
At the time (in microsecond) after the capacitor is
connected will the voltage across it reach 25 V?
(A) 62.25 (B) 69.3
(C) 73.25 (D) 77.3
As the frequency of current i is increased, the impedance
Solution:  Armature resistance
(Z) of the network varies as:
Ra = 0.1Ω
rated field current and rated rotor speed open circuit
voltage
(A) E1 = 200 V
With half rated field current and half rated speed
φ N
i.e., 1 and 1
2 2
Capacitance C = 1000 µF
E ∝ N
(B) E1 φ N
⇒ = 1 1
E2 φ N
2 2
200 φ1N1
⇒ = ⇒ E2 = 50 V
E2 φ1 × N1
(C) 2 2
τ = RC = 0.1 × 1000 × 10–6
E = 2000e–t/RC −6
50 = 2000e −t/( 0.1× 1000 × 10 )
t = 69.3 µs
(D) Hence, the correct option is (B).
Question Number: 49 Question Type: MCQ
The self inductance of the primary winding of a single
Solution: phase, 50 Hz, transformer is 800 mH, and that of the
1
j ωL × secondary winding is 600 mH. The mutual inductance
Z L ZC j ωc
Z= = between these two windings is 480 mH. The secondary
Z L + ZC 1
j ωL + winding of this transformer is short circuited and the
j ωc

F06_Gate_EE_2015_978-93-325-7605-6_P-II.indd 164 6/19/2017 7:20:02 PM


GATE 2015 Solved Paper Electrical Engineering: Set – 2 | clxv

primary winding is connected to a 50 Hz, single phase, `/MWh along with the generation limits for the two
sinusoidal voltage source. The current flowing in both units are given below:
the windings is less than their respective rated currents. C1 (P1) = 0.01P12 + 30P1 + 10;
The resistance of both windings can be neglected. In 100 MW ≤ P1 ≤ 150 MW
this condition, what is the effective inductance (in mH)
C2 (P2) = 0.05P22 + 10P2 + 10;
seen by the source?
100 MW ≤ P2 ≤ 180 MW
(A) 416 (B) 440
(C) 200 (D) 920 The incremental cost (in `/MWh) of the power plant
when it supplies 200 MW is _______.
Solution:
L1 = 800 mH Solution:  Given
L2 = 600 mH G (P1) = 0.01P12 + 30P1 + 10;
M = 480 mH 100 MW ≤ P1 ≤ 150 MW
 C2 (P2) = 0.05P22 + 10P2 + 10;
ω2 M 2 
jXL =  jX1 +  10 MW ≤ P2 ≤ 180 MW
 jX 22 
 P1 + P2 = 200 MW (1)
dC1 dC2
 314 2 × 0.48  =
Leff =  314 × 0.8 −  314 dP1 dP2
 0.6 × 314 

0.02P1 + 30 = 0.1P2 + 10
130.744 0.1P2 - 0.02P1 = 20 (2)
=
314 By solving Equations (1) and (2)
Leff = 416 mH P2 = 200; P1 = 0
Hence, the correct option is (A). But P2 is more than maximum limit therefore the load
distribution between the units is P1 = 100 MW and
Question Number: 50 Question Type: MCQ
P2 = 100 MW
An 8-bit, unipolar Successive Approximation Register
dC2
type ADC is used to convert 3.5 V to digital equivalent ⇒ = 20 `/MWh
output. The reference voltage is + 5 V. The output of dP2

the ADC, at the end of 3rd clock pulse after the start of Hence, the correct Answer is (20).
conversion, is
(A) 1010 0000 (B) 1000 0000 Question Number: 52 Question Type: MCQ
(C) 0000 0001 (D) 0000 0011 Determine the correctness or otherwise of the following
Assertion [a] and the Reason [r].
Solution:  8-bit SAR type ADC, reference voltage
= +5 V Assertion: Fast decouples load flow method gives
approximate load flow solution because it uses several
5
k = resolution = = 0.0195 assumptions.
28
Reason: Accuracy depends on the power mismatch
= 3.5 V
input vector tolerance.
(A) both [a] and [r] are true and [r] is the correct
reason for [a].
(B) both [a] and [r] are true but [r] is not the
correct reason for [a].
(C) both [a] and [r] are false.
Hence, the correct option is (A).
(D) [a] is false and [r] is true.
Question Number: 51 Question Type: NAT
Solution:
Consider the economic dispatch problem for a power Hence, the correct option is (D).
plant having two generating units. The fuel costs in
Question Number: 53 Question Type: NAT

F06_Gate_EE_2015_978-93-325-7605-6_P-II.indd 165 6/19/2017 7:20:03 PM


clxvi | GATE 2015 Solved Paper Electrical Engineering: Set – 2

A 50 Hz generating unit has H-constant of 2 mJ/MVA. Question Number: 55 Question Type: NAT
The machine is initially operating in steady-state at The circuit shown in the figure has two sources
synchronous speed, and producing 1 pu of real power. connected in series. The instantaneous voltage of the
The initial value of the rotor angle δ is 5°, when a AC source (in Volts) is given by v (t) = 12 sin t. If the
bolted three phase to ground short circuit fault occurs circuit is in steady-state, then the rms value of the
at the terminal of the generator. Assuming the input current (in Ampere) flowing in the circuit is _______.
mechanical power to remain at 1 pu, the value of δ in
degrees, 0.02 second after the fault is _____.
Solution:  Given
H = 2 mJ/MVA
P = 1 p.u.
δo = 5° Solution:  Given
tcr = 0.02 V = 12 sin t
Pm = 1 p.u. V = 8 V
2H ( δcr − δo ) Idc = = 8 A
8
tcr =
πf Pm 1

12
2 × 2 (δcr − 50°) IAC max = = 8.485 A
0.02 = 1 + 12
2
π × 50 × 1

2
δcr = 5.90°  I AC m 
2
Irms = I dc + 
Hence, the correct Answer is (5.90°).  2
 
Question Number: 54 Question Type: MCQ
A sustained three-phase fault occurs in the power 8.4852
= 82 + = 9.99 A
system shown in the figure. The current and voltage 2
phasors during the fault (on a common reference), after Hence, the correct Answer is (9.99 A).
the natural transients have died down, are also shown.
Where is the fault located? Question Number: 56 Question Type: NAT
In a linear two-port network, when 10 V is applied to
Port 1, a current of 4 A flows through Port 2 when it is
short-circuited. When 5 V is applied to Port 1, a current
of 1.25 A flows through a 1Ω resistance connected
across Port 2. When 3 V is applied to Port 1, the current
(in Ampere) through a 2Ω resistance connected across
Port 2 is ______.
Solution:  Consider Y-parameter two port network
I1 = y11 V1 + y22 V2
I­2 = y21 V1 + y22 V2
When port-2 short circuited
(A) Location P (B) Location Q V2 = 0
(C) Location R (D) Location S I 4
y21 = 2 = = 0.4
Solution:  The Voltage magnitude of BUS 1 is small V 10
1
therefore the fault is at Q (or) R. The direction of I2 and
I4 are quite opposite then there is not fault at R. In second case: 1.25 = 0.4 × 5 + 1.25y22
y22 = -0.6
Hence, the correct option is (B).
⇒ at 3 V supply I2 = 3 × 0.4 - 0.6 (2I2)

F06_Gate_EE_2015_978-93-325-7605-6_P-II.indd 166 6/19/2017 7:20:05 PM


GATE 2015 Solved Paper Electrical Engineering: Set – 2 | clxvii

⇒ I2 = 0.545 A under the same armature voltage conditions, the speed


Hence, the correct Answer is (0.5454). rises to 520 rpm. Neglecting the rotational losses, the
full load armature current (in Ampere) is ______.
Question Number: 57 Question Type: NAT
Solution:
In the given circuit, the parameter k is positive, and the
Given N1 = 1000 rpm
power dissipated in the 2Ω resistor is 12.5 W. The value
of k is ______ V = 200 V
Ra = 1Ω
No load back e.m.f.
Eb1 = 200 V
N2 = 500 rpm
T3 = 0.5T2
N3 = 520 rpm
Eb 1000
1
= ⇒ Eb2 = 100 V
Solution:  Given Eb 500
2

100 = V - Ia2 (1)


Eb 1000
1
= ⇒ Eb3 = 104
Eb 500
2

104 = V - Ia3 (2)


T2 I T I I
∝ a 2 ⇒ 2 = a 2 ⇒ Ia3 = a2 (3)
Power dissipation in 2Ω resistor T3 I a3 0.5T2 I a3 2

V2 Solving Equation (1), (2) and (3)
= 12.5 W
R Ia2 = 8 A
⇒ V02 = 12.5 × 2 Hence, the correct Answer is (8).
⇒ Vo = 5 V
Question Number: 59 Question Type: NAT
Current in 2Ω resistor
V A DC motor has the following specifications: 10 hp,
= o = 2.5 A 37.5 A, 230 V; flux/pole = 0.01 Wb, number of poles
2 = 4, number of conductors = 666, number of parallel
KCL-nodal analysis 2.5 + kVo = 5 paths = 2. Armature resistance = 0.267Ω. The armature
2.5 reaction is negligible and rotational losses are 600 W.
K= = 0.5
The motor operates from a 230 V DC supply. If the
5
motor runs at 1000 rpm, the output torque produced
Hence, the correct Answer is (0.5).
(in Nm) is ________.
Question Number: 58 Question Type: NAT
Solution:  Given flux/pole
A separately excited DC motor runs at 1000 rpm on f = 0.01ωb
no load when its armature terminals are connected to a
P = 4
200 V DC source and the rated voltage is applied to the
field winding. The armature resistance of this motor is Z = 666
1Ω. The no-load armature current is negligible. With Parallel path A = 2
the motor developing its full load torque, the armature Armature resistance
voltage is set so that the rotor speed is 500 rpm. When Ra = 0.267Ω
the load torque is reduce to 50% of the full load value
Rotational losses = 600w

F06_Gate_EE_2015_978-93-325-7605-6_P-II.indd 167 6/19/2017 7:20:07 PM


clxviii | GATE 2015 Solved Paper Electrical Engineering: Set – 2

Supply voltage V = 230 V Z1 = 1 + j6;  Z2 = 0.8 + j4.8


Speed N = 1000 rpm SL = 1000 kVA
φZN P cos f = 0.8
Eb = × Z1 6.08
60 A ST2 = S × = 1000 ×
Z1 + Z 2 10.94
0.01 × 666 × 1000 4
Eb = × = 555.75 kVa
60 2
Hence, the correct Answer is (555.75 kVA).
Eb = 222 V
V − Eb 230 − 222 Question Number: 62 Question Type: MCQ
Ia = =
Ra 0.267 In the signal flow diagram given in the figure, u1 and
u2 are possible inputs whereas y1 and y2 are possible
= 29.96 A outputs. When would the SISO system derived from
Output power = Eb Ia - 600 this diagram be controllable and observable?
= 222 × 29.96 - 600 (A) When u1 is the only input and y1 is the only
= 6051.12 W output.
P 6051.12 (B) When u2 is the only input and y1 is the only
Output torque = out = output.
W 2π × 1000
(C) When u1 is the only input and y2 is the only
60
= 57.78 N-m output.
(D) When u2 is the only input and y2 is the only
Hence, the correct Answer is (57.78). output.
Question Number: 60 Question Type: NAT Solution:  From the given block diagram
A 200/400 V, 50 Hz, two-winding transformer is  X 1   5 −2   X1  1 0 
rated at 20 kVA. Its windings are connected as an  X  =  2 1   X  + 1 u1 +  1  u2
auto transformer of rating 200/600 V. A resistive load  2    2    
of 12Ω is connected to the high voltage (600 V) side  y1  1 0   X1 
of the auto-transformer. The value of equivalent load  y  = 1 −1  X 
resistance (in Ohm) as seen from low voltage side  2  2
is ______. By considering U2 is input and y1 is output. The state
Solution:  Transformation ratio equations becomes
600  X 1   5 −2   X1   0 
K= =3  X  =  2 −1   X  +  1  u2
200  2   2  
Resistive load = 12Ω
y1 = [1 0 ]  1 
X
Load resistance on primary side  X2 

12
= = 1.33Ω  5 −2   0   −2 
9 AB =  =
2 −1   1   1 
Hence, the correct Answer is (1.33Ω). 
 0 −2 
Question Number: 61 Question Type: NAT Qc = [ B AB ] = 
 1 1
Two single-phase transformers T1 and T2 each rated
at 500 kVA are operated in parallel. Percentage det (QC) = 2 (∴ controllable)
impedances of T1 and T2 are (1 + j6) and (0.8 + j4.8),  5 −2 
CA = [1 0 ]  = [ 5 −2 ]
respectively. To share a load of 1000 kVA at 0.8 lagging 2 1

power factor, the contribution of T2(in kVA) is _____.
C   1 0 
Qo =   = 
Solution:  Given A 5 −2 
  

F06_Gate_EE_2015_978-93-325-7605-6_P-II.indd 168 6/19/2017 7:20:11 PM


GATE 2015 Solved Paper Electrical Engineering: Set – 2 | clxix

det (Q0) = -2 (observable)


Hence, the correct option is (B).
Question Number: 63 Question Type: MCQ
The transfer function of a second order real system
with a perfectly flat magnitude response of unity has a
pole at (2 - j3). List all the poles and zeroes.
(A) Poles at (2 ± j3), no zeroes.
(B) Poles at (±2 - j3), one zero at origin.
(C) Poles at (2 - j3), (–2 + j3), zeroes at (–2 - j3), G1 G2
(A) +
(2 + j3) 1 − HG1 1 − HG2

(D) Poles at (2 ± j3), zeroes at (–2 ± j3)
G1 G2
Solution:  The second order real system for flat mag- (B) +
1 + HG1 1 + HG2
nitude is
G1 + G2
(C)
1 + H ( G1 + G2 )

G1 + G2
(D)
1 − H ( G1 + G2 )

Solution:  The given block diagram can be analyzed
by signal flow graph two forward paths of magnitude
= G1, G2.
Hence, the correct option is (D). Two loops of magnitude
Question Number: 64 Question Type: MCQ = -G1 H, –G2H
Y (s) By masons gain formula
Find the transfer function of the system given
X (s) G1 + G2
below. T  (s) =
1 + G1H + G2 H

Hence, the correct option is (C).

Question Number: 65 Question Type: MCQ


The open loop poles of a third order unity feedback
system are at 0, -1, -2. Let the frequency corresponding
to the point where the root locus of the system transits
to unstable region be K. Now suppose we introduce
a zero in the open loop transfer function at -3, while
keeping all the earlier open loop poles intact. Which
one of the following is TRUE about the point where the
root locus of the modified system transits to unstable
region?
(A) It corresponds to a frequency greater than
K
(B) It corresponds to a frequency less than K
(C) It corresponds to a frequency K
(D) Root locus of modified system never transits
to unstable region.

F06_Gate_EE_2015_978-93-325-7605-6_P-II.indd 169 6/19/2017 7:20:13 PM


clxx | GATE 2015 Solved Paper Electrical Engineering: Set – 2

Solution:  Given open loop transfer function K ( S + 3)


K T2 =
T1 = S ( S + 1) ( S + 2 )
S ( S + 1) ( S + 2 )

The root locus plot is like.
By adding zero the transfer function and root locus
plots

So, in second case the root locus plot never goes to


right hand side.
Hence, the correct option is (D).

F06_Gate_EE_2015_978-93-325-7605-6_P-II.indd 170 6/19/2017 7:20:14 PM


Unit 1
Electric Circuits and Fields

Chapter 1:  Network Elements 1.3


Chapter 2:  Network Graph 1.22
Chapter 3:  Network Theorems 1.24
Chapter 4:  Transient Response 1.33
Chapter 5:  Sinusoidal Steady State Analysis 1.44
Chapter 6:  Three-phase Circuits 1.60
Chapter 7:  Two Port Networks 1.64
Chapter 8:  Miscellaneous1.69
Chapter 9:  Fields1.76

Chapter 01.indd 1 11/10/2015 5:58:41 PM


Chapter 01.indd 2
Exam Analysis
Exam Year 92 93 94 95 96 97 98 99 00 01 02 03 04 05 06 07 08 09 10 11 12 13 14

1 Mark Questions 6 3 6 4 5 5 5 6 2 5 2 4 3 3 3 2 2 2 4 3 4 3 7

2 Marks Questions 2 – 2 – 1 9 2 7 3 4 7 11 6 6 5 11 10 6 4 4 5 4 6

4 Marks Questions – – 1 – – – – – – – – – – – – – – – – – – – –

5 Marks Questions – – – – – 5 5 4 2 1 4 – – – – – – – – – – – –

Total Marks 8 3 9 4 6 19 12 17 7 10 13 15 9 9 8 13 12 8 8 7 9 7 13

Network Elements 3 – – – 1 6 2 5 2 3 1 3 3 1 – 1 3 4 3 – – 2 5

Network Graph – – 1 – – – – – – – 1 – – – – 1 1 – – – – – –

Network Theorems – – 1 – – 2 1 – 1 – 1 3 – 1 1 – – 2 – 1 4 2 –

Transient Response 2 – 1 – 1 2 1 2 – 1 3 1 1 4 1 1 1 1 1 – 1 – 2

Sinusoidal Steady State – 2 2 2 2 1 4 5 1 2 4 1 2 1 1 3 – – – 5 3 1 2


Analysis

Three-phase Circuits 1 – 1 – – – – – – – – 1 – – – 2 – – – – – – –

Two Port Networks – – 1 1 – 1 – – 1 1 1 1 1 2 1 – – – 1 – 1 – –

Miscellaneous – – – – – 1 – 1 – – – – – – 1 2 4 – 2 – – – –

Fields 2 1 3 1 2 7 4 4 2 3 2 5 2 – 3 3 3 1 1 1 – 2 4

11/13/2015 2:06:24 PM
Chapter 1
Network Elements
Total power absorbed will be
One-mark Questions P = P1 + P2 + P3
1. The three circuit elements shown in the figure are = (100 − 640 − 30) W = 330 W.
part of an electric circuit. The total power absorbed
2. C0 is the capacitance of a parallel plate capacitor
by the three circuit elements in watts is 
with air as dielectric (as in Figure (a)). If, half of
 [2014-S1]
the entire gap as shown in Figure (b) is filled with a
10 A 8A
dielectric of permittivity εr, the expression for the
100 V 80 V
modified capacitance is [2014-S1]
15 V


(a) (b)
Solution: C
Consider the figure given below (a) 0 (1 + ε r ) (b)
(C0 + ε r )
2
Node
C
10 A 8A (c) 0 ε r (d)
C0 (1 + ε r )
2
100 V 80 V
Solution: (a)
15 V Capacitance of parallel plate capacitance will be
2A

ε 0 (1) A
C0 =
d
Applying Kirchoff’s current law at node, current Cmodified = C ′ + C ″
through 15 V voltage source = 2 A. ε (1) A ε 0 (ε r ) A
C′ = 0 +
Power absorbed by 100 V voltage source 2d 2d
(P1) = 10 × 100 = 1000 W. ε0 A  1 εr 
Cmodified = +
Power absorbed by 80 V voltage source is d  2 2 
P2 = −(80 × 8) = −640 W. C
Cmodified = 0 [1 + ε r ]
Power absorbed by 15 V voltage source is 2
P3 = −(15 × 2) = −30 W. Hence, the correct option is (a).

Chapter 01.indd 3 11/10/2015 5:58:42 PM


1.4 | Electric Circuits and Fields

3. Two identical coupled inductors are connected in Solution: (b)


series. The measured inductances for the two possi- For delta connection
ble series connections are 380 mH and 240 mH. Their
mutual inductance in mH is .  [2014-S2] Rb Rc
RA = .
Ra + Rb + Rc
Solution:
The two possible series connection are shown If each value in delta is scaled by a factor k, then,
below:
(i) Additive connection k 2 Rb Rc
RA =
k ( Ra + Rb + Rc )
L eqv = L1 + L2 + 2 M = 380 µH.
Rb Rc
L1 L2 =k⋅ .
I Ra + Rb + Rc

M If all elements of the delta connection are scaled by


a factor k, the elements of the corresponding star
(ii)
Subtractive connection
equivalent will be scaled by a factor of k.
L eqv = L1 + L2 − 2 M = 240 µH.
Hence, the correct option is (b).
Let the mutual inductance be M 5. If the electrical circuit figure (b) is an equivalent of
I
L1 L2 the coupled tank system of figure (a), then, [2010]

M

Now, we have
L1 + L2 + 2 M = 380 µH (1) B D

and A C
h1
h2
L1 + L2 − 2 M = 240 µH (2)
Solving Equations (1) and (2), we get,
4M = 140 mH.
140 (a)
A, B are resistances and C, D capacitances.
M = . (b)
A, C are resistances and B, D capacitances.
4
M = 35 mH. (c)
A, B are capacitances and C, D resistances.
(d)
A, C are capacitances and B, D resistances.
4. Consider a delta connection of resistors and its
equivalent star connection as shown below. If all Solution: (d)
elements of the delta connection are scaled by a fac- Consider the electrical equivalent circuit of the
tor k, k > 0, the elements of the corresponding star given system as shown below:
equivalent will be scaled by a factor of [2013] h1 b2
1 q2 R1 2 q1 R2
Ra RC RB

RA t(qr) C1 C2
Rb Rc

(a)
k2 (b)
k From the above circuit we conclude that A, C are
1 capacitances and B, D resistances.
(c) k
(d)
k Hence, the correct option is (d).

Chapter 01.indd 4 11/10/2015 5:58:43 PM


Chapter 1  Network Elements  |  1.5

6. As shown in the figure, a 1 Ω resistance is connected (a) Not possible (b) 4


across a source that has a load line v + i = 100. The (c) 3 (d) 2
current through the resistance is [2010] Solution: (d)
i Power consumed by a single lamp P = 200 W.
+
Source v 1Ω Voltage V = 220 V.
− If R is the resistance of a single incandescent lamp,
Now using the relation,
(a) 25 A (b) 50 A
V2
(c) 100 A (d) 200 A P=
So,         .
R
Solution: (b)
A resistor has linear characteristics, 2202
⇒ 200 = .
i.e., V = Ri R
⇒  V = i. ⇒ R = 242 Ω.
Load line If n number of lamps are connected in series across
V + i = 100. voltage V = 200 V,
i + i = 100. Total resistance of n lamps = nR = 242n.
Current through resistance Total power consumed

=i
100
= 50 A. V2
2
P=
Req
Hence, the correct option is (b)
2202
7. The current through the 2 kΩ resistance in the cir- ⇒ 100 = .
cuit shown in figure is [2009] 242n
⇒ n = 2.
1 kΩ C 1 kΩ
Hence, the correct option is (d).
A B
2 kΩ 9. The Thevenin’s equivalent of a circuit operating at 
ω = 5 rad/s,  has V∝ = 3.71 ∠ − 15.9° V and Z0 = 2.38
D − j0.667 Ω. At this frequency, the minimal realiza-
1 kΩ 1 kΩ
tion of the Thevenin’s impedance will have a  [2008]
(a) resistor and a capacitor and an inductor.
6V
(b) resistor and a capacitor.
(a) 0 mA (b) 1 mA (c) resistor and an inductor.
(c) 2 mA (d) 6 mA (d) capacitor and an inductor.
Solution: (a) Solution: (b)
Nodes C and D are at same potential, therefore, Given Voc = 3.71 ∠ − 15.9° V.
bridge is balanced. Therefore, current through 2 kΩ
Z 0 = 2.38 − j 0.667 Ω.
resistor is zero.
Hence, the correct option is (a). Impedance for resistor and inductor will be,

200 W Z = R + jX L .
8. How many incandescent lamps connected
220 V Impedance for resistor and capacitor will be,
in series would consume the same total power as a
100 W Z = R − jX c .
single incandescent lamp? [2009]
220 V Hence, the correct option is (b).

Chapter 01.indd 5 11/10/2015 5:58:44 PM


1.6 | Electric Circuits and Fields

10. In the figure given below the value of R is [2005] For t = 2 seconds to t = 4 seconds, energy absorbed
R
by the inductor will be
8A

4  di  4
EL2 = ∫ Li   dt = ∫ 2 ⋅ 6 ⋅ 0 dt = 0 J.
100 V 10 Ω 10 Ω 2
 dt  2

In a pure inductor, energy is only stored, it does not


dissipates energy. Some energy is dissipated in the
(a) 2.5 Ω (b) 5.0 Ω
resistor due to resistance. We know that the total
(c) 7.5 Ω (d) 10.0 Ω
energy absorbed by the inductor will be the sum of
Solution: (c) energy stored in the inductor and the energy dis-
Voltage V = 100 V. sipated in the resistor.
Current I = 8 A. We can calculate the energy dissipated by the
When viewed from voltage source, resistance, resistance in 4 seconds as given below
V 100
R
= = = 12.5. T 2 4
ER = ∫ i 2 R dt = ∫ (3t ) 2 × 1 dt + ∫ 62 × 1 dt
I 8 0 0 2

The total resistance can be calculated as, 2


2 4 t3
= ∫ (9t ) dt + 36 ∫ 1 dt = 9 ×
4
R + 10 10 = 12.5
2
+ 36 t 2
0 2 30
∴ R = 12.5 − 10 10 = 12.5 − 5 = 7.5 Ω.
Hence, the correct option is (c). 8
= 9 ×   + 36 × 2 = 24 + 72 = 96 J.
1 1. Figure shows the waveform of the current passing 3
through an inductor of resistance 1 Ω and induct-
Now, the total energy absorbed by the inductor in
ance 2 H. The energy absorbed by the inductor in
4 seconds will be
the first four seconds is [2003]
= 96 J + 36 J = 132 J.

6A Hence, the correct option is (c).


12. A segment of a circuit is shown in figure vR = 5 V,
t vC = 4 sin 2t. The voltage vL is given by [2003]
0 2s 4s
(a) 144 J (b) 98 J
Q
(c) 132 J (d) 168 J
Solution: (c) +
For 0 < t < 2s current varies linearly with time and 5Ω vR
given as i(t) = 6 A.
2A − 1F
For first 2 seconds, energy absorbed by the induc-
P + − R
tor neglecting the resistance will be, vC
+
T di 2H vL
EL = ∫ Li dt = EL1 + EL 2 .
0 dt −
2  di  2
EL1 = ∫ Li   dt = ∫ 2 × 3t × 3 dt S
0
 dt  0

2
2 t2 4  (a) 3 - 8 cos 2t (b) 32 sin 2t
= 18∫ t dt = 18 × = 18 ×  − 0  = 36 J.
0 2 0 2  (c) 16 sin 2t (d) 16 cos 2t

Chapter 01.indd 6 11/10/2015 5:58:45 PM


Chapter 1  Network Elements  |  1.7

Solution: (b) For series connection, current through both the


Consider the labelled figure given below bulbs will be same.
Hence, the 40 W bulb glows brighter.
Q
Therefore, the correct option is (d).
1A 14. Given two coupled inductors L1 and L2, their
+
mutual inductance M satisfies [2001]
5Ω vR
( L + L2 )
− M = L12 + L22 (b)
(a) M > 1
2A 1F iC 2
P R
O + − (c) M ≤ L1 L2
M > L1 L2 (d)
+ vC
Solution: (d)
2H vL
The mutual inductance is given by relation

iL
M = K L1 L2 ,
S
Now K = coefficient of coupling and its value lies
between 0 and 1, that is, 0 < K < 1.
vC (t ) = 4 sin(2t )
dvC (t ) ∴ M ≤ L1 L2 .
iC (t ) C=
= 8 cos( 2t ).
dt
Hence, the correct option is (d).
For capacitance C = 1 F.
15. A voltage waveform v(t ) = 12t 2 is applied across a
If we apply KCL at node O, then the current
1 H inductor for t ≥ 0, with initial current through
through inductance can be expressed as
it being zero. The current through the inductor for
iL (t ) = 1 + 2 − 8 cos( 2t ) = 3 − 8 cos(2t ). t ≥ 0 is given by [2000]
(a) 12t (b) 24t
d (c) 12t3 (d) 4t3
∴ vL (t ) = L iL (t ) = 32 sin(2t ).
dt
Solution: (d)
Hence, the correct option is (b). For an inductor, the relationship between voltage
13. Two incandescent light bulbs of 40 W and 60 W and current is
rating are connected in series across the mains. t
Then [2001] d 1
v(t ) = L i (t )  or  i (t ) = ∫ v(t ) dt.
(a) the bulbs together consume 100 W. dt L −∞
(b) the bulbs together consume 50 W.
For v(t) = 12t2 for t ≥ 0, the current through the
(c) the 60 W bulb glows brighter.
inductor, L = 1 H is
(d) the 40 W bulb glows brighter.
t
Solution: (d) 1
The relationship between power and resistance is
i (t ) =
10∫ 12 t 2 dt for t≥0

as given below
t3
1 = 4 t 3 A.
= 12
∵ P∝ . 3
R
From the above relation we conclude that resist- Hence, the correct option is (d).
ance and power are inversely related. Therefore, 16. For the circuit shown in figure, the capacitance
resistance of 40 W bulb will be greater than resist- measured between terminals B and Y will be
ance of 60 W bulb.  [1999]

Chapter 01.indd 7 11/10/2015 5:58:47 PM


1.8 | Electric Circuits and Fields

R Solution: (c)
For an inductor the relationship between voltage
Cc Cc
and current will be,
t
d 1
B Y v(t ) = L i (t )  or  i (t ) = ∫ v(t ) dt.
dt L −∞
Cc
Cs Cs When v(t ) is an ideal impulse = δ (t ),
1
=i (t ) constant
= ,
L
t

C
Cc +  s
(a)
 C 
Cs +  c 
As  ∫ δ (t ) dt = 1 for t ≥ 0.
 (b) −∞
 2   2 
Therefore, v(t) must be an ideal impulse.
(C + 3Cc ) Hence, the correct option is (c).
(c) s 3Cc + 2Cs
(d)
2 18. An ideal voltage source will change an ideal
capacitor [1997]
Solution: (c)
Simplifying the given circuit in problem we get, (a) in infinite time (b) exponentially
(c) instantaneously (d) None of these
Cc
Solution: (c)
B Y
Voltage and current are related as given below for
charging of a capacitor.
Cs Cc Cs Cc t
1
V (t ) = ∫ I (t ) dt.
C −∞
R t
CV (t ) = ∫ I (t ) dt.
−∞
From the above circuit, the net capacitance be-
tween points B and Y will be, Differentiating both siddes of above equation
wrt t we get,
C + CC C + 3CC
CBY = S + CC = S . d d
t
2 2 [CV (t )] = ∫ I (t ) dt.
dt dt −∞∞
Hence, the correct option is (c).
dV (t )
1 7. In the circuit shown in figure, it is desired to have a C = I (t ).
constant direct current i(t) through the ideal induc- dt
tor L. The nature of the voltage source v(t) must be dV (t )
I (t ) = C .
 [1998] dt
i(t ) From the above expression we conclude that an
ideal voltage source will charge an ideal capacitor
instantaneously.
v (t ) L
Hence, the correct option is (c).
19. A practical current source is usually represented
by [1997]
(a) constant voltage (a) a resistance in series with an ideal current
(b) linearly increasing voltage source.
(c) an ideal impulse (b) a resistance in parallel with an ideal current
(d) exponentially increase voltage source.

Chapter 01.indd 8 11/10/2015 5:58:48 PM


Chapter 1  Network Elements  |  1.9

(c) a resistance in parallel with an ideal voltage C C


source. = [− cos 4π + cos 0] = [−1 + 1] = 0.
2 2
(d) None of the above
Solution: (b) Hence, the correct option is (c).
Figure given below shows a practical current 2 1. The V-I characteristic as seen from the terminal
source which is represented by a resistance in par- pair (A, B) of the network of Figure (1) is shown
allel with an ideal current source. in Figure (2). If an inductance of value 6 mH is
A connected across the terminal pair (A, B), the time
constant of the system will be [1996]
R I
Network of I
Linear Resistors + A
B V
and Independent −
Sources B
Hence, the correct option is (b).
2 0. Energy stored in a capacitor over a cycle, when
excited by an AC source is [1997] Figure 1
(a) the same as that due to a DC source of equiva-
I
lent magnitude.
(b) half of that due to a DC source of equivalent
magnitude. 4 mA
(c) zero.
(d) None of the above
Solution: (c) V
(0, 0)
When excited by an AC source, energy stored in a 8V
capacitor over a complete cycle is zero. Figure 2
Consider the instantaneous voltage across the ca-
(a) 3 ms
pacitor to be V (t ) = sin ωt , then the instantaneous
(b) 12 sec
current across the capacitor can be expressed as
(c) 32 sec
dV (t ) d (sin ωt ) (d) unknown, unless the actual network is specified
I (t ) = C =C
dt dt Solution: (a)
= C (ω cos ωt ) = ωC cos ωt. Inductance L = 6 mH.
Consider the V-I graph given below:
Instantaneous power across the capacitor can be
I
calculated as
P(t ) = I (t )V (t ) 4 mA
= ωC cos ωt × sin ωt = ωC sin ωt cos ωt
ωC
= sin 2ωt [since, sin 2ωt = 2 sin ωt cos ωt ].
2 V
(0, 0)
For one complete cycle, the energy stored in the 8V
capacitor will be Resistance between the terminals A and B can be
calculated
2π /ω 2π /ω
ωC
E= ∫ P(t ) dt = ∫ 2
sin 2ωt dt
RAB =
∆V
=
8 V−0 V
0 0
∆I 4 mA − 0 mA
2π /ω
ωC 2π /ω ωC  − cos 2ωt  8
=
2 0 ∫ sin 2ωt dt =
2  2ω  0
=
4 × 10−3
Ω = 2 × 103 Ω.

Chapter 01.indd 9 11/10/2015 5:58:49 PM


1.10 | Electric Circuits and Fields

Time constant of the system after connecting the 23. All the resistances in figure are of 1 Ω each. The
inductance can be calculated using value of current I is [1992]

L 6 × 10−3 H
τ= =
RAB 2 × 103 Ω
I
= 3 × 10−6 s = 3 µs.
Hence, the correct option is (a). +
1V
2 2. All resistances in the circuit in figure are of R Ω −
each. The switch is initially open. What happens
to the lamp’s intensity when the switch is closed?
 [1992]
220 V 1 2
Lamp (a) A (b) A
+ − 15 15
4 8
(c) A (d) A
15 15
Solution: (d)
Consider the labelled circuit given below

(a) Increases D
C
(b) Decreases E
(c) Remains the same I B
F
(d) Answer depends on the value of R A H
Solution: (c) G
+
Redraw the circuit given in problem as given below 1V
− J
220 V Lamp
+ −

B For upper half of the circuit:
S Resistance between points D and F will be parallel
A C combination of two 1 Ω resistances.

1× 1
RDF = = 0.5 Ω.
D 1+1
Closed loop ABCDA forms a Wheatstone bridge Resistance between path C, D and F will be series
and the bridge is balanced, therefore, no current combination of 1 Ω and RDF.
will flow through the arm BD. Hence, the current
and voltage through BD will be zero. RCDF = 1 + 0.5 = 1.5 Ω.
Intensity of the lamp depends upon the current Resistance between points E and F will be parallel
flowing through the lamp. The opening or closing combination of two 1 Ω resistances.
of switch will not have any effect on the current
through the lamp because the equivalent resistance 1× 1
REF = = 0.5 Ω.
between terminals A and C will be same since, 1+1
the bridge is balanced. Therefore, the intensity of Resistance between path C, E and F will be series
the lamp remains unchanged when the switch is combination of 1 Ω and and REF.
closed.
Hence, the correct option is (c). RCEF = 1 + 0.5 = 1.5 Ω.

Chapter 01.indd 10 11/10/2015 5:58:50 PM


Chapter 1  Network Elements  |  1.11

Total resistance between points C and F will be 4.5m[t − 26] = 11.


parallel combination of RCDF and RCEF. 11
⇒ t − 26 = .
1.5 × 1.5 3 4.5m
RCF = = Ω. ⇒ tfinal = 2470.4 °C.
1.5 + 1.5 4
Total resistance between points B, C, D and F will  I
2. In the figure, the value of resistor R is  25 +  Ω,
be series combination of 1 Ω resistance and RCF.  2
where I is the current in amperes. The current I
3 7 is  [2014-S1]
RBCDF = 1 + Ω = Ω.
4 4
I
Similarly, the total resistance between points B, G, R
300 V
J and F will be
3 7
RBGJF = 1 + Ω = Ω.
4 4 Solution:
 I
Total resistance between points B and F will be Resistance R =  25 +  Ω.
 2
parallel combination of RBCDF and RBGJF.
Current I = (2R - 50).
7 7 Consider the figure given below
Ω× Ω
RBF = 4 4 = 7 Ω.
7 7 I
Ω+ Ω 8
4 4 300 V R
Net resistance between points A and F will be

7 15 Applying KVL in given loop, we have,


RAF = 1 + = Ω.
8 8 300 = IR.
The current I can be calculated as 300 = (2R - 50) × R.
V 1 8 R = 30 Ω.
I= = = A. R = -5 Ω.
RAF  15  15
 
8 Neglecting negative value of resistance we get
R = 30 Ω.
Hence, the correct option is (d).
Thus, the required current
I = (2 R − 50) = (2 × 30 − 50) A = 10 A.
Two-marks Questions
3. Three capacitors C1, C2 and C3 whose values are
1. An incandescent lamp is marked 40 W, 240 V. If 10 mF, 5 mF and 2 mF respectively, have breakdown
resistance at room temperature (26 °C) is 120 Ω voltages of 10 V, 5 V and 2 V respectively. For the
and temperature coefficient of resistance is 4.5 interconnection shown, the maximum safe voltage
× 10-3/°C, then its ‘ON’ state filament tempera- in Volts that can be applied across the combination
ture in °C is approximately  [2014-S1] and the corresponding total charge in mC stored in
Solution: the effective capacitance across the terminals are
For ON state respectively [2013]
C2 C3
[V ]2 (240) 2
RON = = = 1440 Ω.
P 40
Rt = R0 [1 + α t ].
1440 = 120[1 + 4.5 × 10−3 [t − 26]1 + 4.5m[t − 26]] = 12.
C1

Chapter 01.indd 11 11/10/2015 5:58:51 PM


1.12 | Electric Circuits and Fields

(a) 2.8 and 36 (b) 7 and 119 (a) 4 (b) 6


(c) 2.8 and 32 (d) 7 and 80 (c) 8 (d) 4
Solution: (c) Solution: (b)
We now show that charge is expressed in terms of Consider the figure given below
capacitance and voltage as 1Ω R
Q = CV . 1A +
1A
−6
Q1 = C1V1 = 10 × 10 × 10 = 100 µC. 2A 12 Ω 12 V 6V
−6 −
Q2 = C2V2 = 5 × 10 × 5 = 25 µC.
Q3 = C3V3 = 2 × 10−6 × 2 = 4 µC. From the loop equation we get

Capacitors C2 and C3 are in series and in series 12 = 1× R + 6.


charge is same. So, the maximum charge on C2 R = 6 Ω.
and C3 will be minimum of (Q2, Q3) = min (25 mC,
Hence, the correct option is (b).
4 mC) = 4 mC = Q23.
5. The equivalent capacitance of the input loop of the
Equivalent capacitance of C2 and C3 in series will
circuit shown is [2009]
be
i1 1 kΩ 1 kΩ
CC 5 × 2 10
C23 = 2 3 = = µF.
C2 + C3 5 + 2 7 Input 1 kΩ 49i1 100 μF
loop
Equivalent voltage
100 mF
Q 4 × 10−6 28
V23 = 23 = = = 2.8 V.
C23 10 × 10−6 10 (a) 2 mF (b) 100 mF
7 (c) 200 mF (d) 4 mF
In parallel, the voltage is same, therefore, Solution: (a)
Consider the figure given below
V
=1 V=
23 2.8 V. i1 1 kΩ 1 kΩ

Charge in capacitor C1,


Input 1 kΩ 49i1 100 μF
Q1 = C1V1 = 10 × 10−6 × 2.8 = 28 µC. V1(t) loop

The total charge in parallel will be 50 i1 100 μF

Q = Q1 + Q23 .
From loop equation we get
Q = 4 + 28.
1
Q = 32 µC. V1 = 2 × 103 i1 + ∫ 50i1 (t ) dt ,
C1
Hence, the correct option is (c). where capacitance C1 = 100 mF.
20. If the 12 Ω resistor draws a current of 1 A as shown 1
C2 ∫
Also V1 = 2 × 103 i1 + i1 (t ) dt.
in the figure, the value of resistance R is [2010]
The equivalent capacitance of the input loop of the
1Ω R C1
circuit C2 = .
50
12 Ω
100
2A 1A 6V For C1 = 100 µF, C2 = = 2 µF.
50
Hence, the correct option is (a).

Chapter 01.indd 12 11/10/2015 5:58:52 PM


Chapter 1  Network Elements  |  1.13

4. For the circuit shown, find out the current flow- 7. In the circuit shown in the figure, the value of the
ing through the 2 Ω resistance. Also identify the current i will be given by [2008]
changes to be made to double the current through 3Ω
1Ω a b
the 2 Ω resistance. [2009] V
+ ab −
i +
5V + 1Ω 1Ω − 4Vab

VS = 4 V +
− IS = 5 A 2Ω

(a) 0.31 A (b) 1.25 A


(a) (5 A; Put VS = 20 V) (c) 1.75 A (d) 2.5 A
(b) (2 A; Put VS = 8 V) Solution: (b)
(c) (5 A; Put IS = 10 A) Consider the figure given below
(d) (7 A; Put IS = 12 A)
1Ω Va 3Ω
Solution: (b)
Voltage across 2 Ω resistance = VS = 4 V.
Vs 4 5V
+
1Ω 1Ω +
4Vab
Current through 2 Ω resistance = = = 2 A. − −
R 2 i1 i2
When connected across voltage source, current
source has no effect.
To double current though 2 Ω resistance, voltage
Applying KVL in loop (1)
source is doubled, i.e., VS = 8 V.
Hence, the correct option is (b). 5 − i1 − i1 = 0.
6. Assuming ideal elements in the circuit shown 5
below, the voltage vab will be [2008] i1 == 2.5 A.
2
a 2Ω ∴ Va = 2.5 V.
+ + −
Applying KVL in loop (2)
vab −
1A + 5V 4Vab = 3i2 + i2 .
i
− 4Vab
b
i2 = = Vab .
4
(a) -3 V (b) 0 V i2 = Vab .
(c) 3 V (d) 5 V
Solution: (a) ∴ Vb = 1× i2 = Vab .
Consider the figure given below Vb = Va − Vb .
a + 2V − Va 2.5
Vb = = = 1.25 V.
+ 2Ω 2 2
− 5V
i2 = i = Vab = Vb .
1A vab
+
i2 = 1.25 A.

b Hence, the correct option is (b).
From the above circuit we get 8. A 3 V DC supply with an internal resistance of 2 Ω
supplies a passive non-linear resistance character-
vab = 2 − 5 = −3 V. ized by the relation VNL = I2NL. The power dissipated
Hence, the correct option is (a). in the non-linear resistance is [2007]

Chapter 01.indd 13 11/10/2015 5:58:53 PM


1.14 | Electric Circuits and Fields

(a) 1.0 W (b) 1.5 W Solution: (c)


(c) 2.5 W (d) 3.0 W Consider the figure given below
Solution : (a) 1 A + 10 V − − 12 V + P 2A
Consider the circuit given below: 10 Ω 3A 6Ω
+
I NL 1A +
6Ω 18 V Vs −
+

R Q
DC Non-linear From the above circuit we have
supply V NL
resistor
E Vs = 12 + 18 = 30 V.
Hence, the correct option is (c).
1 0. In the figure, Ra , Rb , and Rc  are 20 Ω, 10 Ω and
10 Ω respectively. The resistances R1, R2 and R3 in
VNL = I NL
2
(1) Ω of an equivalent star-connection are [2004]
a a
VNL = E − I NR R.
R1
Rb Rc
We have E = 3 V and R = 2 Ω.
R3 R2
VNL = 3 − 2 I NL = I NL
2
.
c Ra b c b
I 2
+ 2 I NL − 3 = 0.
NL
(a) 2.5, 5, 5 (b) 5, 2.5, 5
I 2
NL = −3A or 1 A. (c) 5, 5, 2.5 (d) 2.5, 5, 2.5
The non-linear resistor is passive and the only ac- Solution: (a)
tive element in the circuit is 3 V DC supply. There- a
a
fore, -3 A is rejected.
Rb Rc R1
So, I NL = 1 A. ∆−Y
R3 R2
Power dissipated in the non-linear resistor will be Trasformation
c Ra b c b
= V=
NL I NL I I= I= I 2
NL NL
3
NL
3
Resistance Ra = 20 Ω.
= 1 W. Resistance Rb = 10 Ω.
Hence, the correct option is (a). Resistance Rc = 10 Ω.
9. In the figure, the value of the source voltage is Now we have
 [2004] Rb Rc 10 × 10
R1 = = = 2.5 Ω.
Ra + Rb + Rc 20 + 10 + 10
10 Ω 6Ω P 2A
Hence, the correct option is (a).
1 1. In the figure, the value of resistance R in Ω is
1A 6Ω +  [2004]
− 10 Ω 2A

Q
10 Ω R
100 V +

(a) 12 V (b) 24 V
(c) 30 V (d) 44 V

Chapter 01.indd 14 11/10/2015 5:58:54 PM


Chapter 1  Network Elements  |  1.15

(a) 10 (b) 20 Potential difference between node x and y = 60 V.


(c) 30 (d) 40 By taking KCL at node y we get,
Solution: (b) 40 − 30
−I − 5 + = 0.
Consider the figure given below: 1
∴ 1 = 5 A.
R
2+
5 10 Ω 60
2A ∴ R= = 12 Ω.
5
+
Hence, the correct option is (d).
10 Ω R 2R
100 V +
− 13. Consider the star network shown in figure. The
R −
resistance between terminals A and B with C open
5
is 6 Ω, between terminals B and C with A open is
Using loop equation, we get 11 Ω, and between terminals C and A with B open
 R is 9 Ω. The values of RA, RB, RC are: [2001]
10  2 +  + 2 R = 100.
 5 A

20 + 2 R + 2 R = 100 ⇒ 4 R = 80.
RA
R = 20 Ω.
Hence, the correct option is (b). RB RC
1 2. In the figure, the value of R is [2003] B

C

14 Ω 1Ω (a) RA = 4 Ω, RB = 2 Ω, RC = 5 Ω
(b) RA = 2 Ω, RB = 4 Ω, RC = 7 Ω
10 A 5A
+ + (c) RA = 3 Ω, RB = 3 Ω, RC = 4 Ω
100 V − 2Ω − 40 V
(d) RA = 5 Ω, RB = 1 Ω, RC = 10 Ω
Solution: (b)
(a) 10 Ω (b) 18 Ω The resistance between terminals A and B with
(c) 24 Ω (d) 12 Ω with C open, will be
Solution: (d) RAB = RA + RB = 6 Ω (1)
By KCL,
The resistance between terminals B and C with A
VP − 40 VP − 100 VP open, will be
∴ + + = 0.
1 14 2
22VP = 660. RBC = RB + RC = 11 Ω (2)
The resistance between terminals C and A with B
∴ VP = 30 V.
open, will be
RΩ I
RCA = RC + RA = 9Ω (3)
14 Ω P 1Ω Solving equations (1) and (2) we get
x y

5A
RC − RA = 5 (4)
10 A
+ Solving equations (3) and (4) we get
+ 2Ω V 40 V
100 V V − 2 RC = 14, RC = 7 Ω.

RA = 2 Ω, RB = 4 Ω.

Hence, the correct option is (b).

Chapter 01.indd 15 11/10/2015 5:58:56 PM


1.16 | Electric Circuits and Fields

14. The circuit shown figure is equivalent to a load of (a) (b)


 [2000]
I 2Ω (c) (d)

+ 2I
Solution: (d)
4Ω − Consider the figure shown below which shows the
relevant circuit.
i
4 8 iR iC
(a) Ω (b) Ω +
3 3 1F
v(t ) 1Ω R C
(c) 4 Ω (d) 2 Ω

Solution: (b)
I I2 2Ω
+ The input is shown in figure given below:
I1
v(t )
4Ω + 2I
V −
1V

− t
0 1 2
Current through 4 Ω resistor, will be
v(t ) dv
V i = iR + iC = + 1⋅ .
I1 = . 1 dt
4
dv
Current through 2 Ω resistor, will be is shown in figure given below:
dt
V − 2I dv (t)
I2 = . dt
2
Total current 1

V V − 2I t
I = I1 + I 2 = + . −2 −1 0 1 2
4 2 −1
V V
⇒ I = + − 1. The figure given below shows current through the
4 2
source i(t).
3
⇒ 2I = V . i(t)
4

V 8 2
1
Load = = Ω.
I 3 t
−2 0 1 2
Hence, the correct option is (b). −1 −1
15. When a periodic triangular voltage of peak ampli-
tude 1 V and frequency 0.5 Hz is applied to a Hence, the correct option is (d).
parallel combination of 1 Ω resistance and 1 F 1 6. When a resistor R is connected to a current
capacitance, the current through the voltage source source, it consumes a power of 18 W. When the
has waveform [1999] same R is connected to a voltage source having

Chapter 01.indd 16 11/10/2015 5:58:57 PM


Chapter 1  Network Elements  |  1.17

the same magnitude as the current source, the Solution: (d)


power absorbed by R is 4.5 W. The magnitude of Consider the figure given below,
the current source and the value of R are [1999]
(a) 18A and 1 Ω (b) 3 A and 2 Ω cos(wt )

(c) 1 A and 18 Ω (d) 6 A and 0.5 Ω


6
sin(wt)
Solution: (b) q
Consider the figure given below in which a resistor q = tan−1 4
10
R is connected to a current source, 3
8 I3 = 53.13°

−cos(wt )
I R

Current I1 = −6 sin(ωt ) mA,


and current I 2 = 8 cos(ωt ) mA.
P I=
= 2
R 18 W. Now we have,
Now, when resistor R is connected to a voltage I 3 = − I1 − I 2 = 6 sin(ωt ) − 8 cos(ωt ).
source
The relevant phasor diagram is shown in figure.
I 3 = 10 sin(ωt − 53.13°).
+ R
V − I 3 = 10 cos(ωt + 180 + 90 − θ )
= 10 cos(ωt + 180 + 36.87°)
= −10 cos(ωt + 36.87°).
V2
P
= = 4.5 W.
R Hence, the correct option is (d).
We know that V = I (in magnitude), therefore, 18. The effective inductance of the circuit across the
terminals A, B in the figure shown below is [1998]
⇒ I 2 R = 18 (1)
A 4H

I = 4.5 (2)
2

R 1H
3H 5H
Solving Equations (1) and (2), we get
2H
B
I = 3 A. 6H
R = 2 Ω.
(a) 9 H (b) 21 H
Hence, the correct option is (b). (c) 11 H (d) 6 H
17. Currents I1, I2 and I3 meet at a junction (node) in a Solution: (c)
circuit. All currents are marked as entering the node. Consider the simplified circuit shown below,
If I1 = −6 sin(ωt ) mA and I 2 = 8 cos(ωt ) mA, then L1
I3 will be [1999] A

10 cos(ωt + 36.87) mA
(a) M12
M13 L2
14 cos(ωt + 36.87) mA
(b)
−14 sin(ωt + 36.87) mA
(c) M23
B
−10 cos(ωt + 36.87) mA
(d) L3

Chapter 01.indd 17 11/10/2015 5:58:58 PM


1.18 | Electric Circuits and Fields

The effective inductance of the circuit across the Solution:


terminals A, B, I 1 Ω V1 I2 1 Ω 8V

L = L1 + L2 + L3 − 2 M 12 + 2 M 23 − 2 M 13 I1 2A I3

= 4 + 5 + 6 − (2 × 1) + (2 × 2) − (2 × 3) = 11 H. E
+
6Ω 4Ω 1Ω

Hence, the correct option is (c).
1 9. A 10 V battery with an internal resistance of 1 Ω
is connected across a non-linear load whose V - I
characteristic is given by 7 I = V 2 + 2V .  The cur-
rent delivered by the battery is A. [1997] Voltage across 4 Ω resistor = 4 × 2 = 8 V.
Solution: 8
Current through 1 Ω resistor, I 3= = 8 A.
Consider the circuit shown below 1
I I 2 = I 3 + 2 = 10 A.
+
I V1 = 8 + 1× 10 = 18 V.
1Ω Non linear
V load Current through 6 Ω resistor,
10 V −
18
I1
= = 3 A.
6
Form non-linear load characteristic we have,
Current through 1 Ω resistor,
7 I = V 2 + 2V . (1)
I = I1 + I 2 = 3 + 10 = 13 A.
From loop equation we have
E = V1 + I ⋅1.
I + V = 10. (2) E = 18 + 13 × 1 = 31 V.
Solving equation (1) and (2), we get, 21. The voltage and current waveforms for an ele-
ment are shown in figure. The circuit element is
V + 2V = 7(10 − V )
2
and its value is . [1997]
 V + 2V = 70 − 7V
2
i(t)
V 2 + 9V − 70 = 0
2A
(V − 5)(V + 14) = 0
V = 5 V , − 14 V .
0 2s Time
As V cannot be negative, therefore neglecting the
negative value and thus, V is taken as 5 V. v(t)

∴ I = 10 − V = 10 − 5 = 5 A. 2V

20. The value of E and I for the circuit shown in figure,


are V and A. [1997]
0 2s Time
I 1Ω 1Ω
Solution:
2A
Inductance L = 2 H.
+ 6Ω 4Ω 1Ω For the given i(t),
E −
di (t )
v(t ) = K ,
dt

Chapter 01.indd 18 11/10/2015 5:59:00 PM


Chapter 1  Network Elements  |  1.19

where, K = 2. Solution:
di (t ) L1
Compare with v(t ) = L . A
dt
M12
Therefore, the circuit element is an inductor with M3 L2
L = 2 H. M23
22. The equivalent inductance seen at terminals A-B
B
in figure is H. [1992] L3

4H
Equivalent inductance seen at terminals A-B will
A be
2H
L = L1 + L2 + L3 − 2 M 12 + 2 M 23 − 2 M 13
1H 4H
1H = 4 + 4 + 4 − (2 × 2) + (2 × 1) − (2 × 1)
B
4H = 8 H.

Chapter 01.indd 19 11/10/2015 5:59:00 PM


1.20 | Electric Circuits and Fields

I1, I2 and I3. Evaluate the power developed in the


Five-marks Questions 10 V voltage source. [1999]

1. In the resistor network shown in figure, all resis- 2V



tor values are 1 Ω. A current of 1 A passes from − +
terminal a to terminal b, as shown in the figure.
4Ω I2 6Ω
Calculate the voltage between terminals a and b.
[Hint: You may exploit the symmetry of the circuit]
 [2002] + 2A I3 +
12 V − − 10 V
I1
1A
b

Solution:
Consider the figure given below:

2V

− +

4Ω I2 6Ω
R=1Ω

+
+ Vx 2A I3 +
a 12 V − − 10 V
I1 −
1A

Solution:
Consider the figure given below Equation for Loop 1 can be expressed as
4( I1 − I 2 ) + Vx = 12 (1)
f 1 Ω 0.5 A 1A
b
0.5 A Equation for Loop 3 can be expressed as

1Ω 1Ω 1Ω Vx = 6( I 3 − I 2 ) + 10 (2)
0.5 A Solving Equations (1) and (2) we get

d e 4( I1 − I 2 ) + 6( I 3 − I 2 ) + 10 = 12
0.5 A
or 4 I1 − 10 I 2 + 6 I 3 = 2
1Ω 1Ω R=1Ω
or 2 I1 − 5 I 2 + 3I 3 = 1 (3)
0.5 A
1Ω Ix Equation for Loop 2 can be expressed as
a c
1A 0.5 A 6( I 2 − I 3 ) + 4( I 2 − I1 ) + 2 I 2 = 2 (4)

Writing the equation for loop acb,we get or    −4 I1 + 12 I 2 − 6 I 3 = 2


or −2 I1 + 6 I 2 − 3I 3 = 1 (5)
0.5 + I x = 0.5, I x = 0. and  I 3 − I1 = 2 (6)
Vab = Vac + Vcc + Vcb = 1.5 V. Solving Equations (3) and (6) we get
2 I1 − 5 I 2 + 3I1 + 6 = 1
2. Solve the circuit shown in figure using the mesh
method of analysis and determine the mesh currents  or I1 − I 2 = −1 (7)

Chapter 01.indd 20 11/10/2015 5:59:02 PM


Chapter 1  Network Elements  |  1.21

Solving Equations (5) and (6), we get Using Equation (6), we get I3 = 3 A.
−2 I1 + 6 I 2 − 3I1 − 6 = 1 ∴ I1 = 1 A, I 2 = 2 A, I 3 = 3 A.
or   −5 I1 + 6 I 2 = 7 (8) Power developed in 10 V voltage source = 10.
Solving Equations (7) and (6), we get I 3 = 10 × 3 = 30 W,
or,  −5 I1 + 6 + 6 I1 = 7 i.e., 10 V source is absorbing 30 W of power.
= I1 1= A, I 2 2 A.

Chapter 01.indd 21 11/10/2015 5:59:02 PM


Chapter 2
Network Graph
Solution: (a)
One-mark Questions Let the number of nodes = N and number of
branches = B.
1. The number of chords in the graph of the given Number of tree branches (or twigs) = (N - 1).
circuit will be [2008]
Number of co tree branches = Number of links (or
chords) = L.
  B = (N - 1)  or  L = B - (N - 1) = B - N + 1.
±
Hence, the correct option is (a).
3. Figure shows a DC resistive network and its graph
is drawn aside. A ‘proper tree’ chosen for analyz-
(a) 3 (b) 4
ing the network will contain the edges: [1994]
(c) 5 (d) 6
Solution: (a)
b b
6 a c a c

2 +
1 3
4 5

1 2 3
d d

4 (a) ab, bc, ad (b) ab, bc, ca


(c) ab, bd, cd (d) ac, bd, ad
Number of branches = b = 6. Solution: (a)
Number of nodes = n = 4. Consider the sub-graphs shown below,
Number of chords = b - (n - 1) (a)
a b c (b)
a b c
= 6 - (4 - 1) = 3.
Hence, the correct option is (a).
d
2. The graph of an electrical network has N nodes and
B branches. The number of links, L, with respect to (c)
a b (d)
a b c
the choice of a tree, is given by [2002] c
(a) B - N + 1 (b) B + N
(c) N - B + 1 (d) N - 2B -1 d d

Chapter 02.indd 22 11/10/2015 5:27:21 PM


Chapter 2  Network Graph  |  1.23

The sub-graphs with the set of edges in option (a), Solution: (b)
(c) and (d) qualify as proper trees. Convention
The sub-graphs with the set of edges in option (b) bj
Ni ⇒ +Ve or +1
does not qualify as a proper tree because it does
not contain all the four nodes of the original graph [When branch bj leaves away node Ni]
and it forms a closed path. bj
Ni ⇒ −Ve or −1
Hence, the correct option is (a).
[When branch bj enters node Ni]
Node incidence matrix is given below
Branches
Two-marks Questions Nodes
b1 b2 b3 b4 b5 b6

N1 1 1 1 0 0 0
1. The matrix A given below is the node incidence
matrix of a network. The columns correspond to N2 0 -1 0 -1 1 0
branches of the network while the rows correspond N3 -1 0 0 0 -1 -1
to nodes. Let V = [v1 v2 … v6]T denote the vector
N4 0 0 -1 1 0 1
of branch voltages while I = [i1 i2 … i6]T that of
branch currents. The vector E = [e1 e2 e3 e4]T denotes For example, Branch b1 enters N3 and leaves N1.
the vector of node voltages relative to a common
ground. [2007] The oriented network graph
b3
N1 N4
1 1 1 0 0 0
 0 −1 0 −1 1 0 
A=  b1
 −1 0 0 0 −1 −1 b2 b6
 
 0 0 −1 1 0 1  b4

Which of the following statements is true?


(a) The equations v1 - v2 + v3 = 0, v3 + v4 - v5 = 0 N2 N3
b5
are KVL equations for the network for some
loops. Using KVL, in loop containing branches b1, b3 and b6.
(b) The equations v1 - v3 - v6 = 0, v4 + v5 - v6 = 0 V1 - V6 - V3 = 0.
are KVL equations for the network for some
In loop containing branches, b4, b5 and b6.
loops.
(c) E = AV. V4 + V6 - V6 = 0.
(d) AV = 0 are KVL equations for the network. Hence, the correct option is (b).

Chapter 02.indd 23 11/10/2015 5:27:21 PM


Chapter 3
Network Theorems

(a) 50 W (b) 100 W


One-mark Questions (c) 5 W (d) 10.1 W
Solution: (a)
1. A source vs (t) = V cos 100 pt has an internal imped- Consider the figure given below
ance of (4 + j3) W. If a purely resistive load connected
ib
to this source has to extract the maximum power out
of the source, its value in W should be [2013] 9 kΩ 1 kΩ 99 ib
(a) 3 (b) 4 1
(c) 5 (d) 7 I +
Solution: (c) 100 Ω Z V
Consider the figure given below: −
2
( 4 + j 2)Ω Impedance is given by
V
Z= .
I
Vs ∼ RL Voltage across (1 kW + 9 kW) = V = -10,000ib. (1)
Applying KCL at node 1 we get
V
= 100ib + I . (2)
The maximum power will transfer to the load when 100
  RL = |Zs|. From equations (1) and (2),
∴ RL = | 4 + j 3 |= 16 + 9 . V V
  =− + I , Z = 50 Ω.
100 100
⇒ RL = 5 W. Hence, the correct option is (a).
Hence, the correct option is (c). 3. In the circuit shown below, the current through the
2. The impedance looking into nodes 1 and 2 in the inductor is [2012]
given circuit is [2012] IL
ib 1Ω j1Ω
1∠0 A
1 kΩ 99 ib 1∠0 V 1∠0 V
− ∼ + + ∼ −
9 kΩ 1∠0 A
1
100 Ω −j1Ω 1Ω
2

Chapter 03.indd 24 11/10/2015 6:57:13 PM


Chapter 3  Network Theorems  |  1.25

2 −1 Using maximum power transfer theorem


(a) A (b) A
1+ j 1+ j RL = Rth.
1 6R
(c) A (d) 0 A 3= .
1+ j 6+ R
Solution: (c) R = 6 Ω.
Consider the figure given below
Hence, the correct option is (c).
IL 5. In the figure the current source is 1∠0 A, P = 1 W,
the impedances are ZC = -jW, and ZL = 2jW. The
1Ω j1Ω
Thevenin equivalent looking into the circuit across
1∠0 A
1∠0 V 1∠0 V
X - Y is [2006]
− ∼ + + ∼ − X
R Z3
1∠0 A
1Ω 1∠0° ZL
−j1Ω
ZC
Y

Applying current division rule in upper part of the


(a) 2∠0 V, (1 + 2 j ) Ω
circuit, we get
(b) 2∠45° V, (1 - 2 j ) W
1 1 (c) 2∠45° V, (1 + 2j) W
IL = × 1∠0 = A.
1+ j 1+ j
(d) 2∠45° V, (1 + j ) Ω
Hence, the correct option is (c). Solution: (d)
4. In the circuit given below, the value of R required X
for the transfer of maximum power to the load hav- 1Ω
ing a resistance of 3 W is [2011] 1∠0° A j2 Ω Vth, Z th
R −j1 Ω
Y
6Ω
+ Thevenin’s equivalent can be calculated from the
10 V 3Ω Load
− above circuit as
Vth = 1∠0° × (1 + j 2 − j1) = 2∠45°.
By opening current source, we get
(a) zero (b) 3 W
(c) 6 W (d) infinity Zth = (1 + j1) W.
Solution: (c) Hence, the correct option is (d).
Voltage source is short-circuited in order to calcu- 6. In the figure Z1 = 10∠-60°, Z2 = 10∠60°, Z3 = 50
late Rth, ∠53.13°.
R Thevenin’s impedance seen from X - Y is [2003]

X
+ Z1 Z3
6
R th 100∠0° Z2

Y

Chapter 03.indd 25 11/10/2015 6:57:15 PM


1.26 | Electric Circuits and Fields

(a) 56.6∠45° (b) 60∠30° From the above circuit we conclude that
(c) 70∠30° (d) 34.4∠65°
 j4 
Solution: (a) VL1 =  100∠53.13° = 80∠90° V.
Applying Thevenin’s theorem, we get  3 + j4 
X Vth = 10 VL1 = 800∠90° V.
Z1 Z3
Hence, the correct option is (c).
100∠0° ∼ Z2 2. Assuming both the voltage sources are in phase,
the value of R for which maximum power is trans-
Y ferred from circuit A to circuit B is [2012]
Z1 × Z 2 2Ω R
Z th = Z X −Y = Z1|| Z 2 + Z 3 = + Z3
( Z1 + Z 2 ) + −j1 Ω +
10∠− 60 × 10∠60 ∼ 10 V ∼ 3V
= + (50∠53.13) − −
(10∠− 60 × 10∠60)
Circuit A Circuit B
= 56.66∠45°.
Hence, the correct option is (a). (a) 0.8 W (b) 1.4 W
7. Superposition principle is not applicable to a net­ (c) 2 W (d) 2.8 W
work containing time–varying resistors. ­(True/False) Solution: (a)
 [1994] Consider the figure given below
Solution:
 The given statement is false because superposition 2Ω R
principle is applicable for both time variant and
time invariant resistors. + +
∼ 10 V −j1 Ω ∼ 3V
− −
Two-marks Questions
Circuit A Circuit B
1. In the circuit shown below, if the source voltage
VS = 100∠53.12° V. Then the Thevenin’s equivalent
voltage in Volts as seen by the load resistance RL is Thevenin equivalent circuit of the above figure is
 [2013] shown below

3Ω j4 Ω j6 Ω 5Ω 2Ω I R
+ − +
V L1
∼ Vs + + 10V RL = 10 Ω
− − L1 + 10 V
− V 3V +

j 40I 2
I1 I2

(a) 100∠90° (b) 800∠0° Circuit A Circuit B
(c) 800∠90° (d) 100∠60°
Solution: (c) The current
Consider the figure given below
7
I= .
3Ω j4 Ω j6 Ω 3Ω R+2
+ − l2 = 0
VL1 And voltage
Vs ∼ + 10 VL1
− Vth
100∠53.13° V 14 10 R + 6
V = 10 − 2 I = 10 − = .
R+2 R+2

Chapter 03.indd 26 11/10/2015 6:57:17 PM


Chapter 3  Network Theorems  |  1.27

Power transferred from circuit A to circuit B will be 4. For the circuit given above, the Thevenin’s resist-
ance across the terminals A and B is  [2009]
10 R + 6 7
P = VI = × . (a) 0.5 kW (b) 0.2 kW
R+2 R+2 (c) 1 kW (d) 0.11 kW
For maximum power Solution: (b)
dP  When independent voltage source is short circuited
= 0.
dR we can calculate the Thevenin’s resistance as shown
(R + 2)2 (10) - (10R + 6) 2(R + 2) = 0. below in figure
2 kΩ 3VAB
5R2 + 20R + 20 = 10R2 + 26R + 12. + −
I
A
5R2 + 6R = 8. +

⇒ R = 0.8 W. 2 kΩ 1 kΩ VAB


Hence, the correct option is (a). −
B
3. If VA - VB = 6 V, then VC - VD is [2012]
R VA 2Ω VB R 3VAB
I
+ − A
+
R R − VAB × 10−3
R R 1Ω R 10 V
+ ≡ VAB
1 kΩ 1 kΩ
R
+ − VC VD (I −VAB × 10−3) −
5V 2A B
Figure 2
(a) -5 V (b) 2 V
(c) 3 V (d) 6 V Using the loop equation we get
Solution: (a) VAB + 3VAB = ( I − VAB × 10−3 )103
Consider the figure given below
= 103 I − VAB .
R VA 2 Ω IAB VB R
5 VAB = 103I.
R R −
R R 1Ω R 10 V VAB 103
+ ∴ Rth = = Ω = 0.2 kΩ.
R I 5
+ − VC IDC VD Hence, the correct option is (b).
5V 2A
5. For the circuit given above, the Thevenin’s voltage
From the given circuit, across the terminals A and B is [2009]
VA - VB = 6 V. (a) 1.25 V (b) 0.25 V
(c) 1 V (d) 05 V
(VA − VB )
I AB = = 3 A = I DC . Solution: (d)
2
 Current source is open-circuited in order to calcu-
Applying KCL at D gives late Thevenin’s impedance as shown below in figure
Hence, the correct option is (a) x

Common Data for Questions 4 and 5: R=1Ω


2 kΩ 3VAB
+− A Zth
ZL = 2j Ω

5V +
− 2 kΩ 1 kΩ
ZC = −j Ω
B y

Chapter 03.indd 27 11/10/2015 6:57:19 PM


1.28 | Electric Circuits and Fields

Thevenin’s impedance will be 7. In the figure, the potential difference between


points P and Q is [2003]
zth = R + z L + zC = 1 + 2 j − j = 1 + jΩ.
Now the open-circuit voltage between terminals 2A
X - Y can be calculated using
2Ω 4Ω
= I × zth P Q

= 1∠0 × (1 + j ) 10 V ∼

= 2∠45° V. 8Ω 6Ω
Hence, the correct option is (d)
6. In the given figure, the Thevenin’s equivalent pair (a) 12 V (b) 10 V
(voltage, impedence), as seen at the terminals (c) -6 V (d) 8 V
P - Q, is given by [2005] Solution: (c)
10 Ω P
Voltage VR = 10 V.
Applying KCL we get,
Unknown
20 Ω 4V 10 Ω network VP − 10 V
+ 2 + P = 0 (1)
2 8
Q VQ − 10 VQ
(a) (2 V, 5 W) (b) (2 V, 7.5 W)
−2+ = 0 (2)
4 6
(c) (4 V, 5 W) (d) (4 V, 7.5 W)
Solution: (a) 2A
Voltage source is short-circuit in order to calculate
the Rth as shown below R
P Q
10 Ω 2Ω + 4Ω

10 V
20 Ω 10 Ω Rth −
8Ω 6Ω

Rth = 10||10 = 5 W. Using equation (1) we get


Vth = Open-circuit voltage at the terminals P - Q. 4(VP - 10) + 2 × 8 + VP = 0.
10 Ω    4VP - 40 + 16 + VP = 0.
P
+   5VP - 24 = 0.
20 Ω 10 Ω Vth    VP = 4.8.
4V
− 6(VQ − 10) − 2 × 4 × 6 + 4VQ = 0.
Q
  10VQ − 108 = 0.
4
Vth = × 10 = 2 V.
10 + 10 ∴ VQ = 10.8.

Thevenin’s equivalent circuit ∴ VP − VQ = −6 V.


  
Rth = 5 Ω
P Hence, the correct option is (c).
Vth = 2 V 8. Two AC sources feed a common variable resis-
tive load as shown in figure. Under the maximum
Q power transfer condition, the power absorbed by
Hence, the correct option is (a). the load resistance RL is [2003]

Chapter 03.indd 28 11/10/2015 6:57:21 PM


Chapter 3  Network Theorems  |  1.29

series with a single resistor with the following


6Ω j8 Ω 6Ω j8 Ω
parameters. [1998]
110∠0° ∼ ∼ 90∠0° A
RL +
10 V 4Ω

+
6Ω 5V

B
(a) 2200 W (b) 1250 W
(c) 1000 W (d) 625 W (a) 5 V source in series with 10 W resistor.
(b) 7 V source in series with 2.4 W resistor.
Solution: (d)
(c) 15 V source in series with 2.4 W resistor.
Vth across the terminals of RL can be calculated us-
(d) 1 V source in series with 10 W resistor.
ing figure given below
Solution: (b)
6Ω j6 Ω 6Ω j6 Ω Consider the figure given below
+ + A + A
∼ 110∠0° Vth 90∠0° ∼
− − B − 5 5
6Ω A 4Ω A
3 4

Using superposition theorem B

Figure 1
110∠0° 90∠0°
Vth = + = 100∠0°.
2 2 2.4 Ω
A A
By short circuiting voltage sources Rth can be cal- +
culated. 35
2.4 Ω A 7V
12

Zth = (6 + j8) || (6 + j8) = (3+ j4) W.
B B
Thevenin’s equivalent about A, B is shown in the Figure 2      Figure 3
Figure below.
Thevenin’s equivalent for the given circuit is shown
(3 + j4)Ω above using source transformations.
Hence, the correct option is (b).
+ Z th
I 10. For the circuit shown in the figure, the Norton
100∠0° ∼ RL
− equivalent source current value is A and
its resistance is W [1997]
For maximum power transfer 2A

2 2
A
RL = 3 + 4 = 5 Ω. +
6V 3Ω
 100 100 104 3Ω

I= , I= , I2 = .
8 + j4 80 80 B
Power absorbed by load will be Solution:
4 Consider the figure given below
10
( RL ) = I 2 RL = × 5 = 625 W. 2A
80 3Ω
A
Hence, the correct option is (d).
9. Viewed from the terminals A, B, the following 6V +
− 3Ω

circuit shown in the figure can be reduced to an


equivalent circuit of a single voltage source in B

Chapter 03.indd 29 11/10/2015 6:57:22 PM


1.30 | Electric Circuits and Fields

Using source transformation Using source transformation


1.5 V 6V 3Ω
6V 3Ω A
A

≡ 2 × 1.5 = 3 V Isc
≡ 6 =2A 3Ω 3Ω
3
B
B
3+ 6
3  3 = 1.5 Ω I SC = = 2 A.
1.5 + 3
RN = 1.5 + 3 = 4.5 Ω.

Chapter 03.indd 30 11/10/2015 6:57:23 PM


Chapter 3  Network Theorems  |  1.31

1Ω X
Five-marks Questions IL
+
1. An electrical network is fed by two AC sources, 0V ∼ ZL

as shown in the figure. Given that Z1 = (1 - j) W,
Z2 = (1 + j) W and ZL = (1 + j0) W. Obtain the
Y
Thevenin equivalent circuit (Thevenin voltage and
impedance) across terminals X and Y, and deter- Figure 2
mine the current IL, through the load IL [2002]
2. Predict the current I in the figure in response to
a voltage of 20∠0° V. The impedance values are
X
given in ohms. Use Thevenin’s theorem. [2000]
Z1 Z2
+ +
ZL IL
∼ ∼ b
− − −j 20 Ω
V1 = 30∠45° V2 = 30∠−45° j40 Ω
I 10 Ω
∼ 20∠0° V a c
Solution:
40 Ω −j 20 Ω

Thevenin voltagex (Vth) can be found using Figure 1.
shown below d

Z1 Z2 Solution:
I X I 
Thevenin’s voltage V th can be found using the cir-
(1− j1) Ω
(1+ j1) Ω cuit shown in Figure 1 shown below
+ +
+
Vth
− V1 = 30∠45° − V2 = 30∠−45° − b
I1 I2
j40 Ω −j20 Ω
Y
∼ 20∠0° V a +V −
ac c
Figure 1
40 Ω −j20 Ω
Using KVL to the closed loop we get
 d
30∠45° − 2 I − 30∠ − 45° = 0
Figure 1

or  2 I = 30e j 45° − 30e − j 45° = j60 sin 45°
20∠0° 2
 30 j 90° I1 = = ∠− 45° = 0.35∠− 45°A.
or     I = e . 40 + j 40 32
2
 20∠0°
30 j 90° I2 = = 0.5∠90°A.
Vth = 30∠45° − e (1 − j1) . − j 40
2
Vth = 30∠45° − 30∠45° = 0. If we apply KVL to loop acd we get

Thevenin’s impedance Zth is found by shorting the Vth = Vac = 40(0.35∠− 45°) − (− j 20)(0.5∠90°)
voltage sources
Zth = (1 - j1)||(1 + j1) = 1 W.  14  14
= 14∠− 45° − 10 =  − 10  − j
 2  2
Thevenin’s equivalent circuit is shown in Figure 2

given below with ZL connected and IL = 0. = −0.1 − j 9.9 = 9.9∠− 90.57° ≅ 10∠− 90° V.

Chapter 03.indd 31 11/10/2015 6:57:25 PM


1.32 | Electric Circuits and Fields

Thevenin’s equivalent impedance Zth can be found Vs 4Ω


+ − 4Vs A
using Figure 2 given below 2Ω +
4Vs 2Ω +
b + 10 V
− Vth Vth
j40 −j 20 4Vs −

a c B
Figure 1
40 −j20

d
Vs = -4Vs × 2,  Vs = 0,  Vth = 10 V.

Zth Thevenin’s resistance can be calculated from the


a c
Figure 2, in which the independent 10 V – voltage
Figure 2 source is short circuited.
Zth = (40 || j40) + (-j20) || (-j20) Vs (I + 4Vs) 4Ω
+ − I
A
 = 20 + j20 - j10 2Ω +
2Ω
 = (20 + j10) W. VAB
Thevenin equivalent circuit in Figure 3 as shown 4Vs
below −
B
Zth Figure 2
a
I   VB = 4I - Vs(1)
20
 Vs = -2(I + 4Vs)
10∠− 90° ∼ 10 Ω
or  Vs = -2I - 8Vs
2
or  Vs = − I .
c 9
From equation(1)
Figure 3
2
VAB = 4 I + I,
 10∠− 90° 9
∴ I= = 0.32∠− 108.43° A.
20 + j10 + 10 VAB 38
Rth = = Ω.
I 9
3. Find the Thevenin equivalent about AB for the cir-
cuit shown in the figure. [1997] Thevenin’s equivalent circuit of given circuit in
problem about A, B is shown in Figure 3 given be-
Vs 4Ω low
+ − A
2Ω 38
2Ω Ω
+ 10 V + 9 − A
− 4Ω

4V s +
10 V 4Ω
B −

Solution:

Consider Figure 1 given below in which Vth is found B
as VOC across the terminals A, B. Figure 3

Chapter 03.indd 32 11/10/2015 6:57:26 PM


Chapter 4

Transient Response
V +c (0) −2
One-mark Questions S
=
S
1
Cs
+ −
1. A combination of 1 μF capacitor with an initial
voltage Vc(0) = -2 V in series with a 100 Ω resistor
is connected to a 20 mA ideal DC current source by +
operating both switches at t = 0s as shown. Which of 1
V s(S) R
s 1
the following graphs shown in the options approxi- − s
mates the voltage Vs across the current source over
the next few seconds? [2014-S1]
Vc
+ − Applying KVL, to the closed loop we get

 −2  I  I 
t=0 + VS ( S ) =   +  R + 
 S  S Cs 
Vs
1 I  1 1 
− =  −2 + IR +  = ( IR − 2) +  .
s Cs  s  Cs 
t=0
Substituting the values, we get

(a) Vs (b)
Vs 1  20 × 10−3  1 
VS ( S ) = (20 × 10−3 × 200 − 2) +  −6 × 
s  10  s
3
1 1  20 × 10
t
= (2 − 2) + 20 × 103 ×  = .
t s s s2
−2 −2
(c) Vs (d)
Vs
20 × 103
   ∴ VS ( S ) = .
s2
Or, VS (t ) = 20000 tu (t ).
t t
−2 −2 ∴ VS ( S ) = (20000) tu (t )….
Solution: (c) The above equation is the equation of a straight
C = 1 μF, vC (0) = -2 V, R = 100 Ω, I = 20 mA. line passing through origin.
For time t > 0 consider the circuit shown below Hence, the correct option is (c).

Chapter 04.indd 33 11/10/2015 5:36:47 PM


1.34 | Electric Circuits and Fields

2. The switch SW shown in the circuit is kept at posi- Now when switch S is closed at t = 0 as shown
tion ‘1’ for long duration. At t = 0+, the switch is below
moved to position ‘2’. assuming | V02 | > | V01 |, the S i(t)
voltage vc(t) across the capacitor is [2014-S2]
+
R ‘2’ C1 VC C2
SW

R
‘1’
V 02
V 01 C VC 2 = VC 1 = VC.
Vc
At t = 0+,
Energy in the circuit of capacitors will be
vc (t ) = −V02 (1 − e − t/ 2 RC ) − V01 .
(a)
1
vc (t ) = V02 (1 − e − t/2 RC ) + V01 .
(b) = (C1 + C2 )VC2 .
2
vc (t ) = (V02 + V01 )(1 − e − t/2 RC ) − V01 .
(c) According to law of conservation of energy,
− t/2 RC
vc (t ) = (V02 − V01 )(1 − e
(d) ) + V01 .
1 1
Solution: (d) C1V12 = (C1 + C2 )VC2 .
2 2
The voltage vc(t) across the capacitor is
C1
− t/τ
VC = V1 ,  for t ≥ 0+ .
vc (t ) = v f + (vi − v f )e C1 + C2
= v02 + (v01 − v02 )e − t/2 RC .
At t = 0, VC 2 changes from 0 V to VC.
Hence, the correct option is (d). Impulse function i(t) can be calculated as
3. In the following figure, C1 and C2 are ideal capaci- d
tors. C1 has been charged to 12 V before the ideal ∴ i (t ) = C2 VC
dt
switch S is closed at t = 0. The current i(t) for all t is
[2012] C1
= C2 V1δ (t ),
S t=0
C1 + C2
V1 = 12 V.
C1 i(t ) C2 We conclude that voltage across a capacitor can-
not change instantaneously unless impulse current
flows through it. In this question voltage at t = 0
(a) zero changes because impulse current flows at t = 0.
(b) a step function
Hence, the correct option is (d).
(c) an exponentially decaying function
(d) an impulse function 4. The switch in the circuit has been closed for a long
time. It is opened at t = 0. At t = 0, the current
Solution: (d)
through the 1 μF capacitor is [2010]
We know that
Since, switch S is open 1Ω
i(t) = 0,  t < 0. t=0
Also VC 2 = 0. 5V 1 μF 4Ω

Energy in the circuit at t = 0- will be


1 (a) 0 A (b) 1 A
= C1V12 ,    V1 = 12 V.
2 (c) 1.25 A (d) 5 A

Chapter 04.indd 34 11/10/2015 5:36:51 PM


Chapter 4  Transient Response  |  1.35

Solution: (b) From the above circuit we conclude that


The circuit is in steady state because the switch has
been closed for a long duration of time. We know vC (∞) = 10 V.
that during steady state, the capacitor behaves as Hence, the correct option is (b).
open circuit
6. A unit step voltage is applied at t = 0 to a series RL
1Ω
circuit with zero initial conditions. [2001]
I (a) It is possible for the current to be oscillatory.
5V Vc(0−) 4Ω (b) The voltage across the resistor at t = 0+ is zero.
(c) The energy stored in the inductor in the steady
Circuit at t = 0 state is zero.
(d) The resistor current eventually falls to zero.
Applying KVL to the closed loop we get
Solution: (b)
S − I − 4 I = 0 ⇒ I = 1 A.
At t = 0+ , inductor behaves as open circuit. There-
VC (0− ) = 4 × 1 = 4 V. fore complete source voltage drops across it and
as a result current through the resistor R becomes

Since, voltage across capacitor cannot change
zero. Therefore, voltage drop across the resistor at
abruptly, therefore,
t = 0+ is zero, and further with time it rises accord-
VC (0+) = VC(0-) = 4 V. ing to VR (t ) = R ⋅ (1 − e − Rt/L )u (t ).
Circuit at t = 0+:
V(t )
+ I
5V 4V C 4Ω R

t
0
Current through capacitor at t = 0+,
4 Hence, the correct option is (b).
I C (0 + ) = = 1 A.
4 7. In the series RC circuit shown in figure, the voltage
Hence, the correct option is (b). across C starts increasing when the DC source is
switched on. The rate of increase of voltage across
5. In the figure given below, the initial capacitor volt-
C at the instant just after the switch is closed (i.e.,
age is zero. The switch is closed at t = 0. The final
at t = 0+), will be [1996]
steady-state voltage across the capacitor is [2005]
t=0
10 Ω
C R
t=0 1V
20 V 10 μF 10 Ω

(a) 20 V (b) 10 V (a) zero (b) infinity


(c) 5 V (d) 0 V 1
(c) RC (d)
Solution: (b) RC
Capacitor behaves as an open circuit in the steady Solution: (d)
state at t = ∞, and the resultant circuit is shown in Consider the figure given below for t > 0.
figure given below.
C R
10 Ω
+ −
VC
i(t )
20 V V c (∞) C 10 Ω 1V
+ −
Sw

Chapter 04.indd 35 11/10/2015 5:36:53 PM


1.36 | Electric Circuits and Fields

From loop equation, we get Solution: (b)


dvC (t ) Inductor will behave as short circuited and capaci-
i (t ) = C . (1) tor will behave as open circuited at steady state,
dt
thus complete 100 V will drop across it.
dv (t )
vC (t ) + RC C = 1, for t > 0. (2) Hence, the correct option is (b).
dt
At t = 0+,
dvC (t )
vC (0+ ) + RC = 1.
dt t = 0+ Two-marks Questions
As vC (0+) = 0,
1. In the Figure shown, all elements used are ideal. For
dvC (t ) 1 time t < 0, S1 remained closed and S2 open. At t = 0,
= .
dt t = 0+ RC S1 is opened and S2 is closed. If the voltage VC 2 across
the capacitor C2 at t = 0 is zero, the voltage across the
Hence, the correct option is (d). capacitor combination at t = 0+ will be [2009]
8. The time constant of the network shown in the
­figure is [1992] S1 S2
R
3V 1F C1 C2 2F
10 V 2R C

(a) 1 V (b) 2 V
(a) 2RC (b) 3RC (c) 1.5 V (d) 3 V
RC 2 RC
(c) (d) Solution: (a)
2 3 For t = 0-, consider Figure 1 given below.
Solution: (d)
S1 A S2
By closing the switch and short circuiting the 10 V
DC source, time constant of the given network can C1
+
be claculated. Consider the resultant circuit shown 3V 3 V 1 F C2 2F

below
R B
Figure 1
2R C
2R C
3 For t = 0 , consider Figure 2 given below.
+

S1 A S2
2R
Time constant = C. C1 +
3 +
3V V C2
Hence, the correct option is (d). 3V −

9. In the circuit of the figure, the switch S is closed at
t = 0 with iL = (0) = 0 and vC = (0) = 0. In the steady B
state, vC equals [1992] Figure 2

S iL L
+ 3 × C1 3 ×1
V= = = 1 V.
100 V
C
+
VC C1 + C2 3
DC −
− Hence, the correct option is (a).
(a) 200 V (b) 100 V 2. The time constant for the given circuit will be
(c) zero (d) -100 V [2008]

Chapter 04.indd 36 11/12/2015 1:17:20 PM


Chapter 4  Transient Response  |  1.37

1F 3Ω We know that the current and voltage across induc-


tor and cpacitor cannot change instantaneously.
1F 1F 3Ω 3A Equivalent circuit at t = 0+ is shown in Figure 2
given below.
I 10 Ω
1 1 + (10 − I ) C
(a) (b) +
9s 4s VL 10 Ω L 10 A I 10 V
(c) 4s (d) 9s −

Solution: (c)
Figure 2
We neglect current source as a open circuit in order
to calculate time constant. Consider the simplified Using loop equation we get
circuit given below
10(10 - I ) = 10I + 10.
1F 3Ω 1F 3Ω ∴  I = 4.5 A.


Hence, the correct option is (d).
1F 1F 3Ω 2F
4. An ideal capacitor is charged to a voltage V0 and
connected at t = 0 across an ideal inductor L. (The
2 circuit now consists of a capacitor and inductor
Time constant = Req Ceq = 6 × = 4sec. 1
3 alone.) If we let ω0 = , the voltage across the
LC
Hence, the correct option is (c). capacitor at time t > 0 is given by [2006]
3. In the circuit shown in figure switch Sw1 is initially (a) V0 (b) V0 cos(ω0 t )
CLOSED and Sw2 is OPEN. The inductor L carries (c) V0 sin(ω0 t ) (d) V0 e −ω0t cos(ω0 t )
a current of 10 A and the capacitor is charged to
Solution: (b)
10 V with polarities as indicated. Sw2 is initially
Untill the voltage across the capacitor becomes ,
CLOSED at t = 0- and Sw1 is OPENED at t = 0.
voltage across capacitor will discharge through in-
The current through C and the voltage across L at
ductor. Electrostatic energy stored in capacitor is
t = 0+ is [2007]
converted into magnetic energy during this period
Sw 2 10 Ω of time and is stored in inductor. Now magnetic
energy in inductor is converted into electrostatic
R1 R2
+ energy in capacitor and inductor will start charging
10 Ω 10 A C 10 V the capacitor,

Sw 1 L
We can obtain the expression for VC(t) in s-domain.
(a) 55 A, 4.5 V (b) 5.5 A, 45 V When the capacitor is charged initially to voltage
(c) 45 A, 5.5 V (d) 4.5 A, 55 V V0, then representation of capacitor in s-domain is
shown below in figure
Solution: (d)
We know that at t = 0-, iL = 10 A, VC = 10 V.
When Sw1 is opened at t = 0 and Sw2 is closed at VC(0+) + V0
+ −
t = 0, For t > 0 consider Figure 1 shown below − s s

Sw 2 10 Ω
C ZC = 1
R2 sC
R1 +
10 Ω 10 A C 10 V

Sw 1 L
As current though the inductor is zero at t = 0, then
Figure 1 we get

Chapter 04.indd 37 11/10/2015 5:36:58 PM


1.38 | Electric Circuits and Fields

the inductance is ideal, the current through the


inductor at t = 0+ equals [2005]
L ≡ sL 10 Ω

10 V 10 mH t=0
For t > 0 the figure is shown below.

V0 (a) 0 A (b) 0.5 A


±
s
(c) 1 A (d) 2 A
V(s) sL
1 Solution: (c)
sC I(s)
For t < 0, consider the Figure 1 given below
10 Ω
V0 iL(t)
s V0 10 V 10 mH
I (s) = = .
1 1
+ sL s 2 L +
sC C
Figure 1
 
V0  1  We know that inductor behaves as short circuit in
I (s) =  .
L  s2 + 1  the steady state, at t = 0-, therefore,
 
 LC  10
iL (0− ) = = 1 A.
Voltage across capacitor = Voltage across inductor 10
= V(s). At t = 0+, consider the Figure 2 given below
10 Ω
V ( s ) = I ( s ) × ( sL)
    L
10 V 1A
V0  1   s  Sw
= 
1  × ( sL) = V0  1 .
L  s2 +   s2 + 
 LC   LC  Figure 2

1 iL (0+ ) = iL (0− ) = 1 A.
ω0 =
As 
LC v L ( 0 + ) = 0.

 s  Hence, the correct option is (c).


V ( s ) = V0  2 2 
.
 s + ω0  Common Data for Questions 6 and 7:
A coil of inductance 10 H and resistance 40 Ω is
Voltage across the capacitor = V (t ) = L−1[V ( s )] connected as shown in the figure. After the switch
 Vs  S has been in contact with point 1 for a very long
= L−1  2 0 2  . time, it is moved to point 2 at t = 0.
 s + ω0  6. If, at t = 0+, the voltage across the coil is 120 V, the
V (t ) = V0 cos ω0 t , value of resistance R is [2005]
1 20 Ω
ω0 =
where  . 1
LC
S 2 10 H
120 V
Hence, the correct option is (b).
R
5. The circuit shown in the figure is in steady state, 40 Ω
when the switch is closed at t = 0. Assuming that

Chapter 04.indd 38 11/10/2015 5:37:00 PM


Chapter 4  Transient Response  |  1.39

(a) 0 Ω (b) 20 Ω Energy decay in the inductor can be expressed as


(c) 10 Ω (d) 60 Ω
Solution: (a) 1 1
EL (t ) = L[i (t )]2 = × 10(2e −10t ) 2 .
Before moving the switch, at t = 0-, is shown in the 2 2
−20 t
figure given below. EL (t ) = 20e .
20 Ω
Energy stored initially at t = 0, EL(t - 0) = 20 J.
95% of the energy is to be dissipated which means
120 V only 5% of the energy is to be stored.
iL(0−)
40 Ω
Therefore,

The circuit is in steady state and we know that in 20e −20t = 0.05 × 20.
steady state the inductor behaves as short-circuit. e −20t = 0.05.
Therefore, −20t = −2.995.
120
iL (0− ) = = 2 A. t = 0.1497 sec ⇒ t ≈ 0.15.
20 + 40
Current through inductor cannot change abruptly Hence, the correct option is (b).
after moving the switch, at t = 0+ as shown in figure 8. In figure, the capacitor initially has a charge of 10
given below Coulomb. The current in the circuit one second
20 Ω after the switch S is closed will be [2004]
+
2Ω
2A
VC S
R +
40 Ω 100 V +
− 0.5 F

iL (0+ ) = iL (0− ) = 2 A. (a) 14.7 A (b) 18.5 A


Now we have, (c) 40.0 A (d) 50.0 A
Solution: (a)
VL = iL (0+ ) × (20 + R). Applng Kirchoff’s voltage law to the closed loop,
120 = 2 × (20 + R). we get,
R = 40 Ω. dq q
100 = R + .
Hence, the correct option is (a). dt C
dq
7. For the value of R obtained in the above question, 100C = RC + q.
the time taken for 95% of the stored energy to be dt
dissipated is close to [2005] q dq 1 t
(a) 0.10 sec (b) 0.15 sec ∫q0 100C − q = RC ∫0 dt.
(c) 0.50 sec (d) 1.0 sec
100C − q = (100C − q0 )e − t/RC .
Solution: (b)
dq (100C − q0 ) −1/RC
Expression for current through inductor, is given by i= = e .
dt RC
i (t ) = I 0 e − t/τ . e − t/RC = 40e −1 = 14.7 A.
i (t ) = 2e − t/τ . Hence, the correct option is (a).
L 10 9. In the circuit shown in the figure, the switch S is
τ= = = 0.1.
R 20 + 40 + 40 closed at time t = 0. The voltage across the induct-
i (t ) = 2e −10t . ance at t = 0+, is [2003]

Chapter 04.indd 39 11/10/2015 5:37:04 PM


1.40 | Electric Circuits and Fields

3Ω 10
F ⋅V = 11×
where,  = 10 V.
+ S
4F 11
10 V 4Ω 4H
− 4Ω 10
τ=
And,  × 103 × 11× 10−9  = 10 μs.
11
(a) 2 V (b) 4 V
(c) -6 V (d) 8 V vC (t ) = 10(1 − e − t/τ ). (1)
Solution: (b)
When we apply a pulse of 10 μs, vC(t) as given in
For t > 0 consider the figure given below
(1) is true under condition 0 < t < 10 μs.

Therefore, peak value of vC(t) occurs at t = 10 μs.
S
+ 4F Peak value = 10(1 - e-1) = 6.32 V.
10 V 4Ω 4H
− 4Ω Hence, the correct option is (c).
11. Consider the circuit shown in the figure. If the fre-
For t = 0+, consider the figure given below. quency of the source is 50 Hz, then a value of t0
3Ω which results in a transient free response is [2002]

+ C 5Ω 0.01 H
10 V 4Ω L V L(0+)
− 4Ω t = t0
∼ sin(wt )
10 × 2
vL ( 0 + ) = = 4 V.
5
(a) 0 ms (b) 1.78 ms
Hence, the correct option is (b). (c) 2.71 ms (d) 2.91 ms
1 0. An 11 V pulse of 10 μs duration is applied to the cir- Solution: (b)
cuit shown in the figure. Assuming that the capacitor For transient free response,
is completely discharged prior to applying the pulse,
the peak value of the capacitor voltage is [2002] ωL
tan(ωt0 ) = .
R
1 kΩ
2π × 50 × 0.01
11 V tan(2π × 50 × t0 ) = .
5
10 kΩ 11 nF
0 π 
10 µs 2π × 50 × t0 = tan −1   .
5
(a) 11 V (b) 5.5 V = 32.14° = 0.561 rad.
(c) 6.32 V (d) 0.96 V 0.561
t0 = = 1.786 ms.
Solution: (c) 100π
Consider the figure given below
Hence, the correct option is (b).
1 kΩ
11 V 1 2. A rectangular voltage pulse of magnitude V and dura-
+ tion T is applied to a series combination of resistance
10 kΩ 11 nF VC (t )
0
10 µs −
R and capacitance C. The maximum voltage devel-
oped across the capacitor is [1999]
vC(0) = 0 V.   −T   VT
V 1− exp 
(a)   (b)
  RC   RC
If the input is 11 u(t),
 −T 
(c) V exp 
V (d) 
vC (t ) = I ⋅V + ( F ⋅V − I ⋅V )(1 − e − t/τ
),  RC 

Chapter 04.indd 40 11/10/2015 5:37:07 PM


Chapter 4  Transient Response  |  1.41

Solution: (a) i.e., vs = V ⋅ [u(t) - u(t - T )]


Consider the figure given below.
∵ vC (t ) = vs (t ) ⋅ (1 − e − t/RC )
R
+ = V (1 − e − t/RC ) ⋅ [u (t ) − u (t − T ) ].

+ V c(t)
Vs(t) C Vc(t)

V(1 − e −t/RC )

Given: t
T
V s(t )
From the above figure we conclude that the capaci-
tor charges till t = T and then discharges.
V Hence  VC (max) = V (1 − e −T /RC ).
t
Hence, the correct option is (a).
0 T

Chapter 04.indd 41 11/10/2015 5:37:08 PM


1.42 | Electric Circuits and Fields

Five-marks Questions 5Ω

7Ω 18 Ω
1. A constant current source is supplying 10 A to a VC
circuit shown in the figure. The switch S, which + −
is initially closed for a sufficiently long time, is 0.04 µF
suddenly opened. Obtain the differential equation 2Ω
governing the behaviour of the inductor current Solution:
and hence obtain the complete time response of the
VC(0) = 12 V.
inductor current. What is the energy stored in L, a
long time after the switch is opened? [2002] For t > 0, the circuit is shown in Figure 1.

iR iL
I 10 Ω R
10 A 5H L S 18 Ω 9Ω
0.04 μF
A
+ − B
Solution: 12 V
For t > 0 consider the figure given below. Figure 1

diL The simplified circuit is shown in Figure 2.


0.5 + iL(t )
dt 11 Ω
diL
10 A 10 Ω 5 V c(t )
dt 5H + −

A + − B
0.04 μF 12 V
From nodal equation we get
Figure 2
diL (t )
0.5 + iL (t ) = 10. VC (0+ ) = 12 V, VC (∞) = 0.
dt
diL (t ) Time constant,
+ 2iL (t ) = 20.
dt RC = 11× 0.04 × 10−6 = 0.44 × 10−6 sec.
iL (0) = 0, iL (∞) = 10 A. −
25
t

Time constant, VC (t ) = 12e 11


, t > 0.
Time t is measured in μs.
L
τ= = 0.5 sec. 3. The switch in the following circuit, shown in the
R
figure has been connected to the 12 V source for a
For step response of 1st order circuit we have long time. At t = 0, the switch is thrown to 24 V.
[1998]
iL (t ) = IV + ( FV − IV )(1 − e − t /τ ).
t=0 L R1
iL (t ) = 10(1 − e −2t ). +
iL iR 2 iC +
24 V +
Energy stored in L as − C V C (t)
12 V R2
− −
1 1
t → ∞ = Li 2 (∞) = × 5 × 100 = 250.
2 2 L = 2 H,  R1 = 10 Ω,  R2 = 2 Ω,  C = 0.25 μF
2. In the given circuit, the capacitor is initially (a) Determine iL(0) and VC(0)
charged to 12 V. Find the mathematical expression (b) Write the differential equation governing VC(t)
for the voltage across the capacitor VC after closing for t > 0
the switch at t = 0. [1999] (c) Compute the steady state value of VC(t)

Chapter 04.indd 42 11/10/2015 5:37:11 PM


Chapter 4  Transient Response  |  1.43

Solution: 6 6 × 106
(a) For t < 0, consider the circuit shown in Figure 1 b= = = 12 × 106.
2C 2 × 0.25
given below. 24
K= = 48 × 106.
2H 10 Ω 2C
iL + 4. In the circuit shown in the figure, eg(t) = 2.5t V. What
+ 0.25 µF are the values of i(t) and vL(t) at t = 4 seconds?
2Ω VC(t)
12 V
− −
[1997]

i(t )
Figure 1 2Ω
eg (t ) ± +
At t = 0- the circuit behaviour is shown in Figure 2 V L(t) 4 H
given below. −

2H 10 Ω Solution:
iL0− +
Consider the figure given below in which trans-
+ form circuit is shown
2Ω VC(t)
12 V
− − I(s)

Figure 2
2
+
12 eg(s) = 2.5/s2 −
iL (0− ) = = 1 A.
12 4s
2
VC (0− ) = 12 × = 2 V.
12
iL (0+ ) = iL (0− ) = 1 A. For eg(t) = 2.5t, R = 2 Ω, L = 2 H,
VC (0+ ) = VC (0− ) = 2 V.
2.5
I (s) = 2
.
(b) For t > 0, the circuit is shown in Figure 3. ( s )(2 + 4 s )
0.625
C
dV c I (s) = 2 .
+
2H 10 Ω dt s ( s + 0.5)
24 V V c(t) −25 1.25 2 .5
− dV c V c I (s) = + 2 + .
C + 0.25 μF V C (t)
dt 2 2Ω 2 s s s + 0.5

For t > 0,
Figure 3
i (t ) = 1.25t − 2.5(1 − e −0.5t ).
Using loop equation we get
di (t )
d 2VC dVC dV vL (t ) = L = 4 [1.25 − 2.5 × 0.5e −0.5t ]
2C + + 10C C + 5VC + VC = 24, t > 0. dt
2
dt dt dt = 5(1 − e −0.5t ).
d 2VC dV
2C + (1 + 10C ) C + 6VC = 24.
dt 2
dt i (4) = 1.25 × 4 − 2.5(1 − e −2 )
d 2VC dVC = 5 − 2.16 = 2.84 A.
+a + bVC = K ,
dt 2 dt vL (4) = 5(1 − e −2 ) = 4.32 V..

1 + 10C 1 + 2.5 × 10−6


a=
where,  = = 2 × 106.
2C 0.5 × 10−6

Chapter 04.indd 43 11/10/2015 5:37:13 PM


Chapter 5
Sinusoidal Steady
State Analysis

1 1
One-mark Questions (a) A (b)
2 2
A

1. The average power delivered to an impedance (c) 1 A (d) 2 A


(4 - J 3) W by a current 5 cos(100πt + 100) A is Solution: (b)
[2012] Vs = 1sin t = Vm sin ωt.
(a) 44.2 W (b) 50 W
Vm = 1 V and ω = 1 rad/sec.
(c) 62.5 W (d) 125 W
Impedance of the branch containing inductor and
Solution: (b)
capacitor can be calculated as
i = 5 cos(100 π t + 100°) A = 5∠100° A.
 1 
Z = (4 − j 3) Ω = 5∠ − 36.87° Ω. Z = j( X L − X C ) = j  ω L − 
 ωC 
v = iZ = 25∠63.13° V.
 1 
Average power is = j  1× 1 −  = 0.
 1 × 1
1 1
P = Vm I m cos φ = × 25 × 5 × cos(36.87°) Since, the impedance of the branch is zero, the
2 2 branch is short-circuited and the whole current
1 4 flow through it
= × 25 × 5 × = 50 W.
2 5 1.0 sin t
Hence, the correct option is (b). = i (t ) = 1.0 sin t.
1
2. The rms value of the current i(t) in the circuit 1
shown below is [2011] rms value of the current = A.
2
1F 1H
Hence, the correct option is (b).
3. The voltage applied to a circuit is 100 2 cos (100πt)
1Ω
volts and the circuit draws a current of
 π
10 2 sin 100π t +  Amperes. Taking the volt-
i(t)  4
(1.0 sin t)V
1Ω age as the reference phasor, the phasor representa-
+ ∼ − tion of the current in amperes is [2011]

Chapter 05.indd 44 11/10/2015 7:11:13 PM


Chapter 5  Sinusoidal Steady State Analysis  |  1.45

π π Z can only be a capacitance given by


10 2∠ −
(a) (b) 10∠ −
4 4
π π 1
(c) 10∠ + (d) 10 2∠ + ZC = .
4 4 jωC
Solution: (a) Or YC = jwC.
Voltage v(t ) = 100 2 cos(100π t )V
 π j  1 
and i (t ) = 10 2 sin 100π t +  A. ∴ YPQ = jωC − = j  ωC − .
4 ωL  ωL 

If we take voltage as the reference phasor, For parallel resonance
Voltage phasor,
1
+
V = 100 2 e − j 0 V. YPQ = 0,  if  ω0 = .
LC
 π
For i (t ) = 10 2 sin 100 π t +  A 1
 4 ∴ C= .
Lω02
 π 3π 
= 10 2 cos 100π t + +  If frequency f0 = 500 Hz and inductance L = 2 H
 4 2 
then we have
 π
= 10 2 cos 100π t + 2π −  1
 4 C= F = 0.05 µF.
π 2 × 4π × 5002
2

= 10 2 cos  100π t −  .
 4 Hence, the correct option is (d).
Therefore, current phasor, I with V as the reference 5. In a series RLC circuit at resonance, the magni-
−j
π tude of the voltage developed across the capacitor
= 10 2e 4
 [2001]
π (a) Is always zero.
= 10 2∠ − . (b) Can never be greater than the input voltage.
4
(c) Can be greater than the input voltage, however,
Hence, the correct option is (a).
it is 90° out of phase with the input voltage.
4. The value of Z in the figure, which is most appropri- (d) Can be greater than the input voltage, and is in
ate to cause parallel resonance at 500 Hz is [2004] phase with the input voltage.
5Ω Solution: (c)
For series RLC circuit, at resonance
2H Z VL = jQVsorce   and  Vc = -jQVsource

Also for Q > 1, |Vc|>|Vsource|
Hence, the correct option is (c).
(a) 125.00 mH (b) 304.20 μF 6. A series RLC circuit when excited by a 10 V sinu-
(c) 2.0 μF (d) 0.05 μF soidal voltage source of variable frequency, exhib-
Solution: (d) its resonance at 100 Hz and has a 3 dB bandwidth
Consider the figure given below of 5 Hz. The voltage across the inductor L at reso-
P nance is [1999]
5Ω (a) 10 V (b) 10 2 V
10
(c) (d) 200 V
2H Z 2V

Solution: (d)
For series RLC circuit
Q
f0 = 100 Hz,  f3dB = 5 Hz.

Chapter 05.indd 45 11/10/2015 7:11:18 PM


1.46 | Electric Circuits and Fields

f3dB for series resonance is given by For Vin = 100 V, R = 20 W,


f0 Current through the circuit,
f 3dB =
.
Q
100
f 100 =I = 5 A.
∴ Q= 0 = = 20. 20
f 3dB 5
Hence, the correct option is (a).
Under resonance, the voltage across the inductor
and capacitance are same in magnitude but oppo- 9. A sinusoidal source of voltage V and frequency f is
site in phase. connected to a series circuit of variable resistance,
R and a fixed reactance, X. The locus of the tip of
VL = j QVin , VC = − j QVin . the current phasor, I, as R is varied from 0 to ∞ is
∴ VL = QVin .  [1998]
V
For Vin = 10 V (a) a semicircle with a diameter of .
X
VL = 20 × 10 = 200 V. R
(b) a straight line with a slope of .
Hence, the correct option is (d). X
7. The rms value of a half - wave rectified symmetri- V
(c) an ellipse with as major axis.
cal square wave current of 2 A is [1999] R
R  V
(a) 2 A (b) 1 A (d) a circle of radius and origin at  0,  .
X  2
1 Solution: (a)
(c) A (d) 3A
2 Consider the Figure 1 given below
Solution: (a)
Consider the figure given below R jX

i(t )

2A

0 T ∼
T T 3T 2T I V at frequency ‘f ’
2 2
rms value of i(t) Figure 1

Phasor current
T
1 2 1 T
I rms = ∫
T0
i (t ) dt =
T
× 4 × = 2 A.
2 I=
V
R + jX
Hence, the correct option is (a).
X
8. The current in the circuit shown in Figure is [1999] V − j tan −1  
= ⋅e R
.
I
R2 + X 2
20 Ω + 200 V −
− V − j 90°
∼ 100 V 200 V For R = 0 I= e .
50 Hz + X
V
R=X I= e − j 45°.
(a) 5 A (b) 10 A 2X
(c) 15 A (d) 25 A
R=∞ 5 I = 0 e j 0.
Solution: (a)
Given circuit is under resonance because the volt- The three points corresponding to R = 0, R = X,
age across inductance is equal and opposite to V
R = ∞ lie on a semicircle of diameter as shown
voltage across the capacitance. X

Chapter 05.indd 46 11/10/2015 7:11:22 PM


Chapter 5  Sinusoidal Steady State Analysis  |  1.47

in Figure 2 given below (a) (3 − 14 ) A (b) (3 + 14 ) A


R=∞ (c) 5 A (d) None of these
V
V Solution: (c)
2X V
Px = P6 V − P1Ω = 6 × 1 − 12 × 1 = 5 W.
√ 2X
R=X By putting the options, it can be concluded that for
V
V 2X i = 5 A,
2X
Px = (6 × 5) − (52 × 1) = 5 W.
R=0 Hence, the correct option is (c).
Figure 2 1 2. A series RLC circuit has the following parameter
values: R = 10 W, L = 0.01 H, C = 100 mF. The
Therefore, locus of the tip of current vector is a Q-factor of the circuit at resonance is [1995]
V
semicircle of diameter . Solution:
X
Hence, the correct option is (a). For series RLC circuit,
10. A circuit with a resistor, inductor and capacitor ωL
Q-factor at resonance = 0 .
in series is resonant at f0 Hz. If all the c­ omponent R
­values are now doubled, the new resonant fre- Substituting the values we get
quency is [1998] 1 1
(a) 2 f0 (b) Still f0 ω0 = =
LC (0.01) × (100 × 10−3 )
f0 f0
(c) (d)
4 2 = 10 10 rad/ sec.
Solution: (d) L 10 10 × 0.01
For series RLC circuit the resonating frequency is Q = ω0 = = 0.032.
R 10
given by
13. In the circuit of Figure, ammeter A2 reads 12 A and
1
f 0 = (1) A3 reads 9 A. A1will read [1995]
2π LC
The new resonating frequency, when all the com- A1
ponents values are doubled will be

1
f new = (2) A2 A3
2π 2 L × 2C
From Equations (1) and (2), we get
Solution:
f0 Consider the figure given below in which the various
f new = .
2 currents are labelled
Hence, the correct option is (d). I
IR IL
11. In the circuit shown in Figure X is an element A1
12 A 9A
which always absorbs power. During a particular ∼
operation, it sets up a current of 1 A in the direc- A2 A3
tion shown and absorbs a power px. It is possible
that X can absorb the same power px for another As current lags the applied voltage in inductor by 90°,
current i. Then the value of this current is [1996]
X
I = (12 - j9) A.
Magnitude of current:
i
∴ I = 122 + 92 = 144 + 81 = 15 A.
+ −
6 1Ω Therefore, the ammeter A1 will read 15 A.

Chapter 05.indd 47 11/10/2015 7:11:25 PM


1.48 | Electric Circuits and Fields

14. At resonance, the given parallel circuit constituted Solution: (d)


by an iron-cored coil and a capacitor behaves like 2 H ≅ j2 Ω

 [1994]
5V
+ 1 F ≅ −j1 Ω 1F
∼ 10 sin t
R −
C
L We know that capacitor acts as open circuit in
steady state for DC voltage of 5 V and no current
flows in the circuit due to DC supply.
(a) an open-circuit. For AC supply of 10 sin (t), with w = 1 rad/sec, the
(b) a short-circuit. inductor
(c) a pure resistor of value R. j
Z = R + jω L −
(d) a pure resistor of value much higher than R. ωC
Solution: (d) = (1 + j1) W.
Consider the figure given below
For R = 1 W, L = 2 H, C = 1 F
A
Phasor current,
R 10
C
I= e − j 45°.
2
L
Current i(t) under steady state will be
B
= 7.07 sin(7 - 45°).
The given RLC the circuit resonates at
Hence, the correct option is (d).
11 16. The following circuit (Figure) resonates at [1993]
f = 1− 2 .
2π LC Q
10 Ω 4H
ZAB = R(1 + Q2). 1F
1F
ZAB is greater than R and for high Q coil
 ωL 
Q = >> 1 Z AB is very much higher than R. ∼
 R 
Under resonance condition, the given parallel cir- (a) all frequencies (b) 0.5 rad/sec
cuit behaves as a pure resistor of value much higher (c) 5 rad/sec (d) 1 rad/sec
than R. Solution: (b)
Hence, the correct option is (d). The impedance of the given circuit will be
15. In the following circuit (Figure) i(t) under steady
state is [1993]  j  j
Z = 10 +  j 4ω −   − 
 ω  ω
i (t)
1Ω 2H  1 
+  4 − ω2 
5 V ∼ 10 sin t 1F = 10 − j  .
−  4ω − 2 
 ω 

(a) zero (b) 5 For resonance imaginary part of impedance Z


(c) 7.07 sin t (d) 7.07 sin(t - 45°) must be equal to zero.

Chapter 05.indd 48 11/10/2015 7:11:27 PM


Chapter 5  Sinusoidal Steady State Analysis  |  1.49

Therefore, If voltage across capacitor = VC2 (t ),


1 Applying the KCL at the node, we get.
4− 2
= 0.
ωres VC2 VC2
+ − 10 sin 5t = 0.
⇒  wres = 0.5 rad/sec. 1 (− j 0.2)

Hence, the correct option is (b). 10 sin 5t


Or, VC2 (t ) = .
(1 + j 5)
Or, VC2 (t ) = 1.97 sin(5t − 78.69°). (2)
Two-marks Questions
Using superposition theorem, voltage across ca-
1. The voltage across the capacitor, as shown in the pacitor will be
figure, is expressed as Vc(t) = A1 sin(w1t - θ1) + A2 VC (t ) = VC1 (t ) + VC2 (t )
sin(w2t - θ1). [2014-S2]
= 2 sin(10t − 84.28°)
1Ω 1H
+ 1.97 sin(5t − 78.69°).
∴ VC (t ) = 2 sin(10t − 84.28°)
20 sin10t ∼ Vc (t ) 1F 10 sin 5t + 1.97 sin(5t − 78.69°). (3)
Given, VC(t) = A1 sin(w1t - θ1) + A2 sin(w2t - θ2).(4)
Comparing Equations (3) and (4), we have:
The values of A1 and A2 respectively, are
(a) 2.0 and 1.98 (b) 2.0 and 4.20 A = 2 and B = 1.97 ≈ 1.98 (closest answer).
(c) 2.5 and 3.50 (d) 5.0 and 6.40 Hence, the correct option is (a).
Solution: (a) 2. The total power dissipated in the circuit, shown in
Applying superposition theorem and consider­ing the figure, is 1 kW. [2014-S2]
the voltage source 20 sin 10t only and 10 sin 5t X c1 XL R
10 A 2A 1Ω
remain open circuited as shown below in figure.

X c2 Load
AC source

∼ V 200 V
VC1(t) 1
20 sin10t ∼ = −j0.10 Ω
j10 × 1

The voltmeter, across the load, reads 200 V. The


The voltage across capacitor, value of XL is .
Solution:
 − j 0.1 
∴ VC1 (t ) =   × 20 sin 10t Consider the figure given below
 1 − j 0.1  200
= (1.99∠ − 84.28°) sin 10t.
∴ VC1 (t ) ≈ 2 sin(10t − 84.28°). (1) VR VL

Considering the current source 10 sin 5t alone and


20 sin 10t voltage source remain short circuited, PT = 22 × 1 + 102 × R = 1000.
Vc2(t) 1 H 1000 − 4
R= = 9.96 Ω.
100
1Ω − j 0.2 Ω 10 sin 5t VR = 10 × 9.96 = 99.6.
VL = 2002 − 99.6 = 173.43.
VL = IX L .

Chapter 05.indd 49 11/10/2015 7:11:30 PM


1.50 | Electric Circuits and Fields

173.43 = 10 × X L . R I
173.43 V2
XL = = 17.3 Ω.
10 RL

Load
3. Two magnetically uncoupled inductive coils have ∼ V1 V3
Q-factors q1 and q2 at the chosen operating fre- X
quency. Their respective resistances are R1 and R2.
When connected in series, their effective Q-factor
at the same operating frequency is [2013] 
    
q1 q2 V1 = V2 + V3 .
(a) q1 R1 + q2 R2 (b) +
R1 R2 V12 = V22 + V32 + 2V2 V3 cos θ .
(q R + q R )
(c) 1 1 2 2 (d) q1 R2 + q2 R1 =
For V1 220
= V, V2 122 V, V3 = 136 V.
( R1 + R2 )
Solution: (c) PF, cos θ ≈ 0.45.
Hence, the correct option is (a).
ω L1 ω L2
q1 = , q2 = . 5. If RL = 5 W, the approximate power consumption
R1 R2
in the load is [2012]
R1 L1 (a) 700 W (b) 750 W
(c) 800 W (d) 850 W
R2 L2 Solution: (b)
Load resistor RL = 5 W.

ω ( L1 + L2 ) q1 R1 + q2 R2 RL
qt = = . ∴ cos φ = .
R1 + R2 R1 + R2 Z
5
Hence, the correct option is (c). ⇒ Z = = 11.11.
0.45
Common Data for Questions 4 and 5: Power consumed by load will be
In the circuit shown, the three voltmeter readings
2
=are V1 220= V, V2 122 V, V3 = 136 V. V   136 
2

PL =  3  RL =   × 5 = 749.1  750 W.
 Z  11.11 
R
 
I
V2 Hence, the correct option is (b).
RL 6. A lossy capacitor Cx rated for operation at 5 kV,
Load

∼ V1 V3
50 Hz is represented by an equivalent circuit with
X an ideal capacitor Cp in parallel with a resistor Rp.
The value of Cp is found to be 0.102 μF and the
value of Rp = 1.25 M W. Then the power loss and
tan δ of the lossy capacitor operating at the rated
4. The power factor of the load is [2012] voltage, respectively, are [2011]
(a) 0.45 (b) 0.50 (a) 10 W and 0.0002 (b) 10 W and 0.0025
(c) 0.55 (d) 0.60 (c) 20 W and 0.025 (d) 20 W and 0.04
Solution: (a) Solution: (c)
Let power factor of the load = cos(θ), Cp = 0.102 μF.
 
where, θ = Angle between VRL and V3 Rp = 1.25 MW.

 

 = Angle between V2 and V3 , V 2 (5 × 103 ) 2 25
  
  Power loss = = = = 20 W.
 as VRL = IRL and V2 = IR. RP 1.25 × 10 6
1.25

Chapter 05.indd 50 11/10/2015 7:11:34 PM


Chapter 5  Sinusoidal Steady State Analysis  |  1.51

R 1 8. The current I C in the figure above is [2011]


tan δ = =
X ωCP RP 1
- j2 A
(a) (b) − j A
1 2
= = 0.025. 1
2 × π × 50 × 0.102 × 10−6 × 1.25 × 106 +j
(c) +j 2 A
A (d)
Hence, the correct option is (c). 2
Solution: (d)
Common Data for Questions 7 and 8:
Using KCL,
An RLC circuit with relevant data is given below.
− I S + I RL + I C = 0.
Is
Ic ⇒ I C = I S − I RL.
+ R
π π
Vs ∼ C = 2∠ − 2∠ − = 2∠90° = + j 2 A.
− L
4 4
Hence, the correct option is (d).
9. The RLC series circuit shown is supplied from a
V s = 1∠0 V. variable frequency voltage source. The admittance
—locus of the RLC network at terminals AB for
π increasing frequency w is [2007]
I s = 2 ∠ A.
4
π
I RL = 2 ∠ − A. A R
4
w ∼
7. The power dissipated in the resistor R is [2011] L
B C
(a) 0.5 W (b) 1 W
(c) 2 W (d) 2 W
Solution: (b) (a)
Img (b)
Img

 VS 1 −j
π
I RL = = = 2e 4 .
R + jω L R + jω L Re w
Re
1 w
∴ = 2.
R 2 + ω 2 L2 Img
(c) Img (d)
ωL π
And − tan −1 =− ,
R 4 w
Re
Re
ωL π w
  = tan = 1, ω L = R.
R 4 Solution: (c)
1 The admittance—locus of the RLC network at ter-
∴ 2
= 2, minals AB for increasing frequency w is
   R + R2
1 1 1
R2 = , R = Ω, YAB = .
  4 2  1 
R + j ωL − 
Power dissipated in the resistor,  ω C
1
2 1 YAB = .
R = I RL R = ( 2 )2 ×= 1 W. 1 
2
2 2
R + ωL − 
Hence, the correct option is (b).  ωC 

Chapter 05.indd 51 11/10/2015 7:11:39 PM


1.52 | Electric Circuits and Fields

 1  Solution: (a)
  ω L − ωC   + +
∠YAB = − tan −1   . I
 R  V∠0° ∼− Vyx
R

  −
  + x y
 1  V∠0° ∼ C
−1
 ω L − ωC  −
θ = − tan  . O
 R 
  Let capacitive reactance = XC.

w V ∠0° + V ∠0° 2V
|YAB| ∠YAB I= = .
R − jX C R − jX C
0 0 90°
Applying KVL, to the closed loop we get
1 1
0 VYX + IR = 0.
LC R
 ⇒ VYX = V - IR.
∞ 0 -90°
 2V 
VYX = V −  R
Phasor diagram is shown is below  R − jX C 
Im
V ( R − jX C ) − 2VR
=
Y-plane R − jX C
w w0 = 1 V ( R + jX C )
w=0 √LC   =− .
( R − jX C )
Re
1
w=∞ R  R + jX C 
VYX = −V  .
 R − jX C 
w  

Hence, the correct option is (c). When, R = 0


1 0. In the figure given below, all phasors are with  0 + jX C 
reference to the potential at point O. The locus VYX = −V   = V.
of voltage phasor VYX as R is varied from zero to  0 − jX C 
infinity is shown by [2007]  XC 
1 + j R 
∴ VYX =  .
V∠0° ∼ R 1 − j X C 
Vyx 
   R 
XY When R → ∞
V∠0° ∼ VYX = -V
O Hence, the correct option is (a).
(a)
0 2 V (b) Locus of VYX
11. The resonant frequency for the given circuit will
VYX be [2007]
0.1 H
VYX
Locus of VYX 0 2V
1F 1Ω
(c)
0 2 V (d) Locus of VYX
VYX

VYX (a) 1 rad/sec (b) 2 rad/sec


Locus of VYX 0 2V (c) 3 rad/sec (d) 4 rad/sec

Chapter 05.indd 52 11/10/2015 7:11:41 PM


Chapter 5  Sinusoidal Steady State Analysis  |  1.53

Solution: (c) Solution: (a)


Impedance Zin is calculated for the circuit shown Consider the figure given below
below
I I
0.1 H
RA RB

1F 1Ω
V2


1 V1
Z in = j 0.1ω + XC VC
1 + jω
(1 − 0.1ω 2 ) + j 0.1ω
= .
(1 + jω ) Z = RA + RB + j ( X L − X C ).
 0.1ω  At resonance,
∠Z in = tan −1  −1
 − tan (ω ).
 1 − 0.1ω
2
 XL = XC.
Impedance
For resonance to take place ∠Z in = 0,
Z = RA + RB.
0.1ω
i.e., ω =  or 1 − 0.1 ω 2 = 0.1. Input impedance is purely resistive in nature and is
1 − 0.1ω 2 minimum, therefore, the input voltage and output
 ω 2 = 9, ω = 3 rad/sec. current are in phase.
Hence, the correct option is (c). Therefore voltage V1 and current I are in phase.
1 2. The circuit shown in the figure is energized by a V1
sinusoidal voltage source V1 at a frequency which V2 = × [ RB + j ( X L − X C )].
RA + RB + j ( X L − X C )
causes resonance with a current of I. [2006]
But XL = XC.
I
V1
V2 = × RB .
RA + RB
V2 Therefore, V1 is in phase with V1 and V2 < V1.
∼ Voltage across the capacitor will be
V1
1
VC VC = I × X C = I × .
jωC
The phasor diagram which is applicable to this cir- 1
VC = ∠ − 90°.
cuit is ωC
(a) V1 (b) Thus, VC lags the current by 90°.
I V2 V2 I
The phasor diagram is shown below
V1
Vc I V2
Vc V1
(c)
Vc (d)
V1

V1 Vc
V2
V2 I I Hence, the correct option is (a).
1 3. The RL circuit of the figure is fed from a constant
Vc magnitude, variable frequency sinusoidal voltage

Chapter 05.indd 53 11/10/2015 7:11:45 PM


1.54 | Electric Circuits and Fields

source VIN. At 100 Hz, the R and L elements each Solution: (d)
have a voltage drop urms. If the frequency of the Consider the figure given below
source is changed to 50 Hz, then new voltage drop
across R is [2005] IR IL IC

E = 10∠0° V
R I
YR YL YC
V INM L

I R = YR E = (0.5 + j 0) × 10∠0° = 5 A.
5 2
(a) urms (b) urms IY = YL E = (0 − j1.5) × 10∠0° = − j15 A.
8 3
I C = YC E = (0 + j 0.3) × 10∠0° = j 3 A.
8 3 I = I R + IY + I C = 5 + (− j15) + j 3 = 5 − j12 A.
(c) urms (d) urms
5 2
Hence, the correct option is (d).
Solution: (c)
Frequency f = 100 Hz, VR = VL = urms, 1 5. In the circuit of the figure, the magnitudes of VL
and VC are twice that of VR. The inductance of the
ωL
i.e., = 1. coil is [2003]
R VR
VIN R
VR = . 5Ω
C VC
R 2 + ω 2 L2 5∠ 0° ∼
When f = 50 Hz, then we have L VL

VIN R VR′ R 2 + ω 2 L2
VR′ = ⇒ = (a) 2.14 mH (b) 5.30 H
ω 
2 VR 2 ω 2 L2 (c) 3.18 mH (d) 1.32 H
R +   L2
2
R +
2 4 Solution: (c)
2 For the RLC series circuit shown below, frequency
 ωL 
1+   of operation is not specified.
 R  2 8 VR
= = = .
 ωL 
2 1.25 5 5Ω
1+   C VC
 2R  ∼ 5∠ 0°
L VL
8 8
∴ VR′ = VR = urms .
5 5 We have,

 
Hence, the correct option is (c). VL = VC = 2 VR , VL = −VC .
1 4. In the figure the admittance values of the elements The given circuit is at resonance
in Siemens are YR = 0.5 + j0, YL = 0 - j1.5, YC = 0 ∴ VR = 5 V, VL = 10 V.
+ j0.3 respectively. The value of I as a phasor when
10
the voltage E across the elements is 10∠0° V is ω L = 10, L = .
 [2004] ω
If frequency is 100 Hz, then the inductance will be
10 50
I YR YL YC E = 10∠0° V L= H= mH = 3.18 mH.
2π × 100 π
Hence, the correct option is (c).
(a) 1.5 + j0.5 (b) 5 - j18 16. In the circuit shown in the figure, what value of C will
(c) 0.5 + j1.8 (d) 5 - j12 cause a unity power factor at the AC source? [2002]

Chapter 05.indd 54 11/10/2015 7:11:48 PM


Chapter 5  Sinusoidal Steady State Analysis  |  1.55

15.49
⇒ f = = 2.46 kHz.
230 V

∼ C ZL = 30∠40°
50 Hz Hence, the correct option is (c).
18. A series RLC circuit has R = 50 W; L = 100 μH and
C = 1 μF. The lower half power frequency of the
(a) 68.1 μF (b) 165 μF circuit is [2002]
(c) 0.681 μF (d) 6.81 μF (a) 30.55 kHz (b) 3.055 kHz
Solution: (a) (c) 51.92 kHz (d) 1.92 kHz
Impedance ZL = 30∠40°. Solution: (b)
1 − j 40° The lower half-power (or 3 dB) frequency of the
Yi = jωC + e current response in series RLC circuit is given by
30
1 1 f1 = f 0 ( 1 + δ 2 − δ ),
= jωC + cos( 40°) − j sin(40°)
30 30 1
1  1  δ= .
= cos( 40°) + j ωC − sin(40°)  . 2Q
30  30 
R = 50 Ω, L = 100 µH,
For frequency f = 50 Hz,
C = 1µF, LC = 10−10.
1 1 1
C= sin(40°) = × 0.643 × 106 µF ω0 = = 105 rad/sec,
30 3000π LC
= 68.2 µF.
ω0 L 105 × 10−4
Hence, the correct option is (a). Q= = = 0.2.
R 50
1 7. A first order, low pass filter is given with R = 50 W
δ = 2.5.
and C = 5 μF. What is the frequency at which the
gain of the voltage transfer function of the filter ω1 = 0.19 × 105 rad/sec, f1 = 3 kHz.
is 0.25? [2002]
Hence, the correct option is (b).
(a) 4.92 kHz (b) 0.49 kHz
(c) 2.46 kHz (d) 24.6 kHz 1 9. In the circuit shown in the figure, it is found that
the input AC voltage (Vi) and current i are in phase.
Solution: (c)
M
Consider the figure given below The coupling coeffoicient is K = , where M
L1 L2
R
is the mutual inductance between the two coils. The
+ +
value of K and the dot polarity of the coil P-Q are
V1 C V0  [2002]
− − K
i (t ) −j 12 Ω 10 Ω
1 + j8 Ω P j8 Ω Q
V0 jωC 1
TF = = = . V(t )
V1 R + 1 1 + jωCR −
  jωC
(a) K = 0.25 and dot at P
1 (b) K = 0.5 and dot at P
∴ Gain = .
1 + (ωCR) 2 (c) K = 0.25 and dot at Q
(d) K = 0.5 and dot at Q
1
0.25 = . Solution: (c)
1 + (ω × 5 × 10−6 × 50) 2 Input AC voltage and current will be in phase only
ω = 15.49 × 103 rad/sec. at resonance condition,

Chapter 05.indd 55 11/10/2015 7:11:52 PM


1.56 | Electric Circuits and Fields

i.e., X C = X L . 1 1
YAB = + .
− j 0.02 R + j 0.01
− j12 = j8 + j8 + 2k ( j8) × ( j8) .
For  R = 0, YAB = -j50 = 50∠-90°.
12 = 8 + 8 + 16k .
For  R = 0.01, YAB = 50.
4 1 For  R = ∞, YAB = j50 =50∠90°.
⇒ k=− = − = −0.25.
16 4 Plotting these three points, we get
Hence coupling will be opposite
Imag (YAB)
\  Dot will be at Q.
Hence, the correct option is (c). R=∞
j50
2 0. The impedance seen by the source in the circuit in
the figure is given by [2000] 50
j100 0 YAB
R = 0.01
4Ω −j 2 Ω
1:4
− j50
R=0
+ ZL = 10∠30°


 Therefore, locus of Y AB is a semicircle having
­center at zero and diameter j100.
(a) (0.54 + j0.313) W (b) (b)(5 - j2) W Hence, the correct option is (a).
(c) (0.54 - j1.69) W (d) (4 + j2) W
22. The voltage phasor of a circuit is 10∠15° V and
Solution: (c) the current phasor is 2∠-45° V. The active and the
The impedance of the given circuit will be reactive powers in the circuit are [1999]
2 (a) 10 W and 17.32 VAR
1
Z = (4 − j 2) +   × 10∠30° = (4.54 − j 1.69) Ω. (b) 5 W and 8.66 VAR
4 (c) 20 W and 60 VAR
Hence, the correct option is (c). (d) 20 2 W and 10 2 VAR
2 1. A fixed capacitor of reactance -j0.02 W is con- Solution: (a)

nected in parallel across a series combination of a For  V = 10∠15° = 10 e j15°
fixed inductor of reactance j0.01 W and a variable 
resistance R. As R is varied from zero to infinity, and  I = 2∠ − 45° = 2 e − j 45°.
the locus diagram of the admittance of this LCR Complex power, is given by
circuit will be [1999] 
S = VI * = 20 e j 60°.
(a) a semi-circle of diameter j100 and center at zero.
(b) a semi circle of diameter j50 and center at zero. Active power,
(c) a straight line inclined at an angle. P = Re(S) = 20 cos 60° = 10 W.
(d) a straight line parallel to the x-axis. Reactive power,
Solution: (a) Q = Im(S) ==
20 sin 60° 10
= 3 VAR 17.32 VAR.
Consider the figure given below
Hence, the correct option is (a).
−j0.02 Ω 2 3. Two identical coils of negligible resistance when
connected in series across a 200 V, 50 Hz source
C
R j 0.01 Ω draws, a current of 10 A. When the terminals of
one of the coils are reversed, then current drawn is
YAB 8 A. The coefficient of coupling between the two
A B coils is [1997]

Chapter 05.indd 56 11/10/2015 7:11:55 PM


Chapter 5  Sinusoidal Steady State Analysis  |  1.57

Solution: Solution: (d)


For series opposing with higher current 10 A, con­ The quality factor is given by relation
sider the Figure 1 given below. ωL
Q= ,
10 A L R
where w = 2π f.
200 V
From the above relation we conclude that quality
50 Hz L factor depends directly on the frequency. When
frequency of operation is doubled, w = 2πf , Q also
get doubled.
Figure 1 2
 V 
For series aiding with lower current 8 A, consider P=I R= 2
 ⋅R
 R 2 + (ω L) 2 
Figure 2 given below.
L V2 ⋅R V2
8A = = .
2
  ω L  2  R (1 + Q 2 )
R 1 +   
200 V   R  
50 Hz L
As per problem Q is high, therefore
∴ Q 2 >> 1.
Figure 2
V2
⇒ P≅ .
  RQ 2
200
10 = (1) From the above relation we conclude that power is
ω (2 L − 2M )
inversely related to square of quality factor. Thus if
200 Q is doubled, P decreases 4 times.
8 = (2)
ω (2 L + 2M ) Hence, the correct option is (d).


25. In the given circuit, the voltage VL has a phase angle
Dividing by Equation (1) by (2) 
of with respect to V s. [1994]
L+M 5 VL
= , 4 L + 4 M = 5 L − 5M . + −
L−M 4
1  17.32 Ω j10
M = L E.
9 VS
1 ∼
M = k L1 L2 = L L.
9 Solution:
1 Consider the figure given below
k= .
9 +
VL

17.32 Ω
24. A coil (which can be modeled as a series RL cir-
j10 Ω
cuit) has been designed for high -Q performance
at a rated voltage and a specified frequency. If the ∼
frequency of operation is doubled, and the coil
VS
is operated at the same rated voltage, then the 
Q-factor and the active power P consumed by the  V S j10
coil will be affected as follows [1996] VL = .
17.32 + j10
(a) P is doubled, Q is halved  
(b) P is halved, Q is doubled V L lags V S by
(c) P remains constant, Q is doubled  10 
(d) P decreased 4 times, Q is doubled
θ = 90° − tan −1   = 90° - 30° = 60°.
 17.32 

Chapter 05.indd 57 11/10/2015 7:11:59 PM


1.58 | Electric Circuits and Fields

Series RLC circuit is shown in Figure 3 given below


Five-marks Questions R C

1. Determine the resonance frequency and the Q-


factor of the circuit shown in the figure. [2001] Vi ∼ L equ
M

R Figure 3
+ C
∼ L1 L2
− L1 = 40 mH, L2 = 10 mH, M = 10 mH

C = 3 µF, R = 10 Ω.
Data: R = 10 W, C = 3 μF 100
Lequ = 40 − = 30 mH,
 L1 = 40 mH, L2 = 10 mH 10
And M = 10 mH Lequ C = 30 × 10−3 × 3 × 10−6 = 9 × 10−8 ,
Solution:
Consider the coupled circuit shown in Figure 1 Leq C = 3 × 10−4.
given below
Resonance frequency, can be expressed as
P M
R C 1 104 104
+ ω0 = = rad/sec, f0 = Hz.
Vi ∼ L1 L2 Leq C 3 6π

ω0 Lequ 104 30 × 10−3
Q Q= = = 10.
R 3 10
Figure 1
2. A circuit consisting of a single resistor R and an
Leq across P, Q in the primary can be calculated
inductor L in series is driven by a 25 V rms, 50 Hz
using Figure 2 given below
sinusoidal voltage source. A capacitor is to be placed
M in parallel with the source to improve the power fac-
+ tor. Given that the average power dissipated in the
L1 L2 R is 100 W and that the reactive power delivered to
Vi
the L is 75 VAR, what value of C will yield a 0.9 pf
I1
− I2 lagging as seen by the source? [1998]
Figure 2 Solution:
Consider the figure given below
Vi = jω L1 I1 + jω MI 2 .
jω L2 I 2 + jω MI1 = 0.
R
−M
I2 = I1 . 25 V,
∼ C
L2 50 Hz
L
 M  2
Vi = j ω  L1 −  I1 .
 L2 

M2 Active power consumed by resistor R will be,


Leq = L1 − .
L2 P = 100 W.

Chapter 05.indd 58 11/10/2015 7:12:00 PM


Chapter 5  Sinusoidal Steady State Analysis  |  1.59

For inductor L the reactive power will be, Solution:


2Ω −j 1 Ω −j 2 Ω
Q1 = 75 VAR.
Power factor angle +
j1 Ω
∼ Vs j3 Ω j3 Ω
Q I1 I2 2 Ω
−1 −
φ = tan  
P
 75  Consider the KVL equations by using dot convention.
= tan −1   = 36.86°.
 100 
Vs = (2 − j1 + j 3) I1 + j1I 2 .
At 0.9 power factor, reactive power demanded by L, Va = (2 + j 2) I1 + j1I 2 (1)
P
Q2 = sin(cos −1 0.9) = 48.43 VAR. j1I1 + (2 − j 2 + j 3) I 2 = 0.
0.9
j1I1 + (2 + j1) I 2 = 0 (2)
Reactive power to be supplied by shunt capacitor
− j1
(QC) = 75 - 48.43 = 26.56 VAR. or  I 2 = I1 (3)
2 + j1
QC = V 2ωC = 26.56 VAR. From Equations(1) and (3)
26.56 (− j1)
C= = 135.5 µF. Vs = (2 + j 2) I1 + j1 I1 .
(25) 2 × (2π × 50) (2 + j1)
3. Determine the impedance seen by the source Vs =  1 
Vs = (2 + j 2) +  I1 .
24∠0° in the network shown in the figure. [1998]  ( 2 + j1) 
−j 1 Ω −j 2 Ω
Vs (2 − j1)
2Ω = ( 2 + j 2) +
I1 5
j1 Ω
I1 I2 = (2 + j 2) + 0.4 − j 0.2 = 2.4 + j1.8.
Vs ∼ j3 Ω j3 Ω 2Ω
V
Z in = s = (2.4 + j1.8) Ω.
I1

Chapter 05.indd 59 11/10/2015 7:12:03 PM


Chapter 6
Three-phase Circuits
∴  For zero neutral current,
Two-marks Questions
I A + I B + I C = 0.
1. A 230 V (phase), 50 Hz, 3-phase, 4-wire system 17.39∠0° + 10∠ − 210° + 10∠ − 150° ≅ 0.
has a phase sequence ABC. A unity power factor
Hence, the correct option is (b).
load of 4 kW is connected between Phase A and
Neutral N. It is desired to achieve zero neutral cur- 2. A 3-phase balanced star connected voltage source
rent through the use of a pure inductor and a pure with frequency w rad/s is connected to a star con-
capacitor in the other two phases. The values of nected balanced load which is purely inductive. The
inductor and capacitor are [2007] instantaneous line currents and phase to neutral volt-
(a) 72.95 mH in Phase C and 139.02 mF in Phase B ages are denoted by (ia, ib, ic) and (van, vbn, vcn) respec-
(b) 72.95 mH in Phase B and 139.02 mF in Phase C tively and their rms values are denoted by V and I. If
(c) 42.12 mH in Phase C and 240.79 mF in Phase B  1 1 
 0 − 
(d) 42.12 mH in Phase B and 240.79 mF in Phase C 3 3 i
  a
Solution: (b)  1 1  
R = [van , vbn , vcn ]  − 0  ib ,
Load current of Phase A =
4 × 103
= 17.39 A.  3 3 
i 
230∠0°  1 1  c
 − 0 
VA = 230∠0°.  3 3 
VB = 230∠ − 120°. then the magnitude of R is [2007]
VC = 230∠ − 240°. (a) 3 VI (b) VI
(c) 0.7 VI (d) 0
From options
Solution: (a)
L = 72.95 mH, X L = 22.91 Ω.
 1 1 
I L ≅ 10∠ − 90° = 139.02 µF, X C = 22.91 Ω,  ib − ic 
 3 3 
I C ≅ 10∠90°.  −1 1 
R = [van vbn vcn ]  ia + ic 
By assuming inductor in Phase B and capacitor in  3 3 
 1 1 
Phase C  ia − ib 
 3 3 
I B = 10∠ − 90 − 120 = 10∠ − 210°. 1
R= [Van (ib − ic ) + Vbn (ic − ia ) + Vcn (ia − ib )] (1)
I C = 10∠90 − 240 = 10∠ − 150°. 3

Chapter 06.indd 60 11/10/2015 6:44:45 PM


Chapter 6  Three-phase Circuits  |  1.61

If the instantaneous phase voltages be the delta connected (capacitive) reactive power
bank necessary to bring the pf to unity is [2006]
Van = Vm sin ωt.
(a) 28.78 (b) 21.60
Vbn = Vm sin(ωt − 120°). (c) 16.60 (d) 12.47
Vcn = Vm sin(ωt + 120°). Solution: (d)
For star connected load rating is given by12 3 kVA,
Since, the load is purely inductive, the current lags 0.8 pf (lag).
the voltage by 90°.
Active power consumed by the load,
∴ ia ( line ) = ia phase = im sin(ωt − 90°).
P = 12 3 × 0.8 × 103 = 16.627 kW.
ib ( line ) = ib phase = im sin(ωt − 120 − 90°)
Reactive power consumed by the load
= im sin(ωt − 210°).
ic ( line ) = ic phase = im sin(ωt + 120 − 90) = 12 3 × sin(cos −1 0.8) × 103.
Q1 = 12.47 kVAR.
= im sin(ωt + 30°).
ib − ic = im [sin(ωt − 210) − sin(ωt + 30°)] At unity power factor reactive power consumed by
the load is
= − 3 im cos(ωt − 90°).
P
ic − ia = im [sin(ωt + 30°) − sin(ωt − 90°)] Q2 = × sin(cos −1 1) = 0.
(1)
= 3 im cos(ωt − 30°).
∴  kVAR to be supplied by the delta connected
ia − ib = im [sin(ωt − 90°) − sin(ωt − 210°)]
capacitor bank = Q1 - Q2.
= 3 im cos(ωt − 30°).
QC = 12.47 kVAR.
∴  Equation (1) becomes
Hence, the correct option is (d).
1
R= [Vm sin ωt (− 3 )im cos(ωt − 90°)
3 4. A balanced delta connected load of (8 + j6) Ω
per phase is connected to a 400 V, 50 Hz, 3-phase
+ Vm sin(ωt − 120°) ⋅ 3 im cos(ωt − 30°) supply lines. If the input power factor is to be
improved to 0.9 by connecting a bank of star con-
+ Vm sin(ωt + 120°) ⋅ 3 im cos(ωt − 30°)].
nected capacitors, the required kVAR of the bank
1 is [2003]
R = Vm im [− sin(2ωt − 150°) + sin(−90°)
2 (a) 42.7 (b) 10.2
+ sin(2ωt − 150°) − sin 90° (c) 28.8 (d) 38.4

+ sin(2ωt − 30°) − sin(−270°)]. Solution: (b)


Consider the figure given below
Vm im
R= (−3).
2 2 R
R = −3VI . 8 8
400 V
∴  Magnitude of R = |− 3VI | = 3VI . 50 Hz j6
3-f j6
Hence, the correct option is (a).
3. A 400 V, 50 Hz, 3-phase balanced source supplies B
8 j6
power to a star connected load whose rating is
12 3 kVA, 0.8 pf (lag). The rating (in kVAR) of Y

Chapter 06.indd 61 11/10/2015 6:44:46 PM


1.62 | Electric Circuits and Fields

Power factor angle of load Solution:


We know that only resistor consumes power and
6 inductor and capacitor do not consume any power.
(φ ) = tan −1   = 36.86°.
8 Power consumed in unbalanced delta connected
V ph2 VL2 (400) 2
Active power consumed by the delta connected = ==
load = 1600 W.
balanced load is R R 100
Power consumed in a balanced star connection con-
P = 3 × V ph × I ph × cos φ taining a set of equal resistors (in all three phases),
each of value
400
= 3 × 400 × × cos 36.86 = 38400 W.
82 + 6 2 V ph2  VL2 (400) 2
RX = 3  = = .
Reactive power consumed by the delta connected  RX  RX R
load is
It is given that, power consumed in both the cases
Q1 = 3 × V ph × I ph × sin φ is equal,
400 (400) 2
= 3 × 400 × × sin 36.86 ∴ 1600 =
82 + 6 2 R
= 28793.36 VAR. ⇒ R = 100 Ω.

Active power consumption remains same even 6. Using Thevenin equivalent circuit, determine the
after capacitor bank is connected. Reactive power rms value of the voltage across the 100 Ω resistor
consumed by the delta connected load at a power after the switch is closed in the 3-phasse circuit
factor of 0.9 will be shown in the figure. [1992]
P
Q2 = × sin(cos −1 0.9) R P
0.9
38400 ∼ 300 Ω 100 Ω
= × sin 25.84
0.9 300 Ω
∼ ∼ 300 Ω
= 18597.96 VAR.
B Y
∴ Q2 = 18597.96 VAR.
Reactive power supplied by star connected capaci-
400 V/√3 Phase Balanced Source
tor bank = Q1 - Q2 = 28793.36 - 18597.96
= 10195 ≅ 10.2 KVAR. Solution:
Figure 1 shows the circuit with switch open
Hence, the correct option is (b).
5. A set of 3 equal resistors, each of value RX, con- P
R
nected in star across RYB of given figure consumes
the same power as the unbalanced delta-connected 400/√3 V 300 Ω Vth 100 Ω
n
load shown. The value of RX is Ω. [1994] 400 V
Q
300 Ω 300 Ω
R B
400 V j 50 Y
100
3-f
-j50 Figure 1
Y
We know that Thevenin voltage is equal to the
B open circuit voltage when the switch is open.

Chapter 06.indd 62 11/10/2015 6:44:47 PM


Chapter 6  Three-phase Circuits  |  1.63

Phase voltage, 300


∴ Z th = = 100 Ω.
3
400
VRn = V.
3 Figure given below shows Thevenin’s equivalent
Thevenin’s equivalent impedance can be calculated circuit across R, n with the switch closed and
if we short circuit the voltage sources as shown in 100 Ω load across P, Q
Figures 2 and 3 given below zth = 100 Ω Sw
R P
R
R
√3
300 Ω zth ∼ Vth = V 100 Ω
n 400

300 Ω 300 Ω
B n Q
Y ∴  rms value of voltage across 100 Ω resistance will
Figure 2 be
400
= = V 115.5 V.
≡ 300 Ω 300 Ω 300 Ω zth 2 3

Figure 3

Chapter 06.indd 63 11/10/2015 6:44:47 PM


Chapter 7
Two Port Networks
When i1 = 0
One-mark Questions
v1 = i2 z1
1. For the 2-port network shown in the figure, the
Z-matrix is given by [2005] v1
⇒ z12 = = z1 (3)
i1 Z2 i2 i2 i1 = 0
+ +
V1 Z1 V2 v2 = i2 ( z1 + z2 )
− −
v2
⇒ z22 = = z1 + z2 (4)
 Z1 Z1 + Z 2   Z1 Z1  i2
(a)
Z + Z (b) i1 = 0

 1 Z 2  Z + Z
 1 Z 2 
2 2
Hence, the correct option is (d).
 Z1 Z2   Z1 Z1  2. A passive 2-port network is in a steady-state.
(c)
Z  (d)
  Compared to its input, the steady state output can
 2 Z2 + Z2   Z1 Z1 + Z 2 
never offer [2001]
Solution: (d) (a) higher voltage (b) lower impedance
Consider the Figure given below (c) greater power (d) better regulation
i1 Z2 i2 Solution: (c)
If the network is passive, then for a two port net-
V1 V2
work, the output power delivered can never be
Z1
greater than the input power.
Hence, the correct option is (c).
When i2 = 0 3. For the circuit shown in the figure the transfer
v1 = v2 = i1z1 function is equal to . [1995]
v1
=
So, z11 = z1 (1) L R
i1 i = 0
2

e1(t) C e0(t)
v2
   z21
= = z1 (2)
i1 i = 0
2

Chapter 07.indd 64 11/10/2015 5:48:36 PM


Chapter 7  Two Port Networks  |  1.65

Solution: +
Consider the Figure given below
A B

Ls I
+ + 1. With 10 V DC connected at Port A, the current
1 drawn by 7 Ω current at Port B is [2012]
E1(S) Cs E 0(S)
3 5
(a) A (b) A
− − 7 7

Using voltage division rule, we get 9


(c) 1 A (d) A
7
 1  Ei ( s )
Eo ( s ) =   ⋅ . Solution: (c)
 Cs  R + Ls + 1 Using Thevenin theorem we get
Cs Observation 1
 1  IL
E (s)   3A
∴ TF = H ( s ) = o =  LC  .
A
10 V Network 1Ω
Ei ( s ) s 2 + R s + 1 B
L LC
Now we have,
Observation 2
ω02
H (s) = , 2A
ωo A
s +
2
s + ω0
2
10 V N/W B 2.5 Ω
Q

where,
1 ωL Simplifying the linear, active, bilateral network
ω02 = , Q= o . across Port 2 according to Thevenin’s theorem we get
LC R
RT IL
4. If a 2-port network is passive, then we have, with the
V th ± B RL
usual notation, the following relationship [1994]
(a) h12 = h21 (b) h12 = -h21
(c) h11 = h22 (d) h11h22 - h12h21 = 1 By substituting, we get
Solution: (b) Vth = 3Rth + 3 (1)
A 2-port network will be passive or reciprocal then
the condition in h-parameter model will be h12 = -h21. Vth = 2Rth + 5 (2)
Hence, the correct option is (b). Solving Equations (1) and (2), we get,
Vth = 9 V
Two-marks Questions Vth = 2 Ω

Common Data for Questions 1 and 2: 2Ω I


With 10 V DC connected at Port A in the linear non- 9V ± 7Ω
reciprocal 2-port network shown below, the following
were observed:
(i) 1 Ω connected at port B draws a current of 3 A I=1A

(ii) 2.5 Ω connected at port B draws a current of 2 A Hence, the correct option is (c).

Chapter 07.indd 65 11/10/2015 5:48:37 PM


1.66 | Electric Circuits and Fields

2. For the same network with 6 V DC connected at  Z11 + 1 Z12 + 1   Z11 + 1 Z12 
7 (a)
  (b)
 
port A, 1 Ω connected at Port B draws A. If 8 V Z
 21 Z 22 + 1  Z
 21 Z 22 + 1 
3
DC connected to Port A, the open circuit voltage at  Z11 + 1 Z12   Z11 + 1 Z12 
Port B is [2012] (c)
  (d)
 
 Z 21 Z 22   Z 21 + 1 Z 22 
(a) 6 V (b) 7 V
(c) 8 V (d) 9 V Solution: (c)
Solution: (d) Consider the Figure given below
Consider the figure given below I1 1 Ω
a
I2
e c
Rth
I V 1ef V 1ab P V2
+ f d
b
10 V RL ⇒ ± Vth RL
V2 does not get affected because 1 Ω resistor is

A B connected in series with the network at Port 1

We know that V1ef = V1ab + I1 × 1


Vth = Z11 I1 + Z12 I 2 + I1 = ( Z11 + 1) I1 + Z12 I 2
3 = (1)
1 + Rth Modified Z-parameter can be expressed as
and  Z + 1 Z12 
Vth =  11 
2 = (2)  Z 21 Z 22 
2.5 + Rth
Hence, the correct option is (c).
Dividing Equations (1) and (2), we can calculate 4. The parameters of the circuit shown in the figure,
the Thevenin’s resistance are Ri = 1 MΩ, Ro = 10 Ω, A = 106 V/V. If Vi =
3 Rth + 2.5 1 mV, the output voltage, input impedance and out-
⇒ =
2 Rth + 1 put impedance respectively are [2006]
Ri Ro
⇒ 2 Rth + 5 = 3Rth + 3
⇒ Rth = 2 Ω +
Vi + AVi


And Thevenin’s voltage will be
Vth = 9 V (a) 1 V, ∞, 10 Ω (b) 1 V, 0, 10 Ω
9
I= = 1 A. (c) 1 V, 0, ∞ (d) 10 V, ∞, 10 Ω
7+2
Solution: (a)
Hence, the correct option is (d). Consider the Figure given below
3. The 2-port network P shown in the figure, has Ports Ri Ro
1 and 2, denoted by terminals (a, b) and (c, d), + + +
respectively. It has an impedance matrix Z with Z in + Zo
Vs Vi − AV Vo
parameters denoted by zij. A 1 Ω resistor is con- i
− − −
nected in series with the network at Port 1 as shown
in the figure. The impedance matrix of the modified For Vi = 1 mV
2-port network (shown as a dashed box) is [2010]
A = 106 , Ri = 1 MΩ
e 1Ω a c Ro = 10 Ω
P Vo = AVi = 106 × 10−6 = 1 V
f b d Z in = ∞

Chapter 07.indd 66 11/10/2015 5:48:38 PM


Chapter 7  Two Port Networks  |  1.67

Source is short circuited. in order to calculate Z0, Solution: (d)


VS = 0, Vi = 0 and AVi = 0 We know that
∴  Z0 = R0 = 10 Ω V1 = 0.9 I1 + 0.2 I 2
Hence, the correct option is (a). V1 = 0.9 I1 + 0.2 I 2
5. Two networks are connected in cascade as shown I2
in the figure with the usual notations the equiva- y22 =
V2
lent A, B, C and D constants are obtained. Given V1 = 0

that, C = 0.025∠45°, the value of Z2 is [2005] 0.9 I1 + 0.2 I 2 = 0


Z 1 = 10∠30° Ω
 0.2 
V2 = 0.2 ×  − I 2  + 0.6 I 2
 0.9 
Z2
0.5
= I 2 = y22
0.9
9
(a) 10∠30° Ω (b) 40∠-45° Ω = = 1.8 
(c) 1 Ω (d) 0 Ω 5
Solution: (b) Hence, the correct option is (d).
Consider the figure given below 7. The h-parameters for a 2- port network are defined
Z 1 = 10∠30° I2  E1   h11 h12   I1 
by   =     . For the 2-port network
I1 I 1 + I2  I 2   h21 h22   E2 
V1 Z2 V2 shown in the figure, the value of h12 is given by
 [2003]
I1 4Ω 2Ω 2Ω I2
V1 = AV2 + BI 2
I1 = CV2 + DI 2 E2
E1 2Ω 4Ω
V2 = Z 2 ( I1 + I 2 ) (1)

I1 (a) 0.125 (b) 0.167


C=
V2 I2 =0 (c) 0.625 (d) 0.25
Solution: (d)
Substituting I2 = 0 in Equation (1), we get,
The h-parameters is
V2 = Z 2 I1
E1
h12 =
V 1 1 E2
⇒ Z2 = 2 = = I1 = 0
I1 I2 =0
C 0.025∠45°
Where, h12 is ratio of E1 to E2 for the input open-
Z 2 = 40∠ − 45°Ω. circuited condition.
Hence, the correct option is (b). I1 4Ω 2Ω 2Ω I2
a b
6. The Z matrix of a 2-port network is given by
0.9 0.2  E1 2Ω 4Ω E2
0.2 0.6  .  The element Y22 of the corresponding
 
Y matrix of the same network is given by [2004] c
(a) 1.2 (b) 0.4
(c) -0.4 (d) 1.8 Using Δ - Y transformation

Chapter 07.indd 67 11/10/2015 5:48:39 PM


1.68 | Electric Circuits and Fields

I1 4 Ω a 0.5 Ω 1Ω b 2 Ω I2 From above figure we get,


1 1 1
y11 = y22 = + = mho
E1 1Ω E2 6 6 3
I1 I2
1
y12 = y21 = − mho
c 6
Hence, the correct option is (b).
E1 = (4 + 0.5) I1 + 1( I1 + I 2 ) = 5.5 I1 + I 2 (1)
9. A 2-port device is defined by the following pair of
E2 = (2 + 1) I 2 + 1( I1 + I 2 ) = I1 + 4 I 2 (2) equations: [2000]
Substituting I1 = 0 in Equations (1) and (2), we get, i1 = 2v1 + v2 and i2 = v1 + v2

E1 = I 2 Its impedance parameters (z11, z12, z21, z22) are


E2 = 4 I 2 given by
(a) (2, 1, 1, 1) (b) (1, -1, -1, 2)
E1 (c) (1, 1, 1, 2) (d) (2, -1, -1, 1)
h12 =
= 0.25.
E2 I = 0 Solution: (b)
1

Hence, the correct option is (d).  2 1


[ y] =  
8. A –2-port network, shown in the figure, is described 1 1
by the following equations:
1  1 −1   1 −1 
[ z ] = [ y ]−1 = =
I1 = Y11 E1 + Y12 E2 2 − 1  −1 2   −1 2 
I 2 = Y21 E1 + Y22 E2 Hence, the correct option is (b).
1 0. For the 2-port network shown in the figure, the
I1 2Ω 2Ω I2 admittance matrix is [1997]

E1 2Ω E2 + I1 10 Ω I2 +
V1 10 Ω 10 Ω V2
− −
The admittance parameters, Y11, Y12, Y21 and Y22 for
the network shown are [2002] Solution:
W W W W Consider the figure given below from which, the
(a) 0.5 , 1 , 2 and 1 respectively.
y-parameters can be found out using inspection.
1 1 1 1
(b)  , − , −  and  respec­tively. 0.1
3 W 6 W 6 W 3W I1 I2
(c) 0.5 , 0.5 , 1.5 and 2 respectively. + Y2 +
V1 0.1 Y 1 Y3 0.1 V2
2 3 3 2
(d) − , − ,  and  respectively. − −
5 7 7 5
Solution: (b)
After converting the T-network to p-network as We know that,
shown in the figure below, we can calculate the ad-
mittance parameters. y11 = Y1 + Y2 ,
y22 = Y2 + Y3 ,

y12 = y21 = −Y2
6Ω 6Ω  0.2 −0.1 
∴ [ y] =  .
 −0.1 0.2 

Chapter 07.indd 68 11/10/2015 5:48:40 PM


Chapter 8
Miscellaneous
If we apply Thevenin’s theorem at the terminals P
One-mark Questions and Q, the resultant circuit will be as shown below.
R1 R2
1. The circuit shown in the figure is [2007] P+
R

+ Ir Load
+
R1 R2 R2 S
+ V V
V− + R1 + R2 − R1 + R2

− I r
R2 Load

r Q

If the short circuit current between the output ter-


rV minals R and S is I, considering op-amp to be ideal,
(a) a voltage source with voltage .
R1 R2
R2
r R2 V = Ir .
(b) a voltage source with voltage V. R1 + R2
R1
V R2
r R2 V I= .
(c) a current source with current ⋅ . r R1 + R2
R1 + R2 r
R2 V Norton’s equivalent circuit across the terminals R,
(d) a current source with current ⋅ . S is shown below.
R1 + R2 r
R
Solution: (d)
Consider the op-amp shown below in figure.
I R0 Load

R1
+ P S
V− +
− The given op-amp circuit acts as a current source
R2 Load with
V R2
I= .
r r R1 + R2

Q Hence, the correct option is (d).

Chapter 08.indd 69 11/10/2015 5:48:15 PM


1.70 | Electric Circuits and Fields

2. A major advantage of active filters is that they can The figure given below shows circuit is s-domain.
be realized without using (1997)
(a) op-amps (b) inductors sL ′
(c) resistors (d) capacitors − V c(0 ) =100
+ s s
Solution: (b) I(s)
1/sC
Active filters can be realized without using bulky
inductors. This is main advantage of this type of
filters. The size of the inductors become bulky Therefore, we have
and occupies more space in order to realize rea-
sonable values of inductance required in filters. 100
Therefore, active filters are realized without using s 100  1 
I (s) = =
inductors.  1  L  s2 + 1 
 sL +   
Hence, the correct option is (b).  sC   LC 

 1 
 
C
Two-marks Questions = 100 
LC 
2 
L 2  1 
s + 
Common Data for Questions 1 and 2:    
  LC  
The L-C circuit shown in the Figure has an inductance
L = 1 mH and a capacitance C = 10 mF. Using inverse Laplace transform we get

L
C 1
i (t ) = L−1[ I ( s )] = 100 sin t
i L LC

t=0 C 100 V
+ 10 × 10−6  1 
= 100 × −3
× sin  t 
1× 10 −
 1× 10 × 100 × 10
3 −6

1. The initial current through the inductor is zero, i (t ) = 10 sin(104 t ) A.
while the initial capacitor voltage is 100 V. The
switch is closed at t = 0. The current i through the Hence, the correct option is (d).
circuit is: [2010]
2. The L-C circuit of Question 7 is used to commu-
(a) 5 cos(5 × 10 t) A
3
(b) 5 sin(10 t) A
4
tate a thyristor, which is initially carrying a current
(c) 10 cos(5 × 103t) A (d) 10 sin(104t) A
of 5 A as shown in the figure below. The values
Solution: (d) and initial conditions of L and C are the same as
Capacitor voltage is charged up to Voltage Vc and in Question 7. The switch is closed at t = 0. If the
initial current through the inductor is zero. forward drop is negligible, the time taken for the
device to turn off is [2010]
Vc(0-) = 100 V.

We know that current through inductor and volt- L


age across cannot change abruptly, therefore, after i
closing the switch we have −
t=0 C 100 V
+
iL(0 ) = iL(0 ) = 0.
+ -

and 5A
100 V 20 Ω
Vc(0 ) = Vc(0 )= 100 V.
+ -

Chapter 08.indd 70 11/10/2015 5:48:15 PM


Chapter 8  Miscellaneous  |  1.71

(a) 52 ms (b) 156 ms Solution: (c)


(c) 312 ms (d) 26 ms Consider the figure given below.
Solution: (a) i(t )
Current i flows through thyristor when the switch
is closed as shown below in the figure. mA
4
i + 2
Vc (t ) A1 A2
− μs
iT 2 μs 5 μs

Area under i - t curve is equal to C charged stored


100 V 20 Ω in the capacitor. Therefore, we get
5A
1
Q = A1 + A2 = (2 × 10−6 ) × (4 × 10−3 )
Load current = I0 = 5 A. 2
1
Net current through thyristor will be + (4 + 2) × 10−3 × (5 − 2) × 10−6
2
it = I0 = i.  6× 3
= 4 +  × 10−9 = 13 nC.
it = 5 - sin 104t.  2 

Let at t = T, circuit get turned off and current iT Hence, the correct option is (c).
becomes zero. 4. The capacitor charged upto 5 ms, as per the cur-
rent profile given in the figure, is connected across
iT = 5 − sin 104 T = 0.
an inductor of 0.6 mH. Then the value of voltage
10 sin 4 T = 5. across the capacitor after 1 ms will approximately
be [2008]
⇒ sin 104 T = 0.5. (a) 18.8 V (b) 23.5 V
(c) -23.5 V (d) -30.6 V
104T = 30° or 0.523 rad.
Solution: (d)
 T ≈ 52 ms. Initial voltage across the capacitor having capaci-
Hence, the correct option is (a). tance C = 0.3 nF will be

Common Data for Questions 3 and 4: Q 13 × 10−9 130


V0 = = = V.
The current i(t) sketched in the figure flows through an C 0.3 × 10−9 3
initially uncharged 0.3 nF capacitor. Consider the figure given below.
P
i(t) mA 6 +
5 V0 C L
4 −
3
2 Q
1 Consider the transform circuit shown below.
0 1 2 3 4 5 6 7 8 9 10 P
t( µs)
1 +
Cs
3. The charge stored in the capacitor at t = 5 ms, +
Ls V 0(s)
will be [2008] 130
3s − −
(a) 8 nC (b) 10 nC
(c) 13 nC (d) 16 nC Q

Chapter 08.indd 71 11/10/2015 5:48:16 PM


1.72 | Electric Circuits and Fields

Voltage across the inductor can be calculated using Solution: (c)


Consider the figure shown below.
130 Ls
VPQ = V0 ( s ) =
3s Ls + 1 RA C
Cs
Z
130 Ls Cs 130s
= = , Vi
3 s LCs + 1 3( s 2 + ω 2 )
2
2RA
130 Vi (s) −
v0 (t ) = cos(ω0 t )u (t ). RA
3 V0(s)
+
For values of inductance and capacitance L = 0.6 mH, RB
C = 0.3 nF, we get Vi
RB
2
1 1 1014
ω02 = = = .
LC 0.6 × 10−3 × 0.3 × 10−9 18
107 For an ideal op-amp, we have
ω0 = r/s.
18 Vi ( s ) V (s)
V0 ( s ) = − Z (s) + i .
2 RA 2
For time t = 1 ms, we get

107 10 V0 ( s ) 1  Z ( s ) 
ω0 t = × 1× 10−6 = rad = 2.357 rad = 1 − ,
Vi ( s ) 2  RA 
18 18
130  10  130 where impedance
v0 (1 µs) = cos  = cos(2.357)
3  18  3 1
RA
130 Cs RA
=− × 0.707 = −30.6 V. Z (s) = = .
3 1 1 + RACs
RA +
Cs
Hence, the correct option is (d).
Common Data for Questions 5 and 6: 1 1  1 RACs
H (s) = 1 − = .
A general filter circuit is shown in the figure. 2  1 + RACs  2 1 + RACs
1
R2 H ( jw) = 0,  at  w = 0 = ,  at w = ∞.
2
Hence, the filter circuit is a high pass filter because
C 1
R1 H ( jw) increases from 0 at w = 0 to at w = ∞.
V1 − 2
Hence, the correct option is (c).
V0
6. The output of the filter in Question 5 is given to the
+
R3 circuit shown in figure. [2008]
R4
RA /2

V in C V0
5. If R1 = R2 = RA and R3 = R4 = RB, the circuit acts as a
 [2008]
(a) all pass filter (b) band pass filter The gain vs frequency characteristic of the output
(c) high pass filter (d) low pass filter (V0) will be

Chapter 08.indd 72 11/10/2015 5:48:17 PM


Chapter 8  Miscellaneous  |  1.73

Gain We can conclude that


(a) 1 2
ω1 = , ω2 = .
w RA C A RA C A
0
Hence, the correct option is (d).
Gain
7. The state equation for the current I1 in the network
(b) shown below in terms of the voltage Vx and the
w
independent source V, is given by [2007]
0
3Ω 0.2 H 5Ω
Gain + Vx −
V ± I1 0.5 H − 0.2V x
(c) +

w
0
dI 5
(a) 1 = −1.4 Vx − 3.75 I1 + V.
Gain dt 4
(d) dI1 5
(b) = 1.4 Vx − 3.75 I1 − V.
w
dt 4
0 dI1 5
(c) = −1.4 Vx + 3.75 I1 + V.
Solution: (d) dt 4
Consider the figure shown below. dI1 5
(d) = −1.4 Vx + 3.75 I1 − V.
RA /2 dt 4
+ +
Solution: (a)
V in(s) C V 2(s) Consider the figure given below.
R 3Ω S 0.2 H P 5Ω M
− −
+ Vx − I1 I2
+ Ix
From the above figure we conclude that transfor- −
V 0.5 H + 0.2V x
mation factor can be expressed as −
1 Q N
V2 1
( s ) = Cs = . dEI1
Vin RA 1 1 + 0.5 RACs H V=
Voltage across 0.5= 0.5 . (1)
+ PQ
dt
2 Cs
Using mesh equation for RPQR we get
V2 ( s )
∴ = Overall TF
Vi ( s ) dI1
VRS = V − Vx − 0.5 . (2)
1 RACs dt
= . VRS 1  dI 
2 (1 + 0.5 RACs )(1 + RACs ) Ix = = V − Vx − 0.5 1  . (3)

3 3 dt  
Figure given below showns the asymptotic bode
plot and the actual curve for the above transforma-
tion factor. Using nodal equation at P we get
I2 = (Ix - I1).(4)
Using mesh equation for PMNQP we get
20 db/dec −20 db/dec
dI1
∴ VPQ = 0.5 = 5 I 2 − 0.2Vx .
Gain dt
Using (4)
0 w1 w2 w
= 5( I x − I1 ) − 0.2 Vx ,

Chapter 08.indd 73 11/10/2015 5:48:18 PM


1.74 | Electric Circuits and Fields

dI1  1  dI   (a) 1, 4 (b) 5, 1


or 0.5 = V − Vx − 0.5 1  − I1  5 − 0.2Vx .
dt  3 
(c) 5, 2 (d) 5, 4
dt  
Solution: (d)
Using (3) We know that capacitor behaves as open circuit
5 5 2.5 dI1 and inductor behaves as short circuit in steady
= V − Vx − − 5 I1 − 0.2Vx . state condition.
3 3 3 dt
R1 R2 Iv
4 dI 5.6 5
or  1 = − Vx − 5 I1 + V.
3 dt 3 3 SC OC I1
dI 5 OC + V

or  1 = −1.4Vx − 3.75 I1 + V. I
dt 4 SC R3

Hence, the correct option is (a).


8. In the circuit shown in the figure, the current Voltage across R3 = V = 5 V.
source I = 1 A, the voltage source V = 5 V, R1 = R2 V 5
= R3 = 1 Ω, L1 = L2 = L3 = 1 H, C1 = C2 = 1 F. The Current through R= 3 I=
1 = = 5 A.
R3 1
currents (in A) through R3 and the voltage source V
respectively, will be [2006] Using KCL we get,
R1 R2 -I + I1 - Iv = 0.
Current though voltage source
L1 C1 L3
+ V = IV = I1 − I
C2 −
= 5 − 1 = 4 A.
I L2 R3
Hence, the correct option is (d).

Chapter 08.indd 74 11/10/2015 5:48:19 PM


Chapter 8  Miscellaneous  |  1.75

Using the nodal equation we get


Five-marks Questions
dvC
i = iL + C
1. For the network shown in figure, obtain the state dt
equations in terms of capacitor voltage, vc and dv 1 1
inductor current iL. [1999] or  C = 0vC − iL + i. (1)
dt C C
Using the mesh equation we get
L iL diL
+
i C VC L + RiL = vC
− dt
R
diL 1 R
 or  = vC − iL + 0 i. (2)
dt L L
Solution:
Vector-matrix form for the state Equations (1) and
Consider the figure given below
(2) are shown below.
+ dvc  dVC   1
L diL C
 dt  0 − 1
C  vC    (3)
iL L dt
+
− dt  =    + C i.
vc  
C
 diL   1 R i
+
− −   L   0 
 dt   L L
iL R Ri L

x = Ax + bu. (4)
or     

Chapter 08.indd 75 11/10/2015 5:48:20 PM


Chapter 9
Fields
Now the relation between voltage V and capaci-
One-mark Questions tance C is
Q = CV
1. A parallel plate capacitor consisting two dielectric
materials is shown in the figure. The middle di- Q
C=
electric slab is placed symmetrically with respect V
to the plates. [2014-S2] 1
C∝ .
10 V V

The relation for capacitance of a parallel plate


e1 e1 ­capacitor is
Kε 0 A
C=
e2 d
εA
d C=
2 d
d
C ∝ ε.
If the potential difference between one of the Voltage is inversely proportional to C and C is pro-
plates and the nearest surface of dielectric inter- portional to ε, therefore
face is 2 V, then the ratio ε1 : ε2 is [2014-S2]
(a) 1 : 4 (b) 2 : 3 1
V∝
(c) 3 : 2 (d) 4 : 1 ε
Solution: (c) V1 ε 2
=
V2 ε1
ε1 V2
=
ε 2 V1
2V 6V 2V ε1 6
=
ε2 4
ε1 3
= .
ε2 2

Voltage ratio is ∴  ε1 : ε2 = 3 : 2
V1 : V2 = 4 : 6 Hence, the correct option is (c).

Chapter 09_1 marks.indd 76 11/10/2015 5:50:11 PM


Chapter 9  Fields  |  1.77

2. The flux density at a point in space is given by ∂


B = 4xax + 2kyay + 8az Wb/m2. The value of constant Vx = −[ x(− sin( xy ) y + cos( xy ))] (1)
∂x
k must be equal to [2013]
(a) -2 (b) - 0.5 ∂
Vy = [ y (− sin( xy ) x + cos( xy ))] (2)
(c) + 0.5 (d) + 2 ∂y
Solution: (a) ∂
Flux density at a point in space is given by Vz = cos( z 2 )2 z. (3)
∂z
B = 4xax + 2kyay + 8az Wb/m2.  
Now the divergence of flux density in free space div (V ) = ∇ ⋅V
will be ∂ ∂ ∂
 = Vx + Vy + Vz .
∇ ⋅ B = 0.   ∂x ∂x ∂x
∂ ∂ ∂
⇒ (4 x) + (2ky ) + (8) = 0 From Equations (1), (2), (3) and (4), we get
∂x ∂y ∂z
⇒ 4 + 2k = 0  ∂ ∂ ∂
⇒ k = −2. div (V ) = Vx + Vy + Vz
∂x ∂x ∂x
Hence, the correct option is (a). = −[ x(− sin( xy ) y + cos( xy ))]
3. Divergence of the 3-dimensional radial vector field +[ y (− sin( xy ) x + cos( xy ))] + cos( z 2 )2 z

F is [2010]
1 = x sin( xy ) y − cos( xy ) − y sin( xy ) x
(a) 3 (b)
r + cos( xy ) + cos( z 2 )2 z = 2 z cos( z 2 ).
(c) i + j + k (d) 3(i + j + k )
Hence, the correct option is (a).
Solution: (a)
5. The concept of an electrically short, medium, and
Consider the radial vector field given below
 
 
 
 long line is primarily based on the [2006]
r = x ax + y a y + z az . (a) nominal voltage of the line.
The divergence of the radial vector field will be (b) physical length of the line.
(c) wavelength of the line.
  ∂ ∂ ∂
div(r ) = (∇ ⋅ r ) = x + y+ z (d) power transmitted over the line.
∂x ∂y ∂z
Solution: (c)
= 1 + 1 + 1 = 3. The concept of an electrically short, medium, and
Hence, the correct option is (a). long line based on the wavelength the signal trans-
4. Divergence of the vector field mitted over the line but is based not on physical
length of the line.
 V(x, y, z) = -(xcos y + y)i + (ycos y)j + (sin z2 + x2 +
y2)k is [2007] Hence, the correct option is (c).
(a) 2zcos z2. (b) sin xy + 2z cos z2. 6. Which of the following statements holds for diver-
(c) xsin xy - cos z (d) None of these gence of electric and magnetic flux densities?
Solution: (a) [2006]
Vector field: (a) Both are zero.
 (b) These are zero for static densities but non-zero
V = V iˆ + V ˆj + V kˆ
x y z for time varying densities.
where the components are (c) It is zero for the electric flux density.
(d) It is zero for the magnetic flux density.
Vx = −[ x cos( xy ) + y ],
Solution: (d)
Vy = y cos( xy ), According to Maxwell’s equation involving diver-
Vz = sin( z 2 ) + x 2 + y 2. gence operation, we have

Chapter 09_1 marks.indd 77 11/10/2015 5:50:14 PM


1.78 | Electric Circuits and Fields

  (a) 1298 pF (b) 944 pF


∇ ⋅ D = div D = ρV
  (c) 354 pF (d) 257 pF
∇ ⋅ B = div B = 0.
Solution: (d)
Where, Width d1 = 6 mm,
 Dielectric constant εr1 = 4
D is the electric flux density in C/m2
ρv is the volume charge density in C/m3 Width d2 = 8 mm,
 Dielectric constant εr2 = 2
B is the magnetic flux density in T or Wb/m2
Hence, the correct option is (d). Area of the plates A = 400 × 400 mm2
7. The inductance of a long solenoid of length 1000 mm Series combination of C1 and C2 is used to find the
wound uniformly with 3000 turns on a cylindrical resultant capacitance
paper tube of 60 mm diameter is [2004]
1 1  d1 d 2 
(a) 3.2 mH (b) 3.2 mH =  + 
(c) 32.0 mH (d) 32 H C ε 0A  ε r1 ε r 2 
Solution: (c) 1 36 π × 108  6 8 
= 4 + 2
We know that inductance, L can be calculated using C 16  
L = m0n2p r2l 1 36 π × 108
= [1.5 + 4]
C 16
where,
1 36 π × 108
length of the solenoid = l = 1000 mm = 1 m = × 5.5
C 16
total number of turns N = 3000
16 160
number of turns per metre, n = 3000 C= F= nF
36 π × 108 × 5.5 36 π × 5.5
diameter of circular cross-section = d = 60 mm,
= 0.257 nF = 257 pF.
radius = r = 30 mm = 30 × 10-3 m
area of cross section = A = π r2 Hence, the correct option is (d).
Substituting the values we get 9. Two conductors are carrying forward and return
current of +I and -I as 
shown
 in the figure. The
L = µ 0 n 2π r 2 l magnetic field intensity H at point P is [2003]

= 4 π × 10−7 × (3000) 2 π (30) 2 × 10−6 × 1 H z +I −I


2 −5 ⊕ d d
= 4 × 9 × 9π × 10 H x
−5
= 3197 × 10 H = 31.97 mH ≈ 32 mH. P

y
Hence, the correct option is (c).
8. A parallel plate capacitor is shown in the figure. It
I  I 
is made of two square metal plates of 400 mm side. (a) a y (b) ax
The 14 mm space between the plates is filled with ≠π d ≠πd
two layers of dielectrics of εr = 4, 6 mm thick and  I 
εr = 2, 8 mm thick. Neglecting fringing of fields at (c) I a y (d) ax
2≠π d ≠π d
the edges the capacitance is [2004]
Solution: (a)
e0 = 8.85 × 10−12 F/m Consider the figure given below
+I −I
er = 4, d = 6 mm
A⊕ B
er = 2, d = 8 mm d d

Chapter 09_1 marks.indd 78 11/10/2015 5:50:16 PM


Chapter 9  Fields  |  1.79

From the above figure we conclude that current I in 11. An electron with velocity u is placed in an electric
A is in the +z direction whereas current I in B is in field, E and magnetic field, B. The force experi-
the -z direction. enced by the electron is given by [2000]

Magnetic field intensity, H1 at P due to A is given by (a) -eE (b) -eu × B
(c) -e(u × E + B) (d) -e(E + u × B)
 I Solution: (d)
aˆ y . H1 =
2π d The force on a charge, Q (Coulombs) in an Electric
 field is given by
Magnetic field intensity, H 2 at point P due to B is  
given by FE = Q E ,
 1 where Q is charge and E is electric field.
H2 = aˆ y .
2π d We know that force acting on a charge particle
moving under the influence of uniform magnetic
Total magnetic field at point is given by relation field is given by relation
  
  FM = Q(v × B)
H
= H1 + H 2
I 1 I The force on a moving charge due to combined
= aˆ y + aˆ y = aˆ . electric and magnetic fields is given by
2π d 2π d πd y
     
F = FE + FM = Q E + Q(v × B)
Hence, the correct option is (a).   
 = Q[ E + (v + B )].
1 0. Given a vector field F , the divergence theorem
states that [2002] For an electron we substitute Q = e, substituting
   the value of charge we get the force on electron as
(a) ∫ F ⋅ d s = ∫ ∇ ⋅ F dV    
s v F = −e[ E + (u + B)]
  

(b) F ⋅ d s = ∫ F dV
∇ × Hence, the correct option is (d).
s v
   1 2. An electromagnetic field is radiated from [1999]
∫ F × d s = ∫ ∇ ⋅ F dV
(c)
s v
(a) a stationary point charge.
   (b) a capacitor with a DC voltage.
∫ F × d s = ∫ ∇ ⋅ F dV
(d)
s v
(c) a conductor carrying a DC current.
(d) an oscillating dipole.
Solution: (a) Solution: (d)
According to divergence theorem which is used to We know that an oscillating dipole produces electro-
converts surface integral into a volume integral, we get magnetic waves, therefore an electromagnetic field
   is radiated form an oscillating dipole. Due to this
∫ F ⋅ d s = ∫ div( F ) dv = ∫ ∇ ⋅ F dv.
s s v
property, an oscillating dipole is used as an antenna,
when alternating current is fed across the dipole.
Now according to Gauss’s law we have Hence, the correct option is (d).
 
∫ D ⋅ d s = Q = ∫ ρv dv.
s v
13. Two parallel wires separated by a distance d are
carrying a current I in the same direction. The
magnetic field along a line running parallel to
Using divergence theorem, we get
these wires and midway between them [1999]
  
∫ D ⋅ d s = Q = ∫ ∇ ⋅ D dv
s v


(a) depends upon I.
(b) is zero.
 (c) depends upon d.
∴ ∇ ⋅ D = ρv
(d) depends upon the permeability of medium be-
Hence, the correct option is (a). tween the wires.

Chapter 09_1 marks.indd 79 11/10/2015 5:50:20 PM


1.80 | Electric Circuits and Fields

Solution: (b) Transmission line P


Consider the figure given below Load
Z0 = 300 Ω
+I +I Z0 = Source impedance 300 Ω
A⊕ ⊕B 300 Ω
d d

From the above figure we conclude that current I in


A is in the +z direction whereas current I in B is in (a) +1 (b) -1
the +z direction. (c) 0 (d) 0.5

Magnetic field intensity, H1 at P due to A is given by Solution: (c)
If ZL is the load impedance and Z0 is the character-
 I istics impedance of the line, then reflection coeffi­
H1 = aˆ y
2 πd cient, r is given by
 Z L − Z0
Magnetic field intensity, H 2 at point P due to B is r= .
given by Z L + Z0
 1 We know that ZL = Z0 = 3000 Ω, since, the line is
H2 = − aˆ y
2 πd matched to the load, therefore
300 − 300
Total magnetic field at point is given by relation r=
300 + 300
  I 1
H = H1 + H 2 = aˆ y − aˆ y = 0aˆ y = 0. ∴  r = 0.
2πd 2πd
Hence, the correct option is (c).
Hence, the correct option is (b). 1 6. In a uniform electric field, field lines and equipo-
1 4. The laws of electromagnetic induction (Faraday’s tentials [1997]
and Lenz’s law) are summarized in the following (a) are parallel to one another.
equation  [1998] (b) interest at 45°.
di (c) intersect at 30°.
(a) e = iR (b) e=L (d) are orthogonal.
dt
dφ Solution: (d)
(c) e=− (d) None of these
dt At equipotential surfaces field lines are always nor-
Solution: (c) mal or orthogonal because field lines are always di-
According to Faraday’s law the induced emf is rected from higher potential to lower potential point
diretcly proportional to the rate of change of flux Hence, the correct option is (d).
dφ 17. The energy stored in the magnetic field of solenoid
e∝ .
dt 30 cm long and 3 cm diameter wound with 1000
turns of wire carrying a current of 10 A is [1996]
Now according to Lenz’s law, the direction of (a) 0.015 J (b) 0.15 J
induced emf is such that it always opposes the (c) 0.5 J (d) 1.15 J
change in flux, therefore
Solution: (b)
dψ Length of solenoid, l = 30 cm.
e=− .
dt Diameter of solenoid, d = 3 cm.
Hence, the correct option is (c). Number of turns, N = 1000.
1 5. The reflection coefficient for the transmission line Current, I = 10 A.
shown in the figure at P is [1998] Now using the relation

Chapter 09_1 marks.indd 80 11/10/2015 5:50:22 PM


Chapter 9  Fields  |  1.81

2 The transverse components Ey and Hz are related by


3 
24π × 10−7 × (1000) 2 × π ×  × 10−2  Ey
µ0 N A  2  = η,
L= =
l 30 × 10−2 Hz
= 2.96 mH.    
=
where, E E=
y ay , H H z az
The energy stored in the solenoid will be
and
1 2 1
E= LI = × 2.96 × 10−3 × 102 µ0
2 2 η = η0 = = 120 π Ω.
ε0
= 0.148 J 0.15 J.
If we consider option (a)
Hence, the correct option is (b).  
1 8. If v, w, q stand for voltage, energy and charge then = E (= x, t ) E y a y
v can be expressed as [1996] and
 
dq dw H (=
= x, t ) H z a z
(a) v= (b) v=
dw dq
dw dq where, E y = E0 cos( kx − ωt )
(c) dv = (d) dv =
dq dw and H z = H 0 cos( kx − ωt )
Solution: (c) Ey E0
and = = η0 , which is real
We know that voltage, charge and energy are related as H z H0
dw Hence, the correct option is (a).
dv = ,
dq 20. Static magnetic fields induce currents in closed
conducting loops. (True/False) [1994]
where v is the voltage, q is the charge and w is the
energy. Solution:
According to Faraday’s law, if φ is the time varying
Hence, the correct option is (c).
magnetic flux linking the surface enclosed by the
19. A monochromic plane electromagnetic wave trav- loop, voltage induced in a closed conducting loop,
els in vacuum in the x-direction (x, y, z system of is given by relation
coordinates). The electric and magnetic fields can

be expressed as [1994] V= .
dt
(a) =E ( x, t ) E0 cos(kx − ωt )aˆ y
According to Lenz’s law, the induced voltage sets
H ( x, t ) H 0 cos(kx − ωt )aˆ z
=
up a current in such a direction which opposes the
E ( x, t ) E0 cos(kx − ωt )aˆ y
=
(b) original flux .
 π In a static magnetic field, where ϕ is not changing
H= ( x, t ) H 0 cos  kx − ωt −  aˆ z
 2 with time,
E ( x, t ) E0 cos(kx − ωt )aˆ y
=
(c) dφ
= 0.
dt
− H 0 cos(kx − ωt )aˆ z
H ( x, t ) =
Therefore V = 0 and current through the loop = 0.
E ( x, t ) E0 cos(kx − ωt )aˆ y
=
(d)
Hence, the correct option is false.
 π
− H 0 cos  kx − ωt −  aˆ z
H ( x, t ) = 21. In electrostatic field ∇ × E = 0 (True/False)
 2
[1994]
Solution: (a)
Solution:
Ex and Hx components along the x-direction are
 We know that, curl of electric field vector can be
zero for a monochromatic (uniform) place EM
expressed as
wave travelling in vacuum (ε = ε0, μ = μ0, σ = 0) in  
the x-direction, curl ( E ) = ∇ × E.

Chapter 09_1 marks.indd 81 11/10/2015 5:50:27 PM


1.82 | Electric Circuits and Fields

The potential difference around a closed path, C in In rectangular (Cartesian) co-ordinates: x, y, z


electrostatic field is given by relation
 
  ∂  ∂  ∂  
E = −∇φ = −  φ ax + φ a y + φ az  ,
C ∫ E ⋅ dL = 0 ∂ x ∂y ∂z 
Using Stoke’s theorem, we get = Ex aˆ x + E y aˆ y + Ez aˆ z .
     ∂ ∂ 
C ∫ E ⋅ dL = ∫ (∇ × E ) ⋅ ds − φ=
Ex = − 2 x y  =−2 y .
C S ∂x ∂x

∇ × E = 0. ∂ ∂ 
− φ=
Ey = − 2 x y 
∂y ∂y
Hence, the correct option is true.
1 −1 x
2 2. The line integral of the vector potential A around −2 x y 2 =
= − .
the boundary of a surface S represents [1993] 2 y
(a) flux through the surface S ∂
(b) flux density in the surface S Ez
= = φ 0.
∂z
(c) magnetic density
(d) current density At the point x = 1 m, y = 1 m the components of
Solution: (a) electric field are
The magnetic flux density is given by relation
Ex = 2, E y = 1, Ez = 0
   
B = curl(A) = ∇ × A,   
∴ E = − [ 2 a x + 1 a x + 0 a x ].

where  A  is the vector magnetic potential Magnitude of electric field intensity
We know that the magnetic flux, φ passing through 
E = E = 22 + 12 = 5 V.
a surface S, is given by 
   Direction of  E can be calculated as given below
φ = ∫∫ B ⋅ ds   E

    2  1  
E = aE = = −  ax + ay  .
= ∫ B ⋅ ds E  5 5 
s
   24. Which of the following equations represents the
= ∫ (∇ × A) ⋅ ds.
s
Gauss’s law in a homogenous isotropic medium?
 [1992]
If C is the closed path encircling the surface S, then
using Stoke’s theorem, a surface integral is converted
(a) ∫∫ D ⋅ d s = ∫∫∫ ρ d v (b) ∇× H = D
into closed surface integral as shown below ρ
(c) ∇ ⋅ J + ρ = 0 (d) ∇⋅E =
     ε
φ = ∫ (∇ × A) ⋅ ds = C ∫ A ⋅ dL. Solution: (a)
s C
According to Gauss’s law, the electric flux pass-
Hence, the correct option is (a). ing through any closed surface is equal to the total
23. An electrostatic potential is given by φ = 2x y V charge in the volume enclosed by that surface.
in the rectangular co-ordinate system. The magnitude     
ψ = c ∫ D ⋅ ds = c ∫ c ∫ D ⋅ ds,
of the electric field at x = 1 m, y = 1 m is s s
V/m. [1992]
Solution:
Q= ∫ ρ dv,
Vol
Electrostatic potential, φ = 2x y V.
 where ρ = volume charge density,
Relation between electric field intensity, E and flux ϕ
 

is given below. ∴ c ∫ c ∫ D ds = ∫∫∫ ρ dv.

E = −grad(φ ) = −∇φ . Hence, the correct option is (a).

Chapter 09_1 marks.indd 82 11/10/2015 5:50:32 PM


Chapter 9  Fields  |  1.83

3 4 ˆ
Two-marks Questions (a) kˆ Wb/m 2 (b) i A/m
2π 3π
3
1. The following four vector fields are given in Cartesian (c) kˆ A/m (d) 0 A/m

co-ordinate system. The vector field which does not
satisfy the property of magnetic flux density is Solution: (c)
[2014-S1]
2A
2 2 2 y
y ax + z a y + x az
(a)
z-direction
2
z 2 ax + x 2 a y + y 2 az
(b) I=4A
x 2 ax + y 2 a y + z 2 az
(c) P
1 (2, 1, 0)
y 2 z 2 ax + x 2 z 2 a y + x 2 y 2 az
(d)
Solution: (c) x
As we know that divergence of magnetic field is
zero, therefore ∇ ⋅ B = 0 From the above figure we conclude that the giv-
en currents are along the x-axis, and the y-axis.
For B = x2ax + y2ay + z2az the above relation becomes The point where magnetic field intensity is to be
∇ ⋅ ( x 2 ax + y 2 a y + z 2 az ) found is (2, 1, 0), this point lie on the xy-plane. In
­order to find the magnetic field intensity we draw
⇒ 2x + 2 y + 2z ≠ 0 a 2-­dimensional figure with the plane of the paper
From the given options, option (c) does not satis- as the xy-plane.
fies the above relation . 4
Magnetic field intensity H ( P) due to 4axA = aZ
Hence, the correct option is (c). 2π l
 A/m. (direction decided by right hand rule)
2. The magnitude of magnetic flux density ( B) at a
point having normal distance d metres from an
infinitely extended wire carrying current of IA is Magnetic field intensity H ( P) due to 2ayA
µ0 I 2
 (in SI units). An infinitely extended wire is = (−aZ ) A/m (right hand rule).
2π d 2π × 2
laid along the x-axis and is carrying current of 4 A Total magnetic field intensity H  at P will be
in the +ve x-direction. Another infinitely extended
wire is laid along the y-axis and is carrying 2 A 2 1 3
= az − az = az A/m.
current in the +ve y-direction. μ0 is permeability of π 2π 2π
free space. Assume  iˆ, ˆj , kˆ  to be unit vectors along
x-, y- and z-axes respectively. [2014-S2] We know that  iˆ, ˆj , kˆ  as the unit vector along x-, y-,
and z-axis respectively. But we have used symbols
y ax, ayand az such that  = ax iˆ=, a y ˆj  and az = kˆ.
Hence, the correct option is (c).
IA µ0I 2A (2, 1, 0)
B= 1 3. A dielectric slab with 500 mm × 500 mm cross-
d 2pd
⊗ section is 0.4 m long. The slab is subjected to a
4A uniform electric field of E = 6ax + 8ay kV/mm.
x
1 2 The relative permittivity of the dielectric material
is equal to 2. The value of the constant ε0 is 8.85
   z
× 10 -12 F/m. The energy stored in the dielectric in
Joules is [2013]
Assume right handed  co-ordinate system, magnet- (a) 8.85 × 10-11 (b) 8.85 × 10-5
ic field intensity, H at co-ordinate (2, 1, 0) will be (c) 88.5 (d) 885

Chapter 09_Part 2.indd 83 11/10/2015 7:13:05 PM


1.84 | Electric Circuits and Fields

Solution: (c) ∴  For a distance of separation, d metre;


Permittivity of free space ε0 = 8.85 × 10-12 F/m Maximum charge in the capacitor, Qm = CVm,
Energy density can be calculated as εA
where C =
d
1 1
WE = ε E 2 = ε 0 × ε r × E 2 . εA
2 2 Qm = Vm ,
d
Substituting εr = 2 εA
Qm = 5 × 106 d = 5 × 106 ε A
and d
= 5 × 106 × 2.26 × 8.85 × 10−12 × 8 × 10−2 C = 8 µC.

 
 
  103
E = 6 ax + 8 a y kV/mm = (6 ax + 8 a y ) × −3 V/m Hence, the correct option is (c).
10

 

= (6 ax + 8 a y ) × 106 V/m. 5. F ( x, y ) = ( x 2 + xy )aˆ x + ( y 2 + xy )aˆ y. Its line inte-
gral over the straight line from (x, y) = (0, 2) to
| E | = 107 V/m. (x, y) = (2, 0) evaluates to [2009]
(a) -8 (b) 4
We get (c) 8 (d) 0
Solution: (d)
1
WE = × 2 × 8.85 × 10−12 × (107 ) 2 Consider the figure given below
2
Y
= 8.85 × 102 = 885 J/m3 .

Energy stored = Energy density × volume (0, 2) A

Volume of the dielectric slab = Cross-section × length


= 500 × 10-3 × 500 × 10-3 × 0.4 = 0.1 m3
Energy stored = 885 × 0.1 J = 88.5 J B
X
Hence, the correct option is (c). 0 (2, 0)
 
 

4. A capacitor is made with a polymeric dielectric We know that  F ( x, y ) = ( x 2 + xy )ax + ( y 2 + xy )a y
having εr of 2.26 and a dielectric breakdown
strength of 50 kV/cm. The permittivity of free Line integral is given by expression:
space is 8.85 pF/m. If the rectangular plates of the B  
capacitor have a width of 20 cm and a length of I = ∫ F ( x, y ) ⋅ dL.
A
40 cm, then the maximum electric charge in the
capacitor is [2011] Along the straight line for A(0, 2) to B(2, 0), slope
(a) 2 μC (b) 4 μC −2
of the line AB = = −1
(c) 8 μC (d) 10 μC 2

Solution: (c) Equation of the line, AB will be y = -1(x - 2) = -x + 2


 
 

Breakdown strength of the dielectric = 50 kV/cm for F ( x, y ) = ( x 2 + xy )ax + ( y 2 + xy )a y
= 5 × 106 V/m  
 
dL = dx ax + dy a y .
Maximum voltage that can be applied, Vm = 5 ×
106d V  
F ( x, y ) ⋅ dL = Fx dx + Fy dy
Area of the rectangular plates,
= ( x 2 + xy )dx + ( y 2 + xy ) dy
2 −2 2
A = 20 × 40 cm = 8 × 10 m . = [ x 2 + x(− x + 2)]dx + [ y 2 + y (− y + 2)] dy
−12
ε = ε 0ε r = 2.26 × 8.85 × 10 F/m. = 2 x dx + 2 y dy.

Chapter 09_Part 2.indd 84 11/10/2015 7:13:09 PM


Chapter 9  Fields  |  1.85

B B Note:
 Here, it is assumed that flux is the flux of
∴ I = ∫ 2 x dx + ∫ 2 y dy. D. We can,
 (and do) define flux for any vector, for
A A
example E .
Hence, the correct option is (d). Hence, the correct option is (c).
6. A coil of 300 turns is wound on a non-magnetic 8. A capacitor consists of two metal plates each 500
core having a mean circumference of 300 mm and a × 500 mm2 and spaced 6 mm apart. The space
cross-sectional area of 300 mm2. The inductance of between the metal plates is filled with a glass plate
the coil corresponding to a magnetizing current of of 4 mm thickness and a layer of paper of 2 mm
3 A will be (Given that μ0 = 4π × 10-7 H/m) [2008] thickness. The relative permittivity of the glass and
(a) 0.31 A (b) 13.5 μC paper are 8 and 2 respectively. Neglecting the fring-
(c) 15.0 μC (d) 113.04 mH ing effect the capacitance will be [2008]
Solution: (d) −12
(Given that ε 0 = 8.85 × 10 F/m)
Length of flux path = 0.3 m (a) 983 33 pF (b) 1475 pF
Area of cross-section of path = 300 × 10-6 m2 (c) 6637.5 pF (d) 9956.25 pF
−3 Solution: (b)
Reference of flux path = εA
µ 0 × 300 × 10−6 Capacitance C of a parallel plate is C =
d
3000 1010 Area of metal plates A = 500 × 500 mm2
= = AT/Wb.
3µ o 4π Thickness of the glass dielectric plated (d1) = 4 mm
Inductance of coil does not depend on current when ε r1 = 8.
core is non magnetic εε A
∴ C1 = 0 r1 .
3002 × 4 π 36 π d1
= 6 H = 36 π µH = 113 µH.
1010 10 For the paper dielectric layer, thickness d2 = 2 mm
Hence, the correct option is (d). ε r 2 = 2.
7. Two point charges Q1 = 10 μC and Q2 = 20 μC εε A
∴ C2 = 0 r 2 .
are placed at co-ordinates (1, 1, 0) and (-1, -1, 0) d2
through a plane Z = 20 will be [2008]
Since, the dielectric interface is parallel to the con-
(a) 7.5 μC (b) 13.5 μC ducting plates, the overall capacitance C is given
(c) 15.5 μC (d) 22.5 μC by series combination of C1 and C2
Solution: (c)
1 1 1 1  d1 d 2 
Since, Z = 0 for both the charges are both on xy- = + =  + 
C C1 C2 ε 0 A  ε r1 ε r 2 
plane.
The plane Z = 20 is parallel to the xy-plane, at a 36 π × 109 × 10−3  4 2  36 π × 108
= + = × 1.5.
distance 20 m above the plane. 500 × 500 × 10−6  8 2  25
Total electric flux produced by a charge q = qc 25 250
∴ C= F= nF
1 36 π × 108 × 1.5 36 π 1.5
If the charge is on xy-plane, by symmetry,   the = 1.474 nF = 1475 pF
2
electrical flux would radiate above the xy-plane Hence, the correct option is (b).
1
and   below the xy-plane. Flux above xy-plane Common Data for Questions 9 and 10:
2
must pass through the Z = 20 plane. An inductor designed with 400 turns coil wound
 ∴  20 μC Causes a flux 10 μC causes a flux 5 on an iron core of 16 cm2 cross-sectional area and
μC through the plane described. Total electric flux with a cut of an air gap length of 1 mm. The coil
through the plane Z = 20 is 10 + 5 = 15 μC. is connected to a 230 V, 50 Hz AC supply. Neglect

Chapter 09_Part 2.indd 85 11/10/2015 7:13:13 PM


1.86 | Electric Circuits and Fields

coil resistance, core loss, iron reluctance and leak- Average force can be calculated as
age inductance. (μ0 = 4π × 10-7 H/m)
1 I
9. The current in the inductor is [2007] F (t )ave = 4002 µ0 (rms value of current)
2 10−6
(a) 18.08 A (b) 9.04 A
(c) 4.56 A (d) 2.28 A 16.8 × 104 × 4 π × 10−7 × 256 × 10−4
= 2.276
Solution: (d) 10−6
Reluctance of flux path (reluctance of iron path is 1 16 × 104
= × −6
4π × 10−7 × 16 × 10−4 × 2.2762
neglected) 2 10
10−3 10−3 = 3.2π × 16 × 2.2762 = 833.2 N.
Rel = =
µ0 16 × 10−4 4π 10−7 × 16 × 10−4 Hence, the correct option is (a).
108 1 1. A solid sphere made of insulating material has a
= AT/Wb. radius R and has a total charge Q distributed uni-
64 π
formly in its volume. What is the magnitude of the
For a current I A in coil; mmf = 400I AT; electric field intensity, E, at distance r(0 < r < R)
400 I × 64 π inside the sphere? [2007]
Flux = Wb
108 1 Qr 3 Qr
(a) (b)
Inductance of coil when no leakage, no fringing, 4πε 0 R 3 4πε 0 R 3
no resistance, and no core loss takes pace is Q 1 QR
(c) 2 (d)
4πε 0 r 4πε 0 r 3
4002 × 64π 1024 π
ε= 8
H− = 0.3217 H Solution: (a)
10 104
Relative permittivity of solid sphere of insulating
Reactance of the coil will be material is not given as per problem. But Assume
= 100π (0.3217) = 101.06 Ω it to be 1. (This is equivalent to assuming that the
230 material is air or free space).
=
Coil current will be  = 2.276 A Consider a point P inside the sphere, OP = r; 0 ≤ r ≤ R.
101.06
Total charge inside the sphere shown by dotted line
Hence, the correct option is (d).
1 0. The average force on the core to reduce the air gap 4  Q r 3Q
=  π r3  = 3 .
will be [2007] 3   4 π R3  R
 
(a) 832.29 N (b) 1666.22 N 3 
(c) 3332.47 N (d) 6664.84 N Volume of uniform volume charge density sphere
Solution: (a) shown inside sphere of radius r dotted.
Force By Gauss’s law and symmetry, we can prove that it
1 d (Re l ) r 3Q
Fe = − φ 2 . is only this charge  3  which produces electrical
2 dx R
Where we have field at P, and that for the point P, it is as though
r 3Q
x d (re l ) 1 this charge  3  is concentrated at the center O.
Reluctance = ⋅ = R
µ0 A dx µ0 A
Hence, electrical field at P
Ignoring the sign (which gives only the direction)
r 3Q 1
2 = ⋅ ar N/C.
1 1 1  400i µ0 A  1 R 4πε 0 r 2
3
Fe = φ 2 =   µ A N.
2 µ0 A 2  x  0 Qr
= N/C In magnitude.
1 400 i
2 2 4 π ε 0 R3
F (t ) = µ0 A ⋅ N ; x = 10−3 m.
2 x2 Hence, the correct option is (a).

Chapter 09_Part 2.indd 86 11/12/2015 1:18:17 PM


Chapter 9  Fields  |  1.87

12. Consider the following statements with reference Maxwell’s divergence equations are
 ∂ρ  
to the equation ∇ ⋅ J = −  [2006] ∇ ⋅ D = ρV  and  ∇ ⋅ B = 0
∂t Statements (3) is false
1.  This is a point form of the continuity equation.
Hence, the correct option is (d).
2. Divergence of current density is equal to the
decrease of charge per unit volume per unit at 13. A point charge of +1 nC is placed in space with
every point. a permittivity of 8.85 × 1012 F/m as shown in the
3.  This is Maxwell’s divergence equation. figure. The potential difference VPQ between two
4.  This represents the conservation of charge. points P and Q at distance of 40 mm and 20 mm
respectively from the point charge is [2003]
Select the correct answer.
(a) Only 2 and 4 are true Q P
(b) 1, 2 and 3 are true 20 mm
(c) 2, 3 and 4 are true 40 mm
(d) 1, 2 and 4 are true 1 nC
Solution: (d)
 (a) 0.22 kV (b) -225 V
If  J  is surface charge density in C/m2 and ρ is the
(c) -2.24 kV (d) 15 V
volume charge density in C/m2 then according to
point form or differential form of continuity equa- Solution: (b)
tion, we have Distance of point P from charge r1 = 20 mm,
   
 ∂ρ Distance of point Q from charge r2 = 40 mm.
∇⋅ J = − (1)
∂t Q P
r1
Therefore, option (1) is true. r2
According to integral form of the Maxwell’s equa-
Q = 1 nC
tion, we have
The potential difference VPQ between two points P
  dQ and Q at distance of 40 mm and 20 mm respec­tively
I = ∫ J ⋅d s = − i from the point charge can be calculated using
S
dt
d ∂  VPQ = VQ − VP
dt ∫
=− ρV dv = − ∫  ρV  dv.
V  ∂t  Q Q Q 1 1
= − =  − .
4π ε o r2 4π ε o r1 4π ε o  r2 r1 
From the above equation we conclude that current
through closed surface or outward flow of positive Q 1× 10−9 × 109 × 36 π
Now = = 9,
charge is balanced by a decrease of positive charge 4π ε o 4π
in the volume enclosed by the closed surface or    
1 1  1 1 
in other words this equation are represents the and − =  − 103 m −1 = −25 m −1
principle of conservation charge which states that r2 r1  40 20 
charges can be neither created nor destroyed. Therefore we get after substituting the values
Therefore option (4) is also true. VPQ = 9 × (-25)V = -225 V

∇ ⋅ J represents divergence of current density Hence, the correct option is (b).

1 4. A parallel plate capacitor has an electrode area of

− ρ represents the decrease of charge unit vol- 100 mm2, with a spacing of 0.1 mm between the
∂t electrodes. The dielectric between the plates is air
ume per unit time at every point
with a permittivity of 8.85 × 10-12 F/m. The charge
Therefore option (2) is also true. on the capacitor is 100 V. The stored energy in the
Hence, Statements (1), (2) and (4) are true capacitor is [2003]

Chapter 09_Part 2.indd 87 11/10/2015 7:13:20 PM


1.88 | Electric Circuits and Fields

(a) 8.85 pJ (b) 440 pJ Now we have


(c) 22.1 nJ (d) 44.3 nJ V C d ε 1 3 3
  2 = 1 = 2× 1 = × = .
Solution: (d) V1 C2 d1 ε 2 0.5 4 2
Electrode area A = 100 mm2.
2
Spacing between the electrodes d = 0.1 mm, V2 + V2 = 100
3
Permitivity of free sace ε 0 = 8.85 × 10−12 F/m, ⇒ 5V2 = 300
    300
Charge on the capacitor V = 100 V. V2 = = 60 V.
5
Capacitance of the capacitor will be
Hence, the correct option is (b).
ε0 A
C= 1 6. Figure shows an ideal single-phasc transformer.
d The primary and secondary coils are wound on the
8.85 × 10−12 × 100 × 10−6 N 
= −3
= 8.85 × 10−12 F. core as shown. Turns ratio  1  = 2. The correct
0.1× 10  N2 
Energy stored in the capacitor is given by phasors of voltages E1, E2 currents I1, I2 and core
flux ϕ are as shown in below [2003]
1 1
CV 2 = × 8.85 × 10−12 × (100 )
2
E= f
2 2 I1 I2

= 4.425 × 10−8 = 44.25 nJ ≈ 44.3 nJ.


N1 N2 E2 R
Hence, the correct option is (d). ∼ E1
1 5. A composite parallel plate capacitor is made up
of two different dielectric materials with different
thickness (t1 and t2) as shown in figure. The two dif-
ferent dielectric materials are separated by a con- (a) E1 (b) E1
I2 I2
ducting foil F. The voltage of the conducting foil is E2
[2003] I1
E2
I1
100 V f f
(c) E1
I1
(d)
I
E1
er 1 = 3, t1 = 0.5 mm 1
F
er 2 = 4, t2 = 1 mm
f f
0V I2
E2 E2
I2
(a) 52 V (b) 60 V Solution: (b)
(c) 67 V (d) 33 V Consider the single phase transformer as shown
Solution: (b) below.
Given: f
I1 I2
ε r1 = 3, ε r 2 = 4,
d1 = 0.5 mm, d 2 = 1 mm.
N1 N2 E2 R
∼ E1
Capacitors C1 and C2 are in series and in series
charge Q remains same across both the capacitors.
Q C=
= 1V1 C2V2 ,
V1 + V2 = 100 V2 = ? The direction of I1 and the direction of ϕ and the
current I2 are also true according to the right hand
ε1 A ε2 A
C1 = , C2 = rule. According to the polarity shown, E1 is in
d1 d2 phase with I1 and E2 is in phase with I2.

Chapter 09_Part 2.indd 88 11/10/2015 7:13:24 PM


Chapter 9  Fields  |  1.89

But The required electric field will be

E1 I2
 Q  
N1 2 E= a
= = = . 4πε r 2 r
E2 I1 N2 1
10−6 1    
= (2ax − 1ax ) V/m.
According to Faraday’s law, voltage induced 4πε 0 × 5 5

dφ 10−6
e=− . = (2i − 1k )
dt 20 5πε 0
  
 
 

In terms of phasors, E = − j ω φ =
Where  ax i,= a y j  and  az = k
 
∴ E lags φ by 90°. Hence, the correct option is (a).
Hence, the correct option is (b). 18. Given the potential function in free space to be
 V ( x) = (50 x 2 + 50 y 2 + 50 z 2 ) V, the magnitude (in
1 7. The electric field E (in V/m) at the point (1, 1, 0) V/m) and the direction of the electric field at a point
due to a point charge of +1 mC located at (-1, 1, 1) (1, -1, 1), where the dimensions are in metres, are
(co-ordinates in metres) is [2001] [2001]
10−6 10−6 (a) 100; (i + j + k )
(a) (2i − k ) (b) (2i − k )
20 5πε 0 20πε 0 100
(b) ; (i − j + k )
−10 −6
−10 −6 3
(c) (2i − k ) (d) (2i − k )
20 5πε 0 20πε 0 100  (−i − j − k ) 
(c) ;  
Solution: (a) 3  3 
100  (−i + j − k ) 
B
(d) ;  
3  3 
E=? Solution: (d)
r
Potential function in free space is given as
V ( x, y, z ) = 50 x 2 + 50 y 2 + 50 z 2 .
A In free space electric field is given by

Q = 1 µC E ( x, y, z ) =  grad(V ) = −∇ V.

Point B = (x2, y2, z2) = B (1, 1, 0)  ∂   ∂   ∂  


= −  V ax + V a y + V az 
Point B = (x2, y2, z2) = B (1, 1, 0)    ∂x ∂y ∂z 
Position vector 
 
 

= − 100 xax + 100 ya y + 100zaz  V/m.

  
 
 

r = AB = [1 − (−1)]ax + (1 − 1)a y + (0 − 1)az The electric field at a point (1, -1, 1) will be

 
  
= 2ax + 0a y −1az .  
 
 

E (1, − 1, 1) = −[100ax − 100a y + 100az ] V/m.
Magnitude of position vector Magnitude of electric field at a point (1, -1, 1) will be
r = AB
E (1, − 1, 1) = 100 (−1) 2 + (1) 2 + (−1) 2
2 2 2
= 2 + 0 + 1 = 5. = 100 3.

 1    
ar = (2ax − 1az ). Direction of the electric field
 is given by the unit
5 vector in the direction of  E.

Chapter 09_Part 2.indd 89 11/10/2015 7:13:31 PM


1.90 | Electric Circuits and Fields

 conductor is a, then the inductance L can be calcu-


 E (1, − 1, 1)
aE = lated using
E (1, − 1, 1)
1        µ 1 −1  d 

= − a + a − a 
z .
L=  + cos h  
3 π 4  2 a 
x y

In unit vector i, j, k notation, µ 1  d 


L=  + ln    ,
 1 π 4  a 
aE = [−i + j − k ].
3 For a << d, substituting the values diameter = 1
Hence, the correct option is (d). cm, radius = a = 0.5 cm, d = 60 cm.
19. The conductors of a 10 km long, single phase, two 4π × 10−7 1  60  
wire line are separated by a distance of 1.5 m. The L=  4 + ln  0.5   H/m
π   
diameter of each conductor is 1 cm. If the conduc- 3
tors are of copper, the inductance of the circuit is   60   10
= 10−7 1 + 4 ln   × −3
mH/km.
[2001]   0.5   10
(a) 50.0 mH (b) 45.3 mH  60 
(c) 23.8 mH (d) 19.6 mH L = 0.1 + 0.4 ln   mH/km.
 0.5 
Solution: (c)
Length of the conductor, l = 10 km Hence, the correct option is (c).
Diameter = 1 cm, 2 1. The capacitance of the arrangement shown in the
figure is pF. [1997]
Radius, a = 0.5 cm
20 cm sq conducting plate
Spacing between the conductors, d = 1.5 m
Note that radius is very small as compared to the
spacing between the lines, therefore a << d. 10 cm e1 e1 = 1
Inductance, L of the circuit can be calculated as e3 e2 = 2
20 cm
given below e3 = 4
10 cm e2
lµ  1  d 
L= + ln   
π  4  a 
10 × 103 × 4π × 10−7
1 20 cm sq conducting plate
 150   10 cm
=
π  4 + ln  0.5  
   Solution:
−3
= 4 × 10 [0.25 + ln(300)] If we consider C1, C2 and C3 as the capacitances
due to dielectrics with ε1, ε2 and ε3 respectively
= 4 × 10−3 [0.25 + 5.7] = 23.8 mH
then voltage between the plates is divided between
Hence, the correct option is (c). C1 and C2 and voltage across C3 in same as the
2 0. For a single phase overhead line having solid cop- voltage between the plates.
per conductors of diameter 1 cm, spaced 60 cm Now we have,
between centres, the inductance in mH/km is
 A  A
[1999] ε 0ε1   ε 0ε 2  
(a) 0.05 + 0.2 ln 60 (b) 0.2 ln 60 C1 =  2  ,    C =  2 .
2
60 d1 d2
   60 
(c) 0.05 + 0.2 ln   (d) 0.2 ln  
 0.5   0.5  and
Solution: (c)  A
ε 0ε 3  
For parallel 2-wire line at low frequencies if d is C3 =  2 .
the distance between centres and radius of the wire d3

Chapter 09_Part 2.indd 90 11/10/2015 7:13:35 PM


Chapter 9  Fields  |  1.91

Therefore, the net capacitance will be series com- 1


bination of capacitors C1 and C2, in parallel with C3 C = 4πε a =
36π × 109
1 4π 10 1
C= + C3 , C= 9
F= F
1 1 36π × 10 100 9 × 1010
+
C1 C2 100
= pF ≈ 11 pF
9
1
∴ C=
1 1 1 23. The velocity of propagation of electromagnetic
+ +
 A  A  A wave n an underground cable with relative per-
ε 0ε1   ε 0ε 2   ε 0ε 3   mittivity of 3 will be m/sec. (Fill in the
2 2 2
blank) [1997]
d1 d2 d3
Solution:
ε0 A ε Aε 1
= + 0 3 Velocity of electromagnetic wave in cable v =
 1 1  2 d3 µε
2 d1  + 
 ε1 ε 2  1 1
v=
  µ0ε 0 εr
 
ε A 1 ε
= 0  + 3 . 
Substituting permittivity of free space or air
2  1 1  d3 
 d1  +   1 1
  ε1 ε 2   ε0 = F/m, = 3 × 108 m/s and εr
36π × 109 µ0ε 0
Substituting the values = 3, we get

d= d= 10 cm, d3 = 20 cm, 3 × 108


1 2 v= m/s = 3 × 108 m/s
3
ε1 = 1, ε 2 = 2 and ε 3 = 4.
A = (20 × 20) cm 2 .

ε0 =
1
F/m. Four-marks Questions
36 π × 109
1. Match the following List-I with List-II [1994]
10−9 × 400 × 10−4  1 4
C= + List-I
36π × 2 × 10−2   1  20 
10 1 +   (a) Line charge
  2   (b) Magnetic flux density
10−10 2  4 (c) Displacement current
=  + 2 = × 10−10 F
18π  3  27π (d) Power flow
400 List-II
= pF = 4.7 pF. (P) Maxwell
27π
(Q) Poynting vector
22. The capacitance of an isolated sphere of radius (R) Transmission line conductors
10 cm in air is equal to pF. [1997] (S) Biot–Savart’s law
Solution: (T) Gauss’s law
(U) Faraday’s law
We know that
Solution:
   Radius of sphere, a = 10 cm. Matching: (a–R), (b–T), (c–P), (d–Q)
Permittivity ε (for air) = ε0 We know that transmission line conductors are
Capacitance of isolated spherical shell of radius a examples of line change, described by ρ L C/m thus
can be calculated using relation the correct mapping is (a–R)

Chapter 09_Part 2.indd 91 11/10/2015 7:13:39 PM


1.92 | Electric Circuits and Fields

According to Gauss law in magnetic fields mag- and


netic flux
 flowing through a closed surfaces is  ∂ 
zero. If B is the magnetic flux density then Jd = D,
∂t
  
∫ B ⋅ ds = 0  
where  D = ε E  is the electric flux density. There-
as there are no enclosed magnetic charges and fore, a time changing electric field gives rise to a
magnetic field lines are closed. Thus the correct current.
mapping is (b–T). Thus the correct mapping is (c–P).
   
According to Maxwell, ∇ × H = J c + J d Power flow in electromagnetic fields is given by


 
where J c is the conduction current density and J d
Poynting vector, P which
 can2 be expressed math-
ematically as P = E × H W/m
is the displacement current density
Hence, the correct mapping is (d–Q).

 
Jc = σ . E

Chapter 09_Part 2.indd 92 11/10/2015 7:13:40 PM


Chapter 9  Fields  |  1.93
In the above relation Bx = -e-y sin(x) and By = −e-y
Five-marks Questions cos(x).
We can calculate the differential force on the con-
1. Themagnetic vector potential in a region is defined ductor using the relation given below
by A = e − y sin( x)aˆ z . An infinitely long conduc-
tor, having a cross-section area, a = 55 mm2 and    
carrying a DC current, I = 5 A in the y-direction, d F= I d L × B= I dy aˆ y × B
passes through this region as shown  in the figure.  ∞

Determine  the expression for (a) B and (b) force


F
= ∫ I dy (aˆ y ) × ( Bx aˆ x + By aˆ y )
density f exerted on the conductor
y =0
[2002]

y I
= ∫
y =0
I dy Bx (− aˆ z )
conductor

∫ 5 dy e sin x dy aˆ z
−y
x =
B
y =0

= −5sin( x) e − y aˆ z
0
I = 5sin( x)aˆ z N

Solution:
Force on a unit length of the conductor and hav-
Figure given below shows infinitely long conduc-
ing cross-section area (a = 5 mm2) will be equal to
tor placed in the y-direction and the magnetic field
force density, and can be expressed as
due the current carrying conductor
y I
 5sin( x)
conductor f = aˆ z N/m3
(a × 1)
x
B  5sin( x)
f = −6
aˆ z N/m3
5 × 10 × 1
= 106 sin( x)aˆ z N/m3
I
 2. (a) Show via the construction of a suitable
Given vector magnetic potential, A = e − y sin( x)aˆ z ,
Gaussian surface, that the capacitance of a spheri-
where= Ax 0=, Ay 0 and Az = e − y sin( x) cal capacitor consisting of two concentric shells of
We know that magnetic flux density, can be calcu- ab
radii a and b is given by C = 4 π ε 0 , where
lated using (b − a )
ε0 is the free space permittivity.
aˆ x aˆ y aˆ z aˆ x aˆ y aˆ z (b) A current I in the short conducting element
  ∂ ∂ ∂ ∂ ∂ shown in the figure. produces a flux density B1
B = ∇× A = = 0 at Point 1. Determine the magnitude and the
∂x ∂y ∂z ∂x ∂y
direction of the flux density vector at Point 2.
Ax Ay Az 0 0 Az  [2000]

1
∂ ∂ d
= aˆ x ( Az ) − aˆ y ( Az )
∂y ∂x
d
aˆ x [−1 e − y sin( x)] − aˆ y e − y cos( x)
=
d
−e − y sin( x)aˆ x − e − y cos( x)aˆ y Wb/m 2
= 2

= Bx aˆ x + By aˆ y Wb/m 2 , 1 B1

Chapter 09_Part 3.indd 93 11/10/2015 5:53:38 PM


1.94 | Electric Circuits and Fields

Solution: If R is the distance from the center and â R is the


(a) Consider the figure given below unit vector along the line from the element to the
−Q point, then the magnetic flux density at any
 point P
due to a differential
 current element I dL is given
+Q
 µ I d L × aˆ R
by B =
a 4π R 2
 
r Magnetic flux density B at Point 1 is given by B1
A b
B P µ IdL sin(90°)  µ IdL
= 2
az =
d  µd 2
4π   (1)
The region between the concentric spherical shells, 2
can be found by assuming a spherical Gaussian 
surface through point P. Magnetic flux density B at point 2 is given by
If Q be the charge on the spherical shells, applying 
 µ I d L × aˆr
Gauss’s law we get B2 =
  4π r 2
∫ D ⋅ d s = Q where r2 = d2 + d2 = 2d2 and
Q 
π r Q=
D 4= 2
,D along aˆr d=L × aˆr dL sin(180 + 45)aˆ z
4π r 2
   Q dL
=D ε E ,=
E aˆr , a < r < b = − aˆ z
4π ε r 2 2
We can calculate the potential difference between  dL 
µ I − aˆ z 
the conducting shells using the formula given below  2  µ I dL
∴ B2 = =

A
  4π (2 d 2 ) 8 2πd
VAB = − ∫ E ⋅ dL
 1 
B
∴ B2 =
− B1
 
 A
Q 8 2
For dL = dr ar , VAB = − ∫ 2
dr
B 4πε r 3. A charge +Q is uniformly distributed through-
a
Q Q 1 1 out the volume of a dielectric sphere of radius R
= = a − b and dielectric constant εR. Based on Gauss’s law,
4πε r 4πε  
b determine the expressions for the electric field,
Q 4πε ab E as a function of distance r from the centre of
C= = = 4πε . the sphere, within the ranges 0 < r < R and R ≤ r.
VAB  1 1  (b − a )
 −  Indicate expression(s) for the critical point(s) on
a b
the sketch. [1999]
(b) Consider the figure given below in which the
Solution:
elemental (or differential) length dL through which
Consider the Figure 1 given below in which the
I is flowing is shown.
y
charge distribution is shown
2
B2

r d
I Q R

dL r
d x

d P
2 P
1 B1 Figure 1

Chapter 09_Part 3.indd 94 11/10/2015 5:53:41 PM


Chapter 9  Fields  |  1.95

Q 3Q The resultant sketch of E as a function of r is


Charge per unit volume = = .
4 3 4 π R 3 shown in Figure 2 with critical points A and B at
πR r = R, i.e., the interface between the two media.
3
For 0 < r < R, when point P is inside the sphere, 4. An infinite number of charges, each equal to q are
assume a spherical Gaussian surface through P. placed along the x = 1, x = 2, x = 4, x = 8, x = 16
Applying Gauss law, we get and so on. Find the potential and electric field at
  point x = 0, due to this system of charges. [1998]
∫ D ⋅ d s = Charge enclosed Solution:
3Q 4 3 Consider the figure given below
D 4π r 2 = ⋅ πr
4π R 3 3 q q q q …
r3 x
2
D 4π r = Q 3 x=0 1 2 4 8
R
=1 =2 =22 =23
D r3
E = =Q 3
ε R ε 0ε R 4π r 2 Now the potential can be expressed as
Q
E= r = K1 r (1) q 1 1 1 1 
4π R 3ε 0ε R V= 1 + 2 + 22 + 23 +  .
4πε  
Q
where K1 = The infinite number of terms in the above summa-
4π R 3ε 0ε R
tion is in geometric series.
Q
as r → R, E1 = . q  First term 
4π ε 0ε R R 2 ∴ V= 1 − Common ratio 
4πε  
From the above relation we conclude that the sketch
of E with r is a straight line passing through the origin. q 1 q
= = .
For R < r < ∞, ε = ε 0 (for air) and point P is out- 4πε 1 2πε
1−
side the sphere, 2

D 4π r 2 = Q  q 1 1 1 1 
E=  + 2 + 4 + 6 + 
Q 1 K2 4πε 1 (2) (2) (2) 
E= =
4πε 0 r 2 r 2 (2) 
 
 q  Firrst term 
(−ax ) = (−ax )
Q 4πε 1 − Common ratio 
where K 2 = q 1 q
4πε 0 = =
4πε  1  3πε
Q 1 − 
as r → R, E2 = .  4
4πε 0 R 2
E1 < E2 as ∈R > 1 
where ax is the unit vector in the +ve direction and
The value of E rises from E1 to E2 along AB at the ε is the permittivity of the medium.
interface between the two medium (r = R). 5. An electron moves in the XY-plane with a speed
E of 106 m/s. Its velocity vector makes an angle of
60° with X-axis. A magnetic field of magnitude
B 10-2 T exists along the Y-axis. Compute the mag-
E2
E1 A
netic force exerted on the electron and its direction.
 [1998]
r Solution:
0 R
 Speed of electron  in XY-plane is given as
Figure 2 
v = 106 m/s along OP as shown figure given below.

Chapter 09_Part 3.indd 95 11/10/2015 5:53:45 PM


1.96 | Electric Circuits and Fields

Y v Q1 = 2000 pC
B B P B B Q2 = 1000 pC
30° Q3 = -1000 pC

E1 at P due to Q1 at point C is given by
60°  Q1 
0 X E1 = 2
along CP (1)
4πε o (CP)
  
Magnetic field B = a y 10−2 T (or Wb/m 2 ) As R >> a, CP ≅ R
Magnetic force on the electron can be calculated as  2000 × 10−12 × 36π × 109 
 

   ∴ E1 = a y = 18 a y V/m.
= −e(v × B ) = −e(v B sin θ )az 4π (1) 2

 
= −e 106 10−12 sin(30°)az
E 2 at P due to Q2 at A is given by
e   
= − 104 az N. Q2
2 E2 = 2 along AP
4π ∈o ( AP )
6. Determine the electric field intensity at the point P As AP ≈ R
for the arrangement shown in figure. [1997]
 1000 × 10−12 × 36π × 109
P E2 =
4π (1)
2
R=1m Q1 = 2000 pC
a = 1 cm Q2 = 1000 pC 
R
= 9 V/m along AP
Q3 = −1000 pC 
 

= 9 cos (θ ) ax + 9sin (θ ) a y (2)
Q1 
where ‘θ’ is shown in the figure.
a a

E3 at P due to Q3 at B is given by
d  
Q3
Q2 Q3 E3 = 2 along BP
a 4πε o ( BP )
Solution: As BP ≈ R,
The configuration of the charges is shown in the
 −1000 × 10−12 × 36π × 109
Figure 1, where the points A, B and C are marked. E3 =
E1 4π (1) 2

E2 = 9 V/m along PB

 

q = 9 cos(θ )ax − 9sin(θ )a y (3)
P

R=1m q y 
a = 1 cm
∴  The resultant E field at P is given by
E3    
R E = E1 + E2 + E3
x
Q1 C z From Equations (1), (2) and (3)
 
 
 
 

E = 18 a y + 9 cos(θ )ax + 9 sin(θ )a y + 9 cos(θ )ax
a a 

d − 9 sin(θ )a y
q 
 

Q2 Q3
A a B = 18 cos(θ )ax + 18 a y .

Q1 = 2000 pC R
From the figure tan θ ≅ = 200.
Q2 = 1000 pC a
Q3 = −1000 pC  
2

Chapter 09_Part 3.indd 96 11/10/2015 5:53:50 PM


Chapter 9  Fields  |  1.97

1 8. A square coil of 10 turns and 10 cm side is moved


∴ cos θ = . through a steady magnetic field of 1 Wb/m2 at a
200
 
 
 constant velocity of 2 m/sec with its plane perpen-
E = (0.09ax + 18a y ) V/m. dicular to the field as shown in the figure Plot the
variation of induced emf as the coil moves along
7. An infinitely long straight wire carries 1000 A of the field. [1997]
current and in the vicinity, there is a circular con-
ducting loop of 100 mm diameter with the centre x x x x x
a x
of the loop 1 m away from the straight conductor. x xB x x B = 1 Wb/m2
v = 2 m/s
Both the wire and the loop are coplanar. Determine a
x x x x x
d = 20 cm
the magnitude and direction of current in the loop x x xV x x
x x x x x a = 10 cm
that procedures a zero flux density at its centre.
 [1997] d

Solution: Solution:
The infinitely long wire PQ with I1 = 1000 A and The square coil ABCD moving into the magnetic
circular loop with center, O at r = 1 m is shown in field is shown in the Figure 1.
the figure.
Y
Q
2R = 100 mm

v = 2 m/s
B = 1 Wb/m2
R N-turn
I1
square coil x x x x x
O
I2 B
x x x x x
C B
a x x x x x X
r 0 10 20 30 40
D A x x x x x in cm
1m y a = 10 cm x x x x x x
20
x
Region of
P z magnetic field

Magnetic flux density, B1 at O due to PQ is given Figure 1


by
 µ0 I1 Let the right side of coil AB be at x = 0 initially at t
= B1 (−aˆ z ) = 0.
2π r
As the coil moves in the +ve direction of x, it
Let I2 be the current through the circular loop such comes under the influence of B (magnetic field)
that B2 produced by it at O cancels with B1 , giving when x > 10 cm (t > 0.05 sec) and ϕ cut by the coil
zero resultant flux density B . increases.
  µ I
B2 is given by B2 = 0 2 aˆ z dφ dφ dx
2R ∴  emf induced = e = N
 =N
dt dx dt
   µ I µI 
B = B1 + B2 =  0 2 − 0 1  aˆ z . dx
 2 R 2π r  where dφ = (a dx) B and =v
dt
I1 R I1 R 1000 50
∴ I2 = = = A = 15.92 A. e = Na Bv = 10 × 10−1 × 1× 2 V = 2 V
πr π r π 1000
The direction of I2 is counter-clockwise. ∴  For 10 < x < 20 or 0.05 < t < 0.10, e = 2 V

Chapter 09_Part 3.indd 97 11/10/2015 5:53:52 PM


1.98 | Electric Circuits and Fields

At t = 0.1 sec, the coil occupies the position 10 < e


x < 20 where the flux cut by the coil reaches its
maximum value ϕmax.
2V
For 20 < x < 30, or for 0.1 < t < 0.15, ϕ does not
change. ∴ emf = 0
x
For 30 < x < 40, or 0.15 < t < 0.2, ϕ decreases with x=0 10 20 30 40
in cm
time. ∴  e = -2 V
For 40 < x, or 0.20 < t, ϕ = 0, ∴  e = 0
t=0 0.0 0.1 0.1 0.2 t (in cm)
Variation of induced emf, e vs x and e vs t is shown
in Figure 2. Figure 2

Chapter 09_Part 3.indd 98 11/10/2015 5:53:52 PM


Unit 2
Signals and Systems

Chapter 1:  CT and DT Signals 2.3


Chapter 2:  LTI Systems (CT and DT) 2.7
Chapter 3:  Periodic Signal: Fourier Series 2.24
Chapter 4:  Signal Fourier Transform 2.32
Chapter 5:  Sampling Theorem 2.35
Chapter 6:  Signal: Laplace Transform 2.37
Chapter 7:  DT Signal: Z-Transform 2.40
Chapter 8:  Miscellaneous 2.42

Chapter 01.indd 1 11/9/2015 5:40:06 PM


Chapter 01.indd 2
Exam Analysis
Exam Year 92 93 94 95 96 97 98 99 00 01 02 03 04 05 06 07 08 09 10 11 12 13 14
1 Mark Questions – 1 2 4 1 1 1 – 1 – 5 1 – 1 1 2 2 1 4 4 2 5 4
2 Marks Questions – – – – 1 – 1 1 – 1 1 1 4 5 8 6 8 3 4 2 3 2 6
4 Marks Questions 1 – 1 – – – – – – – – – – – – – – – – – – – –
5 Marks Questions 1 – – – – 2 – – – 1 1 – – – – – – – – – – – –
Total Marks 2 1 3 4 2 3 2 1 1 2 7 2 4 6 9 8 10 4 8 6 5 7 10
CT and DT Signals – – 2 1 – – – – – – 1 – – – – 1 1 – – 1 – 1 1
LTI Systems (CT and DT) 2 1 1 1 1 2 1 – 1 1 3 2 2 – 4 5 5 3 3 3 2 4 3
Periodic Signal: Fourier Series – – – 1 1 1 – – – 1 2 – 2 3 1 1 1 1 2 1 – – 1
Signal Fourier Transform – – – – – – – – – – – – – – – – 1 – 1 – 1 – 2
Sampling Theorem – – – – – – – – – – – – – – – 1 – – – – – 1 1
Signal: Laplace Transform – – – 1 – – 1 1 – – 1 – – 2 1 – 1 – – 1 1 1 –
DT Signal: Z-Transform – – – – – – – – – – – – – 1 1 – 1 – – – – – 2
Miscellaneous – – – – – – – – – – – – – – 2 – – – 2 –1 – – –

11/9/2015 5:40:07 PM
Chapter 1
CT and DT Signals
fundamental frequency
One-mark Questions
1 50
f0 = =
1. For a periodic signal v(t )30 sin 100t + 10 cos 300t T π
 π ⇒ ω0 = 2π f 0 = 100 rad/sec
+ 6 sin  500t +  , the fundamental frequency in
 4
rad/sec is [2013] 2. A point z has been plotted in the complex plane, as
shown in the figure below: [2011]
Solution:
The periodic signal is
 π Im
v(t ) = 30 sin 100t + 10 cos 300t + 6 sin  500t + 
 4 Unit circle
We get Z
Re
ω1 = 100, ω2 = 300, ω3 = 500
2π 2π 2π
⇒ T1 = , T2 = , T3 =
100 300 500
2π 1
The plot of the complex number y =  is
T1 100 3 z
= = ,
T2 2π 1
(a) Im
300
Unit circle

T1 100 5
= = Re
T3 2π 1 y
500
Fundamental period,
T = x × T1

(b) Im
x = LCM (1, 1) = 1
Unit circle

T1 =
100 Re

2π π
⇒ T = 1× = y
100 50

Chapter 01.indd 3 11/9/2015 5:40:08 PM


2.4 | Signals and Systems

(c) Im Solution: (d)


Unit circle The voltage across the capacitor will be
y t t
1 1
Re vc (t ) = ∫ i(t ) dt = C −∞∫ 5 δ (t ) dt
C −∞
5 d
  = C u (t ), as δ (t ) = dt u (t )
Hence, the correct option is (d).
(d) Im
4. The convolution of the functions f1 (t ) = e −2t u (t )
Unit circle
and  f 2 (t ) = et u (t ) is equal to . [1995]
Re Solution:
1
y f1 (t ) = e −2t u (t ) 
→ F1 ( s ) =
s+2
Solution: (d) 1
f 2 (t ) = et u (t ) 
→ F2 ( s ) =
z lies inside the unit circle and in the first quadrant   s −1
as per the given location of z shown in Figure 1,
1 1
given below −
1
F1 ( s ) F2 ( s ) = = 3 + 3
Im ( s + 2)( s − 1) s + 2 s − 1
Unit circle
f1 (t ) * f 2 (t ) 
As  → F1 ( s ) F2 ( s )
Z
Re The convolution function will be
 1 1 
f1 (t ) * f 2 (t ) =  − e −2t + et  u (t )
 3 3 
Figure 1 +6
6. The value of the integral ∫ e −2t δ (t − 1) dt is equal to
Let z = re jθ, −5
. [1994]
where r < 1 and 0° < θ < 90° Solution:
Now, we have +6
e −2t
∫ e δ (t − 1) dt =
−2 t
= e −2
1 1 − jθ 1 jθ t =1
y= = e = e , φ = −θ −5
z r r t2

1 ∫ x(t )δ (t − t ) = x(t ) t
0 0 1 ≤ t0 ≤ t 2 = 0
∴ > 1 and  −90° < φ < 0° . t1
r   Otherwise
y
∴   lies in the fourth quadrant and outside unit
circle.
Two-marks Questions
Hence, the correct option is (d).
3. A current impulse, 5δ (t ), is forced through a capac- 1. The function shown in the following figure can be
itor C. The voltage, vc (t ), across the capacitor is represented as [2014-S1]
given by [2002]
1
5u (t ) − C
(a) 5t (b)

5 5u (t )
(c) t (d) t
C C 0 T 2T

Chapter 01.indd 4 11/9/2015 5:40:12 PM


Chapter 1  CT and DT Signals  |  2.5

(t − T )  t − 2T  (a) First delay  x[n]   by 3 samples to generate 


u (t ) − u (t − T ) +
(a) u (t − T ) −   u (t − 2T )
T  T  z1[n],  then pick every 4th sample of z1[n]  to
t t generate  z2 [n],  and then finally time reverse 
u (t ) +
(b) u (t − T ) − u (t − 2T ) z2 [n]  to obtain y[n]
T T
(b) First advance  x[n]  by 3 samples to generate
(t − T ) (t − 2T ) z1[n],  then pick every 4th sample of z1[n] to
u (t ) − u (t − T ) +
(c) u (t ) − u (t )
T T generate  z2 [n],  and then finally time reverse
(t − T ) (t − 2T ) z2 [n]  to obtain  y[n]
u (t ) +
(d) u (t − T ) − 2 u (t − 2T ) (c) First pick every fourth sample of x[n]  to gen-
T T
erate v1[n], time reverse v1[n]  to obtain  v2 [n],
Solution: (a) and finally advance v2 [n]  by 3 samples to ob-
x(t ) tain  y[n]
(2T, 1) (d) First pick every fourth sample of  x[n] to gener-
1
ate v1[n], time reverse v1[n]  to obtain v2 [n], and
finally delay v2 [n]  by 3 samples to obtain y[n]
t Solution: (b)
0 T 2T
(T, 0) Advance  x(n)  by 3 samples,  z1 (n) = x(n + 3)
(T , 0) (2T , 1) Picking up every fourth sample,
1− 0 z2 (n) = z1 (4n)
x(t ) − 0 = (t − T )
2T − T = x(4n + 3)
1 t
x(t ) = (t − T ) = − 1 T < t < 2T
T T Time reversing,
dx(1) 1
dx T
y (n) = z2 (−n) = x(−4n + 3)
1
∴ y (n) = x(3 − 4n)
t
T 2T
It is instructive to see what y (n) would be obtained
−1
if procedures in options (a), (c) and (d) are followed:
dx(t ) 1
= δ (t ) − δ ( t − T ) + [u (t − T ) − u (t − 2T )] z1 (n) : x(n − 3)
(a) 
dt T
z2 (n) : x(4n − 3)
1
x(t ) = u (t ) − u (t − T ) + [(r (t − T ) − r (t − 2T ))] y (n) : x(−4n − 3)
T
r (t ) = t ⋅ u (t )
(c)  v1 (n) = x(4n)
1 v2 (n) = x(−4n)
x(t ) = u (t ) − u (t − T ) + [(t − T )u (t − T )
T
y (n) = x[−4(n + 3)]
− (t − 2T )u (t − 2T )]
(t − T ) = x(−4n − 12)
x(t ) = u (t ) − u (t − T ) + u (t − T )
T (d)  v1 (n) = x(4n)
(t − 2T )
− u (t − 2T ) v2 (n) = x(−4n)
T
Hence, the correct option is (a). y (n) = x[−4(n − 3)]
2. Given a sequence x[n], to generate the sequence = x(−4n + 12)
y[n] = x[3 − 4n], which one of the following proce-
Hence, the correct option is (b).
dures would be correct? [2008]

Chapter 01.indd 5 11/9/2015 5:40:18 PM


2.6 | Signals and Systems

3. If u (t ), r (t ) denote the unit step and unit ramp (c) f(t )
functions, respectively and u (t ) * r (t )  their convo- 2
lution, then the function u (t + 1) * r (t − 2)  is given
by [2007] 1

1 t
  (t − 1)(t − 2)
(a) 0 1
2 (d) f(t )
1
  (t − 1)(t − 2)
(b) 2
2
1
1
  (t − 1) u (t − 1)
2
(c)
2 0 1 2
t
(d) None of the above f(t)
Solution: (c) tu (t −1) (Q)
(P) (t + 1) u (t − 1)
1 tu (t ) (S)
(R) (t +1)u (t )
u (t ) ↔
s (
(T) t −1 )u (t ) (t − 1)u (t − 1)
(U)
es
u (t + 1) ↔ Solution:
s Matching: (a, R), (b, U), (c, S), (d, P)
1 (a) f 1(t )
r (t ) ↔
s2
e −2 s 1
r (t − 2) ↔
s2 t
0 1
s −2 s −s
e e e f=
u (t + 1) * r (t − 2) ↔ ⋅ 2 ↔ 3 (R) 1 (t ) tu=
(t ) 1r (t ) unit ramp function
s s s
(b) f 2(t )
t2 1
u (t ) ↔ 3
2 s
1
(t − 1) 2 e− s
u (t − 1) ↔ 3 0 1 2
t
2 s
f 2 (t ) = f1 (t − 1) = (t − 1)u (t − 1) = 1r (t − 1)
(U)
Hence, the correct option is (c).
(c) f 3(t )
Four-marks Questions 2

1. Match the waveforms on the left-hand side with 1


the correct mathematical description listed on the
t
right-hand side. [1994] 0 1

(a) f(t ) f 3 (t ) = u (t ) + f1 (t ) = 1u (t ) + tu (t ) = (t + 1) u (t )
(S)

1 (d) f 4(t )
t 2
0 1
(b) f(t ) 1

0 1 2
t
1
t f 4 (t ) = f 3 (t − 1) = tu (t − 1)
(P)
0 1 2

Chapter 01.indd 6 11/9/2015 5:40:22 PM


Chapter 2
LTI Systems (CT and DT)
2. Consider an LTI system with transfer function
One-mark Questions 1
H (s) = . If the input to the system is
s ( s + 4)
1. x(t) is non-zero only for Tx < t < Tx′,  and similarly, cos(3t ) and the steady-state output is A sin(3t + α ),
y(t) is non-zero only for Ty < t < Ty′. Let z(t) be then the value of A is [2014-S2]
convolution of x(t) and y(t). Which one of the fol- 1 1
lowing statements is TRUE? [2014-S1] (a) (b)
30 15
(a)  z(t) can be non-zero over an unbounded interval
3 4
(b) z(t) is non-zero for t < Tx + Ty (c) (d)
4 3
(c) z(t) is zero outside of Tx + Ty < t < Tx′ + Ty′
(d) z(t) is non-zero for t > Tx′ + Ty′ Solution: (b)
1
Solution: (c) Transfer function H ( s ) =
s ( s + 4)
Consider the figure given below:
Input of the system = cos (3t)
x(t ) Steady-state output of the system is A sin(3t + α )
The frequency response of the system is
t 1
Tx T x′ H ( jω ) =
jω ( jω + 4)
x(t) is non-zero for Tx < t < Tx′ 1
We conclude that y(t) is non-zero for Ty < t < Ty ′ . H (ω ) =
ω ω 2 + 16
y(t )
x(t ) = A cos(ω0 t + φ ) is input then the output is
If 
y (t ) = A H (ω0 ) cos(ω0 t + φ + ∠H ω0 )
t
Ty T y′ x(t ) = cos(3t )ω0 = 3
1 1 1
Z (t ) = x(t ) * y (t ) H (ω0 ) = A = = =
  3 9 + 16 3 × 5 15
z(t ) Hence, the correct option is (b).
3. Consider an LTI system with impulse response h(t)
Tx + Ty T x′ + T y′
t = e −5t u (t ). If the output of the system is y (t) = e-3tu (t)
− e −5t u (t )  then the input, x(t) is given by [2014-S2]
∴ z (t ) is zero outside of Tx + Ty < t < Tx′ + Ty ′ . (a) e −3t u (t ) (b) 2 e −3t u (t )
Hence, the correct option is (c). (c) −5t
e u (t ) (d) 2 e −5t u (t )

Chapter 02.indd 7 10/31/2015 2:53:35 PM


2.8 | Signals and Systems

Solution: (b) e− s 1 e− s
Impulse response h(t ) = e −5t u (t ) Y ( s) = X (s) H (s) = × = 3
s s2 s
Output of the system y (t ) = e −3t u (t ) − e −5t u (t )
Taking inverse Laplace transforms we get
1 y (t ) = p (t −1)
H (s) =
s+5 where p(t) = parabolic function
1 1 2 1 t2
Y ( s) = − = = u (t )
s + 3 s + 5 ( s + 3)( s + 5) s3 2
Y ( s) 2 e − s (t − 1) 2
X (s) = = = u (t − 1)
H (s) S + 3 s3 2
x(t ) = 2e −3t u (t ) Hence, the correct option is (c).
6. Two systems with impulse responses h1(t) and h2(t)
Hence, the correct option is (b).
are connected in cascade. Then the overall impulse
4. Assuming zero initial condition, the response y(t) response of the cascaded system is given by [2013]
of the system given below to a unit step input u(t) (a) product of h1(t) and h2(t)
is . [2013] (b) sum of h1(t) and h2(t)
U(s) 1 Y(s) (c) convolution of h1(t) and h2(t)
s (d) subtraction of h2(t) from h1(t)
(a) u(t) (b) tu(t) Solution: (c)
Consider the figure given below in which two sys-
t 2
(c) u (t ) (d) e − t u (t ) tems with impulse responses h1(t) and h2(t) are
2 connected in cascade.
Solution: (b) x(t) y(t)
h1(t) h 2(t)
The given system is
U(s) 1 Y(s) Now, we have
s
y (t ) = [h1 (t ) * h2 (t )]* x(t )
1
Input of system is u(t), then u ( s ) = . Overall impulse response = h1 (t ) * h2 (t ) = Convolu­
s tion of h1(t) and h2(t)
1 1 1 1
Y ( s) = u (s) × = × = 2 Hence, the correct option is (c).
s s s s
Output y (t ) = tu (t ) 7. A low-pass filter with a cut-off frequency of 30 Hz
is cascaded with a high-pass filter with a cut-off
Hence, the correct option is (b).
frequency of 20 Hz. The resultant system of filters
5. The impulse response of a system is h(t ) = tu (t ). will function as [2011]
For an input u(t – 1), the output is _______. (a) an all-pass filter
 [2013] (b) an all-stop filter
t2 t (t − 1) (c) a band stop (band-reject) filter
(a) u (t ) (b) u (t − 1)
2 2 (d) a band-pass filter
(t − 1) 2 t 2 −1 Solution: (d)
(c) u (t − 1) (d) u (t − 1)
2 2 Figure 1 given below shows the frequency response
Solution: (c) H1(t) of LPF with cut-off frequency, 30 Hz
Impulse response of system h(t ) = tu (t ) H 1(f )

Input x(t ) = u (t −1)


The output will be
f
y(t) = x(t)*h(t) 0 10 20 30 40

Taking Laplace transform on both sides we get Figure 1

Chapter 02.indd 8 10/31/2015 2:53:39 PM


Chapter 2  LTI Systems (CT and DT)  |  2.9

Figure 2 given below shows the frequency response (c) causal but not linear
H2(t) of HPF with cut-off frequency 20 Hz. (d) neither linear nor causal
H 2(f ) Solution: (b)
Input x(t) gives output
5t

f
y (t ) = ∫ x(τ ) dτ , t>0
−30 −20 0 20 30 −∞
5t

Figure 2 Let x1(t) gives  y1 (t ) = ∫ x (τ ) dτ ,


−∞
1 t>0
In Figure 3 if two filters are cascaded, the over fre- 5t
quency response of the resultant system will be Let x2(t) gives y2 (t ) = ∫ x (τ ) dτ ,
−∞
2 t>0
H ( f ) = H1 ( f ) H 2 ( f ) ,
x′(t ) = ax1 (t ) + bx2 (t )  gives
Then 
H(f )
5t
y ′(t ) = ∫ [ax (τ ) + bx (τ )] dτ ,
−∞
1 2 t>0

f 5t 5t
−30 −20 0 20 30 = a ∫ x1 (τ ) dτ + b ∫ x2 (τ ) dτ , t > 0
−∞ −∞
Figure 3
As  y ′(t ) = ay1 (t ) + b y2 (t ),  the system is linear.
BPF with bandwidth, B = 10 Hz The output y at time t is depending upon input x
Hence, the correct option is (d). up to the future time 5t, therefore the system is not
8. Given two continuous time signals x(t ) = e − t and causal.
−2 t
y (t ) = e  which exist for t > 0, the convolution ∴  The system is linear but not causal.
z (t ) = x(t ) * y (t )  [2011] Hence, the correct option is (b).
(a) e − t − e −2t (b) e −3t 2
+t −t −2 t 10. For the system , the approximate time taken
(c) e (d) e +e ( s + 1)
Solution: (a) for a step response to reach 98% of its final value
First continuous time signal  x(t ) = e −t is [2010]
(a) 1 s (b) 2 s
Second continuous time signal  y (t ) = e −2t (c) 4 s (d) 8 s
−t 1
For  x(t ) = e , t > 0 → X ( s ) = LT
Solution: (c)
s +1
2
1 H (s) =
−2 t
For  y (t ) = e , t > 0 → Y ( s ) = LT
s +1
s+2 x(t ) = u (t )
=
If  z (t ) x= (t ) * y (t ), z ( s ) X ( s )Y ( s ) 1
X (s) =
1 1 1 s
Z (s) = = − 2 2 2
( s + 1)( s + 2) s + 1 s + 2 Y (s) = X (s) H (s) = = −
s ( s + 1) s s + 1
Taking inverse LT, z (t ) = (e − t − e −2t )u (t ) −t
y (t ) = 2(1 − e )u (t )
Hence, the correct option is (a). y (∞ ) = 2
9. The system represented by the input-output rela- 0.98 × 2 = 2(1 − e −T )
5t
tionship: y (t ) = ∫ x(τ ) dτ , t > 0 is
−∞
[2010] 1 − e −T = 0.98 ⇒ e −T = 0.02
e +T = 50 ⇒ T = ln 50 = 4 sec
(a) linear and causal
(b) linear but not causal Therefore, the answer is (c).

Chapter 02.indd 9 10/31/2015 2:53:44 PM


2.10 | Signals and Systems

11. A linear time-invariant system with an impulse (c) Both Statement (I) and Statement (II) are false
response h(t) produces output y(t) when input x(t) (d) Both Statement (I) and Statement (II) are true
is applied. When the input x(t − τ ) is applied to a Solution: (d)
system with response h(t − τ ), the output will be An LTI system is causal if the impulse response,
 [2009] h(t) = 0 for t < 0
(a) y(t) (b) y (2(t − τ ))
The principle of superposition holds for a linear
(c) y (t − τ ) (d) y (t − 2τ )
system.
Solution: (d)
Therefore, both statements I and II are true.
If the input is x(t) to an LTI system with impulse
response h(t), the output y(t) is given by Hence, the correct option is (d).
14. Let a signal a1 sin(ω1t + φ1 )  be applied to a stable
y (t ) = x(t ) * h(t )
linear time-invariant system. Let the corresponding
If the input is x(t − τ )  to the same LTI system steady-state output be represented as a2 F (ω2 t + φ2 ).
with impulse response h(t − τ ), the output y1 (t ) Then which of the following statements is TRUE?
is given by y1 (t ) = x(t − τ ) * h(t − τ ) = y (t − τ − τ )  [2007]
= y (t − 2τ )  (Property of convolution is used) (a)  F is not necessarily a ‘sine’ or ‘cosine’ func-
tion but must be periodic with ω1 = ω2.
Hence, the correct option is (d).
(b) F must be a ‘sine’ or ‘cosine’ function with
12. A signal e −1α t sin(ωt ) is the input to a real linear a1 = a2 .
time-invariant system. Given K and ϕ are con- (c)  F must be a ‘sine’ function with ω1 = ω2  and
stants, the output of the system will be of the form φ1 = φ2 .
Ke − β t sin(υ t + φ ) where [2008] (d) F must be a ‘sine’ or ‘cosine’ function with
(a) b need not be equal to a but υ is equal to w ω1 = ω2 .
(b) υ need not be equal to w but b is equal to a Solution: (d)
(c) b is equal to a and υ is equal to w Let  H (ω ) be the frequency response of stable LTI
(d) b need not be equal to a and υ need not be system.
equal to w For input,  x(t ) = a1 sin(ω1t + φ1 ),  the steady-state out-
Solution: (c) put, y(t) = a1 H (ω1 ) sin(ω1t + φ1 + ∠H (ω1 ))
 If the input x(t ) = sin(ωt )  to a real linear time-
Comparing with  y (t ) = a2 F (ω2 t + φ2 ) given in the
invariant system with frequency response H (w)
question:
= Ke jφ ,  then the output will be
F is a sine function with
y (t ) = K sin(ωt + φ )
ω2 = ω1 , a2 = a1 H (ω1 )
If the input is a damped sinusoid = e −α t sin(ωt ) to
the same system, then the output can be shown to φ2 = φ1 + ∠H (ω1 )
−α t
be Ke sin(ωt + φ ). F may become a cosine function if
∴  For the output given in the form Ke − β t sin(υ t + φ ) π
< [φ1 + ∠H (ω1 )] < π ,  but  ω2 = ω1
b = a and υ = w 2
Hence, the correct option is (c). a2 ≠ a1, unless H (ω1 ) = 1
13. The impulse response of a casual linear time- φ2 ≠ φ1 , unless ∠H (ω1 ) = 0
invariant system is given as h(t). Now consider the Hence, the correct option is (d).
following two statements: [2008] 1 5. A control system is defined by the following mathe-
Statement (I): Principle of superposition holds d2x dx
matical relationship  2 + 6 + 5 x = 12(1 − e −2t ).
Statement (II): h(t) = 0 for t < 0 dt dt
Which one of the following options is correct? The response of the system as t → ∞ is [2003]
(a) Statement (I) is true and Statement (II) is false (a) x = 6 (b) x = 2
(b) Statement (II) is true and Statement (I) is false (c) x = 2.4 (d) x = –2

Chapter 02.indd 10 10/31/2015 2:53:51 PM


Chapter 2  LTI Systems (CT and DT)  |  2.11

Solution: (c) y (t ) = te − t , t > 0.  The transfer function of the sys-


d2x dx tem is [2000]
Given + 6 + 5 x = 12(1 − e −2t )
dt 2 dt 1 1
(a) 2 (b)
1 1  ( s + 1) s ( s + 1) 2
12  − 
s s+2 s
X ( s ) = 2
1
(c) 2 (d)
s + 6s + 5 ( s +1) s ( s + 1)
24 Solution: (c)
=
s ( s + 2)( s + 1)( s + 5) Step response is given as
24 y (t ) = te − t , t > 0
Response, x(t ) = Lim [ sX ( s )] = = 2.4 (Final val-
x →0 10 1
Y (s) =
ue theorem of LT is used) ( s + 1) 2
Hence, the correct option is (c). 1
u (t ) → U ( s ) =
16. Let s(t) be the step response of a linear system with s
zero initial conditions. Then the response of this Y ( s) s
∴ TF = H ( s ) = =
system to an input u(t) is [2002] U ( s ) ( s + 1) 2
t
Hence, the correct option is (c).
∫ s(t − τ )u (τ ) dτ
(a)
0 1 8. The output of a linear time-invariant control sys-
tem is c(t) for a certain input r(t). If r(t) is modi-
d  
t
(b)  ∫ s (t − τ )u (τ ) dτ  fied by passing it through a block whose transfer
dt  0  function is e–s and then applied to the system, the
modified output of the system would be [1998]
t
t 
∫0
(c) s (t − τ )  ∫ u (τ 1 )dτ 1  dτ c(t ) c(t )
(a) t (b)
0  1+ e 1+ e − t
t (c) c(t )u (t −1)
c(t − 1)u (t − 1) (d)
∫ s(t − τ ) u (τ ) dτ
2
(d)
0
Solution: (c)
The given data is represented as shown in Figure 1
Solution: (b)
and Figure 2
Output, y(t) of LTI system to an arbitrary input,
u(t) is given by y (t ) = h(t ) * u (t ) (Property of LTI r (t ) c(t )
H(s)
system is used) where h(t) is the impulse response.
Figure 1
d
If s(t) is the step response, h(t ) = s (t ) (Property
dt r (t)
e −s
r (t − 1)
H(s)
c′(t )
of LTI system is used)
d  d Figure 2
∴ y (t ) =  s (t )  * u (t ) = [ s (t ) * u (t )]
 dt  dt
Assuming that r(t) is causal and LTI system is also
(Property of convolution operation is used) causal, then c(t) will be causal,
d  
t
i.e., c(t ) = 0, t < 0
 =  ∫ s (t − τ )u (τ ) dτ  (Definition of convolution
dt τ = 0  So output c(t ) ⋅ u (t )
is used) According to the property of LTI system if input is
Hence, the correct option is (b). delayed output is also delayed by same amount.
17. A linear time-invariant system initially at rest, when Output for input r (t − 1) will be c(t − 1)u (t − 1)
subjected to a unit-step input, gives a response  Hence, the correct option is (c).

Chapter 02.indd 11 10/31/2015 2:53:56 PM


2.12 | Signals and Systems

19. A differentiator has transfer function whose 22. s(t) is the step response and h(t) is impulse response
 [1997] of a system. Its response y(t) for any input u(t) is
(a) phase increase linearly with frequency given by [1993]
(b) amplitude remains constant d
t

(c) amplitude increases linearly with frequency (a) ∫ s (t − τ )u (τ ) dτ


dt 0
(d) amplitude decreases linearly with frequency
t
Solution: (c)
Input-output relation of differentiator:
(b) ∫ s(t − τ )u (τ ) dτ
0

d t τ
y (t ) = x(t )
dt ∫ ∫ s(t − τ1 )u (τ1 ) dτ1 dτ
(c)
0 0
Y (s)
TF H=
= (s) =s d
t
X (s) (d) ∫ h(t − τ )u (τ ) dτ
dt 0
Frequency response H ( jω ) = jω ⇒ H ( jω ) = ω ,
amplitude increases linearly with frequency Solution: (a)
π If s(t) is step response and h(t) is impulse response
 ∠H ( jω ) = rad/sec, phase is constant –90° for of an LTI system, then response, y(t) for any input,
2
all w. u(t) is given by
Hence, the correct option is (c). y (t ) = u (t ) * h(t )
20. The impulse response of an initially relaxed lin- d
ear system is e −2t u (t ). To produce a response of = u (t ) * s (t )
dt
te −2t u (t ), the input must be equal to [1995] d
1 −2t = [u (t ) * s (t )]
−t
(a) 2e u (t ) (b) e u (t ) dt
2 d

dt τ =∫−∞
−2 t −t
(c) e u (t ) (d) e u (t ) = u (τ ) s (t − τ ) dτ
Solution: (c) t
d
dt τ ∫= 0
−2 t LT 1 = u (τ ) s (t − τ ) dτ ,
h(t ) = e u (t ) → H ( s ) =
s+2
1 If u(t) and s(t) are causal functions.
y (t ) = te −2 t u (t ) LT → Y ( s ) =
( s + 2) 2 1. Remember the property of LTI system: If the in-
Y ( s) 1 put is differentiated, output is also differentiated.
TF = H ( s ) = = 2.  Remember the property of convolution:
X (s) s + 2
Y (s) 1 d dx(t )
∴ X (s) = = [ x(t ) * g (t )] = * g (t )
H (s) s + 2 dt dt
d
Hence  x(t ) = e −2t u (t ) = x(t ) * g (t )
dt
Hence, the correct option is (c).
Hence, the correct option is (a).
21. If f (t) is the step-response of a linear time-invariant
system, then its impulse response is given by
. [1994] Two-marks Questions
Solution:
If f (t) is the step response of an LTI system, then 1. The impulse response of a continuous time system
its impulse response h(t) is given by is given by h(t ) = δ (t − 1) + δ (t − 3). The value of
the step response at t = 2 is  [2013]
d (a) 0 (b) 1
h(t ) = f (t ).
dt (c) 2 (d) 3

Chapter 02.indd 12 10/31/2015 2:54:00 PM


Chapter 2  LTI Systems (CT and DT)  |  2.13

Solution: (b) I1 is bounded, I1 → ∞ as t → ∞


Impulse response of a continuous time system is
Thus we conclude that the system is not stable.
h(t ) = δ (t − 1) + δ (t − 3)
Hence, the correct option is (d).
Step response will be
3. Let y[n] denote the convolution of h[n] and g[n],
δ (t ) = h(t ) * u (t ) n
1
where h[n] =   u[n] and g[n] is a causal sequence.
= (δ (t − 1) + δ (t − 3)) * u (t ) 2
= u (t − 1) + u (t − 3) 1
If y[0] = 1  and y[1] = , then g[1] equals [2012]
2
s(t ) 1
(a) 0 (b)
2
2
3
(c) 1 (d)
1 2
Solution:
t n
0 1 2 3 1
h( n) =   u ( n)
Value of step response at t = 2 is s(2) = 1 2
Hence, the correct option is (b). n
y ( n) = h( n) * g ( n) = ∑ h( n − k ) g ( k )
2. The input x(t) and output y(t) of a system are k =0
t
related as y (t ) = ∫ x(τ ) cos(3τ ) dτ . The system is y ( 0) = ∑ h ( − k ) g ( k ) = h ( 0) g ( 0)
 −∞ [2012] k =0

(a) time-invariant and stable 1 = 1g (0), g (0) = 1


(b) stable and not time-invariant
1
(c) time-invariant and not stable y (1) = ∑ h(1 − k ) g (k )
(d) not time-invariant and not stable k =0

Solution: (d) = h(1) g (0) + h(0) g (1)


t
x(t ) → y (t ) = ∫
τ =−∞
x(τ ) cos(3τ ) dτ 1 1
= × 1 + 1 g (1), g (1) = 0
2 2
x(t − t0 ) → O /P Hence, the correct option is (a).

t
4. The response h (t) of a linear time-invariant system
= ∫
τ =−∞
x(τ − t0 ) cos(3τ ) dτ  Let τ − t0 = τ 1
to an impulse δ (t), under initially relaxed condi-
t tion is h(t ) = e − t + e −2t .  The response of this sys-
= ∫ x(τ 1 ) cos(3t0 + 3τ 1 ) dτ 1 tem for a unit step input u(t) is [2011]
τ1 =−∞
(a) u (t ) + e − t + e −2t
t − t0
(e − t + e −2t )u (t )
(b)
y (t − t0 ) = ∫ x(τ ) cos(3τ ) dτ
τ =−∞
(1.5 − e − t − 0.5 e −2t )u (t )
(c)
O /P ≠ y (t − t0 )
e − t δ (t ) + e −2t u (t )
(d)
From the above relation we conclude that the sys-
tem is time-varying. Solution: (c)
For an LTI system with input δ (t), response
For a bounded input,  x(t ) = cos(3t )u (t )
t h(t ) = e −1 + e −2t , t > 0
y (t ) = ∫ cos (3τ ) dτ
2 t

0 Now, we know that u (t ) = ∫ δ (t ) dt.


−∞
t t
1 1
= ∫ dτ + ∫ cos(6τ ) dτ = I1 + I 2 We also know that output is also integrated for an
2 2 LTI system, if the input is integrated,
0 0

Chapter 02.indd 13 10/31/2015 2:54:05 PM


2.14 | Signals and Systems

∴ For unit step input, 1u(t), the response s(t) is (a) each system in the cascade is individually
given causal and unstable
t (b) at least one system is unstable and at least one
s (t ) = ∫ h(t )dt system is causal
−∞ (c) at least one system is causal and all systems
s (t ) = 0, t < 0 are unstable
t (d) the majority are unstable and the majority are
= ∫ (e − t + e −2t )dt , t > 0 causal
0
t
Solution: (b)
e −2t
−t We know that a cascade of three LTI systems is
= −e −
2 0
causal and unstable. The overall impulse response
because the systems are LTI systems is given by
e −2t 3
= −e − t − + , for t > 0 h(n) = h1 (n) * h2 (n) * h3 (n) (1)
2 2
∴ s (t ) = (1.5 − e − t − 0.5e −2t )u (t ) and transfer function is given by

Hence, the correct option is (c). H ( z ) = H1 ( z ) H 2 ( z ) H 3 ( z ) (2)


5. Given the finite length input x[n] and the corre- We know that a causal LTI system is unstable, if its
sponding finite length output y[n] of an LTI system H(z) has at least one pole located outside the unit
as shown below, the impulse response h[n] of the circle in the Z-plane and this pole may be due to
system is [2010] any one of the three systems.
∴  At least one of the systems is unstable.
h[n]
x[n] = {1, −1} y [n] = {1, 0, 0, 0, −1}  From (1) according to the width property of
convolution at least one of the three systems is
causal e.g., if h1(n) and h2(n) are non-causal with
(a) { ↑}
h[n] = 1, 0, 0,1 (b) { }
h[n] = 1, 0,1
↑ h1 (n) = 0  for  n ≤ − N1  and  h2 (n) = 0  for  n ≤ − N 2
h3(n) may be causal with h3 (n) = 0 for n ≤ N1 + N 2
h[n] = {1,1,1,1} (d)
(c) h[n] = {1,1,1}  
↑ ↑ Thus we conclude that at least one system is unsta-
Solution: ble and at least one system is causal.
y (n) = δ (n) − δ (n − 4) → Y ( z ) = 1 − z −4 Hence, the correct option is (b).
x(n) = δ (n) − δ (n − 1) → X ( z ) = 1 − z −1 7. The z-transform of a signal x[n] is given by
4 z −3 + 3 z −1 + 2 − 6 z 2 + 2 z 3. It is applied to a system,
Y ( z) with a transfer function H ( z ) = 3 z −1 − 2 . Let the
∴ H ( z) =
X ( z) output be y(n). Which of the following is TRUE?
1 − z −4  [2009]
= (a) y(n) is non-causal with finite support
1 − z −1
(b) y(n) is causal with infinite support
(1 + z −2 )(1 + z −1 )(1 − z −1 ) (c) y (n) = 0; n > 3
=
(1 − z −1 ) (d) Re[Y ( z )]z = e jθ = − Re[Y ( z )]z = e− jθ ;
= 1 + z −1 + z −2 + z −3 Im[Y ( z )]z = e jθ = Im[Y ( z )]z = e− jθ ; − π ≤ θ < π
∴ h(n) = δ (n) + δ (n − 1) + δ (n − 2) + δ (n − 3) Solution: (a)
∴ {
h(n) = 1,1,1,1,
↑ } Y ( z) = X ( z)H ( z)
Hence, the correct option is (c). = 12 z −4 − 8 z −3 + 9 z −2 − 4 − 18 z + 18 z 2 − 4 z 3
6. A cascade of three linear time-invariant systems is ∴ y (n) = 12δ (n − 4) − 8δ (n − 3) + 9δ (n − 2) − 4δ (n)
causal and unstable. From this, we conclude that − 18δ (n + 1) + 18δ (n + 2) − 4δ (n + 3)
 [2009] y (−1) = −18, y (−2) = 18, y (−3) = −4

Chapter 02.indd 14 10/31/2015 2:54:11 PM


Chapter 2  LTI Systems (CT and DT)  |  2.15

is not a causal signal with 3 (finite num-


∴ y(n) ∴  System is non-causal, time-variant and unstable.
ber) non-zero samples at n = −1, − 2 and − 3 It may be noted that the system is linear.
∴  Hence, y(n) is non-causal with finite support. Hence, the correct option is (d).
∴  Option (a) is true and option (b) is not true. 9. A signal x(t ) = sin c(α t ) where a is a real con-
It may be noted that y(n) = 0 for n ≤ −4 and n ≥ 5.
 sin(π x) 
Hence, option (c) is not true. stant  sin c( x) =  is the input to a linear
 πx 
y(n) is a real signal time-invariant system whose impulse response
∴ Re[ y (e jθ )] Is even and Im[ y (e jθ )] is odd, h(t ) = sin c( β t ) where b is a real constant. If
i.e., Re[Y ( z )]z = e jθ = Re[Y ( z )]z = e− jθ ; min(a, b) denotes the minimum of a and b, and
similarly max(a, b) denotes the maximum of a
Im[Y ( z )]z = e jθ = − Im[Y ( z )]z = e− jθ and a and K is a constant, which one of the fol-
lowing statements is true about the output of the
Therefore, option (d) is not true.
system? [2008]
Hence, the correct option is (a). (a) It will be of the form K sin c(γ t ) where
8. A system with input x(t) and output y(t) is defined γ = min(α , β )
−2 t
by input-output relation: y (t ) = ∫ x(τ ) dτ . The sys- (b) It will be of the form K sin c(γ t ) where
tem will be −∞
[2008] γ = max(α , β )
(a) Causal, time-invariant and unstable (c) It will be of the form K sin c(α t )
(b) Causal, time-invariant and stable (d) It cannot be a sin c type of signal
(c) Non-causal, time-invariant and unstable
(d) Non-causal, time-variant and unstable Solution: (a)
For the linear time-invariant system with impulse
Solution: (d)
−2 t response,
y (t ) =
Given  ∫ x(τ ) dτ
−∞
h(t ) = sin c( β t ) FT
→ H ( f )
Y at time t is depending on values of x(t) in the Input, x(t ) = sin c(α t ) FT
→ X ( f )  is shown in
range of t = 0 to (–2t). Figure 1
i.e., y for negative values of time is depending on x
x(f )
at positive values of time.
Hence system is non-causal. 1
a
For a bounded input like a step function the output
is not bounded, as the input is integrated.
a a f
Hence system is unstable. − 0
−2 t 2 2
For input, x (t) output y (t ) = ∫ x(t ) dt
−∞
Figure 1

−2 t Output Y ( f ) = X ( f ) H ( f ) as shown in Figure 2


For x(t − t0 ), output = ∫
t =−∞
x(t − t0 ) dt
y(f )
−2 t − t0

= ∫
t1 =−∞
x(t1 ) dt1 (1)
K
t1 = t − t0
where 
−2 ( t − t0 )
f
but y (t − t0 ) = ∫
−∞
x(t ) dt (2) −
g
2
0 g
2
As Equations (1) and (2) are not equal, system is
time-variant. Figure 2

Chapter 02.indd 15 10/31/2015 2:54:16 PM


2.16 | Signals and Systems

1 x(n) +
∴ y (t ) = K sin c(γ t ),  where  K =
  and Σ G(z) y(n)
αβ +
γ = min(α , β ).
K
Hence, the correct option is (a).
1 0. The transfer function of a linear time-invariant Y ( z) G( z)
H ( z) = = (1)
1 X ( z ) 1 − KG ( z )
system is given as G ( s ) = 2 . The steady-
s + 3s + 2 The system is stable if the poles of H(z) obtained
state value of the output of this system for a unit from
impulse input applied at time instant t = 1 will be 1 − KG ( z ) = 0 (2)
 [2008]
(a) 0 (b) 0.5 lie inside the unit circle in the z-plane.
(c) 1 (d) 2 For the given
Solution:(a) g (n) = 1δ (n − 1) + 1δ (n − 2), G ( z ) = 1z −1 + 1z −2 (3)
Transfer function of LTI system From Equations (2) and (3)
1 1 1 − Kz −1 − Kz −2 = 0  or  z 2 − Kz − K = 0
G (s) = =
s + 3s + 2 ( s + 1)( s + 2)
2

K ± K2 + 4K
For the input,  x(t ) = 1δ (t − 1) Poles are given by p1 , p2 = for sta-
2
1± 5
X ( s) = 1e− s bility  p1 < 1  and  | p2 | < 1  for K = 1; p1, p2 = ,
2
Y ( s) = X (s) G ( s)
∴ Output  | p1 |, | p2 | > 1
∴ For K = 1 and 2, system is unstable.
e− s
= ∴  Options (b), (c) and (d) are eliminated for
( s + 1)( s + 2)
−1 ± j 3
Steady-state value of y(t) K = −1; p1 , p2 =, p1 , p2 = 1
2
= Lt sG ( s ) = 0 1 3
s →0 +
1
Hence, the correct option is (a). For K = ; p1 , p2 = 2 2
2 2
11. Consider the discrete-time system shown in the fol-
= 1, − 0.5, p1 = 1, p2 = 0.5
lowing figure where the impulse response of G(z)
1
is g(0) = 0, g(1) = g(2) = 1, g(3) = g(4) = … = 0 ∴ The system is stable for −1 < K < or in the
 [2007] 2
 1
range   −1, 
+
Σ G(z)
 2
+ Hence, the correct option is (a).
K 12. X ( z ) = 1 − 3 z −1 , Y ( z ) = 1 + 2 z −2  are Z-transforms of
two signals x[n], y[n],  respectively. A linear time-
This system is stable for range of values of K. invariant system has the impulse response h[n]
defined by these two signals as h[n] = x[n −1]* y[n]
 1
 −1,
(a)
2 
(b) [–1, 1] where x denotes discrete-time convolution. Then
 the output of the system of the input δ [n −1]
 1   1   [2007]
(c) − 2 , 2
 − 2 , 1 (d) −1
(a) Has Z-transform  z X ( z )Y ( z )
   
Solution: (a) (b) Equals δ [n − 2] − 3δ [n − 3] + 2δ [n − 4]
If the input is x(n) and the output is y(n) as shown -6δ[n - 5]
in the following figure, the overall transfer func- (c) Has Z-transform 1 − 3 z −1 + 2 z −2 − 6 z −3
tion, H(z) is given by (d) Does not satisfy any of the above three

Chapter 02.indd 16 10/31/2015 2:54:23 PM


Chapter 2  LTI Systems (CT and DT)  |  2.17

Solution: (b) (a) α = −β


α = β (b)
For X ( z ) = 1 − 3 z −1
α = β −(1/ 3)
α = β (1/ 3) (d)
(c)
x(n) = 1δ (n) − 3δ (n − 1)
   Solution: (a)
Delaying by 1 For  G ( z ) = α z −1 + β z −3
x(n − 1) = 1δ (n − 1) − 3δ (n − 2) jω − jω i3 jω
  G (e ) = α e + β e
For Y ( z ) = 1 + 2 z −2 The frequency response is given by G (e jω ) = e − j 2ω
y (n) = 1δ (n) + 2δ (n − 2) (α e jω + β e − jω ) = 2α cos(ω )e − j 2ω , for α = β
h(n) = x(n − 1) * y (n) Hence, the correct option is (a).
= δ (n − 1) − 3δ (n − 2) + 2δ (n − 3) − 6δ (n − 4) 1 5. A discrete real all-pass system has a pole at
z = 2 ∠300. It, therefore, [2006]
h(n) = Output of the LTI system due to the input δ (n)
1
∴  Output due to the input δ (n − 1) = h(n − 1) (a) also has a pole at  ∠30°
2
= 1δ (n − 2) − 3δ (n − 3) + 2δ (n − 4) − 6δ (n − 5)
(b) has a constant phase response over the z-plane:
Note that the following statements and properties arg H ( z ) = const
are used:
∞ (c) is stable only if it is anti-causal
X ( z) = ∑
n =−∞
x ( n) z − n (d) has a constant phase response over the unit
circle: arg H (e jΩ ) = const
g ( n) * δ ( n) = g ( n)
g (n) * δ (n − n0 ) = g (n − n0 ) Solution: (c)
For a causal stable system, all the poles should lie
g (n − n1 ) * δ (n − n2 ) = δ (n − n1 − n2 )
inside the unit circle in the z-plane. For an anti-caus-
If the input to an LTI system is shifted by units, the al stable system, all the poles should lie outside the
output is also shifted by n0  units. unit circle in the z-plane. As the pole 2∠30°  is out-
Hence, the correct option is (b). side the unit circle in the z-plane the system is stable
only if it is anti-causal.
Common Data for Questions 13 and 14:
Also note the following points:
13. A signal is processed by a causal filter with trans-
fer function G(s). For a distortion free output sig- 1. A discrete real all-pass system is characterized
nal waveform, G(s) must [2007] by its frequency response.
− jω
(a) provide zero phase shift for all frequencies    H (e ) =  Constant for all w(1)
(b) provide constant phase shift for all frequencies M  −λ * + z −1 
(c) provide linear phase shift that is proportional 2. Its H(z) is given by H ( z ) = ± π  i  (2)
i =1 1 − λ z −1
to frequency  i 
(d) provide a phase shift that is inversely propor- 3. From (2) it can be seen that for a pole at  z = λi ,
tional to frequency 1
there is a zero at  z =  (mirror symmetry)
Solution: (c) λi
For distance free output, the frequency response 4. From a given pole location, another pole loca-
of the system should satisfy both the following tion cannot be determined.
conditions:
5.  arg [ H (e jω )] ≠ Constant
1.  G ( jω ) =  Constant
Hence, the correct option is (c).
2.  ∠G ( jω ) ∝ ω
16. x[n] = 0; n < −1, n > 0, x[−1] = −1, x[0] = 2 is the
Hence, the correct option is (c). input and y[n] = 0; n < -1, n > 2, y[–1] = –1 = y[1],
14. G ( z ) = α z −1 + β z −3  is a low-pass digital filter with y[0] = 3, y[2] = –2 is the output of a discrete-time
a phase characteristic same as that of the above LTI system. The system impulse response h[n]
question if [2007] will be [2006]

Chapter 02.indd 17 10/31/2015 2:54:29 PM


2.18 | Signals and Systems

h[n] = 0; n < 0, n > 2, h[0] = 1, h[1] = h[2] = −1


(a) 18. y(n) denotes the output and x[n] denotes the input
of a discrete-time system given by the difference
(b) h[n] = 0; n < −1, n > 1, h[−1] = 1, h[0] = h[1] = 2
equation  y[n] − 0.8 y[n − 1] = x[n] + 1.25 x[n + 1]. Its
(c) h[n] = 0; n < 0, n ≥ 3, h[0] = −1, h[1] = 2, h[2] = 1 right-sided impulse response [2006]
(d) h[n] = 0; n < −2, n > 1, h[−2] = h[1] = −2, (a) causal (b) unbounded
(c) periodic (d) non-negative
h[−1] = −h[0] = 3
Solution: (d)
Solution: (a) From the given difference equation
x(n) ≠ 0 from n = –1 to n = 0
y (n) − 0.8 y (n − 1) = x(n) + 1.25 x(n + 1)
y(n) ≠ 0 from n = –1 to n = 2
Y ( z ) 1 + 1.25 z1 1 1.25
As y (n) = x(n) * h(n), h(n) ≠ 0, from n = 0, to n = 2 H ( z) = = −1
= −1
+z
from width property of convolution X ( z ) 1 − 0.8 z 1 − 0.8 z 1 − 0.8 z −1
This is satisfied only for option in (a). For right-sided impulse response, h(n) the ROC is 
Detailed analysis follows: z > 0.8.
From the given data ∴ h(n) = (0.8) n u (n) + 1.25(0.8) n +1 u (n + 1)
 x(n) = −1δ (n + 1) + 2δ (n)  and h(n) ≠ 0, n < 0
y (n) = −δ (n + 1) + 3δ (n) − 1δ (n − 1) − 2δ (n − 2)  So, it can be verified that h(n) is non-causal,
X ( z ) = − z1 + 2 bounded and not periodic.
h(n) ≤ M , M < ∞; h(n) is bounded.
Y ( z ) = − z1 + 3 − z −1 − 2 z −2
 h(n) is not periodic as it is a monotonically decay-
Y ( z ) − z1 + 3 − z −1 − 2 z −2 ing sequence.
H ( z) = =
X ( z) − z1 + 2 → h(n) is non-negative.
= 1 − z −1 − z −2 Hence, the correct option is (d).
∴ h(n) = 1δ (n) − 1δ (n − 1) − 1δ (n − 2) 19. The unit impulse response of a second order under-
h(0) = 1, h(1) = h(2) = −1 damped system starting from rest is given by
c(t ) = 12.5 e −6t sin 8t , t ≥ 0.  The steady-state value
h(n) = 0, n < 0
of the unit step response of the system is equal to
h(n) = 0, n > 2  [2004]
Hence, the correct option is (a). (a) 0 (b) 0.25
1 7. A continuous-time system is described by y(t) (c) 0.5 (d) 1.0
− x (t )
=e ,  where y(t) is the output and x(t) is the Solution: (d)
input. y(t) is bounded [2006] Impulse response
(a) only when x(t) is bounded
(b) only when x(t) is non-negative h(t ) = 12.5e −6t sin(8t ), t ≥ 0
(c) only for t ≥ 0 if x(t) ≥ 0 is bounded for t ≥ 0 8
sin(8t )u (t ) LT
→ 2
(d) even when x(t) is not bounded s + 64
Solution: (d) 8
− x ( t)
e −6t sin(8t )u (t ) →
 y (t ) = e , | x(t ) |  is always positive for positive ( s + 6) 2 + 64
as well as negative values of x(t). 100
h(t ) LT
→ H ( s ) =
∴  When x(t) is bounded, y(t) is bounded. ( s + 6) 2 + 64
Even if x(t) is not bounded If the step response is y(t),
i.e.,  x(t ) = ± ∞  for any t t

y (t ) = e −∞ = 0
y (t ) = ∫
t =−∞
h(t ) dt ,

∴  y(t) is bounded even when x(t) is not bounded. H (s) 100


Y ( s) = =
Hence, the correct option is (d). s s[( s + 6) 2 + 64]

Chapter 02.indd 18 10/31/2015 2:54:34 PM


Chapter 2  LTI Systems (CT and DT)  |  2.19

(Integration property of LT is used) 1 1  1  1


s 2 + s + = 0  or   s +   s +  = 0
100 2 18  3  6
∴ yss = Lim y (t ) = Lim [ sY ( s )] = =1
t →∞ s →0 100 1 1
∴ p1 = − , p2 = −
(Final value theorem of LT is used) 3 6
Hence, the correct option is (d). 1 1
T1
Time constants are= T2
= 3 s  and= = 6s
2 0. In the system shown in the following figure, the p1 p2
input x(t ) = sin t . In the steady-state, the response Hence, the correct option is (b).
y(t) will be [2004]
22. The transfer function of the system described by 
x(t ) s y(t )
d 2 y dy du
s+1 + = + 2u  with u as input and y as out-
dt 2 dt dt
1 1 put is [2002]
(a) sin(t − 45°) (b) sin(t + 45°)
2 2 ( s + 2) ( s + 1)
(a) 2 (b)
s +s ( s 2 + s)
sin(t + 45°)
sin(t − 45°) (d)
(c)
2 2s
Solution: (b) (c) 2 (d)
( s + s) ( s 2 + s)
s jω
Y (s) =
Given  , Y ( jω ) =
s +1 1 + jω Solution: (a)
x(t ) = sin(t ), ω = 1,
For  d 2 y dy du
Given 2 + = + 2u
dt dt dt
j
∴ Y ( j1) = Taking LT with zero initial conditions:
1+ j
1 1s 2Y ( s ) + 1sY ( s ) = sU ( s ) + 2U ( s )
Y ( j1) = ,
2 Y ( s) s + 2
∴ TF = =
∠Y ( j1) = 90° − 45° = 45° U (s) s 2 + s
∴ yss = Y ( j1) sin(t + ∠Y ( j1))
Hence, the correct option is (a).
1
= sin(t + 45°) 23. Given the relationship between the input u(t) and
2 t
the output y(t) to be y (t ) = ∫ (2 + t − τ )e −3( t −τ ) u (τ )dτ
Hence, the correct option is (b). 0

21. A control system with certain excitation is governed Y ( s)


the transfer function is [2001]
by the following mathematical equation [2003] U (s)
2e −2 s s+2
d x 1 dx 1
2
(a) (b)
+ = x = 10 + 5e −4t + 2e −5t s+3 ( s + 3) 2
dt 2 2 dt 18
The natural time constants of the response of the 2s + 5 2s + 7
(c) (d)
system are s+3 ( s + 3) 2
(a) 2 s and 5 s (b) 3 s and 6 s Solution: (a)
1 1 Relation between input, u(t) and output, y(t):
(c) 4 s and 5 s (d) s  and  s
3 6 t
Solution: (d) y (t ) = ∫ (2 + t − τ )e −3( t −τ ) u (τ ) dτ
d 2 x 1 dx 1 0
Given 2 + + x = 10 + 5e −4t + 2e −5t
dt 2 dt 18 Observe that the above integral is a convolution
The natural time constants are related to the poles operation between the inputs u(t) and the impulse
of the system, given by response of the system.

Chapter 02.indd 19 10/31/2015 2:54:41 PM


2.20 | Signals and Systems

y (t ) = [(2 + t )e −3t u (t )]* u (t )


∴ h(t ) = (2 + t )e −3t u (t )
Four-marks Questions
Use the LT pairs: 1. Match the following transfer functions and impulse
responses. [1992]
1
e −3t u (t ) →
s+3 Transfer functions
−3t 1
te u (t ) → 1 1
( s + 3)
2 (a) (b)
s ( s + 1) ( s + 1) 2
2 1 ( 2 s + 7)
H (s) = + = 1 1
s + 3 ( s + 3) 2
( s + 3) 2 (c) (d)
s ( s + 1) + 1 s 2
+1
Y (s)
TF =
U (s) Impulse responses
2s + 7 (P)
= H (s) = h(t )
( s + 3) 2
Hence, the correct option is (d).
t
24. The unit impulse response of a system is given as 0
c(t ) = −4e − t + 6e −2t. The step response of the same
system for t ≥ 0 is equal to [1996] h(t )
(Q)

−3e −2t + 4e − t − 1
−3e −2t − 4e − t + 1 (b)
(a)
−2 t −t
−3e − 4e − 1 (d)
(c) 3e −2t + 4e − t − 1
t
0
Solution: (b)
Unit impulse response, h(t )
(R)
−t −2 t
h(t ) = −4e + 6e , t ≥ 0
t
0 t
Unit step response, s (t ) = ∫ h(t ) dt
−∞

s (t ) = 0, t ≤ 0 h(t )
(S)
t
= ∫ (−4e − t + 6e −2t ) dt , t ≥ 0
0
0 t
−2 t t
= 4e − t − 3e , t≥0
0
−t −2 t
s (t ) = 4e − 3e − 1, t ≥ 0
Solution:
Hence, the correct option is (b). Matching: (a, Q), (b, P), (c, S), (d, R)

Chapter 02.indd 20 10/31/2015 2:54:44 PM


Chapter 2  LTI Systems (CT and DT)  |  2.21

H(s) h(t) Sketch of h(t)

1 1 1 (1− e − t )u (t ) (Q)  h(t )


(a)  = −
s ( s + 1) s s + 1

t
0

1 te − t u (t ) (P)  h(t )
(b) 
( s + 1) 2

t
0

1 1 2  3  (S)  h(t )
(c)  = e − t / 2 sin  t 
s ( s + 1) + 1 s 2 + s + 1 3  2 
2 3 0 t

= 3 2
2
1  3
2

 s +  + 
 2   2 

1 sin t (R)  h(t )


(d) 
s +1
2

0 t

Chapter 02.indd 21 10/31/2015 2:54:46 PM


2.22 | Signals and Systems

2. A first order system is initially at rest an excited by


Five-marks Questions a step input at time t = 0. Its output becomes 1.1 V
is in 4 seconds and eventually reaches a steady
1. A single input single output system with y output state value of 2 V. Determine its time. [1993]
and u as input, is described by Solution:
d2y dy du If IV = initial value = 0; FV = Final value = 2 and
+ 2 + 10 y = 5 − 3u
dt 2 dt dt τ = Time constant, the general formula for step
­response of 1st order system is given below
For the above system find an input u(t), with zero
initial condition, that produces the same output as y(t) = IV + (FV - IV)(1 - e -t/τ)
with no input and with the initial conditions.[2001]
y(t) = 2(1 - e -t/τ)
Solution:
We k now that It is given that y(4) = 1.1 therefore,
d y
2
dy du
+ 2 + 10 y = 5 − 3u (1) 2(1 − e −4 /τ ) = 1.1, 1 − e −4 /τ = 0.55, e −4 /τ = 0.45
dt 2 dt dt
1 20 4 20
Taking LT with zero initial conditions: e 4 /τ = = , = log e = 0.7985
0.45 9 τ 9
s 2Y ( s ) + 2 sY ( s ) + 10Y ( s ) = 5sU ( s ) − 3U ( s )
4
(5s − 3) τ= = 5 sec.
∴ Y (s) = 2 U (s) (2) 0.7985
( s + 2 s + 10)
3. The impulse response of a network is h(t) = for
With no input but with initial conditions y(0− ) = 1 0 ≤ t < 1 and zero otherwise. Sketch the impulse
dy (0−1 ) response of two such networks cascade, neglecting
and = −4 (3)
dt loading effects. [1993]
Equation (1) becomes Solution:
d2y dy Given h(t) = 1, for 0 < t < 1 = 0, otherwise, Figure 1
+ 2 + 10 y = 0 (4)
dt 2
dt
h(t )
Taking LT of Equation (2) we get
1
dy (0− )
s[ sY ( s ) − y (0− )] − + 2[ sY ( s ) − y (0− )] + 10Y ( s )
dt t
0 1
=0
Using the initial conditions in (3), we get Figure 1

s 2Y ( s ) − s + 4 + 2 sY ( s ) − 2 + 10Y ( s ) = 0 h(1) = u(t) - u(t - 1)

Y ( s )[ s 2 + 2 s + 10] = s − 2 Neglecting loading effect,


s−2 y (t ) = x(t ) * h(t ) * h(t )
Y ( s) = (5)
s + 2 s +102
= x(t ) *[h(t ) * h(t )]. (1)
If Y(s) in Equations (2) and (5) are to be same:
Overall impulse response, h′(t) is given by
(5s − 3)U ( s ) = ( s − 2)
s−2 ( s − 2) h′(t ) = h(t ) * h(t )
∴ U (s) = = 0.2 t
5s − 3 ( s − 0.6) d 

( s − 0.6 − 1.4)
= ∫  dt h(t )  * h(t ) dt
t =−∞  
 1.4 
= 0.2 = 0.2 1 −  t
( s − 0.6)  s − 0.6 
∴ u (t ) = 0.2[δ (t ) − 1.4e0.6 t u (t )]
h′(t ) = ∫
t =−∞
g (t ) dt (2)

Chapter 02.indd 22 10/31/2015 2:54:49 PM


Chapter 2  LTI Systems (CT and DT)  |  2.23

d  Using Equations (2) and (3) and Figure (4), h′(t) is


where g (t ) =  h(t )  * h(t ) sketched in Figure 4
 dt 
h′(t )
= [δ (t ) − δ (t − 1)]* h(t )
1
= h(t ) *1δ (t ) − h(t ) * δ (t − 1),
t
Using Figure 2 0 1 2
d Figure 4
h(t ) = d(t ) − d(t − 1)
d(t )
1 h′(t ) = 1t , 0 ≤ t ≤1
1 = −1(t − 2), 1 ≤ t ≤ 2
t
0
= 0, otherwise
−1
Figure 2

g(t) = h(t) - h(t - 1) Figure 3 (3)


g(t )

t
0 1 2

−1

Figure 3

Chapter 02.indd 23 10/31/2015 2:54:50 PM


Chapter 3
Periodic Signal:
Fourier Series

(a) a0 and bn, n = 1, 3, 5, … ∞


One-mark Questions (b) a0 and an, n = 1, 2, 3, … ∞
(c) a0, an and bn, n = 1, 2, 3, … ∞
1. For a periodic square wave, which one of the fol- (d) a0 and an, n = 1, 3, 5, … ∞
lowing statements is TRUE? [2014-S1]
Solution: (d)
(a) The Fourier series coefficients do not exist
Consider the periodic function f (t) shown in Fig-
(b) The Fourier series coefficients exist but the re-
ure 1 given below:
construction converges at no point
(c) The Fourier series coefficients exist and the
f (t )
reconstruction converges at most points
(d) The Fourier series coefficients exist and the A
reconstruction converges at every points
Solution: (c)
t
The Fourier series coefficients exist and the recon- −T0 0 T0 T0
struction converges at most points for a periodic 2
square wave.
f 1(t)
Hence, the correct option is (c).

2. The Fourier series expansion f (t ) = a0 + ∑ an A
n =1 2
cos nωt + bn sin nωt of the periodic signal shown
below will contain the following non-zero terms: t
0
 [2011] A

2
f (t )
Observation 1: From the given figure, average

A value of periodic function
T0
1 A
0 T0
t a0 =
T0 ∫ f (t ) dt = 2 ,
0
a0 ≠ 0

Chapter 03.indd 24 11/13/2015 2:02:35 PM


Chapter 3  Periodic Signal: Fourier Series  |  2.25

Observation 2: From the above figure, periodic Solution: (a)


function has even symmetry also, From the given figure we observe that the given
i.e.,  f (t) = f (-t). waveform f (t) is half wave symmetric or its has
 T
∴  Sine terms are absent in f (t) rotation symmetry, i.e., f (t) = -f  t −  .
 2
Observation 3: The function f1(t) = [  f (t) - a0] has The waveform over any half period is a replica of
half-wave symmetry or odd harmonic symmetry: the adjacent waveform but with opposite values.
In the given waveform, even harmonics are absent
 T 
i.e., f 1 (t ) = − f1  t − 0 . and only odd harmonics are present.
 2 
Amplitude of the second harmonic (even harmonic)
∴  Even harmonics are absent. component is zero.
From observations (2) and (3) above, Hence, the correct option is (a).
bn = 0, for all values of n 5. x(t) is a real valued function of a real variable with
period T. It’s trigonometric. Fourier series expan-
an = 0, for even values of n 2π ( 2k )
sion contains no terms of frequency π = ;
an ≠ 0, for n = 1, 3, 5, … T
Hence, the correct option is (d). k = 1, 2, …. Also, no sine terms are present. Then,
x(t) satisfies the equation [2006]
 π
3. The period of the signal x(t ) = 8 sin  0.8πt +  is x(t ) = − x(t − T )
(a)
 4
 [2010] x(t ) = x(T − t ) = − x(−t )
(b)
(a) 0.4 πs (b) 0.8 πs
 T
(c) 1.25 s (d) 2.5 s (c) x(t ) = x(T − t ) = − x  t − 
 2
Solution: (d)
 T
The given signal is (d) x(t ) = x(t − T ) = x  t − 
 2
π Solution: (c)

x(t ) = 8 sin  0.8πt +  . Period of x(t ) = T
 4 1
Fundamental frequency (1st harmonic), f 0 =
2 T
The angular frequency of the signal is and angular frequency ω0 = , nω0 = nth harmonic.
T
ω0 = 0.8 π r/s, Since, the signal contains no terms of frequency
2
ω= 2 K; K = 1, 2, … or, ω = 2ω0, 4ω0, … (even
2π T
T0 = = 2.5 sec. harmonics), given signal is half-wave symmetric,
ω0
therefore,
Hence, the correct option is (d).   T 
x(t ) = − x t −    (1)
4. The second harmonic component of the periodic   2 
waveform given in the figure has an amplitude of The signal is even because no sine terms (odd
 [2010] functions) are present in the signal, therefore,
f (t ) x(t ) = x(−t ) (2)
We known that x(t) is periodic with period, T there-
1
fore Equation (2) becomes
0 t
T/2 T x(t ) = x(t − T ) = x[−(t − T )] = x(T − t ) (3)
−1
From Equations (1) and (3)
  T 
(a) 0 (b) 1 x(t ) = x(T − t ) = − x t −   
2   2 
(c) (d) 5
π Hence, the correct option is (c).

Chapter 03.indd 25 11/13/2015 2:02:39 PM


2.26 | Signals and Systems

6. The rms value of the voltage v(t ) = 3 + 4 cos(3t ) is Hence FS contains only cosine terms odd harmonics.
 [2005] Note that this restriction is satisfied only the FS in
(a) 17 V (b) 5 V option (c).
(c) 7 V (d) (3 + 2 2 ) V Hence, the correct option is (c).
Solution: (a) 8. What is the rms value of the voltage waveform
The rms voltage is given as shown in the figure? [2002]
v(t )
v(t ) = 3 + 4 cos(3t ).

Assuming that the peak value of cosine term as 4 V,


+100 V
2
 4 
MSV = 32 +   = 9 + 8 = 17. 0 2p
t
p 2p
 2 3 3
p
−100 V
rms value = 17.
Hence, the correct option is (a). 200 100
(a) (b)
7. Fourier series for the waveform, f (t) shown in the πV πV
following figure is [2002] (c) 200 V (d) 100 V
f (t)
Solution: (d)
For the given rectangular waveform with period,
1
T = π and peak values = ±100
−1 1 2 3 π
t 1 2
π ∫0
0 MSV = v (t ) dt
−1

1 
π /3 2π / 3 π
8  1 1 
(a) 2 sin(πt ) + sin(3πt ) + sin(5πt ) + 
= 
π ∫ (100) 2 + ∫ (100) 2 + ∫ (100) 2 

π  9 25  0 π /3 2π / 3

1
8  1 1  = [(100) 2 π] = (100) 2 .
(b) 2 sin(πt ) − cos(3πt ) + sin(5πt ) +  π
π  9 25  rms = MSV
8  1 1  ∴ RMSV = 100 V.
(c) 2 cos( πt ) + cos(3πt ) + cos(5πt ) + 
π  9 25  Note that the DC value of v(t) is also 100 V.
8  1 1  Hence, the correct option is (d).
(d) 2 cos(πt ) − sin(3πt ) + sin(5πt ) + 
π  9 25  9. A periodic rectangular signal, x(t) has the wave-
form shown in the following figure. Frequency of
Solution: (c)
the fifth harmonic of its spectrum is [1996]
For the given periodic waveform, f (t), peak to peak
2 x(t )
value = 2, T0 = 2, ω0 = = π.
T0
1. It has DC value = 0 t (ms)
−4 −2 0 2 4
2. It is even, f (t) = f (-t), hence no sine terms in its FS.
3. It is halfwave symmetric,

 2π  (a) 40 Hz (b) 200 Hz


f (t ) = − f  t −  (c) 250 Hz (d) 1250 Hz
 T0 
Solution: (d)
∴ Even harmonics are absent, i.e., only harmon-
 For the given square wave
ics are present Period = T0 = 4 ms

Chapter 03.indd 26 11/13/2015 2:02:41 PM


Chapter 3  Periodic Signal: Fourier Series  |  2.27

1 103 (a) x(t) has finite energy because only finitely


(1st harmonic)1 f=
0 = = 250 Hz. many coefficients are non-zero
T0 4
(b) x(t) has zero average value because it is
5th harmonic = 5 × 250 = 1250 Hz. periodic
Hence, the correct option is (d). (c) The imaginary part of x(t) is constant
10. The rms value of the periodic waveform e(t), shown (d) The real part of x(t) is even
in the following figure is [1995] Solution: (c)
e(t ) We know that a periodic signal x(t) has always in-
finite energy, that is,
+A


2
T/2 T
t x(t ) dt = ∞.
−A t =−∞

Therefore, option (a) is not true.


3 2 We know that the average value a0 of a periodic
(a) A (b) A signal need not be zero.
2 3
a0 = j2 as given in this question.
1
2A
(c) A (d) Therefore, option (b) is not true.
3
Also the complex value of a0 indicates that x(t)
Solution: (b) should be complex.
For the given periodic waveform
Let x(t) = xR(t) + jXI (t) where XR(t) and XI (t) are the
2A T real and imaginary parts of x(t).
e(t ) = t, 0 < t < T0 T
T 2 1 1 0
T
a0 =
T0 ∫ xR (t ) dt + T0 ∫0
j xI (t ) dt
= − A, <t <T 0
2 For a0 to be purely imaginary = j2, the first integral
where T is the period should be zero and the second integral should not
be zero.
1 T
T /2
4 A2 2 That is; xR(t) should be either 0 or not odd function.
MSV = 
T ∫
0 T 2
t dt + A2 
2 Therefore, option (d) is not true.
1  4 A 2T 3 T  A2 A2 2 2 and xI(t) should be a constant or an even function
=  2 + A2  = + = A. so that the integral is not zero.
T T × 3× 8 2 6 2 3
Therefore, option (c) is true.
rms = MSV
Hence, the correct option is (c).
2 2. Let x(t) be a periodic signal with time period
RMSV = A
3 T. Let y(t) = x(t - t0) + x(t + t0) for some t0. The
Fourier series coefficients of y(t) are denoted by bk.
Hence, the correct option is (b). If bk = 0 for all odd k, then t0 can be equal to
 [2008]
T T
Two-marks Questions (a) (b)
8 4
T
1. The Fourier series coefficients, of a periodic signal (c) (d) 2T
2
x(t), expressed as x(t ) = ∑ k =−∞ ak e j 2πkt/T are given

Solution: (b)
by a-2 = 2 - j1; a-1 = 0.5 + j0.2; a0 = j2; a1 = 0.5 Consider the kth harmonic component of x(t) be
- j0.2; a2 = 2 + j1; and ak = 0; for k > 2. Which ak and the kth harmonic component of the given
one of the following is TRUE? [2009] y(t) = x(t - t0) + x(t + t0) be bk.

Chapter 03.indd 27 11/13/2015 2:02:44 PM


2.28 | Signals and Systems

Now using the time shifting property we get 2π


Fundamental frequency of signal = ω0 =
2T

bk = ak e − jkω0t0 + ak e jkω0t0 , π  T
=  , x1 (t ) = x  t +  shown in Figure 2
T

2π  4
where, ω0 = = 2ak cos( kω0 t0 ), bk = 0 for all odd x 1(t )
k if T
1
π 3π 5π
k ω0 t 0 = , , ….
2 2 2 t
−T −T 0 T T
π
ω0 t 0 = 2 2
2 −1
 2π  π Figure 2
  t0 = Therefore, the signal possesses even symmetry
 T  2
and also half-wave symmetry.
T
∴ t0 = ∴  Fundamental Fourier term of
4
Hence, the correct option is (b). 2× M π 
x1 (t ) = cos  t ,
3. A signal x(t) is given by π T 

 T 3T where M = peak to peak value of x1(t) = 2


1, − 4 < t ≤ 4   T 
 As x(t ) = x1  t −   
 3T 7T   4 
x(t ) = −1, <t ≤ ,
4 4 Fundamental component of

− x(t + T ) 4  π  T  4 π π
 x(t ) =
cos   t −   = cos  t − 
 π  T  4  π T 4
which among the following gives the fundamental Hence, the correct option is (a).
Fourier term of x(t)? [2007] 4. The Fourier series for the function f (x) = sin2x is
4  πt π  π  πt π   [2005]
(a) cos  −  (b) cos  + 
π  T 4 4  2T 4  (a) sin x + sin 2 x (b) 1 − cos 2 x
(c) sin 2 x + cos 2 x (d) 0.5 − 0.5 cos 2 x
4  πt π  π  πt π 
(c) sin  −  (d) sin  +  Solution: (d)
π  T 4 4  2T 4 
f ( x) = sin 2 ( x) = 0.5 − 0.5 cos(2 x)
Solution: (a)
The signal x(t) is a periodic waveform with period The FS contains DC term a0 = 0.5 and a cosine
T0 = 2T. term with ω0 = 2 rad/sec and amplitude a1 = -0.5.
The signal also satisfies half-wave symmetry that is Hence, the correct option is (d).
5. For the triangular waveform shown in the figure,
 2T 
x(t ) = − x  t ±  = − x(t ± T ) the rms value of the voltage is equal to [2005]
 2  v (t )

As shown below in Figure 1.


1
x(t )
0 T T 3T 2T
1 2 2

1 1
(a) (b)
−9T −5T −T 0 3T 7T 6 3
4 4 4 4 4
−1 1 2
(c) (d)
Figure 1 3 3

Chapter 03.indd 28 11/13/2015 2:02:48 PM


Chapter 3  Periodic Signal: Fourier Series  |  2.29

Solution: (a) 7. The rms value of the periodic waveform g in the


2 T T following figure is [2004]
v(t ) = t , 0 ≤ t < = 0, <t ≤T
T 2 2
2
e(t )
T /2
1 2 
MSV =
T ∫t =0  T t  dt 6A

T /2 0 t
T/2 T
4 t3 4 T3 1
= 3 = 3 = −6 A
T 3 0
3T 8 6
1
RMSV =
6 (a)
2 6 A (b)
6 2A

Hence, the correct option is (a). 4


(c) A (d) 1.5 A
6. The rms value of the resultant current in a wire 3
which carries a DC current of 10 A and sinusoidal Solution: (a)
alternating current of peak value of A is[2004] 12 T T
Given v(t ) = − t , 0 < t < = 6, <t <T
(a) 14.1 A (b) 17.3 A T 2 2
(c) 22.4 A (d) 30.0 A 1 144
T /2
36T 
MSV = ∫t dt +
2
Solution: (b)  
T  T2 0
2 
Given DC current = 10 A, and sinusoid with peak
value = 20 A 1  144 T / 2 
=  2 t3 + 18 T 
T  3T 0

i (t ) = 10 + 20 sin ω0 t 1  144 T 3 
2 =  + 18 T 
20 T  3T 2 8 
MSV = 102 + = 300
2 = 6 + 18 = 24.
RMSV = 300 = 10 3 = 17.3 A
RMSV = 24 = 2 6 A.
Hence, the correct option is (b). Hence, the correct option is (a).

Chapter 03.indd 29 11/13/2015 2:02:49 PM


2.30 | Signals and Systems

3 5

Five-marks Questions b1 = 0.4  ∫ sin(0.4πt )dt − ∫ sin(0.4πt )dt 
0 3 
1. Consider the voltage waveform V, shown in the fol­ 0.4 
− cos(0.4πt ) 0 + cos(0.4πt ) 3 
3 5
=
lowing figure. Find [2001] 0.4π  
1
v = [− cos(1.2π) + cos(0) + cos(0) − cos(1.2π)]
π
1V
1 2
0 t (ms) = [2 − 2 cos(1.2π)] = [1 − cos(1.2π)] (4)
3 5 8 10 13 π π
−1 V
From Equations (2), (3) and (4)
2
(a) The DC component of V, A1 = sin 2 (1.2 π ) + 1 + cos 2 (1.2 π ) − 2 cos(1.2 π )
π
(b) The amplitude of the fundamental component
2 2 2 2
of V, and = 1 + cos(36°) = 1.81
(c) The rms value of the AC part of V π π
= 1.21 V
Solution: (b) 5
For the given periodic waveform v(t): T = 5 ms, 1 2
5 ∫0
(c) MSV of v(t ) = v (t ) dt
f 0 = 200 Hz = 0.2 KHz, ω0 = 0.4 πk r/s
1
v(t ) = 1, 0 < t < 3 = −1, 3 < t < 5 = [(1× 3) + (1× 2)] = 1 V 2
  5
MSV of DC component = ( 0.2 ) = 0.04 V 2
2
(a) The DC component of voltage waveform can
be calculated as MSV of AC part of v(t ) = 1 − 0.04 = 0.96 V 2
1 of v(t )
rms value of AC part= =
0.96 V 0.98 V
v(t ) = a0 = [(1× 3) − (1× 2)] = 0.2 V
5 Hence, the correct option is (b).

(b) Fundamental component 2. Compute the amplitude of the fundamental com-
ponent of the waveform given in the figure.[1997]
= A1 cos(1ω0 t + φ ) x(t )

= a1 cos(1ω0 t ) + b1 sin(1ω0 t ) (1) 2


1
A1 = Amplitude of the 1st harmonic (1ω0 ) or fun-
 p 2p
0 t
damental component = a12 + b12 (2) p 2p 4p 5p
3 3 3 3
−1
Now, v(t) is neither even nor odd. It is also not half-
−2
wave symmetric.
T Solution:
2
T ∫0
∴ a1 = v(t ) cos(ω0 t ) dt Figure 1 given below shows the periodic wave-
form, x(t)
x(t )
2 
3 5
a1 =  ∫ cos(0.4πt ) dt − ∫ cos(0.4πt ) dt  2
5 0 3 
1
0.4  p 2p
0 t
sin(0.4 πt ) 0 − sin(0.4 π t ) 3 
3 5
= p 2p 4p 5p
0.4 π   3 3 3 3
−1
−2
1 2
= [sin(1.2π) + sin(1.2π)] = sin(1.2π) (3)
π π Figure 1

Chapter 03.indd 30 11/13/2015 2:02:52 PM


Chapter 3  Periodic Signal: Fourier Series  |  2.31

From the above figure we get 2  π  2π   π 


= sin   + 2 sin   − 2 sin   
T0 = 2π, π   3   3  3 
1 2
f0 = , = [2 cos(30°) − sin(60°)]
2π π
ω0 = 1 2 3 3 3
= 2 − =
Fundamental component of the waveform π 2 2  π

x1 (t ) = a1 cos(1t ) + b1 sin(1t ) (1) a1 =


3
π
= A1 cos[1t + φ1 ] (2)
4  
π/3 2π/3

2π  ∫0 ∫
A1 = Amplitude of the fundamental component b1 =  sin(t ) dt + 2 sin(t ) dt 
π/3 
= a12 + b12 (3) 2
= − cos(t ) 0 + 2 cos(t ) π / 3 
π/3 2π/3

b  π  
φ1 = − tan −1  1  (4)
2  π  2π  π
 a1  = −  cos   − 1 + 2 cos   − 2 cos 
Therefore, x(t) has half-wave symmetry. π 3  3  3
2 π  2 π 
4
T /2 = −  − cos − 1 + 2 cos  
π 3  3 
∴ a1 =
T ∫ x(t ) cos(1t )dt
0 2 1 1 3
= − − − 1 − 2 ×  =
4
π
π 2 2 π
2π ∫0
a1 = x(t ) cos(1t ) dt
From Equation (3),
2 
π/3 2π/3

π  ∫0 ∫π / 3 cos(t ) dt 
=  1 cos(t ) dt + 2 3 25 28
A1 = + = = 1.68
2 π2 π2 π
sin(t ) 0 + 2 sin(t ) π / 3 
π/3 2π/3
=
π  −1  5 
From Equation (4), φ1 = − tan  .
 3

Chapter 03.indd 31 11/13/2015 2:02:54 PM


Chapter 4
Signal Fourier Transform
If f (t) is even f (-t) = f (t) then
Two-marks Questions g (t ) = 2π f (t )
1. Let f (t) be a continuous time signal and F(ω) be its g (t ) ∝ f (t )

Fourier transform defined by f (t ) = ∫ f (t )e − jωt dt. Hence, the correct option is (b).
Define g(t) −∞ 2. A 10 kHz even-symmetric square wave passed
∞ through band-pass filter with centre frequency at
∫ F (u )e
− jut
g (t ) = du. 30 kHz and 3 dB pass-band of 30 kHz. The filter
−∞ output is [2014-S1]
What is the relationship between f (t) and g(t)? (a) a highly attenuated square wave at kHz.
 [2014-S1] (b) nearly zero.
(a) g(t) would always to be proportional to f (t). (c) a nearly perfect cosine wave at 30 kHz.
(b) g(t) would be proportional to f (t) if f (t) is an (d) a nearly perfect sine wave at 30 kHz.
even function. Solution: (c)
(c) g(t) would be proportional to f (t) only f (t) is a Centre frequency = 30 kHz
sinusoidal function. Pass-band = 3 dB
(d) g(t) would never proportional to f (t).
Consider the Figure given below:
Solution: (b)
Continuous time signal = f (t) A x(t )

Fourier transform of signal = F(ω)


∞ t
F (ω ) = ∫
−∞
f (t )e − jωt dt

1
∞ −A
f (t ) =
2π ∫
−∞
F (ω )e jωt d ω
h(t )

1
∫ F (ω )e
jω t
2π f (t ) = dω
  −∞

f
∫ F (ω )e
jω t
2π f (−t ) = d ω and given −30 kHz 30 kHz
−∞

6 kHz
∫ F (u )e
− jut
g (t ) = du
−∞ The given symmetrical wave exhibits even and
 g (t ) = 2π f (−t ) half-wave symmetry an exists for odd n Cosine

Chapter 04.indd 32 10/31/2015 3:52:55 PM


Chapter 4  Signal Fourier Transform  |  2.33

terms for odd harmonic fundamental frequency of x(t )


square wave is 10 kHz.
1
It contains, 10 kHz, 30 kHz, 50 kHz, …, but BPF
cut-off frequency is 30 kHz.
∴  We will retain 30 kHz cosine-wave.
t
−1 0 1
Hence, the correct option is (c).

3. The Fourier transform of a signal h(t) is H(t) = (2 (a) 2 (b) 2π
cos ω)(sin2ω)/ω. The value of h(0) is
[2011] (c) 4 (d) 2π
1 1 Solution: (d)
(a) (b) Consider the figure given below:
4 2
(c) 1 (d) 2 x(t)

Solution: (c) i
Given as
Fourier transform of a signal h(t) is
(2 cos ω )(sin 2ω ) −1
t
H (t ) = . 0 1
ω
As per the Parseval’s theorem for FT pair, If x(t)
(2 cos ω )(sin 2ω ) → X(ω), X(  f  )
H ( jω ) = .
ω Energy of the signal, x(t) is
Consider the Figure given below: ∞ ∞ ∞
1
∫ ∫ ∫
2 2 2
x(t ) E= x(t ) dt = X (ω ) d ω = X ( f ) df
−∞
2π ω =−∞ f =−∞

1
= Area under x2(t) = 2 for the given signal, x(t)

shown in Figure 1
∞ ∞

∫ ∫
t 2
X (ω ) d ω = 2π
2
x(t ) dt = 2 π × 2 = 4 π
−1 0 1
−∞ −∞

2 sin ω  2 sin(ω )  Hence, the correct option is (d).


↔ x(ω ) = = 2 cos 2 ω  
ω  (ω )   1
5. Let x(t ) = rect  t −  (where rect (x) = 1 for
= [1 + cos(2ω )] X ( jω ),  2
1 1
− ≤ x ≤ and zero otherwise). Then if sin c(x) =
2 2
2 sin(ω )  e j 2ω + e − j 2ω  sin(π x)
X ( jω ) = = X ( jω ) + X ( jω )   , the Fourier transform of x(t) + x(-t) will
(ω )  2 

πx
x(t − 2) + x(t + 2) be given by [2008]
h(t ) = x(t ) +
2  ω 
sin c 
(a) 
1  2π 
h(0) = x(0) + [ x(−2) + x(2)] = x(0) = 1
2  ω 
2 sin c 
(b) 
Hence, the correct option is (c).  2π 
4. x(t) is a positive rectangular pulse from t = -1 to  ω  ω 
t = +1 with unit height as shown in the figure. The 2 sin c 
(c)  cos  
∞  2π  2

2
value of X (ω ) d ω {where x(ω) is the Fourier  ω  ω 
−∞ sin c 
(d)  cos  
transform of x(t)} is [2010]  2π  2

Chapter 04.indd 33 10/31/2015 3:52:58 PM


2.34 | Signals and Systems

Solution: (b) FT [ x(t ) + x(−t )] = X (ω ) + X (−ω )


Consider the figure given below: − jω jω
 ω   −ω   ω 
rect (t ) =e 2
⋅ sin c   + e 2
⋅ sin c   sin c  
 2π   2π   2π 
 −ω 
= sin c   (even function )
 2π 
t
−1 0 1  ω   − 2jω jω

= sin c   ⋅ e + e 
2
2 2
 2π   
ω   ω  ω 
←
FT
→ Sa   = sin c  ω 
 = 2 cos   ⋅ sin c  
2  2π  2  2π 
 1
x(t ) = rect  t − 
 2 Hence, the correct option is (b).
− jω
ω 
X (ω ) = e 2
⋅ sin c  
   2π 

Chapter 04.indd 34 10/31/2015 3:52:59 PM


Chapter 5
Sampling Theorem
|U(j w)| or |U(f )|
One-mark Question M

1. The frequency spectrum of a signal is shown in the


following figure. If this signal is ideally sampled at f(in kHz)
0 1
intervals of 1 ms, then the frequency spectrum of
the sampled signal will be [2007] Figure 1

|U( j w)| Sampling interval, Ts = 1 ms, fs = 1 kHz, fs = fh.


Therefore aliasing or overlap of the adjacent spec-
w = 2pf tra occurs in the sampled spectrum because, fs < 2fh.
The sampled spectrum,
f(in kHz) ∞
0 1
U * ( jω ) = U * ( f ) = f s ∑ U ( f − nf ).
n =−∞
s

|U ∗(jw)|

(a) as shown in Figure 2.


f |U ∗(j w)| = |U ∗(f )|
0 1
fsM

(b) f(in kHz)


−2 −1 0 1 2 3
f
0
Figure 2

The resultant spectrum, U * ( jw) is constant for all


(c) f as shown in Figure 3 which is the same figure
0 1
f given in option (b)
|U ∗(j w)|
fsM
(d)
f
0 1 f
0
Solution: (b)
Figure 3
Highest frequency of the input signal,  fh = 1 kHz
as shown in its spectrum of Figure 1. Hence, the correct option is (b).

Chapter 05.indd 35 11/2/2015 11:54:25 AM


2.36 | Signals and Systems

Y ( Z ) 1 1 − Z − N 
Two-marks Questions =  
X ( Z ) N  1 − Z −1 
1. An input signal x(t) = 2 + 5 sin(100pt) is sampled N =8
with a sampling frequency of 400 Hz and applied to 1 1 − Z −8 
the system whose transfer function is represented Y (Z ) =   ⋅ X (Z )
8  1 − Z −1 
Y ( z ) 1 1 − z − N 
by =   where, N ­represents the Final value theorem
X ( z ) N  1 − z −1 
number of samples per cycle. The output y(n) of y (∞) = Lt (1 − Z −1 )Y ( Z )
z →1
the system under steady state is [2014-S2]
(a) 0 (b) 1 1 1 − Z −8 
y (∞) = Lt (1 − Z −1 )   X (Z )
(c) 2 (d) 5 Z →1 8  1 − Z −1 
Solution: (a) 1 − Z −8
The output of the sampling process is x(nTs) = 2 y (∞) = Lt X (Z )
Z →1 8
+ 5 sin(100 × p × n × Ts) y (∞) = 0.
1 Hence, the correct option is (a).
Ts =
400 2. A band-limited signal with a maximum frequency
 1  of 5 kHz is to be sampled. According to the sam-
x(n) = 2 + 5 sin 100 × π × n × 
 400  pling theorem, the sampling frequency in kHz
which is not valid is [2013]
 nπ  (a) 5 (b) 12
x(n) = 2 + 5 sin  ,
 4  (c) 15 (d) 20
π Solution: (a)
ω0 = According to sampling theorem.
4
Sampling frequency ( fs) ≥ 2 × maximum signal
2π 2π frequency
No = m= m
ω0 π ⇒  fs ≥ 2fm, Given, fm = 5 kHz
4
∴  fs ≥ 2 × 5 kHz  ⇒  fs ≥ 10 kHz
No = 8m
No = 8m is the number of samples per cycle Hence, the correct option is (a).

Chapter 05.indd 36 11/2/2015 11:54:26 AM


Chapter 6
Signal: Laplace Transform
Solution: (b)
One-mark Questions Unilateral Laplace transform of f (t) will be
1
1. Which one of the following statements is NOT f (t ) LT
→ F ( s ) =
TRUE for a continuous time causal and stable LTI s2 + s + 1
system? [2013] Unilateral Laplace transform of tf (t) will be
(a) All the poles of the system must lie on the left
d 2s + 1
side of the jw -axis. t f (t ) LT
→ − F (s) = 2
(b) Zeros of the system can lie anywhere in the ds ( s + s + 1) 2
s-plane. Hence, the correct option is (d).
(c) All the poles must lie within | s | = 1 2. Let Y (s) be the Laplace transformation of the
(d)  All the roots of the characteristic equation function y (t), then the final value of the function is
must be located on the left side of the jw -axis.  [2002]
Solution: (c) (a) lim Y ( s ) (b) lim Y ( s )
s →0 s →∞
For a continuous time causal and stable LTI sys-
tem, all the poles of the system must lie on the left lim sY ( s ) (d)
(c) lim sY ( s )
s →0 s →∞
side of the jw-axis and zeros of the system can lie Solution: (c)
anywhere in the s-plane. Also all the roots of the Final value theorem of LT:
characteristic equation must be located on the left
Final value of y (t ) = lim y (t )
side of the jw-axis. t →∞

Thus options (a), (b) and (d) are true. = lim[ sY ( s )],
s →0
For both causality and stability is Laplace domain the function is Y (s) has poles strictly in the left half
all poles must have negative real parts, i.e., left of the s-plane.
half of the s-plane.
Hence, the correct option is (c).
Hence, the correct option is (c).
2. The Laplace transformation of f (t) is F(s). Given
2. The unilateral Laplace transform of f (t) is ω
1 F (s) = − 2 , the final value of f (t) is [1995]
. The unilateral Laplace transform of s + ω2
s2 + s + 1 (a) infinity (b) zero
tf (t) is [2012] (c) one (d) none of these
s 2s + 1 Solution: (d)
(a) − 2 (b) − 2
( s + s + 1) 2
( s + s + 1) 2 ω
F (s) = 2 ,
s 2s + 1 s + ω2
(c) 2 (d)
( s + s + 1) 2
( s + s + 1) 2
2 ∴ f (t ) = sin(ωt )u (t )

Chapter 06.indd 37 10/31/2015 4:01:56 PM


2.38 | Signals and Systems

The Final value of f (t), i.e.,  Lt f (t ) is indetermi- Where, δ (t) is the delta function. Assuming zero
t →∞
nate (not defined) as f (t) may have any value be- initial condition and denoting the unit step func-
tween -1 and +1. Note that the final value theorem tion by u (t), y (t) can be of the form
of LT is not applicable because F(s) has poles on (a) et (b) e-t
the imaginary axis in the s-plane and not strictly in (c) e u (t) (d)
t
e-tu(t)
the left half of the s-plane. Solution: (d)
Hence, the correct option is (d). The given differential equation is

dy (t )
+ y (t ) = δ (t )
dt
Two-marks Questions
In the s-domain the given differential equation
1. Let the Laplace transform of a function f (t) which becomes
exists for t > 0 be F1(s) and the Laplace transform
of its delayed version f (t - τ) be F2(s). Let F1*(s)be Y ( s )[ s + 1] = 1
the complex conjugate of F1(s) with the Laplace 1
Y (s) =
F ( s ) ⋅ F1* ( s ) s +1
variable set as s = σ + jw. If G ( s ) = 2 2
,
F1 ( s ) ∴ y (t ) = e − t u (t )
then the inverse Laplace transform of G(s) is
[2011] Hence, the correct option is (d).
t
(a) an ideal impulse δ (t) 3. The running integrator, given by y (t ) = ∫ x(t ′) dt ′
(b) an ideal delayed impulse δ (t - τ) ∞

(c) an ideal step function u (t)  [2006]


(d) an ideal delayed step function u (t - τ) (a) has no finite singularities in its double sided
Laplace transform Y(s).
Solution: (b) (b) produces a bounded output for every causal
f (t ), t > 0 → F1 ( s ) LT bounded input.
(c) produces a bounded output for every anti-
→ F2 ( s ) = e −τ s F1 ( s )
f (t − τ ), t > τ LT causal bounded input.
(d) has no finite zeros in its double sided Laplace
F2 ( s ) F1*( s ) transform Y(s).
G (s) = 2
F1 ( s ) Solution: (d)
e −τ s
F1 ( s ) F ( s )
* If x(t) is the input and y (t) is the output then the
= 1
2 Running integrator is given by
F1 ( s )
t

F1 ( s ) F1*( s ) = F1 ( s ) ,
2
y (t ) = ∫ x(t ) dt
−∞
∴ G ( s ) = e −τ s
Let the double sided laplace transform of x (t)
g (t ) = 1δ (t − τ ) be x (s).
The inverse LT of G (s) is a unit ideal impulse de-
x(t ) 
→ X ( s ), ROC = R
layed by τ.
Hence, the correct option is (b). Then the double-sided Laplace transform of y(t) is
2. A function y (t) satisfies the following differential given by
equation: [2008]
1
Y ( s) =X ( s ),
dy (t ) s
+ y (t ) = δ (t )
dt ROC = R1 = R ∩ Re( s ) > 0

Chapter 06.indd 38 10/31/2015 4:01:58 PM


Chapter 6  Signal: Laplace Transform  |  2.39

We conclude that, 1
(a)
1.  Y (s) has pole or singularity at s = 0, i.e., Y(s) s
has finite singularity.
2. y (t) is bounded for bounded causal input x(t) 1
  exp (−Ts )
(b)
= u(t). s
3. y (t) is unbounded for bounded anti-causal input
x (t) = u (-t). 1
  exp (Ts )
(c)
4. Y(s) has zero at s = ∞, i.e., Y (s) has no finite s
zero.
1
∴  Option (d) is correct.
(d)   [1 − exp(−Ts )]
s
Hence, the correct option is (d). Solution: (d)
4. For the equation x(t ) + 3 x (t ) + 2 x(t ) = 5,  the solu- The rectangular pulse of current duration, T and
tion x (t) approaches which of the following values magnitude, 1 is
as t → ∞ ?  [2005]
5 p (t ) = 1u (t ) − 1u (t − T )
(a) 0 (b) 1 1 1
2 P( s ) = − e −Ts = (1 − e −Ts )
(c) 5 (d) 10 s s s
Solution: (b) Hence, the correct option is (d).
For the given differential equation 7. The Laplace transform of (t2 - 2t)u(t - 1) is [1998]
x(t ) + 3 x (t ) + 2 x(t ) = 5,
 2 −s 2 −s
(a) e − 2e
5 s3 s
X (s) =
s ( s + 3s + 2)
2
2 −2 s 2 − s
(b) e − 2e
5 s3 s
Lim x(t ) = Lim s X ( s ) =
t →∞ s →0 2 2 −s 1 −s
(c) e − e
Hence, the correct option is (b). s3 s
(d) None of the above
5. The Laplace transform of a function f (t) is F(s)
5s 2 + 23s + 6 Solution: (c)
= .  As t → ∞,  approaches [2005] Let x (t) = (t2 - 2t) u (t - 1)
s ( s 2 + 2 s + 2)
      = (t - 1)2 u (t - 1) - u (t - 1)
(a) 3 (b) 5
17 Using the pairs and properties of LT:
(c) (d) ∞
2 1 1
Solution: (a) u (t )  → , u (t − 1)  → e− s
s s
Laplace transform of a function f (t)
1 2
5s 2 + 23s + 6 tu (t )  → 2 , t 2 u (t ) → 3,
F (s) = s s
s ( s 2 + 2 s + 2)
2
6 (t − 1) 2 u (t − 1)  → 3 e −1s
Lim f (t ) = Lim sF ( s ) = = 3 s
t →∞ s →0 2
2 1
Hence, the correct option is (a). ∴ X ( s ) = 3 e −1s − e − s
s s
6. A rectangular current pulse of duration T and mag-
nitude 1 has the Laplace transform [1999] Hence, the correct option is (c).

Chapter 06.indd 39 10/31/2015 4:02:02 PM


Chapter 7
DT Signal: Z-Transform

Two-marks Questions  X 1 (k + 1)   a a − 1  X 1 (k ) 
 X (k + 1)  =  a + 1 a   X 2 (k ) 
.
 2  
1
1. Let X ( z ) = be the z-transform of causal
1 − z −3 It has initial conditions X=
1 ( 0) X 2 (0) = 0.
1;=
signal x[n]. Then, the value of x[2] and x[3] are The pole locations of the system for a = 1 are
 [2014-S1]  [2014-S2]
(a) 0 and 0 (b) 0 and 1 (a) 1 ± j 0 (b) −1 ± j 0
(c) 1 and 0 (d) 1 and 1 (c) ±1 + j 0 (d) 0 ± j1
Solution: (b)
Solution: (a)
z-transform of causal signal x[n]
Difference equation of discrete system is
1 z3
X ( z) = = .  X 1 (k + 1)   a a − 1  X 1 (k ) 
1 − z −3 z 3 − 1  X (k + 1)  =  a + 1 .
 2   a   X 2 (k ) 
Numerator and denominator polynomials are de-
scending power of z for a causal sequence. State equation of a discrete system is
 3  X [ K + 1] = AX [ K ] + Bu[ K ].
z 3 − 1 z 1 + z −3 + z −6
 z3 −1 
______ O/P equation is
1
Y ( K ) = CX ( K ) + Du ( K ).
1 − z −3
________
State transition matrix is = Z −1[( ZI − A) −1 Z ]
z −3
z −_3 _____
____ − z −6 1 0 
−6 A= 
z 0 1
 Z 0  1 0 
X ( z ) = 1 + z −3 + z −6 +  ( ZI − A) =  − 
∞ 0 Z   0 1
X ( z) = ∑ x ( n) z −n
x(2) = 0, x(3) = 1 Z −1 0 
n =−∞ = .
 0 Z − 1
Hence, the correct option is (b).
2. A discrete system is represented by the difference Indicates the poles at Z = 1
equation Hence, the correct option is (a).

Chapter 07.indd 40 11/13/2015 12:21:08 PM


Chapter 7  DT Signal: Z-Transform  |  2.41

z
3. Given X ( z ) = with |z| > a, the residue of ∞
3n 2 n
( z − a)2 For the given x(n), X(z) = ∑ z
n −1 n =0 2 + n
X ( z ) z at z = a for n ≥ 0 will be [2008]
1
(a) n−1
a (b) a n = +1z2 + 
2
(c) n a n (d) n a n−1
1
Solution: (d) ∴ x(n) = δ (n) + 1δ (n + 2) + . (3)
2
z
For the given X ( z ) = with | z | > a From the options given, according to the definition
( z − a)2
of ZT:
zn 2
n
Let Y ( z ) = z n −1 X ( z ) = . y1 (n) =   u (n), y3 (n) = 2−|n|
( z − a)2 3
1 d y4 (n) = 1δ (n) + 3 δ (n − 2) + 2 δ (n − 4).
Residue of Y ( z ) = [( z − a ) 2 Y ( z )]z = a .
1! dz ∞
For Y2 ( z ) = ∑ (5n − n)z − ( 2 n +1) .
d
= [zn ] n =0
dz z =a = 1z −1 + 4 z −3 + 23 z −5 + 
n −1 n −1
=nz = na ∴ y2 (n) = 1δ (n − 1) + 4δ (n − 3) + 23δ (n − 5).
z =a

1 n It can be seen that Equation (2) is satisfied only


na .
=
a when y (n) = y2 (n).
Hence, the correct option is (d). Hence, the correct option is (b).
4. The discrete-time signal 5. If u(k) is the unit step and δ (k ) is the unit impulse
1
3n 2 n function, the inverse z-transform of F ( z ) =
z[n] ↔ X ( z ) = ∑ n = 0

z , z +1
2+n
for k > 0 is [2005]
where ↔ denotes a transform-pair relationship, is
 (a) (−1) k δ (k ) (b) δ (k ) − (−1) k
orthogonal to the signal [2006] (c) (−1) k u (k ) (d) u (k ) − (−1) k
n
∞ 2 Solution: (b)
y1[n] ↔ Y1 ( z ) = ∑ n = 0   z − n
(a)
1
3 Unit impulse function F ( z ) =
z +1
y2 [n] ↔ Y2 ( z ) = ∑ n = 0 (5n − n) z − ( 2 n +1)

(b) To find f (k ), k > 0 i.e., to find right sided sequence,
y3 [n] ↔ Y3 ( z ) = ∑ n =−∞ 2−|n| z − n

(c) ROC is | z |> 1, express F ( z ) as

y4 [n] ↔ Y4 ( z ) = 2 z −4 + 3 z −2 + 1
(d) z +1− z z
F ( z) = = 1− .
Solution: (b) z +1 z +1
Two discrete time signals x(n) and y(n) are said to Use ZT pairs: δ (k ) → 1,

be orthogonal, if ∑ x(n) y *(n) = 0, whose * denotes z
n =0 a k u (k ) → , | z| > |a|
complex conjugation (1) z−a
∞ ∴ f (k ) = δ (k ) − (−1) k u (k ).
\  or ∑ x(n) y(n) = 0, for real x(n) and y(n)(2)
n =0 Hence, the correct option is (b).

Chapter 07.indd 41 11/13/2015 12:21:15 PM


Chapter 8
Miscellaneous
g(t )
One-mark Question
1 2 1
1. Given f ( z ) = − . If C is a counter clock-
z +1 z + 3 t
wise path in the z-plane such that z + 1 = 1, the 0 3 5
1
value of
2π j 
C
∫ f ( z ) dz is [2012] 1. g(t) can be expressed as [2010]
g (t ) = f (2t − 3)
(a)
(a) −2 (b) −1
t 
(c) 1 (d) 2 g (t ) = f  − 3 
(b)
Solution: (c)  2 
1 2  3
Given f ( z ) = − (c) g (t ) = f  2t − 
z +1 z + 3  2
 t 3
(d) g (t ) = f  − 
1 1  1 2  2 2

2π j C
f ( z ) dz = ∫
2 π j C z + 1
dz − ∫
C
dz 
z +3  Solution: (d)
1 1 3
f t −
= ∫
2π j C z + 1
dz − 0 f (t ) 2

1 1
= f (-1) where f (z) = 1 = 1 1
t
0 3 5
t
Hence, the correct option is (c). 0 1   2 2
3
Shifting by units
2
Two-marks Questions g(t ) = f
t 3

2 2
Common Data for Questions 1 and 2:
1 1
Given f (t) and g(t) as shown below:
f (t ) t
0 3 5

1 1
Scale by units
2
t
0 1 Hence, the correct option is (d).

Chapter 08.indd 42 11/10/2015 6:03:44 PM


Chapter 8  Miscellaneous  |  2.43

2. The Laplace transform of g(t) is [2010] with corner frequency


1 1 −5 s −3 s 1
(a) (e3 s − e5 s ) (b) (e − e ) ω1 = = 1 rad/sec  (2)
s s RC
e −3 s 1 5s 3s 1 1 1
(c) (1 − e −2 s ) (d) (e − e ) H (s) = , H ( jω ) = =  (3)
s s 1+ s 1 + jω 1+ ω2
Solution: (c)
R
g (t ) = u (t − 3) − u (t − 5) + 1Ω +
V i (t ) 1F C V 0(t)
Using LT pair and property
− −
1
u (t ) 
→ Figure 1
s
e − t0 s |H(jw)|
u (t − t0 ) 

s
0 dB
e −3 s e −5 s e −3 s
∴ LT of g (t ) = G ( s ) = − = (1 − e −2 s ) −20 dB/dec
s s s
w
Hence, the correct option is (c). 0.1 1
Common Data for Questions 3 and 4: Figure 2
It is required to design an anti-aliasing filter for an
8-bit ADC. The filter is a first order RC filter with The Bode (asymptotic) magnitude plot is shown in
R = 1Ω and C = 1 F. The ADC is designed to span Figure 2 with its roll-off or slope = -20 dB/decade
a sinusoidal signal with peak to peak amplitude (4) above the corner frequency
equal to the full-scale range of the ADC. Hence, the correct option is (a).
R = 1Ω
4. What is the SNR (in dB) of the ADC? Also find the
frequency (in decades) at the filter output at which
C=1F A/D the filter attenuation just exceeds the SNR of the
ADC. [2006]
(a) 50 dB, 2 decades (b) 50 dB, 2.5 decades
3. The transfer function of the filter and its roll-off, (c) 60 dB, 2 decades (d) 60 dB, 2.5 decades
respectively are [2006] Solution: (b)
1 If the n = 8 - bit ADC is designed to span sinusoi-
(a) , − 20 dB/decade dal signal with peak to peak amplitude equal to its
(1 + RCs )
full-scale range,
(1 + RCs ) , − 40 dB/decade
(b) SNR (in dB) = 1.8 + 6n = 1.8 + 6 × 8 = 49.8
1 ≈ 50 dB (5)
(c) , − 40 dB/decade For the derivation of this formula refer to the PCM
(1 + RCs )
topic in Communication systems. From Equation
 RCs  (3) in Question 3,
(d)
  , − 20 dB/decade
 (1 + RCs )  |H(ω)|dB = -10 log(1 + ω2) = -10 log(1 + 105). At ω2
= 105 or ω = 102.5 (or 2.5 decades) ≅ -10 × 5 = -50
Solution: (a) (< -50) gain ≅ -50 dB, ∴  Attenuation = 50 dB
The transfer function of the RC low-pass filter
 ∴  The frequency at the filter output at which the
shown in Figure 1 is given by
attenuation just exceeds the SNR of the ADC given
1 Equation (5) = 2.5 decades.
H (s) = (1)
1 + RCs  Hence, the correct option is (b).

Chapter 08.indd 43 11/10/2015 6:03:46 PM


This page is intentionally left blank.

Chapter 08.indd 44 11/10/2015 6:03:46 PM


Unit 3
Electric Machines

Chapter 1:  DC Machines 3.3


Chapter 2:  Transformers 3.18
Chapter 3:  Induction Motors 3.44
Chapter 4:  Synchronous Machines 3.67

Chapter 01.indd 1 11/9/2015 6:00:59 PM


Chapter 01.indd 2
Exam Analysis
Exam Year 92 93 94 95 96 97 98 99 00 01 02 03 04 05 06 07 08 09 10 11 12 13 14
1 Mark Questions 5 2 3 5 11 – 5 4 6 7 2 4 5 5 4 4 3 2 2 3 1 2 7
2 Marks Questions – 4 – 1 1 3 4 7 7 3 3 9 13 7 5 8 10 9 5 3 3 2 10
5 Marks Questions 2 – 3 4 2 9 2 3 2 5 7 – – – – – – – – – – – –
Total Marks 7 6 6 10 14 12 11 14 15 15 12 13 18 12 9 12 13 11 7 6 4 4 17
DC Machines 2 1 – 2 2 5 2 3 5 4 3 1 2 3 1 2 2 1 2 2 1 – 2
Transformers 2 1 2 2 3 3 5 6 3 6 4 3 3 1 4 2 3 7 2 1 1 2 5
Induction Motors – 1 4 4 6 2 2 3 4 – 3 6 6 6 1 5 5 2 1 1 2 2 5
Synchronous Machines 3 3 – 2 3 2 2 2 3 5 2 3 7 2 3 3 3 1 2 2 – – 5

11/9/2015 6:00:59 PM
Chapter 1

DC Machines
(c) DC series motor.
One-mark Questions (d) DC compound motor.

1. The no-load speed of a 230 V separately excited Solution: (b)


DC motor is 1400 rpm. The armature resistance Use of 4-point starter:
drop and the brush drop are neglected. The field These starters are used in shunt motors in above
current is kept constant at rated value. The torque rated speeds to control the speed. At above rated
of the motor in Nm for an armature current of 8 A speeds the field of starter weakens and thus current
is . [2014: S3] reduces. In case of 3-point starter the holding coil
is unable to hold the plunger in ON position.
Solution:
Hence, the correct answer is (b).
Voltage, V = 230 V; Speed, N = 1400 rpm; ­Armature
current: Ia = 8 A. 3. The DC motor, which can provide zero speed regu-
At no-load speed, the angular speed of the motor is lation at full-load without any controller, is[2007]
given as (a) Series
(b) Shunt
2π 2π (c) Cumulative compound
ω= ×N = × 1400 = 146.6 rad/s.
60 60 (d) Differential compound
Power developed in motor is given by Solution: (d)
P = T ×ω N
P EI
T= = a.
ω ω
As losses are neglected then E ≅ V = 230 V.
Speed

230 × 8
Torque, T = = 12.55 Nm.
146.6
O Torque Ta
2. A 4-point starter is used to start and control the
speed of a [2011]
(a)  DC shunt motor with armature resistance 
The zero speed regulation for any DC motor
­control. can be studied by observing the speed–­
current
(b) DC shunt motor with field weakling control. c haracteristics. It is observed by studying
­

Chapter 01.indd 3 11/9/2015 6:01:00 PM


3.4 | Electric Machines

c­ haracteristics that only differential compound DC Solution: (b)


motor shows the zero regulation at full-load. (a) Field control:
Hence, the correct answer is (d). The constant power drive is provided by field
control. It is known that speed is inversely
4. In a DC machine, which of the following state-
ments is true? [2006]  1
proportional to flux per pole  ω ∝  , due to
(a)  Compensating winding is used for neutralizing  φ
armature reaction while interpole winding is this the field cannot be increased. It can only
used for producing residual flux. be weakened. This kind of control is good for
(b) Compensating winding is used for neutralizing speed control and directly proportional to flux
armature reaction while interpole winding is per pole T ∝ φ . This kind of control is adopted
used for improving commutation. for torque value below base torque.
(c)  Compensating winding is used for improving (b) Armature Control:
commutation while interpole winding is used  Armature control provides constant-torque
for neutralizing armature reaction. drive. The full-load torque can be obtained
(d) Compensating winding is used for improving at full-load armature current by keeping field
commutation while interpole winding is used current at maximum value at all speeds. This
for producing residual flux. control is adopted for speeds below base speed.
Solution: (b)
The compensating winding in a DC motor serves the

Torque
purpose to reduce armature reaction. Also they pre-
vent the main field to crowd in one side. These are se-
ries coils that are placed in the slots of poles coinciding
the axis of brushes in the motor. They are connected Speed
to armature in series. The magnetic field is produced Armature Field
by these series connected windings, which vary ac- control control
cording to the armature current. These compensating
windings are wound so that they can neutralize the ef- Hence, the correct option is (b).
fects generated by field of armature. To give speed in 6. A DC series motor fed from rated supply voltage
commutation process the reactance voltage should be is overloaded and its magnetic circuit is saturated.
neutralized and this is done by injecting the suitable The torque–speed characteristic of this motor will
polarity dynamical voltage into commutating coil. be approximately represented by which curve of
Due to restriction over injecting such voltage, narrow figure? [2002]
interpolars are provided in interpolar region. (a) Curve A (b) Curve B
Hence, the correct option is (b). (c) Curve C (d) Curve D
5. The speed/torque regimes inthe control method- Solution: (b)
sanda DC motorsuitable for the same are given C motor is fed by rated supply voltage and it is
respectively in List-I and List-II. [2003] overloaded and its magnetic circuit is saturated.
The curve for such a motor can be represented as
List-I List-II
A.  Field control 1.  Below base speed
B.  Armature control 2.  Above base speed
Speed

3.  Above base torque


4.  Below base torque
Codes:
A B
   (a)  1 3 Torque
(b)  2 4
    (c)  2 3 At saturation point linear characteristics are used.
    (d)  1 4 Hence, the correct option is (b).

Chapter 01.indd 4 11/9/2015 6:01:00 PM


Chapter 1  DC Machines  |  3.5

7. In case of an armature controlled separately excited Solution: (c)


DC motor drive with closed loop speed control, an Extracting the data from given problem:
inner current loop is useful because it  [2001] Number of poles are 4, power developed is P watts,
(a) limits the speed of the motor to a safe value. voltage is E volts,number of brushes present is 2.
(b) helps in improving the drive energy efficiency.
Circuit diagram represent a DC generator with
(c)  limits the peak current of the motor to the per-
complete 4 brushes:
missible value.
(d) reduces the steady state speed error. Ia
Solution: (c) 2
Inner current loop in a separately excited DC ­motor + +
with closed loop speed control is very useful as it
limits the peak current due to close loop ­system.
Ia Ia
Hence, the correct option is (c). −
4 4
Ia Ia −
8. The compensating winding in a DC machine Ia
4 4
 [2000] 2
(a) is located in armature slots for compensation Ia −
Ia
of the armature reaction. 2 +
2
(b) is located on commutating poles for improving
the commutation.
(c) is located on pole shoes for avoiding the flash-
over at the commutator surface. Circuit diagram represent a DC generator with 2
(d) is located on poles shoes to avoid the sparking adjacent brushes:
at the brushes.
Solution: (c)
Compensating windings are used to overcome the
− Ia
commutation and cross-magnetization of armature
4
reaction, which causes high flux density in trail-
+
ing pole tip. Also the coil under this tip can develop
the induced voltage that is high enough to cause a −
+
flashover between nearby commutator segments. Ia
­Physically, this coil is much closer to the commuta- 4
tion zone in which the air temperature can be high
due to commutation process.
It is observed that E ′ = E.
Therefore, the compensating windings are located
on pole shoe to avoid flashover. I
I a′ = a
4
Hence, the correct option is (c).
Power: P = EI a .
9. A 4-pole lap-wound DC generator has a devel-
Then, new power will be
oped power of P watts and voltage of E volts. Two
adjacent brushes of the machine are removed as ( EI a ) P
P ′ = EI a′ = =
they are worn out. If the machine operates with 4 4
the remaining brushes, the developed voltage and Therefore, the voltage and power obtained from
power that can be obtained from the machine are p
machine are E , .
 [1999] 4
(a) E, P (b) E P Hence, the correct option is (c).
,
2 2 10. A 200 V, 10 kW lap-wound DC generators has 10
P P poles and 500 conductors on its armature. If the
(c) E, (d) E ,
4 2 pole face covers 80% of the pole pitch, the pole

Chapter 01.indd 5 11/9/2015 6:01:01 PM


3.6 | Electric Machines

face conductor required to fully compensate for Hence, torque–speed characteristics of repulsion
armature reaction will be conductors/ motor resembles with series motor.
pole. [1997] Hence, the correct option is (c).
Solution: 12. A differentially compounded DC motor with inter-
Extracting information from the given problem: poles and with brushes on the neutral axis is to be
Number of poles in lap wound DC generator are driven as a generator in the same direction with the
10, number of armature conductors: Z = 500. same polarity of the terminal voltage. It will then
The pole face is covered 80% of pole pitch.  [1995]
(a)  be a cumulatively compound generator but the
Then, number of poles:
interpole coil connections are to be reversed.
P = 10 × 80% = 8. (b) be a cumulatively compounded generator with-
A= P= 8. out reversing the interpole coil connections
Because of lap wound number of parallel path is (c)  be a differentially compounded generator with-
equal to number of poles. out reversing the interpole coil connections.
Compensating conductors/pole (d)  be a differentially compounded generator but
the interpole coil connections are to be reversed
Z 500
= × 0.8 = × 0.8 = 6.25 ≈ 7. Solution: (b)
AP 8×8
 A differential compound DC motor can be driven as
Here, A is number of parallel path and P is number a generator by reversing the direction of armature
of poles current and its nature will found to be cumulative.
Therefore, the number of poles would be 7. The reversing of current direction changes the
11. The torque–speed characteristics of a repulsion ­interpole flux nature. Therefore, there is no need to
motor resembles with which of the following DC reverse the connection of motor.
motor characteristic? [1996] Hence, the correct option is (b).
(a) Separately excited (b) Shunt 13. A cumulative compound long shunt motor is driv-
(c) Series (d) Compound ing a load at rated torque and rated speed. If the
Solution: (c) series is shunted by a resistance equal to the resist-
Torque–speed characteristics of series: ance of the series field, keeping the torque ­constant,
 [1993]
Torque

Series
field
Shunt
Supply
field

Speed
Torque–speed characteristics of repulsion motor:
(a) The armature current increases
(b) The motor speed increases
(c) The armature current decreases
Torque

(d) The motor speed decreases


Solution: (a)
Torque equation is given as T = kφ I a .
In a cumulative compound long shunt motor, a load
Speed is being driven at rated torque and rated speed and

The torque–speed characteristics of repulsion its series field is shunted by a resistance equal to
motor is almost same as the series DC motor. series field resistance with constant torque. Due to

Chapter 01.indd 6 11/9/2015 6:01:02 PM


Chapter 1  DC Machines  |  3.7

new resistance the current is halved of its original Substitute 240 V for Eb, 10 for Ia and 1500 rpm for N.
value. Therefore, the armature current will have to 240 × 10 × 60
increase to keep torque constant. T= × 60 = 15.2788 Nm.
2π × 1500
 Hence, the armature current increases to keep
torque constant. Therefore, torque developed armature is 15.2788 Nm.
Hence, the correct option is (a).
14. Neglecting all losses, the developed torque (T) of Two-marks Questions
a DC separately excited motor, operating under
constant terminal voltage, is related to its output
1. A separately excited 300 V DC shunt motor under
power (P) as under: [1992] no-load runs at 900 rpm drawing an armature cur-
(a) T ∝ P      (b)  T ∝ P rent of 2 A. The armature resistance is 0.5 W and
(c) T 2 ∝ P 3      (d)  T independent of P leakage inductance is 0.01 H. When loaded, the
Solution: (b) armature current is 15 A. Then the speed in rpm is
 Relation of output power and torque under con- ________. [2014-S1]
stant terminal voltage can be given as Solution:
T = kφ I a Voltage: V = 300 V, at no-load speed: No = 900 rpm,
armature current: Ia = 2 A, armature resistance:
T ∝ I a (1) Ra = 0.5 W, Leakage reactance is given as 0.01 H.
Output power for separately excited motor is given The emf at no-load is
as P = Eb I a . E = V − I a Ra = 300 − 2 × 0.5 = 299 V.
If voltage supply is constant then power is Armature current at full-load: I 0 = 15 A.
P ∝ I a . (2) Emf induced at no-load,
From equation (1) and (2), E0 = V − I 0 Ra = 300 − 15 × 0.5 = 292.5 V.
T ∝ P. Leton-load speed be N.
Therefore, the right answer is T ∝ P. E ∝ φ N.
Hence, the correct option is (b). E ∝ N (φ is constant).
15. A separately excited DC motor has an armature The ratio of emf and speed can be equated at on-
resistance of 0.5 W. It runs from a 200 V DC supply load and no-load condition.
drawing an armature current of 20 A at 1500 rpm.
N E
For the same field current, the torque developed = .
for an armature current of 10 A will be [1992] N 0 E0
Solution: E  292.5 
N= × N0 =   × 900 = 880.43 rpm.
Calculate the EMF of the DC motor. E0  299 
V = Eb + I a Ra .
2. A 250 V DC shunt machine has armature circuit
Here, V is voltage, Eb is back emf, Ia is armature resistance of 0.6 W and field circuit resistance of
current, Ra is armature resistance. 125 W. The machine is connected to 250 V sup-
Substitute 250 V for V, 20 A for Ia and 0.5 W for Ra. ply mains. The motor is operated as a generator
250 = Eb + 20 × 0.5. and then as a motor separately. The line current of
the machine in both the cases is 50 A. The ratio of
Eb = 240 V. the speed as a generator to the speed as a motor is
Calculate the torque. . [2014-S2]
EI Solution:
T = b a × 60.
2π N Voltage: V = 250 V, armature resistance: Ra = 0.6 W,
Here, T represent the torque; N represent the num- field circuit resistance: Rf = 125 W and line current:
ber of turns. I1 = 50 A.

Chapter 01.indd 7 11/9/2015 6:01:05 PM


3.8 | Electric Machines

Case 1: It is known that E ∝ φ N .


V 250 Here, N is the speed of motor, E is the emf and ϕ is
Field current, I=
f = = 2 A.
R f 125 the flux, which is constant.
The armature current is calculated as: E fl N fl
= .
I a = I l − I f = 50 − 2 = 48 A. E N0
The emfis calculated as:  E fl   202.4 
N fl =   × N0 =   × 1400.
Em = V − I a Ra = 250 − 48 × 0.6 = 221.2 V.  E   228.4 
Case 2: The full-load speed of motor is calculated as
Machine running as a generator, 1240.63 rpm.
I a = I l + I f = 50 + 2 = 52 A.
4. A 220 V, 15 kW, 1000 rpm shunt motor with arma-
The emfis calculated as: ture resistance of 0.25 W,has a rated line current of
Eg = V + I a Ra = 250 + 52 × 0.6 = 281.2 V. 68 A and a rated field current of 2.2 A. The change
Emf can be given as Ea ∝ N for a DC shunt in field flux required to obtain a speed of 1600 rpm
­machine. while drawing a line current of 52.8 a and a field
current of 1.8 A is [2012]
Eg N g
= . (a) 18.18% increase  (b)  18.18% decrease
Em N m (c) 36.36% increase  (d)  36.36% decrease
N g 281.2 Solution: (d)
= = 1.271.
N m 221.2 Extracting the data from given problem:
V = 200 V, N1 = 1000 rpm, armature resistance:
3. A 15 kW, 230 V DC shunt motor has armature cir-
Ra = 0.25 W, rated line current: IR = 68 A, rated
cuit resistance of 0.4 W and field circuit resistance of
current: If 1 = 2.2 A, N2 = 1600 rpm, line current
230 W. At no-load and rated voltage, the motor runs
at 1400 rpm and the line current drawn by the motor I L = 52.8 A and field current: I f 2 = 1.8 A .
is 5 A. At full-load, the motor draws a line current of The amount of field flux is calculated as follows:
70 A. Neglect armature reaction. The full-load speed I a1 = I R − I f 1 = 68 − 2.2 = 65.8 A.
of the motor in rpm is __________. [2014-S1]
I a 2 = I L − I f 2 = 52.8 − 1.8 = 51 A.
Solution:
A supply voltage: V = 230 V, circuit resistance: Ra The induced emf in armature can be given as
= 0.4. W, field resistance: Rf = 230 W, the speed of  Ea1 = V − I a1 Ra = 220 − ( 65.8 × 0.25 ) = 203.55 V.
motor: N0 = 1400 rpm, the line current at no-load:
I0 = 5 A. At full-load the line current: If 1 = 70 A. Ea 2 = V − I a 2 Ra = 220 − ( 51× 0.25 ) = 207.25 V.
Armature reaction is neglected. It is known that induced emf can also be written as
The speed of motor at full is calculated as: Ea = Kφωm .
V 230
The field current is = If = = 1 A. Ea1 φ1 ωm1
Rf 230 = × .
E a 2 φ2 ω m 2
The no-load armature current is
203.55 φ1 1000
I a = I lo − I f = 5 − 1 = 4 A. = × .
207.25 φ2 1600
The induced emf at no-load is given as
φ1
E = V − I a0 Ra = 230 − 4 × 0.4 = 228.4 V. = 1.571.
φ2
At full-load the armature current will be
φ2 1
I afl = I fl − I f = 70 − 1 = 69 A. = .
φ1 1.571
The induced emf at full-load is given as
It is observed from this ratio that the field flux
E fl = V − I fl Ra = 230 − 69 × 0.4 = 202.4 V.
­reduces.

Chapter 01.indd 8 11/9/2015 6:01:09 PM


Chapter 1  DC Machines  |  3.9

Percentage of change in flux is: Here, T1 is torque generated due to flux (φ1) and
φ2 − φ1 φ2 1 T2 is torque generated due to flux (φ2 ) .
= −1 = − 1 = -0.3636 = -36.36%.
φ1 φ1 1.571 φ1 I a1 = φ2 I a 2 .
The negative sign shows the decrease in flux. φ1 × 10 = 0.9 × φ1 I a 2 .
Hence, the correct option is (d). I a 2 = 11.11 A.
5. A 220 V, DC shunt motor is operating at a speed Ea 2 = V − I a 2 ( Ra + RExt ).
of 1440 rpm.The armature resistance is 1.0 W and 189 = 220 − 11.11(1 + RExt ).
armature current is 10 A. If the excitation of the RExt = 1.79 Ω.
machine is reduced by 10%, the extra resistance to
The external resistance in circuit is RExt = 1.79 W.
be put in the armature circuit to maintain the same
speed and torque will be [2011] Hence, the correct option is (a).
(a) 1.79 W (b) 2.1 W Common Data for Questions 6 and 7:
(c) 3.1 W (d) 18.9 W A separately excited DC motor runs at 1500 rpm
Solution: (a) under no-load with 200 V applied to the armature.
Extracting the data from given problem: The field voltage is maintained at its rated value.
Supply voltage: V = 200 V, speed: N = 1400 rpm, The speed of motor, when it delivers a torque of
armature resistance: Ra = 1 W and armature cur- 5 Nm, is 1400 rpm as shown in the figure. The ro-
rent: Ia1 = 10 A. tational losses and armature reaction are neglected.
 The extra resistance put in armature circuit at same Speed (rpm)

speed and torque is calculated as follows:


1500
Armature voltage:
1400
Ea1 = V − I a1 Ra = 220 − (10 × 1) = 210 V.
Now consider the case when external resistance is
included in the circuit
I
+
Ia
0 5 torque (Nm)
If
Rext Hence, the correct option is (a).
6. The armature resistance of the motor is [2010]
Ra
Vt (a) 2 W (b) 3.4 W
Shunt
field (c) 4.4 W (d) 7.7 W
+
Solution: (b)
Ea

Extracting the data from given problem:
− Speed: N1 = 1500 rpm, the supply voltage: V1 = 200 V,
torque T1 = 5 Nm, N = 1400 rpm. Losses are neglected.
Ea ∝ φ N . Armature resistance is calculated as follows:
φ2 = 0.9φ1 . Consider that speed is directly proportional to emf.
At constant speed Ea ∝ φ . N = 1400 rpm.
N 1400
φ 0.9φ1 Ea = E0 × = 200 × = 186.67 V.
Ea2 = 2 Ea1 = × 210 = 189 V. N0 1500
φ1 φ1 Armature current:
Torque ∝ φ I a. 2π
5 × 1400 ×
Torque is also constant then, Tω 60 = 3.925 A.
Ia = =
T1 = T2. Ea 186.67

Chapter 01.indd 9 11/9/2015 6:01:12 PM


3.10 | Electric Machines

Armature resistance: (a) 


VAB
I a Ra 13.33
Ra = = = 3.39 Ω.
Ia 3.925
wt
0
Therefore, the calculated armature resistance is 0.2p 0.4p 0.6p 0.8p p
Ra = 3.39 W.
(b) 
VAB
Hence, the correct option is (b).
7. For the motor to deliver a torque of 2.5 Nm at
1400 rpm , the armature voltage to be applied is
0 wt
 [2010]
0.2p 0.4p 0.6p 0.8p p
(a) 125.5 V (b) 193.3 V
(c) 
(c) 200 V (d) 241.7 V V AB

Solution: (b)
 Using the same data as given in previous question,
the speed of motor is 1400 rpm at torque 2.5 Nm. wt
Armature voltage can be calculated as below: 0
0.2p 0.4p 0.6p 0.8p p
Voltage required by motor: (d) 
VAB
V = Ea + I a Ra = 186.67 + I a (3.39). (1)

Armature current can be calculated as:


wt
T  0
 2.5  0.2p 0.4p 0.6p 0.8p p
I a′ = I a  1  = 3.925   = 1.9625 A.
 T2   5  Solution: (a)
The brush position from interpolar axis has been
The voltage applied to the motor can be calculated changed to other position A and A′. The figure is
as follows: given as below:

V2 = Ea + I 2 Ra = 186.67 + (1.9625 × 3.39) = 193.3 V. N S

Hence, the correct option is (b). B′ A′


+
8. Figure shows the extended view of a 2 pole DC B A
+
machine with 10 armature conductors. Normal
brush positions are shown by A and B, placed at
1 2 3 4 5 1′ 2′ 3′ 4′ 5′
the interpolar axis. If the brushes are now shifted,
in the direction of rotation, to A′ and B ′ as shown, Rotation at speed w rad/sec
the voltage waveform VA′B′ will resemble [2009]
The figure above shows the extended view of two
poles and ten armature conductors of a DC machine.
Consider the action of DC machine as a DC mo-
N S tor. Thus, when the brushes move in the direction
of DC motor then the fields will get magnetized.
B′ A′
+ After magnetization two fields will act. One is ar-
B A mature field flux and other is main field flux (due
+
to poles of motor). Influence of armature field on
main field flux causes armature reaction. Due to
1 2 3 4 5 1′ 2′ 3′ 4′ 5′
occurrence of armature reaction the leading tips
Rotation at speed w rad/sec of both South Pole and North Pole will get more

Chapter 01.indd 10 11/9/2015 6:01:14 PM


Chapter 1  DC Machines  |  3.11

magnetized and trailing tip will get demagnetized. 31.1 W


(a) (b) 31.9 W
To visualize the movement of brushes in DC motor 15.1 W
(c) (d) 15.9 W
the figure below is represented: Solution: (a)
The rated value of armature current is 12 A.
N
Leading tip Trailing tip During braking, the value of armature current is
N I B = 1.25 × 12 = 15 A.
B′ far
The total armature resistance of braking is the sum
of external resistance and armature resistance,
A′
RB = Ra + RExt . (1)
Trailing tip Leading tip Force
 E +V (2)
S RB = .
IB

Statement for linked answer Questions 9 and 10: Equating both of these equations,
A 240 V, DC shunt motor draws 15 A while sup- E +V
plying the rated load at a speed of 80 rad/s. The Ra + R Ext = .
IB
armature resistance is 0.5 W and the field winding
resistance is 80 W. 474
0.5 + R Ext = .
Hence, the correct option is (a). 15
RExt = 31.1 Ω
9. The net voltage across the armature resistance at
the plugging will be  [2008] Hence, the correct option is (a).
(a) 6V (b) 234 V
(c) 240 V (d) 474 V 11. A 220 V DC machine supplies 20 A at 200 V as a
generator. The armature resistance is 0.2 W. If the
Solution: (d)
machine is now operated as a motor at same termi-
 The supply voltage: V = 240 V, current is 15 A, the
nal voltage and current but with the flux increased
speed is of 80 rad/s, armature resistance is given as
by 10% the ratio of motor speed to generator speed
0.5 W and field winding resistance: Rf = 80 W.
is [2006]
The net voltage can be calculated as follows: (a) 0.87 (b) 0.95
Current through the field winding, (c) 0.96 (d) 1.06
V 240 Solution: (a)
If = = = 3 A.
Rf 80 Extracting the data from given problem:
Supply voltage: 220 V at 20 A and armature resist-
Load current, I L = I f + I a
ance: Ra = 0.2 Ω.
I a = I L − I f = 15 − 3 = 12 A. Generator speed can be found as follows:
Back emf can be calculated as, For generator,
Ea = V − I a Ra = 240 − 12 × 0.5 = 234 V. Eg = V + I a Ra = 200 + 20 × 0.2 = 204.
Plugging results in the reversal of armature winding For motor,
connections. If the connections of armature winding
Em = V + I a Ra = 200 − 20 × 0.2 = 196.
arereversed, strong braking torque can be achieved.
The net voltage comes out to be, Ea + V = 240 + 234 Ratio calculated as follows,
= 474 V. The emf, speed and flux of a DC motor and gen-
Hence, the correct option is (d). erator has a ratio as following:
10. The external resistance to be added in the arma- Eg Ng φg
ture circuit to limit armature current to 125% of = × .
its rated value is  [2008] Em Nm φm

Chapter 01.indd 11 11/9/2015 6:01:16 PM


3.12 | Electric Machines

Here, (b) Developed Torque: Torque can be defined as


rotational force acting on body and causes it to
204 N g 1 rotate in a particular direction. As the torque is
= × .
196 N m 1.1 developed in armature and tends to rotate it,its
expression can be given as
Nm
= 0.87. T = k aφ I a .
Ng
Here, ka is the proportionality constant, ϕ is
Hence, the correct option is (a). flux per pole and Ia is the armature current. So
it is observed that torque depends upon flux
1 2. In relation to DC machines, match the following and armature current.
and choose the correct combination [2005] (c) Developed Power: Developed power is the
product of induced voltage in motor and ar-
List-I List-II
mature current. The expression for this can be
Performance Proportional to written as follows
Variables P = EI a = (kaφω ) I a .
A.  Armature emf (E) 1. Flux (ϕ), speed P = (kaφ I a )ω = T ω.
(w) and armature So, P depends on ϕ, w and Ia.
current (Ia)
Hence, the correct option is (d).
B. Developed torque (T) 2.  ϕ and w only 13. A 50 kW DC shunt motor is loaded to draw rated
C. Developed power (P) 3.  ϕ and Ia only armature current at any given speed. When driven
(1) At half the rated speed by armature voltage
4.  Ia and w only
control and
5.  Ia only (2) At 1.5 times the rated speed by field control,
the respective output powers delivered by the
Codes: motor are approximately [2005]
(a) 25 kW in (1) and 75 kW in (2)
A B C
(b) 25 kW in (1) and 50 kW in (2)
(a) 3 3 1 (c) 50 kW in (1) and 75 kW in (2)
(b) 2 5 4 (d) 50 kW in (1) and 50 kW in (2)
Solution: (b)
(c) 3 5 4
(i) The output power delivered by motor is when
(d) 2 3 1 a DC shunt motor is driven at half the rated
speed using armature voltage control method.
Solution: (d)
The armature control method provides the con-
Performance variables of DC machines and their
stant torque drive. Consider the field current at
proportionality:
maximum value and the full-load torque can be
(a) Armature emf: Armature emf is emf in- obtained at full-load armature current at all speeds.
duced in armature of DC machine due to
T = constant.
cutting of flux lines by armature conduc-
tors. Its expression can be given as below: T ω = output power (Po ).

T = constant.
E = kaφω Po
= constant.
ω
Here, ka is the proportionality constant, ϕ rep-
(Po )1 = 50 kW.
resents the flux per pole and w is the speed.
This expression shows that emf depends on ω1
ω2 = .
flux per pole and angular speed of motor. 2

Chapter 01.indd 12 11/9/2015 6:01:18 PM


Chapter 1  DC Machines  |  3.13

( P0 )1 ( P0 ) 2
= .
ω1 ω2
C ′2
50 ( P0 ) 2 C ′1
= . S N
ω1 ω1 C1
C2
2
( P0 ) 2 = 25 kW.
(ii) Now the motor is driven at 1.5 times the rated From the given figure, it is observed that the two
speed by using field control method. poles are revolving in clockwise direction. R­ elative
to motion it is assumed that the magnetic poles are
The field control method provides the con-
stationary and the coils are rotating in counter-
stant-power drive. It is known that speed is in-
clockwise direction.
versely proportional to flux per pole given by
 1 The magnetic flux will flow from North to South Pole.
expression  ω ∝  . At first consider the coils C1 and C1′, the direction
 φ
of induced voltage can be find by Fleming’s right-
The torque is proportional to flux per pole giv- hand rule, which states that the thumb represents
en by expression (T ∝ φ ). the direction of force on conductor, the fore-finger
The output power can be given as will represent the magnetic-field direction and the
k  middle finger will represent the induced current
T ω =  1  ( k2φ ) = k1k2 = constant. ­direction in conductor.
φ 
So according to this rule the induced emfis given as
This means that the delivered power will re-  
emf = v × B l = Blv sin θ .
main constant, 50 kW.
Hence, the correct option is (b).
B
14. Two magnetic poles revolve around a stationary
armature carrying two coils (C1 − C1′, C2 − C2′ ) as
shown in figure. Consider the instant when the C1
V
poles are in a position as shown. Identify the cor- Since, the emf comes out to be positive, therefore
rect statement regarding the polarity of the induced the direction of induced emf will be upward, C1 = .
emf at this instant in coil sides C1 and C2. [2005] Consider the case of coil C2 and C2′ .
The magnetic field intensity is anti-parallel to the
C ′1 C ′2
induced voltage, thus the angle θ = 180°.
The induced voltage can be given as
S N
C2 C1 emf = Blv sin θ = Blv sin 180° = 0.
It means no emf will induce in coil C2 .
Hence, the correct option is (a).
(a)  in C1 , no emf in C2 15. A 8 pole, DC generator has a simplex wave-wound
(b) ⊗ in C1 , no emf in C2 armature containing 32 coils of 6 turns each. Its
(c)  in C2 , no emf in C1 flux per pole is 0.06 Wb. The machine is running at
250 rpm. The induced armature voltage is [2004]
(d) ⊗ in C2 , no emf in C1
(a) 96 V (b) 192 V
Solution: (a) (c) 384 V (d) 768 V
Extracting the data from given problem: Solution: (c)

The two coils of a stationary armature are Extracting the data from given problem:
(C1 − C1′, C2 − C2′ ) . Around these coils two mag- Number of poles: P = 8, number of coils is 32,
netic poles are revolving. The instant of pole posi- number of turns is 6, flux per pole: φ = 0.06 Wb
tion is considered. and machine speed: N = 250 rpm.

Chapter 01.indd 13 11/9/2015 6:01:21 PM


3.14 | Electric Machines

The induced armature voltage can be found out as The power developed by motor can be given as
follows: Pon = Ea − on I a − on
Total number of turns = No. of coils × No. of turns = 22.2 × 3.5 = 77.7 W.
= 32 × 6 = 192.
The power at shaft
Each turn consists of two conductors.
Psh = Pon − No load loss = 77.7 − 35.7 = 42 W.
Total number of conductors can be given as
Z = 192 × 2 = 384. Efficiency can be calculated as
It is known that for simplex wave winding arma- output power
Efficiency = × 100
ture, number of parallel path: A = 2. input power
The induced emf can be calculated as P0
= × 100
PZ VI a − on
E= φ N.
60 A 42
= × 100 = 48%.
Here, P is number of poles, Z represents number 25 × 3.5
of conductors, φ is flux per pole and N represents Hence, the correct option is (a).
speed of motor.
17. A DC series motor driving an electric train faces
8 × 384 a constant power load. It is running at rated speed
E= × 0.06 × 250 = 384 V.
60 × 2 and rated voltage. If the speed has to be brought
Therefore, the induced armature voltage calculated down to 25 pu the supply voltage has to be approx-
is 384 V. imately brought down to [2003]
Hence, the correct option is (c). (a) 0.75 pu (b) 0.5 pu
16. The armature resistance of a permanent magnet DC (c) 0.25 pu (d) 0.125 pu
motor is 0.8 W. At no-load, the motor draws 1.5 A Solution: (b)
from a supply voltage of 25 V and runs at 1500 rpm. Extracting the data from given problem:
The efficiency of the motor while it is operating on- An electric train is being driven by a DC series mo-
load at 1500 rpm drawing a current of 3.5 A form tor has constant power load. The speed is brought
the same source will be [2004] down to 0.25 pu, now the value for supply voltage
(a) 48.0% (b) 57.1% can be found as follows:
(c) 59.2% (d) 88.8% The back emf equation in a series motor can be
Solution: (a) given as
Extracting the information from given data: Eb = V − I ( RA + Rs ).
A permanent magnet DC motor has armature re- Suppose,
sistance of 0.8 W. ( RA + Rs ) = R.
At no-load, current drawn is 1.5 A, supply voltage Eb = V − IR.
is 25 V, and speed is given as 1500 rpm.
K nφ N = V − IR.
At on-load, current drawn is 3.5 A and speed is
1500 rpm. V − IR
N= .
Back emf at no-load, K nφ
Eb 0 = Vt − I a 0 Ra = 25 − 1.5 × 0.8 = 23.8 V. Current and flux are proportional to each other in a
The motor power can be calculated as, series motor, then φ ∝ I.
P0 = Eb 0 I a 0 = 23.8 × 1.5 = 35.7 W. At constant power load,
At on-load, E × I = T × ω = constant.
 I a − on = 3.5 A. T = K Aφ I = K A I 2.
If  ωn  decreases by one-fourth of its rated value,
Ea − on = V − I a − on Ra
then torque will increase four times to maintain the
= 25 − 3.5 × 0.8 = 22.2 V. power constant.

Chapter 01.indd 14 11/9/2015 6:01:24 PM


Chapter 1  DC Machines  |  3.15

As it is known torque is directly proportional to 19. To conduct load test on a DC shunt motor, it is cou-
square of current, then it can be observed that cur- pled to a generator which is identical to the motor.
rent will increase two times to brought down the The field of the generator is also connected to the
supply voltage to 0.5 pu same supply source as the motor. The armature
Hence, the correct option is (b). of the generator is connected to a load resistance.
The armature resistance is 0.02 pu Armature reac-
18. Following are some of the properties of rotating
tion and mechanical losses can be neglected. With
electrical machines:
rated voltage across the motor, the load resistance
(a) Stator winding current is DC, rotor-winding
across the generator is adjusted to obtain rated
current is AC
armature current in both motor and generator. The
(b) Stator winding current is AC, rotor-winding
p.u value of this load resistance is [2003]
current is DC
(a) 1.0 (b) 0.98
(c) Stator winding current is AC, rotor-winding
(c) 0.96 (d) 0.94
current is AC
(d) Stator has salient poles and rotor has commutator Solution: (c)
(e) Rotor has salient poles and slip rings and sta- Extracting the data from given problem:
tor is cylindrical A load test is conducted on a DC motor, which is
(f) Both stator and rotor have poly-phase windings coupled to an identical generator. The generator field
DC machines, synchronous machines and induc- and motor are connected to the same supply source.
tion machines exhibit some of the above properties The generator’s armature is connected to a load resist-
as given in the following table. Indicate the correct ance having value 0.02 pu The losses and armature
combination from this table[2003] ­reactions are neglected. The load resistance across the
generator is adjusted to find out the armature current
DC Synchronous Induction in motor and generator. The per unit (pu) value for
Machine Machines Machines load resistance can be found as follows:
(a) p, s q, t r, u + Ig
(b) q, u p, t r, s Im
(c) p, s r, u q, t RL
V M G Vg
(d) r, s q, u p, t
Solution: (a) −
The properties of rotating electrical machines: All the calculation will be done in pu system as
DC Machines: The field winding is provided on load resistance value is found in this system.
the stator, which is a stationary part and the ar- The rated voltage to which motor is connected is 1 pu
mature is provided on the rotor, which is a rotat- The armature current will flow in motor and gen-
ing part of DC machine. The AC current in rotor erator then I= I= 1 pu
g m
winding gets converted into DC with the help of
The armature resistance of both motor and genera-
commutator and slip-rings.
tor is R=m R=g 0.02 pu
Synchronous Machines: In this machine the arma- The value of back emf can be calculated as
ture is provided in rotor because of insulation, num-
Eb = V − I m Rm = 1 − 1× 0.02 = 0.98 pu
ber of slip rings, slip ring rating and high rating of
armature. So, because of all these factors armature The mechanical output power can be given as
is provided in stator and field is provided in rotor. Eb I m = 0.98 × 1 = 0.98 pu
Induction Machines: These are asynchronous The power obtained from motor is given to the
machines and drive at speed less than synchronous generator.
speed. They run on AC supply only means they Thus,
draw excitation by means of same supply and no Input power of generator = output power of motor.
external field supply is required in these machines. Eg I g = Eb I m .
Therefore, the correct option for this problem is (a).
Eg = Eb .

Chapter 01.indd 15 11/9/2015 6:01:25 PM


3.16 | Electric Machines

The generator’s voltage can be evaluated as


Vg = Eg − I g Rg = 0.98 − 1× 0.02 = 0.96 pu
Vg 0.96
RL =

Torque
= = 0.96 Ω.
Ig 1
Therefore, the value of armature resistance is 0.96 W.
Hence, the correct option is (c).
20. A 200 V, 2000 rpm, 10 A separately excited DC motor Speed
has an armature resistance of 2 W. Rated DC voltage Hence, the correct option is (d).
is applied to both the armature, draws 5 A from the 2 2. A permanent magnet DC commutator motor has
source, the torque developed by the motor is [2002] a no-load speed of 6000 rpm when connected to a
Solution: 120 V DC supply. The armature resistance is 2.5 W
Extracting the data from given problem: and other losses may be neglected. The speed of
Supply voltage: V = 200 V, speed: N = 2000 rpm and the motor with supply voltage of 60 V developing
current: I a1 = 10 A, armature resistance: Ra = 2 W, a torque 0.5 Nm, is [2000]
armature current I a 2 = 5 A. (a) 3000 rpm (b) 2673 rpm
Back emf at rated voltage is given as (c) 2836 rpm (d) 5346 rpm
( Eb ) rated = V − I a1 Ra = 200 − 2 × 10 = 18 V. Solution: (b)
The torque at rated Extracting the data from given problem:
1800 × 60 The no-load speed: N1 = 6000 rpm, armature re-
(T ) rated = = 8.598 Nm.
2π × 2000 sistance: Ia = 2.5 Ω, the other losses are neglected.
At 5 A current the torque, The new speed has to find out with supply voltage
T2 I 60 V and torque: T = 0.5 Nm.
= a2 The voltage equation can be written as V = Eb + IaRa.
Trated I a1
5 At no-load the armature current is zero, then the
T2 = × 8.598 = 4.299 Nm. back emf for first case will be 120 V, the back emf
10
for second case can be calculated as
Therefore, the torque developed by the motor is
N × 1200
4.299 Nm. Eb2 = 2 = 0.02 N 2 .
21. An electric motor with ‘constant output power’ will 6000
N2 = 50Eb2.(1)
have a torque–speed characteristic in the form of a
 [2001] Similarly, the voltage equation for second case can
(a) straightline through the origin. be written as
(b) straightline parallel to the speed axis.  V = Eb2 + IaRa.
(c) circleabout origin. 60 = Eb2 × 2.5.
(d) rectangularhyperbola.   Eb2 = (60 − 2.5 Ia).(2)
Also,
Solution: (d)
Eb 2 I a = T × ω.
 The torque-speed characteristic of an electric ­motor
having constant output power will be a rectangular Eb 2 0.5 × 2π × N 2
(60 − Eb 2 ) = .
hyperbola. Consider the power equation,   2.5 60
2π N 2.5π
P= T. Eb 2 (60 − Eb 2 ) = × Eb 2 × 50.
60 60
If multiplication of two variables is a constant term Eb 2 = 53.455 V.
then it will be an equation for rectangular hyperbola.
And the speed
Say, xy = C. x and y are the variables and C represent
N 2 = 2673.75 rpm.
the constant term. The constant power can relate to
the equation of rectangular hyperbola. The torque- N 2 ≈ 2673 rpm.
speed characteristic curve can be plotted as follows: Hence, the correct option is (b).

Chapter 01.indd 16 11/9/2015 6:01:27 PM


Chapter 1  DC Machines  |  3.17

23. A 240 V DC series motor takes 40 A when giv- 25. A 240 V DC shunt motor with an armature resist-
ing its rated output at 1500 rpm. Its resistance is ance of 0.5 W has a full-load current of 40 A. Find
0.3 W. The value of resistance which must be the ratio of the stalling torque to the full-load
added to obtain rated torque at 1000 rpm is[2000] torque when a resistance of 1 W is connected in
(a) 6 W (b) 5.7 W series with the armature? [1998]
(c) 2.2 W (d) 1.9 W (a) 4 (b) 12
Solution: (d) (c) 6 (d) None of these
Extracting the data from given problem: Solution: (a)
Supply voltage: V = 240 V, current: Ia = 40 A, Extracting the data from given problem:
speed: N1 = 1500 rpm and N2 = 1000 rpm, and re- Armature resistance: Ra = 0.5 W, full-load current
sistance: Ra = 0.3 W. is given as 40 A.
As it is known the torque is constant, thus arma- New resistance added in series is 1 W.
ture current and flux will also remain constant. The Consider the stall condition (speed, N = 0),
voltage equation can be given as
Eb = 0
V = Eb + I a Ra .
V = I stall ⋅ RA.
240 = Eb + 40 × 0.3.
240 = I stall ⋅ (0.5 + 1) Ω.
Eb = 228 V.
I stall = 160 A.
At constant flux,
N 1000 Ratio can be found as
Eb 2 = 2 Eb1 = × 228 = 152 V. Tstall I stall 160
N1 1500 = = = 4.
Now, Tstart I start 40
240 = 152 + 40 (0.5 + Rnew) Therefore, the ratio of stall torque to start torque is 4.
Rnew = 1.9 Ω Hence, the correct option is (a).
Hence, the correct option is (d). 26. A 4-pole dynamo with wave wound armature has
51 slot containing 20 conductors in each slot. The
24. A DC shunt motor is running at 1200 rpm, when
induced emf is 357 V and the speed is 8500 rpm.
excited with 220 V DC. Neglecting the losses and
The flux pole will be [1995]
saturation, the speed of the motor when connected
(a) 3.5 mWb (b) 1.2 mWb
to a 175 V DC supply is [1999]
(c) 14 mWb (d) 21 mWb
(a) 750 rpm (b) 900 rpm
(c) 1050 rpm (d) 1200 rpm Solution: (b)
Solution: (d) Extracting the data from given problem:
Extracting the data from given problem: Number of poles: P = 4, number of slots is 51,
Speed of DC shunt motor is 1200 rpm, excitation volt- number of conductor in each slot is 20, the induced
age is 220 V. New excitation voltage is given as 175 V emf: Eb = 357 volts, speed: N = 85000 rpm.
DC. In a DC shunt motor, the field current is given as Total number of armature conductors: Z = Number
V of slots × number of conductors in each slot.
If = .
Rf Back emf can be given as
The flux of DC motor is φ = K ′V. φ PZN
Eb = .
60 A
Voltage is also equal to back emf
Substitute all the values given in problem to find flux.
V = Eb = kφ N .
φ × 4 × 51× 20 × 8500
V V 1 357 = .
N= = = . 60 × 2
kφ KK ′V KK ′
357 × 60 × 2
Now, it is observed that speed is constant and inde- φ= = 1.235 mWb ≈ 1.2 mWb.
pendent of voltage. Therefore, the value of voltage 4 × 51× 20 × 8500
is 1200 rpm. Therefore, the flux per pole comes out to be 1.2 mWb.
Hence, the correct option is (d). Hence, the correct option is (b).

Chapter 01.indd 17 11/9/2015 6:01:28 PM


Chapter 2
Transformers
Method 2:
One-mark Questions 1100 11
aauto = = .
1000 10
1. A single phase, 50 kVA, 1000/100V two winding
transformer is connected as an autotransformer as  11 
Sauto aauto   11 10
shown in the figure. [2014-S2] = =  10  = × = 11.
S 2 wind aauto − 1  11  10 1
 
 10 − 1 
100 V Sauto = 11× S 2 winding = 11× 50 kVA = 550 kVA..
  
1100 V
2. Assuming an ideal transformer, the Thevenin’s equiv-
1000 V alent voltage and impedance as seen from the termi-
nals x and y for the circuit in figure are [2014-S2]

The kVA rating of the autotransformer is _______. x

Solution:
sin(wt )
Method 1:
500 A y
1: 2

550 A (a) 2sin (ω t ), 4 Ω      


(b) 1sin (ω t ), 1 Ω
1100 V (c) 1sin (ω t ), 2 Ω     (d) 2sin (ω t ), 0.5 Ω
50 A
1000 V Solution: (a)
2
V2 N 2 Z2  N2 
= = 
50 × 103 V1 N1 Z1  N1 
Current in HV side (winding) = = 50 A. 2
1000 N  N 
V2 =  2  ⋅V1 Z 2 =  2  ⋅ Z1
50 × 103  N1   N1 
Current in LV winding = = 500 A.
100 2
The direction and magnitude of current are accord- = ⋅ sin ωt = ( 2) 2 ⋅ 1
1
ing to the polarity shown.
V2 = 2 sin ωt Z2 = 4 Ω
Therefore, kVA rating of autotransformer = 100 ×
550 = 550 kVA. Hence, the correct answer is (a).

Chapter 02.indd 18 11/12/2015 10:01:49 AM


Chapter 2  Transformers  |  3.19

3. For a specified input voltage and frequency, if the Fraction x of full-load corresponding to maximum
equivalent radius of the core of a transformer is Iron loss
reduced by half, the factor by which the number of efficiency =
full load copper loss
turns in the primary should change to maintain the
same no-load current is [2014-S1] Pcore 64
(a) 1 (b) 1 = =
Pfl cu
= 0=
100
.8 80%.
4 2  
(c) 2 (d) 4
Solution: (c) Hence, the correct option is (c).
For a transformer, the induced emf equation is, 5. A single-phase air core transformer, fed from a
V1 = 2π f φ m − N . rated sinusoidal supply, is operating at no-load.
The steady state magnetizing current drawn by the
V1  2 π f ( BmA) N transformer from the supply will have the wave-

For maintaining the same no-load current, flux in form [2011]
V (a)
the core should be constant, i.e., 1 = constant.
f i

∴ 2 π Bm AN = Constant or A ⋅ N = constant

A = (2π r ) ⋅ l t

r = Radius of core; l = length of core.


∴ r ⋅ N = Constant (b)
i
r A 
Given, r2 = 1 ∵ A2 = 1 
2 2
t
r1 N1 = r2 N 2 .
r 
N 2 =  1  N1 = 2 N1 .
 r2 
(c)
 ∴ N 2 = 2 N1 . i
Therefore, number of turns required is twice that
of initial. t
Hence, the correct option is (c).
4. A single-phase transformer has no-load loss of 64 W
as obtained from an open-circuit test. When a short-
circuit test is performed on it with 90% of the rated (d)
i
currents flowing in its both LV and HV windings, the
measured loss is 81 W. The transformer has maxi-
mum efficiency when operated at [2013] t
(a) 50.5% of the rated current
(b) 64.0% of the rated current
(c) 80.0% of the rated current
(d) 88.8% of the rated current Solution: (c)
The transformer is fed from sinusoidal supply, so
Solution: (c)
flux (ϕ) set up in the transformer is also sinusoi-
For 90% current,
dal. As the transformer is air-cored, there will be
Pcu = (0.9) 2 Pfl cu . no saturation and hysteresis effect in transformer.
So, B-H curve is linear, due to which magnetizing
81
P
=fl cu = 100 W. current is also sinusoidal.
0.81

Chapter 02.indd 19 11/12/2015 10:01:51 AM


3.20 | Electric Machines

sectional area 20 cm2 and both the horizontal arms


f f of cross sectional area 10 cm2. If the two windings
Linear shown were wound instead on opposite horizontal
Sinusoidal i arms, the mutual inductances will [2009]

i
Sinusoidal

Hence, the correct option is (c).


6. A single-phase transformer has a turns ratio of 1 : 2,
and is connected to a purely resistive load as shown (a) Double     (b) Remain same
in the figure. The magnetizing current drawn is 1 A, (c) Be halved    (d) Become one quarter
and the secondary current is 1 A. If core losses and Solution: (c)
leakage reactances are neglected, the primary cur- Reluctance of magnetic circuit remains same in
rent is [2010] both cases
N2
1: 2 1A Self inductance of coil, L =
Reluctance
∼ L ∝ N2

Vertical arm,
( L1 )v ∝ ( N1 )v2
(a) 1.41 A (b) 2 A
(c) 2.24 A (d) 3 A ( L2 )v ∝ ( N 2 )v2
Solution: (c) Av = 20 cm 2
Secondary current, I2 = 1∠0° A.
Assuming square cross sectional area,
Secondary current referred to primary side,
av = Av
N 
I ′2 =  2  I 2 Length of coil = 1
 N1 
Number of turns,
2
=   × 1 ∠ 0° = 2∠0° A. l l
1 Nv = ∝
4av Av
 s the core losses are neglected, magnetizing cur-
A 1
rent (Im) will be in phase with flux (ϕ). Therefore, ( L1 )v ∝ ( N1 )v2 ∝
Av
Im lays the induced emf by 90°.
1
I m = 1∠− 90° A Similarly, ( L2 )v ∝
Av
Primary Current, Mutual inductance,

I1 = I m + I 2′ mv ∝ ( L1 )v ( L2 )v
= 1∠− 90° + 2 ∠0° 1 1
∝ ⋅
= 2.24∠− 26.56° A. Av Av
1
Hence, the correct option is (c). ∝ .
Av
7. The single phase, 50 Hz iron core transformer
in the circuit has both the vertical arms of cross Similarly, when coils placed on horizontal arms,

Chapter 02.indd 20 11/12/2015 10:01:53 AM


Chapter 2  Transformers  |  3.21

1 VR = 0 = I ( R cos φ − X sin φ )
mH ∝
AH R
tan φ =
mH A 20 X
= v = .
mv AH 10  o, zero voltage regulation is possible for leading
S
mH = 2 mv . power factor when,
 hen, mutual inductance gets double when wind-
T R
tan φ =
ings are placed on horizontal arms. X
Hence, the correct option is (c). Hence, the correct option is (c).
8. It is desired to measure parameters of 230/115 V, 10. In transformers, which of the following statements
2 kVA, single-phase transformer. The following is valid? [2006]
watt meters are available in a laboratory: [2008] (a) In an open-circuit test, copper losses are ob-
W1 : 250 V, 10 A, low power factor tained while in short-circuit test, core losses
W2 : 250 V, 5 A, low power factor are obtained

W3 : 150 V, 10 A, high power factor (b) In an open-circuit test, current is drawn at high
power factor
W4 : 150 V, 5 A, high power factor
(c) In a short-circuit test, current is drawn at zero
 he watt meters used in open-circuit test and short-
T power factor
circuit test of the transformer will respectively be (d) In an open-circuit test, current is drawn at low
(a) W1 and W2 (b) W2 and W4 power factor
(c) W1 and W4 (d) W2 and W3
Solution: (d)
Solution: (d)
Open-Circuit Test: In open-circuit test, no-load Io
current Io is very small (it is usually 2–6% of the Ii Im
rated current).
V1 Gi Bm E1
No-load current Io lags E by slightly less than 90°,
so power factor is very low.
Therefore, wattmeter W2 is suitable for open cir-
cuit. Short-Circuit Test: In short-circuit test, Vsc
needed to circulate the full-load current is very low.
Ii
Under these conditions, Io is only about 0.1 to E1
0.5% of full-load current. As Io is highly lagging
but it is very small as compared to full-load cur-
rent, therefore the power factor is high. Im Io
So, wattmeter W3 is suitable for the short-circuit test.
Hence, the correct option is (d). f
9. In a transformer, zero voltage regulation at full-
load is [2007] In open-circuit test, the transformer draws only ex-
(a) Not possible citing current. The exciting current is only magnet-
(b) Possible at unity power factor load izing in nature and is proportional to the sinusoidal
(c) Possible at leading power factor load flux and in phase with it. This is represented by Im
(d) Possible at lagging power factor load lagging the induced emf by 90°.
Solution: (c) However, the presence of eddy currents, and hys-
Voltage regulation = I(R cos ϕ + X sin ϕ) for lag- teresis both demand the flow of active power into
ging pt the system and as a consequence the exciting cur-
= I ( R cos φ − X sin φ ) for lagging pt rent Io has another component Ii in phase with E1.

Chapter 02.indd 21 11/12/2015 10:01:54 AM


3.22 | Electric Machines

Then, the exciting current lags the induced emf by Solution: (d)
an angle slightly less than 90° making power factor As per lenz’s law,
very low.

Hence, the correct option is (d). E1 = N1
dt
11. Which three-phase connection can be used in a
transformer to introduce a phase difference of 30° The positive direction of this emf opposes the pos-
between its output and corresponding input line itive current direction.
voltages [2005] Let ϕ = ϕ m sin wt
(a) Star–Star (b) Star–Delta
d
(c) Delta–Delta (d) Delta–Zigzag E1 = N1 (φm sin ωt )
dt
Solution: (b)
= N1φmω cos ωt
30° phase difference is produced by:
Δ/Y, Y/Δ, Y/Zigzag Y. E1 leads flux by 90°.
Hence, the correct option is (b). Similarly,
12. Figure shows an ideal single-phase transformer. dφ
The primary and secondary coils are wound on the E2 = N 2 (φm sin ωt ) and E2 loads the flux by 90°.
dt
N 
core as shown. Turns ratio  1  = 2.  The correct So, E1 and E2 are in phase.
 N2 
phasors of voltages E1, E2, currents I1, I2 and core E1 N1
flux Φ are as shown in [2003] = = 2
E2 N2
f
I1
E1 = 2 E2  or  E1 > E2
I2
E2
∼ E1 N1 N2 E2 R
I2 =
  R
Because of resistive load, E2 is in phase with E2.
I1 N 2 1
= =
 (a)  E1   (b)  E1 I 2 N1 2
I2 I2
I2 I1 =
2
I1 < I 2
E2 E2
I1 I1 and I2 are in phase because, I1 = I2 + exciting
I1
current (Im).
f Im is very small.
f
So, I1 ∼ I2.
   (c)  E1    (d)  E1
I1 On the basis of above analysis, phasor diagram is:
E1
I1
I2 E2
f f
I1

90°
f
E2 Im
I2 E2
I2 Hence, the correct option is (d).

Chapter 02.indd 22 11/12/2015 10:01:57 AM


Chapter 2  Transformers  |  3.23

13. In the protection of transformers, harmonic restraint 16. If an AC voltage wave is corrupted with an arbi-
is used to guard against [2001] trary number of harmonics, then the overall voltage
(a) Magnetizing inrush current waveform differs from its fundamental frequency
(b) Unbalanced operation component in terms of [2000]
(c) Lighting (a) Only the peak values
(d) Switching (b) Only the rms values
Solution: (a) (c) Only the average values
Harmonic restraint is to guard against magnetizing (d) All the three measures (peak, rms and average
inrush current because these currents contain both values)
even and odd of which 2nd harmonic is dominant. Solution: (d)
Hence, the correct option is (a). Peak voltage is not same as that of fundamen-
14. The core flux of a practical transformer with a tal frequency if waveform consists harmonics
resistive load [2001] and so average and rms value also won’t be the
(a) Is strictly constant with load changes same.
(b) Increases linearly with load Hence, the correct option is (d).
(c) Increases as the square root of the load
17. In a constant voltage transformer (CVT), the out-
(d) Decreases with increased load
put voltage remains constant due to [2000]
Solution: (a) (a) Capacitor      (b) Input inductor
Supply voltage (c) Saturation      (d)  Tapped windings
φ=
Supply frequency Solution: (d)
CVT is designed on basic principle of ferroreso-
Then, core flux is independent of load variations nance, in which output winding is resonated with
and remains constant with load changes. the help of select value or high stability capaci-
tor. This is most reliable product as there is no
Hence, the correct option is (a). moving part, then bringing maintenance cost to
15. A single phase transformer is to be switched to zero.
the supply to have minimum inrush current. The Hence, the correct option is (d).
switch should be closed at [2001]
(a) Maximum supply voltage 18. The magnetizing current in a transformer is rich in
(b) Zero supply voltage  [1998]
1 (a) 3rd harmonic (b) 5th harmonic
(c) Maximum supply voltage
2 (c) 7th harmonic (d) 13th harmonic
1 Solution: (a)
(d) Maximum supply voltage
2 Magnetization curve becomes highly non-linear
and with such a core, a sinusoidal flux may be
Solution: (a)
obtained with a peaky magnetizing current that
When the input voltage is maximum, rate of
contains dominant peaky third harmonic compo-
change of flux is minimum; as both are 90° out of
nent. Since, magnitude of harmonic decreases on
the phase in case of sinusoidal input.
increasing order, third harmonic dominates.
Hence, the correct option is (a).
19. The efficiency of a 100 kVA transformer is 0.98 at
full as well as at half load. For this transformer at
full-load the copper loss [1998]
(a) is less than core loss
(b) is equal to core loss
(c) is more than core loss
Hence, the correct option is (a). (d) None of the above

Chapter 02.indd 23 11/12/2015 10:01:57 AM


3.24 | Electric Machines

Solution: (c) Solution: (d)


kVA out × pt The phase sequence, or the order in which the
Efficiency η = phases reach their maximum positive voltages,
kVA out × pt + wi + wcu
must be identical for two parallel transformers,
Let, pt = 1 otherwise during the cycle each pair of phases will
0.5 × kVA be short circuited.
At half load, 0.98 =
0.5 kVA + Pi + 0.25 PC Δ - Y and Y - Δ both belong to -30 phasor group,
hence they can be used for parallel operations.
kVA
= . (1) Hence, the correct option is (d).
kVA + 2 Pi + 0.5 PC
22. Autotransformer is used in transmission and distri-
kVA bution [1996]
At half load, 0.98 = (2)
kVA + Pi + PC (a) When operator is not available
From (1) and (2), (b) When iron losses are to be reduced
(c) When efficiency considerations can be ignored
Pi + PC = 2 Pi + 0.5 PC (d) When the transformation ratio is small
0.5 PC = Pi
Solution: (d)
PC = 2Pi If a is turn ratio of autotransformer,

Hence, the correct option is (c). Copper in autotransformer 1


= 1−

2 0. The laws of electromagnetic induction (Faraday’s Copper in 2 − winding transformer a
and Lenz’s law) are summarized in the following
equation: [1998]  100 
di Percentage saving of copper =  %
(a) e = iR (b) e= L  a 
dt

(c) e=− (d) None of these  a 
dt Sauto =   × S 2 wind
   a −1 
Solution: (c)
According to Lenz’s law, the direction of induced
Thus, advantages of autotransformer will be more
emf is such that if it allowed to cause a current by
with 100% tapping, i.e., transformation ratio near
short circuiting required, then the current so pro-
to unity.
duced has an effect that opposes the cause.
Hence, the correct option is (d).

e=± 23. The function of oil in a transformer is [1996]
dt (a) to provide insulation and cooling
According to Faraday’s law, induced emf ∝ rate of (b) to provide protection against lightning
change of flux linkages. (c) to provide protection against short circuit
(d) to provide lubrication
dN φ
e∝ Solution: (a)
dt
Oil in transformer construction serves the dou-
Hence, the correct option is (c).
ble purpose of cooling and insulating. All oils are
21. Keeping in view the requirement of parallel opera- good insulators. Heat developed in cores and coils
tion. Which of the 3-phase connections given below is passed to the oil and heat to the tank walls, from
are possible? [1996] which it is dissipated.
(a) Delta–Delta to Delta–Star
Hence, the correct option is (a).
(b) Delta–Delta to Star–Delta
(c) Star–Star to Delta–Star 24. Supply to one terminal of a Δ - y connected three-
(d) Delta–Star to Star–Delta phase core type transformer which is on no-load,

Chapter 02.indd 24 11/12/2015 10:01:59 AM


Chapter 2  Transformers  |  3.25

fails. Assuming magnetic circuit symmetry, volt- Solution: (b)


ages on the secondary side will be [1995] Equivalent circuit for transformers:
(a) 230, 230, 115 (b) 230, 115, 115 IA IB
(c) 345, 115, 115 (d) 345, 0, 345
jXA jXB
Solution: (b) Za ZB

Load
RA RB

230 V + +
EA ∼ ∼ EB
− −
115 V 115 V
230 V 230 V 115 V
Since, the transformers are connected in parallel,
they share the same source and have common load
1:1 which implies,
115 V

E A = EB .
Hence, the correct option is (b).
With this, the circuit becomes,
25. The percentage impedance of a 100 kVA. 11 kV/ IL
400 V, delta/wye, 50 Hz transformer is 4.5%. For IA IB
the circulation of half the full-load current dur- jXB
jXA
ing short-circuit test, with low voltage terminals
ZB
shorted, the applied voltage on the high voltage ZA
side will be _________. [1995] V Load
(a) 200 V (b) 247.5 V RA RB
(c) 250 V (d) 230 V

Solution: (b) EA = EB ∼
For half the rated current, the voltage should be half
of that voltage which will give full-load current, 

 Z (Ω) 

i.e., 4.5× 11× 10 = 495 V
3
I A =  B  × IL
Z A (Ω) + Z B (Ω)
100 

 Z (Ω) 

For half of the rated current, I B =  A  × IL
Z A (Ω)) + Z B (Ω)
495 S A = VI A*
V
= = 247.5 V
2 *
S A* = V I A
Hence, the correct option is (b). 
 Z (Ω ) 
 
= V   B 
 Z (Ω) + Z (Ω)  × I L 
*
26. Two transformers of identical voltage but of dif-
 B A  
ferent capacities are operating in parallel. For sat- 
isfactory load sharing [1994] Z (Ω )
S A* =  B  × S L*
(a) Impedances must be equal Z A (Ω ) + Z B (Ω )
(b) Per-unit impedances must be equal
X Similarly,
(c) Per-unit impedances and ratios must be  *
R S B* = VI B
equal 
X  Z (Ω ) 
=   A  S*.
(d) Impedances and
R
ratios must be equal  Z (Ω) + Z (Ω)  × L
 A B 

Chapter 02.indd 25 11/12/2015 10:02:01 AM


3.26 | Electric Machines


  28. Two transformers of different kVA ratings working
I j Z j (Ω) = Constant
in parallel share the load in proportional to their
ratings when their [1992]
1
Ij ∝ . (a) Per unit leakage impedances on the same kVA
Z j (Ω) base are the same
Since, V * is constant, (b) Per unit leakage impedances on their respec-

 tive ratings are equal
1
V * I j ∝  (c) Ohmic values of the leakage impedances are
Z j (Ω ) inversely proportional to their ratings
1 (d) Ohmic values of the leakage magnetizing re-
S *j ∝  actances are the same
Z j (Ω )
Solution: (b)
1
S *j ∝  Equivalent circuit for transformers:
Z j (pu ) × Z j (base)
IA IB
1
S ∝ 
*
j jXA jXB
Z j (pu )(V 2 rated/S j rated )
Za ZB
S j rated Load
S *j ∝  RA RB
Z j pu
+ +
S *j 1
∝  EA ∼ ∼ EB
S j rated Z j pu − −

For proportional load staring,


Since, the transformers are connected in parallel,
S j ∝ S j rated they share the same source and have common load
Sj which implies,
= Constant
S j rated E A = EB
  ⇒ Z pu = Constant
With this, the circuit becomes,
Then, for propotional load sharing, per unit leak-
age impedances on their respective rates are equal. IL
IA IB
Hence, the correct option is (b).
jXA jXB
27. When a transformer winding suffers a short circuit,
ZB
the adjoining turns of the same winding experience ZA
[1994] V Load
RA RB
(a) An attractive force
(b) A repulsive force
(c) No force ∼
EA = EB
(d) None of the above
Solution: (a)
 According to Lorentz principle, two conductors 

 Z (Ω) 

carrying current in the same direction will experi- I A =  B  × IL .
ence force of attraction and the conductors carrying Z A (Ω) + Z B (Ω)
current in opposite direction with experience force 

 Z (Ω) 

of repulsion. Since, the direction of current in ad- I B =  A  × IL .
joining terms is same, it will feel force of attraction. Z A (Ω) + Z B (Ω)
Hence, the correct option is (a). S A = VI A* .

Chapter 02.indd 26 11/12/2015 10:02:03 AM


Chapter 2  Transformers  |  3.27

S A* = V * I A 29. Two transformers of the same type, using the same


 grade of iron and conductor materials, are designed
 Z B (Ω)   to work at the same flux and current densities; but
= V  
*
 ×I .
 Z (Ω) + Z (Ω)  L  the linear dimensions of one are two times those
 B A  
 of the other in all respects. The ratio of kVA of the
Z (Ω ) two transformers closely equals [1992]
S A* =  B  × S L*
Z A (Ω ) + Z B (Ω ) (a) 16 (b) 8
(c) 4 (d) 2
Similarly, Solution: (a)
 Both the developed emf and current carrying capa-
S B* = V I B*
 bility are the functions of core area.
 Z (Ω ) 
=   A  ×S . I = JA
 Z (Ω) + Z (Ω)  L
   A B 

  Where J is current density which is constant and
I j Z j (Ω) = Constant E = 4.44 Bm An and Tpn.
E ∝ An
1
Ij ∝
Z j (Ω) An is the net area of core material.

As the area has increased by ‘4’ times, current car-
Since, V * is constant, rying capacity has also increased by ‘4’ times and
induced emf has also increased by ‘4’ times.

 1
V * I j ∝  . kVA 2 = E2 J 2 = 4 E1 × 4 J1 = 16 J1 E1 = 16 kVA1
Z j (Ω )
1 Hence, the correct option is (a).
S j* ∝  .
Z j (Ω )
1 Two-marks Questions
S j* ∝  .
Z j (pu ) × Z j (base)
1. The core loss of a single phase, 230/115 V, 50 Hz
1 power transformer is measures from 230 V side
S ∝ 
*
j .
Z j (pu )(V 2 rated/S j rated ) by feeding the primary (230 V side) from a vari-
able voltage variable frequency source while keep-
S j rated ing the secondary open circuited. The core loss is
S j* ∝ 
Z j (pu) measured to be 1050 W for 230 V, 50 Hz input.
The core loss is a gain measured to be 500 W for
S j* 1
∝  138 V, 30 Hz input. The hysteresis and eddy cur-
S j ratted Z j (pu) rent losses for the transformer for 230 V, 50 Hz
input are respectively [2014-S1]
For proportional load staring, (a) 508 W and 542 W  (b)  468 W and 582 W
S j ∝ S j rated (c) 498 W and 552 W  (d)  488 W and 562 W
Solution: (a)
Sj
= Constant (1) At 230 V, 50 Hz,
S j rated
Wi = 1050 W.
⇒ Z (pu) = Constant
V11 = 230 V.
Then, for propotional load sharing, per unit leak-
age impedances on their respective rates are equal. V11 230
= = 4.6.
Hence, the correct option is (b). f1 50

Chapter 02.indd 27 11/12/2015 10:02:06 AM


3.28 | Electric Machines

(2) At 138 V, 30 Hz, Bf 22 − Bf12


= × 100
Wi = 500 W. Bf12
V12 = 138 V. B(1.1) 2 f12 − Bf12
f 2 = 30 Hz. = × 100 = 21%.
Bf12

V12 230
= = 4.6. Change in hysteresis loss = 10%.
f2 30 Change in Eddy current loss = 21%.
V
Since, is constant, Hence, the correct option is (a).
F
Wi = Af + Bf 2 is valid. 3. The following arrangement consists of an ideal
transformer and an attenuator which attenuates by a
For (1), factor of 0.8. An AC voltage VWX1 = 100 V is applied
1050 = A(50) + B(50) 2 (1) across WX to get an open-circuit voltage VYX1 across
For (2), YZ. Next an AC voltage VYZ2 = 100 V is applied
500 = A(30) + B(30) 2 (2) across YZ to get an open-circuit voltage VWX2 across
Solving (1) and (2), V V
WX. Then,  YZ 1 , WX 2  are respectively. [2013]
A = 10.1667. VWX 1 VYZ 2
B = 0.2167. W
1:1.25
Then, at 230 V, 50 Hz
Wn = Af = 10.1667 × 50 = 508.33 W.
Y
We = Bf 2 = 0.2167 × (50) 2 = 541.75 W.
Hence, the correct option is (a).
2. For a single phase, two winding transformer, the X Z
supply frequency and voltage are both increased by 125 80 100 80
(a) and (b) and
10%. The percentage changes in the hysteresis loss 100 100 100 100
and eddy current loss, respectively are [2014-S2] 100 100 80 80
(a) 10 and 21 (b) −10 and 21 (c) and (d) and
100 100 100 100
(c) 21 and 10 (d) −21 and 10 Solution: (b)
Solution: (a) VYZ1 = 100 × 1.25 × 0.8 = 100 V.
V
Bmax ∝ . In second case, when 100 V is applied at YZ termi-
f nals, this whole 100 V will appear across the sec-
V2 = 1.1V1 . ondary winding.
f 2 = 1.1 f1 . Hence,
Bmax 2 V2 f1 1 100
= ⋅ = 1.1× = 1. V=WX 2 = 80 V.
Bmax 1 V1 f 2 1.1 1.25
VYZ 1 100
⇒ B is constant. = .
VWX 1 100
Wn = Af .
VWX 2 80
We = Bf 2 . = .
VYZ 2 100
A( f 2 − f1 )
% Change in Wn = Hence, the correct option is (b).
Af1
4. A single phase 10 kVA, 50 Hz transformer with
A(1.1 f1 − f1 )
= = 10%. 1 kV primary winding draws 0.5 A and 55 W, at
Af1 rated voltage and frequency, on no-load. A second
We 2 − We1 transformer has a core with all its linear dimen-
% Change in We = sions of the first transformer. The core material
We1

Chapter 02.indd 28 11/12/2015 10:02:10 AM


Chapter 2  Transformers  |  3.29

and lamination thickness are the same in both (a) (3 + j 0) Ω (b) (0.866 − j 0.5) Ω
transformers. The primary windings of both the (c) (0.866 + j 0.5) Ω (d) (1 + j 0) Ω
transformers have the same number of turns. If a Solution: (d)
rated voltage of 2 kV at 50 Hz is applied to the pri- Let rated VA rating of transformer be S. Rated
mary of the second transformer, then the no-load voltage (line to line) on secondary side = 200 V
current and power, respectively, are [2012] S
rated current = .
(a) 0.7 A, 77.8 W (b) 0.7 A, 155.6 W 3.200
(c) 1 A, 110 W (d) 1 A, 220 W 200
Solution: (b) 3 2002
= ZB = .
S 5
E = 2π Nf φ m.
200 3
E1 φm1 1
= = . 4 4 4S
E 2 φm 2 2 Z pu ,=sec = = .
Z B 2002 2002
φm 2 = 2φm1 .
5
mNiA Z pu , sec = Z pu , piirm.
φm = .
d
4S
φm1 ie1 A1d 2 ie1 1 Z pu = .
= = = . 2002
φm 2 ie 2 A2 d1 ie 2 2 On primary side,
ie 2 = 2ie1 . 100
Vpi, pn =.
1 3
ie 2 = 2 × = 0.7 A. S S
2 I pu, pn =
= .
Core loss ∝Volume of core. 3Vu 3.100
V 100
∴ Pc 2 = 2 Pc1
V1 3 1002
= ZB = .
S S
= 2 2 × 55 = 155.6 W.
100 3
Hence, the correct option is (b). Load impedance referred to primary side, -Z1
5. A balanced star-connected and purely resistive load = base impedance (primary side) × pu impedance
is connected at the secondary of a star-delta trans- 1002 4 S
former as shown in the figure. The line-to-line volt- = ×
S 2002
age rating of the transformer is 110/220 V. Neglecting
the non-idealities of the transformer, the impedance = (1 + j 0) Ω.
Z of the equivalent stat-connected load, referred to Hence, the correct option is (d).
the primary side of the transformer, is: [2010] Common Data for Questions 6 and 7:
R A a

B b

Y C c

B N
110/220 V r S1 S2
R The star-delta transformer shown above is excited
Z 4 kΩ on the star side with balanced, 4-wire, 3-phase, si-
Z Z 4 kΩ 4 kΩ nusoidal voltage supply of rated magnitude. The
b transformer is under no-load condition.
Y
6. With both S1 and S2 open, the core flux waveform
B y
will be [2009]

Chapter 02.indd 29 11/12/2015 10:02:14 AM


3.30 | Electric Machines

(a) A sinusoid at fundamental frequency


i(t )
(b) Flat-topped with third harmonic
(c) Peaky with third-harmonic
10 A
(d) None of these
Solution: (b) 0 $
5 ms 10 ms 15 ms $$ 25 ms 10 ms
As S2 is open, third harmonic current cannot flow
in delta winding. 10 A
As S1 is open, neutral gets isolated, therefore no
third harmonic current in star side.
Since, third harmonic current cannot flow in either
side, the magnetizing current I is almost sinusoidal. 8. The peak voltage across A and B, with S open is
 [2009]
For a sinusoidal excitation, flux is a flat topped with
400
3rd harmonic and since, there is no closed path for (a) V (b) 800 V
π
circulation of 3rd harmonics currents, no compen-
sating flux is produced for 3rd harmonics flux. 4000 800
(c) V (d) V
π π
Hence, the flux remains as flat topped wave.
Hence, the correct option is (b). Solution: (d)
7. With S2 closed and S1 open, the current waveform   (i) During 0 < t ≤ 5 ms, i (t ) increases linearly
in the delta winding will be [2009] with time,
(a) A sinusoid at fundamental frequency 10
(b) Flat-topped with third harmonic i (t ) = t = 2000t.
5 × 10−3
(c) Only third-harmonic di (t )
(d) None of these VAB = Lφ
dt
Solution: (c) 400 d (2000t )
As S1 is open, third harmonic current cannot flow in = × 10−3 ×
π dt
star side, but S2 is closed, therefore delta winding 800
provides path for third harmonic current to flow. = V.
π
As transformer is under no-load condition, only
3rd harmonic current will be flowing in delta con-  (ii)  During 5 ms < t ≤ 15 ms,
nected secondary. i (t ) = −2000t + 20.
Hence, the correct option is (c). di (t )
VAB = Lφ
Common Data for Questions 8 and 9: dt
The circuit diagram shows a two-winding, loss- 400 d
= × 10−3 × (−2000t + 20)
less transformer with no leakage flux, excited π dt
from a current source, i(t), whose waveform is also 800
shown. The transformer has a magnetizing induct- =− V.
400 π
ance of  .
π mH
≠ (iii) During 15 ms < t ≤ 25 ms,

A i (t ) = 2000t − 40.
S di (t )
1:1
VAB = Lφ
dt
i(t) 400 d
= × 10−3 × (2000t − 40)
30 Ω π dt
800
= V.
B π

Chapter 02.indd 30 11/12/2015 10:02:18 AM


Chapter 2  Transformers  |  3.31

800 Solution: (a)


Therefore, peak voltage A and B = V.
π
IL
Hence, the correct option is (d). C

9. If the waveform of i (t ) = 10 sin(100π t ) A, the peak I DC


voltage across A and B with S closed is  [2009] A
D V out
(a) 400 V (b) 240 V I in
(c) 320 V (d) 160 V Vin I AB

Solution: (a)
i (t ) = 10 sin(100π t ).
N1 = 4000
di (t )
VAB = Lφ N 2 = 6000
dt
400 d E1 N1
= × 10−3 × (10 sin 100π t ) =
π dt E2 N 2
400 × 10−3 N2 6000
= × 10 × 100π cos 100π t E2 = ⋅ E1 = × 400 = 600 V
π N1 4000
= 400 cos 100π t
= Vm cos ωt. The two coils are connected to obtain step up auto-
400 V
ω = 100π . transformer  .
1000 V
VAB = Vm = 400 V.
Vin = 400 V.
Hence, the correct option is (a).
Vout = VAB + VCD
Common Data for Questions 10 and 11:
= 400 + 600 = 1000 V
A C Hence, the correct option is (a).
1 1. In the autotransformer obtained in Question 10,
Coil 1 Coil 2 the current in each coil is [2009]
(a) Coil-1 is 25 A and Coil-2 is 10 A
(b) Coil-1 is 10 A and Coil-2 is 25 A
B D (c) Coil-1 is 10 A and Coil-2 is 15 A
(d) Coil-1 is 15 A and Coil-2 is 10 A
The figure above shows coils 1 and 2, with dot Solution: (d)
markings as shown, having 4000 and 6000 turns Load connected = S = 10 kVA.
­respectively. Both the coils have a rated current of
25 A. Coil is excited with single phase, 400 V, 50 Hz S
I2 = Load current =
supply. Vout
10. The coils are to be connected to obtain a single- 10 × 103
400 = = 10 A.
phase, V, autotransformer to drive should be 1000
1000 I DC= I= 10 A.
exercised to realize the required autotransformer? 2

[2009] Input current of autotransformer,


(a) Connect A and D; Common B
(b) Connect B and D; Common C S 10 × 103
I in = =
(c) Connect A and C; Common B Vin 400
(d) Connect A and C; Common D I in = 25 A.

Chapter 02.indd 31 11/12/2015 10:02:21 AM


3.32 | Electric Machines

Applying KCL, f
p r
I in = I DC + I AB . + +

25 = 10 + I AB . cpq 100 200 e rs

− −
Current through coil 1 = IAB = 15 A. q S
Hence, the correct option is (d).
1 2. Three single-phase transformers are connected to The induced emf (ers) in the secondary winding as
form a 3-phase transformer bank. The transform- a function of time will be of the form [2008]
ers are connected in the following manner.[2008] (a) 
e rs

A1 A2 a2 a1
24 V
B1 B2 b2 b1 2 2.5
0 1 t(s)
C1 C2 c2 c1 −48 V
Primary Secondary

(b) e rs
The transformer connection will be represented by
Yd 0 (b)
(a) Yd1 48 V
(c)
YS 6 (d) Yd11 0 1
Solution: (b) 2 2.5 t(s)
−24 V
A 2(A) c1
a 2(a)

c2(c)
(c) e rs
B 1 C1 b1
A1 48 V
24 V
b2(b) a
C 2(C) B 2(B) 1 0 1 2 2.5 t(s)
Primary Secondary

It is observed from the phasor diagram that phase


a to neutral voltage (equivalent star basis) on delta
(d) e rs
side lags -30° to the phase to neutral voltage on
the star side. This connection is known as -30 con-
nection on Yd1 connection.
Hence, the correct option is (b). 0 1 2 2.5

13. The core of a two-winding transformer is sub- t(s)


−24 V
jected to a magnetic flux variation as indicated in 48 V
the figure.
f(Wb) Solution: (b)
0.12

ers = − N 2
dt
0 1 2 2.5 t(s) N 2 = 200

Chapter 02.indd 32 11/12/2015 10:02:24 AM


Chapter 2  Transformers  |  3.33

0 ≤ t < 1,
For  At full-load,
x = 1, cos φ = 1, S = 50.
φ = 0.12t
1× 50 × 1
dφ d (0.12t ) 0.95 = .
ers = − N 2 = −200 × = −24 V. 1× 50 × 1 + losses
dt dt
Losses = 2.63 = Iron loss + Copper loss.
1 ≤ t < 2,
For  In autotransformer,
φ = 0.12 I2
C
d (0.12)
ers = − 0 = 0 V.
dt 200 A
300 A A V 2 = 750 V
For 2 ≤ t < 2.5, ϕ decreases with time,
V 1 = 500 V
φ = 0.24(25 − t ). 100
dφ d [0.24(2.5 − t )] B
ers = − N 2 = −200 × = 48 V.
dt dt
I 2 = 200 A.
Hence, the correct option is (b). V2 = 750 V.
14. A single-phase 50 kVA, 250 V/500 V two wind-
kVA rating of autotransformer,
ing transformer has an efficiency of 95% at full-
load, unity power factor. If it is reconfigured as Sauto = V2 I1
a 500 V/750 V autotransformer, its efficiency at = 750 × 200 = 150 kVA.
its new rated load at unity power factor will be
 [2007] As current through winding and voltage across
(a) 95.752% (b) 97.851% windings are equal in two-winding transformers and
(c) 98.276% (d) 99.241% autotransformers losses remain same at full-load,
Solution: (c) xSauto cos φ
%η = × 100
xSauto cos φ + losses
A C
1× 150 × 1
I1 I2 = × 100 = 98.276%
1× 150 × 1 + 2.63
V1 V2 Hence, the correct option is (c).
1 5. Two transformers are to be operated in parallel
such that they share load in proportion in their
B D kVA ratings. The rating of the first transformer is
500 kVA and its pu leakage impedance is 0.05 pu
In two winding transformer, If the rating of second transformer is 250 kVA, its
pu leakage impedance is [2006]
V1 = 500 V (a) 0.20 (b) 0.10
V2 = 250 V (c) 0.05 (d) 0.025
50 × 103 Solution: (c)
I1 = = 100 A. The currents carried by two transformers are pro-
500
portional to their ratings if their per-unit imped-
50 × 103
I2 = = 200 A. ances on their own ratings are equal.
250
xS cosφ Z=
2 ( pu ) Z=
1 ( pu ) 0.05 pu
η= .
xS cosφ + losses Hence, the correct option is (c).

Chapter 02.indd 33 11/12/2015 10:02:28 AM


3.34 | Electric Machines

Common Data for Questions 16 and 17: (a) 50.0% (b) 70.7%
A 300 kVA transformer has 95% efficiency at full- (c) 141.4% (d) 200.0%
load 0.8 pf lagging and 96% efficiency at half load, Solution: (b)
unity pf. Copper loss at any load, Pcu = x 2 × Full-load cop-
16. The iron loss (Pi) in kW, under full-load operation per loss = x 2 ⋅ Pfl , cu .
are [2006] x = Fraction of load.
(a) = Pc 4=.12, Pi 8.51  = (b)  Pc 6= .59, Pi 9.21 For maximum efficiency,
(c)  Pc 8=
= .51, Pi 4.12  = (d)  Pc 12 =.72, Pi 3.07  Copper loss = iron loss.
Solution: (c) x 2 Pfl , cu = Pi
xS cos φ x 2 × 600 = 300
Efficiency, η = .
xS cos φ + Pi + x 2 Pc 300
η = 95%, x = 1, cosφ = 0.8, S = 300 kVA .
(i) x= = 0.707 = 70.7%.
600
1× 300 × 0.8 Hence, the correct option is (b).
0.95 = .
(1× 300 × 0.8) + Pi + Pc fl 19. A 50 kVA, 3300/230 V single-phase transformer is
Pi + Pc = 12.68 kW connected as an autotransformer shown in figure.
(1) The nominal rating of the autotransformer will be
η = 96%, x = 0.5, cosφ = 1, S = 30 kVA.
(ii)  [2004]
0.5 × 300 × 1
0.96 = N2
(0.5 × 300 × 1) + Pi + (0.5) 2 Pcu Vout =
3530 V
Pi + 0.25 Pc = 6.25 kW (2) Vin = 3300 V N1
On solving (1) and (2),
=Pi 4=
.12 kW, Pcu 8.51 kW. (a) 50.0 kVA (b) 53.5 kVA
(c) 717.4 kVA (d) 767.4 kVA
Hence, the correct option is (c).
Solution: (d)
1 7. What is the maximum efficiency (in %) at unity pf
load? [2006] I1 I2
(a) 95.1 (b) 96.2
(c) 96.4 (d) 98.1 V1 = 3300 V I 1 I2 V2 = 220 V

Solution: (b)
For maximum efficiency, at upf,
Iron loss = Copper loss. 50 × 103
I1 = = 15.15 A.
3300
Pi = x 2 Pc
50 × 103
4.12 = x 2 (8.51) I2 = = 217.34 A.
230
x = 0.696 When connected as autotransformer,
0.696 × 300 × 1 I out
η max = × 100 = 96.2%
(0.696 × 300 × 1) + 2 × 4.12
230
Hence, the correct option is (b). I in
V out
1 8. A 500 kVA, 3-phase transformer has iron losses
of 300 W and full-load copper losses of 600 W. 15.8 A
V in 3300
The percentage load at which the transformer is
expected to have maximum efficiency is [2004]

Chapter 02.indd 34 11/12/2015 10:02:33 AM


Chapter 2  Transformers  |  3.35

Vin = 3300. Output = x S cos φ .


Vout = 3300 + 230 = 3530 V. S = Rating of the machine.
X = % of full-load.
I out = I 2 = 217.34 A. cos ϕ = Power factor.
I in = I out + 15.15 = 232.49 A. Pi = Iron or core loss.
Pcu = Full-load copper loss.
kVA rating of the autotransformer,
For maximum efficiency,
Sauto = Vin I in
Pi = x 2 Pcu .
= 3300 × 232.49 × 10−3 = 767.4 kVA.
x S cos φ
Hence, the correct option is (d). η max = .
x S cos φ + 2 Pi
20. The resistance and reactance of a 100 kVA,
1× S × 1
11000/400 V, D - Y distribution transformer are 0.02 0.9 = .
and 0.07 pu respectively. The phase impedance of 1× S × 1 + 2 Pi
the transformer referred to the primary is  [2004] Pi = 0.055 S .
(a)  (0.02 + j 0.07) Ω   (b)  (0.55 + j 1.925) Ω
Efficiency at half load, unity power factor.
(c)  (15.125 + j 52.94) Ω  (d)  (72.6 + j 254.1) Ω
Solution: (d) 1
x= .
On primary side, 2
1
Vl −l = 11000 V. × S ×1
η= 2 × 100
VP = 11000 V. 2
1 1
Rated kVA × 103 × S × 1 + 0.055 S +   × 0.55 S
Il = 2 2
3Vl −l  87.8%.
100 × 103 Hence, the correct option is (d).
= .
3 × 11000 22. Figure shows a Y-connected 3-phase distribution
I 100 × 103 transformer used to step down the voltage from
IP = P = .
3 3 × 1000 11000 V to 415 V line-to-line. It has two switches S1
V and S2. Under normal conditions S1 is closed and S2
11000
ZB = P = = 3630 Ω. is open. Under certain superior conditions S1 is open
I P  100 × 103  and S2 is closed. In such a case the magnitude of the
 
 3 × 11000  voltage across the LV terminals a and c is [2003]
HV LV
Pu base impedance = 0.02 + j 0.07 pu A a
Phase Impedance = Base phase impedance
× (0.02 + j 0.07) = (72.6 + j 254.1) Ω. B b
Hence, the correct option is (d). S2
21. A single phase transformer has a maximum efficiency
of 90% at full-load and unity power factor. Efficiency C c
S1
at half load at the same power factor is [2003]
(a) 86.7% (b) 88.26%
(c) 88.9% (d) 87.8% (a) 240 V (b) 480 V
(c) 415 V (d) 0 V
Solution: (d)
Solution: (b)
xS cos φ As the transformer is ∆ − y connected, it means
Efficiency, η = .
xS cos φ + Pi + x 2 Pcu line-top-line voltage on primary side induces line

Chapter 02.indd 35 11/12/2015 10:02:37 AM


3.36 | Electric Machines

to neutral for phase voltage on secondary side due (a) (−10 + j 10) A (b)
(−10 − j 10) A
to transformer. (c) (10 + j 10) A (d) (10 − j 10) A
N1 11 kV 11 3 Solution: (c)
Turn ratio = =a= = × 103.
N2 415 3 415
N1 I1 = N 2 I 2 + N 3 I 3 .
=VAB aV
=a ; VBC aV
=b ; VAC aVc . N2 N
I1 = I 2 + 3 I3 .
When S2 is closed, B and C are at same potential, N1 N1
So, VBC = VB − VC = 0. I1 = a2 I 2 + a3 I 3 .
Applying KVL in D connection, E2  N 2  1 2 1
I2 = =  ⋅ E1 ⋅ =   400∠0°×
R  N1  R 4 10
VAB + VBC + VAC = 0.
⇒ VAC = −VAB . = 20 ∠ 0 A.
0

E3 N  1 1
I3 = =  3  ⋅ E1 =   400∠0°×
Let VAB = 11× 103 ∠0°. − j × c  N1  4 − j 2.5

VAC = −11× 103 ∠0° = 11× 103 ∠180°. = 40∠90°A.


   I1 = 10∠0° + 10∠90° = (10 + j10) A.
Voltage across terminals a and c,
Hence, the correct option is (c).
Vac = Va − Vc
2 4. A 1 kVA, 23 V/100 V, single phase 50 Hz trans-
VAB VAC 1 former having negligible winding resistance and
= − = (VAB − VAC )
a a a leakage inductance is operating under saturation,
415 while 250 V, 50 Hz, sinusoidal supply is connected
= (11× 102 ∠0° − 11× 102 ∠180°)
11 3 × 102 to the high voltage winding. A resistive load is
415 connected to the low voltage winding which draws
= × 11× 103 × 2  480 V. rated current. Which one of the following quanti-
11 3 × 103 ties will not be sinusoidal? [2002]
(a) Voltage induced across the low voltage winding
23. Figure shows an ideal three winding transformer, (b) Core flux
the three windings 1, 2, 3 of the transformer are (c) Load current
wound on the same case as shown. The turns ratio (d) Current drawn from the source
N1 : N 2 : N 3 is 4 : 2 : 1. A resistor of 10 W is connected
Solution: (d)
across winding -2. A capacitor of reactance 2.5 W
A sinusoidal flux wave (required by a sinusoidal
is connected across winding -3. Winding -1 is
applied voltage) demands a magnetizing current
connected across a 400 V, as supply. If the supply
with an harmonic content. But a supply of strictly
voltage phasor V1 = 400∠0°, the supply current
sinusoidal voltage cannot supply a harmonic cur-
phasor I1 is given by [2003]
rent. If a sinusoidal magnetization current is fur-
nished, however the flux wave will fail to reach its
I1 sinusoidal peak value and will become flat topped.
The emf induced by it will then be peaky with third
V1 ∼ 1 N1 N2 2 R = 10 Ω (and other) harmonics. This to produce sinusoidal
N3 flux during saturation, the current drawn from sup-
ply should be peaky wave and not sinusoidal.
3 Hence, the correct option is (d).
25. A 400V/200V/200V, 50Hz three winding trans-
former is connected as shown in figure. The read-
X 0 = 2.5 Ω ing of the voltmeter ‘V’ will be [2002]

Chapter 02.indd 36 11/12/2015 10:02:42 AM


Chapter 2  Transformers  |  3.37

28. A 3-phase delta/star transformer is supplied 6000 V


400 V ∼ on delta connected side. The terminal voltage on
V
50 Hz the secondary side when supplying full-load at
0.8 lagging power factor is 415 V. The equivalent
resistance and reactance drops for the transformer
400:200:200
are, 1% and 5% respectively. The turns ratio of the
(a) 0 V (b) 400 V transformer is [2000]
(c) 600 V (d) 800 V (a) 14 (b) 24
Solution: (a) (c) 42 (d) 20
The 200 turn windings are connected in additive Solution: (b)
polarity. Hence, the output voltage will be 400 V. Percentage voltage regulation = ( R cos φ + X sin φ )
The difference of this 400 V and the voltage in-
duced in first windings (i.e., 400 turns) is same. = 1× 0.8 + 5× 0.6
Hence, the correct option is (a).   = 3.8%.
26. The hysteresis loop of a magnetic material has an Voltage Regulation,
area of 5 cm2 with the scales given as 1 cm = 2 AT
and 1 cm = 50 MWb. At 50 Hz the total hysteresis Induced Voltage − Terminal Voltage
loss is [2001]   =
Induced Voltage
(a) 15 W (b) 20 W
(c) 25 W (d) 50 W 3.8 IV − 415
= = .
Solution: (c) 100 IV
Area under one hysteresis loop 415
  = hysteresis loss/cycle 50 Hz IV = = 431.392 V.
3.8
  = 50 Cycles. 1−
100
Scale: On x-axis: 1 cm = 2 AT.
On y-axis: 1 cm = 50 MWb. 431.392
(IV ) per=
phase = 249.07 V.
Area = 5 cm2. 3
Hysteresis loss 1 cycle = 5 × 2 AT × 50 × 10−3 AT.Wb 6000
−3
Turn=
ratio = 24.089 = 24.
  = 500 × 10 W. 249.07
\ For 50 cycles, hysteresis loss = 500 × 10−3 × 50 Hence, the correct option is (b).
  = 25 W. V1
Hence, the correct option is (c). 29. The windings of a Q kVA, V, three phase
V2
27. A 3-phase transformer has rating of 20 MVA, 220 Delta connected core type transformer are recon-
kV (star)/33 kV(Delta) with leakage reactance nected to work as a single phase transformer. The
12%. The transformer reactance (in ohms) referred maximum voltage and the power ratings of the
to each phase of the LV delta connected side is new configuration are [1999]
 [2001]
3V V1 Q
(a) 23.5 (b) 19.6 (a) 1 , 3Q (b) ,
(c) 18.5 (d) 8.7 3V2 V2 3
Solution: (b) 2V1 2Q
(c) 3 V1 , 2Q (d) ,
(Vphase ) 2 (33 kV) 2 3V2 2V2 3
[ Z bone ]Per phase = = .
[ S bone ]phase 20 Solution: (d)
MVA
3 If we neglect the effect of saturation and failure of
3 × (33) 2 12 insulation (as maximum voltage is being asked),
Z actual = Z bone × Z pu = × = 19.6 Ω.
20 100 the voltage ratings will be three times of their
Hence, the correct option is (b). ­previous values and same will be the kVA ratings.

Chapter 02.indd 37 11/12/2015 10:02:47 AM


3.38 | Electric Machines

The practical problems like failure of insulation Solution: (a)


cannot be incorporated as nothing is mentioned in The autotransformer connections can be made be
the question. made as below:
Hence, the correct option is (d). 100 A
30. A 10 kVA 400 V/200 V single phase transformer
with 10% impedance draws a steady short-circuit 500 V 75 A
line current of [1999] 25 A
(a) 50 A (b) 150 A 400 V
(c) 250 A (d) 350 A
Solution: (c)
The current corresponding to rated applied voltage 10 × 1000
IP = = 25 A.
during short-circuit test on transformer is steady 400
short-circuit current. To get rated short-circuit cur- 400
rent, 10% of rated voltage to be applied, IS = × 25 = 100 A.
100
10 × 103 Rating  = 500 V × 100 A = 50 kVA.
Rated current = = 25 A.
400
Hence, the correct option is (a).
I ∝ V.
3 3. A 10 kVA, 400/200 V, single phase transformer
I 2 V2 with a percentage resistance of 3% and percentage
= .
I1 V1 reactance of 6% is supplying a current of 50 A to
400 × 25 a resistive load. The value of the load voltage is
I2 = .  [1999]
0.1× 400
(a) 194 V (b) 390 V
I 2 = 250 A.
(c) 192 V (d) 196 V
Hence, the correct option is (c). Solution: (a)
31. The percentage resistance and percentage reactance IR cos φ + IX sin φ
% Voltage regulation = × 100
of a 10 kVA, 400 V/200 V, 3-phase transformer are I
  = ( R cos φ + X sin φ ) × 100.
2% and 10% respectively. If the constant losses in

the machine are 1%, the maximum possible per-
Being a resistive load, φ = 0.
centage efficiency of the transformer is [1999]
(a) 98.32 (b) 97.25 % Regulation = 3 × cos 0° + 6 × sin 0°
(c) 96.85 (d) 96.12 = 3% of 200
Solution: (b) 3 × 200
= = 6 V.
   100
Iron loss 0.01
x= = = 0.707. \  Terminal voltage = 200 − 6 = 194 V.
FL copper loss 0.02
0.707 × 1 Hence, the correct option is (a).
η max = × 100 3 4. A 50 Hz transformer having equal hysteresis and
0.707 × 1 + 0.01 + 0.01
eddy current losses at rated excitation is operated
= 97.248% = 97.25%.
at 45 Hz at 90% of its rated voltage. Compared
Hence, the correct option is (b). to rated operating point the core losses under this
condition [1998]
32. A 400 V/100 V, 10 kVA two winding transformer
(a) Reduce by 10%  (b)  Reduce by 19%
is reconnected as an autotransformer across a suit-
(c) Reduce by 14.5%  (d)  Remain unchanged
able voltage source. The maximum rating of such
an arrangement could be [1999] Solution: (c)
(a) 50 kVA (b) 15 kVA Wn1= We1.
(c) 12.5 kVA (d) 8.75 kVA Wn1: Initial hysteresis loss.

Chapter 02.indd 38 11/12/2015 10:02:50 AM


Chapter 2  Transformers  |  3.39

We1: Initial Eddy current loss. current of the transformer is 2 sin(314.16 t − 85°) ,
Total iron loss, Wi1 = Wn1+ We1 = 2Wn1. then magnetization branch impedance will be
approximately [1997]
f 2 = 45 Hz V2 = 0.9 V1 (a) 141∠90 (b) 200 ∠−85
∆f = 90% ∆V = 90% (c) 200 ∠85 (d) 282 ∠−80

V1 (0.9)V1 Solution: (c)
Bmax ∝ ∝ = Constant. Magnetizing branch impedance,
f1 (0.9) f1
 400 
Wn ∝ f . VPh   ∠0°
Zo = =  2
= 200 ∠85°.
We ∝ f 2 . I Ph  2 
  ∠− 85
We 2 = (0.9) 2 We1 Wn 2 = (0.9)Wn1  2
= 0.81We1 = 0.81Wn1 . Hence, the correct option is (c).
Wi1 = We 2 + Wn 2 37. A 3-phase transformer bank consists of three identi-
= 0.81Wn1 + 0.90 Wn1 = 1.71Wn1 . cal 2300/230 V, 15 kVA single-phase transformers
connected in delta/delta. The bank supplies a 20 kVA,
2 Wn1 − 1.71Wn1 unity pf 3-phase load. If one of the single-phase trans-
% Reduction core loss = × 100
2 Wn1 formers develops a fault, and is removed, the load
= 14.5%. carried by each of the two transformers now operat-
Hence, the correct option is (c). ing in open delta will be kVA. [1997]
(a) 10 kVA (b) 20 kVA
35. The low voltage winding of a 400/230 V, 1-phase, (c) 15 kVA (d) None of these.
50 Hz transformer is to be connected to a 25 Hz,
the supply voltage should be [1997] Solution: (a)
(a) 230 V (b) 460 V Maximum kVA supplied,
(c) 115 V (d) 65 V 1
= × (Rated capacity of bank)
Solution: (c) 3
Magnetizing component current demand should 1
= × 3 × 15 kVA  26 kVA.
remain same. If increased, it may damage trans- 3
former due to increased resultant current. In case of bank of similar or identical transform-
V ers, each transformer generally shares the load in
Bmax ∝ 1 . equal proportion.
f
20
V Hence, load shared by one transformer =
Thus   1   is to be maintained constant. Also in a kVA
2
 t  = 10 kVA.
transformer, core flux is constant.
Hence, the correct option is (a).
φf
V ∝ Bmf ∝ . 38. A 200/440 V, 50 kVA single phase transformer
A
operates on 220 V, 40 Hz supply with secondary
V winding. Then [1993]
= K.
f (a) The eddy current loss and hysteresis loss of
the transformer decreases
V2 f 25
= 2 V2 = × 230 = 115 V. (b) The eddy current loss and hysteresis loss of
V1 f1 50 the transformer increases
(c) The hysteresis loss of the transformer increas-
Hence, the correct option is (c).
es while eddy current loss remains the same
36. A voltage v = 400 sin 314.16t is applied to a (d) The hysteresis loss remains the same whereas
1-phase transformer on no-load. If the no-load eddy current loss decreases

Chapter 02.indd 39 11/12/2015 10:02:55 AM


3.40 | Electric Machines

Solution: (c) Then, Eddy current losses depend only on voltage


Eddy current loss, and are independent of frequency. Since, voltage is
constant, losses will remain constant.
Pe ∝ f 2 Bm 2 ∝ ( f Bm) 2 (1)
Hysteresis loss, Pn ∝ f Bm n
V = 2 π f φ m N1
∝ f φm  v n
∝ f ⋅ 
∝ f ( Bm × Ac)  f
∝ f Bm (2) Vn 1
      ∝ f n− 1 ∝ f n− 1
From (1) and (2), Thus, hysteresis loss increases on decreasing the
Pe ∝ V 2 = Constant frequency.
Hence, the correct option is (c).

Chapter 02.indd 40 11/12/2015 10:02:56 AM


Chapter 2  Transformers  |  3.41

Secondary reactance X B = 0.01 Ω , referred to pri-


Five-marks Questions mary,

1. In a single phase 3 winding transformer the turns 0.01


X B′ =
2
= 0.68 Ω.
ratio for primary: secondary: tertiary windings is  4
20 : 4 : I. With the lagging currents of 50 A at a  
 33 
power factor of 0.6 in the tertiary winding find the
Secondary voltage V2 = 400 referred to primary,
primary current and power factor. [2002]
Solution: V2 400
V2′ = = = 3300 V.
I1 N 3  N2   4 
= .    
I 3 N1  N1   33 
 1  The equivalent circuit,
I =   ⋅ 50∠− cos −1 0.6
 20  XA = 05 Ω X1 = 1 Ω X2′ = 0.82 Ω XB′ = 0.68 Ω
= 2.5∠− cos −1 0.6.
Im1 Im Im2

Primary current = 2.5 A. + Xm = 300 Ω ∼ V1
V1 ∼ 2
Power factor = 0.6.
2. A single phase 6300 kVA, 50 Hz, 3300/400 V dis-
tribution transformer is connected between two
50 Hz supply systems, A and B as shown in figure. As V 1 = V 2 = 3300∠0°V, I m1 + I m 2 = I m .
The transformer has 12 and 99 turns in the low and In loops,
high voltage windings respectively. The magnet- V 1 = jI m1 ( X A + X 1 ) + jI m × m
izing reactance of the transformer referred to the
high voltage side is 500 Ω. The leakage reactance = j ( I m1 ( X A + X 1 + X m ) + I m 2 × m).
of the high and low voltage windings are 1.0 Ω 50 Ls = I m1 + 500 I m 2 = − j 3300. (1)
and 0.012 Ω respectively. Neglect the winding In loops,
resistance and core losses of the transformer. The
Thevenin voltage of system A is 3300 V while that V21 = j[ I m 2 ( X B′ + X 2′ ) + I m × m]
of system B is 400 V. The short-circuit reactance
of system A and B are 0.5 Ω and 0.010 Ω respec-
tively. If no power is transferred between A and B, = j[ I m1 × m + I m 2 ( X B′ + X 2′ + X m )].
so that the two system voltages are in phase, find 500 I m1 + 501.5 I m 2 = − j 3300. (2)
the magnetizing ampere turns of the transformer.
[2002] Solving (1) and (2),

A 0.5 Ω 0.01 Ω B I m1 = − j 3.26.

3300 V 400 V I m 2 = − j 3.33.


∼ ∼
50 Hz 50 Hz
Resultant magnetizing current referred to primary
side,
Solution:
I m = I m1 + I m 2 = − j 6.59.
N 2 12 4
= = . So, magnetizing mmf or ampere turns,
N1 99 33
Secondary reactance referred to primary, f m = N1 I m = 99 × 6.59 = 652.41AT 1 .
X2 0.012 In the currents log behind voltage by 90°, Pf = 0,
X 2′ = 2
= 2
= 0.82 Ω.
 N2   4 so P = 0.
   
 N1   33  Hence, no power is transferred.

Chapter 02_Five Marks.indd 41 11/10/2015 6:22:50 PM


3.42 | Electric Machines

3. An ideal transformer has a linear B-H character- d φm


is negative and constant.
istic with a finite slope and a turns ratio of 1:1. dt
The primary of the transformer is energized with
an ideal current source, producing a signal as dfm
shown in figure. Sketch the shape (neglecting the dt 6
scale factor) of the following signals, labeling 1 2 3 5
4 t
the time axis clearly [2001]
i

t V2
1 3 5 7 4
1 2 3 5 6 t

(a) The core flux φ∝ with the secondary of the In s/c condition,
transformer open = im 0=and i2 i.

(b) The open circuited secondary terminal voltage i
V2 (t ) im
s/c
(c) The short circuited secondary current i2 (t ) i Lm V2 = 0
(d) The core flux φsc, with the secondary of trans-
former short circuited
Solution: 03
As im = 0  and  φmsc = 0.
The ideal transformer can be represented by mag-
netizing inductance Lm or reactance X m . 4. Two single phase transformers A and B have the
following parameters.
+
im Transformer A: 400/200 V, 10 kVA, percentge re-
sistance and percentage reactance are 3% and 4%
Lm V2
respectively.
Transformer B: 5 kVA, 400 / 200 V, percenage re-

sistance and percentage reactance are 4% and 3%
Magnetizing current im = i, respectively.
BH curve is linear than B ∝ H . These two transformers are connected in parallel
B ∝ φm . and they share a common load of 12 kW at a power
H ∝ i. factor of 0.8 lag. Determine the active and reactive
Thus, magnetizing waveform will be, power delivered by transformer A. [1999]
Solution:
fm Load shared by transformer A,
3 6
 ZB 
1 2 4 5 t SA =   ⋅ SL
 Z A + ZB 
(4 + j 3) 12
Induced emf in o /c, = ⋅  
dφ (3 + j 4) + (4 + j 3)  0.8 
V2 = − N m .
dt 4 + j3
= 15 ×
For 0 < t < 1, 7 + j7
d φm = 7.5 − j1.07 kVA.
is positive and constant.
dt P = 7.5 kW.
For 1 < t < 3, Q = 1.07 kVAR.

Chapter 02_Five Marks.indd 42 11/10/2015 6:22:55 PM


Chapter 2  Transformers  |  3.43

5. In a 50 kVA, 11 kV/400 V transformer, the iron and full load iron loss
copper losses are 500 W and 600 W respectively Percentage load, x =
under rated conditions. Calculate the efficiency on full load copper loss
unity power factor at full-load. Find the load for 500 5
maximum efficiency and the iron and copper losses = = .
600 6
corresponding to this load [1998]
5
Solution: Thus, load for maximum efficiency = × 50 × 1
6
At full-load efficiency,
= 45.64 kW.

S cos φ Copper losses at maximum efficiency load
η=
S cos φ + Pi + Pcu , µ 2
 5
50 × 1 =   × 600
=  6
50 × 1 + 500 + 600
= 0.978 = 97.8%   = 500 W.
For maximum efficiency, Iron losses = 500 W.

Chapter 02_Five Marks.indd 43 11/10/2015 6:22:57 PM


Chapter 3
Induction Motors
Solution: (c)
One-mark Questions An induction generator always operates at negative
slip, i.e.,
1. An 8-pole, 3-phase, 50 Hz induction motor is
operating at a speed of 700 rpm. The frequency Ns − Nr
of the rotor current of the motor in Hz is = S < 0 Ns < Nr .
Ns
 [2014-S1]
120 × f As generator speed is maintained at constant speed
Solution: Synchronous speed = N s = .
P of 1500 rpm, synchronous speed will be less than
Number of poles, P = 8. 1500 rpm in both cases.
Supply frequency, f = 50 Hz. Ns = 1500 rpm for P = 4 and f = 50 Hz.
For N s < 1500 rpm f1 , f 2 < 50 Hz.
120 × 50
Ns = = 750 rpm. Also, when load decreases, then frequency increases.
8
Hence, f 2 > f1 .
Ns − Nr Hence, the correct option is (c).
Slip, S =
Ns 3. In a constant V/f control of induction motor, the
750 − 700 1 ratio V/f is maintained constant from 0 to base fre-
= = .
750 15 quency, where V is the voltage applied to the motor
∴   Frequency of rotor current, at fundamental frequency f. Which of the follow-
ing statements relating to low frequency operation
1 of the motor is TRUE? [2014-S2]
f 2 = Sf1 = × 50 = 3.33 Hz.
15 (a)  At low frequency, the stator flux increases
2. A 3-phase, 4-pole, self excited induction generator from its rated value
is feeding power to a load at a frequency f1. If the (b) At low frequency, the stator flux decreases
load is partially removed, the frequency becomes from its rated value
f2. If the speed of the generator is maintained at (c) At low frequency, the motor saturates
1500 rpm in both the cases, then [2014-S2] (d) At low frequency, the stator flux remains un-
changed at its rated value
(a) f1 , f 2 > 50 Hz and f1 > f 2
Solution: (b)
(b) f1 < 50 Hz and f 2 > 50 Hz
At low frequencies, effect of resistance cannot be
(c) f1 , f 2 < 50 Hz and f 2 > f1 neglected in comparison to reactance.
(d) f1 > 50 Hz and f 2 < 50 Hz
V/f ratio constant implies that torque is constant.

CH 03.indd 44 11/12/2015 10:02:19 AM


Chapter 3  Induction Motors  |  3.45

But, at low frequencies, because of resistive effect V


(c)
magnitude of air gap flux decreases and hence the
(rs + rr ) + ( X m + xr ) 2
2
maximum torque also decreases at lower frequencies.
Hence, the correct option is (b). V
(d)
4. Leakage flux in an induction motor is [2013] rs + ( X m + xr ) 2
2

(a) Flux that leaks through the machine


(b) Flux that links both stator and rotor windings Solution: (a)
(c) Flux that links none of the windings Starting current in induction motor can be as
(d) Flux that links the stator winding or the rotor large as 5 to 6 times the full-load current. As
windings but not both compared to starting current, exciting current is
Solution: (d) very small.
Some stator flux may not cross the air gap—this Therefore, shunt branch can be neglected in the
flux linking the stator winding but not the rotor circuit modes as given below:
winding is called stator leakage flux.
Similarly, the rotor produced flux, not linking the
Ist rs Xs Xr
stator winding is called rotor leakage flux.
Thus, leakage flux in induction motor links the ei-
ther stator winding or rotor winding but not the both.
Hence, the correct option is (d). V
I st = .
5. The slip of an induction motor normally does not (rs + rr ) + j ( X s + X r )
depend on [2012]
V
(a) Rotor speed   (b)  Synchronous speed I st = A.
(c) Shaft torque   (d)  Core-loss component (rs + rr ) + ( X s + X r ) 2
2

Solution: (d) Hence, the correct option is (a).


N − Nr
Slip = s . 7. A 3-phase squirrel cage induction motor supplied
Ns
from a balanced 3-phase source drives a mechani-
From this, slip depends on synchronous speed (Ns) cal load. The torque–speed characteristics of the
and rotor speed (Nr). motor (solid curve) and of the load (dotted curve)
 As the shaft torque depends upon rotor speed, are shown. Of the two equilibrium points A and B,
therefore slip also depends on shaft torque. which of the following options correctly describes
Slip is independent of core loss. the stability of A and B? [2009]
Hence, the correct option is (d).
6. A balanced 3-phase voltage is applied to a star-
connected induction motor, the phase to neutral
Torque

voltage being V. The stator resistance, rotor resist-


ance referred to the stator, stator leakage reactance, A B
rotor leakage reactance referred to the stator, and
the magnetizing reactance are denoted by rs, rr, rs,
rr and Xm respectively. The magnitude of the start- N
0 1.0
ing current of the motor is given by: [2010]
V
(a)
(rs + rr ) 2 + ( xs + xr ) 2 (a) A is stable B is unstable
(b) A is unstable B is stable
(b) V (c) Both are stable
rs + (rs + X m ) 2
2 (d) Both are unstable

CH 03.indd 45 11/12/2015 10:02:22 AM


3.46 | Electric Machines

Solution: (a) Motor will accelerate and motor speed will keep
on increasing.
IL
So, motor is unstable at B as equilibrium is not get-
A
D ting restored on disturbance.
Torque B
C E F Hence, the correct option is (a).
8. The electromagnetic torque Te of a drive and its
connected load torque TL are shown below. Out of
the operating points A, B, C and D, the stable ones
NC NA ND NE NB NF are [2007]
Speed

Let, Te TL Te
T T TL
Torque developed by motor = Tm. A

Load torque = TL. B


Accelerating Torque, Ta = Tm − TL . Speed
Speed  
At point A,
Tm = TL . Te Te
C
Ta = Tm − TL = 0. T TL T D

So, no acceleration and motor runs at speed NA, TL


with a small disturbance, speed decreases to NC.
Speed   Speed
At point C,
Tm > TL . (a) A, C, D (b) B, C
Ta > 0. (c) A, D (d) B, C, D
So, motor will accelerate and comes to point A. Solution: (a)
At point A, C, D,
When speed increases to ND.
At point D, TL = Te .
Tm < TL . Ta = Te − TL = 0.
Ta < 0. If due to some disturbance speed decreases,
Motor will decelerate and comes to A. Te > TL .
At B, Ta = Te − TL > 0.
Tm = TL .
So, rotor accelerates and speed increases
Ta = 0.
If speed increases due to some disturbances,
With small disturbance, if speed decreases to NE,
Te < TL .
Tm < TL .
Ta = Te − TL < 0.
Ta < 0.
Motor will decelerate and speed will keep on de- So, rotor decelerates and speed decreases.
creasing. So, points A, C, D are stable equilibrium is restored
If disturbance increases speed to NF, if any disturbance occurs.
At point F, At point B,
Tm > TL . Te = TL .
Ta > 0. Ta = Te − TL = 0.

CH 03.indd 46 11/12/2015 10:02:26 AM


Chapter 3  Induction Motors  |  3.47

If speed decreases, due to some disturbance, Pm = Gross power output


  = Air gap power – Rotor copper loss
Te < TL .
= Pg − SPg
Ta = Te − TL < 0.
= Pg (1 − S ).
So rotor decelerates and rotor speed keeps on
Pm
decreasing. = (1 − S ).
Pg
If speed increases,

Te > TL . Hence, the correct option is (b).


Ta = Te − TL > 0. 11. On the torque/speed curve of induction motor shown
in the figure four points of operation are marked as W,
Rotor accelerates, and rotor speed keeps on X, Y and Z. Which one of the represents the operation
increasing. at a slip greater than 1? [2005]
So, point B is unstable since, after disturbance is
applied, equilibrium is not restored.
Y
Hence, the correct option is (a). X
9. For a single phase capacitor start induction motor W
which of the following statements is valid? 0 Speed
[2006] Z
(a) The capacitor is used for power factor im-
provement (a) W (b) X
(b) The direction of rotation can be changed by (c) Y (d) Z
reversing the main winding terminals.
(c) The direction of rotation cannot be changed Solution: (a)
(d) The direction of rotation can be changed by In breaking mode, the motor run in opposite direction
interchanging the supply terminals. to rotating field, i.e., Nr (rotor speed) is negative.

Solution: (b) NS − ( Nr ) NS + Nr
S= = > 1.
If the supply terminals are interchanged, Ia and Im NS NS
will flow in the opposition direction, so torque will
act in same direction. Therefore, direction of rota- Hence, the correct option is (a).
tion will remain same. 1 2. For a linear electromagnetic circuit, the following
Thus, direction of rotation can be changed by re- statement is true [2004]
versing either main winding terminals or auxiliary (a) Field energy is equal to the co-energy
winding but not both. (b) Field energy is greater than the co-energy
(c) Field energy is lesser than the co-energy
Hence, the correct option is (b).
(d) Co-energy is zero.
10. For an induction motor, operating at a slip s, the
Solution: (a)
ratio of gross power output to air gap power is
equal to [2005] 13. The direction of rotation of a 3-phase induc-
tion motor is clockwise when it is supplied with
(a) (1− s )
(1 − s ) 2 (b) 3-phase sinusoidal voltage having phase sequence
(1− s )
(c) (1− s ) (d) A-B-C. For counterclockwise rotation of the
motor, the phase sequence of the power supply
Solution: (b) should be [2004]
Let Pg = Air gap power. (a) B-C-A      (b) C-A-B
Pcu = Rotor copper loss = SPg. (c) A-C-B      (d) B-C-A or C-A-B

CH 03.indd 47 11/12/2015 10:02:28 AM


3.48 | Electric Machines

Solution: (c) 16. List-I lists different applications and List-II lists
To reverse the direction of rotation, phase sequence motors for these applications. Match the applica-
of the supply has to be reversed. For clockwise di- tion with the most suitable motor and choose the
rection, the phase sequence was A-B-C, thus for right combination among the choices given these
counter clockwise direction, the phase sequence after [2003]
has to be C-B-A. List-I List-II
Hence, the correct option is (c).
A.  Food mixer 1. Permanent magnet DC
14. The type of single-phase induction motor having motor
the highest power factor at full-load is [2004]
B. Cassette tape 2. Single-phase induction
(a) Shaded pole type  (b)  Split-phase type
recorder motor
(c) Capacitor-start type  (d)  Capacitor-run type
Solution: (d) C. Domestic water 3.  Universal motor
In capacitor run motors, the motor would operate pump
as a balanced two phase induction motor, back- D. Escalator 4. 3-phase induction motor
ward rotating flux, would therefore, be absent and
the motor would have improved efficiency and bet- 5.  DC series motor
ter operating power factor. 6.  Stepper motor
Hence, the correct option is (d).
Codes:
15. No-load test on a 3-phase induction motor was   A  B   C   D
conducted at different supply voltages and a plot of (a) 3   6   4    
5
input power versus voltage was drawn. This curve (b) 1   3   2    
4
was extrapolated to intersect the y-axis. This inter- (c) 3   1   2    
4
section point yields [2003] (d) 3   2   1    
4
(a) Core loss
(b) Stator copper loss Solution: (c)
(c) Stray load loss A → 3:
(d) Friction and windage loss In food mixer, universal motor is used as it gives
very high speed.
Solution: (d)
Power input at no-load (Po) provided losses only as B → 1:
the shaft output is zero. These losses comprise Pi Permanent magnet DC motor in used in cassette
(iron/core loss) and Pωt (windage and friction loss). tape recorder as it provides fairly constant speed
like DC shunt motor.
Extrapolation
C → 2:
Domestic water pump uses 1ϕ induction motor as
Pout this water pump is of upto 2 kW capacity so 1ϕIM
is best suited.
D → 4:
V(rated) V In case of escalators, lifts, cranes, etc., i.e., for trac-
tion purpose there is requirement of high starting
torque so either DC series motor or 3ϕIM is used.
As voltage is reduced below the rated value, the
core-loss decreases as the square of voltage. Hence, the correct option is (c).
Since, the slip does not increase significantly, the 17. If a star connected, 3 phase squirrel cage induction
windage and friction loss remains almost constant. motor is operated from a supply, the torque that the
motor can now provide while drawing rated cur-
When PoV/SV is extrapolated to V = 0,
rent from the supply [2002]
Gives Pωt as Pi = 0 at zero voltage. (a) Decreases
Hence, the correct option is (d). (b) Increases

CH 03.indd 48 11/12/2015 10:02:29 AM


Chapter 3  Induction Motors  |  3.49

(c) Remains same 20. A 3-phase squirrel cage induction motor has a full-
(d) Increases or decreases depending on rotor load efficiency of 0.8 and a maximum efficiency
resistance of 0.9. It is operated at a slip of 0.6 by applying a
Solution: (a) reduced voltage. The efficiency of the motor at this
operating points is [1998]
R2 60 (a) Less than 0.4
T = 3I 2 ⋅ ⋅
S 2π N S (b) Greater than 0.6
(c) In the range of 0.8 ± 0.1
As the motor is drawing rated current from the (d) None of the above
supply,
Solution: (d)
1 Efficiency of rotor cannot be determined without
T∝ .
NS knowing actually about the fixed losses, because
if losses are not known, we cannot determine me-
1
T∝ . chanical output power.
frequency
Hence, the correct option is (d).
Hence, as frequency increases, torque decreases. 21. When the supply voltage to an induction motor is
Hence, the correct option is (a). reduced by the maximum torque will decrease by
1 8. The following starting method for an induction approximately [1997]
motor is inferior in view of the poor starting torque (a) 5% (b) 10%
per ampere of the line current drawn; [1999] (c) 20% (d) 40%
(a) Direct on line starting Solution (c)
(b) Autotransformer method of starting Te(max) ∝ V 2 .
(c) Series inductor method of starting 2 2
(d) Star-delta method of starting V   0.9V1 
T2 =  2  ⋅ T1 =   ⋅ T1 = 0.81T1 .
Solution: (c)  V1   V1 
Compared to other methods, in this method reduc-
tion in torque is more for the same amount of re- T2 − T1
% Change = = −19%
duction in current. T1
Hence, the correct option is (c). Decrease by 19%
19. Starting torque can be obtained in the case of a Hence, the correct option is (c).
single phase induction motor with identical main 2 2. In an induction motor, if the air gap is increased
and auxiliary windings by connecting [1999]  [1996]
(a) A capacitor across the mains. (a) Speed will reduce
(b) A capacitor in series with the machine. (b) Efficiency will improve
(c) A capacitor in series with the auxiliary winding (c) Power factor will be lower
(d) The main and the auxiliary windings in series (d) Breakdown torque will reduce
Solution: (c) Solution: (c) and (d)
All single phase induction motors have two stator If the air gap increases, leakage flux increases. This
windings, a main (running) winding and an auxil- increased leakage flux will cause the motor current
iary (starting) winding. If the two winding currents more logging in nature and hence pf gets deterio-
are shifted in time phase, a rotating field is cre- rated. Also, as breakdown torque is inversely pro-
ated which is necessary for production of starting portional to leakage reactance, it will also reduce.
torque. The time phase displacement between aux- Hence, the correct option is (c), (d)
iliary winding current Ia and main winding current 23. In case of a split phase motor, the phase shift between
Im is obtained by putting suitable capacitor in series currents in the two windings is around [1996]
with auxiliary winding. (a) 30 degrees (b) 70 degrees
Hence, the correct option is (c). (c) 90 degrees (d) 120 degrees

CH 03.indd 49 11/12/2015 10:02:30 AM


3.50 | Electric Machines

Solution: (a) Solution: (c)


Va When the number of stator slots is equal to the
number of rotor slots or it is it’s integral multiple,
f
then the alignment forces between the stator teeth
Ia and rotor teeth at start may become so strong as to
Im prevent movement of the rotor. This phenomenon
Ia: Auxiliary winding current. is known as cogging.
Im: Main winding current. Cogging is mainly prevented by skewing the ro-
Main winding is highly inductive so lags Va by tor. Skewing of the rotor, results in reduction in vi-
larger angle whereas auxiliary winding is highly bration and noise as the effective air gap becomes
resistive φ ≅ 30°. more uniform.
Hence, the correct option is (a). Hence, the correct option is (c).
24. Unbalanced supply voltage given to a 3-phase, delta- 27. A three phase slip ring induction motor is fed from
connected induction motor will cause [1996] the rotor side with stator winding short circuited.
(a) Zero sequence currents The frequency of the currents flowing in the short
(b) Less heating of the rotor circuited stator is [1993]
(c) Negative sequence components (a) Slip frequency
(d) All of these (b) Supply frequency
(c) Frequency corresponding to rotor speed
Solution: (c)
(d) Zero
Any unbalance in supply system will cause nega-
tive sequence component. Solution: (a)
The frequency of the currents flowing in the short
Hence, the correct option is (c).
circuited stator is slip frequency.
25. An induction motor is fed from a balanced 3-phase
Hence, the correct option is (a).
supply at rated voltage and frequency through a
bank of three single phase transformers connected
in delta-delta. One unit of the bank develops fault Two-marks Questions
and is removed. Then [1995]
(a) Single phasing will occur and the machine 1. A 3 phase, 50 Hz, 6 pole induction motor has a
fails to start rotor resistance of 0.1 Ω and reactance of 0.92 Ω.
(b) Single phasing will not occur but the motor Neglect the voltage drop in stator and assume that
terminal voltage will become unbalanced and the rotor resistance is constant. Given that the full-
the machine can be loaded to the extent of load slip is 3%, the ratio of maximum torque to
57.7% of its rating full-load torque is [2014-S1]
(c) The machine can be loaded to the extent of 57.7% (a) 1.567 (b) 1.712
of its rating with balanced supply at its terminals (c) 1.948 (d) 2.134
(d) The machine can be loaded to the extent of
2 Solution: (c)
66 % of its rating with balanced supply at
3
its terminals R2′ 2 2
Sm = = =
Solution: (c) X 2′ S m S f l 0.1087
+
0.03
+
This is the case of open delta connection which S f l Sm 0 . 03 0 . 1087
gives balanced supply voltage with reduced rating. 2 3
= = .
Hence, the correct option is (c). 3.623 + 0.275 3.898
26. Skew is used in induction motors in order to reduce Tem 3.898
= = 1.949.
torque due to [1994] Tf L 2
(a) Time harmonics  (b) Space harmonics
(c) Slot harmonics   (d) Reverse rotating fields Hence, the correct option is (c).

CH 03.indd 50 11/12/2015 10:02:31 AM


Chapter 3  Induction Motors  |  3.51

2. A 3-phase slip-ring induction motor, provided at rated conditions is 50 A. Neglecting losses and
with a commutator winding, is shown in the figure. magnetizing current, the approximate blocked rotor
The motor rotates in clockwise direction when the line current drawn when the motor is connected to a
windings are closed. 236 V, 57 Hz supply is [2012]
3-phase ac, f Hz (a) 58.5 A (b) 45.0 A
(c) 42.7 A (d) 55.6 A
Solution: (b)
At standstill, the rotor current is
f2
Prime Slip ring induction
mover E2
motor I2 = .
fr R + X2
2

As losses are zero,


f1
E2 E
If the rotor winding is open circuited and the sys- I2 = = 2 .
X jWL
tem is made to run at rotational speed fr with the
help of prime-mover in anti-clockwise direction, I 2 E2 W ′
= × .
then the frequency of voltage across slip rings is I 2′ E2′ W
f1 and frequency of voltage across commutator 236 × 50
brushes is f2, the values of f1 and f2 [2014-S2] I 2′ = × 50 = 45 A.
230 × 57
(a) f + f r and f (b) f − f r and f
Hence, the correct option is (b).
(c) f − f r and f + f r (d) f + f r and f − f r
5. A 3-phase 440 V, 6 pole, 50 Hz, squirrel cage induc-
Solution: (a) tion motor is running at a slip of 5%. The speed of
Frequency of voltage across slip ring = frequency stator magnetic field with respect to rotor magnetic
of voltage induced in rotor winding. field and speed of rotor with respect to stator mag-
For stator frequency let speed = NS. netic field are [2011]
For rotor frequency let speed = Nr. (a) Zero, -5 rpm (b)  Zero, 955 rpm
Thus, frequency of rotor induced emf = f + fr = f1. (c) 1000 rpm, -5 rpm (d)  1000 rpm, 955 rpm
 Also, frequency of induced voltage across commu-
Solution: (a)
tator = Stator frequency = f.
Slip= S= 5% = 0.05.
Hence, the correct option is (a).
Synchronous speed,
3. A 4-pole induction motor, supplied by a slightly
unbalanced 3-phase 50 Hz source is rotating at 1440 120 f 120 × 50
N1 = = = 1000 rpm.
rpm. The electrical frequency in Hz of the inducted P 6
negative sequence current in the rotor is [2013]
(a) 100 (b) 98 Rotor speed = (1 − S ) N S
(c) 52 (d) 48 = (1 − 0.05) × 1000
Solution: (b) = 950 rpm.
120 × 50
NS = = 1500 rpm. The speed of rotor field wrt rotor = N S − N r
14
1500 − 1440 = SN
=1 50 rpm.
S= = 0.04.
1500 It is in direction of stator field.
S ′ = 2 − S = 1.96.
Hence, wrt stator both, the magnetic field rotate at
f ′ = S ′f = 1.96 × 50 = 98 Hz. same speed NS.
Hence, the correct option is (b). Both the magnetic fields are stationary wrt each
4. The blocked rotor current in a 3-phase, star con- other.
nected 15 kW, 4-pole, 230 V, 50 Hz induction motor So, speed of stator field wrt rotor field = 0 rpm .

CH 03.indd 51 11/12/2015 10:02:37 AM


3.52 | Electric Machines

Speed of rotor wrt stator field = N r − N S VS 220


Im = = = 22∠ − 89.427°A.
= 950 − 1000 Z m 0.1 + j10
= −50 rpm. Current through auxiliary winding,
Hence, the correct option is (a).
VS 220
6. A 200 V, 50 Hz, single-phase induction motor has Ia = = = 0.22∠ − 89.942.
Z a 1 + j1000
the following connection diagram and winding
orientations shown. MM3 is the axis of the main Taking Vs as reference,
stator winding (M1M2) and AA3 is that of the aux- V
iliary winding (A1A2). Directions of the winding
axis indicate direction of flux when currents in the
windings are in the directions shown. Parameters
of each winding are indicated. When switch S is
closed, the motor [2009] Im
Ia

M1 Im leads Ia, the fields created by two current also


rm = 0.1 Ω have same difference thereby constituting an un-
Lm = 0.1/pH balanced field system.
ra = 1 Ω
La = 10/pH M2 The result is production of starting torque.
S A1 A2 M Space orientation of field,
220 V A A′ Field due to Ia
50 Hz
Rotor
M′

Field due to Im
(a) Rotates clockwise
The motor rotates in the direction of leading phase
(b) Rotates anticlockwise
to lagging phase.
(c) Does not rotate
(d) Rotates momentarily and comes to a halt In this case, motor rotates anti-clockwise.
Solution: (b) Hence, the correct option is (b).
F = 50 Hz. Common Data for Questions 7 and 8:
Impedance of main winding A 3-phase, 440 V, 50 Hz, 4 pole, slip ring induc-
tion motor is feed from the rotor side through an
Z m = rm + j 2π f Lm autotransformer and the stator is connected to a
0.1 variable resistance as shown in the figure.
= 0.1 + j 2π × 50 ×
π
= (0.1 + j10) Ω.

Impedance of auxiliary winding,

Z a = ra + j 2π fLa t$
Induction
10 Rex
= 1 + j 2π × 50 × . motor
π 3-phase
50 Hz, Supply
Z a = 1 + j1000 Ω.
Auto transformer +220 V
Current through main winding,

CH 03.indd 52 11/12/2015 10:02:40 AM


Chapter 3  Induction Motors  |  3.53

The motor is coupled to a 220 V, separately excited Power Output = PO = (1 − S ) Pin


DC generator feeding power to fixed resistance

= (1 − 0.06)1600 = 1504 W.
of 10 Ω. Two wattmeter method is used to meas-
ure the input power to induction motor. The vari- All losses are neglected,
able resistance is adjusted such that motor runs at Induction motor power output
1410 rpm and the following reading were recorded = DC generator Input
W1 = 1800 W, W2 = −200 W.
 = DC generator Output
7. The speed of rotation of stator magnetic field with
respect to rotor structure will be [2008]   = 1504 W.
(a) 90 rpm in the direction of rotation
∴ I 2 Rex = 1504.
(b) 90 rpm in the opposite direction of rotation
(c) 1500 rpm in the direction of rotation 1504
I= = 12.26 A.
(d) 1500 rpm in the opposite direction of rotation IO
Solution: (c) Hence, the correct option is (c).
120 × f 120 × 50 9. A 230 V, 50 Hz, 4 pole, single-phase induction
N S (syn speed ) = = = 1500 rpm.
P 4 motor is rotating in the clockwise (forward) direc-
tion at a speed of 1425 rpm. If the rotor resistance
If the three phase supply is given to the rotor wind-
at standstill is 7.8 Ω, then the effective rotor resist-
ings through an autotransformer, the three phase
ance in the backward branch of the equivalent cir-
rotor current will generate a rotating field in the
cuit will be [2008]
air-gap, rotating at synchronous speed wrt rotor.
(a) 2 Ω (b) 4 Ω
If rotor is allowed to move, it will rotate as per the (c) 78 Ω (d) 156 Ω
Lenz’s law, opposite to the rotation of the rotating
Solution: (a)
field, decreased the induced voltage in the stator
Rotor resistance at stand still = 7.8 Ω.
winding. Thus, at a particular speed, the frequency of
120 f
the stator circuit will correspond to the slip speed. Synchronous speed, N S =
P
Rotor speed,  N r = 1410 rpm.
120 × 50
N − N r 1500 − 1410 = = 1500 rpm.
Slip,  S= S = = 0.06.         4
NS 1500
Slip, S = 1500 − 1425 = 0.06.
Slip frequency,  S f = 0.06 × 50 = 3 Hz . 1500
120 × 3 Slip of rotor wrt backward field = 2 − S
Slip speed = = 90 rpm.
4       = 2 − 0.05 = 1.95.
As stator magnetic field rotates 90 rpm in the op- R2
posite direction of the rotation of rotor, therefore, Effective rotor resistance =
2(2 − S )
speed of the stator wrt rotor = 1410 + 90 1500 rpm.
Hence, the correct option is (c). 7.8
= = 2 Ω.
    2 × 1.95
8. Neglecting all losses of both the machines, the DC
generator power output and the current through Hence, the correct option is (a).
resistance (Rex) will respectively be [2008] 1 0. A 400 V, 50 Hz, 30 hp, 3-phase induction motor is
(a) 96 W, 3.10 A (b) 120 W, 3.46 A drawing 50 A current at 0.8 power factor lagging.
(c) 1504 W, 12.26 A (d) 1880 W, 13.71 A The stator and rotor copper losses are 1.5 kW and
Solution: (c) 900 W respectively. The friction and windage
losses are 1050 W and the core losses are 1200 W.
90
= = 0.06.
Slip The air-gap power of the motor will be [2008]
1500 (a) 23.06 kW (b) 24.11 kW
Power Input = 1800 − 200 = 1600 W. (c) 25.01 kW (d) 26.21 kW

CH 03.indd 53 11/12/2015 10:02:45 AM


3.54 | Electric Machines

Solution: (c) Common Data for Questions 12–14:


Input power = 3Vl I l cos φ A three phase squirrel cage induction motor has a
starting current of seven times the full-load current
= 3 × 400 × 50 × 0.8 and full-load slip of 5%.
= 27.71 kW. 12. If an autotransformer is used for reduced voltage
Air gap power (Pg) is the power that is transformer starting to provide 1.5 per unit starting torque, the
from the stator to the rotor via the air gap magnetic autotransformer ratio (%) should be [2007]
field. (a) 57.77% (b) 72.56%
Pg = Input Power-core loss–core loss-stator (c) 78.25% (d) 81.33
Solution: (c)
= 27.71 − 1.5 − 1.2
= 25.01 kW. I st = 7 ⋅ I f l .

Hence, the correct option is (c). S f l = 0.05.


11. A 400 V, 50 Hz, 4 pole, 1400 rpm, star connected squir- In case of autotransformer, starting torque can be
rel cage induction motor has the following parameters expressed as ratio of full-load.
referred to the stator: R = 1.0 Ω, X s = X r′ = 1.5 Ω.
Neglect stator resistance and core and rotational losses 2
TS I 
of the motor. The motor is controlled from a 3-phase = x 2 ⋅  SC  ⋅ S f l .
voltage source inverter with constant V/f control. The Tf l  I FL 
stator line-to-line voltage (rms) and frequency to
obtain the maximum torque at starting will be:[2008] x : Autotransformer ratio.
(a) 206. V, 2.7 Hz (b) 133.3 V, 16.7 Hz TS : 1.5T f l .
(c) 266.6 V, 33.3 Hz (d) 323.3 V, 40.3 Hz 2
Solution: (b) 1.5T f l I 
Rr′ ⇒ = x ⋅  Sc  ⋅ 0.05.
2

For maximum torque, slip = S m = . Tf l  I fl 


 

X Sm + X Sm
For Starting torque S m = 1 . x 2 = 0.6122.
′ = Rr ′. x = 0.7825 = 78.25%
X Sm + X rm
2π f m LS + 2π f m Lr ′ = 1. Hence, the correct option is (c).

1 13. If a star-delta starter is used to start this induction


fm = . motor, the per unit starting torque will be [2007]
2π ( LS + Lr )
(a) 0.607 (b) 0.816
Xs 1.5 (c) 1.225 (d) 1.616
LS = = .
2π × 50 2π × 50
Solution: (b)
1.5
Lr ′ = . In case of star delta transformer,
2π × 50
1 50 1
∴ fm = = = 16.7 Hz. x= .
1.5 1.5 3 3
+
50 50 2 2
TS 1  I sc  1  7I f l 
Vl 400 = 
  ⋅ S f l = ⋅   ⋅ 0.05.
In comt. V/f control method = = 8. Tf l 3  I f l  3  I fl
fo 50 
V TS = 0.816T f l .
∴ 2 = 8.
f2
TS = 0.816 pu
V2 = f 2 × 8 = 16.7 × 8 = 13.3 V.
Hence, the correct option is (b). Hence, the correct option is (b).

CH 03.indd 54 11/12/2015 10:02:50 AM


Chapter 3  Induction Motors  |  3.55

14. If a starting torque of 0.5 per unit is required then 1.5 2Sm
the per unit starting current should be [2007] ⇒ = .
3 S m2 + 1
(a) 4.65 (b) 3.75
(c) 3.16 (d) 2.13 S m 2 − 4 a + 1 = 0.
Solution: (c) S m = 3.73 or 2.679.
TS = 0.5T f l .
 S m = 3.73 is neglected because for this value of
S f l = 0.05. slip, motor works in breaking mode.
In case of direct line starting, Therefore, for motoring mode, Sm = 0.2679 = 26.79%.
x = 1. Hence, the correct option is (d).
2 16. The speed of a 4-pole induction motor is controlled
T I  by varying the supply frequency while maintaining
⇒ S =  sc  ⋅ S f l .
T f l  I f l  the ratio of supply voltage to supply frequency (V/f  )
2
constant. At rated frequency of 50 Hz and rated volt-
0.5T f l I  age of 400 V its speed is 1440 rpm. Find the speed at
=  sc
 I fl  ⋅ 0.05. 30 Hz, if the load torque is constant [2006]
Tf l   (a) 882 rpm (b) 864 rpm
I sc = 3.16 I f l . (c) 840 rpm (d) 828 rpm
Solution: (c)
or I S = 3.16 pu V
As, = constant,
Hence, the correct option is (c). F
15. A 3-phase squirrel cage induction motor has a V1 V2
= .
starting torque of 150% and a maximum torque of F1 F2
300% with respect to rated torque at rated voltage
F   30 
and rated frequency. Neglect the stator resistance V2 =  2  ⋅V1 =   × 400 = 240 V.
and rotational losses. The value of slip for maxi- F
 1  50 
mum torque is [2007] Torque developed in induction motor,
(a) 13.48% (b) 16.42%
V 2 R2
(c) 18.92% (d) 26.79% 3
T= ⋅ S .
Solution: (d) WS  R2′  2
 S  + ( X 2′ )
2

TS 2  
= .
Tmax S Sm In stable region slip is low.
+
Sm S R2′
⇒ >> X 2′ .
S
Where Sm is slip for maximum torque,
V 2 R2
Tst 2 2Sm 3 3 SV 2
⇒ =
S
= . ⇒ T= ⋅ S 2= ⋅ 2 .
Tmax 1 S m2 + 1 WS  R1  WS R2′
+ m 2
Sm 1  
 S 
Given, SV 2
T∝ .
Tst = 1.5Trated. F
2
Tm = 3Trated. T1 S1  V1  F2
=   × .
Tst 1.5 T2 S 2  V2  F1
= .
Tm 3 As load torque is constant, T1 = T2 ,

CH 03.indd 55 11/12/2015 10:02:54 AM


3.56 | Electric Machines

2 No-Load Test: 400 V 6A 1002 W .


V   F 
S 2 = S1  1  ⋅  2  Blocked Rotor Test: 90 V 15A 762 W .
 V2   F1 
Neglecting copper loss in No-Load Test and core
2
 400   30  loss in Blocked Rotor Test, estimate motor’s full-
= 0.04 ×   ×   = 0.067. load efficiency. [2006]
 240   50 
(a) 76% (b) 81%
120 × F (c) 82.4% (d) 85%
NS = .
P Solution: (b)
120 × 50 No-load= =
losses 1002 W 1.002 kW.
N S1 = = 1500 rpm.
4 This is independent of load.
1500 − 1440 Copper loss at 15 A,
S1 = = 0.04.
1500 Pcu = 762 W.
120 × 30 Full-load at 20 A , i.e., full-load,
NS 2 = = 900 rpm.
4 2
 20 
N r 2 = N S 2 (1 − S 2 ) = 900 (1 − 0.067) = 840 rpm. Pcu, sl =   ⋅ 762 = 1.354 kW.
 15 
Hence, the correct option is (c). Total losses at full-load = P + Pcu, fl = 1.002 + 1.354
1 7. A 3 phase, 4 pole, 400 V, 50 Hz, star connected
.356 kW P2 .
= 2=
induction motor has following circuit parameters
r1 = 1.0 Ω, r2 = 0.5 Ω, x1 = x2′ = 1.2 Ω, xm = 35 Ω. = P=
Output power on full-load O 10 kW.
The starting torque when the motor is started Input power on full-load = Pi = P2 + Po
direct-on-line is (use approximate equivalent cir-
cuit model) [2006]           = 10 + 2.356 = 12.356 kW.
(a) 63.6 Nm (b) 74.3 Nm 10
Efficiency, η = × 100 = 81%
(c) 190.8 Nm (d) 222.9 Nm 12.356
Solution: (a) Hence, the correct option is (b).
Line to line voltage, VL = 400 V. 19. Under no-load condition, if the applied voltage to
an induction motor is reduced from the rated volt-
400
Phone to neutral voltage, VP = V. age to half the rated value. [2005]
3 (a) The speed decreases and the stator current in-
120 × 50 creases
Synchronous speed N S = = 1500 rpm.
4 (b) Both the speed and the stator current decreases
2π × N S 2π × 1500 (c) The speed and the stator current remain practi-
WS = = = 157.08 rad/sec. cally constant
60 60
(d) There is negligible change in the speed but the
S st = 1. stator current decreases
3 V22P r2′ Solution: (d)
⇒ Tst = ⋅
WS (r1 + r2′) + ( X 1 + X 2′ ) 2
2
Let no-load current = I O
 400 
2
Equivalent circuit at no-load:
  × 0.5 Io
3  3
= × = 63.6 N-m.
157.08 (1 + 0.5) 2 + (1.2 + 1.2) 2
Hence, the correct option is (a).
Rc Xo
18. A 3 phase, 10 kW, 400 V, 4 pole, 50 Hz, star con-
nected induction motor draws 20 A on full-load. Its
no-load and blocked rotor test data are given below,

CH 03.indd 56 11/12/2015 10:02:59 AM


Chapter 3  Induction Motors  |  3.57

V (a) Low starting torque  (b)  Quick acceleration


IO = .
RC + jX φ (c) High efficiency    (d)  Reduced size
IO ∝ V . Solution: (b)
For induction motor,
If voltage is reduced, no-load current reduces.
SV 2 Total torque, Ttotal = T f − Tb .
Torque ∝ 1 .
R2 T f = Torque developed due to forward field.
1 Tb = Torque developed due to backward field.
S∝ 2.
V1
Im 2 R2 1 1 
Thus, when voltage is halved, slip becomes one ⇒ Ttotal =  − .
2WS  S 2 − S
fourth, but slip is extremely low at no-load, so
there is negligible change is speed. Ttotal ∝ R2 .
Hence, the correct option is (d). Higher the rotor resistance (R2), more is the total
20. A 3-phase cage induction motor is started by torque developed (Ttotal) which results in quick
direct-on-line (DOL) switching at the rated volt- acceleration.
age. If the starting current drawn is 6 times the Hence, the correct option is (b).
full-load current, and the full-load slip is 4%, then 22. Assertion (a): Under V/f control of induction
ratio of the starting developed torque to the full- motor, the maximum value of the developed torque
load torque is approximately equal to [2005] remains constant over a wide range of speed in the
(a) 0.24 (b) 1.44 sub-synchronous region.
(c) 2.40 (d) 6.00
Reason (b): The magnetic flux is maintained
Solution: (b) almost constant at the rated value by keeping the
Let the starting current = I st . ratio V/f constant over the considered speed range.
Full-load current = I f l .  [2005]
Given, I st = 6 I f l . (a) Both [A] and [R] are true and [R] is the correct
reason for [A]
Slip at full-load, S f l = 0.04.
(b) Both [A] and [R] are true but [R] is not the
Torque in induction motor is given by, correct reason for [A]
3 R′ (c) Both [A] and [R] are false
T= ⋅ ( I 2 )2 ⋅ 2 . (d) [A] is true but [R] is false
WS S
Full-load torque, Solution: (a)
3 R′ Since, V/f is constant,
Tf l = ( I 2 f l )2 ⋅ 1 . Then at any frequency f,
WS S fl
3 R′  f 
Tst = ⋅ ( I st ) 2 ⋅ 2 . V =   ⋅Vo .
WS S st  fo 
3  f 
Tst = ⋅ ( I st ) 2 ⋅ R2′ . X 2′ =   ⋅ X 20
′.
WS
 fo 
Tst  I 2′ st   f 
⇒ =  sfl WS =   ⋅W50 .
T f l  I 2′ sl   fo 
= 62 × 0.04 = 1.44.   f 2 
Hence, the correct option is (b).  0.5   Vo2 
f 3 0.5Vo2
⋅   o  
3
2 1. In a single phase induction motor driving a fan Tmax =
 f   f   W ⋅ X′ .
=
load, the reason for having a high resistance rotor   50⋅ W  ⋅
  20 X ′ 50 20

f
 1  f 
is to achieve [2005]   o 

CH 03.indd 57 11/12/2015 10:03:03 AM


3.58 | Electric Machines

Then, torque is constant, independent of f. Current in auxiliary winding,


Hence, the correct option is (a). V 230∠0°
Ia = =
2 3. A rotating electrical machine having its self- Za  1 
inductance of both the stator and the rotor wind- 8− j − 6
 ω C 
ings, independent of the rotor position will be
definitely not develop [2004]  1 
 −6
−1 ω C
(a) Starting torque = I a ∠ tan  .
(b) Synchronizing torque  8 
(c) Hysteresis torque  
(d) Reluctance torque
Capacitor is provided to make angle between Ia
Solution: (d) and Im90°.
N S2 N r2 So,  ∠I m + ∠I a = 90°.
=LS = ; Lr .
RI S RI r
∠I a = 90° − 33.7° = 56.3°.
Where,
 1 
Ls and Lr are self inductances of stator and rotor. Ns  −6
−1 ω C
and Nr are number of turns energized stator and rotor, tan   = 56.3.
 8 
RIs = Reductance seen by stator flux.  
RIr Reductance seen by rotor flux. 1
= 18.
As Ls and Lr are independent of rotor position, the ωC
reluctances RIs and RIr does not vary with the rotor ω = 2π f = 2π × 50 rad/sec.
movement.
C = 176.84 µF.
1 2 dLS 1 dL
IS and I r2 r become zero.
2 dθ 2 dθ Hence, the correct option is (a).
Hence, the correct option is (d). 2 5. A 400 V, 15 kW, 4 pole, 50 Hz Y-connected induc-
24. A single-phase, 230 V, 50 Hz, 4 pole, capacitor- tion motor has full-load slip of 4%. The output
start induction motor has the following stand-still torque of the machine at fuH load is [2004]
impedances: (a) 1.66 Nm (b) 95.50 Nm
(c) 99.47 Nm (d) 624.73 Nm
Main winding Z m = 6.0 + j 4.0 Ω.
Solution: (c)
Auxiliary winding  Z a = 8.0 + j 6.0 Ω.
The value of the starting capacitor required to pro- 120 × F 120 × 50
NS = = = 1500 rpm,
duce 90° phase difference between the currents in P 4
the main and auxiliary windings will be [2004]
(a) 176.84 μF (b) 187.24 μF Torque developed, T = P
(c) 265.26 μF (d) 280.86 μF WS (1 − S )
Solution: (a) 15 × 103 × 60
=
V 230 ∠ 0° 2π × 1440
Im = = = 31.89 ∠ − 33.7°A.
Zm 6 + j4  
= 99.47 N - m.
Capacitor C will be connected in series with auxiliary Hence, the correct option is (c).
winding, so modified auxiliary winding impedance, 2 6. Following are some of the properties of rotating
electrical machines [2003]
1  1 
Za = 8 + j6 − = 8+ j6 −  P. Stator winding current is DC, rotor winding
ωC  ωC 
current is AC
 1  Q. Stator winding current is AC, rotor winding
= 8− j − 6 .
 ω C  current is DC

CH 03.indd 58 11/12/2015 10:03:06 AM


Chapter 3  Induction Motors  |  3.59

R. Stator winding current is AC, rotor winding C Stator


current is AC c Airgap
S. Stator has salient poles and rotor has commutator
Rotor
T. Rotor has salient poles and sliprings and stator B b D Fs
is cylindrical Stator mmf axis
d
U. Both stator and rotor have poly-phase windings
d
DC machines, synchronous machines and induc- a Fr
tion machines exhibit some of the above properties Rotor mmf axis
A
as given in the following table. Indicate the correct
combination from this table

DC Synchronous Induction The following table gives four sets of statements


Machines Machines Machines as regards poles and torque. Select the correct
set corresponding to the mmf axes as shown in
(a) P, S Q, T R, U figure.
(b) Q, U P, T R, S
(c) P, S R, U Q, T Stator Stator Rotor Rotor
Surface Surface Surface Surface
(d) R, S Q, U P, T
ABC CDA abc cda
Solution: (a) froms froms froms froms Torque
DC Machines → P, S. (a) North South North South Clock
P: Stator winding, i.e., field winding current is DC Pole Pole Pole Pole wise
and current in rotor winding, i.e., current in arma- (b) South North North South Counter
ture coil is AC, with the help of commutator and Pole Pole Pole Pole Clock
brush, it is converted to DC. wise
S: Stator has salient poles to have  B −  wave closer to (c) North South South North Counter
rectangular and rotor is provided with commutator Pole Pole Pole Pole Clock
to convert AC generated emf in armature into DC. wise
Synchronous Machine → Q, T (d) South North South North Clock
Q: Rotor field winding is supplied by DC and induced Pole Pole Pole Pole wise
current in 3φ winding is 3φ balanced AC currents.
 T: The DC field winding on rotor is supplied Solution: (c)
through slip ring if can be salient pole type or cy- For a particular bar magnet or coil, the direction
lindrical but stator is always cylindrical. of flux or mmf is from N-pole to S-pole outside
Induction Machine → R, U. the bar magnet or coil and direction of flux of
R & U: Both rotor and stator are wound for 3φ mmf is from S-pole to N pole inside the magnet
balanced winding and carry AC currents. or coil.
Hence, the correct option is (a). Torque is always produced in such a direction that
27. When stator and rotor windings of a 2-pole rotat- the rotor field axis always tries to align with stator
ing electrical machines are excited, each would field axis.
produce a sinusoidal mmf distribution in the air- Hence, the correct option is (c).
gap with peak values Fs and Fr respectively. The
rotor mmf lags stator mmf by a space angle δ at 28. A 3-phase induction motor is driving a constant
any instant as shown in figure. Thus, half of sta- torque load at rated voltage and frequency. If
tor and rotor surfaces will form one pole with both voltage and frequency are halved, follow-
the other half forming the second pole. Further, ing statements relate to the new condition if sta-
the direction of torque acting on the rotor can be tor resistance, leakage reactance and core loss
clockwise or counter-clockwise. [2003] are ignored [2003]

CH 03.indd 59 11/12/2015 10:03:07 AM


3.60 | Electric Machines

1. The difference between synchronous speed and T1 = T2 .


actual speed remains same S1V12 S V2
2. The airgap flux remains same = 2 2 .
WS 1 R2′ WS 2 R2′
3. The stator current remains same
4. The pu slip remains same among the above, 2
WS 1 2
correct statements are (WS ) 2  V1  2  V1 
S2 = ×   S1 = ×  ⋅ S1 .
(a) All (b) 1, 2 and 3 (WS )1  V2  WS 1  V1 / 2 
(c) 2, 3 and 4 (d) 1 and 4 1
Solution: (b) S2 = × 4 × S1 = 2 S1 .
2
For induction machine, V ∝ f φ . Difference between synchronous speed and actual
V speed,
V2 = 1 .
V2 WS − Wr = SWS .
F1 (WS )1 − (Wr ) 2 = S 2 (WS ) 2
F2 = .
2 (WS )1
V1 F1φ1 = 2 S1 ×
= S1 (WS )1 .
= . 2
V2 F2φ2 Therefore, the difference between synchronous
V   F   V /2   F  speed and actual speed remains same.
φ2 =  2  ⋅  1  ⋅ φ1 =  1  ⋅  1  ⋅ φ1 = φ1 . Stator current,
 V1   F2   V1   F1 / 2 
So, air gap flux remains the same. V
IS = .
2
( X 2′ )1 =  Nominal rotor stand still reactance re-  R2 
 S  + ( X 2′ )
2

ferred to stator (at frequency F1 ).  


F  As R2′ /S >> X 2′
( X 2′ ) 2 =  2  ⋅ ( X 2′ )1
 F1 
SV
 F1  IS = .
  R21
(X ′)
=  2  ⋅ ( X 2′ )1 = 2 1 . V1
 F1  2 I S 1 = S1 .
R2′
 F1 
 F2    (W ) V2
(WS ) 2 =   ⋅ (WS )1 =  2  ⋅ (WS )1 = S 1 . I S 2 = S2 ⋅
F
 1 F
 1 2 R2′
Torque developed in induction motor,  V1 
V 2 R2  2  SV
= (2S1 ) ×   = 1 11 .
3 S R2′ R2
T= ⋅ .
WS  R2′  2
I S1 = I S 2 .
 S  + ( X 2′ )
2

 
For normal steady state operating region, slip in Therefore, stator current remains the same.
very low, Hence, the correct option is (b).
R2 2 9. A single phase induction motor with only main
>>> X 2′ . winding excited would exhibit the following
S
response ar synchronous speed [2003]
V 2 R2
(a) Rotor current is zero
3 3SV 2
T= ⋅ S 2= . (b) Rotor current is non-zero and is at slip frequency
WS  R2′  WS R2′ (c) Forward and backward rotating fields are equal
 S 
  (d) Forward rotating field is more than the back-
As load torque is constant, ward rotating field

CH 03.indd 60 11/12/2015 10:03:09 AM


Chapter 3  Induction Motors  |  3.61

Solution: (d) 120 × 50


Synchronous speed, N s = = 1000 rpm.
Under running condition, 6
Forward induced emf = S f E2 f . 1000 − 975
S=
Slip,  = 0.025.
Backward induced emf  = Sb E2b . 1000
S f =  Forward slip. Gross power  = (1 − s ) × 39 kW = 38.025 kW.
Sb =  Backward slip. New Power = 38.025 − 2 = 36.025 kW.
Let, current due to forward field  = I 2 f . 36.025
Efficiency, η = = 0.90 = 90%.
And current due to backward field = I 2b . 40
Hence, the correct option is (b).
I 2 f < I 2b 32. A three phase, wound rotor induction motor is to
⇒ φ 2 f < φ2 b be operated with slip energy in the constant torque
mode, when it delivers an output power Po at slip s.
⇒ φRf > φRb .
Then theoretically, the maximum power that is
available for recovery at the rotor terminals, is
Where, φRf = φ f − φ2 f .
equal to [2000]
φRb = φb − φ2b. (a) Po (b) Po s
Where, φ2 f =  Rotor forward flux. P P ⋅S
(c) o (d) o
φ2b =  Rotor backward flux. (1− s ) (1 − s )
φ f =  Stator forward flux. Solution: (d)
φb =  Stator backward flux. Let, the input power be Pin ,
Hence, the correct option is (d). Gross power, Po = (1 − s ) Pin .
s
30. A 3-phase, 4-pole squirrel cage induction motor Slip power which can be recovered  = sPin = Po .
has 36 stator and 28 rotor slots. The number of 1− s
Hence, the correct option is (d).
phases in the rotor is [2000]
33. An induction motor having full-load torque of
(a) 3 (b) 9
60 Nm when delta-connected develops a starting
(c) 8 (d) 7
torque of 120 N-m. For the same supply voltage, if
Solution: (c) the motor is changed to star-connection, the start-
Number of phases on rotor ing torque developed will be [1996]
(a) 40 Nm (b) 60 Nm
Number of slot on rotor 28 (c)
= = = 7. 90 Nm (d) 120 Nm
Number of poles 4
Solution: (a)
Hence, the correct option is (c). When the connection is delta ( I L ) ∆ = 3 ( I P ) ∆ .
31. The power input to a 415 V, 50 Hz, 6 pole, 3-phase When the connect is star ( I L ) r = ( I P ) r .
induction motor running at 975 rpm is 40 kW. The
stator losses are 1 kW and friction and windage losses Tst ∝ (Tst ) 2 .
2
total 2 kW. The efficiency of the motor is [2000] (Tst ) r  ( I P ) r 
(a) 92.5% (b) 90% =  .
(Tst ) ∆  3I P 
(c) 91% (d) 88%
Solution: (b) 1
(Tst ) r = × 120 = 40 N-m.
Air gap power = stator output = 40 kW - 1 kW 3
= 39 kW. Hence, the correct option is (a).

CH 03.indd 61 11/12/2015 10:03:15 AM


3.62 | Electric Machines

frequency, rated voltage and rated horse-power, has


Five-marks Questions a speed of 1175 rpm and an efficiency of 92.1%.
Determine the new operating speed if a system
1. The rotor of a three phase, 5 kW, 400 V, 50 Hz, slip disturbance causes 10% drop in voltage and 6%
ring induction motor is wound for 6 pole while its drop in frequency. Assume the friction, windage
stator is wound for 4 poles. The approximate aver- and stray power losses remain constant [2000]
age no-load steady state speed when this motor is
Solution:
connected to 400 V, 50 Hz supply is [2002]
As we know,
Solution:
180 SE 2 R
The above question violates the basic working princi- Torque ( Z ) = ⋅ 2 221 2 .
ple of the induction motor, where stator poles ≠ rotor 2π N s ( R2 + S X 1 )
poles and hence the motor will not start or work ac-
120 × 60
cording to given set of data. Ns = = 1200 rpm.
6
2. A 440 V, 50 Hz, 6 pole, 960 rpm star connected
induction machine has the following per phase N − Nr
= s
Slip 
parameters referred to the stator: Rs = 0.6 W, Ns
R1 = 0.3 W, Xs = 1 W = X1. The magnetizing reac-
tance is very high and is neglected. The machine is 1200 − 1175
= = 0.020.
connected to the 440 V, 50 Hz supply and a certain   1200
mechanical load is coupled to it. It is found that
the magnitude of the stator current is equal to the Since, slip is small, neglecting S 2 X 22 term,
rated current of the machine but the machine is SE12 R2
Torque ∝ .
running at a speed higher than its rated speed. Find R22
the speed at which the machine is running. Also
find the torque developed by the machine. [2002] Now, it is given F2 = 0.94 × F1 and V2 = 0.90 × V1 .
Solution: 120 × 0.94 × 60
Ns = = 1128.
Given, 6
50 Hz; 6 Pole induction machine. S1V12 S 2V22
= .
120 × F 120 × 50 N s1 Ns2
Ns = = = 1000 rpm.
P 6 0.0208 × V12 S 2 × (00.9) 2 × V12
= .
N − Nr 1200 1128
= s
Slip 
Ns 1128 × 0.0208
S2 = = 0.024.
1000 − 960 0.81× 1200
= = 0.04.
  1000 N 2 = 1128(1 − 0.024)
440 = 1128 × 0.97 = 1100 rpm.
Current ( I 2 ) = .
 R′ 
3  R1 + 2  + [ X 1 + X 2 ]2 \  New operating speed of motor N2 = 1100 rpm.
 S 
4. A 3 kW, 400 V/200 V, Delta/Star, 50 Hz, three
440 phase, 6-pole induction motor is found to draw a
I1 = = 30.447 Amp. line current of 25 A at a power factor of 0.4, when
2
 0.3  a blocked rotor test is conducted at the rated
3 0.6 + + (1 + 1) 2
 0.04  voltage. Determine the stator and rotor winding
resistance in ohms per phase, if the torque devel-
3. A 230 V, 20 hp, 60 Hz, 6-pole, 3-phase induction oped by the motor under the above conditions is
motor driving a constant torque load at rated 25 N-m. [1999]

CH 03_five marks question.indd 62 11/9/2015 5:55:34 PM


Chapter 3  Induction Motors  |  3.63

Solution: Mechanical power developed = 50 hp × 746 Walts


V  200/ 3 
Transformer ratio T1 = 2 = 
V1  400 
 = 0.288.   = 37.3 kW.

120 × F 120 × 50 Total power developed = (37.3 + 1.2) kW


Ns = = = 1000 rpm.
P 6 = 38.5 kW.

VBR Rotor Cu loss S
Impedance/Phase = Z = = .
I BR Total mech Power (1 − S )
= [400/25/ 3 ] 0.04
= 27.7 Ω. Copper loss = × 38.5
1 − 0.04
ROr = Z cos φ = 27.7 × 0.4 = 11.08 Ω/Ph.   = 1604 Watts.
YOr = Z sin φ = 25.38 Ω/Ph.
Rotor copper loss = 1604 − 300 Watts
Torque produced = Rotor input × WS = 1304 Watts
R2 60 = 3I R2 R.
= 3 × I r2 × .
  S 2π N S
Now, 3 × (40) 2 × R = 1304.
1304
 400   400  R= .
I r = I1 ×   = 25 ×   = 86.6 A. 1600 × 3
 200/ 3   200/ 3  R = 0.27 Ω/Phase.
60 6. A 3-phase, 20 kW, 400 V, 1470 rpm, 50 Hz squirrel
25 = 3 × (86.6) 2 × R2 ×
 (S = 1, Since,
2 × 3.14 × 1000 cage induction motor develops a torque of ­100 N-m
Ns − Nr N = 0 connected to a 30 Hz supply, for keeping the same
; r blocked rotor).
Ns air-gap flux, then supply voltage should be ….V and
for the same load torque, the new speed will be rpm.
151000  [1997]
= R2 .
3 × (86.6) 2 × 60 Solution:
R2 = 0.11 Ω/Ph . We know,
5. A 50 hp, 6 pole, 50 Hz slip-ring induction motor Voltage
Flux = .
runs at 960 rpm on full-load with a rotor current Frequency
of 40 A. Allowing 300 W for the copper loss in Given,
the short-circuiting gear and 1200 W for mechani- V1 = 400 Volts.
cal losses, find the resistance R2 per phase of the
3-phase rotor winding. [1998] F1 = 50 Hz.
Solution: F2 = 30 Hz.
Given, V2 = ?
Nr = 960 rpm 50 HP and Pole 50 Hz motor. Flux is kept constant.
120 × 50 V1 V2
NS = = 1000 rpm. = .
6 F1 F2
1000 − 960 400 V2
=
Slip  = .
1000 50 30
40 400 × 30
= = 0.04. V2 = = 240 Volts.
1000 50

CH 03_five marks question.indd 63 11/9/2015 5:55:40 PM


3.64 | Electric Machines

Now,  I2 
120 × 50 T ∝ 
P= = 4.08 ≈ 4.  s 
1470 2 2
Now, Tst  I st   XI 
=   S fL =  sc  S fl
120 × 50  
T fL  I fl   I
Ns (Synchronous speed) = = 1500 rpm.  fL 
4 2
Ns − Nr  0..612 × 266.67 
Slip =
=  × 0.05
Ns  72.16 
Tst
1500 − 1470 30 1 = 0.256
= = = . T fL
  1500 1500 50
Torque ∝ Slip × (Voltage)2. 8. Out of the several characteristics shown in figure,
Here, Torque = constant. identify the appropriate ones to match the follow-
ing for a 3-phase induction motor. [1996]
S1V12 = S 2V22 . x-axis   y-axis
1 (a) Load  Efficiency     (P) Curve I
× (400) 2 = S 2 × (240) 2 .
50 (b)  Speed   Current     (Q)  Curve II
3200 (c)  Speed   Power factor   (R)  Curve III
S2 = = 0.05.               (S) Curve IV
57600
              (T) Curve V
N 2 = N s (1 − S 2 )
= 1500(1 − 0.05) = 1425 rpm. III
II
7. A 50 kVA, 400 V, 3-phase, 50 Hz squirrel cage
induction motor has full-load slip of 5%. It is
started using a tapped autotransformer. If the max- IV
imum allowable supply current at the time of start-
ing is 100 A, then calculate the tap position and the I
ratio of starting torque to full-load torque. [1997]
Solution: 0 25% 50% 75% 100%
Squirrel cage induction motor full load slip
5 Solution:
( S n ) = 5% = = 0.05 (a) Full-load efficiency of the motor is about 95–
100
97% so the graph suiting the above statement
Impedence = 0.866 Ω/ph is curve V. (a ) → (T ).
Start circuit current/phase during starting.
 400  (b) Torque ∝ φ I a cos φ (This statement/relation is
  1 1
= 3  best speed ∝ ⇒ defined by
 0.866  Torque φ Ia
Rated full load current curve III) (b) → ( R).
50 × 103 1
= 72.16A (c) Speed ∝ .
3 × 400 φ I a cos φ
If x is the % tapping
The above relation is met by curve I. (c) → ( P),
X  2 ISc = 100
100 100 9. An induction motor runs stably under constant
X2 = ⇒ X = = 0.612 torque load at 1250 rpm off a 50 Hz supply, its
266.87 266.67
number of poles is . [1995]
% tapping = 61.2%

CH 03_five marks question.indd 64 11/12/2015 1:20:18 PM


Chapter 3  Induction Motors  |  3.65

Solution: Solution:
The condition for an induction motor to run stably (a) Typical speed–torque characteristics of I-M is
is given below:
Speed > Synchronous (NS).
120 × f

300 600 900 1200


NS = .
P
120 × 50

Speed
P= = 48.
1250
So, for motor to run above the synchronous speed P,
So, P should be a whole integer hence P = 4.
10. When started by means of an autotransformer with −40 −32 −24 −16 −8
50% tapping, supply current at start of an induc- Torque
tion motor is reduced to of that when
started by means of a star-delta starter. [1995] The following characteristics is best specified
by the graph P.
Solution:
Hence (a) → P.
By autotransformer starting
Supply current (IS1) = x2 × Short circuit current (b) We know
(ISC1).
3 2 Rr′
By Δ - Y Starting method, T= Ir .
Wms S
1
Supply current ( I S 2 ) = × I SC .
3
So, T ∝ I .
I S1 x 2 × I SC
= = (0.5) 2 × 3 = 0.433. The following characteristics is best satisfied
I S 2 1/ 3 × I SC
by the graph U.
I S 1 = 0.433 × I S 2 . Hence (b) → U.
11. Motor characteristics [1995] (c) If frequency is constant and voltage is varied
R the iron losses increases WL ∝ V 2 (Where
W = iron losses)
U
V ∝ WL .
Current, Speed
Power factor,

T
The following characteristics is best satisfied
Q P by graph T.
Hence (c) → (T).
S
(d) DC dynamic braking speed torque curve:

Torque voltage Wm
(a) Speed–torque characteristics of induction ma- Rr 2′
chine under motoring operation Rr1′
(b) Current–torque characteristics of a DC series
motor
(c) Power factor variation with voltage of an induc
tion motor under no-load operation Rr 2′ > Rr1′ O
(d) Speed–torque characteristics of induction ma- Torque
chine under DC injection dynamic braking
operation The following characteristics is best satisfied
(e) Speed–torque characteristics of DC series motor bygraph R.

CH 03_five marks question.indd 65 11/9/2015 5:55:47 PM


3.66 | Electric Machines

Hence (d) → R. 1000 − 1400


1 as Power = Speed = Clearly slip is negative.
(e) N∝ × Torque. 1000
T Hence, the motor is working in the generating mode.
So the following relation is best satisfied by
graph Q. 14. In a variable frequency induction motor drive,
the voltage must be varied to the
Hence (e) → Q.
frequency. [1994]
12. A 3-phase induction motor coupled to a pump is Solution:
operating at normal speed. If one line gets dis- As, we know
connected, the motor stops (state whether true or
flux (φ ) = V /f .
false). [1994]
So to maintain a constant value of flux, voltage
Solution:
must be varied proportional to the frequency.
The given statement is false.
If flux is not constant, the motor may draw excess
If one of the phase of 3 - ϕ. Induction motor fails,
magnetizing current which may saturate the core
a hunting sound will be produced and motor will
and terminal voltage and current waveform may
continue to run with slightly lower speed and in-
diston.
creased current with chances of burning of motor.
Constant torque
13. A six pole 50 Hz induction motor rotating at
1400 rpm is in mode. [1994]
Solution: Voltage

120 × f 120 × 50
NS = = = 1000 rpm.
P 6
Rated Frequency
N − Nr
= S
Slip 
NS 
So, voltage cannot be increased beyond rated
voltage. Hence, for speeds above rated speed
frequency is varied to obtain speed control.

CH 03_five marks question.indd 66 11/12/2015 1:20:31 PM


Chapter 4
Synchronous Machines
Actual value of ZS at rated voltage = Z S ( sat ) Pu × Z base
One-mark Questions = 0.65 × 22.094.
Z S = 15.06 Ω.
1. A star connected 400 V, 50 Hz, 4 pole synchronous
machine gave the following open circuit and short 2. A three-phase salient pole synchronous motor
circuit test results: is connected to an infinite bus. It is operated at
Open circuit test: VOC = 400 V (rms, line-to line) at no-load at normal excitation. The field excita-
field current, If = 2.3 A. tion of the motor is first reduced to zero and then
increased in the reverse direction gradually. Then
Short circuit test: ISC = 10 A (rms, phase) at field the armature current [2011]
current, If  = 1.5 A. (a) Increases continuously
The value of per phase synchronous impedance in (b) First increases and then decreases steeply
Ω at rated voltage is . [2014-S1] (c) First decreases and then increases steeply
Solution: (d) Remains constant
Given, V for VOC = 400 V (rms) Solution: (b)
400 As field current reduces, the flux will start reducing,
= = 220.94 (phase value). to keep this flux constant the armature drawn high
3
current from bus. At zero field current the motor
I f = 2.3 A.
acts as synchronous reluctance motor in this case
For I SC = 10 A (rms phase). the magnetizing current fully taken from bus. If we
I f = 1.5 A increase field current in reverse direction to keep
VOC flux constant more draws more currents. During this
∴  Base impedance, Z base = process the load angle increases. At one point, the
I SC
reverse field force dominates reluctance torque and
230.94 rotor slips are pole pitch and align to opposite pole.
=
10 The instant angle aligns to opposite pole the flux
= 23.094 Ω will be very high to reduce this flux current drop
For finding synchronous impedance at rated voltage, steeply to synchronous motor value. This torque is
naturation effect of machine has to be considered. reluctance + synchronous motor torque.
Hence, the correct option is (b).
If for VOC
SCR =
If for I SC 3. Distributed winding and short chording employed
in AC machines will result in [2008]
1 2.3
= = = 0.65 pu (a) Increase in emf and reduction in harmonics
Z S (Sat ) P1 1.5 (b) Reduction in emf and increase in harmonics

Chapter 04.indd 67 11/12/2015 10:11:05 AM


3.68 | Electric Machines

(c) Increase in both emf and harmonics At δ2, Ef cos d < V, i.e., machine operating at under
(d) Reduction in both emf and harmonics excitation for motor.
Solution: (d) V
θin = (V − E f cos δ )
Distribution factor in distributed winding, XS
 mm Y  Then θin is positive, i.e., taking lagging VARS and
sin   therefore operating at lagging power factor.
K d nth harmonic =  2 .
Y  Hence, the correct option is (b).
m sin  
2 6. A synchronous generator is feeding a zero power
nα factor (lagging) load at rated current the armature
Pitch factor, K p ≠ nth harmonic = cos .
2 reaction is [2006]
Kd < 1 and Kp < 1. So, emf reduces.
(a) Magnetizing (b) Demagnetizing
nα nα
If is chosen in such a way that cos = 0, nth (c) Cross magnetizing (d) Ineffective
2 2
harmonic is eliminated. Solution: (b)
Hence, the correct option is (d). Xs

4. In a stepper motor, the detent torque means Ia


[2008] +
(a) Minimum of the static torque with phase wind- Ef ∼ Vt

ing excited
(b) Maximum of the static torque with phase wind-
ing excited jIaXs
(c) Minimum of the static torque with phase wind- Ef
Vt
ing unexcited
(d) Maximum of the static torque with the phase
Ia
winding unexcited
Solution: (d) E f > Vt , the generator is overexcited. Therefore,

 Detent torque is the amount of torque that is pro- armature reaction is demagnetizing in nature, i.e.,
duced when it is not energized. No current is flow- φar opposes φ f .
ing through the windings. Hence, the correct option is (b).
Hence, the correct option is (d). 7. In relation to the synchronous machines, which of
5. A 3-phase synchronous motor connected to AC the fallowing statements is false? [2005]
mains is running at full-load and unity power factor. (a) In salient pole machines, the direct axis syn-
If its shaft load is reduced by half with field current chronous reactance is greater than the quadra-
held constant, its new power factor will be [2007] ture axis synchronous reactance
(a) Unity (b) Damper bars help the synchronous motor self start
(b) Leading (c) Short circuit ratio is the ratio of the field cur-
(c) Lagging rent required to produce the rated voltage on
(d) Dependent on machine parameters open circuit to the rated armature current
Solution: (b) (d) The  V curve of a synchronous motor represents
Excitation E is constant. the variation in the armature current with field
Load angle δ decreases from δ1 to δ2 as load decreases. excitation, at a given output power
E
Solution: (c)
jIa2Xs
SCR is defined as the ratio of the field current re-
quired to produce rated voltage on open circuit to the
E field current required to produce rated armature cur-
d2 Ia1 rent with the armature terminals shorted while the
d1 Ia2 machine is mechanically run at synchronous speed.
Hence, the correct option is (c).
At μPf, Ef cos d = V.

Chapter 04.indd 68 11/12/2015 10:11:07 AM


Chapter 4  Synchronous Machines | 3.69

8. For a given stepper motor, the following torque has Assuming Ra = 0.


the highest numerical value. [2004] Ef = Vf + jIa × S.
(a) Detent torque (b) Pull in torque Vertical distance,
(c) Pull out torque (d) Holding torque PQ = E f − V f = jI a × S .
Solution: (c)  It represents voltage drop across synchronous
The pull out torque is of highest numerical value. ­reactance.
When motor is running, if load torque is increased, at a Hence, the correct option is (a).
particular value of load torque, the motor will stop, i.e.,
11. A stand alone engine driven synchronous generator
it pulls out from operation this is the pull out torque.
is feeding a partly inductive load. A capacitor is now
Hence, the correct option is (c). connected across the load to completely nullify the
9. The following motor definitely has a permanent inductive current. For this operating condition.[2003]
magnet rotor [2004] (a) The field current and fuel input have to be
(a) DC commutated motor ­reduced
(b) Brushless DC motor (b) The field current and fuel input have to be
(c) Stepper motor ­increased
(d) Reluctance motor (c) The field current has to be increased and fuel
Solution: (c) input left unaltered
The rotor of a permanent magnet stopper motor is (d) The field current has to be reduced and fuel
made of ferrite or rare earth material which is per- input left unaltered
manently magnetized. Solution: (d)
Hence, the correct option is (c). With addition of capacitor, P f improves from lagging
to unity so field current reduces to reduce excitation.
10. Curves X and Y in the figure denote open circuit and
full-load zero power factor (zpf) characteristics of a Active power been supply will be same as the pre-
synchronous generator, Q is a point on the zpf char- vious because there is no change in resistance. So
acteristics at 1.0 pu voltage. The vertical distance the fuel input would remain same.
PQ in figure gives the voltage drop across [2003] Hence, the correct option is (d).
P X 12. It is desirable to eliminate 5th harmonic voltage from
the phase voltage of an alternator. The coils should
Y be short-pitched by an electrical angle of [2001]
1.0 Q (a) 30° (b) 36°
(c) 72° (d) 18°
Solution: (b)
Field current For fifth harmonics, S β = 180°.
(a) Synchronous reactance β = 36°.
(b) Magnetizing reactance
Hence, the correct option is (b).
(c) Potier reactance
(d) Leakage reactance 13. Figure shows the magnetization curves of an alter-
nator at rated armature current, unity power factor
Solution: (a)
and also at no-load. The magnetization curve for
Point P represents Ef for field current If .
rated armature current, 0.8 power factor leading is
Point Q represents Vf for same field current If and given by [2001]
rated armature current is given by alternator. V no load rated
armature
Xs current
unity pf
Ia

Ef ∼ Vt
C D
A B

Chapter 04.indd 69 11/12/2015 10:11:07 AM


3.70 | Electric Machines

(a) Curve A (b) Curve B (a) The field current is reversed keeping the direc-
(c) Curve C (e) Curve D tion of rotation same
Solution: (c) (b) The field current remains the same but the di-
For any particular rated voltage, leading power rection of rotation is reversed
factor terms are to left of uφ f curve and lagging (c) The field current is reversed and the number of
power factor terms are to right. poles is doubled
Hence, the correct option is (c). (d) The number of poles is doubled without re-
versing the field current
14. Xd, Xd′ and X d″ are steady state d-axis synchronous
reactance, transient d-axis reactance and sub-­ Solution: (b)
transient d-axis reactance of a synchronous machine The phase sequence can be change if direction of
respectively, which of the following statements rotation changes. Change of phase sequence is in-
is true? [2001] dependent of field current.
(a) Xd > X′d > X″d (b) X″d  > X′d  > Xd Hence, the correct option is (b).
(c) X′d > X″d > Xd (d) Xd > X″d > Xd′ 16. Higher synchronous reactance is preferred in the pre-
Solution: (a) sent day alternators, because one can have [1999]
The path of armature flux (When armature mmf (a) Reduced sub-transient currents
along d-axis), (b) Reduced harmonic currents
(c) Reduced transient currents
(d) Higher voltage regulation with load
Solution: (c)
When any short circuit on all rotors occurs, the
short circuit current is very high. Transient period is
for long duration because of low resistance of field
Sub-transient Transient Steady state
condition condition condition winding. Thus, to prevent alternator from affects
of transients, it is designed with high synchronous
In sub-transient condition, there is induced current ­reactance.
in both the field windings and damper winding so
φa cannot be linked with either winding. Hence, the correct option is (c).
In transient condition, there is induced current only 17. A synchronous generator connected to an infinite
in field winding, so φa cannot be linked with field. bus is overexcited. Considering only the reactive
It can only be linked with damper. power, from the point of the system, the machine
In steady state, no induced current in either FW or acts as [1998]
in DW, so flux φa can be linked with both. (a) A capacitor (b) An inductor
(c) A resistor (d) None of these
By assuming reluctance or iron path negligible and
reluctance mainly due to air path. Solution: (b)
For generator,
Sub-transient Transient Transient
> > V
reluctance reluctance reluctance Qout = ( E f cos δ − V ).
(R″d ) (R′d ) (Rd) XS
l When, E f cos δ > V , i.e., over excited generator,
Inductance, L ∝ .
Rl then Qout is positive, i.e., supplying lagging VARS
Ld > Ld′ > Ld′′. and therefore operating at lagging pf, thus, behav-
ing like inductor.
X d > X d′ > X d′′.
Hence, the correct option is (b).
Hence, the correct option is (a).
18. In the figure below, the characteristics that corre-
15. The phase sequence of a three-phase alternator
sponds to the variation of synchronous reactance of
will reverse if [2000]
synchronous motor with field current is [1996]

Chapter 04.indd 70 11/12/2015 10:11:09 AM


Chapter 4  Synchronous Machines | 3.71

(c) Stator mmf wave and resultant flux density wave


I
IV (d) Stator mmf wave and resultant mmf wave
III II
Xs Solution: (a)
The angle between stator field axis and rotor field
axis is called torque angle.
If Hence, the correct option is (a).
(a) Curve I (b) Curve II 22. The developed electromagnetic force and/or torque
(c) Curve III (d) Curve IV in electro–mechanical energy conversion systems
act in a direction that tends [1992]
Solution: (b) (a) To increase the stored energy at constant mmf
E (b) To decrease the stored energy at constant flux
ZS  X S  .
I (c) To decrease the co-energy at constant mmf
Upto knee point: Both OCC and SCC are linear. (d) To decrease the stored energy at constant mmf
Above knee point: OCC is non-linear and SCC is Solution: (b)
linear. This is according to Lenz’s law.
⇒ ZS or XS decreases during saturated condition. Hence, the correct option is (b).
Hence, the correct option is (b).
19. During hunting of synchronous motor [1996] Two-marks Questions
(a) Negative phase sequence currents are generated
1. A 3-phase synchronous generator is to be connected
(b) Harmonics are developed in the armature circuit
to the infinite bus. The lamps are connected as shown
(c) Damper bar develops torque
in figure for the synchronization. The phase sequence
(d) Field excitation increases
of bus voltage is R - Y - B and that of incoming gen-
Solution: (c) erator voltage is R′ - Y ′ - B′. [2014-S1]
When the motor is running at synchronous speed, RY B
the relative speed b/w damper bars and rotating air R′
gap flux is zero. Because of zero relative speed, no Y′ ∼
flux cutting action takes place and emf generated
in damper bars is zero—consequently no damping B′
∼ ∼
torque is developed. The damper winding comes
La
into play only during rotor hunting, when rotor
speed departs from synchronous speed. Lb
Hence, the correct option is (c). Infinite Incoming
bus Lc generator
20. A synchronous motor on load draws a current at a
leading power factor angle φ . If the internal power It was found that the lamps are becoming dark in
factor angle, which is the phase angle between the the sequence La - Lb - Lc. It means that the phase
excitation emf and the current in time phasor dia- sequence of incoming generator is
gram is ψ, then the air gap excitation mmf lags the (a) O pposite to infinite bus and its frequency is
armature mmf by [1995] more than infinite bus
≠p (b) O pposite to infinite bus and its frequency is
(a) ψ (b) +ψ
2 less than infinite bus
(c) π/2 - ψ (d) ψ+ φ (c) Same as infinite bus and its frequency is more
Solution: (c) than infinite bus
21. The torque angle of a synchronous machine operat- (d) Same as infinite bus and its frequency is less
ing from a constant voltage bus, is usually defined than infinite bus
as the space angle between [1992] Solution: (a)
(a) Rotor mmf wave and stator mmf wave As La is connected between R - R′.
(b) Rotor mmf wave and resultant flux density wave
Lb is connected between Y - Y′.

Chapter 04.indd 71 11/12/2015 10:11:10 AM


3.72 | Electric Machines

Lc is connected between B - B′. Solution:


Therefore, the given method of synchronization is Given, E1 = 0.3 pu
called “Synchronization by dark lamp method”. X s = 1.1 pu
In this method of synchronization, correct phase V = 1 pu
sequence of incoming generator is obtained when P = 0.6 pu
all the three lamps become dark together and bright Neglecting stator resistance, i.e., θ1 = 90°.
together.
VE f
Since, lamps are becoming dark in the sequence Output power delivered, P = , sin δ .
XS
La  - Lb - Lc therefore, the phase sequence of in-
coming generator is opposite to the infinite bus. P × S 0.6 × 1.1 6.6
sin δ = = = = 0.5.
Since, lamps dark sequence is La - Lb - Lc, there- VE f 1× 1.3 13
fore, frequency of incoming generator is more than δ = 30° = Power angle.
infinite bus.
Hence, the correct option is (a). 3
cos δ = cos 30 = .
2
2. A 20-pole alternator is having 180 identical stator Reactive power is given by,
slots with 6 conductors in each slot. All the coils of a
VE f V2 
phase are in series. If the coils are connected to real- θ = cos δ − 
ize single-phase winding, the generated voltage is  XS X1 
V1. If the coils are reconnected to realize three-phase 1× 1.3 3 12
star-connected winding, the generated phase voltage = × − .
1.1 2 1.1
is V2. Assuming full pitch, single-layer winding, the
ratio V1/V2 is [2014-S2] θ = 1.023 − 0.909
= 0.114 pu = 0.11 pu
1 1
(a) (b) 4. There are two generators in a power system.
3 2
No-load frequencies of the generators are 51.5 Hz
(c) 3 (d) 2 and 51 Hz, respectively, and both are having droop
Solution: (d) constant of 1 Hz/MW. Total load in the system is
2.5 MW. Assuming that the generators are oper-
E ph (1−φ ) K d 180 Tph (1−φ ) ating under their respective droop characteristics,
= ×
E ph ( 3−φ ) K d 60 Tph ( 2 −φ ) the frequency of the power system in Hz in the
 180  steady state is . [2014-S2]
sin  T
 Solution:
=  2  × 60 × 1
 
60 80 T No-load frequency of Generator-1 = 51.5 Hz.
si n  
 2  3 No-load frequency of Generator-2 = 50 Hz.
1 1 Total load = 2.5 MW.
= × × 3 = 2. Drop of both m/c = 1 Hz/MW.
0.5 3
Let M/C1 share load of P1MW.
Hence, the correct option is (d).
⇒ M/C2 will share load of (2.5 - P1) MW.
3. A synchronous generator is connected to an infi-
nite bus with excitation voltage E = 1.3 pu The
f
Let steady state frequency of system be f.
generator has a synchronous reactance of 1.1 pu 51.5 − f
and is delivering real power (P) of 0.6 pu to the = 1 = droop.
P1
bus. Assume the infinite bus voltage to be 1.0 pu
Neglect stator resistance. The reactive power (Q) P1 = (51.5 − f ) (1)
in pu supplied by the generator to the bus under
51 − f
this condition is  [2014-S2] Also, = 1.
2.5 − P1

Chapter 04.indd 72 11/12/2015 1:21:52 PM


Chapter 4  Synchronous Machines | 3.73

⇒ 51 - f = 2.5 - P1. (c) The magnitude of terminal voltage decreases,


 P1 = f = 48.5 (2) and the field current increases
Solving (1) and (2), (d) The magnitude of terminal voltage does not
change, and the field current decreases
51.5 f − f = f − 48.5.
Solution: (a)
2 f = 51.5 + 48.5 = 100. As field voltage is held constant, so field current
100 does not change.
f = = 50 Hz.
2 When the generator is connected with open-circuit
5. The direct axis and quadrature axis reactances transmission line, line draws charging current,
of a salient pole alternator are 1.2 pu and 1.0 pu therefore Vt > Eg.
respectively. The armature resistance is negligible.
If this alternator is delivering rated kVA at upf and Xs Ic
at rated voltage then its power angle is [2011] Eg ∼ Vt
(a) 30° (b) 45°
(c) 60° (d) 90°
Solution: (b)
Direct
But, when generator is disconnected from the line,
axis no charging current is delivered by generator, i.e.,
Ic = 0  ⇒  Vt = Eg.
E′
d Quadrature
So, terminal voltage decreases.
Ef
jIa X q axis Hence, the correct option is (a).
Ia
Vt 7. A separately excited DC machine is coupled to a
Power angle is angle b/w Ef and Vt. 50 Hz, three-phase, 4-pole inductions machine as
shown in the figure. The DC machine is energized first
But as E′ and Vt are in phase, in this case, angle
and the machines rotate at 1600 rpm. Subsequently
b/w E′ and -Vt = Power angle = δ.
the inductions machine is also connected to a 50 Hz,
Rated power is delivered at upf, so Ia and Vt are in phase, three-phase source, the phase sequence being con-
I a = 1 ∠0° pu sistent with the direction of rotation. In steady state
Quad nature - axis function,  [2010]
Induction
X a = 1 pu machine
DC machine
E1 = Vt + jI a × a 4 pole, 50 Hz
t 50 Hz,
1 balanced
= 1 + j1× 1 = ∠45°.
2 three-phase

δ = 45°.
Hence, the correct option is (b). (a) Both machine act as generators
6. A 50 Hz synchronous generator is initially con- (b) The DC machine acts as a generator, and the
nected to a long lossless transmission line which is induction machine acts as a motor
open circuited at the receiving end. With the field (c) The DC machine acts as a motor, and the in-
voltage held constant, the generator is disconnected duction machine acts as a generator
from the transmission line. Which of the following (d) Both machines act as motors
may be said about the steady state terminal voltage Solution: (c)
and field current of the generator? [2010]
120 × f
(a) The magnitude of terminal voltage decreases, Synchronous speed, N S =
and the field current does not change P
(b) The magnitude of terminal voltage increases, 120 × 50
= = 1500 rpm.
and the field current does not change 4

Chapter 04.indd 73 11/12/2015 10:11:13 AM


3.74 | Electric Machines

Speed of rotors of both DC and IM are same as 9. The excitation voltage and load angle will respec-
they are mechanically coupled. tively be [2008]
Nr = 1600 rpm. (a) 0.8 pu and 36.86 degree lag
(b) 0.8 pu and 36.86 degree lead
For, IM Nr > Ns. (c) 1.17 pu and 30.96 degree lead
Slip, S = -Ve. (d) 1.17 pu and 30.96 degree lag
Hence, IM is acting as generator, input mechani- Solution: (d)
cally power is taken from rotor of DC machine and
I a = 0.6∠0°.
output power is delivered to 3φ AC supply.
E ∠δ = Vt − jI a × S
Hence, the correct option is (c).
= 1∠0° − j 0.6 × 1 = 1.17∠ − 30.96.
8. A field excitation of  20  A in certain alternator
results in an armature current of 400 A in short cir- E = 1.17.
cuit and a terminal voltage of  2000 V on open ­circuit. δ = 30.96 lag.
The magnitude of the internal voltage drop with in
the machine at a load current of 200 A. [2009] Hence, the correct option is (d).
(a) 1 V (b) 10 V 10. Keeping the excitation voltage same, the load on
(c) 100 V (d) 1000 V the motor is increased such that the motor current
Solution: (d) increases by 20%. The operating power factor will
become [2008]
If = 20 A.
(a) 0.995 lagging (b) 0.995 leading
VOC = 2000 V. (c) 0.791 lagging (d) 0.848 leading
ISC = 400 A. Solution: (a)
If Ra is neglected, Let P.f. = cos φ .
     
Ef = Vt + jIa X S .
I a = 0.6× 1.2 = 0.72 P.u.
For a given field current, under short circuit condition,
E ∠δ = Vt − j ( I a ∠ − φ ) × ( X S ).
=Ia I=
SC ; Vt 0.
E (cos δ + j sin δ ) = 1 ∠ 00 − jI a X S (cos φ − j sin φ )
Ef
XS = . = 1 − j 0.72 × 1(cos φ − j sin φ )
I SC
= (1 − 0.72 sin φ ) − j 0.72 cos φ .
At open circuit,
Ef = VOC. E 2 = (1 − 0.72 sin φ ) 2 + (0.72 cos φ ) 2
Ia = 0. = 1 + 0.722 sin 2 φ − 1.44 sin φ + 0.722 cos 2 φ .
With linear assumption,
1.17 2 = 1 + 0.722 − 1.44 sin φ .
VOC sin φ = 0.1038.
XS =
I SC If = constant φ = 5.96.
2000 cos φ = 0.995 lagginng.
= = 5 Ω.
400
Hence, the correct option is (a).
Internal voltage drop = Ia × S = 200 × 5 = 1000 V.
11. A 3-phase, 3-stack, variable reluctance stepper motor
Hence, the correct option is (d). has 20 poles on each rotor and stator stack. The step
Common Data for Questions 9 and 10: angle of this stepper motor is [2007]
A synchronous motor is connected to an infinite bus at (a) 3 degree (b) 6 degree
1.0 pu voltage and draws 0.6 pu at unity power factor. (c) 9 degree (d) 18 degree
Its synchronous reactance is 1.0 pu and resistance is Solution: (b)
negligible. Number of teeth = Number of poles = 20.

Chapter 04.indd 74 11/12/2015 10:11:14 AM


Chapter 4  Synchronous Machines | 3.75

360° Solution: (c)


Step angle = In double layer winding,
(no. of stacks) × (no. of teeth )
Number of coils = Number of slots = 48.
360°
= = 6°. Each coil has 10 turns.
3 × 20
Total number of turns = 48 × 10 = 480.
Hence, the correct option is (b). 480
= T=
Number of turns/phase = 160.
12. A 100 kVA, 415 V star connected synchronous Ph
3

machine generates rated open circuit voltage 415 V
180
at a field 15 A 10 A is equal to the rated armature Slots/pole/phase = m = =4.
­current. The per unit saturated synchronous reac- 4×3
tance is [2007] 180 × poles 180 × 4
x = Slot angle = = = 15°.
(a) 1.731 (b) 1.5 slots 48
(c) 0.666 (d) 0.5777
mx sin  4 × 15 
Solution: (c) sin  
Distribution factor, K d = 2 =  2 
At rated open circuit voltage, IOC = 15 A. x
m sin  15 
Short armature current at field current 15 A. 4 sin  
2  2
100
Rated armature current = = 139.12 A. = 0.957.
3 × 415 Short pitching angle, a = 36°
Short circuit armature current at field current 15 A, nα
15 For nth harmonic, pitch factor, K P = cos .
I a1 = 139.12 × = 208.68 A. 2
10 Pitch factor due to fundamental component,
Phase voltage,
α
VOC ( line ) 415 K P = cos = cos 18° = 0.951.
= VP = . 2
3 3 Winding factor,
V Kw = Kd ⋅ Kp = 0.910.
Saturated synchronous reactance = X s = P
I a1 φ
Induced emf or phase,
415 EPh = 4.44 K w Tphφ f
= 3 = 4.44 × 0.910 × 160 × 0.025 × 50
208.68
= 808 V.
= 1.148 Ω.
    =
  Eline 3 EPh 1400 V.
=
4152
Base impedance, Z B = = 1.722 Ω.
100 × 102 Hence, the correct option is (c).
XS 1.48 14. The line to line induced emf, for two phase con-
= =
Pu saturated synchronous reactance
Z B 1.722 nection is [2006]
= 0.666 pu (a) 1143 (b) 1332
Hence, the correct option is (c). (c) 1617 (d) 1791
Common Data for Questions 13–15: Solution: (c)
A 4 pole, 50 Hz, synchronous generator has 48 slots In this case,
in which a double layer winding is house. Each coil has mγ = 90°.
10 turns and is short pitched by an angle to 36 degree γ = 15°.
­electrical. The fundamental flux per pole is 0.025 Wb 48
13. The line to line induced emf, for a 3-phase star m= = 6.
4× 2
connection is approximately [2006]
sin(90 / 2)
(a) 808 (b) 888 factor, K d
Distribution = = 0.903.
(c) 1400 (d) 1538 6 sin(15/ 2)

Chapter 04.indd 75 11/12/2015 10:11:18 AM


3.76 | Electric Machines

480 E2 = 233.729 × 1.01 = 236 V.


TPh = = 240.
2
∴ I a X = E22 − V
K P = 0.951.
2
E = 4.44 K w f TPhφ  400 
= 2362 −  
= 4.44 × 0.903 × 0.951× 50 × 240 × 0.025  3
= 1617.65 V. = 48.93.
Hence, the correct option is (c). I a = 4.893 A.
15. The fifth harmonic component of phase emf, for a 4.893
% Load =
3-phase star connection is [2006] 7.21
(a) 0 (b) 269 = 67.86%.
(c) 281 (e) 808 Hence, the correct option is (a).
Solution: (a)
Pitch factor due to 5th harmonic components, Common Data for Questions 17 and 18:
A 1000 kVA, 6.6 kV, 3-phase star connected cylindrical
 5 × 36 
( K P ) S = cos   = cos 90 = 0. pole synchronous generator has a synchronous reactance
 2  of 20 ohms. Neglect the armature resistance and ­consider
As, pitch factor is zero, induced emf due to 5th operation at full-load and unity power factor.
harmonic component is zero. 17. The induced emf is close to (line to line) [2005]
Hence, the correct option is (a). (a) 5.5 kV (b) 7.2 kV
(c) 9.6 kV (d) 12.5 kV
16. A 3-phase 400 V, 5 kW, star connected synchro-
nous motor having an internal reactance of 10 Solution: (b)
ohms is operating at 50% load, unity power factor. The answer is (b).
Now, the excitation is increased by 1%. What will 18. The power angle is close to [2005]
be the new load in percent, if the power factor is (a) 13.9 degree (b) 18.3 degree
to be kept same? Neglect all losses and consider (c) 24.6 degree (d) 33.0 degree
linear magnetic circuit. [2006] Solution: (c)
(a) 67.9% (b) 56.9% Rated output power = S = 1000 kVA.
(c) 51% (d) 50%
Rated line to line voltage = VL - L = 6.6 kV.
Solution: (a)
S
5 × 102 Rated current, I a =
IfL = = 7.21 A. 3 ⋅ V µ cos φ
3 × 400 × 1
1000
For synchronous motor, =
3 × 6.6 × 1
E 2 = (V cos θ − I a Ra ) 2 + (V sin θ − I a × S ) 2. = 87.477 A.
X s = 10. As it is star connected,
400 6.6
V= . =
Phase to neutral voltage,Vt = 3.81 kV.
3 3
cos θ = 1. Taking Vt as reference,
I fL Vt = 3.81 ∠0° kV.
Ia = = 3.6.
2 I a = 87.477 ∠0° A.
2
 400   10 
2
Et = Vt + I a × S
E=   +  = 233.729 V.
 3   3.6  = 3.81 ∠0° + (87.477 ∠0°)(20 × 10−3 ∠90°)
There is 1% increase E1 = 4.19 ∠24.6° kV.

Chapter 04.indd 76 11/12/2015 10:11:20 AM


Chapter 4  Synchronous Machines | 3.77

Induced emf (line to line) = 3E f 21. For a 1.8 degree, 2 phase bipolar stepper motor,
the stepping rate is 100 steps/second. The rota-
= 3 × 4.19 tional speed of the motor in rpm is [2004]
= 7.2 kV. (a) 15 (b) 30
The power angle (torque angle) = 24.6°. (c) 60 (d) 90
Hence, the correct option is (c). Solution: (b)
360
= = 200 steps.
Steps required for one revolution
19. The synchronous speed for the seventh space har- 1.8
monic mmf wave of a 3-phase, 8 pole, 50 Hz induc- ∴ Time required for one revolution = 2 seconds.
tion machine is [2004] Rev/second = 0.5 rps.
(a) 107.14 rpm in forward direction Rev/minute = 30 rpm.
(b) 107.14 rpm in reverse direction
Hence, the correct option is (b).
(c) 5250 rpm in forward direction
(d) 5250 rpm in reverse direction 22. A 400 V, 50 kVA, 0.8 pf leading delta connected
50 Hz synchronous machine has a synchronous reac-
Solution: (a)
tance of 2 ohm and negligible armature resistance. The
Synchronous speed corresponding to fundamental
friction and windage losses are 2 kW and the core loss
component,
is 0.8 kW. The shaft is supplying 9 kW load at a power
120 f
Ns = . factor of 0.8 leading. The line current drawn is[2004]
P (a) 12.29 A (b) 16.24 A
Fiction number of poles corresponding to rth har- (c) 21.29 A (d) 36.88 A
monic = rp.
Solution: (c)
So speed of rth harmonic rotating field,
Power available at shaft = Pshaft = 9 kW.
120 f 120 × 50 Losses = Core loss + Friction and windage loss
N sr = = = 107.14 rpm.
P 7×8 + copper loss
Direction of rotation will be forward, i.e., same as = 0.8 kW + 2 kW + 0 = 2.8 kW.
that of rotor. Input power = P = Pshaft + losses
Hence, the correct option is (a). = 9 + 2.8
= 11.8 kW
20. Two 3-phase, star connected alternators are to be par-
P
allel to a set of common bus bars. The armature has Line current drawn =
a per phase synchronous reactance of 1.7 ohm and 2VL cos φ
negligible armature resistance. The line voltage of the 11.8 × 103
first machine is adjusted to 3300 V and that of the = = 21.29 A.
3 × 400 × 0.8
second machine is adjusted to 3200 V. The machine
voltages are in phase at the instant they are paralleled. Hence, the correct option is (c).
Under this condition, the synchronizing current per 23. A 500 MW 3-phase star connected synchronous gen-
phase will be [2004] erator has a rated voltage of 21.5 kV at 0.85 pf. The
(a) 16.98 A (b) 29.41 A line current when operating at full-load rated condi-
(c) 33.96 A (d) 58.82 A tions will be [2004]
(a) 13.43 A (b) 15.79 kA
Solution: (a)
E − E2 (c) 23.25 kA (d) 27.36 kA
Per phase synchronizing current = 1 .
Z s1 + Z s 2 Solution: (b)
E1 and E2 are in phase. P
Rated line current I L = .
3300 3200 3 ⋅VL ⋅ cos φ

3 3 500 × 106
Z s1 = Z s 2 . I sy = = 16.98 A. IL =
2 × 1.7 3 × 21.5 × 103 × 0.85
= 15.79 KA.
Hence, the correct option is (a).
Hence, the correct option is (b).

Chapter 04.indd 77 11/12/2015 10:11:21 AM


3.78 | Electric Machines

24. A 4 pole, 3-phase, double layer winding is housed (a) 0.82 (b) 0.47
in a 36 slot stator for an AC machine with 60 (c) 0.39 (d) 0.92
degree phase spread. Coil span is 7 slot pitches.
Solution: (d)
Number of slots in which top and bottom layers
belonging to different phases is  [2003] E f ∠δ = V ∠0 + I a ∠φ . Z S ∠θ
(a) 24 (b) 18 = V ∠0 + I a Z S ∠(φ + θ ).
(c) 12 (d) 0
Solution: (a) Squaring on both sides,
2 2 2
36 E f = V + I a ⋅ Z s + 2 V I a cos(θ + φ ).
= = 9 slots.
Pole Pitch
4
Coil spam = 7 slots. For zero regulation, E f = V .
Slot/pole/pitch = 3.
Ia Zs 20 × 10
If chorded by two slots, cos(θ + φ ) = − =− = −0.3936.
2V 2×
440
Out of 3 → 2 have different phases.
3
Out of 36 → 24 have different phases.
θ + φ = 113.178.
Hence, the correct option is (a).
φ = 113.1788 − 90 = 23.178.
25. The flux per pole in a synchronous motor with the
cos φ = 0.9192.
field circuit ON and the stator disconnected from the
supply is found to be 25 mWb. When the stator is Hence, the correct option is (d).
connected to the rated supply with the field excitation 27. A single-phase, 2000 V alternator has armature
unchanged, the flux per pole in the machine is found resistance and reactance of 0.8 ohms and 4.94
to be 20 mWb while the motor is running on no-load. ohms respectively. The voltage regulation of the
Assuming no-load losses to be zero, the no-load cur- alternator at 100 A load at 0.8 leading power-­
rent drawn by the motor from the supply [2002] factor is [2000]
(a) Lags the supply voltage (a) 7% (b) −8.9%
(b) Leads the supply voltage (c) 14% (d) 0%
(c) Is in phase with the supply voltage
(d) Is zero Solution: (b)

Solution: (b) E f ∠δ = V ∠0 + I a ∠φ ⋅ Z S ∠φ
Armature reaction is trying to deteriorate the main = 2000∠0 + (100∠ cos −1 0.8)(0.8 + j 4.94)
flux. So, motor must be operating with leading
power factor. = 1822.316 ∠14.07.
Hence, the correct option is (b). | E | − |V |
V.R. = × 100
|V |
26. A star-connected 440 V, 50 Hz alternator has per
phase synchronous reactance of 10 Ω. It supplies a 1822.316 − 2000
= × 100 = −8.884%.
balanced capacitive load current of 20 A, as shown 2000
in the per phase equivalent circuit of figure. It is
Hence, the correct option is (b).
desirable to have zero voltage regulation. The load
power factor should be [2001] 28. A 3-phase alternator is wound with a 60 degree
phase-spread armature windings and develops
j10 W 20 A 300 kVA. If the armature is reconnected utiliz-
ing all the coils for single-phase operation with a
load phase spread of 180 degrees, the new rating of the

machine is [1999]
E ph = 440 (a) 100 kVA (b) 200 kVA
√3
(c) 250 kVA (d) 300 kVA

Chapter 04.indd 78 11/12/2015 10:11:23 AM


Chapter 4  Synchronous Machines | 3.79

Solution: (b) (a) Would be equal to If


E′ ∝ Kd KP N f . (b) Would be equal to 2If
(c) Would be equal to If /2
π  (d) Cannot be predicted due to insufficient data
sin  
K d1 =  3 = 3. Solution: (a)
π
  π For short circuit calculation,
3
  Et E
2 I SC =  t .
π  ZS XS
sin  
 2 = 2. Ef ∝ f φ and XS ∝ f .
Kd 2 =
π π
Which gives I SC ∝ φ .
2
Since, the armature conductors are the same, cur- Hence, short circuit current is only function of ex-
rent carrying capability will be same. citation.
kVA Rating ∝ E ′ ∝ K d . Hence, the correct option is (a).
3 31. A synchronous motor operates at 0.8 p.f. lagging.
300 π 3 If the field current of the motor is continuously
= = .
(kVA)π 2 2 increased [1993]
π (1) The power factor decreases upto a certain val-
ue of field current and thereafter it increases
(kVA)π = 200 kVA.
(2) The armature current increases upto a certain
Hence, the correct option is (b). value of field current and thereafter it decreases
29. The armature of a single-phase alternator is com- (3) The power factor increases upto a certain ­value
pletely wound with T single turn coils distributed of field current and thereafter it decreases
uniformly. The induced voltage in each turn is 2 V (4) The armature current decreases upto a certain
(rms). The emf of the whole winding is [1998] value of field current and thereafter it i­ ncreases.
(a) 2T V (b) 1.11T V From these the correct one is
(c) 1.414T V (d) 1.273T V (a) 1, 2 (b) 2, 3
Solution: (d) (c) 3, 4 (d) 1, 3
 mx  Solution: (c)
sin  
Distribution factor, K d =  2 .
 mx 
  Ia
 2  p.f.
For single-phase machine mx = π .
sin(π /2) 2 lag lead
⇒ Kd = = .
π /2 π ‘v ’ curve Inverted ‘v ’ curve

K p = 1 For full pitched winding. Hence, the correct option is (c).


2 32. Two 550 kVA alternators operate in parallel to sup-
E = K d ⋅ K p ⋅ N .E ′ =
⋅1⋅ T ⋅ 2 = 1.273T .
π ply the following loads
Hence, the correct option is (d). (1) 250 kW at 0.95 power factor lag
(2) 100 kW at 0.8 power factor lead one machine
30. A 3-phase alternator has negligible stator resistance.
is supplying 200 kW at 0.9 power factor lag.
A short circuit test is conducted on this a­ lternator.
The power factor of the other machine must be
At a particular speed a field current of If is required
 [1993]
to drive the rated armature current. If the speed of
(a) 0.89 lead (b) 0.95 lead
the alternator is reduced to half, the field current
(c) 0.95 lag (d) 0.89 lag
required to maintain rated armature current [1993]

Chapter 04.indd 79 11/12/2015 10:11:25 AM


3.80 | Electric Machines

Solution: (a) Using power conservation,


Total kVA supplied to load by machine (250 − j82.17) + (100 + j 75)
250 = (200 − j 96.864) + S 2 .
M1 = ∠ − cos −1 (0.95) = (250 − j82.17) kVA.
0.95 S 2 = 174.65∠30.814°.
100
M2 = ∠ cos −1 0.8 = (100 − j 75) kVA. P.f. = 0.8588 Lead.
0.8
Hence, the correct option is (a).
Now,
200
Total kVA of M 1 = ∠ cos −1 0.9
0.9
  = (200 − j 96.864) kVA.

Chapter 04.indd 80 11/12/2015 1:22:17 PM


Chapter 4  Synchronous Machines | 3.81

P = 3VI cos θ
Five-marks Questions
= 3 × 415 × 50 × 0.985
1. A 415 V, 2 pole, 3-phase, 50 Hz, star connected, = 35.38 kW.
non-salient pole synchronous motor has synchro-
nous reactance of 2 Ω per phase and negligible 2. A 50 kW synchronous motor is tested by driving
stator resistance. At a particular field excitation, it by another motor. When the excitation is not
it draws 20 A at unity power factor from a 415 V, switched on, the driving motor takes 800 W. When
3-phase, 50 Hz supply. The mechanical load on the the armature is short-circuited and the rated arma-
motor is now increased till the stator current is equal ture current of 10 A is passed through it, the driv-
to 50 A. The field excitation remains unchanged. ing motor takes 1800 W. Calculate the efficiency
Determine: of the synchronous motor at 50% load. Neglect the
(A) The per phase open circuit voltage Eo losses in the driving motor. [2001]
(B) The developed power for the new operating con- Solution:
dition and corresponding power factor [2002] A 50 kW synchronous motor is tested by driving it
Solution: by another motor. When the excitation is switched
The applied voltage per phase, on, the driving motor takes 800 W. When the arma-
ture is short circuited and the rated armature current
415
V
= = 239.6 V. at 10 A is passed through it, the driving motor re-
3 quired 2500 W. On open-circuiting the armature with
f = 50 Hz. rated excitation, the driving motor takes 1800 W.
P = 2. Calculate the efficiency of synchronous motor at
Armature resistance,  Ra  0. 50% load. Neglect the losses in the driving motor.
Synchronous reactance, XS = 2 Ω. 3. Two identical synchronous generators, each of 100
MVA, are working in parallel supplying 100 MVA at
At a particular excitation, say emf E per phase, 0.8 lagging pf at rated voltage. Initially, the machines
Armature current, Ia = 20 A at upf. are sharing load equally. If the field current of first
cos θ = 1 sin θ = 0. generator is increased by 5% and of the second gen-
erator is increased by 5%, find the sharing of load
E = V + ( I a × S )2
2 2
(MW and MVAR) between the generators.
= (239.6) 2 + (20 × 2) 2 . Assume Xd = Xq = 0.8 pu, no field saturation and
When load is increased, Ia 50 A. rated voltage across load. Reasonable approxima-
Ifo2 > Ifo1 thus, motor is at underexcitation and lag- tions may be made. [2001]
ging pf cos θ, Solution:
As Xd = Xq , it means a cylindrical rotor with syn-
V = 239.6 V E = 242.92 V.
chronous reactance XS = 0.8 pu
I a = 50 A cos θ lag.
Total load = 100 MVA at 0.8 pf leading.
At lag pf,
Let (MVA)base = 100; (V)base = rated KV.
E 2 = (V cos θ − I a Ra ) 2 + (V sin θ − I a X S ) 2 .
    S L = 1 pu; cos θ = 0.8.
(242.92) 2 = (239.6 cos θ ) 2 + (239.6 sin θ − 50 × 2) 2 .
S L = PL + j θ L .
⇒ 47920 sin θ = 8398.034. PL = S L cos θ = 0.8 pu
    sin θ = 0.175. θ 2 = S L sin θ = 0.6 pu
cos θ = 0.985 lag. Load shared by each generator,
As motor is lossless ( Ra  0)  the power developed SL
will be same as power input. S1 = S 2 = = 0.4 + j 0.3.
2
Electrical power input, S1 = S L = 0.5 pu

Chapter 04_Five Marks Question.indd 81 11/9/2015 6:00:36 PM


3.82 | Electric Machines

Both are operating at rated voltage, i.e., terminal 1.344 × 1


voltage, 0.4 = × sin δ 2′ .
0.8
Let, V1 = V2 = 1 pu δ 22 = 13.77.
V1 = V2 = V = 1∠0° pu E2′ = E2′ ∠δ 2 = 1.344∠13.77° pu
The generator currents,
S L = VI pu
1 = 1× I . E1′ − V 1.216∠15.16 − 1∠0°
I1′ = =
I = 1 pu j 0.8 j 0.8
= 0.45 ∠ − 28.54 pu
Pf , cos θ = 0.8.
θ = 36.9. Complex power,
I = 1∠ − 36.9 pu S1′ = V ⋅ I1 = 1 ∠0°× 0.45 ∠28.54.
Both are identical so,
S1′ = (0.4 + j 0.22) pu
I Load and Pf are same,
I1 = I 2 = L = 0.5∠ − 36.9 pu
2 S L = 0.8 + j 0.6.
E1 = E2 = V + jI1 × S
= 1∠00 + j 0.8 × 0.5∠ − 36.9 Since,   S L = S1 + S 2 ,
= 1.28∠14.47° pu     S 2 = S L − S1 = 0.4 + j 0.38.
Excitation emfs, From    S = P + jQ and (MVA B ) = 100.
E=
1 E=
2 1.28 pu P1 0=
= Q1 0=
.4 40 MW = .22 22 MVAR.
If field If 1of 1st generator is reduced by 5%, P 0=
= .4 40 MW = Q2 0= .38 38 MVAR.
     2
As saturation is neglected, E ∝ I f . 4. A 2300 V, 3-phase synchronous motor driving a
E1′ = 0.95 E1 = 1.216 pu pump is provided with a line ammeter and a field
rheostat. When the rheostat is adjusted such that
Field of 2nd generator is increased by 5%, the AC line current is minimum, the ammeter reads
E2′ = 1.05 E1 = 1.344 pu 8.8 A. What is the power being delivered to the
pump, neglecting losses? How should the rheostat
In case of changing field excitation, the active be adjusted so that the motor operates at 0.8 lead-
power output remains the same, only operating pfs ing power-factor? How many kVARs is the motor
or reactive power changes. supplying to the system at this new power-factor?
P=
1 P=
2 0.4 pu  remains same.  [2000]
Terminal voltage V = 1 pu is to be maintained con- Solution:
stant. For varying excitation, current is minimum for
unity power factor.
E1V
P1 = sin δ1′. Since, losses are neglected,
XS
Power output, i.e., power delivered to pump
1.216 × 1
0.4 = sin δ1′.
0.8 = 3 × 230 × 8.8 × 1
δ1′ = 15.26. = 350.56 kW.
E1′ = E1∠δ1′ = 1.216∠15.16 pu To make the power 0.8 leading, excitation is to be
increased. On increasing excitation, active power
E2′V remains constant.
P2 =
Similarly,  ⋅ sin δ 2′ .
XS Active power component of current is constant.

Chapter 04_Five Marks Question.indd 82 11/9/2015 6:00:42 PM


Chapter 4  Synchronous Machines | 3.83

I a1 cos φ1 = I a 2 cos φ2 . 7. The per unit voltage of two synchronous machines


connected through a lossless line are 0.95∠10°
I a1 cos φ1 8.8 × 1
Ia2 = = = 11 A. and 1.0 ∠ 0°.  Match the two sides in the following
cos φ2 0.8  [1996]
So, excitation is to be increased to make 11 A cur- (a) Real power of Machine 1
rent flow. (b) Reactive power of Machine 1
3 VL I L sin φ (c) Power factor of Machine 1
kVAR Supplied to system =
1000 (P) Positive real power
= 26.292 kVAR. (Q) Positive reactive power
(R) Negative real power
5. A single-phase alternator has a synchronous reac-
(S) Negative reactive power
tance of 2 ohms and negligible resistance. If it sup-
(T) Leading power factor
plies10 A to a purely capacitive load at 200 V, then
(U) Lagging power factor
generated emfwill be … V and the regulation will
be …% [1997] Solution:
The alternator which is having greater δ will de-
Solution:
liver real power, while the other one will absorb
E ∠δ = V ∠0 + I a ∠φ Z S φ real power.
= 200 + 10∠90 ⋅ 2∠90 = 200 − 20 = 180 V. The alternator which has more per unit voltage, de-
| E f | − |V | 180 − 200 livers reactive power and operates at lagging power
V.R. = = × 100 = −10%. factor, while the other one absorbs reactive power
V|
|V 200
and operates at leading power factor.
6. A 5 MVA, 11 kV, 3-phase star connected alterna- Thus the correct mapping is (a) → (P), (b) → (S),
tor is synchronized to the bus bars and is operating and (c) → (T).
with an induced EMF of 125% of the rated volt-
age. If the load current is 500 A. What is the power 8. The distribution factor for a 36 slot stator with
factor of operation? The machine has a synchro- three-phase, 8-pole winding, having 120° phase
nous reactance of 5 Ω and negligible resistance per spread is ____________ [1995]
phase. [1997] Solution:
Solution: 36
m= = 1.5.
E ∠δ = V ∠0 + ( I a ∠φ )( Z S φ ). 3× 8
Slots/Ph (36 / 3) 12 4 × 3
X S = 5 Ω. = = = . S K = 3.
Pole 8 8 4× 2
I 2 = I P = 500 A.
E = 1.25 × 11 kV = 13.75 kV. K: Highest common factor.
EPh = 7938.56 V.  120 
sin  
VPh = 11 kV/ 3 = 6350.85 V. Kd =  2  = 0.844.
 120 
Z S = X = 5. 3 sin  
 2×3 
EPh = VPh + jX SPh [ I aph cos φ − I aph sin φ ]
9. Figure depicts the characteristics of an isolated
= [VPh + I aph X SPh sin φ ] + jI aph X SPh cos φ . three-phase alternator, running at constant speed.
| EPh |2 = (VPh + I aph X SPh sin φ ) 2 + (500 × 5 cos φ ) 2 . Match the following sets of operating conditions
with the given characteristics. Disregard the effects
(7938.5) 2 = (6350.8 + 500 × 5 sin φ ) 2 of saliency, saturation and stator resistance.[1992]
+ (500 × 5 cos φ ) 2 .  (a) 
Constant excitation and non-zero leading
sin φ = 0.518. ­power-factor
(b)  Constant excitation and zero power-factor,
Power factor, cosφ = 0.855 (lag ). leading

Chapter 04_Five Marks Question.indd 83 11/9/2015 6:00:45 PM


3.84 | Electric Machines

(c) Constant terminal voltage and zero power-­ On increasing the load current beyond rated
factor, leading value, cross magnetizing effect dominates
(d) Constant terminal voltage and none-zero lead- then magnetizing effect and terminal voltage
ing power-factor decreases.
Q (B) For zero power factor and constant excitation,
P
armature reaction is completely magnetizing,
then terminal voltage increases.
PU Terminal voltage
or Excitation voltage

1 (C) For constant terminal voltage and zero power


R factor leading, on increasing the load to main-
tain constant terminal voltage excitation volt-
S age is reduced due to increase in magnetiza-
tion effect.
0 1.0 (D) 
For constant terminal voltage and non-zero
PU Stator current leading power factor, to maintain the constant
terminal voltage excitation must be reduced
Solution: for increase in load current upto rated value.
(A) For constant excitation and non-zero leading But beyond the rated current the cross mag-
power factor load with increase of load cur- netization effect of armature reaction is domi-
rent, armature reaction is partly magnetizing nant. Hence, the excitation voltage must be
and partly cross magnetizing. This increases increased.
the terminal voltage.

Chapter 04_Five Marks Question.indd 84 11/9/2015 6:00:45 PM


Unit 4
Power Systems

Chapter 1:  Transmission and Distribution 4.3


Chapter 2:  Economics of Power Generation 4.28
Chapter 3:  Symmetrical Components and Faults 4.34
Chapter 4:  Power System Stability 4.48
Chapter 5:  Protection 4.58
Chapter 6:  Circuit Breaker 4.65
Chapter 7:  Generating Stations 4.68
Chapter 8:  Load Flows 4.70
Chapter 9:  HVDC 4.78
Chapter 10:  Per Unit System 4.80

Chapter 01.indd 1 11/12/2015 1:57:47 PM


Chapter 01.indd 2
Exam Analysis
Exam Year 92 93 94 95 96 97 98 99 00 01 02 03 04 05 06 07 08 09 10 11 12 13 14
1 Mark Questions 3 3 2 4 7 4 3 5 4 2 4 6 6 3 5 4 3 2 3 3 3 2 6
2 Marks Questions – 1 1 1 1 – 5 5 5 5 3 10 10 8 8 6 7 5 6 5 2 4 7
3 Marks Questions – – – – – – – – – – – – – – – – – – – – – – –
5 Marks Questions 1 – 3 5 1 2 3 4 3 4 4 – – – – – – – 1 – – – –
Total Marks 4 4 6 10 9 6 11 14 12 11 11 16 16 11 13 10 10 7 10 8 5 6 13
Transmission and Distribution 2 1 3 4 2 1 5 9 3 4 3 6 7 3 3 5 2 2 4 3 1 1 4
Economics of Power Generation – – – 1 – – 1 1 1 1 – 1 – 1 – 1 1 1 – 1 1 – 1
Symmetrical Components and Faults – – 1 1 1 1 1 2 2 1 2 2 3 3 1 1 4 – 1 – 1 – 5
Power System Stability – 1 1 – 1 2 2 1 2 1 2 2 1 1 2 1 1 1 – – 1 1 2
Protection 1 1 – 2 2 1 1 – 3 1 1 1 1 – 1 – 2 1 1 1 – – 1
Circuit Breaker – – – 1 1 1 – 1 – – – 1 – – 1 1 – – 1 – – – –
Generating Stations – – – – 1 – – – 1 1 1 – 3 – – – – 1 – – – – –
Load Flows 1 1 – 1 1 – – – – 1 2 2 – 1 3 – – 1 – 1 1 4 –
HVDC – – 1 – – – 1 – – – – 1 – 1 1 1 – – 1 – – – –
Per Unit System – – – – – – – – – 1 – – 1 1 1 – – – 2 2 – – –

11/12/2015 1:57:47 PM
Chapter 1
Transmission and
Distribution
Solution:
One-mark Questions Power PIm = 12 MW, cos φIm = 0.6 lag

1. The undesirable property of an electrical insulat-


ing material is [2014-S1]
(a) high dielectric strength C Im
(b) high relative permittivity
(c) high terminal conductivity
(d) high insulation resistivity After connecting capacitor bank, overall pf cosϕ2
Solution: (b) = 0.8 lag
An electrical insulator should have: Without shunt capacitor bank, cosϕ1 = 0.6 lag
1. High dielectric strength for better insulation Reactive power to be injected by shunt capacitor bank.
­capability.
2. High insulation resistivity for getting high re- Qc = P[tan φ1 − tan φ2 ]
sistance and low current. = 12[tan(cos −1 0.6) − tan(cos −1 0.8)]
3. High thermal conductivity. = 7 MVAR.
4.  High relative permittivity is an undesirable
4. A single load is supplied by a single voltage source.
property of an electrical insulating material as
If the current flowing from the load to the source
losses increases due to the increase in dielectric
is 10∠-150° A and if the voltage at the load termi-
loss at high relative permittivity because of the
nals is 100∠60° V, then the [2013]
increase in capacitance value.
(a) load absorbs real power and delivers reactive
(Dielectric loss ∝ C0 ∝ εr) power
Hence, the correct option is (b). (b) load absorbs real power and absorbs reactive
2. A single phase induction motors draws 12 MW power
power at 0.6 lagging power. A capacitor is con- (c) load delivers real power and delivers reactive
nected in parallel to the motor to improve the power power
factor of the combination of motor and capacitor (d) load delivers real power and absorbs reactive
to 0.8 lagging. Assuming that the real and reactive power
power drawn by the motor remains same as before, Solution: (b)
the reactive power delivered by the capacitor in Consider the figure given below
MVAR is [2014-S2]

Chapter 01.indd 3 11/12/2015 1:57:48 PM


4.4 | Power Systems

10∠−150° We know that power flow in a line is inversely pro-


+ portional to the reactance of the line therefore as
reactance of line decreases, power flow in the line
VS ∼ 100∠60°
Ca increases.
Hence, the correct option is (c).
6. Consider a step voltage wave of magnitude 1 pu
Impedance across the load will be
travelling along a lossless transmission line that ter-
−VL −100∠60° −(50 + j86.6) minates in a reactor. The voltage magnitude across
ZL = = = .
IL −10∠ − 150° −8.66 − j 5 the reactor at the instant the travelling wave reaches
Z L = (8.66 + j 5) Ω. the reactor is [2010]
A
So load is RL. Therefore load absorbs real power
and also absorbs reactive power.
Reactor
Hence, the correct option is (b).
3. A nuclear power station of 500 MW capacity is
located at 300 km away from a load centre. Select
the most suitable power evacuation transmission
configuration among the following options. [2011]
(a) -1 pu (b) 1 pu
(a)
∼ Load center
(c) 2 pu (d) 3 pu
132 kV, 300 km Double circuit
Solution: (c)
(b)
∼ Load center Step voltage V1 = 1.0 pu
132 kV, 300 km single circuit with 40%
V 1.0 pu
series capacitor compensation
A
(c)
∼ Load center
400 kV, 300 km single circuit Reactor
(d)
∼ Load center
400 kV, 300 km double circuit
Solution: (d)
V 2 400 × 400 The reactor is initially open circuit
SIL = = = 400 MW.
ZC 400 V2 = V + V1 = 1.0 + 1.0 = 2.0 pu
Hence, the correct option is (c).
The carry 500 MW a double circuit 400 kV line is
used. 7. Consider two buses connected by an impedance of
(0 + j5) Ω. The bus 1 voltage is 100∠30° V, and bus
Hence, the correct option is (d).
2 voltage is 100∠0° V. The real and reactive power
5. For enhancing the power transmission in a long supplied by bus 1, respectively, are [2010]
EHV transmission line, the most preferred is to (a) 100 W, 268 V Ar
connect a [2011] (b) -1000 W, -134 V Ar
(a) series inductive compensator in the line (c) 276.9 W, -56.7 V Ar
(b) shunt inductive compensator at the receiving end (d) -276.9 W, 56.7 V Ar
(c) series capacitive compensator in the line
Solution: (a)
(d) shunt capacitive compensator at the sending end
Bus 1 voltage is V1 = 100∠30° V and bus 2 voltage
Solution: (c) is V2 = 100∠0° V.
We know that series capacitance compensation re-
Since, δ1 > δ2, current will flow from bus 1 to bus 2.
duces reactance of the line.
Impedance will be Z = j5 Ω.

Chapter 01.indd 4 11/12/2015 1:57:49 PM


Chapter 1  Transmission and Distribution  |  4.5

V1 − V2 The wavelength for one complete sinusoidal varia-


The current will be I = 2π
Z tion will be λ = .
100∠30° − 100∠0° β
=
j5 2π
λ= = 4947.4 km.
= 10.35∠15° A. 0.00127

Complex power can be calculated using expression Line Length 300


∴ = = 0.0606 = 6.06%.
 S = P + jθ = V1I* Wavelength 4947.4
P + jQ = 100∠30° × (10.35∠15°)
Hence, the correct option is (d).
 ≈ 100 + j268
10. Consider the transformer connections in a part of a
P = 100 W  and  Q = 268 VAR powder system shown in the following figure. The
Hence, the correct option is (a). nature of transformer connections and phase shifts
8. For a fixed value of complex power flow in a trans- are indicated for all but one transformer. Which of
mission line having a sending end voltage V, the the following connections and the corresponding
real power loss will be proportional to [2009] phase shift θ, should be used for the transformer
(a) V (b) V 2 between A and B? [2007]
1 1 ∼
(c) 2 (d)
V V Y

Solution: (c) −30° A


15 kV
Let the complex power be S, mathematically it can Δ
be expressed as Δ
B
S = VI 30°
Y 0°
S
I=
V 400 kV
Autotransformer
If R is the resistance of transmission line
2 (a) Star - Star (θ = 0°)
S (b) Star - Delta (θ = -30°)
Real power loss = I 2 R =   R
V  (c) Delta - Star (θ = 30°)
S2R (d) Star - Zigzag (θ = 30°)
= Solution: (a)
V2
Transformer is y - y
As S and R are constant therefore
1 T1 = Δ y
Real power loss α 2
V T2 = Δ y
Hence, the correct option is (c). T3 = yy
9. An extra high voltage transmission line of length  T4 = y  (Auto transformer)
300 km can be approximate by a lossless line hav- Hence, the correct option is (a).
ing propagation constant b = 0.00127 radius per
11. Consider a bundled conductor of an overhead line
km. Then the percentage ratio of the line length to
consisting of three identical sub-conductors placed
wavelength will be given by [2008]
at the corners of an equilateral triangle as shown
(a) 24.24% (b) 12.12%
in the figure. If we neglect the charges on the other
(c) 19.05% (d) 6.06%
phase conductors and ground and assume that spac-
Solution: (d) ing between sub-conductors is much larger than
Line length = 300 km their radius, the maximum electric field intensity is
Propagation constant = 0.00127 rad/km experienced at [2007]

Chapter 01.indd 5 11/12/2015 1:57:51 PM


4.6 | Power Systems

Y (a) 28.78 (b) 21.60


(c) 16.60 (d) 12.47
X
Z Solution: (d)
W
Active power demand of the load = PL
(a) Point X (b) Point Y = 12 3 × 0.8 = 16.63 kW
(c) Point Z (d) Point W
cos θ = 0.8 ⇒ sin θ = 0.6.
Solution: (b)
Electric field intensity at various points are shown Reactive power demand of the load = QL
as follows: = 12 3 sin θ
∈1
∈3 ∈2 ∈3 ∈2 = 12 3 × 0.6
X 1
1 ⇒ QL = 12.47 kVAR.
∈1
For unity pf, the total reactive power is zero.
Assuming k VAR rating of capacitor bank QC.
2 Point X 3     2 Point Y 3 QL + QC = 0
1 1
∈3 QC = −QL = −12.47 kVAR
∈2 ∈3 ∈2
Hence, the correct option is (d).
∈1
∈1 1 4. The insulation strength of an EHV transmission
2 Point Z 3    2 Point W 3 line is mainly governed by [2005]
(a) lower power factor
From above diagrams we conclude that minimum (b) switching over voltage
cancellation of vector occurs at point Y, therefore, (c) harmonics
maximum electric field intensity is experienced at (d) corona
point Y.
Solution: (b)
Hence, the correct option is (b).  Switching over voltage governs the insulation
12. The concept of an electricity short, medium and strength of an EHV transmission line.
long line is primarily based on the [2006] Hence, the correct option is (b).
(a) nominal voltage of the line
15. The rated voltage of a 3-phase power system is
(b) physical length of the line
given as [2004]
(c) wavelength of the line
(a) RMS phase voltage
(d) power transmitted over the line
(b) PEAK phase voltage
Solution: (b) (c) RMS line to line voltage
The concept of electricity short, medium and long (d) PEAK line to line voltage
line is based length (L) of the line.
Solution: (c)
If L ≤ 80 km, the line is short line. Peak line to line voltage is the rated voltage of a
If 80 km < L ≤ 250 km, the line is medium line. 3-phase power system.
If L > 250 km, the line is long line. Hence, the correct option is (c).
Hence, the correct option is (b). 16. The phase sequence of the 3-phase system shown
in the following figure is [2004]
13. A 400 V, 50 Hz, 3-phase balanced source supplies
power to a star connected load whose rating is R
12 3 kVA, 0.812 3 kVA, 0.8 pf (lag). The rat-
ing (in kVAR) of the delta connected (capacitive)
reactive power bank necessary to bring the pf to Y
unity is [2006] B

Chapter 01.indd 6 11/12/2015 1:57:52 PM


Chapter 1  Transmission and Distribution  |  4.7

(a) RYB (b) RBY Vm I m


(c) BRY (d) YBR =3 cos φ − 0 [∵  All are ∠20°
2 2 displace vector]
Solution: (b) = 3V ph I ph cos φ
The phase sequence of the given figure is RBY.
RYB, BRY and YBR represent the same phase se- Hence, the correct option is (b).
quence of the figure (given below): 1 8. Bundled conductors are mainly used in high volt-
R age overhead transmission lines to [2003]
(a) reduce transmission line losses
(b) increase mechanical strength of the line
B (c) reduce corona
Y (d) reduce sag
Hence, the correct option is (b). Solution: (c)
17. Total instantaneous power supplied by a 3-phase By bonding of conductors, the self GMD of the
AC supply to a balanced RL load is [2004] conductors is increased.
(a) zero Corona loss ∝(V - V0)2.
(b) constant
(c) pulsating with zero average  V0 is approximately directly proportional to the
(d) pulsating with non-zero average size of the conductor Hence larger the size of the
conductor, larger will be the critical disruptive
Solution: (b) voltage and smaller will be the factor (V - V0)2 and
Phase voltage hence, smaller will be the corona loss.
Va = Vm sin ωt Hence, the correct option is (c).
Vb = Vm sin(ωt − 120°), 19. A 3-phase 11 kV generator feeds power to a constant
Vc = Vm sin(ωt − 240°) power unity power factor load of 100 MW through a
Phase current, ia = im sin(ωt - ϕ) 3-phase transmission line. The line-to-line voltage at
the terminals of the machine is maintained constant at
ib = im sin(ωt - 120 - ϕ), 11 kV. The per unit positive sequence impedance of
ic = im sin(ωt - 240 - ϕ) the line based on 100 MV A and 11 kV is j 0.2. The
Instantaneous total 3 phase line to line voltage at the load terminals is measured
Power = (Vm sin ωt )(im sin(ωt − φ )) + to be less than 11 kV. The total reactive power to be
less than 11 kV. The total reactive power to be injected
(Vm sin(ωt − 120°)) at the terminals of the load to increase the line-to-line
(im sin(ωt − 120 − φ )) + (Vm sin(ωt − 240°)) voltage at the load terminals to 11 kV is [2003]
(sin(ωt − 240°))(im sin(ωt − 240 − φ )). (a) 100 MV AR (b) 10.1 MV AR
(c) -100 MV AR (d) -10.1 MV AR
cos(ωt − ωt + φ ) − cos(2ωt − φ ) + 
cos(ωt − 120 − ωt + 120 + φ ) −  Solution: (b)
Vm I m   | VS | = | VR | = 11 kV pu impedance of the line =
= cos(ωt − 120 + ωt − 120 − φ ) + 
2   j 0.2  pu
cos(ωt − 240 − ωt + 240 + φ ) −  Impedance of the line = pu impedance × base
cos(ωt − 240 + ωt − 240 − φ )  ­impedance
112
 cos(2 − φ ) + cos(2ωt −   x = j 0.2 × = j 0.242 Ω.
Vm I m    100
= 3 cos φ  240 − φ ) + cos(2ωt −   Active power at receiving end = PR = 100 MW
2 
  480 − φ )  
|VS ||VR |
PR = sin δ
cos( 2ωt − φ ) + cos(2ωt −  X
3
= Vm I m cos φ − 120 − φ ) + cos(2ωt − 
 ⇒ 100 =
11× 11
sin δ .
2 0.242
 240 − φ ) 

Chapter 01.indd 7 11/12/2015 1:57:54 PM


4.8 | Power Systems

At receiving end the reactive power will be (c) has flat voltage profile and unity power factor
VS VR VR
2 at all points along it
QR = cos δ − (d) has sending end voltage higher than receiving
X X end voltage and unity power factor at sending end
11× 11 112 Solution: (c)
= cos 11.54 − = −10.1 MVAR.
0.242 0.242 In a flat line the magnitude of the voltage through-
Since, power factor = 1 out its length is same. For a surge impedance load-
Load reactive power should be zero. Therefore, ing, load impedance and line impedance is purely
reactive power to be injected at terminals = - QR resistive in nature.
= -(-10.1) = 10.1 MVAR Therefore, the power factor is unity throughout its
Hence, the correct option is (b). length.
20. Consider a long, two-wire line composed of solid Hence, the correct option is (c).
round conductors. The radius of both the conduc- 23. An overhead line having a surge impedance of 400 Ω
tors is 0.25 cm and the distance between their is connected in series with an underground cable
centres is 1 m. If this distance is doubled, then the having a surge impedance of 100 Ω. If a surge of
inductance per unit length [2002] 50 kV travels from the line towards the cable junc-
(a) doubled tions, the value of the transmitted voltage wave at the
(b) halves ­junction is [1999]
(c) increases but does not double (a) 30 kV (b) 20 kV
(d) decreases but does not halve (c) 80 kV (d) -30 kV
Solution: (c) Solution: (b)
Radius of both conductors r = 0.25 cm Surge impedance of overhead line ZC 400Ω
Distance between the centers of the conductors = d Surge impedance of underground cable ZL = 100Ω.
d Surge V = 50 kV
L ∝ ln  
 r′   The transmitted voltage calculated in transient
ln(2d ) > ln d condition will be
Hence, the correct option is (c).
 ZL 
21. A long wire composed of a smooth round conduc-  V2 = 2V  .
tor runs above parallel to the ground (assumed to  Z L + ZC 
be a large conducting plane). A high voltage exists
between the conductor and the ground. The maxi-  100 
∴ = 2 × 50 × 103 ×   = 20 kV.
mum electric stress occurs at [2002]  100 + 400 
(a) the upper surface of the conductor
(b) the lower surface of the conductor Hence, the correct option is (b).
(c) the ground surface 2 4. The load carrying capability of a long AC trans-
(d) midway between the conductor and the ground mission line is [1999]
Solution: (b) (a) always limited by the conductor size
If high voltage exists between the conductor and (b) limited by stability considerations
the ground, then the maximum electric stress oc- (c) reduced at low ambient temperature
curs at the lower surface of the conductor. (d) decreased by the use of bundled conductors of
Hence, the correct option is (b). single conductors
22. A lossless radial transmission line with surge Solution: (b)
impedance loading [2001] The load carrying capability of a long AC transmis-
(a) takes negative VAR at sending end and zero sion line is limited by stability consideration because
VAR at receiving end reactance is more for long transmission line it effects
(b) takes positive VAR at sending end and zero the stability.
VAR at receiving end Hence, the correct option is (b).

Chapter 01.indd 8 11/12/2015 1:57:54 PM


Chapter 1  Transmission and Distribution  |  4.9

25. Corona losses are minimized when [1999] (a) reduce the line loading
(a) conductor size is reduced (b) improve the stability of the system
(b) smoothness of conductor is reduced (c) reduce the voltage profile
(c) sharp points are provided in the line hardware (d) improve the protection of the line
(d) current density in conductors is reduced Solution: (b)
Solution: (d) Synchronizing power
Corona loss ∝  (V - V0)2 3EV
Psyn = cos δ .
d  X
Also,V0 ∝ r ln   X↓, Psy↑
r
Hence, the correct option is (b).
⇒  V0 ∝  r and V0 ∝  -ln r
2 8. For a 500 Hz frequency excitation, a 50 km short
   ∵ e x > x (This relation always holds for x > 0)
power line will be modeled as [1996]
As r being radius of conductor, r > 0 (a) short line
(b) medium line
⇒ er > r
(c) long line
r > ln r (d) data insufficient for decision
With this we can conclude that VC ∝ r Solution: (c)
1 1 For one full wave variation, the length of the line
Corona loss ∝ ∝ (1) for 50 Hz supply will be given by
V0 r
f λ = υ
1
Current density ∝
(2) where f = Supply frequency
r
Corona loss ∝  current density λ = Wavelength, i.e., the length of the line
Methods of option (a), (b) and (c) will increase co- υ = Velocity of wave = 3 × 108 m/sec.
rona losses, if applied. υ 3 × 108
 ∴ λ = = = 6000 km.
Hence, the correct option is (d). f 50
26. The reflection coefficient for the transmission line For 6000 km wave length, the line with more than
shown in the following figure at P is [1998] 160 km length is treated as long line.
p 3 × 108
Transmission line For 500 Hz wave,  λ = = 600 km.
500
Z 0 = 300 Ω
Z 0 = Surge Impedance Hence with 500 Hz supply, a line with more than
Load 16 km length is treated as long line, where line pa-
300 Ω rameters are distributed.
Hence, the correct option is (c).
29. The insulation level of a 400 kV, EHV overhead
transmission line is decided on the basis of [1995]
(a) +1 (b) -1
(a) lightning over voltage
(c) 0 (d) 0.5
(b) switching over voltage
Solution: (c) (c) corona inception voltage
As load impedance and surge impedance are same, (d) radio and TV interference
the voltage and current wave forms are not going Solution: (b)
to experience any reflection. Hence reflection coef- In any transmission line lightning overvoltages are
ficient is zero. more severe compared to switching over-voltages.
Hence, the correct option is (c). The lightning voltages are external voltages. Hence
27. Series capacitive compensation in EHV transmis- the insulation is provided for switching voltage
sion lines is used to [1998] Hence, the correct option is (b).

Chapter 01.indd 9 11/12/2015 1:57:56 PM


4.10 | Power Systems

30. The insulation resistance of a cable of length 10 km 33. A three phase overhead transmission line has its con-
is 1 MΩ. For a length of 100 km of the same cable, ductors horizontally spaced with spacing between
the insulation resistance will be [1995] adjacent conductors equal to d. If now the conduc-
(a) 1 MΩ (b) 10 MΩ tors of the line are rearranged to form an equilateral
(c) 0.1 MΩ (d) 0.01 MΩ triangle of sides equal to d then [1993]
Solution: (c) (a) average capacitance will increase
Insulation resistance, (b) average capacitance will increase and induct-
ance will increase
ρ R (c) average capacitance will increase and induct-
Rins = ln   Ω. ance will decrease
2π l  r 
(d) surge impedance loading of the line increases
R2  l1  Solution (c)
⇒ =  .
R1  l2  L ∝ GMD
 10  1
R2 = 1 MΩ  C∝
 = 0.1 MΩ. GMD
 100 
Hence, the correct option is (c). 2d
3 1. The surge impedance of a 400 km long overhead
transmission line is 400 Ω. For a 400 km length of d d
the same line, the surge impedance will be [1995] d d
d
(a) 200 Ω (b) 800 Ω
GMD = 21/3d    GMD = d
(c) 400 Ω (d) 100 Ω
Solution: (c) Hence, the correct option is (c).
Surge impedance for a given transmission line is 3 4. The inductance of a power transmission line
constant and is independent of length of the trans- increase with [1992]
mission line and frequency of surge. It depends (a) decrease in line length
only on magnitude of inductance/km and capaci- (b) increase in diameter of conductor
tance/km (c) increase in spacing between the phase conductors
(d) increase in load current carried by the conductors
 Inductance/km  Solution: (c)
∵ Z c = 
 Capacitance/km  Inductance per phase of a transmission line is
d
Hence, the correct option is (c).    ( L ph ) = 0.2 ln  1 
r 
3 2. In a 400 kV network, 360 kV is recorded at a 400 kV where d is the space between two conductors, r′ is
bus. The reactive power absorbed by a shunt rated the effective radius of conductor
for 50 MV AR, 400 kV connected at the bus is
∴  Lph a ln(d )
[1994]
(a) 61.73 MV AR (b) 55.56 MV AR [∵  r′ = 0.7788r] = Constant for a given conductor
(c) 45 MV AR (d) 40.5 MV AR Hence, the correct option is (c).
Solution: (d) 35. The selection of size of conductors for a distribu-
tor in a distribution system is governed by [1992]
V2 (360) 2 (a) corona loss (b) temperature rise
Qabsorbed
= =
X rated (400) 2 (c) radio interference (d) voltage drop
50 Solution: (d)
= 40.5 MVAR.  ρl 
Resistance of a conductor  R =  , where A = Area
 A
Hence, the correct option is (d). of cross-section of conductor. In a distribution

Chapter 01.indd 10 11/12/2015 1:57:57 PM


Chapter 1  Transmission and Distribution  |  4.11

system, if the drop of voltage (IR) is more, means 2R R R R


the current [(or) load] is more for a given length I1 + ( I1 − 200) − ( I 2 − 200) − I 2 + 400
400 =
5 5 5 5
of conductor. The size of conductor must be in-
2 I1 − 200 200 − I 2 I 2
creased, so that the resistance of conductor is re- or  I1 + + − =0
duced and hence the voltage drop. 5 5 5 5
Hence rated voltage is maintained at the load or  2 I1 + I1 − 200 + 200 − I 2 − I 2 = 0
­terminal. or  3I1 − 2 I1 = 0
Hence, the correct option is (d). 2
or I1 = I 2 (1)
3
Two-marks Questions Also,  I1 + I2 - 400 = 100
or  I1 + I2 = 500 (2)
1. A distribution feeder of 1 km length having resist- Solving Equations (1) and (2), we have,
ance, but negligible reactance, is fed from both the I1 = 200 A  and  I2 + 300 A
ends by 400 V, 50 Hz balanced sources. Both volt-
age sources S1 and S2 are in phase. The feeder sup- Contribution of S1 in 100 A at location
plies concentrated loads of unity power factor as P = I1 - 200 = 0 A  and
shown in the figure [2014-S1] Contribution of S2 in 100 A at location
P = I2 - 200 = 300 - 200 = 100 A
S1 S2
400 m 200 m 200 m 200 m Therefore, S2 alone supplies the total load at loca-
∼ ∼ tion P.
P
400 V 400 V
200 A 100 A 200 A Hence, the correct option is (d).
50 Hz 50 Hz
2. The horizontally placed conductors of a single-
The contributions of S1 and S2 in 100 A current phase line operating at 50 Hz are having outside
supplied at location P, respectively, are diameter of 1.6 cm and the spacing between cen-
(a) 75 A and 25 A (b) 50 A and 50 A tres of the conductors is 6 m. The permittivity of
(c) 25 A to 75 A (d) 0 A and 100 A free space is 8.854 × 10-12 F/m. The capacitance to
Solution: (d) ground per kilometre of each line is [2014-S2]
Assume the resistance of complete length of feeder (a) 4.2 × 10-9 F (b) 8.4 × 10-9 F
be R Ω, length of feeder = 1000 m. (c) 4.2 × 10 F -12
(d) 8.4 × 10-12 F
R Solution: (b)
Resistance per unit length = Ω /meter
1000 a b
Resistance of 400 m length
R 2R r
= × 400 = Ω d
1000 5 Radius of the conductors r = 0.8 cm, spacing be-
R tween the conductors d = 6 m, permittivity of free
Resistance of 200 m length = Ω
5 space ε = 8.854 × 10 –12 F/m.
Let the current supplied by the sources be I1 and I2. Capacitance to ground per kilometer of each line
can be calculated as
I1 I1 (I1 − 200)A (I2 − 200) I2 I2
2π ε 0ε r
S1 S2 Can = F/m
∼ ∼
 6 
ln  −2 
2R R Ω RΩ R
⎯Ω  0.8 × 10 
400 V ⎯Ω 5 5 400 V
5 5
50 Hz 200 A 100 A 200 A
50 Hz = 8.404 × 10−12 F/m
100 = (I1 + I2 − 400)
= 8.404 × 10−9 F/km.
If we apply KVL to source S1 to S2, then we get, Hence, the correct option is (b).

Chapter 01.indd 11 11/12/2015 1:57:59 PM


4.12 | Power Systems

3. For the system shown below, SD1 and SD2 are com- 1
plex power demands at bus 1 and bus 2, respec- tanδ =
ωCR
tively. If | V2 | = 1 pu, the VAR rating of the capacitor
1
(QG2) connected at bus 2 is [2012] tanδ = = 0.025.
314 × 0.102 × 10−6 × 1.25 × 106
Bus 1 Bus 2
V 1 = 1∠0 pu V2 Hence, the correct option is (c).
S G1 ∼ Z = j 0.5 pu 5. Consider a 3-core, 3-phase, 50 Hz, 11 kV cable
Q G2 whose conductors are denoted as R, Y and B in the
figure. The inter-phase capacitance (C1) between
S D1 = 1 pu S D2 = 1 pu each pair of conductors is 0.2 mF and the capaci-
(a) 0.2 pu (b) 0.268 pu tance between each line conductor and the sheath
(c) 0.312 pu (d) 0.4 pu is 0.4 mF. The per-phase charging current is[2010]
Solution: (b) C2
|V ||V |
Real power Pr = S r sin δ R
|X | C1 C1
1.0 × 1.0
I= sin δ B Y
0.5 C2
C1
⇒ δ = sin −1 (0.5) = 30°. C2
(VS )(Vr ) (V ) 2
Reactive power Qr = cos δ ′ −
1× 1 (X )
Outer sheath
1.0 × 1.0 12
= cos 30 − (a) 2.0 A (b) 2.4 A
0.5 0.5
(c) 2.7 A (d) 3.5 A
 3
  Solution: (a)
2 
= −2 The per phase capacitance will be
1
  C /ph = C2 + 3 C1
2
⇒ 1.732 − 2 = −0.268. = 0.4 × 10−6 + 3 × 0.2 × 10−6
But Qr + QC = 0 = 1× 10−6 = 1 µF.
QC = -Qr  ⇒  0.268 pu. ∴  Per phase charging current = Vph WCph
Hence, the correct option is (b).
1I
4. A loss y capacitor CX Rated for operation at 5 kV, = × 103 × 2π × 50 × 1× 10−6 = 2 A.
3
50 Hz is represented by an equivalent circuit with
an ideal capacitor CP is found to be 0.102 mF and Hence, the correct option is (a).
the value of RP = 1.25 MΩ. Then the power loss and
6. Consider a 3-phase, 50 Hz, 11 kV distribution
δ of the loss y capacitor operating at the rated volt-
system. Each of the conductors is suspended by
age, respectively, are [2011]
an insulator string having two identical porcelain
(a) 10 W and 0.0002 (b) 10 W and 0.0025
insulators. The self-capacitance of the insulator is
(c) 20 W and 0.025 (d) 20 W and 0.04
5 times shunt capacitance. The voltages drop each
Solution: (c) disc is [2010]
V 2 5 × 103 × 5 × 103 (a) 3.46 kV
Power loss = =
R 1.25 × 106 (b) 34.6 kV
25 (c) 346 kV
= = 20 W. (d) None
1.25

Chapter 01.indd 12 11/12/2015 1:58:01 PM


Chapter 1  Transmission and Distribution  |  4.13

Solution: (a) ing VARs to control the voltage across the load to
within certain desirable limit.
C 5C
e1
Series capacitor compensation reduces the series
1
impedance of the line. Power flow in line ∝ ,
5C XL
e2 power flow in line increases, as XL decreases.
Hence, the correct option is (b).
8. A lossless transmission line having surge imped-
ance loading (SIL) of 2280 MW. A series capacitive
e1 = e2 (1 + K )
compensation of 30% is emplaced. Then SIL of the
11 compensated transmission line will be [2008]
e1 + e2 = .
3 (a) 1835 MW (b) 2280 MW
C 1 (c) 2725 MW (d) 3257 MW
K
= = = 0.2. Solution: (c)
5C 5
Let characteristics impedance
11
∴ e2 (1 + K ) + e2 = × 103 Z 1.0
3 (Z
= c) = = 1pu
Y 1.0
11 impedance/km X
e2 (1 + K ) = × 103 = = .
3 admittance/km B
e2 = 2.8867 ≅ 2.89 kV
Given that for a given line 30% series capacitive
e1 = e2 (1 + K ) = 2.8867 × 1.2 = 3.46 kV. compensation is provided. Hence the series imped-
ance of line is 0.7 or (70%) of original value.
Hence, the correct option is (a).
7. Match the items List-I (To) with the items in List-II 0.7
∴ Z new = = 0.836 pu.
(Use) and select the correct answer using the codes 1.0
given below the lists. [2009]
V2
Surge impedance loading (SIL) =
List-I List-II Zc
A.  Improve power factor 1.  Shunt reactor
1
B.  Reduce the current ripples 2.  Shunt capacitor ⇒ SIL ∝
Zc
C. Increase the power flow in 3.  Series capacitor
(SIL) 2 Z c1
line =
(SIL)1 Z c 2
D.  Reduce the Ferranti effect 4.  Series reactor
(a) a → 2, b → 3, c → 4, d → 1 1.0
(SIL 2 ) = × 2280 × 106
(b) a → 2, b → 4, c → 3, d → 1 0.836
(c) a → 4, b → 3, c → 1, d → 2 = 2725 × 106 = 2725 MW.
(d) a → 4, b → 1, c → 3, d → 2 Hence, the correct option is (c).
Solution: (b) 9. 230 V (Phase) 50 Hz, 3-phase, 4-wire, system has a
Shunt capacitors are used to provide part of the re- sequence ABC. A unity power-factor load of 4 kW
active VARs required by the load to keep the volt- is connected between phase A and neutral N. It is
age within desirable limits and to improve factor. desired to achieve zero neutral current through
Series reactor reduces current ripple. the use of a pure inductor and pure capacitor in
Shunt reactors are used across capacitive loads or the other two phases. The values of inductor and
lightly loaded lines to absorb some of the lead- capacitor are [2007]

Chapter 01.indd 13 11/12/2015 1:58:03 PM


4.14 | Power Systems

(a) 72.95 mH in phase C and 139.02 mF in phase B V V V


+ + =0
(b) 72.95 mH in phase B and 139.02 mF in phase C R XC X L
(c) 42.12 mH in phase C and 240.79 mF in phase B 1 1
(d) 42.12 mH in phase B and 240.79 mF in phase C + ωC ∠− 30 + ∠+ 30° = 0
R ωL
Solution: (b) 1 1
IR + ωC cos 30° + cos 30° ≠ 0
A R ωL
1
ωC sin 30 = sin 30.
R V∠0° ωL
In
First condition can never be zero, because all the
positive parts never becomes zero.
XC
V∠−120°
Hence, the correct option is (b).
IB V∠+120° 10. The total reactance and total susceptance of a loss-
Iy
B less overhead EHV line, operating at 50 Hz are
given by 0.045 pu and 1.2 pu, respectively. If the
C
velocity of wave propagation is 3 × 105km/s, then
IR + Iy + IB = In = 0. the approximately length of the line is [2007]
(a) 122 km (b) 172 km
V2 2302
= 4000
= ,R = 13.225 (c) 222 km (d) 272 km
R 4000
Solution: (c)
V ∠0° V ∠−120° Reactance = X (in pu) = 0.045 pu.
⇒ In = 0 = + + VωC ∠+ 120∠+ 90
R ω∠ + 90 Suspectance = Y (in pu) = 1.2 pu.
V V Let base impedance = ZB
⇒ + ∠− 210° + VωC ∠+ 210° = 0
R ωL X (in Ω) = 0.045Z B
V V
⇒ + cos 210 + V ωC cos 210° = 0 1.2
X (in Ω) = Ω
R ωL ZB
V
⇒− sin 210 + V ωC sin 210° = 0. Assuming impedance of line = LH/km
ωL
ω
1 X=
ω= (1) L
LC Capacitance of line CF/km
1  ω 2 LC + 1  3 ω
= × Y=
R  ωL  2 C
L = 72.9 mH where, l = length of the line
C = 139.02 µF X 0.045 Z B Y 1.2
L= =  and  C = =
ωl ωl ωl ωl Z B
If suppose ‘XC’ on phase B, XL on phase C Velocity of propagation is given by
A 1
VC =
LC
R V∠0°
1 ωl
VC = = .
 0.045 Z B   1.2  0.45 × 1.2
 ωl   ωl Z 
XL   B 
V∠−120°
XC
V∠+120° 2 × π × 50 l
3 × 105 = ⇒ l ≈ 222 km
B 0.045 × 1.2
C Hence, the correct option is (c).

Chapter 01.indd 14 11/12/2015 1:58:06 PM


Chapter 1  Transmission and Distribution  |  4.15

11. Single line diagram of a 4-bus single source dis- 3. If e3 is opened
tribution system is shown below. Branches e1, e2,
e3 and e4 have equal impedances. The load current ∼
values indicated in the figure are in per unit.
1A
∼ 7A
e1 e2

e1 e2 e4
1A 5A
2A

2A
1 + jo e 3 e 4 5 + jo
Total losses = 12R + 72R + 22R = 54R

2 + jo
4. If e4 is opened

Distribution Company’s policy requires radial ∼


system operation with minimum loss. This can be
achieved by opening of the branch [2007]
3A 5A
(a) e1 (b) e2
e1 e2
(c) e3 (d) e4
Solution: (d)
e3 5A
Assuming impedance of each branch R 1A 2A
1. If e1 is opened
∼ 2A

Total losses = 52R + 32R + I 2R = 38R


8A Operation with minimum loss can be achieved by
e2 opening line e4.
e4 Hence, the correct option is (d).
1A
1A 5A 12. The A, B, C, D constant of a 220 kV line are:
e3 3A
A = D = 0.94∠10, B = 130∠730, C = 0.001∠900.
If the sending-end voltage of the line for a given
2A
load delivered at nominal voltage is 240 kV, the %
Total losses = 8 R + 32R + I 2R = 74R
2
voltage regulation of the line is [2006]
2. If e2 is opened (a) 5 (b) 9
(c) 16 (d) 21
∼ Solution: (c)
Sending-end voltage = VS = 240 kV
8A Full-load receiving-end voltage = (VR)FL = 220 kV
e1 240
Vs( per phase) = kV.
3
e3 e4
1A As we know,
7A 5A
5A Vs ( per phase) = AVR ( per phase) + BI R
At no-load,
2A
IR = 0.
Total losses = 82R + 72R + 52R = 138R No-load receiving-end voltage (per phase)

Chapter 01.indd 15 11/12/2015 1:58:07 PM


4.16 | Power Systems

VS ( per phase) Solution: (c)


= (VR ) NL ( per phase) = Let the initial power factor angle = ϕ1
A
240 255.32 After connecting a capacitor, the power factor angle
= = kV. = ϕ2
3 × 0.94 3
Given ϕ2 = cos-1 0.97 = 14.07°
No-load receiving-end voltage (line to line)
P(tan ϕ1 - tan ϕ2) = kVAR Supplied by capacitor
= (V
=R ) NL 255.32 kV. 4 × 106P(tan ϕ1 - tan 14.07) = 2 × 106
(VR ) NL − (VR ) FL φ1 = 36.89°
% Voltage regulation = × 100.
(VR ) FL cos φ1 = 0.8 lag
255.32 − 220
= × 100 ≈ 16%. Hence if the capacitor goes out of service the load
220 power factor becomes 0.8 lag.
Hence, the correct option is (c).
Hence, the correct option is (c).
13. At an industrial sub-station with a 4 MW load, a
capacitor of 2 MVAR is installed to maintain the 15. A 800 kV transmission line is having per phase line
load factor at 0.97 lagging. If the capacitor goes out inductance of 1.1 mH/km and per phase line capaci-
of service, the load power factor becomes [2005] tance of 11.78 nF/km. Ignoring the length of the line,
(a) 0.85 (b) 1.00 its ideal power transfer capability in MW is [2004]
(c) 0.80 lag (d) 0.90 lag (a) 1204 MW (b) 1504 MW
Solution: (d) (c) 2085 MW (d) 2606 MW
By opening the branch e4 we can allow only the Solution: (c)
possible minimum current through each branch Ideal power transfer capability is nothing but the
hence the drop is minimum. surge impedance loading of the line

L /Km
∴  Surge impedance Z C =
C /Km
e1 e2
1.1× 10−3
=
11.68 × 10−9
e3
5 + i0 = 306.88 Ω
1 + i0
V2
2 + j0
∴  Surge impedance loading =
ZC
Hence, the correct option is (d).
(800 × 103 ) 2
1 4. Two networks are connected in cascade as shown =
306.88
in the figure. With the usual notations the equiva-
= 2085.47 MW.
lent A, B, C and D constants are obtained. Given
that, C = 0.025∠45°, the value of Z2 is [2005] Hence, the correct option is (c).
Z 1 = 10∠30° W 16. A 110 kV, single core coaxial, XLPE insulated
power cable delivering power at 50 Hz, has a capac-
itance of 125 nF/km. If the dielectric loss tangent of
Z2 XLPE is 2 × 10-4, then dielectric power loss in this
cable in W/km is [2004]
(a) 5.0 (b) 31.7
(a) 10∠30° Ω (b) 40∠-45° Ω (c) 37.8 (d) 189.0
(c) 1 Ω (d) 0 Ω

Chapter 01.indd 16 11/12/2015 1:58:08 PM


Chapter 1  Transmission and Distribution  |  4.17

Solution: (b) load at the receiving end is 50 MW at 220 kV with a


power factor of 0.9 lagging, then magnitude of line to
line sending end voltage should be [2004]
I (a) 133.23 kV (b) 220.00 kV
IC (c) 230.78 kV (d) 246.30 kV
IR
Solution: (c)
A = 0.936 ∠0.98° and B = 142 ∠76.4° Ω
V IC I 220
R C = kV = 127∠0° kV
3
50 × 106
d
q IR = = 145.8 A.
I 3 × 220 × 103 × 0.9
V
V/R
Equivalent of a cable Phasor diagram IR = 0.1458 ∠-25.84° kA (Power factor = 0.9 lagging)

VS (phase voltage) = AVR + BIR
110
V (Phase voltage) = kV VS = (0.936 ∠0.98) × (127∠0) + (142∠76.4)
3
C = 125 nF/km ×(0.1458∠− 25.84)

tan δ = 2 × 10−4 = 133.23∠7.77 kV.


Line to line sending end voltage
Dielectric power loss in cable = P = V2 wC tan d
2 = |VS |l −l = 3 × 133.23 kV
 110 × 103  −9 −4
P=  × 2 × π × 50 × 125 × 10 × 2 × 10 = 230.78 kV.
 3 
Hence, the correct option is (c).
≈ 31.7 W/km
19. The ABCD parameters of a 3-phase overhead trans-
Hence, the correct option is (b). mission line are A = D = 0.9 ∠0. B = 200 ∠90° Ω
and C = 0.95 × 10-3 ∠90° S. At no-load condition a
17. A lightning stroke discharges impulse current of
shunt inductive, reaction is connected at the receiv-
10 kA (peak) on a 400 kV transmission line having
ing end of the line to limit the receiving-end voltage
surge impedance of 250 Ω. The magnitude of tran-
to be equal to the sending-end voltage. The ohmic
sient over voltage travelling waves in either direction
value of the reactor is [2003]
assuming equal distribution from the point of light-
(a) ∞ Ω (b) 2000 Ω
ning strike will be [2004]
(c) 105.26 Ω (d) 1052.6 Ω
(a) 1250 kV (b) 1650 kV
(c) 2500 kV (d) 290 kV Solution: (b)
At no-load condition
Solution: (a)
As the current is distributed equally, current on Active power at receiving end = PR = 0
each side = 5000 A Reactive power at receiving end = QR = reactive
Surge impedance of transmission line = 205 Ω power absorbed by the reactor to make | VS | = | VR |.
∴ The magnitude of transient over voltage = 250 A = D = 0.9∠0°
× 5000 = 1250 kV. | A | = 0.9  and  a = 0°
Hence, the correct option is (a). B = 200∠90° Ω
18. The generalized circuit constant of a 3-phase 220 kV B | = 200  and  b = 90°
| 
rated voltage, medium length transmission line are A |VS ||VR | A
= D = 0.936 + j0.01 = 0.936∠0.98°, B = 33.5 + j138 SR = ∠( β − δ ) − |VR |2 ∠( β − α )
| B| B
= 142.0∠76.4° Ω, C = (-5.18 + j914) × 10-6 Ω. If the

Chapter 01.indd 17 11/12/2015 1:58:10 PM


4.18 | Power Systems

The above equation is expressed in real and im- where z1 = z2


aginary parts. We can write the real and reactive Parameters of cable
powers at the receiving-end as
Inductance = LC = 0.4 mH/km and
|VS ||VR | A Capacitance = CC = 0.5 mF/km
PR = cos( β − δ ) − |VR |2 cos( β − α ) (1)
| B| B
Surge impedance of the cable = ZC
|VS ||VR | A
QR = sin c( β − δ ) − |VR |2 sin( β − α ) (2) LC 0.4 × 10−3
| B| B = = = 28.28 Ω.
CC 0.5 × 10−6
Since, PR = 0, equation (1) becomes
Parameters of OH transmission lines
|V |2 0.9
0 = R cos(90° − δ ) − |VR |2 cos(90° − 0) Inductance = L1 = 1.5 mH/km and
200 200
Capacitance = C1 0.015 mF/km
|V |2
0 = R sin δ Surge impedance of the lines = Z1 = Z2
200
δ = 0°. L1 1.5 × 10−3
Equation (2) becomes = = = 316.22 Ω.
C1 0.015 × 10−6
|VR |2 0.9 2 E = 20 kV.
QR = sin(90° − 0) − |VR | sin(90° − 0)
200 200
Junction voltage = Ej
0.1
QR = |VR |2. (3)
200 1
2E ×
Reactive power absorbed by reactor of reactance x ZC
=
|V |2 1 1 1
Q = R (4) + +
x Z C Z1 Z 2
Equating Equations (3) and (4) 1
2 × 20 ×
= 28.28 = 33.93 kV.
0.1 |V |2 1 2
|VR |2 = R +
200 x 28.28 316.22
⇒ x = 2000 Ω.
Hence, the correct option is (d).
Hence, the correct option is (b).
21. A balanced delta connected load of (8 + j6) Ω per
2 0. A surge of 20 kV magnitude travels along a lossless phase is connected to a 400 V, 50 Hz 3-phase supply
cable towards its junction with two identical lossless lines. If the input power factor is to be improved to
over head transmission lines. The inductance and the 0.9 by connecting a bank of star connected capaci-
capacitance of the cable are 0.4 mH and 0.5 mF per tors the required kVAR of the bank is [2003]
km. The inductance and capacitance of the overhead (a) 42.7 (b) 10.2
transmission lines are 1.5 mH and 0.015 mF per km. (c) 28.8 (d) 39.4
The magnitude of the voltage at the junction due to
surge is [2003] Solution: (b)
(a) 36.72 kV (b) 18.36 kV When load is connected to 3 - ϕ supply
(c) 6.07 kV (d) 33.93 kV
Solution: (d) 400
ZL

Z1
ZC
Load impedance = ZL = 8 + j6 Ω
Z2 As, load is delta connected phase to neutral voltage
T-junction (VP) = Line to line voltage (V1 - 1)

Chapter 01.indd 18 11/12/2015 1:58:12 PM


Chapter 1  Transmission and Distribution  |  4.19

VP = V1 - 1 = 400∠0° V Given that VP - VQ = 3 V, assume current in 0.1 Ω


section distributors is ‘I’
Phase current = IP
By applying Loop,
VP 400∠0°
= =
ZL 8+ j6 VP − VQ = 0.1 I + 0.15( I − 20) + 0.2( I − 50)
= 40∠− 36.87° A. 3 = 0.1 I + 0.15 I − 0.15 × 20 + 0.2 I − 0.2 × 50

= S=
Load complex power L 3VP I P* VP VQ
0.1 Ω Q 0.15 Ω S 0.2 Ω
= 3 × 400∠0°× (40∠− 36.87°)* P 35.55 A 15.55 A 14.45 A Q
S L = 38.4 + j 28.8 kVA
PL + jQL = 38.4 + j 28.8
10 A 20 A 30 A 15 A
PL = 38.4 kW and QL = 28.8 kVAR
When capacitors banks are connected, power fac- Minimum voltage occurs at point s and this value
tor changes to 0.9. given as 200 V.
Assuming required kVAR of the bank = Qcap Now VP
cos θ = 0.9 = 0.1(35.55) + 0.15(15.55) + Vs (or )Vmi
⇒ θ = 25.842° = 225.887 V.
Q QL − Qcap Hence, the correct option is (b).
tan θ = =
PL PL 2 3. The conductor of a 10 km long, single-phase, two-
28.8 − Qcap wire line are separated by a distance of 1.5 m. The
tan 25.842° = diameter of each conductor is 1 cm. If the conductors
38.4
are of copper, the inductance of the circuit is [2001]
Qcap ≈ 10.2 kVAR.
(a) 50.0 mH (b) 45.3 mH
Hence, the correct option is (b). (c) 23.8 mH (d) 19.6 mH
2 2. A DC distribution system is shown in the figure with Solution: (c)
load currents as marked. The two ends of the feeder
are fed by voltage sources such that VP - VQ = 3 V. d 
Lab = 0.4 ln   mH/km
The value of the voltage VP for a minimum voltage r
of 220 V at any point along the feeder is [2003]  
 150 
VP VQ = 0.4 ln   mH/km = 2.3815 mH/km.
0.1 Ω R 0.15 Ω S 0.2 Ω  0.7788 × 1 
P Q  2
For 10 km long line
Lab = 23.815 mH/km
10 A 20 A 30 A 15 A
Hence, the correct option is (c).
(a) 225.89 V (b) 222.89 V 2 4. Consider the model shown in the figure of a trans-
(c) 220.0 V (d) 228.58 V mission line with a series capacitor at its mid-point.
Solution: (b) The maximum voltage on the line is at the location
VP VQ [2001]
R 1 − 20 A S 1 − 50 A j 0.1 pu j 0.1 pu
I P1 P2 P3 P4
P 0.1 Ω 0.15 Ω 0.2 Ω Q
− j 0.15 pu I = 1 pu
V s = 1 pu Pf = 1 V
f
∠0°
10 A 20 A 30 A 15 A

Chapter 01.indd 19 11/12/2015 1:58:14 PM


4.20 | Power Systems

(a) P1 (b) P2 Solution: (d)


(c) P3 (d) P4
V  f + 25 
Solution: (c) Pcorona = 2.41×   (V − V0 ) W/km/ph
2

d 8 
VP 2 = 1 pu − 1 pu × j 0.1 pu P2 f + 25
⇒ = 2 .
VP 3 = 1 pu + 1 pu × j 0.1 pu P1 f1 + 25
VP 4 = 1 pu − 1 pu × j 0.1 pu 85
P2 = × 1 = 1.13 kW/km/ph.
75
⇒  P3 is maximum
Hence, the correct option is (d).
Hence, the correct option is (c).
27. For a single-phase overhead line having solid copper
25. A transmission line has equal voltages at the two
conductors of diameter 1 cm, spaced 60 cm between
ends. Maintained constant by two sources. A third
centres, the inductance in mH/km is [1999]
source is to be provided to maintain constant voltage
(a) 0.05 + 0.2 ln 60 (b) 0.2 ln 60
(equal to end voltages) at either the midpoint of the
 60   60 
line or at 75% of the distances from the sending end. (c) 0.05 + 0.2ln   (d) 0.2 ln  
Then the maximum power transfer capabilities of  0 . 5   0.5 
the line in the original case and the other two cases Solution: (c)
respectively will be in the following ratio: [2000] a
I
1: 2 :1 r
(a) 1 : 1 : 1 (b)
0.75 d
(c) 1 : 2 : 4 (d) 1 : 4 : 6 I
Solution: (b) b
V2
Pmax = (1) Given r = 0.5 cm, d = 60 cm.
x
Inductance per conductor
With compensator at mid point,
V2 δ ( La ) = La internal + La external
P′ = sin
x 2 µ0µr µ0µr  d 
= + ln   H/m.
2 8π 2π r
′ = 2 Pmax
Pmax (2)
Since, mr = 1 for conductors and air medium
With compensator at 75% distance x′ = 0.75 x
4π × 10−7 × 1 4π × 10−7 × 1  d 
V2 1 La = + ln   H/m.
′′ =
Pmax = Pmax (3) 8π 2π r
0.75 x 0.75
 60 
From (1), (2) and (3) La = 0.05 + 0.2 ln   mH/km.
 0.5 
1
The ratio is 1 : 2 : Hence, the correct option is (c).
0.75
Hence, the correct option is (b). 2 8. A 220 kV, 20 km long, 3-phase transmission line
26. The corona loss on a particular system at 50 Hz has the following A, B, C, D constants: A = D =
is 1 kW/km per phase. The corona loss at 60 Hz 0.96∠3°, B = 55∠65° Ω / phase, C = 0.5 × 10-4
would be ∠90°S/phase. Its correct charging current per
[2000] phase is [1999]
(a) 1 kW/km per phase 11
(a) A (b) 11 A
(b) 0.83 kW/km per phase 3
(d) 1.2 kW/km per phase 220
(c) 220 A (d) A
(e) 1.13 kW/km per phase 3

Chapter 01.indd 20 11/12/2015 1:58:16 PM


Chapter 1  Transmission and Distribution  |  4.21

Solution: (a) (a) 4.5 V (b) 31.5 V


 Vphase
1 (c) 30 V (d) 20 V
Charging current/phase I c = ;  XC = 
XC  Y Solution: (c)
220 × 10 3 VC − (40 × j 0.35) − (40 + 30∠− cos −1 (0.8)) × j 0.25 = VA
=
 1  VC − A = 16.6207∠74.29° + 14∠90°
3× −4
∠ − 90°
 0.5 × 10  = 30.34∠81.468° V.
220 × 103 × 0.5 × 10−4 Hence, the correct option is (c).
= ∠90°
3 3 1. A cable has the following characteristics. L = 0.201
11 mH/m and C = 196.2 pF/m. The velocity of wave
= ∠90° A. propagation through the cable is [1998]
3
(a) 32 m/s (b) 159.24 m/ms
Hence, the correct option is (a). (c) 0.0312 m/s (d) 159.24 m/(m - s)
2 9. A 3-phase, 11 kV, 50 Hz, 200 kW load has a power Solution: (d)
factor of 0.8 lag. A delta connected 3-phase capaci-
1 1
tor is used to improve the power factor to unity. The v= =
capacitance per phase of the capacitor in microfar- LC 0.201× 10 × 196.2 × 10−12
−6

ads is [1999] 1000


= = 159.24 m/µ
µs.
(a) 3.948 (b) 1.316 0.201× 196.2 µs/m
(c) 0.439 (d) 11.844
Hence, the correct option is (d).
Solution: (b)
3 2. A shunt reactor of 100 MVAR is operated at 98%
200 × 103 of its rated voltage and at 96% of its rated fre-
IL = ∠ − cos −1 (0.8) quency. The reactive power absorbed by the reac-
3 × 11× 103 × 0.8
tor is [1998]
= 10.4973 − j 7.8729.
(a) 98 MVAR (b) 104.02 MVAR
The reactive part of the line current is to be sup- (c) 96.04 MVAR (d) 100.04 MVAR
plied by capacitor bank in order to have unity pf. Solution: (d)
7.8729 V2 V2
I C ( phase ) = . (Δ connection) Q= =
3 X L 2π fL
2
V 7.8729 Q2  V2   f1 
= . =   × 
XC 3 Q1  V1   f 2 
11000 × 3  0.98V1   f1 
2
XC = = 2420.0178 Ω.. Q2 = 100 MVAR 
7.8729  
 V1   0.96 f1 
1 = 100.042 MVAR.
or, C = = 1.31532 µF
2π × 50 × 2420.02
Hence, the correct option is (d).
Hence, the correct option is (b). 3 3. For equilateral spacing of conductors of an
3 0. A single-phase AC distributor supplies two single- untransposed 3-phase line, we have [1996]
phase loads as shown in the figure. The voltage (a) balanced receiving end voltage and no com-
drop from A to C is [1999] munication interference.
A 0 + j 0.25 Ω B C (b) unbalanced receiving end voltage and no com-
230 V 0 + j 0.35 Ω munication interference.
(c) balanced receiving end voltage and communi-
30 A, 40 A, cation interference.
0-8 leg upf (d) unbalanced receiving end voltage and commu-
nication interference.

Chapter 01.indd 21 11/12/2015 1:58:18 PM


4.22 | Power Systems

Solution: (a) Solution:


For the transmission line with equilateral spacing The rated load of an underground cable is always
of conductors less than its natural loading (surge impedance
(Inductance/phase) is equal loading).
(Current/phase) is equal 35. The charging current of a 400 kV transmission
line is more than that of a 220 kV line of the same
(Flux/phase) is equal
length. [1994]
(Voltage drop/phase) is equal
Solution:
Hence no communication interference and also the
receiving end voltages are balanced. I C = 2π f V ph C ph .
Hence, the correct option is (a). I C ∝ −V ph .
34. The rated load of an underground cable is always Hence, the answer is true.
its natural loading (surge impedance
­loading). [1995]

Chapter 01.indd 22 11/12/2015 1:58:18 PM


Chapter 1  Transmission and Distribution  |  4.23

6.6 × 103
Five-marks Questions =
 1.5 
= 12.69 kV/cm 2 (rms)
0.75 × ln  
1. Consider a 3-phase, 50 Hz, 11 kV distribution sys-  0.75 
tems. Each of the conductors in suspended by an = 12.69 × 2 kV/cm(Peak )
insulator string having two identical porcelain insu- = 17.95 kV/cm(Peak )
lators. The self-capacitance of the insulator is 5
times the shunt capacitance between the link and the Hence, the correct option is (b).
ground, as shown in the figure. The voltages across 2. A long lossless transmission line has a unity power
the two insulators are [2010] factor (UPF) load at the receiving end and an AC
voltage source at the sending end. The parameters
e2 5C C of the transmission line are as follows:
e1 5C Characteristics impedance Zc = 400 W propagation
constant b = 1.2 × 10-3 rad/km, and length 1 = 100
km. The equation relating sending- and receiving-
(a) e1 = 3.47 kV,  e2 = 2.61 kV end questions is Vs = Vr cos h( β l ) + jZ c sin h( β l ) I R
(b) e1 = 3.46 kV,  e2 = 2.89 kV Complete the maximum power that can be trans-
(c) e1 = 6.0 kV,  e2 = 4.23 kV ferred to the UPF load at the receiving end if
(d) e1 = 5.5 kV,  e2 = 5.5 kV |Vs |= 230 kV.  [2002]
Solution: (b) Transmission lines IR
l3 ∼
UPF Load
e2 5C VS VR
A
e1 l2 C Solution:
5C
l1 It
Transmission lines VR

VS
At ‘A’ point I1 = I2 + I3
UPF Load
e1 (5Cω ) = e2 (ωC ) + e2 (5Cω ) VS = VR cos h( β1 ) + jZ C sin h( β1 ) I R
5e1 = 6e2 (1)
From standard form VS = AVR + BI R
11 k
e1 + e2 =
3 ∴ A = cos h( β1 ), B = jZ C sin h( β1 )
e1 + e2 = 6.35 kV (2) A = cos h(1.2 × 10−3 × 100) = 1.0
  
B = j 400 sin h(1.2 × 10−3 × 100)
From Equations (1) and (2) we can get
2πε 0ε r = 48.11 ∠90°.
(b) Capacitance, C = F/m
R Since given transmission line is of 100 km length,
ln  
r let us consider the line as short line.
2π × 8.854 × 10−12 × 3.5 VS  VR = 230 kV
C= = 0.281 nF/m
 1.5  Maximum power transferred Pmax is given as
ln  
 0.75  VS Vr A 2
−9 Pmax = − Vr cos( β − α )
Total capacitance,Ct = 0.281× 10 × 4000 = 1.12 µF B B
(c) Maximum electric stress in the insulation, A 2
Qmax = − VR sin( β − α )
V B
Emax = kV ⋅ cm/rms
R 230 × 230 × 106 1
r ln   ∴ Pmax = − (230 × 103 ) 2 cos(90 − 0)
r 48.11 48.11

Chapter 01_5 MARKS.indd 23 11/4/2015 5:03:15 PM


4.24 | Power Systems

Pr max = 1102.083 MW (a) Calculate the receiving-end voltage on open


1 2
circuit using justifiable assumptions.
∴ Qr max = − 230 sin(90 − 0) × 106 (b) What load at the receiving end will result in a
48.11
flat voltage profile on the line?
= −1102.83 MVA
AR (c) If the flat voltage profile is to be achieved at
Total power transferred 1.2 times the loading in (b), what will be the
= (1102.083 − j1102.83) MVA nature and quantum of uniformly distributed
compensation required?
3. A 132 kV transmission line AB connected a cable
Solution:
BC. The characteristics impedances of the overhead
(a) Lossless for long line parameter, A = cos bI
line and the cable are 400 W and 80 W, respectively.
Assume that these are purely resistive. A 250 kV β = ω LC = x /km ⋅ b /km
switching surge travels from A to B. [2001]
(a) Calculate the value of this voltage surge when = 0.05 × 3 × 10−6
−4
it first reaches C.   = 3.8729 × 10 rad/km
(b) Calculate the value of the reflected component
Vs ( LL)
of this surge when the first reflection reaches A. Vro ( L − L ) =
(c) Calculate the surge current in the cable BC. A
Solution: 275 kV
= 278.33 kV
B 800 Ω C 0.988
400 Ω
A (b) Flat voltage profile comes for SIL case at which
Surge voltage = 250 kV z x
Z=
L Z=
c = (For lossless line)
(a) Value of this surge voltage, when it first reaches y b
‘c’ is
0.05
ZL ZL = = 12.9.09 Ω
(V2 ) = 2V × 3 ××10−6
Z L + ZC
(c) If Z c = 129.09 Ω, SIL
80
= 2 × 250 × 103 × V 2 (275) 2
80 + 400 = = = 585.58 MW
Z c 129.09
= 83.33 kV  
If loading = 1.2 time SIL= 1.2 × 585.8 MW
(b) Value of reflected voltage when first reflection = 703 MW
reaches A is
To make SIL of line as 703 MW ‘Z’ of line has
 Z − ZC  to be modified by putting compensation.
V1 = V  L 
 Z L + ZC  New value of Zc required, Zc (new) =
v2
 80 − 400  SIL new
= 250 × 103   = −166.67 kV
 80 + 400  2752
Z c (new ) = = 107.575 Ω
703
(c) Surge current in cable
Zc Value can be reduced either by placing se-
V 83.33 × 103 ries (or) shunt capacitance compensation
BC ( I 2 ) = 2 = = 1041.625 A
ZL 80 We know that Z c = xL ⋅ xc
xc → Series inductive reactance
4. A 275 kV, 3-phase, 50 Hz, 400 km lossless line has
following parameters: xc → Shunt capacitive reactance
 x = -0.05 Ω/km charging susceptance, y = 3.0 mi- 1
xc = = 333.3 × 103 Ω
cro-Siemens/k [2000] b

Chapter 01_5 MARKS.indd 24 11/4/2015 5:03:18 PM


Chapter 1  Transmission and Distribution  |  4.25

(i) with series capacitor of x reactance, Y


= j 3 × 10−4 S/ph
zc = ( xL − x) ⋅ xc 2
37.5 Ω j 75 Ω
10.575 = (0.05 − x) × 333.3 × 103

x = 0.0153 Ω/km → Series compensation 66 kV


∼ 66 kV − j 3 × 10−4 Ω
Vr 0
√3 √3
With shunt compensation of x (or) capacitance
x ⋅ xc
⇒  Equivalent shunt reactance =
x + xc ZY
A = 1+
Parameter  = 0.9778∠0.6°
2
 x ⋅ xc  | V ( L − L) |
Z c (new ) = xL ⋅   Vr 0 ( L − L) = x
   x + xc  |A |
x × 333.33 k 66 kV
10.575 = 0.05 × = = 67.5 kV
x + 333.33 k 0.9778

(231.45 k )( x + 333.3 k ) = x(333.33 k ) 6. A 6.6 kV, 50 Hz single core lead sheathed cable
X = 757.26 × 10 Ω 3 has the following data:
Conductor diameter:1.5 cm, Length: 4 km
Shunt suceptance of capacitor banks,
Internal diameter of the sheath: 3 cm
1
b= Resistivity of insulation: 1.3 × 1012 Ω-m
x Relative permittivity of insulation: 3.5
1
b= Calculate
757.26 × 103 (a) The insulation resistance
= 1.32 µs/km → Series compensation (b) The capacitance and
5. A 66 kV, 3-phase, 50 Hz, 150 km long overhead (c) The maximum electric stress in the insulation
transmission line is open circuited at the receiving  [1999]
end. Each conductor has a resistance of 0.25 W/km, Solution:
an inductive reactance of 0.5 W/km and a capacitive Given V = 6.6 kV, f = 50 Hz Single-core lead
admittance to neutral of 0.04 × 10-4 S/km [1999] sheathed cable,
Solution: Conductor diameter, d = 1.5 cm;
Nominal -p equivalent circuit of given transmis- Conductor Radius, r = 0.75 × 10−2 m,
sion line is shown below.
Length of cable, 1 = 4 km = 4000 m
R iX
Internal diameter of sheath, D = 3 cm;
66 kV

Y Y Radius of sheath, R = 1.5 × 10−2 m,
√3 2 2
Resistivity of insulation, ρ = 1.3 × 1012 Ω-m
Total resistance of each conductor, R = 0.25 × 150 Relative permittivity of insulation, ε r = 3.5
= 37.5 W/ph (a) Insulation resistance,
Total inductive reactance of each conductor, X ρ R
= 0.5 × 150 = 75 W/ph Ri = × ln  
2π l r
Total capacitive admittance to neutral,
1.3 × 1012  1.5 
Y = j 0.04 × 10−4 × 150 = j 6 × 10−4 s/ph = × ln  Ω
 = 35.85 M-Ω
2π × 4000  0.75 
Y
= 3 × 10−4
2 7. Each conductor of a 33 kV, 3-phase system is sus-
Z = R + jx = 37.5 + j 75 Ω /ph pended by a string of three similar insulators. The

Chapter 01_5 MARKS.indd 25 11/4/2015 5:03:21 PM


4.26 | Power Systems

ratio of shunt capacitance to mutual capacitance cap-pin junction and tower is one-fifth of the
is 0.1. Calculate the voltage across each insulator capacitance C of each insulation unit. A guard
and the string efficiency. [1998] ring, effective only over the line-end insulator unit
Solution: is fitted so that the voltages on the two units near-
est the line-end are equal. [1998]
(a) Calculate the voltage on the line-end unit.
(b) Calculate the value of capacitance CX required.
C′ C V1 Solution:
C V c
C′ 2 V1
I1 (C/5)
C I2 I
1
V3 Cx V2 (C/5) 1
11 I3 1
I2
I2 V3
c
C = Insulator capacitance
C′ = Shunt capacitance Static capacitance ‘CX’ of guard ring compensates
C′ the shunt capacitance (C1)charging current.
K= = 0.1
C i.e., I 21 = I 211 (1)

V1 + V2 + V3 = String voltage ∴ I 3 = I 2 ≠ I1  and V3 = V2 ≠ V1 (2)



Applying KCL at point ‘a’
33
V1 + V1 (1 + k ) + V1 (1 + 3k + k 2 ) = × 103 V I 2 = I1 + I11
3
C 
33 V2ωC = V1ωC + (V1 )ω  
V1 (1 + 1 + k + 1 + 3k + k ) =
2
× 10 V
3
5
3
V 6
33 V2 = V1 + 1 = V1   = V3
V1 (3 + 0.1 + 0.3 + 0.01) = × 103 V 5 5
  3
6V
V1 = 5.587 kV ∴ V2 = V3 = 1
5
V2 = V1 (1 + k ) = 5.587(1 + 0.1) × 103 Given that V1 + V2 + V3 = 20 kV
= 6.146 kV  6V 
V1 + 2  1  = 20 kV
V3 = V1 (1 + 3k + k )
2
 5 
= 5.587(1 + 3 × 0.1 + 0.01) × 103  12 
V1  1 +  = 20 kV
= 7.32 kV  5
V1 = 5.88 kV
V1 + V2 + V3
String efficiency = × 100 6
3V3 (a) V2 = V3 = V1   = 7.06 kV
5
 33  (b) From Equation (1)
 × 10
3

= 
3 I 21 = I 211
× 100
3 × 7.32 × 103 ωC
= 86.76 % (V1 + V2 ) = = V3ωCx
5
8. In a transmission line each conductor is at 20 V  C
Cx =  1 + 1 × [∵ V2 = V3 ]
kV and is supported by a string of three suspen-
 V2  5
sion insulators. The air capacitance between each
Cx = 0.166 C.

Chapter 01_5 MARKS.indd 26 11/4/2015 5:03:24 PM


Chapter 1  Transmission and Distribution  |  4.27

9. A factory draws 100 kW at 0.7 pf lagging from a Since transmission line is a Y connected one
3-phase, 11 kV supply. It is desired to raise the pf
to 0.95 lagging using series capacitors. Calculate I Line = I ph
the rating of the capacitor required. [1997] 69.152 × 103
X C /ph = = 755.125 Ω
Solution: 3 × (5.252) 2
Load rating = 100 kW at 0.7 power factor (lagging)
1
3-phase supply voltage = 11 kV kVAR demand of = 755.125
load at 0.7 pf is 2π fC
1
100 × 103 C /ph = = 4.215 µF
Q1 = × sin(cos −1 0.7) 2π × 50 × 755.125
0.7
= 102.020 kVAR Since series capacitors are connected in phase
manner kVAR rating of each capacitor
kVAR demand of load at 0.95 pf is
69.152
= = 23.05 kVAR
100 × 10 3
3
Q2 = × sin(cos −1 0.95)
0.95
∴ Rating of capacitor is, 4.215 μF, 23.05 kVAR,
= 32.868 kVAR
11 kV
Hence to operate the load at 0.95 power factor 10. The increase in resistance due to non-uniform
kVAR to be supplied by using series capacitors distribution of current in a conductor is known as
effect. [1994]
= (102.020 − 32.868) kVAR Solution:
= 69.152 kVAR The increase in resistance due to non-uniform dis-
tribution of current in a conductor is known as skin
3I 2 X C = 69.152 kVAR effect.
3VL I L cos φ = 100 × 103
3 × 11× 103 × I L × 0.95 = 100 × 103
I L = 5.525 A

Chapter 01_5 MARKS.indd 27 11/4/2015 5:03:25 PM


Chapter 2

Economics of Power
Generation
From the given data,
One-mark Questions dPL dPL
= 2(0.5) PG1 = PG1 and =0
dP1 dPG 2
1. The figure shows a two-generator system supply-
ing a load of PD = 40 MW, connected at bus 2. 1 1
× 10, 000 = × 12, 500
(1 − PG1 ) (1 − 0)
Bus 1 Bus 2 G
2 100
G1 ∼ 1 − PG1 = ⇒ PG1 = 0.2 pu
∼ PG2 125
PG1 ⇒ PG1 = 0.2 *100 = 20 MW
=
But PL 0= .5 P12 0.5(0.2) 2 = 0.02 pu
PD = 40 MW PL = 0.02 *100 = 2 MW
The fuel cost of generators G1 and G2 are: C1 PG1 + PG 2 = PD + PL
(PG1) = 10,000 Rs/MW hr and C2(PG2) = 12,500 Rs/ 20 + PG 2 = 40 + 2 ⇒ PG 2 = 22 MW
MW hr and the loss in the line is Ploss(pu) = 0.5 PG21( pu ) , Hence, the correct option is (a).
where the loss coefficient is specified in pu on a 2. The incremental cost curves in Rs/MW hr for two
100 MVA base. The most economic power genera- generators supplying a common load of 700 MW
tion schedule in MW is [2012] are shown in the figures. The maximum and mini-
mum generation limits are also indicated. The opti-
= PG1 20
(a) = =
, PG 2 22 (b) PG1 22
= , PG 2 20
mum generation schedule is [2007]
= PG1 20
(c) =, PG 2 20 (d)
=PG1 0=
, PG 2 40
Incremental Cost Rs/MW hr
Solution: (a)
For economic load dispatch,
650
dF1 dF2
L1 = L2 450
dPG1 dPG 2
1 dF1 1 dF2
=
dPL dP1  dP  dP2 P
1− 1 −
L
 200 MW 450 MW
dPG1  dPG 2  Generator A

Chapter 02.indd 28 11/4/2015 5:06:50 PM


Chapter 2  Economics of Power Generation  |  4.29

Incremental Cost Rs/MW hr Diversity


Sum of individual maximum demands
800 = (> 1.0)
650 maximum demand on the sysstem
Installed capacity of a power system is based on the
simultaneous maximum demand on the system. If the
simultaneous maximum demand is less than installed
capacity is less and hence fixed cost is reduced.
P Hence load factor and diversity factors must be high.
150 MW 400 MW
Hence, the correct option is (d).
Generator B

(a) Generator A: 400 MW,


Generator B: 300 MW
Two-marks Questions
(b) Generator A: 350 MW,
1. The fuel cost functions of two power plants are
Generator B: 350 MW
(c) Generator A: 450 MW, Plant P1 : C1 = 0.05 Pg12 + APg1 + B
Generator B: 250 MW Plant P2 : C2 = 0.10 Pg 22 + 3 APg 2 + 2 B
(d) Generator A: 425 MW, where, Pg1 and Pg2 are the generator powers of
Generator B: 275 MW two plants and A and B are the constants. If the
Solution: (c) two plants optimally share 1000 MW load at incre-
Maximum incremental cost in Rs/MW hr for gen- mental fuel cost of 100 Rs/MW hr, the ratio of load
erator A = 600 (at 450 MW ) shared by plants P1 and P2 is [2014-S1]
Minimum incremental cost in Rs/MW hr for gen- (a) 1 : 4 (b) 2 : 3
erator B = 650 (at 150 MW ) (c) 3 : 2 (d) 4 : 1
As maximum value of incremental cost of A is less Solution: (d)
than minimum value of B. Cost functions C1 = 0.05 P12 + AP1 + B
∴ Generator A will operate at its maximum (O/P) C2 = 0.1P2 2 + 3 APg 2 + 2 B
450 MW and B at (700 − 450) = 250 MW
λ = 100 Rs/MW hr
Hence, the correct option is (c).
3. In order to have a lower cost of electrical energy   PD = Pg1 Pg 2 = 1000 MW
generation [1995] λ = Ic1 = Ic2 (1)
(a) The load factor and diversity factor should be ∂c1
low Ic1 = = 0.1Pg1 + A (2)
∂Pg1
(b) The load factor should be low but diversity
factor should be high ∂c2
  Ic2 = = 0.2 Pg 2 + 3 A (3)
(c) The load factor should be high but diversity ∂Pg 2
factor should be low From Equations (1) and (2),
(d) The load factor and diversity factor should be 0.1Pg1 + A = 100 (4)

high
From Equations (1) and (3),
Solution: (d)
   0.2 Pg 2 + 3A = 100 (5)
Average Load
Load factor = (< 1.0) Pg1 + Pg 2 = 1000 (6)
Maximum demand
Average load is always less than the maximum de- Solving (4), (5) and (6),
mand. If the average load is nearer to maximum = Pg1 800 = MW; Pg 2 200 MW
demand then the effective utilization of steam will
Pg
= : Pg =
800 : 200 4 : 1
occur, which will results in saving in fuel. Hence 1 2

running cost will be reduced. Hence, the correct option is (d).

Chapter 02.indd 29 11/4/2015 5:06:53 PM


4.30 | Power Systems

2. A load centre of 120 MW deliver power from two Transmission losses = PL


power stations connected by 220 kV transmission
lines of 25 km and 75 km as shown in the figure = I12 R + I 22 3R + I 32 3R
below. The three generators G1, G2 and G3 are of 100 PL ∝ ( P12 + 3P22 + 3P32 )
MW capacity each and have identical fuel cost char-
acteristics. The minimum loss generation schedule PL = K [ P12 + 3P22 + 3P32 ]

for supplying the 120 MW load is [2011] Option (a)
P1 = 80 MW
25 km 75 km ∼ G2
∼ P2 = 20 MW
G1 ∼ G3
P3 = 20 MW
120 km
PL = K [802 + 3 × 202 + 3 × 202 ] = 8800 kMW

P1 = 80 MW + losses
(a) Option (b)
P2 = 20 MW P1 = 60 MW
P2 = 30 MW
P3 = 20 MW + losses
P1 = 60 MW
(b) P3 = 30 MW
P2 = 30 MW + losses PL = K [602 + 3 × 302 + 3 × 302 ] = 9000 kMW
P3 = 30 MW Option (c)
P1 = 40 MW
(c)
P1 = 40 MW
P2 = 40 MW
P2 = 40 MW
P3 = 40 MW + losses
P3 = 40 MW
P1 = 30 MW + losses
(d)
PL = K [402 + 3 × 402 + 3 × 402 ] = 11200 kMW
P2 = 45 MW
Option (d)
P3 = 45 MW
P1 = 30 MW
Solution: (a)
P2 = 45 MW
I2
I1 R 3R ∼ G2 P3 = 45 MW
∼ I3
∼ G3 PL = K [302 + 3 × 452 + 3 × 452 ] = 13050 kMW

120 MW So, option (a) gives minimum losses.


Hence, the correct option is (a).
Let load centre is connected at point x.
3. Three generators are feeding a load of 100 MW.
If r is resistance/km of line The details of the generators’ rating, efficiency and
Resistance of section between 1 and x regulation are shown below:

R1 = r × 25 = R (pu) on
(MW) % 100 MVA base
Resistance of section between x and 2
Generator-1 100 20 0.02
R2 = r × 75 = 3R Generator-2 100 30 0.04

Current (I ) fed by generator ∝ power produced by Generator-3 100 40 0.03


the generator.

In the event of increased load power demand,
I1 ∝ P1 , I 2 ∝ P2 and  I 3 ∝ P3 which of the following will happen? [2009]

Chapter 02.indd 30 11/4/2015 5:06:56 PM


Chapter 2  Economics of Power Generation  |  4.31

(a) All the generators will share equal power 5. A load centre is at an equidistant from the two
(b) Generator-3 will share more power as com- thermal generating stations G1 and G2 as shown in
pared to Generator-1 the figure. The fuel cost characteristics of the gen-
(c) Generator-1 will share more power as com- erating stations are given by [2005]
pared to Generator-2
F1 = a + bP1 + cP12 Rs/hour
(d) Generator-2 will share more power as com-
pared to Generator-3 F2 = a + bP2 + cP2 2 Rs/hour
Solution: (c)
As all the generators are of equal rating, the gen- ∼ ∼
erator with less value of regulation will share more
load and also the load sharing is independent of the G1 Load G2
efficiency of alternators.
where P1 and P2 are the generations in MW of G1
Hence, the correct option is (c). and G2, respectively. For most economic generation
4. A lossless power system has to serve a load of 250 to meet 300 MW of load P1 and P2, respectively, are
MW. There are two generators (G1 and G2) in the (a) 150, 150 (d) 100, 200
system with cost curves C1 and C2, respectively (c) 200, 100 (d) 175, 125
defined as follows:
Solution: (c)
C1 ( PG1 ) = PG1 + 0.055 × PG21 dF1
= b + 2CP1
C2 ( PG 2 ) = 3PG 2 + 0.03 × PG22 dP1

where PG1 and PG2 are the MW injections from dF2


= b + 4CP2
generator G1 and G2, respectively. Thus, the mini- dP2
mum cost dispatch will be [2008] For most economic generation
=
(a) PG1 250
= MW; PG 2 0 MW
dF1 dF2
=
(b) PG1 150
= MW; PG 2 100 MW =
dP1 dP2
=
(c) PG1 100= MW; PG 2 150 MW
b + 2CP1 = b + 4 CP2
=
(d) PG1 0= MW; PG 2 250 MW
P1 = 2 P2
Solution: (c)
Given P1 + P2 = 300
C1 ( PG1 ) = PG1 + 0.055 PG21
dC1 ∴ 2 P2 + P2 = 300
= 1 + 0.11PG1 P1 = 200 MW, P2 = 100 MW
PG1
C2 ( PG 2 ) = PG 2 + 0.03PG22 Hence, the correct option is (c).
dC2 6. Increment fuel costs (in some appropriate unit) for
= 3 + 0.06 PG 2 a power plant consisting of three generating units
PG 2
are IC1 = 20 + 0.3P1 , IC2 = 30 + 0.4 P2 , IC3 = 30.
For minimum cost analysis Assume that all the three units are operating all the
dC1 dC2 time. Minimum and maximum loads on each unit
=
PG1 PG 2 are 50 MW and 300 MW, respectively. If the plant
is operating on economic load dispatch to supply
    1 + 0.11PG1 = 3 + 0.06 PG 2 (1)
the total power demand of 700 MW, the power gen-
PG1 + PG 2 = 250 MW (2) erated by each unit is [2003]
Solving Equations (1) and (2), we get =
(a) P1 =
242 . 86 MW ; P2 157 . 14 MW ;
and  P3 = 300 MW
PG1 = 100 MW  and  PG 2 = 150 MW
=
(b) P1 157= .14 MW; P2 242.86 MW;
Hence, the correct option is (c). and  P3 = 300 MW

Chapter 02.indd 31 11/4/2015 5:07:00 PM


4.32 | Power Systems

= P1 300
(c) = .00 MW; P2 300.00 MW; Solution: (b)
and  P3 = 100 MW dF1
= 20 + 0.1P1
= P1 242
(d) = .86 MW; P2 157.14 MW; dP1
 and P3 = 300 MW dF2
= 16 + 0.2 P2
Solution: (a) dP2
When P1 is minimum, i.e.,  P1 = 50 MW
P1 + P2 = Pdemand = 200 MW
Given 
IC1 = 20 + 0.3 × 50 = 35
∴ P1 + P2 = 200 (1)
When P2 is minimum, i.e., P2 = 50 MW
IC2 = 30 + 0.4 × 50 = 50 For economic operationw

For minimum value of P1 and P2 dF1 dF2


λ= =
IC1   and  IC2 > IC3 dP1 dP2
20 + 0.1P1 = 16 + 0.2 P2
Therefore,  P3 = 300 MW [maximum load is assigned
to unit 3] 4 + 0.1P1
= P2
Remaining power is to be shared by unit 1 and 2. 0.2
P2 = 20 + 0.5 P1 (2)
P1 + P2 = 700 − 300 = 400 MW (1)
So, 
Solving (1) and (2)
For optimal operation
=P1 120
= MW, P2 80 MW
IC1 = IC2
20 + 0.3P1 = 30 + 0.4 P2 Hence, the correct option is (d).
0.3P1 − 0.4 P2 = 10 (2) 8. An industrial consumer has a daily load pattern of
2000 kW, 0.8 lag for 12 hrs and 2000 kW UPF for
Solving Equations (1) and (2), we get
12 hrs. The load factor is [1999]
P1 = 242.86 MW   and  P2 = 157.14 MW (a) 0.5 (b) 0.75

Hence, the correct option is (a). (c) 0.6 (d) 2.0
7. The incremental cost characteristics of two gen- Solution: (b)
erators delivering 200 MW are as follows: [2000] Actual number of units generated
Load factor =
maximum load × Total no. of hours
dF1 dF2
= 20 + 0.1P1 , = 16 + 0.2 P2
dP1 dP2 (2000 × 12) + (1000 × 12)
Load factor = = 0.75
2000 × 24
For economic operation, the generations P1 and P2
should be Hence, the correct option is (b).
(a) P=
1 P=
2 100 MW
= P1 80
(b) = MW, P2 120 MW
= P1 200
(c) = MW, P2 0 MW
= P1 120
(d) = MW, P2 80 MW

Chapter 02.indd 32 11/4/2015 5:07:05 PM


Chapter 2  Economics of Power Generation  |  4.33

Solving (1) and (2)


Five-marks Questions
= PG1 258= .33 MW, PG 2 341.67 MW
1. A power system has two generators with the fol- 2. In a power system, the fuel inputs per hour of plant
lowing cost curves [2001] 1 and 2 are given as: [1998]
Generator 1: C1 ( PG1 ) = 0.006 P G1 + 8 PG1 + 350
2 F1 = 0.20 P1 + 30 P1 + 100  Rs/hr
2

(Thousand Rs/hr) F = 0.25 P 2 + 40 P + 150  Rs/hr


2 2 2
Generator 2: C2 ( PG 2 ) = 0.006P 2G 2 + 7PG 2 + 400 The limits of generators are
(Thousand Rs/hr)
20 ≤ P1 ≤ 80 MW
The generator limits are

40 ≤ P2 ≤ 200 MW
100 MW ≤ PG1 ≤ 650 MW
Find the economic operating schedule of genera-

⋅ 50 MW ≤ PG 2 ≤ 500 MW tion, if the load demand is 130 MW, neglecting


transmission losses.
A load demand of 600 MW is supplied by the gen-
Solution:
erators in an optimal manner. Neglecting losses in
the transmission network, determine the optimal F1 = 0.2 P12 + 30 P1 + 100 Rs/hr
generation of each generator.
F2 = 0.25 P22 + 40 P2 + 150 Rs/hr
Solution:
dF1
C1 ( PG1 ) = 0.006 = 0.4 P1 + 30
dP1
PG21 + 8 PG1 + 350 Thousand Rs/hr dF2
= 0.5 P2 + 40
C2 ( PG 2 ) = 0.006 dP2
PG22 + 7PG 2 + 400 Thousand Rs/hr
For economic operation
dC1
= 0.012PG1 + 8 dF1 dF2
dPG1 λ= =
dP1 dP2
dC2
= 0.012PG 2 + 7 ⇒ 0.4 P1 + 30 = 0.5 P2 + 40
dPG 2
0.4 P1
⇒ = P2
Given load demand, 0.5
PD = PG1 + PG 2 = 600 MW
P2 = 0.8 P1 − 20 (1)

∴ PG1 + PG 2 = 600 (1) P1 + P2 = 130 (2)


Given 

For an optimal generation Solving (1) and (2)

dC1 dC2 P1 = 83.33 MW, P2 = 46.67 MW


λ= =
dPG1 dPG 2 P1 > P1 min ⇒ P1 = 80 MW; P2 = 50 MW
0.01PG1 + 8 = 0.012 PG 2 + 7
1 + 0.012 PG 2
PG 2 =
0.012
PG 2 = 83.33 + PG1 (2)

Chapter 02.indd 33 11/4/2015 5:07:08 PM


Chapter 3
Symmetrical Components
and Faults

•• Switch 1′ or 2′ is closed when winding is in Δ.


One-mark Questions Given, for transformer T1, switch 1 is closed and 2
is open.
1. A 2-bus system and corresponding zero sequence
network are shown in the figure. [2014-S1] Therefore, transformer will be connected in pri-
mary with grounded neutral and secondary in star
Bus 1 T1 T2 Bus 2
with isolated neutral (i.e.,↓ λ λ).
∼ ∼
For transformer T2, switch 1 is closed and 2′ is
xMa closed. Therefore, primary winding is star con-
nected with neutral grounded whereas secondary
(a) will be delta connected (i.e., ↓ λ ∆)
X0G X0T1 X0L X0T2 X0M Hence, the correct option is (b).
2. Three phase to ground fault takes place at loca-
3xMa tions F1 and F2 in the system shown in the figure.

K=0 IF1
IF2
The transformers T1 and T2 are connected as A B
F1 F2
(a) and Δ (b) and Δ ∼
EA ∠d EB ∠0
(c) Δ and Δ (d)
Δ and Δ
VF1 VF2
Solution: (b)
We have to use switch diagram for the zero-sequence If the fault takes place at location F1, then the
network of transformer. voltage and the current at bus A are FF1 and IF1
1
X0
2 ­respectively. If the fault takes place at location F2,
then the voltage and the current at bus A are AF2 and
IF2 respectively. The correct statement about volt-
1′ 2′ ages and currents during at F1 and F2 is [2014-S1]
•• Switch 1 or 2 is closed when winding in Y and (a) VF1 leads IF1 and VF2 leads IF2.
neutral are grounded. (b) VF1 leads IF1 and VF2 leads IF2.

Chapter 03.indd 34 11/12/2015 10:16:14 AM


Chapter 3  Symmetrical Components and Faults  |  4.35

(c)
VF1 leads IF1 and VF2 leads IF2. S base (3φ ) = 3VL I L .
(d)
VF1 leads IF1 and VF2 leads IF2.
S 100× 106
Solution: (c) VL = ⇒ = 137 kV.
3 IL 3 × 437.38
(i)  Fault at F1:
It is a base voltage.
A B
F1 IF1 pu Voltage = 0.9.
EA ∠d EB ∠0
Ix Va
Vpu = .
VF 1 ∠ 0° Vb
Va = Vpu × Vb ⇒ 0.9 × 132 = 118 kV.
For a fault F1:
Both generator 1 and generator 2 are supplying the 4. The sequence components of the fault current are
fault current. The voltage at bus A is due to gen- as follows:
erator 2. The angle of generator is zero so that the I positive = j 1.5 pu , I negative = − j 0.5 pu , I zero = − j1 pu.
voltage angle at A is negative. Hence, VF1 lags IF1 Ix
fault current will be I x ∠ − 900 . The type of fault in the system is [2012]
(a) LG (b) LL
I F 1 = − I x = (1∠180°) I x ∠ − 90°. (c) LLG (d) LLLG
I F 1 = I x ∠90. Solution: (c)
→ VF 1 lags I F 1 . For LLG fault, I positive = I negative + I zero .
Hence, given data satisfies the above condition.
(ii) Fault at F2:
Hence, the correct option is (c).
A B 5. A 3-phase transmission line is shown in figure:
F1 IF1 F2
EA ∠d EB ∠0
∆Va
VF 2 ∠ 0° Ia

For a fault F2:


Both generator 1 and generator 2 are supplying the ∆Vb
fault current the voltage at bus A due to genera- Ib
tor 1. The angle of generator is d and it is positive
so that the voltage angle at bus A is also positive.
Hence, VF2 leads IF2 . ∆Vc

Now I F 2 is also ∠ − 90 . Ic

⇒  VF2 leads IF2. Voltage drop across the transmission line is given
Hence, the correct option is (c). by the following equation:
3. A three phase star-connected load is drawing power
at a voltage of 0.9 pu and 0.8 power factor lagging.  ∆Va   Z s Z m Z m  ia 
    
The three phase base power and base current are  ∆Vb  =  Z m Z s Z m  ib  .
100 MVA and 437.38 A respectively. The line-to-  ∆Vc   Z m Z m Z s  ic 
line load voltages in kV is . [2014-S2]
Solution: Shunt capacitance of the line can be neglected. If
the line has positive sequence impedance of 15 Ω
S base (3φ ) = 100 MVA.
and zero sequence in impedance of 48 Ω, then the
I base = 437.38 A. values of Zs and Zm will be [2008]

Chapter 03.indd 35 11/12/2015 10:16:16 AM


4.36 | Power Systems

(a) Z s = 31.5 Ω; Z m = 16.5 Ω During unbalanced grounded faults, negative and


(b) Z s = 26 Ω; Z m = 11 Ω zero sequence components exist at fault point,
which are maximum in value.
(c) Z s = 16.5 Ω; Z m = 31.5 Ω
(d) Z s = 11 Ω; Z m = 26 Ω Hence, the correct option is (b).
Solution: (b)
Positive sequence impedance = negative sequence
impedance Two-marks Questions
= Z s − Z m = 15 (1)
Zero-sequence impedance 1. In an unbalanced three phase system phase cur-
rent Ia = 1∠(-90°) pu, negative sequence ­current Ib2
= Z s + 2 Z m = 48  (2)
= 4∠(-150°) pu, zero sequence current Ic0 = 3∠90° pu.
Solving Equations (1) and (2), we get The magnitude of phase current Ib in pu is
Z s = 26 Ω and Z m = 11Ω  [2014-S1]
Hence, the correct option is (b). (a) 1.00 (b) 7.81
6. For a fault at the terminals of a synchronous gen- (c) 11.53 (d) 13.00
erator, the fault current is maximum for a [1997] Solution: (c)
(a) 3-phase fault. Given, I a = 1∠(−90°) pu
(b) 3-phase to ground fault.
I b2 = 4∠(−150°) pu
(c) line-to-ground fault.
(d) line-to-line fault. I c0 = 3∠(90°) pu

Solution: (c)
As zero sequence current in all the three phases of
In case of line to ground fault I = I= I R 2 and
R0 R1
unbalanced 3 - f system are equal, therefore,
fault current is I f = 3I R1 .
I a 0 = I b 0 = I c 0 = 3∠90° pu
Whereas in case of 3-phase fault, I= R2 I=R0 0
and the fault current I f = I R1. The fault current in Ia2
3-phase fault contains only positive sequence com-
ponent, whereas the negative and zero sequence Ia0
components are zeroes, since, it is a balanced fault.
Ib0
Hence, the correct option is (c).
7. For an unbalanced fault, with paths for zero Ic0 Ic2 Ib2
sequence currents, at the point of fault[1996] Zero sequence currents   Negative sequence currents
(a) the negative and zero sequence voltages are
minimum. Also, I b2 = 4∠ − 150°
(b) the negative and zero sequence voltages are
= Negative phase sequence current of phase b,
maximum.
therefore, referring to the negative phase sequence
(c) the negative sequence voltage is minimum and
phasor,
zero sequence voltage is maximum.
(d) the negative sequence voltage is maximum I a 2 = 4∠(−150° − 120°)
and zero sequence voltage is minimum. = 4∠ − 270°pu
Solution: (b)
We know that, phase current vectors in matrix
Normally in an alternator, negative and zero
form are given by
sequence components don’t exist, only positive
­
sequence components exist. Negative sequence  I a  1 1 1   I a 0 
components exist due to unbalanced fault (or) un-     
 I b  = 1 a a   I a1 
2
balanced load, whereas zero sequence components
exist only during grounded faults.  I c  1 a a 2   I a 2 

Chapter 03.indd 36 11/12/2015 10:16:20 AM


Chapter 3  Symmetrical Components and Faults  |  4.37

r
(where, operator a = 1∠120° ).
R
Here, I a = I a 0 + I a1 + I a 2.
b
Given, I a = 1∠ − 90° and I a0 = 3∠90°pu and
I a 2 = 4∠ − 270° pu (as obtained above)
Therefore, Y
I a1 = I a − ( I a 0 + I a 2 ) B y

= (1∠ − 90°) − (3∠90° + 4∠270°) (a)


R r

or, I a1 = 8∠ − 90° pu G
(b)
R r
Therefore,
G
I b = I a 0 + a 2 I a1 + a I a 2
R r
= (3∠90°) + (1∠120°) 2 (8 − ∠90°) (c)
+ (1∠120°)(4∠ − 270°) G

= 3∠90° + 8∠150° + 4∠ − 150° R


(d) r

I b = 11.53∠154.3° pu G
∴  Magnitude of phase current, Solution: (b)
Ib = 11.53 pu. R r

Hence, the correct option is (c). G


2. A three phase, 100 MVA, 25 kV generator has Primary side connected in star, secondary connect-
solidly grounded neutral. The positive, negative, ed in delta, so here is no path for zero sequence
and the zero sequence reactances of the genera- current flow, so preferable option is (b).
tor are 0.2 pu, 0.2 pu, and 0.05 pu, respectively, Hence, the correct option is (b).
at the machine base quantities. If a bolted single
Common Data for Questions 4–6:
phase to ground fault occurs at the terminal of the
unloaded generator, the fault current in amperes Vs1 X ZL Y Vs2
Zs1 Zs2
immediately after the fault is  [2014-S2] ∼ ∼
Solution: Ix
IF
Due to solid LG Fault at generator terminals F
Given that: Vs1 = Vs2 = 1 + j 0 pu, +ve sequence imped-
3 Eq1
If = . ances are Zs1 = Zs2 = 0.001 + j0.01 pu and ZL = 0.006
z1 + z2 + z0 + j 0.06 pu, 3f. Base MVA = 100, voltage base = 400
=
Given z1 j=
0.2, z2 j 0.2, z0 = j 0.05 pu kV(L - L).
Nominal system frequency = 50 Hz. The reference
Assume prefault voltage Eq1 = 1 pu
voltage for phase a is defined as V(t) = Vm cos (wt).
3 ×1 A symmetrical 3f fault occurs at centre of the line, I
If = = − j 6.667 pu point F at time t0 the +ve sequence impedance from
j 0.2 + j 0.2 + j 0.05
source S1 to point F equals (0.004 + j 0.04) pu. The
sbase (MVA) 100 M wave form corresponding to phase a fault current from
I base = = = 2309.4 A
3 × kVbase ( L − L) 3 × 25 k bus × reveals that decaying DC offset current is -ve
I f (A) = 6.667 × 2309.4 A = 15396 A and in magnitude at its maximum initial value. Assume
that the negative sequence are equal to +ve sequence
3. The zero-sequence circuit of the three phase trans- impedance and the zero sequence (Z ) are 3 times +ve
former shown in the figure is [2010] sequence (Z ).

Chapter 03.indd 37 11/12/2015 10:16:22 AM


4.38 | Power Systems

4. The instant (t0) of the fault will be [2008] Z1eq = (0.004 + j 0.04) (0.004 + j 0.04).
(a) 4.682 ms (b) 9.667 ms Z1eq = 0.002 + j 0.02 pu.
(c) 14.667 ms (d) 19.667 ms
Solution: (a) ER1 = Vth = 1.0 pu  [∴ pre fault voltage not specified].
Voltage, V = Vm cos wt. 1.0
∴ If = = 49.75 − ∠84.289° pu.
The current after fault 0.002 + j 0.02

2V 100 × 106
i (t ) = Ae
− ( R / L )t
+ cos (ωt − θ ). I f base = = 144.3 A.
Z 3 × 400 × 103

∴ I f actal = I f pu × I f base
At the instant of fault t = t0, i = 0.
= 49.75 × 144.3 = 7.18 kA
2V
0 = Ae − ( R / L ) t0 + cos (ωt0 − θ ). 7.18
Z = IX = = 3.59 kA.
2
− ( R / L ) t0 2V
Ae =− cos (ωt0 − θ ). Hence, the correct option is (a).
Z
6. Instead of the three phase fault, if a single line to
Maximum value of DC offset ground fault occurs on phase a at point F with zero
fault impedance, then the rms of the AC component
2V
Ae − ( R / L ) t0 = − cos (ωt0 − θ ) = Negative max . of fault current (Ix) for phase a will be [2008]
Z
(a) 4.97 pu (b) 7.0 pu
Z = 0.004 + j 0.004. (c) 14.93 pu (d) 29.85 pu
Z = Z ∠θ = 0.0402∠84.3°. Solution: (c)
Z1 = 0.002 + j 0.02 pu
θ = 84.3°.
Z 2 = Z1 = 0.002 + j 0.02 pu
ω t 0 − θ = 0.
Z 0 = 3Z1 = 0.006 + j 0.006 pu
θ
t0 = = 4.682 ms.
ω Z1 Ia1
Hence, the correct option is (a). Vs ∼
5. The rms value of the AC component of fault cur-
rent (Ix) will be [2008] Z2 Ia2

(a) 3.59 kA (b) 5.07 kA Ia0 = Ia1 = Ia2


(c) 7.18 kA (d) 10.15 kA
Z0 Ia0
Solution: (a)
For a 3-phase fault
ER1
Fault current ( I f ) = .
Z1eq
Vs
I a 0 = I a1 = I a 2 =
0.003 + j 0.03 F 0.003 + j 0.03 Z1 + Z 2 + Z 0
Vs
=
0.001 + j 0.01 0.001 + j0.01 Z1 + Z 2 + 3Z1
Z1eq Vs 1∠0°
= =
∼ ∼ 5 Z1 5 × (0.002 + j 0.02)
= 9.95∠ − 84.29 pu

Chapter 03.indd 38 11/12/2015 10:16:24 AM


Chapter 3  Symmetrical Components and Faults  |  4.39

=
Fault current I=
f 3 Ia0 Va  Ia 
pu
   
where, Vb  = [ Z ]  I b  (1)
= 3 × 9.95 = 29.85 pu
Vc   I c 
If 29.85
I xf =
pu
= = 14.93 pu
2 2
Ia  1 1 1  I p 
Hence, the correct option is (c).    2 α 1  I 
 I b  = K α   n 
7. Suppose we define a sequence transformation    α α 1 I 
2
between ‘a-b-c’ and ‘p-n-o’ variables as follows:  I c   o 
 [2007]
I p 
 fa  1 1 1  f p   
   2 α 1  f  , = KA  I n  .
 f b  = K α  n   
 f c   α α 2 1  f o   I o 

j
where α = e 3 and K is a constant. Similarly, Vph = K [ A]Vsequence
Now, if it is given that:
0.5 0 0 
V p  0.5 0 0  i p 
     = K [ A]  0 0.5 0  [ I sequence ]
Vn  = K  0 0.5 0  in   0 0 2 
V   0 0 2.0  io 
 o
0.5 0 0  −1
Va  ia  [A ]
   

= K [ A]  0 0.5 0  I ph (2)
and Vb  = ib  then, K
 0 0 2 
Vc  ic 
Comparing Equations (1) with (2)
 1.0 0.5 0.75
z = 0.75 1.0 0.5 
(a) 0.5 0 0 
 0.5 0.75 1.0  ∴ Z = A  0 0.5 0  [ A−1 ]
 0 0 2 
1.0 0.5 0.5
z = 0.5 1.0 0.5
(b)
1 1 1 0.5 0 0  1 α α 2 
0.5 0.5 1.0  1 2  
= α α 1  0 0.5 0  1 α 2 α 
3
 1.0 0.75 0.5   α α 2 1  0 0 2  1 1 1 
z = 3K  0.5 1.0 0.75
(c) 2

0.75 0.5 1.0   1 0.5 0.5


= 0.5 1 0.5 .
 1.0 −0.5 −0.5 0.5 0.5 1 
K2 
z=
(d) −0.5 1.0 −0.5
3  Hence, the correct option is (b).
 −0.5 −0.5 1.0 
8. Three identical star connected resistors of 1.0 pu
Solution: (b)
are connected to an unbalanced 3 phase supply.
V p  0.5 0 0  I p  The load neutral is isolated. The symmetrical
    
Here,   Vn  =  0 0.5 0   I n  components of the line voltages in pu calculations
V   0 are with the respective base values, the phase to
 o 0 2.0   I o 
neutral sequence voltages are [2006]

Chapter 03.indd 39 11/12/2015 10:16:27 AM


4.40 | Power Systems

Van 1 = X ∠(θ1 + 30°), Van 2 = Y ∠(θ 2 − 30°)


(a) ∴ Van 1 = X θ1 − 30°, Van 2 = Y θ 2 + 30°.

Van 1 = X ∠(θ1 − 30°), Van 2 = Y ∠(θ 2 + 30°)


(b) [Here X and Y are in per unit system, hence, are
same for line and phase values].
1 Hence, the correct option is (c).
Van 1 =
(c) X ∠(θ1 − 30°),
3
Common Data for Questions 9 and 10:
1
Van 2 = Y ∠(θ 2 − 30°) At a 200 kV sub-station of a power system, it is
3 given that the three-phase fault level is 4000 MVA
1 and single line to ground fault level is 5000 MVA.
Van 1 =
(d) X ∠(θ1 − 60°), Neglecting the reactance and the shunt suscept-
3
ances of the system.
1
Van 2 = Y ∠(θ 2 − 60°) 9. The positive sequence driving point reactance at
3
the bus is [2005]
Solution: (c) (a) 2.5 Ω (b) 4.033 Ω
Given the per unit line voltages (c) 5.5 Ω (d) 12.1 Ω
Vab1 = X ∠θ1 , Vab 2 = Y ∠θ 2 , Solution: (d)
−Vbn1 The short circuit MVA = 3VbaseIsc,
Vcn1 Vab1
where Isc = fault current.
Vbase
For 3-phase fault I sc = .
30° X 1eq
Van1

Vab1 = Van1 − Vbn1 Vbase


(+ve sequence) ∴ 3Vbase = 4000.
X 1eq
Vbn1 3 × Vbase
2

Vbn2
X 1eq = .
4000
2
 220 
3×  
X 1eq =  3 .
Van2
30°
4000
X 1eq = 12.1 Ω.

Vab2
Hence, the correct option is (d).
Vcn2 −Vbn2
1 0. The zero sequence driving point reactance at the
Vab1 = Van1 − Vbn1 bus is [2005]
(−ve sequence)
(a) 2.2 Ω Ω
(b) 4.84 
where Van 1, Vbn 1, Vcn 1, are +ve sequence phase (c) 18.18 Ω Ω
(d) 22.72 
voltages and Van 2, Vbn 2, Vcn 2, are -ve sequence
Solution: (b)
phase voltages. Vab 1, Vab 2, are +ve sequence and
For single line to ground fault,
-ve sequence line voltages respectively.
From the above phase or diagrams, 3 × Vbase
I sc = .
Line voltage leads phase voltages by 30° in +ve X 1eq + X 2 eq + X 0 eq
sequence,
3 × Vbase
Line voltage lags phase voltages by 30° in -ve ∴ 3 × Vbase × = 5000.
sequence. ( X 1eq + X 2 eq + X 0 eq )

Chapter 03.indd 40 11/12/2015 10:16:29 AM


Chapter 3  Symmetrical Components and Faults  |  4.41

9 × V 2 base So, Eg = 1.0 ∠0° pu


X 1eq + X 2 eq + X 0 eq = .
5000 Initial symmetrical current
2
 220  Eg 1∠0°
9×  =I= = .
X 1eq + X 2 eq + X 0 eq =  3 . X d′′ j 0.19
5000 I = 5.26∠ − 90° pu
X 1eq + X 2 eq + X 0 eq = 29.04 Ω.
(MVA) Base 100
But X 1eq = X 2 eq = 12.1 Ω. Base current = = kA.
3 (kV) Base 3 × 11
∴ X 0 eq = 29.04 − 24.2 = 4.84 Ω. I = 5.26 × Base current
Hence, the correct option is (b). 100
= 5.26 × kA
1 1. The parameters of transposed overhead transmis- 3 × 11
sion line are given as: self reactance Xs = 0.4 Ω/km = 30.37 kA.
and mutual reactance Xm = 0.1 Ω/km. The positive
Hence, the correct option is (c).
sequence reactance X1 and zero sequence reac-
tance X0 respectively in Ω/km are [2005] 1 3. A 3-phase transmission line supplies Δ- ­connected
load Z. The conductor c of the line develops an
(a) 0.3, 0.2 (b) 0.5, 0.2
open circuit fault as shown in figure. The ­currents
(c) 0.5, 0.6 (d) 0.3, 0.6 in the lines are as shown on the diagram. The +ve
Solution: (d) sequence current component in line a will be
Positive sequence reactance  [2004]
a
X1 = X s − X m Ia = 10∠0° A
= 0.4 − 0.1
Z Z
= 0.3 Ω/km.
Ib = 10∠180° A
Zero sequence reactance b
Ic = 0 Z
c
X0 = Xs + 2 Xm
= 0.4 − 2(0.1) (a) 5.78∠− 30° (b) 5.78∠− 90°
(c) 6.33∠− 90° (d) 10.00∠− 30°
= 0.6 Ω/km.
Solution: (a)
Hence, the correct option is (d).
I = 10∠0°A; I b = 10∠180°   and  I c = 0 A.
1 2. A 3-phase generator rated at 110 MVA, 11 kV   a
is connected through circuit breakers to a trans-   α = 1 ∠120°.
former. The generator is having direct axis sub-
transient reactance X ″d = 26% and synchronous Positive sequence current component in line a
reactance = 130. The generator is operating at no 1
load and rated voltage when a three phase short I a1 = [ I a + α I b + α 2 I c ]
3
circuit fault occurs between the breakers and the
transformer. The magnitude of initial symmetrical 1
= [10∠0° + (1∠120°) × (10∠180°) + (1∠240°) × 0].
rms current in the breakers will be [2004] 3
(a) 4.44 kA (b) 22.20 kA I a1 = 5.77∠ − 30°A.
(c) 30.39 kA (d) 38.45 kA Hence, the correct option is (a).
Solution (c) 1 4. A 500 MVA, 50 Hz 3-phase turbo-generated pro-
The generator is operating at no-load and rated duces power at 22 kV. Generator is Y-connected
voltage and its neutral is solidly grounded. Its sequence

Chapter 03.indd 41 11/12/2015 10:16:33 AM


4.42 | Power Systems

reactances are X = 1 X=2 0.15 X 0 = 0.05 pu. It is the transmission line. The voltage of the alternator
operating at rated voltage and disconnected from neutral with respect to ground during the fault is
the rest of the system (no load). The magnitude  [2003]
of the sub-transient line current for single line to (a) 513.8 V (b) 889.9 V
ground fault at the generator terminal in pu will be (c) 1112.0 V (d) 642.2 V
 [2004] Solution: (d)
(a) 2.851 (b) 3.333 For alternator,
(c) 6.667 (d) 8.553 Z1g = j 0.1 pu
Solution: (d) Z 2 g = j 0.1 pu
For single line to ground fault
And Z 0 g = j 0.04 pu.
Ia1
For line
Ea ∼ Z1
Z1l = j 0.1 pu.
Z2 Z 2l = j 0.1 pu.
Ia1 = Ia2 = Ia0 Z 0l = j 0.3 pu.
Ia2
Z n = j 0.05 pu.
Z0
Equivalent sequence impedances;
Ia0 Z1eq = Z1g + Z1l = j 0.1 + j 0.1
Since, the generator is operating at no-load and = j 0.2 pu.
rated voltage Z 2 eq = Z 2 g + Z 2l = j 0.1 + j 0.1
Ea = 1∠0 pu = j 0.2 pu.
Z1 = jX 1 = j 0.15 pu Z 0 eq = Z 0 g + Z 0 l + 3Z n
Z 2 = jX 2 = j 0.15 pu = j 0.04 + j 0.3 + 3 × j 0.05
Z 0 jX
And = = j 0.05 pu = j 0.49 pu.
0
For single line to ground fault
Ea
I a1 = I a 2 = I a 0 = Z1eq
Z1 + Z 2 + Z 0
1∠0° 1 ∠0° ∼
=
j 0.15 + j 0.15 + j 0.05
Z2eq
= 2.857 ∠90° pu
Ia1 = Ia2 = Ia0
Fault current = 3I a1 = 3 × 2.857 = 8.57 pu.
Hence, the correct option is (d). Z0eq
15. A 20 MVA, 6.6-kV, 3-phase alternator is con-
nected to a 3-phase transmission line. The per unit
positive-sequence, negative-sequence and zero-
sequence impedance of the alternator are j 0.1, j 0.1 1∠0
and j 0.04 respectively. The neutral of the alternator I a1 = I a 2 = I a 0 =
Z1eq + Z 2 eq + Z 0 eq
is connected to ground through an inductive reac-
tor of j 0.05 pu The per unit positive-, negative- and 1∠0
=
zero-sequence impedances of the transmission line j (0.2 + 0.2 + 0.49)
are j 0.1, j 0.1 and j 0.3, respectively. All per unit = − j 1.1236 pu.
values are based on the machine ratings. A solid
ground fault occurs at one phase of the far end of Fault current = I f = 3I a1 = − j 3.3708 pu

Chapter 03.indd 42 11/12/2015 10:16:36 AM


Chapter 3  Symmetrical Components and Faults  |  4.43

Voltage of neutral wrt ground 17. A generator is connected to a transformer which


feeds another transformer through a short feeder.
Vn = I f × Z n = − j 3.3708 × j 0.05 = 0.16854 pu.
The zero sequence impedance values are expressed
Per phase base voltage = Vp in pu on a common base and are indicated in figure.
The Thevenin equivalent zero sequence imped-
6.6 6.6
= KV = × 103 V. ance at point B is [2002]
3 3
6.6
Vn = × 0.16854 × 103 = 642.2 V. X 0 = 0.03 X 0 = 0.1 X 0 = 0.05 X 0 = 0.07
3 B

Hence, the correct option is (d).
1 6. A three-phase alternator generating unbalanced
voltages is connected to an unbalanced load
through a 3-phase transmission line as shown in
figure. The neutral of the alternator and the star
0.25 0.25
point of the load are solidly grounded. The phase
voltages of the alternator are Ea = 10∠0° V,
Eb = 10∠ − 90° V, Ec = 10∠120° V. The positive (a) 0.8 + j0.6 (b) 0.75 + j0.22
sequence component of the load current is [2003] (c) 0.75 + j0.25 (d) 1.5 + j0.25
Ea j1.0 Ω j1.0 Ω Solution: (b)

j 0.1 j 0.05 j 0.07 3 × 0.25


Eb j1.0 Ω j2.0 Ω B

j 0.03

Ec j1.0 Ω j3.0 Ω Z th
3 × 0.25
1.310∠ − 107°A (b)
(a) 0.332∠ − 120°A
0.996∠ − 120°A (d)
(c) 3.510∠ − 81°A
Solution: (d) Zth = 0.75 + j0.22

Since, both sides are grounded, we can take it as 3 Hence, the correct option is (b).
independent circuits. 18. A 50 Hz alternator is rated 500 MVA, 20 kV, with
E 10∠00 X d = 1.0 per unit and X d′′ = 0.2 per unit. It sup-
Hence, I a = a = = −5 jA.
Za 2j plies a purely resistive load of 400 MW at 20 kV.
The load is connected directly across the generator
Eb 10∠ − 90 terminals when a symmetrical fault occurs at the
Ib = = = 3.33∠ − 180
Zb 3j load terminals. The initial rms current in the gen-
Ec 10∠120 erator in per unit is [2001]
Ic = = = 2.5∠30°A. (a) 7.22 (b) 6.4
Zc 4j
(c) 3.22 (d) 2.2
1
I a1 = ( I a + α I b + α 2 I c ) Solution: (b)
3
For a purely resistive load the MVA and MW limits
1
= [5∠ − 90 + 3.33∠(−180 + 120) + 2.5∠(240 + 30)] will be same.
3
For three phase symmetrical fault
1
= (5∠ ∠90 + 3.33∠ − 60 + 2.5∠270)
3 Ef
If = .
= 3.510∠ − 81°A. X1
Hence, the correct option is (d). X 1 = 0.2 pu at 500 MVA base.

Chapter 03.indd 43 11/12/2015 10:16:38 AM


4.44 | Power Systems

400 ∴  Inductive reactance required for neutral


( X 1 ) 400 MVA = 0.2 × = 0.16. grounding, Xn = 0.0166 pu
500
1 Hence, the correct option is (a).
( I f ) pu = = 6.25 pu.
0.16 20. For the network shown in figure the zero sequence
reactances in pu are indicated. The zero sequence
Hence, the correct option is (b). driving point reactance of the node 3 is [1998]
1 9. The severity of line-to-ground and three phase
faults at the terminals of an unloaded synchronous 1 2 3
generator is to be same. If the terminal voltage is ∼ ∼
1.0 pu and Z1 = Z2 = j0.1 pu, Z0 = j0.05 pu, for the
X10 = 0.05
alternator, then the required inductive reactance Xgo = 0.1 Xgo = 0.2
for neutral grounding is [2000] ∆

(a) 0.0166 pu (b) 0.05 pu X10 = 0.15


(c) 0.1 pu (d) 0.15 pu
Solution: (a) (a) 0.12 (b) 0.30
For an unloaded alternator the fault current for line (c) 0.10 (d) 0.20
to ground fault is given by Solution: (c)
3 ER1
I f1 = .
X 1eq + X 2 eq + X 0 eq 0.15 0.05
For an unloaded alternator, fault current for 3-phase Xeq
0.1 0.2
fault is given by
ER
If2 = 1 .
X 1eq
Given that If1 = If2. X eq 0=
= .2 0.2 0.1 pu.

3 ER1 E Hence, the correct option is (c).
= R1 .
X 1eq + X 2 eq + X 0 eq X 1eq
3 X 1eq = X 1eq + X 2 eq + X 0 eq .
3(0.1) = 0.1 + 0.1 + X 0 + 3 X n .
0.1 = 0.05 + 3 X n .
X n = 0.0166 pu.

Chapter 03.indd 44 11/12/2015 10:16:39 AM


Chapter 3  Symmetrical Components and Faults  |  4.45

(b) Inter connection of sequence networks for line


Five-marks Questions to ground fault is as shown below.
IR1
1. A single line-to-ground fault occurs on an

(Xf = 0)
Fault element
unloaded generator in phase positive, negative,
and zero sequence impedances of the generator are 0.25
j 0.25 pu, j 0.25 pu and j 0.15 pu, respectively. The VR1
generator neutral is grounded through a reactance Vth = ER1 ∼ 1 pu
of j 0.05 pu. The prefault generator terminal volt- IR1
age is 1.0 pu [2002] n

LR1 = LR2 = LR0


(a) Draw the positive, negative, and zero sequence IR2
networks for the fault given.
(b) Draw the interconnection of the sequence net-
j0.25 VR2
works for the fault analysis.
(c) Determine the fault current. IR2

Solution: n j 0.15
Positive sequence impedance n
XG0
Negative sequence impedance
3Xn 0.15 VR0
Zero sequence impedance
IR0
Neutral reactance of generator
G
Prefault generator terminal voltage
Vth
IR1 I R1 =
(c)
X 1eq + X 2 eq + X 0 eq
X1eq j 0.25 1.0
=
VR1 = ER1 − IR1 j 0.25 + j 0.25 + j 0.3
Vth = ER1 ∼ 1 pu
= 1.25∠ − 90°.
n
Fault current I f = 3 I R1 = 3 × 1.25∠ − 90°
Positive sequence network:
(a)
  = 3.75∠ − 90°.
Negative sequence network:
2. For the configuration shown in figure, the breaker
IR2 connecting a large system to bus 2 is initially open.
The system 3-phase fault level at bus 3 under this
condition is not known. After closing the system
X2eq j 0.25 VR2 = − IR3 X2eq breaker, the 3-phase fault level at bus 1 was found
to be 5.0 pu. What will be the new 3-phase fault
n level at system bus 3, after the interconnection?
All per unit values are on common base. Prefault
Zero sequence network: load currents are neglected and prefault voltages
are assumed to be 1.0 pu at all buses. [2000]
XG0
4 1 2 3
n j 0.15 IR0
∼ ∼
IR0 3Xn = j 0.15 VR0 Gen System

Eg = 1.0 pu X1 = 0.2 pu
G Xd = 0.2 pu XLine = 0.3 pu

Chapter 03.indd 45 11/12/2015 10:16:41 AM


4.46 | Power Systems

=Eg 1=
.0 pu X 1 0.2 pu 4. Determine the required MVA rating of the circuit
breaker CB for the system shown in given figure.
=X d 0=
.2 pu X Line 0. 3 pu
Consider the grid as infinite bus. Choose 6 MVA
Solution: as base. [1999]
0.2 pu 0.3 pu 33
1 3 Transformer 3-phase, kV, 6 MVA, 0.01 + j 0.08
11
0.2 pu X pu impedance. Load 3-phase 11 kV, 5800 kVA, 0.8
Xg lag, j0.2 pu impedance. Impedance of each feeder
1 pu ∼ ∼ 1 pu
9 + j18 Ω.
Feeder 1 T

Ef Grid Feeder 2 Load


I 3φ = .
X1 CB

At bus (1) Solution:


(0.4)(0.3 + X ) The per unit impedance of transformer = 0.01 +
X1 = .
(0.4) + (0.3 + X ) j 0.08 pu
Ef By taking transformer rating as base per unit im-
= 5 pu (given ). 2
X1  6   11 
pedance of load = j 0.2 ×  × 
 5.8   11 
or X + 0.7 = 2(0.3 + X ).
= j 0.207 pu
⇒ X = 0.1 pu
Per unit impedance of each feeder
Now at bus (3)
6
(0.2 + 0.2 + 0.3) × 0.1 0.7 × 0.1 = (9 + j 18) × .
X 1eq = = . (33) 2
(0.2 + 0.2 + 0.3 + 0.1) 0.8
    = (0.04958 + j 0.09917) pu
1 pu 0.8
∴ If = = = 11.4285 pu
X eq 0.1× 0.7 The per unit reactance diagram of the given single
line diagram is
3. Determine the magnitudes of the symmetrical
components (IR1, IR2, and IR0) of the currents in at 0.04958
three phase (RYB) three wire system, when a short j 0.08 0.01 F j0.207
j 0.09917 CB
circuit occurs between R and Y phase wires, the
fault current being 100 A. [1999] 0.04958 j 0.09917
Solution:

1
I R 0 = [ I R + IY + I B ] = 0.
3
1
I R1 = I F ∠ − 30°.
3
1
= × 100∠ − 30°
3 If a fault occurs at F, then the equivalent imped-
ance at F is
= (50 − j 28.8675)A
1 Z eq = (0.03479 + j 0.12958) // ( j 0.207)
IR2 = × 100∠30°
3 = (0.013 + j 0.08)pu
= (50 + j 28.867)A. = 0.082∠80.87°.

Chapter 03.indd 46 11/12/2015 10:16:43 AM


Chapter 3  Symmetrical Components and Faults  |  4.47

Short circuit MVA rating of For Line to Line to ground fault:


MVA base 6 VR
=CB = = 73.17 MVA. V=
R0 V=
R1 VR 2 = .
Z eq 0.082 3

5. The positive sequence component of the voltage Fault current I f = 3I R 0 pu


at the point of fault in a power system is zero for a
fault. [1995]  X 2 eq 
I R 0 = − I R1  ,
Solution:  X 2 eq + X 0 eq 
In a 3-phase fault positive sequence component of
ER1
voltage at the fault point is zero. I R1 = pu.
X 2 eq ⋅ X 0 eq
Note: X 1eq +
X 2 eq + X 0 eq
For Line to Ground fault:
IR For Line to Line fault (or) 3-phase fault:
I=
R0 I=
R1 IR2 = .
3
VR1 0=
= , I R 2 I R 0 = 0,
Fault current I=
f I=
R 3I R 0 pu,
Fault current If = IR = IR1.
ER1 6. In a power-system, the 3-phase fault MVA is
I R0 = pu.
X 1eq + X 2 eq + X 0 eq always higher than the single-line-ground fault
MVA at a bus (State True or False) [1994]
For Line to Line fault: Solution:
= I R 0 0=
, I R 2 I R1 . False.
Fault current I f = 3I R1∠ − 90° pu,
ER1
I R1 = pu.
X 1eq + X 2 eq

Chapter 03.indd 47 11/12/2015 10:16:46 AM


Chapter 4

Power System Stability


3. A round rotor generator with internal voltage E1
One-mark Questions = 2.0 pu and X = 1.1 pu is connected to a round rotor
synchronous motor with internal voltage E2 = 1.3 pu
1. Shunt reactors are sometimes used in high voltage and X = 1.2 pu. The reactance of the line connect-
transmission system to [2014-S2] ing the generator to the motor is 0.5 pu. When the
(a) limit the short circuit current through the line. generator supplies 0.5 pu power, the rotor angle dif-
(b) compensate for the series reactance of the line ference between the machines will be [2003]
under heavily loaded condition. (a) 57.42° (b) 1°
(c) limitover-voltages at the load side under l­ ightly (c) 32.58° (d) 122.58
loaded condition. Solution: (c)
(d) compensate for the voltage drop in the line
E1 = 2.0 pu,   X = 1.1 pu
­under heavily loaded condition.
Solution: (c) X1 = 0.5 pu
# ∼ #
Limit over voltages at the load side under lightly
loaded condition. E1 = 2.0 pu E2 = 1.3 pu
X = 1.1 pu X = 1.2 pu
Hence, the correct option is (c).
2. The angle δ in the swing equation of a synchro- Electrical power delivered by generator (Pe) = 0.5 pu
nous generator is the [2013]
EV EE
(a) angle between stator voltage and current. Pe = sin(δ ) = 1 2 sin(δ1 − δ 2 ).
(b) angular displacement of the rotor with respect X X eq
to the stator. 2.0 × 1.3
0.5 = sin(δ1 − δ 2 ).
(c) angular displacement of the stator mmf with 1.1 + 0.5 + 1.2
respect to a synchronously rotating axis.
(d) angular displacement of an axis fixed to the rotor ∴ δ1 − δ 2 = 32.58°.

with respect to a synchoronously rotating axis. Hence, the correct option is (c).
Solution: (d) 4. Out of the consideration (i) to (iv) listed below,
Rotor axis (i) No distance limitation related to steady state
Synchronously
stability.
d (ii) No reactive power requirement from the sys-
rotating
wst
reference frame tem at the two terminals.
(iii) No substantial effect on fault level of the two
Reference
systems at the terminals in spite of the inter
Hence, the correct option is (d). connection.

Chapter 04.indd 48 11/4/2015 5:11:51 PM


Chapter 4  Power System Stability  |  4.49

(iv) No corona problems. The considerations which Solution: (a)


constitute advantages of HVDC transmission are When the synchronous machine swings from A to
 [2000] B due to disturbance and settles at a point C, then
(a) All of the above. (b) (i) and (iii). during this process, rotor angle δ increases until
synchronous speed is achieved and the mechanical
(c) (iii) and (iv). (d) (ii) and (iv).
input and electrical output are balanced. The rotor
Solution: (b) will be having synchronous speed at point A and
Reactive power compensation is required at the also at point B before it finally settles.
converter stations. The inter connection of two Pc
systems is alone by using isolation transformers so
that the fault on one side will not be reflected on to P max
other side. By using DC transmission, corona can B
x
be reduced but not completely avoided. C
A
Hence, the correct option is (b).
d
5 Steady state stability of a power system is the abil-
ity of the power system to [1999] Hence, the correct option is (a).
(a) maintain voltage at the rated voltage level.
8. The transient stability of the power system can be
(b) maintain frequency exactly at 50 Hz.
effectively improved by [1993]
(c) maintaina spinning reserve margin at all times.
(a) excitation improved by.
(d) maintain synchronism between machines and
(b) phase shifting transformer.
on external tie lines.
(c) single pole switching of circuit breakers.
Solution: (d) (d) increasing the turbine valve opening.
Hence, the correct option is (d).
Solution: (c)
6. A100 MVA, 11 kV, 3-phase, 50 Hz, 8-pole synchro- The transient stability of a power system can be
nous generator has an inertia constant H equal to improved by fast excitation system, use of high
4 MJ/MVA. The stored energy in the rotor of the speed circuit breakers, use of high speed gover-
E nors, single pole circuit breakers, and by dynamic
generator at synchronous speed will be H =
G resistance switching. In case of alternator terminal,
 [1997] the line to ground fault is more severe. In order
to maintain transient stability, employ single pole
(a) 100 MJ (b) 400 MJ
breaker operation.
(c) 800 MJ (d) 12.5 MJ
In case of a transmission line the frequently
Solution: (b) ­occurring fault is line to ground fault. In this case
Inertia constant (H) instead of opening all the 3 lines if we open one
line on which fault is occurred then the net electri-
kinetic energy stored in rotor in MJ cal power transferred during fault will not be zero.
= .
Machine rating in MVA(S) Hence, accelerating power is reduced and hence,
the transient stability is improved.
Energy stored = 4 × 100 = 400 MJ.
Hence, the correct option is (b). [∵ Pacc = Pmechanical − Pelectrical ].
7. During a disturbance on synchronous machine the Hence, the correct option is (c).
rotor swings from A to B before finally settling
down to a steady state at point C on the power
angle curve. The speed of the machine during Two-marks Questions
oscillation is synchronous at point (s) [1996]
(a) A and B (b) A and C 1. A two bus power system shown in figure supplies
(c) B and C (d) Only at C load of 1.0 + j0.5 pu. [2014-S1]

Chapter 04.indd 49 11/4/2015 5:11:52 PM


4.50 | Power Systems

Bus 1 Bus 2 2. A cylindrical rotor generator delivers 0.5 pu power


G1 1∠0°
V 1∠d2 in the steady-state to an infinite bus through a trans-
∼ mission line of reactance 0.5 pu. The generator no-
j 0.1 load voltage is 1.5 pu and the infinite bus voltage is
j2
1.0 + j 0.5
1 pu. The inertia constant of the generator is 5 MW-s/
MVA and the generator reactance is 1 pu. The criti-
cal clearing angle, in degrees, for a three-phase dead
The value of V1 in pu and δ2 respectively are short circuit fault at the generator terminal is [2012]
(a) 0.95 and 6.00° (b) 1.05 and -5.44° (a) 53.5 (b) 60.2
(c) 1.1 and -6.00° (d) 1.1 and -27.12° (c) 70.8 (d) 79.6
Solution: (b)
Two bus power system Solution: (d)

V1∠0° 1∠d2 Xd XT
j0.1 pu

G1 j2 P2 + jQ2 E∠d° ∼ Infinite bus ∼ V∠0°
1.0 + j 0.5 pu

E V
Pe = sin δ 0 .
P2 = 1.0 pu; Q2 = 0.5 pu ( X d + XT )
V1V2 1.5 × 1
P2 = sin(−δ 2 ). 0.5 = sin δ 0 .
X (1 + 0.5)
V π
Q2 = 2 (V1 cos δ 2 − V2 ). δ 0 = sin −1 (0.5) = 30° = .
X 6
VX
From P2 equation, 1 = 1 1 sin(−δ 2 ). Critical clearing angle
0.1
10V1 sin δ 2 = −1. (1) δ cr = cos −1[(π − 2δ 0 ) sin δ 0 − cos δ 0 ]

From Q2 equation  π 1 3
= cos −1  π −  × − 
1  3 2 2 
0.5 = [V1 cos δ 2 − 1]. = cos −1 (0.18) = 79.56°.
0.1  
0.5 = 10V1 cos δ 2 − 10.
Hence, the correct option is (d).
⇒ 10V1 cos δ 2 = 10.5 (2)
3. A 500 MW, 21 kV, 50 Hz, 3-phase, 20-pole syn-
Equation 1 add Equation 2, square on both sides chronous generator having a rated pf = 0.9, has a
(10V1 sin 2 δ 2 ) 2 + (10V1 cos δ 2 ) 2 = 12 + (10.5) 2 . moment of inertia 27.5 × 102. Then inertia constant
(H) will be [2009]
100 V12 (sin 2 δ 2 + cos 2 + δ 2 ) = 12 + (10.5) 2 .
(a) 2.44 s (b) 2.71 s
100 V12 = (1) 2 + (10.5) 2 . (c) 4.88 s (d) 5.42 s
V1 = 1.05475 pu Solution: (a)
−1 Inertia constant (H)
From1 sin δ 2 =
10V1 Kinetic energy stored in rotor (MJ)
= .
−1 MVA rating of alternatoor (S)
= = δ 2 = −5.44°.
10.5475 1 2
Kinetic energy stored in rotor = Iω .
Hence, the correct option is (b). 2

Chapter 04.indd 50 11/4/2015 5:11:54 PM


Chapter 4  Power System Stability  |  4.51

2πN s Case 1:
ω= . When only one generator is connected
60
120 × 50 Mechanical input to the generator (Pm1) = Electri-
Ns = = 3000 rpm. cal power delivered by the generator (Pe1)
2
2π × 3000 ⇒ Pm1 = Pe1 = 1 pu
ω= = 314.15 rad/sec.
  60 Case 2:
When two generator connected in parallel
1
∴ KE = × 27.5 × 103 × (314.15) 2 = 1357 MJ. Electrical power delivered by each generator
2
1357 (Pe2) = 0.5 pu
H= = 2.44 MJ/MVA.
 500  Mechanical input to each generator
 
 0.9  ( Pm 2 ) = Pe 2
Hence, the correct option is (a). Pm1
P=
m2 P=
e2 0.5 pu = .
4. A lossless single machine infinite bus power sys- 2
tem is shown below:
Assuming δ0 in each case same
1.0∠d pu 1.0∠0 pu
∼ δ 01 = δ 02 = δ 0 .
1.0 pu
So, δcr will also be same in both cases, (from Equa-
The synchronous generator transfers 1.0 per unit of tion (1))
power to the infinite bus. The critical clearing time
of circuit breaker is 0.28 s. If another identical syn- 1
tcr ∝  (δcr, H and δ0 are same).
chronous generator is connected in parallel to the Pm
existing generator and each generator is scheduled
to supply 0.5 per unit of power, then the critical Pm1
∴ tcr 2 = × tcr1 .
clearing time of the circuit breaker will [2008] Pm 2
(a) reduce to 0.14 s.
Pm1
(b) reduce but will be more than 0.14 s. tcr 2 = × 0.28
(c) remain constant at 0.28 s. Pm1
(d) increase beyond 0.28 s. 2
Solution: (d) = 2 × 0.28 = 0.396 s.
Pc
Hence, the correct option is (d).
Pmax 5. Consider a synchronous generator connected to an
A2 infinite bus by two identical parallel transmission
Pm lines. The transient reactance x′ of the generatoris
A1 0.1 pu. Due to previous disturbance, the rotor angle
d (δ ) is undergoing an undamped oscillation, with
d0 dcr dmax
the maximum value of δ (t) equal to 130°. One of
For such faults the parallel lines trip due to relay mal-operation at
δcr is given by an instant when δ (t) = 130° as shown in the figure.
The maximum value of the per unit line reactance x
δ cr = cos −1[(π − 2δ 0 ) sin δ 0 − cos δ 0 ]. (1) such that the system does not lose synchronism sub-
and tcr = critical cleaning time sequent to this tripping is [2007]
1/ 2 x′ = 0.1 pu x 1.0∠0
 2 H (δ cr − δ 0 )  (2) ∼
=  .
 π f Pm  1.0∠d x

Chapter 04.indd 51 11/4/2015 5:11:56 PM


4.52 | Power Systems

One line trips Solution: (c)


d
Before fault
130°
Mechanical input to alternator
(Ps) = Electrical output (Pe) = 1.0 pu
t
Given  δ = 30°, V = 1.0 pu
(a) 0.087 (b) 0.74 During fault
(c) 0.67 (d) 0.54
1
Solution: (c) X eq =
pu
0.8
Pe =E 1=
.1 pu , V 1.0 pu
Pe1
‘δ ’ value cannot change instantaneously.
∴  Initial accelerating power
Pe2 ( Pa ) = Ps − Pe .
Ps
1.1× 1.0
d Pa = 1.0 − sin 30°.
130°  1 
(dmax)  
 0.8 
As the alternator is already under going undamped Pa = 0.56 pu.
oscillations with δmax = 130°, at this instant if any
fault occurs even then also to maintain stability ‘δ’ Hence, the correct option is (c).
value must not go beyond 130°, for a given mechani- 7. If the initial accelerating power is X pu, the initial
cal input. acceleration in elect deg/sec, and the inertial constant
in MJ-sec/elect deg respectively will be [2006]
EV
∴ Pe 2 = Ps , sin δ = Ps . (a) 31.4 X, 18 (b) 1800 X, 0.056
X eq X X
(c) , 0.056 (d) , 18
E = 1.0∠d v = 1.0∠0°
1800 31.4
Solution: (b) GH
# ∼
Inertia constant (in MJ-s/elec deg) = M = ,
Ps = 1.0 pu 180 f
where,
Hence, the correct option is (c). G = Machine rating = 100 MVA.
Common Data for Questions 6 and 7: H = Inertia constant in MJ/MVA = 5 MJ/MVA.
 A generator feeds power to an infinite bus
through a double circuit transmission line. A3 GH 5 × 100
M = = = 0.056 MJ-s/elec deg.
phase fault occurs at the middle point of one of 180 f 180 × 50
the lines. The infinite bus voltage is 1 pu, the H 5
transient internal voltage of the generator is 1.1 M ( in pu ) = = .
180 f 180 × 50
pu and the equivalent transfer admittance dur-
ing fault is 0.8 pu The 100 MVA generator has a = acceleration in elect deg/sec.
an inertial constant of 5 MJ/MVA and it was
delivering 1.0 pu power prior tothe fault with M α = Pa .
rotor power angle of 30°. The system frequency Pa = Accelerating power = X.
is 50 Hz.
Pa X
6. The initial accelerating power (in pu) will be α= = = 1800 X elec deg/sec.
M  5 
 [2006]  × 50 
(a) 1.0 (b) 0.6  180 
(c) 0.56 (d) 0.4 Hence, the correct option is (b).

Chapter 04.indd 52 11/4/2015 5:11:58 PM


Chapter 4  Power System Stability  |  4.53

8. A generator with constant 1.0 pu terminal voltage Mechanical input (Ps1) = electrical output.
supplies power through a step-up transformer of 0.12
pu reactance and a double circuit line to an infinite ( Pe1 ) = 500 × 0.8 × 106 = 400 MW.
bus bar as shown in the figure. The infinite bus volt- During fault electrical output
age is maintained at 1.0 pu Neglecting the resistances
and susceptances of the system, the steady state sta- ( Pe2 ) = 0.6 × 400 × 106 = 240 MW.
bility power limit of the system is 6.25 pu If one of During fault mechanical input
the double-current is tripped, then resulting steady ( Ps 2 ) = 400 MW.
state stability power limit in pu will be [2005]
Accelerating power,
1 2 3
x
∼ x Pa = Ps2 − Pe2 = 400 − 240 = 160 MW.
120 × 50
(a) 12.5 (b) 3.125 pu Ns = = 1500 rpm.
4
(c) 10.0 pu (d) 5.0 pu
2π N s 2π × 1500
Solution: (d) ωs = = = 157 rad/sec.
60 60
Induced emf of the generator = |tg| = 1 pu
Accelerating torque
Terminal voltage = |Vt| = 1 pu
Reactance of transformer = Xt = 0.12 pu Pa 160 × 106
Ta = = = 1.018 MN-m.
When double-circuit line is connected ωs 157
X Hence, the correct option is (b).
Reactance of= line X= || X .
2 1 0. A generator delivers power of 1.0 pu to an infinite
Steady state stability power limit bus through a purely reactive network. The maxi-
mum power that could be delivered by the gen-
| Eg | |Vt | | x| erator is 2.0 pu. A three-phase fault occurs at the
= P1 = = .
X X terminals of the generator which reduces the gen-
Xt + 0.12 +
2 2 erator output to zero. The fault is cleared after tc
1 second. The original network is then resorted. The
6.25 = . maximum swing of the rotor angle is found to be
X
0.12 + δmax = 110 electrical degree. Then the rotor angle
2
in electrical degrees at t = tc is [2003]
X = 0.08 pu
(a) 55 (b) 70
When one of the double circuit is tripled, steady (c) 69.14 (d) 72.4
state stability power limit Solution: (c)
| Eg | |Vt | Pe
| x|
= P2 = = = 5 pu
X1 + X 0.12 + 0.08 Pmax
A2
Hence, the correct option is (d). Pm
9. A 50 Hz, 4-pole, 500 MVA, 22 kV turbo-generator A1
is delivering rated megavolt-amperes at 0.8 power d
d0 dcr dmax
factor. Suddenly losses and assume constant power
input to the shaft. The accelerating torque in the Power generated by the generator
generator in MNm at the fault will be [2004]
Pe = Pmax sinδ.
(a) 1.528 (b) 1.018
(c) 0.840 (d) 0.509 At δ0
Solution: (b) Pe= 1 pu
Before fault Pmax = 2 pu

Chapter 04.indd 53 11/4/2015 5:11:59 PM


4.54 | Power Systems

δcr = Rotor angle at t = tc. steady state power transfer limit of the transmis-
sion line is [2002]
Pe = Pmax sin δ .
(a) 9.8 pu (b) 4.9 pu
1 = 2 sin δ 0 . (c) 19.6 pu (d) 5 pu
δ 0 = 30°. Solution: (a)

•• During fault, Pe become zero and the fault is cleared |V1| |Vm| |V2|
at δ cr . Mechanical input to the generator remains
j 0.1 j 0.1
constant, Pm = 1 pu Compensation
•• Applying equal areal criterion
A1 = A2 .
δ cr
V
=1 V=
2 1 pu
  A1 = ∫ (P
δ0
m − 0)d δ = Pm (δ cr − δ 0 ) (1) =Vm 0=
.98 pu , X 0.2 pu.

δ max Steady state power transfer limit,


A2 = ∫ ( Pmax sin δ − Pm )d δ | Vm || V1 | 0.98 × 1
δ cr Pmax = = = 98 pu
( X / 2) 0.1
= Pmax (cos δ cr − cos δ max ) − Pm (δ max − δ crr ) (2)
Hence, the correct option is (a).
Equating Equations (1) and (2)
12. A power station consists of to synchronous genera-
Pm (δ cr − δ 0 ) = Pmax (cos δ cr − cos δ max ) − Pm (δ max − δ cr ). tors A and B of ratings 250 MVA and 500 MVA with
Pm inertia 1.6 pu and 1.0 pu, respectively on their own
cos δ cr = (δ max − δ 0 ) + cos δ max .
Pmax base MVA ratings. The equivalent pu inertia constant
for the system on 100 MVA common base is [1998]
1 π
cos δ cr = × (110 − 30) × + cos 110. (a) 2.6 (b) 0.615
2 180 (c) 1.625 (d) 9.0
δ cr = 69.14°.
Solution: (d)
Hence, the correct option is (c). 250
H A′ = 1.6 × = 4.0 pu
1 1. A transmission line has a total series reactance of 100
0.2 pu. Reactive power compensation is applied at 500
H B′ = 1× = 5.0 pu
the midpoint of the line and it is controlled such 100
that the midpoint voltage of the transmission line H = 4.0 + 5.0 = 9.0 pu
is always maintained at 0.98 pu. If voltage at both
ends of the linear is maintained at 1.0 pu, then the Hence, the correct option is (d).

Chapter 04.indd 54 11/4/2015 5:12:01 PM


Chapter 4  Power System Stability  |  4.55

As the common base MVA is the generator MVA


Five-marks Questions
∴ S = 1 pu

1. A synchronous generator is to be connected to an ∴  Moment of inertia,
infinite bus through a transmission line of reac- (1.0)(5)
tance X = 0.2 pu, as shown in figure the generator M =
3.14 × 50
data is as follows:
= 0.0318 MJ-sec/electrod.
X1 = 0.1 pu, E1 = 1.0 pu, H = 5 MJ/MVA, mechan-
ical power Pm = 0.0 pu, ωB = 2p × 50 rad/s. All d δ 2
∴ 0.0318 2 = 0.0 − 3.33 sin δ .
quan­tities are expressed on a common base. dt
The generator is initially running on open circuit From the solution of swing equation
with the frequency of the open circuit voltage 1/ 2
slightly higher than that of the infinite bus. If at the dδ  2 
dt  M ∫
= Pa d δ  .
dδ 
instant of switch closure δ = 0 and ω = = ωinit ,
dt dδ  2
1/ 2

dt  0.0318 ∫
compute the maximum value of ωinint  so that the = (−3.33 sin δ ) d δ  .
generator pulls into synchronism. 

 2H  = [62.9(3.33)(cos δ )] .
1/ 2
(Hint: Use the equation ∫   ω d ω + Pe d δ = 0) dt
 ωB  dδ
 [2002] ωinitial = = [209.43(cos δ )]1/ 2 .
dt
pe 50H
dδ 
Infinite bus. (ωinitial ) max =   = (209.43)
1/ 2

X′ X  d t  max
1.0∠0°
∼ E ′∠d = 14.47 rad/sec.

2. A synchronous generator is connected to an infinite


bus through a loss less double circuit transmission
Solution: line. The generator is delivering 1.0 per unit power
pe 50 Hz at a load angle of 30° when a sudden fault reduces
Infinite bus. the peak power that can be transmitted to 0.5 per
X ′ = 0.1 pu X = 0.2 pu unit. After clearance of fault, the peak power that
1.0∠0°
E ′∠d ∼ 1 pu can be transmitted becomes 1.5 per unit. Find the
critical clearing angle. [2001]
Solution:
Inertia constant H = 5 MJ/MVA. Before fault, mechanical input (Ps1) = electrical
Mechanical power (Pm) = 0.0 pu output.
Swing equation of synchronous generator is
( Pe1 ) = Pmax sin δ 0 = 1.0 pu
d 2δ
M 2 = Pacc ,
dt δ 0 = 30° = 0.5235 rad.
Given 
where Pacc = Pe .
Pmax sin 30° = 1.
EV 1.0 × 1.0
Pe = sin δ = sin δ = 3.333 sin δ . Pmax = 2 pu = Pm1.
X ( 0.1 + 0.2 )
SH (Pm = Maximum power transferred to load).
M = (MJ-Sec)/ (elect − rad ).
πf Given that during fault, Pm2 = 0.5 pu

Chapter 04.indd 55 11/4/2015 5:12:03 PM


4.56 | Power Systems

After fault, Pm3 = 1.5 pu Pa


α=
where  .
 P (δ − δ 0 ) + Pm3 cos δ max − Pm 2 ⋅ cos δ 0  M
δ C = cos −1  s max . Pa
 Pm 3 − Pm 2  ∴ ∆δ1 = ∆δ 0 + α (∆t ) 2 = ∆δ 0 + (∆t ) 2 .
M
 P  Let us consider the inertia constant of generator as
δ max = 108 − sin −1  s 
 Pm 3  H = 1.0 pu
 1  SH 1.0 × 1.0
= 108 − sin −1  ∴ M = = = 1.11× 10−4 pu
 180 f 180 × 50
 1.5 
= 138.2° = 2.41 rad. We have to calculate the accelerating power as the
average of accelerating power before and after fault.
∴ δ C = cos −1 P +P
Hence, Pa = Pa avg = a1 a 2 = 0 + P
1× (2.41 − 0.5235) + 1.5 cos(138.2°) − 0.5 cos 30°  2
 1.5 − 0.5 . 0 + Ps − Pe 2
  =
2
1.8865 + (−1.118) − 0.43  1.0 − Pm 2 sin δ 0
δ C = cos −1  . = .
 1  2

δ C = cos [ 0.335].
−1
During fault
δ C = 70.4°.
Pm 2 = 0 ⇒ Pe 2 = 0.
3. A synchronous generator, having a reactance of 1.0
∴ Pa = = 0.5 pu
0.15 pu, is connected to an infinite bus through two 2
identical parallel transmission lines having reac- 0.5
tance of 0.3 pu each. In steady state, the generator is ∴ ∆δ1 = 0 + (0.05) 2 .
1.11× 10−4
delivering 1 pu power to the infinite bus. For a three
∆δ1 = 11.26°.
phase fault at the receiving end of one line, calcu-
late the rotor angle at the end of first time step of
Rotor angle δ1 = δ 0 + ∆δ1
0.05 seconds. Assume the voltage behind transient
reactance for the generator as 1.1 pu and infinite = 15.826 + 11.26°
bus voltage as 1.0 pu. Also indicate how the accel- = 27.087°.
erating powers will be evaluated for the next time
step if the breaker clears the fault [2000] (i) If the breaker clears fault at the end of 2nd
step (or) interval then the accelerating power
(i) at the end of an interval
will be considered as the average of before and
(ii) at the middle of an interval.
­after faults.
Solution: (ii) If the breaker clears the fault at the middle of
To find the rotor angle at the end of first time step: 2nd interval then we have to consider the ac-
EV celerating power same as that of beginning of
Before fault Pm1 = that interval, i.e.,
X 1eq
Pa = Ps − Pm 2 sin δ 0 ,
1.1× 1.0
= = 3.67 pu
0.15 + (0.3 || 0.3) where Ps = mechanical input (or) shaft ­power.
4. An alternator is connected to an infinite bus as
P   1.0 
δ 0 = sin −1  S  = sin −1   = 15.826°. shown in figure. It delivers 1.0 pu current at 0.8 pf
P
 m1   3.67  lagging at V = 1.0 pu. The reactance Xd of the alter-
∆δ n = ∆δ n −1 + α ( ∆t ) ,
2 nator is 1.2 pu. Determine the active power out-
put and the steady state power limit. Keeping the

Chapter 04.indd 56 11/4/2015 5:12:06 PM


Chapter 4  Power System Stability  |  4.57

active power fixed, if the excitation is reduced, find But Pe1 = Pmax sin δ 0 .
the critical excitation corresponding to operation
0.5 Pmax = Pmax sin δ 0 .
at stability limit [1998]
  δ 0 = sin −1 0.5 = 30°.
∼ v = 1.0 ∠ 0°
Xd The load on the motor is suddenly reduced to 80%, so
E∠d
Solution: Ps2 = (0.8) × 0.5 Pmax .
E f ∠δ = V ∠0 + I ∠φ Z S ∠θ 0.4 Pmax = Pmax sin δ1 .
= 1∠0 + 1∠ − cos −1 ( 0.8 ) × 1.2∠90° δ1 = 23.578°.
= 1.969777∠29.1676° pu. From equal are criterion
δ2
Active power output
= VI cos φ ∫ P dδ = 0.
δ0
a

= 1 pu × 1 pu × 0.8 δ1 δ2

= 0.8 pu ∫ Pa dδ + ∫ Pa dδ = 0.
δ0 δ1
Steady state power limit δ1

=
|E ||V | 1.96977 × 1
= = 1.433.
∫ (P
δ0
max sin δ − 0.4 Pmax ) d δ +
X 1.2 δ2

If active power and remains constant, ∫ (P


δ1
max sin δ − 0.4 Pmax ) d δ = 0.
|E ||V | 0.4(δ 2 − δ 0 ) + cos δ 2 − cos δ 0 = 0.
= 0.8 pu
X π
0.4(δ 2 − 30) × + cos δ 2 − cos 30° = 0.
| E | ×1 pu 180
or  = 0.8 pu
1.2 pu (6.98 × 10−3 )δ 2 − 0.2094 + cos δ 2 − 0.866 = 0.
|E |= 0.96 pu By trial and error method. δ2 = 17.2°
5. A synchronous motor is receiving 50% of the So the rotor angle will be swinging between 17.2° to
power it is capable to receiving from an infinite 30° with respect to the final settling angle 23.578°.
bus. If the load on the motor is suddenly reduced 6. In a sytem, there are two generators operating in
to 80% of the previous value, swing of the motor parallel. One generator, of rating 250 MVA, has
around its new equilibrium position. [1997] an inertia-constant of 6 MJ/MVA while the other
Solution: generator of 150 MVA has an inertia-constant of
Pe 4 MJ/MVA. The inertia-constant for the combined
Pmax
system on 100 MVA common base is
MJ/MVA. [1994]
Solution:
0.5 Pmax
For 250 MVA generator
0.4 Pmax
250
H′ = × 6 = 15 MJ/MVA.
100
d
d2 d1 d0 For 150 MVA generator
Initially, mechanical input (Ps1) = 150
H ′ = 4× = 6 MJ/MVA.
Electrical output (Pe1) = 0.5 Pmax. 100
H = 15 + 6 = 21 MJ/MVA.

Chapter 04.indd 57 11/4/2015 5:12:09 PM


Chapter 5
Protection

the phase a, are given by Va volts and Ia Amperes,


One-mark Questions respectively. Then, the impedance measured by the
ground distance relay located at the terminal X of
1. A negative sequence relay is commonly used to line XY will be given by [2008]
protect [2011] Z1 Z
(a) an alternator (b) a transformer (a) Ω (b) 0

2

2

(c) a transmission line (d) a bus bar


( Z + Z1 ) Va
Solution: (a) (c) 0 Ω (d) Ω
2 Ia

Solution: (d)
X Y
Va
∼ ∼
Ia
F
Negative
R sequence
relay
Va
Impedance seen by the relay = Ω
Ia
Hence, the correct option is (d).
The negative sequence current segregating network 3. In a biased differential relay the bias is defined as
is used, the output of which is proportional to the a ratio of [2006]
generator negative phase sequence current and is fed (a) number of turns of restraining and operating coil.
into a relay with an inverse square law characteristic. (b) operating coil current and restraining coil current.
(c) fault current and operating coil current.
Hence, the correct option is (a).
(d) fault current and restraining coil current.
2. A two machine power system in shown below figure.
Solution: (a)
Transmission line XY has positive sequence imped-
ance of Z1 W and zero sequence impedance of Z1 W. I1 Protected Element I2
X Y RC
∼ ∼
F Nr Nr
2 N 2
0
An a phase to ground fault with zero fault imped- k(I1 − I2)
OC
ance occurs at the centre of the transmission line.
Bus voltage at X and line current from X to F for

Chapter 05.indd 58 11/9/2015 6:09:19 PM


Chapter 5  Protection  |  4.59

Nr a transmission system figure. The Zones 1 and


N 0 k I1 − I 2 > k I1 + I 2
2 2 setting for both the relays are indicated on the
N I +I diagram. Which of the following indicates the cor-
I1 − I 2 > r 1 2 rect time setting for the Zone 2 of relays R1 and R2
N0 2
 [2002]
where N0 = number of turns of the operating coil,
Zone 2 (R 2)
Nr = Number of turns of the restraining coil. Zone 1 (R 2)

 The relay operates when the magnitude of the


difference in the currents at the two ends of a
R1 R2
protected element exceeds a certain percentage
  Nr   Zone 1 (R 1)
   of half the magnitude of their sum.
  N0   Zone 2 (R 1)
 
 100 
Hence, the name percentage differential relay. = TZ 2 R1
(a) .6 s, TZ 2 R 2 0.3 s
0=
Hence, the correct option is (a). = TZ 2 R1
(b) .3 s, TZ 2 R 2 0.6 s
0=
4. The transmission line distance protection relay = TZ 2 R1
(c) .3 s, TZ 2 R 2 0.3 s
0=
having the property of being inherently directional = TZ 2 R1
(d) .1s, TZ 2 R 2 0.3 s
0=
is [2004] Solution: (a)
(a) Impedance relay. (b) MHO relay. From above figure, it is clear that Zone 2 of relay
(c) OHM relay. (d) Reactance relay. 1 and relay 2 are overlapped. If there is a fault in
Solution: (b) overlapped section (line 2), the fault should be
MHO relay has the property of being inherently clear by relay 2. Hence, Zone 2 operating time of
directional. relay 2 must be less than Zone 1 operating time.
Note: (TZ2R1> TZ2R2).
Q = K 3 V I cos(θ − τ ) − (− K1 ) V
2 Hence, the correct option is (a).
6. In the protection of transformers, harmonic
For relay operation
restraint is used to guard against [2001]
Q>0 (a) magnetizing in rush current.
2
K 3 V I cos(θ − τ ) − (− K1 ) V >0 (b) unbalanced operation.
(c) lightning.
V K3
= Z < cos(θ − τ ) (1) (d) switching over-voltages.
I − K1 Solution: (a)
The right-hand side of Equation (1) is a circle with When the supply switch of transformer is closed
central located on the line determined by the pa- then the magnetic inrush current having harmon-
rameter τ and passing through, the origin. ics, predominantly 2nd harmonic will flow. Due to
X this the relay will operate, through it is not a fault.
Block So, as to prevent this a harmonic restraint coil is
used in the relay.
1 K3 Hence, the correct option is (a).
Trip = Zrs
2 −K1
7. In an inverse definite minimum time, electromag-
τ = ∠Zrs netic type over-current relay, the minimum time
R feature is achieved because of [2000]
(a) saturation of the magnetic circuit.
Hence, the correct option is (b). (b) proper mechanical design.
5. Consider the problem of relay coordination for (c) appropriate time delay element.
the distance relays R1 and R2 on adjacent lines of (d) electromagnetic damping.

Chapter 05.indd 59 11/9/2015 6:09:21 PM


4.60 | Power Systems

Solution: (a) 10. If the fault current is 200 A, the relay setting is
50% and CT ratio is 400:5, the plug setting multi-
Operating
time in
plier will be [1996]
seconds (a) 25 A (b) 15 A
(c) 50 A (d) 10 A
(b)
Solution: (d)
(a)
2000
Fault current
Multiples of PSM = = 80 = 10.
plug setting Relay current settings 0.5 × 5

In curve (a) the time of operation of relay remains Hence, the correct option is (d).
same over the working range, because before the 1 1. A Buchholz relay is used for [1992]
saturation of core, the torque T = KI 2. (a) protection of a transformer against all internal
Where as during early saturation as the current I faults.
increases K decreases. Hence, the torque remains (b) protection of a transformer against external faults.
constant and hence, the time of operation. (c) protection of a transformer against both inter-
nal and external faults.
If we made the core to saturate at later time the
(d) protection of induction motors.
characteristic is as curve (b), which is the known as
inverse definite minimum time characteristics. The Solution: (a)
time current characteristics is inverse over some When a fault occurs inside the transformer tank,
range and then assumes the definite time form. gas is usually generated, slowly for an incipient
fault and violently for heavy faults. Most of the
Hence, the correct option is (a).
short circuits are developed by impulse breakdown
8. In a 3-step distance protection, the reach of the [insulation failure] between adjacent turns at the
three zones of the relay at the beginning of the end turns of the winding. The heat produced by the
first line typically extends upto [2000] high local current causes the transformer oil to de-
(a) 100% of the first line, 50% of the second line compose and produce gas which can be made use
and 20% of the third line. of to detect the winding faults.
(b) 80% of the first line, 50% of the second line
Buchholz relay is the simplest form of protection
and 20% of the third line.
which is commonly used in all transformers pro-
(c) 80% of the first line, 20% of the second line
vided with conservator. It will act (or) close the
and 10% of the third line.
breaker circuit, whenever there is sudden rise of
(d) 50% of the first line, 50% of the second line
oil pressure inside the transformer or there is more
and 20% of the third line.
gas collection. So, this device will only sense the
Solution: (b) internal fault of transformers.
Hence, the correct option is (b). Hence, the correct option is (a).
9. The Reactance relay is normally preferred for pro-
tection against [1997]
(a) Earth faults (b) Phase faults Two-marks Questions
(c) Open-circuit faults (d) None of these
1. The over current relays for the line protection
Solution: (a) and loads connected at the buses are shown in the
If the measurement of earth is taken in impedance figure.
form, it will be a variable value due to resistance
variation throughout its length and relay operation G1 A B C
will be affected with impedance measurement. This ∼
RA RB
can be over come by measuring reactance of earth.
Hence, the correct option is (a). 300 A 200 A 100 A

Chapter 05.indd 60 11/9/2015 6:09:21 PM


Chapter 5  Protection  |  4.61

The relays are IDMT is natural having the charac- 400 400
CT ratio CT ratio
5 5
teristic
(220 + j0) A (250 + j0) A
0.14 × time multiplier setting
top =
(plug setting multiplier )0.02 − 1

The maximum and minimum fault current at


bus B are 2000 A and 500 A, respectively. As-
suming the time multiplier setting and plug set- Operating coil
ting for relay RB to be 0.1 and 5 A, respectively,
the operating time of RB (in seconds) is (a) 0.1875 A (b) 0.2 A
 [2014-S1] (c) 0.375 A (d) 60 kA
Solution: (c)
Solution:
400 400
Minimum and maximum fault currents are CT ratio CT ratio
5 5
=I f min 500
= A, I f max 2000 A
(220 + j 0) A (250 + j 0) A
3.125 A
For relay RB : TMS = 0.1, plug setting = 5 A

1f (3.125 − 2.75) A
2.75 A
PSM =
I pk × CT ratio
Operating coil
At relay RB: 5 5
× 200 × 250
400 400
500
With I f min ⇒ PSM1 = = 2.5 \ Current through the operating coil = 3.125
200
×5 – 2.75 = 0.375 A
5
Hence, the correct option is (c).
2000 3. Match the items in List-I (Type of transmission
With I f max ⇒ PSM 2 = = 10 line) with the items in List-II (Type of distance
200
×5 relay preferred) and select the correct answer
5
using the codes below the lists. [2009]
Operating time with TMS = 0.1, and from IDMT
characteristics List-I List-II
A.  Short Line 1.  Ohm Relay
0.14 × 0.1 B.  Medium Line 2.  Reactance Relay
top 2 = = 0.7569 sec C.  Long Line 3.  Mho Relay
(2.5)0.02 − 1
0.14 × 0.1 (a) A → 2, B → 1, C → 3
top 2 = 0.02 = 0.2970 sec (b) A → 3, B → 2, C → 1
10 − 1
(c) A → 1, B → 2, C → 3
For maximum fault current minimum time. (d) A → 1, B → 3, C → 2
Note: GATE had given range 0.22 to 0.24.
Solution: (a)
2. Consider a stator winding of an alternator with Impedance Relay: Impedance relay is a voltage
an internal high resistance ground fault. The cur- restrained over-current relay.
rents under the fault condition are as shown in the
figure. The winding is protected using differential T = k1 I 2 − k2V 2
current scheme with current transformer of ratio Reactance Relay: Reactance relays is an over-
400 current relay with directional restraint.
A as shown. The current through the operat-
5
ing coil is [2010] T = k1 I 2 − k3VI sin θ

Chapter 05.indd 61 11/9/2015 6:09:24 PM


4.62 | Power Systems

Mho Relay: Mho relay is a voltage restrained di- (B) The under frequency relay is used at alternator so
rection relay. as to keep the frequency of voltage and current at
load side with in the permissible limits i.e., ±5%.
T = k3VI cos(θ − t ) − k2V 2
(C) The differential relay scheme is used for bus
Mho relay is inherently directional. bar protection by comparing the current of
Hence, the correct option is (a). transformer with the current of transformer
4. Voltage phasors at the two terminals of a transmis- with the currents of feeder.
sion line of length 70 km have a magnitude of 1.0 (D) The Buchholz relay is used for the protection
per unit but are 180 degrees out of phase. Assuming of transformer for internal faults.
1 Hence, the correct option is (a).
that the maximum load current in the line is th
5 6. The plug setting of a negative sequence relay is
of minimum 3-phase fault current. Which one of
the following transmission line protection schemes 0.2 A. The current transformer ratio is 5 : 1. The
will NOT pick up for this condition? [2008] minimum value of line-to-line fault current for the
operation of the relay is [2000]
(a)  Distance protection using mho relays with
zone-1 set to 80% of the line impedance. 1
(a) 1 A (b) A
(b) Directional over current protection set to pick 1.732
up at 1.25 times the maximum load current. 0.2
(c) Pilot relaying system with directional com- (c) 1.732 A (d) A
1.732
parison scheme. Solution: (c)
(d) Pilot relaying system with segregated phase The minimum value of current required for relay
comparison scheme. operation is the plug setting value of current.
Solution: (a) \ Minimum value of negative sequence current
Distance protection using mho relays with zone-1 5
required for relay operation = 0.2 × = 1 A.
set to 80% of the line impedance will not provide 1
But for a line-to-line fault I R2 = − I R1
protection.
Hence, the correct option is (a). (I f )
And fault current = 3I R2 1.732 A
=
5. A list of relays and the power system components \  Minimum fault current required = 1.732 A.
protected by the relays are given in List-I and Hence, the correct option is (c).
List-II, respectively. Choose the correct match the 7. The neutral of 10 MVA, 11 kV alternator is earthed
four choices given below: [2003] through a resistance of 5 ohms. The earth fault
relay is set to operate at 0.75 A. The CT’s have a
List-I List-II
100
A.  Distance relay 1. Transformer ratio of . What percentage of the alternator
B.  Under frequency relay 2. Turbines 5
winding is protected? [1998]
C.  Differential relay 3. Busbars
D.  Buchholz relay 4.  Shunt capacitors (a) 85% (b) 88.2%
5. Alternators (c) 15% (d) 11.8%
6.  Transmission lines Solution: (b)
Codes Percentage protected winding
A B C D
 5 × 0.75 × 1000 
(a) 6 5 3 1  
 IR  5
(b) 4 3 2 1 = 1 −  × 100% = 1 −  × 100%
 V   11 
(c) 5 2 1 6  
(d) 6 4 5 3  3 × 1000 
Solution: (a)  0.75 × 3 
= 1 −  × 100% = 88.19%
(A) The distance relay scheme is used in transmis- 11
 
sion lines only, since, the fault current travels
more distance in transmission lines. Hence, the correct option is (b).

Chapter 05.indd 62 11/9/2015 6:09:27 PM


Chapter 5  Protection  |  4.63

8. The disturbance system shown in figure is to be Solution: (b)


protected by over current system of protection. For If a fault occurs in between relays 1 and 2, the di-
proper fault discrimination directional over cur- rection of current in relay ‘2’ changes. Similarly if
rent relays will be required at locations  [1993] a fault occurs in between relays 3 and 4 the direc-
33 kV 1 2 3 4 33 kV tion of current in relay ‘3’ changes.
Supply Supply Hence, these relays ‘2’ and ‘3’ must be directional
5
in nature.
Load
Hence, the correct option is (b).
(a) 1 and 4 (b) 2 and 3
(c) 1, 4, and 5 (d) 2, 3, and 5

Chapter 05.indd 63 11/9/2015 6:09:27 PM


4.64 | Power Systems

2. The distance relay with inherent directional prop-


Five-marks Questions erty is known as relay. [1995]
Solution:
1. The per unit voltages of two synchronous machines The distance relay with inherent directional property
connected through a lossless line are 0.95∠10° is Admittance (or) mho relay.
and 1.0∠0°. Match the two sides in the following
 [1996] In this relay the operating force is directional and
opposing force is non-directional.
a. Real power of P.  Positive real power Operating force F1 = K1V1cos(φ − τ )
machine 1 Opposing force F2 = K2V2
b. Reactive power Q. Positive reactive power For the relay to operate F1 > F2
of machine 1
3. Type of Relay most suited for [1995]
c. Power factor R.  Negative real power
A.  Buchholz relay. P. Feeder.
of machine 1
B.  Translay relay. Q. Transformer.
S. Negative reactive power
C. Carrier current, phase R.  Radial distributed.
T.  Leading power factor
comparison relay.
U. Lagging power factor
D. Directional over S. Generator.
Solution: current relay.
(a) Circuit breaker is an equipment which consists E. Negative sequence T. Ring main
of current carrying contacts and electrodes. relay. distributor.
Whenever the relay acts due to over current Solution:
immediately the breaker tripcoil is energized (a) Buchholz relay is used to protect transformer
and so it trips to avoid over current due to any for internal faults.
fault. (b) Translay relay is a voltage balance differential
(b) When the switch is closed for a given trans- scheme. It is suitable for pilot circuits up to
mission line, then the instantaneous voltages a loop resistance of 800 ohms. It is used for
which are expressed interns of peak values feeder protection.
travel over transmission line and should not (c) Pilot wires are economical for a length of
enter transformer. Hence at the end of trans- around 10 km. So, the percentage differential
mission line before transformer lightning ar- relay for transmission line can be employed
restor is provided, which is made of non-linear with carrier current pilot wires. It is a voltage
resistor. balance scheme.
[Example: Zinc oxide] (d) Directional over current relay is used in inter
connected system and also in ring main distri-
(c) Governor is used to control the steam input
bution system.
to the prime mover of generator, by which the
(e) Whenever the alternator supplies to unbalanced
power output of generator is controlled.
load then negative sequence currents will flow
(d) By controlling excitation, generated voltage can from load to generator. In order to protect alter-
be controlled and hence the terminal voltage. nator a negative sequence relay is employed.

Chapter 05.indd 64 11/9/2015 6:09:27 PM


Chapter 6
Circuit Breaker

3. Three sections of a feeder are provided with circuit


One-mark Questions breakers CB1, CB2, CB3, CB4, CB5 and CB6. For a
fault F as indicated in figure. [1999]
1. Keeping in view the cost and overall effectiveness,
the following circuit breaker is best suited for F
capacitor bank switching [2006] CB1 CB2 CB3 CB4 CB5 CB6
(a) Vacuum (b) Air blast
(c) SF6 (d) Oil (a) CB5 must be set to trip after CB1 trips.
(b) CB5 must be set to trip after CB3 and CB4 trip.
Solution: (a)
(c) CB5 must be set to trip after CB2 trips.
The arc time constant is the least (a few us) in vac-
(d) CB5 must be set to trip before CB1, CB2, CB3
uum circuit breakers as compared to other breaker
and CB4 trip.
types. The rapid building up of dielectric strength
after final arc extinction (20 kV/µs) is unique fea- Solution: (a)
tures of VCBs. These arc, therefore, ideally suited The vacuum circuit breakers are specifically
for capacitor switching. used for low cost switch having low fault inter-
rupting capacity, but capable of large number of
Hence, the correct option is (a).
load switching operations without maintenance
2. The interrupting time of a circuit breaker is the and in some applications capable of interrupt-
period between the instant of [2003] ing line charging (or) capacitor current without
(a) initiation of short circuit and the arc extinction restrike.
on an opening operation.
Hence, the correct option is (a).
(b) energizing of the trip circuit and the arc extinc-
tion on an opening operation. 4. The use high-speed circuit-breakers [1997]
(c) initiation of short circuit and the parting of pri- (a) reduces the short circuit current
mary arc contacts. (b) improves system stability
(d) energizing of the trip circuit and the parting of (c) decreases system stability
primary arc contacts. (d) increases the short circuit current
Solution: (b) Solution: (d)
Note: When a fault occurs in the protected circuit, As the fault is in the middle of circuit breakers CB5
the relay connected to the CT actuates and closes its and CB6. Either CB5 (or) CB6 (or) both must trip
contact. Current flows from the battery in the trip first, before other C.Bs trip.
circuit. As the trip coil of the breaker is energized Hence, the correct option is (d).
the circuit breaker operating mechanism is actuated. 5. Resistance switching is normally employed in
Hence, the correct option is (b).  [1996]

Chapter 06.indd 65 11/10/2015 5:40:00 PM


4.66 | Power Systems

(a) allbreakers 2. Consider the protection system shown in the fol-


(b) bulkoil breakers lowing figure. The circuit breakers numbered from
(c) minimum oil breakers 1 to 7 are of identical type. A single line to ground
(d) airblast circuit breakers fault with zero fault impedance occurs at the mid-
Solution: (b) point of the line (at point F), but circuit breaker 4
The high-speed circuit breakers are used to im- fails to operate (‘Stuck breaker’). If the relays are
prove transient stability. coordinated correctly, a valid sequence of circuit
breaker operation is [2007]
Hence, the correct option is (b).
1 Transmission 3 Bus C
Line
Two-marks Questions Struck
4 6
2 breaker
1. A three-phase, 33 kV oil current breaker is rated
1200 A, 2000 MVA, 3 s. The symmetrical breaking Bus A
5 7
current is [2010] Transmission
(a) 1200 A (b) 3600 A Bus B Line
(c) 35 kA (d) 104.8 kA
(a) 1, 2, 6, 7, 3, 5 (b) 1, 2, 5, 5, 7, 3
Solution: (c) (c) 5, 6, 7, 3, 1, 2 (d) 5, 1, 2, 3, 6, 7
Rated MVA of circuit breaker = 200 MVA 3
Solution: (c)
× |V(line)|rated × Symmetrical breaking current
The relays are coordinated in such a manner that
= Rated MVA of CB
the relay nearer to the faulty point will trip the cor-
200 responding circuit breaker first and the others act
Symmetrical breaking current = = 35 kA.
3 × 33 as back up protection.
Hence, the correct option is (c). Hence, the correct option is (c).

Chapter 06.indd 66 11/10/2015 5:40:00 PM


Chapter 6  Circuit Breaker  |  4.67

Five-marks Question Solution:

1. The inductance and capacitance of a power system 1 L 1 1


RP = = = 5 kΩ.
network up to a circuit breaker location are 1 H 2 C 2 0.01 × 10−6
and 0.01 µF, respectively, the value of the shunt
resistor across the circuit breaker, required for crit-
ical damping of the restriking voltage is
 [1995]

Chapter 06.indd 67 11/10/2015 5:40:00 PM


Chapter 7
Generating Stations

maintains its speed constant at the rated value,


One-mark Questions while the other generators (G2 and G3) have gover-
nors with a droop of 5%. If the load of the system
1. In thermal power plants, the pressure in the work- is increased, then in steady state. [2002]
ing fluid cycle is developed by [2004] (a) Generation of G2 and G3 is increased equally
(a) condenser (b) superheater while generation of G1 is unchanged
(c) feedwater pump (d) turbine (b) Generation of G1 alone is increased while gen-
Solution: (c) eration of G2 and G3 is unchanged
Note: (c) Generation of G1, G2 and G3 is increased
Condenser: The functions of condenser are equally
(d) Generation of G1, G2 and G3 is increased in the
(i) to provide vacuum at outlet of steam turbine.
ratio 0.5 : 25 : 0.25.
(ii) to condense the steam and pass on the conden-
sate to boiler feed. Solution: (a)
Generators with more speed regulation will share
The super-heater is a device used to convert satu-
more load.
rated steam or wet steam into dry steam used for
As G2 and G3 have same regulation they share load
power generation process.
equally and G1 will not share any load as its speed
Hence, the correct option is (c). regulation is 0.
2. For harnessing low variable water heads, the suit- Hence, the correct option is (a).
able hydraulic turbine with high percentage of
4. In a thermal power plant, the feed water coming to
reaction and runner adjustable vanes is [2004]
the economizer is heated using [2000]
(a) Kaplan (b) Francis
(a) HP steam
(c) Pelton (d) Impeller
(b) LP steam
Solution: (a) (c) Direct heat in the furnace
Kaplan turbine is having adjustable vanes and is (d) Flue gases
used for low variable heads of (2–20 m) and is re- Solution: (d)
action type turbine. Hence, the correct option is (d).
Hence, the correct option is (a). 5. Which material is used in controlling chain reac-
3. Consider a power system with three identical tion in a nuclear reactor? [1996]
generators. The transmission losses are negligible. (a) Thorium (b) Heavy water
One generator (G1) has a speed governor, which (c) Boron (d) Beryllium

Chapter 07.indd 68 11/4/2015 2:37:58 PM


Chapter 7  Generating Stations  |  4.69

Solution: (c) 2. A power system has two synchronous generators.


Boron and cadmium rods are used as control rods. The Governor-turbine characteristics correspond-
Hence, the correct option is (c). ing to the generators are
6. Out of the following plant categories P1 = 50(50 − f ), P2 = 100(51 − f ),
(a) Nuclear (b) Run-of-river
where f denotes the system frequency in Hz, and
(c) Pumped-storage (d) Diesel
P1 and P2 are, respectively, the power outputs (in
Solution: (c) MW) of turbines 1 and 2. Assuming the genera-
The pumped-storage plants supply power-during tors and transmission network to be lossless, the
peak loads. system frequency for a total load of 400 MW is
Note: Base load-plant  [2001]
(i) Low operating cost (a) 47.5 Hz (b) 48.0 Hz
(ii) Capability of working continuously long period. (c) 48.5 Hz (d) 49.0 H
Hence, the correct option is (c). Solution: (b)
P1 = 50 (50 - f  ) MW (1)
Two-marks Questions P2 = 100(51 - f  ) MW (2)
Also P1 + P2 = 400 MW
1. A hydraulic turbine having rated speed of 250 rpm From Equations (1) and (2)
is connected to a synchronous generator. In order
to produce power at 50 Hz, the numbers of poles 50(50 − f ) + 100(51 − f ) = 400
required in the generator are [2004] (50 − f ) + 2(51 − f ) = 8
(a) 6 (b) 12 3 f = 152 − 8 = 144
(c) 16 (d) 24 144
Solution: (d) f = = 48Hz
3
120 f 120 f Hence, the correct option is (b).
Ns = ⇒ P=
P Nf
120 × 50
P=
250
P = 24
Hence, the correct option is (d).

Chapter 07.indd 69 11/4/2015 2:37:58 PM


Chapter 8
Load Flows

y′12 y′31
One-mark Questions Y11 = + + y12 + y31 = − j 13
2 2
1. The bus admittance matrix of a three-bus three- y12 = − y12 = j 10
 −13 10 5 y23 = − y23 = j 10

line system is y = j  10 −18 10  . y31 = − y31 = j 5
 5 10 −13
′ = 2 [ − j 13 + j 10 + j 5] = j 4.
∴ y′12 + y31
If each transmission line between the two buses is
represented by an equivalent p- network, the mag- Similarly,
nitude of the shunt susceptance of the line con-
necting bus 1 and 2 is [2012] y′12 + y′23 = 2[− j 18 + j 10 + j 10] = j 4
(a) 4 (b) 2 y′23 + y′31 = 2[− j 13 + j 10 + j 5] = j 4
(c) 1 (d) 0
⇒ y′12 = j 6 − j 4 = j 2
Solution: (b)
yik = Series admittance of the line connecting buses Hence, the correct option is (b).
i and k. 2. A power system consists of 300 buses out of which
y′ik 20 buses are generator bus, 25 buses are ones with
= Half line charging admittance in bus admit-
2 reactive power support and 15 buses are the ones
tance matrix. with fixed shunt capacitors. All the other buses are
load buses. It is proposed to perform a load flow
1 analysis in the system using Newton–Raphson

y31 ′
y12 method. The size of the Newton–Raphson Jacobian
2 2 matrix is [2003]
(a) 553 × 553 (b) 540 × 540
y 31 y 12

y31 ′
y12 (c) 555 × 555 (d) 554 × 554
2 2
Solution: (b)
y 23 From the given 300 Buses
3 2
•• 20 buses are generator buses. For these generator

y23 ′
y23 buses the specified quantity magnitude of volt-
2 2 age is only used in Jacobian Matrix.
∴  Number of equations = 20 × 1 = 20.

Chapter 08.indd 70 11/12/2015 10:17:40 AM


Chapter 8  Load Flows  |  4.71

•• 25 buses are reactive power support buses and 15


buses are with fixed shunt capacitance. For there Two-marks Questions
buses [load buses] reactive power consumption
is assumed as zero. 1. For a power system network with n nodes, Z33 of
∴ Only specified value used in Jacobian Matrix its bus impedance matrix is j 0.5 per unit. The volt-
is P. age at node 3 is 1.3∠-10° per unit. If a capacitor
having reactance of - j 3.5 per unit is now added
∴  Number of equations = (25 + 15) × 1 = 40.
to the network between node 3 and the reference
Remaining buses are load buses. node, the current drawn by the capacitor per unit is
∴  Load buses = 300 - [20 + 15 + 25] = 240.  [2013]
For load buses the specified values are P and Q. (a) 0.325∠-100° (b) 0.325∠-80°
∴  Number of equations = 240 × 2 = 480. (c) 0.371∠-100° (d) 0.433∠-80°
Total number of equations = 20 + 40 + 480 = 540. Solution: (d)
Size of Jacobian Matrix = 540 × 540. Z 33 = j 0.5 pu → Thevenin’s impedance wrt bus (3).
Hence, the correct option is (b). V3 = 1.3 ∠ − 10° → Thevenin’s voltage at bus.
3. If the reference bus is changed in two load flow A capacitor of reactance - j 3.5 pu connected be-
runs with same system data and power obtained tween node (3) and reference.
for reference bus taken as specified P and Q in the j 0.5
latter run [1996]
(a) the system losses will be unchanged but com- Z 33 IC
−j 3.5
V3
plex bus voltages will change.
(b) the system losses will change but complex bus
voltages remain unchanged.
(c) the system losses as well as complex bus volt- V3
IC =
age will change. Z 33 − j 3.5
(d) the system losses as well as complex bus volt-
1.3 ∠ − 10
age will be unchanged. = = 0.433 ∠80° pu
j 0.5 − j 3.5
Solution: (a)
The system losses are a function of system proper- Hence, the correct option is (d).
ties which are independent of choosing a type of
bus whereas the complex bus voltage is a function Common Data for Questions 2 and 3:
of parameters which are being calculated. In the following network, the voltage magnitudes
Hence, the correct option is (a). at all buses are equal to 1 pu, the voltage phase
angles are very small, and the line resistances are
4. In load flow analysis, the load connected at a bus is negligible. All the line reactances are equal to j1Ω.
represented as [1993] Bus 1 (slack) j1 Ω Bus 2
(a) constant current drawn from the bus
∼ ∼
(b) constant impedance connected at the bus
P 2 = 0.1 pu
(c) voltage and frequency dependent source at the
bus
(d) constant real and reactive power drawn from j1 Ω j1 Ω
the bus.
Solution: (d) Bus 3
P 3 = 0.2 pu
The bus to which load is connected is known as
a load bus. For load bus real (P) and reactive (Q) 2. The voltage phase angles in rad at buses 2 and 3 are
powers are specified, voltage magnitude (V) and its  [2013]
phase angle d are to be found. (a) θ 2 = −0.1, θ3 = −0.2 (b) θ 2 = 0, θ3 = −0.1
Hence, the correct option is (d). (c) θ 2 = 0.1, θ3 = 0.1 (d) θ 2 = 0.1, θ3 = 0.2

Chapter 08.indd 71 11/12/2015 10:17:41 AM


4.72 | Power Systems

Solution: (b)  0.3 −0.2 0


P1 + P2 + P3 = 0. (a) j  −0.2 0.12 0.08 
P1 + j 0.1 − j 0.2 = 0.  0 0.08 0.02 
P1 = j 0.1 pu
 −15 5 0
No power flow in between bus 1 and 2. (b) 
j  5 7.5 −12.5 
P12 ∝ sin(θ1 − θ 2 ) = 0.  0 12.5 2.5
θ1 − θ 2 = 0.
 0.1 0.2 0

j 0.2 0.12 −0.08 
As bus 1 is a slack bus (c)
θ1 = 0.  0 −0.08 0.10 
⇒ θ 2 = 0.
 −10 5 0
V2V3
Now P23 = sin(θ 2 − θ3 ) (d) 
j  5 7.5 12.5
X
 0 12.5 −10 
1× 1
or 0.1 = sin(θ 2 − θ3 )
1 Solution: (b)
⇒ θ 2 − θ3 = sin −1 (0.1) 1 2 3
= 0.10016  0.1
j0.2 −j0.08
θ3 = −0.1
j0.1 j0.1
Hence, the correct option is (b).
3. If the base impedance and the line-to-line base
voltage are 100 Ω and 100 kV, respectively, then
Z10 = j 0.1
the real power in MW delivered by the generator
connected at the slack bus is [2013] 1 1
⇒ y10 = = = − j10 pu
(a) −10 (b) 0 Z10 j 0.1
(c) 10 (d) 20
Z12 = j 0.2
Solution: (c)
P1 = 0.1 pu 1 1
⇒ y12 = = = − j 5 puu
Z12 j 0.2
(100 kV) 2
= 0.1× = 10 MW.
100 Z 23 = − j 0.8
Note: IIT Bombay has given marks to all students
1 1
in questions 9.21 and 9.22. ⇒ Y23 = = = j 12.5 pu
Z 23 − j 0.08
Hence, the correct option is (c).
4. A three-bus network is shown in the figure below   Z 30 = j 0.1.
indicating the pu impedance of each element
1 1
 [2011] ⇒ y30 = = = − j10 pu
Z 30 j 0.1
1 2 3
y11 = y10 + y12 = − j10 − j 5 = − j15 pu
j0.2 y12 = y21 = − y12 = −(− j 5) = j 5 pu
−j 0.08
j 0.1 j 0.1
Y31 = Y13 = − y13 = 0
Y22 = Y12 + Y23 = − j 5 + j12.5 = j.75 pu
The bus admittance matrix, Y-bus, of the network
is Y33 = Y30 + Y23 = − j10 + j12.5 = j 2.5 pu

Chapter 08.indd 72 11/12/2015 10:17:44 AM


Chapter 8  Load Flows  |  4.73

 −15 5 0   − j 8.75 j 1.25 j 2.50 


Ybus = j  5 7.5 −12.5 . Ybus =  j 1.25 − j 6.25 j 2.50 
 0 −12.5 2.5   j 2.50 − j 2.50 − j 5.00 
 j 0.16 j 0.08 j 0.12 
Z bus =  j 0.08 j 0.16 
Hence, the correct option is (b).
j 0.24
5. For the Y-bus matrix of a 4-bus system given in per
unit, the buses having shunt elements are [2009]  j 0.12 j 0.16 j 0.34 
The pre-fault voltage are 1.0 pu at all the buses.
The system was unloaded prior to the fault. A solid
 −5 2 2.5 0  3 phase fault takes place at bus 2.
 2 −10 2.5 4 
7. The post fault voltages at buses 1 and 3 in per unit
Ybus = j 
 2.5 2.5 −9 4  respectively are [2006]
  (a) 0.24, 0.63 (b) 0.31, 0.76
 0 4 4 −8
(c) 0.33, 0.67 (d) 0.67, 0.33
Solution: (d)
(a) 3 and 4 (b) 2 and 3
(c) 1 and 2 (d) 1, 2 and 4 V20
If = .
Solution: (c) Z 22
If sum of the elements in the row are zero, there is 1.0
V1′ = V10 − I f Z12 = 1.0 − × 0.08 = 0.64 pu
no shunt element. If sum of the elements are not 0.24
zero, then there is shunt element, i.e., at bus 1, bus 1.0
2 there are shunt elements. V3′ = V30 − I f Z 23 = 1.0 − × 0.16 = 0.33 pu
0.24
Hence, the correct option is (c). Hence, the correct option is (d).
6. The Gauss–Seidel load flow method has follow-
8. The per unit fault feeds from generators connected
ing disadvantages. Tick the incorrect statement.
to buses 1 and 2 respectively are [2006]
 [2006]
(a) 1.20, 2.51 (b) 1.55, 2.61
(a) Unreliable convergence
(c) 1.66, 2.50 (d) 5.00, 2.50
(b) Slow convergence
(c) Choice of slack bus affects convergence Solution: (c)
(d) A good initial guess for voltages is essential Yij = Sum of the admittance directly connected to
for convergence ith bus.
Solution: (a)
YiL = YiO + Yi1 …Yin (Yij excluded )
1. The time taken to perform one iteration of the
computation is relatively smaller in case of G–S Yik = −Yik
method but the number of iterations required
by G-S method for a particular system and they So, Y11 = Y10 + Y12 + Y13
increase with the increase in the size of the sys- Y12 = −Y12
tem which results in slow convergence.
Y13 = −Y13
2. The convergence characteristics of G–S method
is sometimes very seriously affected by the Y10 = Y11 + Y12 + Y13
selection of a slack bus and the selection of a = − j8.75 + j1.25 + j 2.5
particular bus may result in poor convergence.  ⇒ Y10 = − j 5 puu
Hence, the correct option is (a). Similarly,
Common Data for Questions 7 and 8:
Y20 = Y22 + Y21 + Y23
For a power system the admittance and impedance
matrices for the fault studies are as follows. Y20 = − j 6.25 + j1.25 + j 2.5

Chapter 08.indd 73 11/12/2015 10:17:46 AM


4.74 | Power Systems

⇒ Y20 = − j 2.5 pu 10. The bus impedance matrix of a 4-bus power sys-
tem is given by
1 1
Z10 = = = j 0.2
y10 − j 5  j 0.3435 j 0.2860 j 0.2723 j 0.277 
1 1  j 0.2860 j 0.3408 j 0.2586 j 0.2414 
Z 20 = = = j 0.4 Z bus = A.
y20 − j 2.5  j 0.2723 j 0.2586 j 0.2791 j 0.2209 
 
EGi − Vi t  j 0.2277 j 0.2414 j 0.2209 j 0.2791
I Gif =
Z i0
Branch having an impedance of j 0.2 Ω is connected
[Current fed by generator I during fault] between bus 2 and the reference. Then the value of
EG1 = EG 2 = 1 ∠0° [The system was unloaded]. Z22, new and Z23, new of the bus impedance matrix of
the modified network are respectively [2003]
Current fed by generator 1 (a) j0.5408 Ω and j0.4586 Ω
EG1 − V1 f 1∠0° − 0.67∠0° (b) j0.1260 Ω and j0.0956 Ω
I Gf1 = = (c) j0.5408 Ω and j0.0956 Ω
Z10 j 0.2 (d) j0.1260 Ω and j0.1630 Ω
= 1.65 ∠ − 90° pu Solution: (b)
Current fed by generator 2,  Z1 j 
 
EG 2 − V2f 1∠0° − 0 Z B (new ) = Z B (old ) −
1  .  [ Z  Z ].
I Gf 2 = =
Z jj + Z b  . 
j1 jn
Z 20 j 0.4  
= 2.5 ∠ − 90° pu  Z nj 

Hence, the correct option is (c). New elements (Zb) is connected between jth and
9. The network shown in the given figure has imped- reference bus.
ance in pu as indicated. The diagonal element Y22 Here, j = 2, b = 4.
of the bus admittance matrix YBUS of the network is
 [2005]  Z12 
1 2 3
 
1  Z 22 
+j 0.1 +j 0.1 ∴   [ Z 21 Z 22 Z 23 Z 24 ]
Z ij + Z b  Z 32 
 
−j 20.0 −j 20.0 −j 10.0  Z 42 
 j 0.2860 
 
(a) -j19.8 (b)
+ j20.0 1  j 0.3408 
=  
(c) + j0.2 (d)
-j19.95 ( j 0.3408 + j 0.2)  j 0.2586 
Solution: (d)  
 j 0.2414 
Y22 = y21 + y22 + y23
[ j 0.2860 j 0.3408 j 0.2586 j 0.2414].
1
y22 = = j 0.05, We are required only changes in Z22Z23.
− j 20
1 − − − −
y21 = = − j10 = y23 −
j 0.1  j 0.2147 j 0.16296 − 
.
Y22 = j 0.05 − j10 − j10 = − j19.95 MHO − − − −
 
Hence, the correct option is (d). − − − −

Chapter 08.indd 74 11/12/2015 10:17:48 AM


Chapter 8  Load Flows  |  4.75

Z 22 ( new ) = Z 22 ( old ) − j 0.2147 (a) the load flow will converge only if the slack
bus is specified in Area 1.
= j 0.1260. (b) the load flow will converge only if the slack
Z 23( new ) = Z 23( old ) − j 0.16296 bus is specified in Area 2.
= j 0.2586 − j 0.16296 (c) the load flow will converge if the slack bus is
specified in either Area 1 or Area 2.
= j 0.0956 Ω.
(d) the load flow will not converge if only one slack
Hence, the correct option is (b). bus is specified. The line Area 1 or Area 2.
1 1. A power system consists of 2 areas (Area 1 and Solution: (d)
Area 2) connected by a single tie-line (figure). It is The connectivity date is not specified. Hence, the
required to carry out a load flow study on this sys- two areas are isolated areas.
tem. While entering the network data, the tie-line Hence, two black buses are required to set the con-
data (connectivity and parameters) is inadvertently vergent criteria.
left out. If the load flow program is run with this Hence, the correct option is (d).
incomplete data, [2002]

Chapter 08.indd 75 11/12/2015 10:17:48 AM


4.76 | Power Systems

Solution:
Five-marks Questions
1. Two transposed 3 phase lines run parallel to each j 0.67 j 0.67
other. The equation describing the voltage drop in Bus 1 j 0.5 Bus 2
both lines given below.
j 0.4
j 0.4 j 0.25
 ∆Va1  0.15 0.05 0.05 0.04 0.04 0.04   I a1 
  0.05  
 ∆Vb1   0.15 0.05 0.04 0.04 0.04   I b1 
 ∆Vc1  Bus 3 Bus 4
0.05 0.05 0.15 0.04 0.04 0.04   I c1  j 0.25
 = j   .
 ∆Va 2  0.04 0.04 0.04 0.15 0.05 0.05  I a 2 
 ∆V  0.04 0.04 0.04 0.05 0.15 0.05  I b 2  Y11 = Y11 + Y12 + Y13 + Y14 = − j 6 (1)
 b2    
 ∆Vc 2  0.04 0.04 0.04 0.05 0.05 0.15  I c 2  Y22 = Y21 + Y22 + Y23 + Y24 = − j10 (2)
Y33 = Y31 + Y32 + Y33 + Y34 = − j 9 (3)
Compute the self and mutual zero sequence im-
pedances of this system, i.e., compute Z011, Z012, Y44 = Y41 + Y42 + Y43 + Y44 = − j 8 (4)
Z021, Z022 in the following equations
From the given YBUS matrix
∆V01 = Z 011 I 01 + Z 012 I 02 y12 = − j 2, y13 = − j 2.5, y14 = 0 (5)
∆V02 = Z 0 21 I 01 + Z 0 22 I 02 , y21 = − j 2, y23 = − j 2.5, y24 = − j 4 (6)
y31 = − j 2.5, y32 = − j 2.5, y34 = − j 4 (7)
where DV01, DV02, DV01, DI02, are the zero sequence
voltage drops and currents for the two lines re- y41 = 0, y42 = − j 4, y43 = − j 4 (8)
spectively. [2002]
From (1) and (5)
Solution:
y11 − j 2 − j 2.5 + 0 = − j 6.
Z s = j 0.15. y11 = − j 1.5 ⇒ X 11 = j 0.67.
Z m = j (0.05 + 0.05 + 0.04 + 0.04 + 0.04)
From (2) and (6)
= j 0.22. − j 2 + y22 − j 2.5 − j 4 = − j 10.
Z eq = Z s + 2 Z m y22 = − j 1.5 ⇒ X 22 = j 0.67.
= j 59. From (3) and (7)
Z 012 = Z 021 = 2 Z m = j 0.44.
− j 2.5 − j 2.5 + y33 − j 4 = − j 9.
y33 = 0 ⇒ X 33 = ∞.
2. For the Y-bus matrix given in per unit values,
where the first, second, third, and fourth row refers From (4) and (8)
to bus 1, 2, 3 and 4 respectively, draw the reactance
0 − j 4 − j 4 + y44 = − j8.
diagram. [2001]
y44 = 0 ⇒ X 44 = ∞.
 −6 2 2.5 0 
 2 −10 2.5 4  The reactance diagram is
Ybus = j  3. In load-flow analysis, a voltage-controlled bus is
 2.5 2.5 −9 4 
  treated as a load bus in subsequent iteration for a
 0 4 4 −8 reactive power limit is violated. [1995]

Chapter 08.indd 76 11/12/2015 10:17:51 AM


Chapter 8  Load Flows  |  4.77

Solution: In load flow studies, each bus (or) node in a ­power


The given statement is true. system is associated with four quantities, real
4. In load flow studies of a power system, the quan- ­power and reactive power, bus voltage magnitude
tities specified at a voltage-controlled bus are and its phase angle. In load flow solution two out
and  [1992] of the four quantities are specified and remaining
two are required to be obtained through the load
Solution:
flow solution. In case of voltage controlled bus,
In load flow studies of a power system, the quanti-
real power and voltage are specified, reactive pow-
ties specified at a voltage controlled bus (generator
er and phase angle of voltage are to be obtained
bus) are the voltage magnitude corresponding to
through load flow solution.
the generation voltage (V) and real power (P).

Chapter 08.indd 77 11/12/2015 10:17:51 AM


Chapter 9
HVDC
(a) Both regions need not have the same frequency.
One-mark Questions (b) The total power flow between the regions
(PAC + PDC) can be changed by controlling the
1. Power is transferred from System A to System B HVDC converters alone.
by an HVDC link as shown in the figure. If the (c) The power sharing between the AC line and
voltage VAB and VCD are as indicated in the figure, the HVDC link can be changed by controlling
and 1 > 0, then [2010] the HVDC converters alone.
Power Flow (d) The directions of power flow in the HVDC
A C link (PDC) cannot be reversed.
1 AC
Solution: (c)
AC
VCD Both regions are connected by HVDC link as well as
System A System B
AC line. So, AC link is possible when both regions
B D have same frequency. By changing fringe angle
Rectifi
(α)of converter, we can change the power sharing,
(a) VAB < 0, VCD < 0, VAB > VCD between the AC line and HVDC link.
(b) VAB > 0, VCD > 0, VAB < VCD Hence, the correct option is (c).
(c) VAB > 0, VCD > 0, VAB > VCD 3. An HVDC link consists of rectifier, inverter trans-
(d) VAB > 0, VCD < 0, VAB > VCD mission line and other equipment. Which on the
Solution: (c) following is true for this link? [2006]
Power is always transferred from high voltage sys- (a) The transmission line produces/supplies reac-
tem to low voltage system. tive power.
Hence, the correct option is (c). (b) The rectifier consumes reactive power and the
inverter supplies reactive power from/to the re-
2. Two regional systems, each having several syn- spective connected consumes.
chronous generators and loads are interconnected (c) Rectifier supplies reactive power and the in-
by an AC line and a HVDC link as shown in the verted consumes reactive power to/from the
figure, which of the following statements is true in respective connected AC systems.
the steady state: [2007] (d)  Both the converters (rectifier and inverter)
PDC consume reactive power from the respective
connected AC systems.
Solution: (b)
Region 1
HVDC link
Region 2 Rectifier acts as an inductor
AC line [For α < 90º]
PDC And inverter acts as a capacitor

Chapter 09.indd 78 11/4/2015 3:04:54 PM


Chapter 9  HVDC  |  4.79

[For α > 90º] (c) minimum line power losses.


Hence, the correct option is (b). (d) simple protection.
4. The high voltage DC (HVDC) transmission is Solution: (c)
mainly used for [2005] Hence, the correct option is (c).
(a) bulkpower transmission over long distances. 7. HVDC transmission is preferred to EHV–AC
(b) inter-connecting two systems with the same because [1994]
nominal frequency. (a) HVDC terminal equipment are inexpensive.
(c) eliminating reactive power requirement in the (b) VAR compensation is not required in HVDC
operation. systems.
(d) minimizing harmonics at the converter stations. (c) Systems stability can be improved.
Solution: (a) (d) Harmonics-problem is avoided.
Hence, the correct option is (a). Solution: (c)
5. Choose the appropriate auxiliary components of Electrical power supplies by an alternator is
HVDC transmission system from the following:
 [2003] EV
Pe = sin δ
1. DC line inductor X
2. AC line inductor
For a given power transfer if the value of reactance
3. Reactive power sources
is reduced then we can operate the generator at
4. Distance relays on DC line
lower values of δ. Hence during fault conditions
5. Series capacitance on AC line
(or) sudden variation in load the maximum swing
(a) 1 and 2 (b) 1 and 3
of rotor will not throw it out of synchronism and
(c) 2 and 4 (d) 4 and 5
hence the stability is improved.
Solution: (b)
In case of HVDC system, the reactance offered by
Hence, the correct option is (b).
the line is zero and hence the stability is improved.
6. Bulk power transmission over long HVDC lines
Note: In HVDC system reactive power compensa-
are preferred, on account of [1998]
tion is required at converter stations.
(a) lowcost of HVDC terminals.
(b) noharmonic problems. Hence, the correct option is (c).

Chapter 09.indd 79 11/4/2015 3:04:55 PM


Chapter 10
Per Unit System

(b) j1.0 j1.0


One-mark Question
1 3 2
j0.25 j0.10
1. The pu parameters for a 500 MVA machine on its
own base are:
∼ ∼
Inertia, M = 20 pu; reactance, X = 2 pu
The pu values of inertia and reactance on 100 MVA
(c) j2.25 j2.25
common base, respectively, are [2005]
(a) 4, 0.4 (b) 100, 10 1 3 2
(c) 4, 10 (d) 100, 0.14 j0.10 j0.10

Solution: (a)
∼ ∼
MVA new 100
X new = X old × = 2× = 0.4 pu
MVA old 500 Solution: (a)
M ∝S New MVA Base = (MVA B ) new = 100 MVA
M 2 S2 100 2
= ⇒ M2 = × 20 = 4 pu (MVA B ) new (kVB )old
M 1 S1 500 Z pu(new) = Z pu(old) × × 2
(MVA B )old (kVB ) new
Hence, the correct option is (a). (kVB )old = (kVB ) new = 15 kV
On 100 MVA and 15 kV base,
Two-marks Questions XG2 = 0.1 pu (remains same)
1. For the above system, the positive sequence dia- 100
X G1 = 0.25 × = 0.1 pu
gram with the pu values on the 10 MVA common 250
 [2011] Positive sequence reactance of L1 and L2
(a) j1.0 j1.0
= X L = 0.225 × 10 = 2.25 Ω
1 3 2
j0.10 j0.10 (kVB ) 2new
= Z=
Base impedance B
(MVA B ) new
∼ ∼
152
= = 2.25 Ω
100

Chapter 10.indd 80 11/10/2015 6:07:56 PM


Chapter 10  Per Unit System  |  4.81

2.25 When the generator is connected with open-circuit


X L (in =
 pu ) = 1 pu {pu reactance of section transmission line, line draws charging current.
2.25
L1 and L2}
IC
Hence, the correct option is (a). Xs
2. In the above system the three-phase fault MVA at
Eg ∼ Vt
the bus 3 is [2011]
(a) 82.55 MVA (b) 85.11 MVA
(c) 170.91 MVA (d) 181.82 MVA
Solution: (d) Therefore, Vt is higher than Eg, i.e., Vt > Eg but
when the generator is disconnected from the line,
1 j 1.0 3 j 1.0 2 no charging current is delivered by the genera-
f f tor, i.e., IC = 0. In this Vt = Eg so, terminal voltage
I I2
j 0.1 1
j 0.1 decreases.
f
I
Hence, the correct option is (a).
∼ 1∠0° pu 1∠0° pu ∼
Common Data for Questions 4 and 5:
The generation units G1 and G2 are connected by
Fault current = I f = I1f + I 2f 15 kV line with a bus at the mid-point as shown
below
1∠0°
I1f = I 2f = = − j 0.9091 pu 1 3 2
j 1.0 + j 0.1
∼ ∼
f 10 km 10 km
  I = − j1.8182 pu G1 15 kV 15 kV G2
Fault level = 1 × 1.8182 = 1.8182 pu
Fault level (in MVA) = 1.8182 × 100 = 181.82 MVA G1 = 250 MVA, 15 kV, Positive sequence X = 25%
on its own base.
Hence, the correct option is (d).
 G2 = 100 MVA, 15 kV, Positive sequence X = 10%
3. A 50 Hz synchronous generator is initially con-
on its own base.
nected to a long lossless transmission line, which
is open circuited at the receiving end. With the L1 and L2 = 10 km, positive sequence reactance
field voltage held constant, the generator is discon- X = 0.225 W/km.
nected from the transmission line. Which of the 4. For the power system shown in the figure below,
following may be said about the steady state termi- the specifications of the components are the
nal voltage and field current of generator? [2010] following:
Long
Transmission Line Receiving G1 : 25, 100 MVA, X = 9%

end G2 : 25 kV, 1100 MVA, X = 12%

(a) The magnitude of terminal voltage decreases, 25 kV


T1 : , 90 MVA, X = 12%
and the field current does not change. 220 kV
(b) The magnitude of terminal voltage increases, 220 kV
T2 : , 90 MVA, X = 12%
and the field current does not change. 25 kV
(c) The magnitude of terminal voltage increases,
Line 1 : 220 kV, X = 150 Ω
and the field current increases.
(d) The magnitude of terminal voltage does not
T1 T2
change and the field current decreases. Line 1
Solution: (a)
As field voltage is held constant, so field current ∼ Bus 1 Bus 2 ∼
does not change. G1 G2

Chapter 10.indd 81 11/10/2015 6:07:57 PM


4.82 | Power Systems

Choose 25 kV as the base voltage at the generator 2


 200   25 
G1, and 200 MVA as the MVA base. The imped- X T 1 = j 0.12 ×   ×   = j 0.27 pu
ance diagram is [2010]  90   25 
MVA base
X1 = X Ω ×
(a) j 0.27 j 0.42 j 0.27 (kVb ) 2
200
X 1 = j150 × = j 0.62 pu
j 0.18 j 0.18 (220) 2
X T 2 = X T1 = j 0.27 pu
G1 ∼ ∼ G2 2
 200   25 
X G 2 = j 0.09 ×     = j 0.18 pu
 100   25 
(b) j 0.27 j 0.62 j 0.27 Hence, the correct option is (b).
5. A generator is connected through a 20 MVA,
j 0.18 j 0.18 13.8/138 kV step down transformer, to a transmis-
sion line. At the receiving end of the line a load
is supplied through a step down transformer of
G1 ∼ ∼ G2 138
10 MVA, kV rating. A 0.72 pu load, evaluated,
69
on load side transformer ratings as base values of
10 MVA and 69 kV in load circuit, the value of
(c) j 0.27 j 0.42 j 0.27
the load (in per unit) in generator circuit will be
 [2006]
j 0.21 j 0.21 (a) 36 (b) 1.44
(c) 0.72 (d) 0.18
G1 ∼ ∼ G2 Solution: (a)
T1 T2
a b
(d) j 0.3 j 0.42 j 0.3 G ∼ Load
Line

20 MVA 10 MVA
j 0.21 j 0.21 13.8 138
kV kV
138 69

G1 ∼ ∼ G2 Base values on the load circuit


(MVA)B = 10 MVA  and  (kV)B = 69 kV.

Solution: (b) 692


=
Base Impedance = 476.1,
  10
XT 1 X1 XT 2
Values of load in ohm = ZL.
j0.27 j0.62 j0.27
XG1
= Value of load in pu × Base impedance
0.18 XG2
⇒ Z L = 0.72 × 476.1 = 342.8 Ω
G1 ∼ ∼ G2 Base values in generator circuit,
(MVA)B = 20 MVA
(kV)B = 13.8 kV
2
 200   25  (kV) 2B 13.82
X G1 = j 0.9 ×   ×   = j 0.18 pu = Z=
Base Impedance B =
 100   25  (MVA) B 20

Chapter 10.indd 82 11/10/2015 6:07:58 PM


Chapter 10  Per Unit System  |  4.83

⇒ Z B = 9.522 Ω Applying KVL in the Loop


Z L 342.8 1.4∠30° − j1.0( I ) − j 0.25( I ) − 0.9 = 0
= = = 36 pu
Z B 9.522 I ( j1.250) = 0.766∠65.947°
Hence, the correct option is (a). I = 0.613∠−24°
6. A new generator having Eg = 1.4∠30° pu [equiv- Vt = 1.430° − ( j1.0 × 0.612∠−24°)
alent to (1.212 + j0.70)pu] and synchronous Vt = 0.973∠8.27°
reactance Xs of 1.0 pu on the system base, is to
be connected to a bus having voltage V, in the Hence, the correct option is (b).
existing power system. This existing power sys- 7. A 75 MVA, 10 kV synchronous generator has
tem can be represented by Thevenin’s voltage Xd = 0.4 pu. The Xd value (in pu) to a base of 100
Eth = 0.9∠0° pu in series with Thevenin’s imped- MVA, 11 kV is [2001]
ance Zth = 0.25∠90° pu. The magnitude of the volt- (a) 0.578 (b) 0.279
age Vt, of the system in pu will be [2004] (c) 0.412 (d) 0.44
(a) 0.990 (b) 0.973
Solution: (d)
(c) 0.963 (d) 0.900
2
Solution: (b)  100   10 
( X d ) new = 0.4 ×     = 0.44077
Z th = 0.25∠90°  75   11 
Hence, the correct option is (d).
j 1.0
E th = 0.9∠0° ∼ I
Vt
∼ 1.4∠30°

Chapter 10.indd 83 11/10/2015 6:07:59 PM


This page is intentionally left blank.

Chapter 10.indd 84 11/10/2015 6:07:59 PM


Unit 5
Control Systems

Chapter 1:  Basic Control Systems 5.3


Chapter 2:  Signal Flow Graph and Block Diagram 5.6
Chapter 3:  Stability 5.9
Chapter 4:  Time Response Analysis 5.14
Chapter 5:  Root Locus Diagram 5.26
Chapter 6:  Frequency Response Analysis 5.32
Chapter 7:  Controllers (or) Compensators 5.48
Chapter 8:  State Space Analysis 5.51

Chapter 01.indd 1 11/9/2015 5:48:42 PM


Chapter 01.indd 2
Exam Analysis
Exam Year 91 92 93 94 95 96 97 98 99 00 01 02 03 04 05 06 07 08 09 10 11 12 13 14
1 Mark Questions 3 5 3 5 7 4 2 3 – 4 3 1 3 3 3 1 1 1 3 2 3 1 2 4
2 Marks Questions 3 – – – – 2 3 2 1 2 – 4 7 8 5 3 7 9 5 3 3 3 3 6
4 Marks Questions – – – 1 – – – – – – – – – – – – – – – – – – – –
5 Marks Questions – – – – 1 – 1 2 – 3 2 3 – – – – – – – – – – – –
Total Marks 6 5 3 6 8 6 6 7 1 9 5 8 10 11 8 4 8 10 8 5 6 4 5 10
Basic Control Systems – – – – 1 – – – – 2 – 1 – 1 – – – – 1 1 – – 1 –
Signal Flow Graph and Block Diagram 1 1 1 – – – – 1 – – – – 1 1 – – 1 – – – – – 1 –
Stability – 1 – 2 1 – 2 2 – – – – 1 2 – – 2 1 1 – – 1 – 3
Time Response Analysis 1 1 – – 4 5 1 1 – 3 – 1 4 3 1 – 3 5 1 1 3 – 1 1
Root Locus Diagram 1 1 – – – – – – – 1 1 2 – – 1 1 – – – 1 1– – 1 –
Frequency Response Analysis 3 1 – 1 – 1 1 1 1 1 3 – 1 3 4 2 1 1 3 1 2 1 1 2
Controllers (or) Compensators – – – 1 – – 1 1 – 1 – – 1 – – – 1 1 – – – 1 – –
State Space Analysis – – 2 2 2 – 1 1 – 1 1 4 2 1 2 1 – 2 2 1 – 1 1 3

11/9/2015 5:48:42 PM
Chapter 1
Basic Control Systems

One-mark Questions  1 
R+ 
Cs
V2 ( s ) =   ⋅V ( s ).
1 1 1
V (s) R+ +
1. The transfer function  2  of the circuit shown Cs Cs
below is V1 ( s ) [2013]
V2 ( s ) 1 + RCs
⇒ = .
100 μF V1 ( s ) 2 + RCs
+ +
R = 100 C = 100 µf.
10 kΩ
V1(s) V2(s)
1
100 μF  81 = 10 × 103 × 100 × (1)
− − 100

V2 ( s ) s + 1
0.5s + 1 3s + 6 So, = .
(a) (b) V1 ( s ) s + 2
s +1 s+2
Hence, the correct option is (d).
s+2 s +1
(c) (d) 2. As shown in the figure, a negative feedback system
s +1 s+2
has an amplifier of gain 100 with ±10% tolerance
Solution: (d) in the forward path, and an attenuator of value
For capacitor with capacitance C farads in Laplace 9/100 in the feedback path. The overall system
1 gain is approximately: [2010]
domain impedance  = .
Cs
1 +
Cs 100 ± 10%

+

R 9/100
V1(s) V2(s)
1
Cs

(a) 10 ± 1% (b) 10 ± 2%
By voltage divide rule (c) 10 ± 5% (d) 10 ± 10%

Chapter 01.indd 3 11/9/2015 5:48:44 PM


5.4 | Control Systems

Solution: (a) Kt
Kts2 (b)
(a)
Since, change is in forward path, s
G dT 1 (c)
Kts (d) Kt
So,  SGT = ⋅ = .
T dG 1 + GH
Solution: (c)
9 For tachometers,
Since, 
= G 100 = ;H .
100
O/p voltage ∞ | Angular velocity of g/p shaft |
1 1
SGT = = . dθ (t )
9 10 V (t ) = K .
1 + 100 ×
100 dt
1 Taking Laplace,
So error in output,  of 10% = 1%.
10 th V ( s ) = K t sθ ( s ).
100 V (s)
Also total Tx function = = 10. = K t s.
9
1 + 100 × θ (s)
100
Total gain = 10 ± 1%. Hence, the correct option is (c).
Hence, the correct option is (a). 5. Feedback control systems are [2000]
3. Errors associated with each respective subsystem (a) Insensitive to both forward and feedback path
G1, G2 and G3 are e1, e2 and e3. The error associated parameter changes
with the output is: [2009] (b) Less sensitive to feedback path parameter changes
than to forward path parameter changes
1 (c) Less sensitive to forward path parameter changes
G1 G2
Input G3 Output than to feedback path parameter changes
(a) e1 − e2 + e3 (b) e1e2/e4 (d) Equally sensitive to forward and feedback path
(c) e1 + e2 − e3 (d) e1 + e2 + e3 parameter changes
Solution: (c) Solution: (c)
GG For −ve f /b system,
Overall  Tx function = 1 2 .
G3
G (s)
Changes due to G1 = SGT1 . TF = .
1 + G ( s) H (s)
Change due to G2 = SGT 2 .
dT ( s ) /T ( s ) G ( s ) dT ( s )
Change due to G3 = SGT 3 . SGT = = ⋅
dG ( s ) /G ( s ) T ( s ) dG ( s )
G1 dT G3 G2 [1 + GH − GH ] 1
SGT1 = × = × = 1. = (1 + GH ) ⋅ = .
T dG1 G2 G3 (1 + GH ) 2
1 + GH
G2 dT G G
SGT 2 =
Similarly,  ⋅ = 3 ⋅ 1 = −1. H ( s ) dT ( s )
T dG2 G1 G3 S HT = ⋅
T ( s ) dH ( s )
G3 dT G 2  −G G 
SGT 3 = ⋅ = 3 ⋅  12 2  = −1. H (1 + GH ) −G 2
T dG3 G1G2  G3       = ⋅ .
G (1 + GH ) 2
Total error  = E1 + E2 − E3 .
Hence, the correct option is (c). −GH
S HT = .
4. For a tachometer, if q (t) is the rotor displacement 1 + GH
in radians, e(t) is the output voltage and Kt is the SGT < S HT .
    
tachometer constant in V/rad/sec, then the transfer
E (s) So f /b systems are more sensitive to f /b path changes.
function , will be [2004]
θ (s) Hence, the correct option is (c).

Chapter 01.indd 4 11/9/2015 5:48:47 PM


Chapter 1  Basic Control Systems  |  5.5

6. A linear time invariant system initially at rest, when 1


subjected to a unit step input, gives a response y (t) L(input ) =
  .
s+2
= te−t, t > 0. The transfer function of the system is
 [2000]  1  −2 t
Input = L−1   = e u (t ).
1 1 s+2
(a) 2 (b)
( s + 1) s ( s + 1) 2 Hence, the correct option is (c).
(c) s 2 (d)
1
( s +1) s ( s + 1)
Two-marks Questions
Solution: (c)
x(t ) = Input = u (t ). 1. The transfer function of the system described by
d 2 y dy du
−t
y (t ) = t e , t > 0. + = + 2u  with u as input and y as out-
dt 2 dt dt
L( y (t )) Y ( s ) put is[2002]
Tx function = =
L( x(t )) X ( s ) ( s + 2) ( s + 1)
(a) 2 (b)
1 ( s + s) ( s 2 + s)
( s + 1) 2 s 2 2s
= = . (c) 2 (d)
1 ( s + 1) 2 ( s + s) ( s 2 + s)
s
Solution: (a)
Hence, the correct option is (c).
d 2 y dy du
7. The impulse response of an initially relaxed lin- + = + 2u.
ear system is e−2tu (t). To produce a response of dt 2 dt dt
(te−2t) u (t), the input must be equal to [1995] Y (s)
TF = .
1 −2t U (s)
(a) 2e−tu(t) (b) e u (t )
2 Taking Laplace of above equation, s2Y(s) + sY(s) =
(c) e−2tu(t) (d)
e−tu(t) sU(s) + 2U(s) [For Tx function, initial condition is
zero].
Solution: (c)
L(input) ⋅ L(impulse response) = L(output) Y ( s) s + 2
= .
1 1 U (s) s 2 + s
⇒ L(input ) ⋅ = .
s + 2 ( s + 2) 2 Hence, the correct option is (a).

Chapter 01.indd 5 11/9/2015 5:48:50 PM


Chapter 2
Signal Flow Graph and
Block Diagram
Solution: (b)
One-mark Questions
G
1 1
1. Signal flow graph is used to obtain the [1993]
(a) stability of a system 1 y
w
1 H H
(b) transfer function of a system
(c) controllability of a system 1
G
(d) observability of a system
Solution: (b)
Forward paths: P1 = 1⋅ G ⋅1 = G.
Signal flow graph is used to obtain transfer func-
tion of the system. P2 = 1⋅ G ⋅1 = G.
Hence, the correct option is (b). P3 = 1⋅ G ⋅ H ⋅ G ⋅1 = G 2 H .
2. The overall transfer function of the system in fig-    
ure, is [1992] P4 = G 2 H .

∆ k ′s: ∆1 = 1; ∆ 2 = 1; ∆ 3 = 1; ∆ 4 = 1.
+
G
Δ = 1- [G2 H2].
+ Δ = 1 − [G2H2].
H + y
w ∑ Pk ∆ k
Using Mason’s gain formula: TF =
H + ∆
+ 2G + 2G 2 H
+
G ⇒ TF =
1− G2 H 2
2G[1 + GH ]
=
[1 + GH ][1 − GH ]
G 2G
(a) (b)
1− GH 1 − GH 2G
= .
GH GH 1 − GH
(c) (d)
1− GH 1− H Hence, the correct option is (b).

Chapter 02.indd 6 11/2/2015 12:29:28 PM


Chapter 2  Signal Flow Graph and Block Diagram  |  5.7

with
Two-marks Questions (a) X = C0s + C1Y = l/(s2 + a0s + a1), Z = b0s + b1
(b) X = 1, Y = (c0s + c1)/(s2 + a0s + a1), Z = b0s + b1
1. The signal flow graph for a system is given below. (c) X = C1s + C0, Y = (b1s + b0)/(s2 + a1s + a0), Z = l
Y (s)
The transfer function   for this system is [2013] (d) X = C1s + C0, Y = l/(s2 + a1s + a), Z = b1s + b0
U (s)
Solution: (d)
1
For figure 1,
1 s −1 s −1 1 P ⋅ c0 P ⋅ c1
u(s) y(s) P1 = ; P2 = .
s 2
s
−a −a Pb Pb
−4 L1 = 1 ; L2 = 20 ; L3 = 20 ; L4 = 1 .
s s s s
−2
P ⋅ c0 P ⋅ s1
s +1 +
(a) s + 1 (b) TF from Figure 1 = s2 s
5s + 6 s + 2
2
s + 6s + 2
2
a1 a0 Pb0 Pb1
1+ + 2 − 2 −
s s s s
(c) s + 1 (d)
1
s + 4s + 2
2
5s + 6 s + 2
2
P [C1 s + C0 ]
  = 2 . (1)
Solution: (d) ( s + a1 s + a0 ) − P (b1 s + b0 )
P1 = s −2 ; P2 = s −1. From Figure 2,

L1 = − 4 s −1 ; L2 = −2 s −2. P1 = XYP; L1 = YPZ

L3 = − 4; L4 = −2 s −1. TF = PXY . (2)


1 − PYZ
∑ Pk ∆ k Comparing 1 and 2,
TF =
∆ Hence, the correct option is (d).
s −2 + s −1 s +1 3. For the block diagram shown in figure, the transfer
= −1 −2
= 2 .
1 + 6 s + 2 s + 4 5s + 6 s + 2 function is equal to [2004]
Hence, the correct option is (d). R(s) 1 + 1 + C(s)
2. The system shown in figure below [2007] S S
+ +

b0 c0 b1 c1 s2 + 1 s2 + s + 1
(a) 2 (b)
+ +
s s2
Σ s + s +1
2
1
+ + + +
(c) (d)
Σ 1/s Σ 1/s P
s s + s +1
2

− −
Solution: (b)
a0 a1 1 1
s s
R(s) C(s)
Figure 1
1 1
Can be reduced to the form
1 1
+ P1 = ; P2 = ; P3 = 1.
X
+
Σ Y P s2 s
1 1
+ +1
s s2 + s + 1
TF = s
2
Z = .
1 s2
Figure 2 Hence, the correct option is (b).

Chapter 02.indd 7 11/2/2015 12:29:31 PM


5.8 | Control Systems

4. The block diagram of a control system is shown G1G2 G3


(a)
in figure. The transfer function G(s) = Y(s)/U(s) of 1+ H 2 G2 G3 + H1G1G2
the system is [2003]
GG G
9 (b) 1 2 3
1+ G1G2 G3 H1 H 2
u(t ) − y (t )
Integrator 2 Integrator
+ − + − G1G2 G3
(c)
1+ G1G2 G3 H1 + G1G2 G3 H 2
3 12
GG G
1 1 (d) 1 2 3
(a)  (b)  1+ G1G2 G3 H1
 s  s   s  s 
18 1 +  1 +  27 1 +  1 +  Solution: (a)
 12   3   6  9  −H 2

1 1 1 G1 G2 G3
(c)  (d)  R(s) C(s)
 s  s   s  s 
27 1 +  1 +  27 1 +  1 + 
 12   9   9  3  −H1

Solution: (b) P1 = G1G2 G3 ; ∆1 = 1.


1
In Laplace domain, integrator  = . L1 = −G1G2 H1 ;
L2 = −G2 G3 H 2 .
s
∑ Pk ∆ k
Replacing, -9 Using Mason’s Gain formula TF = .

1 1 G1G2 G3
s 2 s ⇒ TF = .
U(s) Y(s) 1 + G1G2 H1 + G2 G3 H 2
-3 -12
Hence, the correct option is (a).
-3 -12
6. The signal flow graph of figure shown below, has
2 ______ forward paths and _____ feedback loops.
P1 = ∆1 = 1.
s2 [1991]
−18 −3 −12 c
L1 = 2 ; L2 = ; L3 = .
s s s
2 b
2 d e
TF = s 2
=
18 3 12 36 s 2 + 15s + 54 x a h
1+ 2 + + + 2
s s s s
2 1 k 1 f
= = .
( s + 6)( s + 9)  s  s  m g y
27 1 +  1 + 
 6  9 
Hence, the correct option is (b). n
Solution:
5. For block diagram shown in Figure C(s)/R(s) is Forward paths: 4
given by [1996]
=P1 abdfg
= P2 ahfg
H2 =P3 aklfg
= P4 akmg

− Loops: 4
R(s) C(s)
G1
+
G2 G3 =L1 C=L2 de
+ −
=L3 lfn
= L4 mn
H1
Hence, 4 forward paths and 4 feedback loops.

Chapter 02.indd 8 11/2/2015 12:29:35 PM


Chapter 3
Stability

d ( 4 s 2 + 4)
One-mark Questions A( s ) = = 8s.
ds
1. In the formation of Routh–Hurwitz array for a So
polynomial, all the elements of a row have zero s1 8 0
values. This premature termination of the array
indicates the presence of [2014-S1] s 0
4 0
(a) only one root at the origin
(b) imaginary roots Roots at jw axis  ⇒ 4s2 + 4 = 0.
(c) only positive real roots ⇒     
s = ± j.
(d) only negative roots
Hence, the correct option is (d).
Solution: (b)
3. The system shown in the figure is  [2007]
Hence, the correct option is (b).
2. The first two rows of Routh’s tabulation of a third
u1 + s−1
order equation are as follows ∑ C1
− s+2
S 3  2  2
S 2  4  4 +
1 u2
This means there are [2009] ∑
C2 1+s +
(a) two roots at s = ± j and one root in right half
s-plane
(b) two roots at s = ± j2 and one root in left half (a) stable
s-plane (b) unstable
(c) two roots at s = ± j2 and one root in right half (c) conditionally stable
s-plane (d) stable for input u1, but unstable for input u2
(d) two roots at s = ± j and one root in left half Solution: (a)
s-plane For input u1,
Solution: (d) s −1
Output C1, Transfer function = s +2
s3 2 2 s −1 1
s2 4 4 1+ ×
s + 2 s +1
s1 0 0 ( s − 1)( s + 1)
= .
Forming auxiliary equation, s 2 + 4s + 1

Chapter 03.indd 9 11/3/2015 2:55:49 PM


5.10 | Control Systems

By Routh, R(s) C(s)


G(s)
s2 1 1 −

s1 4 H(s)
s0 1

Hence, stable. Solution:


s −1 True.
For output C2, Transfer function = . 7. The number of positive real roots of the equation
( s + 1)( s + 2)
s3 - 2s + 2 = 0 is  [1994]
Since, roots are negative. Hence, stable.
For input u2, Solution:
−( s − 1)
Output C1, Transfer function = . s 3 − 2 s + 2 = 0.
( s + 2)( s + 1)
Roots negative, so stable. Roots: −1.7693, 0.8846 ± j 0.5897.
1
Hence, no positive real roots.
Output C2, TF = 1 + s .
( s − 1) 8. For what range of K is the following system
1− (figure) asymptotically stable. Assume K ≥ 0.
­
( s + 2)( s + 1)
 [1992]
s+2
Transfer function = .
s + 2s + 3
2
+ s−5
s2 1 3 − s+4
s1 2
s0 3 K

Hence, stable.
Hence, the correct option is (a). Solution:
4. None of the poles of a linear control system lie in
G ( s)
the right half of s-plane. For a bounded input, the TF =
output of this system [1998] 1 + G ( s) H (s)
(a) is always bounded s −5
(b) could be unbounded s+4
(c) always tends to zero =
k ( s − 5)
(d) none of the above 1+
( s + 4)
Solution: (b)
s −5
If more than one poles lie on jw axis then it will be = .
unstable.   s (1 + k ) + 4 − 5k
Hence, the correct option is (b). By Routh array table.
5. Closed loop stability implies that [l + G(s) H(s)]
has only in the left half of the s-plane. s1 1 + k.
 [1995]
s 0
4 − 5 k.
Solution:
Zeroes and poles. For stability, 1 + k ≥ 0  ⇒  k ≥ -1 and 4 -5 k ≥ 0
6. The closed loop system, of figure, is stable if the 4
C (s) ⇒ k= .
transfer function T ( s ) = is stable. 5
R( s) 4
Hence, the correct answer is 0 ≤ k ≤ .
Transfer Function (TF)[1994] 5

Chapter 03.indd 10 11/3/2015 2:55:51 PM


Chapter 3  Stability  |  5.11

3. The feedback system shown below oscillates at 2


Two-marks Questions rad/s when [2012]

1. For the given system, it is desired that the system


+ K (s + 1)
be stable. The minimum value of a for this condi- − s 3 + as 2 + 2s + 1
tion is _________ [2014-S1]

R(s) (s + a) C (s)
+
− s 3 + (1 + a)s 2 + (a − 1)s + (1 − a)

(a) K = 2 and a = 0.75  (b)  K = 3 and a = 0.75


(c) K = 4 and a = 0.5   (d)  K = 2 and a = 0.5
Solution: Solution: (a)
Characteristic Equation: s 3 + (1+ ∝) s 2 + ∝ s − 1 = 0 Characteristic Equation: s 3 + as 2 + s (2 + K ) + 1+ K = 0.

s3 1 ∝ s3 1 2+ K
s2 1+ ∝ 1 s2 a 1+ K
∝ 2 + ∝ −1 a (2 + K ) − (1 + K )
s1 0 s1 0
∝ +1 a
s0 1 s0 1+ K

For stability, 1 + ∝ > 0 ⇒ ∝ > -1 and ∝2 + ∝ - 1 > 0 For system to oscillate,


⇒ ∝ > 0.62.
Hence, the minimum value is 0.62. a (2 + K ) − (1 + K ) = 0.
Hence, the correct answer is 0.62. ⇒ K (a − 1) = 1 − 2a.
2. A system with the open loop transfer function 1 − 2a
⇒ K= (1)
K a −1
G ( s) = is connected in a neg-
s ( s + 2)( s 2 + 2 s + 2)
Also to oscillate at 2 rad/sec
ative feedback configuration with a feedback gain
of unity. For the closed loop system to be margin- A(s)  ⇒  as2 + 1 + K = 0  At  s = ±2j
ally stable, the value of K is _____. [2014-S2] ⇒ − 4a + 1 + K = 0
Solution:   ⇒ K = 4 a − 1. (2)
Characteristic Equation: s4 + 4s3 + 6s2 + 4s + K. Solving Eqs. 1 and 2
s 4
1 6 K =K 2=
and a 0.75.
s3 4 4 Hence, the correct option is (a).
s2 5 K 4. Figure shows a feedback system where K > 0
[2008]
20 − 4 K
s1 +
5 Σ
K
s0 K s(s + 3)(s + 10)

20 − 4 K
For marginal stable, = 0.
5 The range of k for which system is stable will be
given by
⇒ K = 5. (a) 0 < K < 30 (b) 0 < K < 39
Hence, the correct answer is 5. (c) 0 < K < 390 (d) K > 390

Chapter 03.indd 11 11/3/2015 2:55:53 PM


5.12 | Control Systems

Solution: (c) 7. The algebraic equation [2006]


Characteristic Equation: s3 + 13s2 + 30s + K.  F(s) = s - 3s + 5s - 7s + 4s + 20 is given. F(s) = 0 has
5 4 5 2

s3 1 30 (a) single complex root with the remaining roots


s 2
13 K being real
390 − K (b) one positive real root and four complex roots,
s1 0 all with positive real parts
13
(c) one negative real root, two imaginary roots,
s0 K
and two roots with positive real parts
390 − K (d) one positive real root, two imaginary roots,
For stability, > 0 ⇒ K < 390.
13 and two roots with negative real parts
K > 0.
Solution: (c)
Hence, the correct option is (c).
5. If the loop gain K of a negative feedback system s5 1 5 4
having a loop transfer function K(s + 3)/(s + 8)2
is to be adjusted to induce a sustained oscillation s4 −3 −7 20
then [2007] 8 32
s3
(a)  the frequency of this oscillation must be 3 3
4 s2 5 20
rad/s
3 s1 0 (10) 0
(b) the frequency of this oscillation must be 4 rad/s
s0 20
(c) the frequency of this oscillation must be 4 or
4
rad/s d (5s 2 + 20)
3 A′( s ) = = 10 s.
(d) such k does not exist ds
Solution: (b)
Characteristic Equation: s2 + s(16 + K) + 64 + 3K. Two sign change → Two poles on right side.
s2 1 64 + 3K Zero in 1st column → Two imaginary poles (±2j)
s1 16 + K 0 Hence, the correct option is (c).
s 0
64 + 3K 8. For the equation, s3 - 4s2 + s + 6 = 0 the number of
For oscillations, 16 + K = 0. roots in the left half of s-plane will be [2004]
⇒ K = −16. (a) Zero (b) One
(c) Two (d) Three
A( s ) = s 2 + 64 + 3K = 0.
Solution: (c)
⇒ s 2 + 16 = 0.
    s = ±4 j = jω ⇒ ω = 4 rad/sec. s3 1 1
Hence, the correct option is (b). s2 −4 6
6. A continuous time system is described by y(t) = s1 2.5
e- |x ( t ) |, where y(t) is the output and x(t) is the input. s0 6
y(t) is bounded [2006]
(a) only when x(t) is bounded Two sign changes in 1st column.
(b) only when x(t) is non-negative Hence, the correct option is (c).
(c) only for t ≥ 0 if x(t) is bounded for t ≥ 0
(d) even when x(t) is not bounded 9. A unity feedback system, having an open loop
K (1 − s )
Solution: (d) gain becomes stable when G ( s ) H ( s ) =
(1 + s )
y (t ) = e −| x ( t )|
 [2004]
For x(t ) = ∞, y (t ) = e −∞ = 0. (a) |K| > 1 (b) K > 1
Hence, the correct option is (d). (c) |K| < 1 (d) K < -l

Chapter 03.indd 12 11/3/2015 2:55:55 PM


Chapter 3  Stability  |  5.13

Solution: (c) 12. The system represented by the transfer function


Characteristic Equation: s(1 - K) + 1 + K. s 2 + 10 s + 24
G (s) = 4 has pole (s) in the
s 1− K s + 6 s − 39 s 2 + 19 s + 84
right-half s-plane. [1997]
s0 1+ K
Solution:
For stable, 1 − K > 0 ⇒ K < 1 . s4 1 − 39 84
1 + K > 0 ⇒ K > −1. s3 6 19 0
Combining both | K | < 1. s2 −42.2 84
Hence, the correct option is (c). s1 30.92 0
1 0. The loop gain GH of a closed loop system is given s0 84
K Since, there are 2 sign changes in list column.
by the following expression  . The
s ( s + 2)( s + 4)
Hence, the correct answer is 2.
value of K for which the system just becomes
unstable is [2003] 1 3. Determine whether the system given by the block
(a) K = 6 (b) K = 8 diagram of figure is stable [1997]
(c) K = 48 (d) K = 96
Solution: (c) R(s) + +

+
1 1 1
Characteristic Equation: s3 + 6s2 + 8s + K = 0. (s + 1) −
4
(s − 4)
3
− (s + 3) C(s)

4 8
s3 1 8
s2 6 K Solution:
48 − K ∑ Pk ∆ k
s1 T ( s) = [By Mason’s gain formula].
6 ∆
s0 K 1 1 1
Forward path = ⋅4⋅ ⋅3⋅
s +1 s−4 s+3
48 − K 12
For system to be just unstable, = 0. = .
6 ( s + 1)( s − 4)( s + 3)
⇒ K = 48.
Hence, the correct option is (c). −16 −8 −3
Loops: L1 = ; L2 = ; L3 = .
1 1. The number of roots on the equation 2S4 + S3 + 3S2 s−4 s+3 ( s + 3)( s − 4)
+ 5S + 7 = 0 that lie in the right half of S plane is
[1998] 12
(a) zero (b) one ( s + 1)( s − 4)( s + 3)
(c) two (d) three T ( s) =
 −16 8 3  128
Solution: (c) 1−  − −  +
 s − 4 s + 3 ( s + 3)( s − 4)  ( s + 3)( s − 4)
s4 2 3 7 12
= .
s 3 + 24 s 2 + 158 s + 135
s3 1 5
s2 −7 7 s3 1 158
s1 6< < s2 24 135
s0 7 s1 152.4
Two sign changes in 1st column. s0 135
Hence, the correct option is (c). Hence, the system is stable.

Chapter 03.indd 13 11/3/2015 2:55:58 PM


Chapter 4
Time Response Analysis
1
One-mark Questions ess = Lt SE ( s ) = Lt
s →0 s →0 1 + G (s)
⋅ R( s).

1. The closed-loop transfer function of a system is


r (t ) = 10[ µ (t ) − µ (t − 1)] −  Given input L[r (t)] = R(s)

4
T ( s) = 2 . The steady state error due 10 − 10e − s
( s + 0.4 s + 4) = .
to unit step input is . [2014-S2] s
Solution:  
4   10[1 − e − s ]
Final value  = Lt ST ( S ) = = 1. 1
s →0 4 ess = Lt s   =0
Hence, O/P = 1. s →0  k (1 + sT1 )  s
1+
 (1 + sT2 ) 
I/P = 1 unit step I/P. 
∴ Error  = Lt [I/P − O/P] = 0.
t →∞ Steady state: error = 0.
2. The steady state error of a unity feedback linear Hence, the correct option is (a).
system for a unit step input is 0.1. The steady state 2
error of the same system, for a pulse input r(t) hav- 3. For the system , the approximate time taken
( s + 1)
ing a magnitude of 10 and a duration of one sec- for a step response to reach 98% of its final value is
ond, as shown in the figure. [2011] [2010]
r (t) (a) 1s (b) 2s
(c) 4s (d) 8s
10
Solution: (c)

2 C (s)
TF = = .
1s t s + 1 R( s) 1
R ( s ) = ( step )
s

(a) 0 (b) 0.1  2 1


C ( s) =   .
(c) 1 (d) 10  s +1 s
Solution: (a)
Error = 0.1.
2
∴  It must be a type 0 system (0.98)(2)
k (1 + sT1 )
Let  G (s) =  H ( s ) = 1. 0 4 sec
(1 + sT2 ) t

Chapter 04.indd 14 11/3/2015 3:38:42 PM


Chapter 4  Time Response Analysis | 5.15

Taking L−1, (a) x = 6 (b) x = 2


−t
C (t ) = 2[1 − e ]u (t ). (c) x = 2.4 (d) x = −2
C (∞) = 2. Solution: (c)
2[1 − e − t ] = 0.98[2]. d 2 x 6dx
+ + 5 x = 12(1 − e −2t ).
T = 4 sec. dt 2
dt
Hence, the correct option is (c). Taking LT
4. A function y (t) satisfies the following differential 1 1 
x( s )[ s 2 + 6 s + 5] = 12  − .
dy (t ) s s + 2
equation:  + y (t ) = δ (t ).  Where δ (t) is the
dt 1 1 
delta function. Assuming zero initial condition, and 12  −
 s s + 2  12[ s + 2 − s ]
denoting the unit step function by u (t), y (t) can be x( s ) = 2 = 2 .
of the form [2008] s + 6s + 5 ( s + 6 s + 5)( s )( s + 2)
(a) et (b) e−t Final value  = Lt x(t ) = Lt sx( s )
(c) e  u(t) (d)
t
e−t u(t) t →∞ s →0

Solution: (d)  (12)(2) 


= Lt s  
dy (t ) s →0
 s ( s + 2 )( s 2
+ 6 s + 5) 
+ y (t ) = δ (t ).
dt (12)(2)
= = 2.4.
Taking LT       (2)(5)
SY ( s ) + Y ( s ) = 1. ∴   Final value  = 2.4 .

1
y(s) = . Hence, the correct option is (c).
s +1
7. A unity feedback system has open loop transfer func-
Taking  L−1[ y ( s )] = y ( s ) = e − t u (t ). tion G(s). The steady state error is zero for [2000]
Hence, the correct option is (d). (a) step input and type-1 G(s)”
5
5. Consider the function  F ( s ) =  where (b) ramp input and type-1 G(s)
s ( s + 3s + 2)
2
(c) step input and type-0 G(s)
F (s) is the Laplace transform of the function f (t). (d) ramp input and type-0 G(s)
The initial value of f (t) is equal to [2004] Solution: (a)
5 Type 1, Step input gives zero steady state error.
(a) 5 (b)
2
5 Steady State error (ess)
(c) (d) 0
3
Ramp parabolic
Solution: (d) Type A A A
Step = = 2 = 3
5 S S S
F (s) = .
s ( s 2 + 3s + 2)
A
Initial value 0 ∞ ∞
1+ K p
s5
f (0) = Lt SF ( s ) = = 0.
s →∞ s ( s 2 + 3s + 2) A
1 0 ∞
Hence, the correct option is (d). Kv
6. A control system is defined by the following math-
ematical relationship A
2 0 0
Ka
d2x dx
+ 6 + 5 x = 12(1 − e −2t ).
dt 2 dt 3 0 0 0
The response of the system as t → ∞ is [2003] Hence, the correct option is (a).

Chapter 04.indd 15 11/3/2015 3:38:45 PM


5.16 | Control Systems

8. A linear time invariant system initially at rest, when 11. The unit-impulse response of a unit-feedback con-
subjected to a unit-step input, gives a response trol system is given by c (t) = −te−t + 2e−t, (t ≥ 0) the
y (t) = te−t, t > 0 The transfer function of the system is open loop transfer function is equal to [1996]
[2000] s +1 2s + 1
(a) (b)
1 ( s + 2) 2 s2
(a) 1 2 (b)
( s + 1) s ( s + 1) 2 s +1 s +1
(c) 2 (d)
( s + 1) s2
s 1
(c) 2 (d)
( s +1) s ( s + 1) Solution: (b)
−1 2
Solution: (c) CLTF = L[−te − t + 2e − t ] = +
( s + 1) 2
( s + 1)
Step response = te − t u (t ).
−1 + 2( s + 1) 2s + 1
1 = = .
( s + 1) 2 ( s + 1) 2
L[Output ] L[te − t u (t )] ( s + 1) 2
TF = = = . CLTF
L[Input ] L[u (t )] 1 ∴ OLTF =
1 − CLTF
s H ( s ) =1

s 2s + 1
TF = .
( s + 1) 2 ( s + 1) 2 2s + 1
= = 2 .
2s + 1 s
Hence, the correct option is (c). 1−
( s + 1) 2
9. The output of a linear time invariant control sys-
tem is c(t) for a certain input r (t). If r (t) is modi- 2S + 1
∴ OLTF = .
fied by passing it through a block whose transfer S2
function is e−s and then applied to the system, the Hence, the correct option is (b).
modified output of the system would be [1998]
1 2. Consider the unit step response of a unity feedback
c(t ) c(t )
(a) t (b) control system whose open loop transfer func-
1+ e 1+ e − t 1
tion is G ( s ) = . The maximum overshoot is
(c) c (t − 1) u (t − 1) (d) c (t) u (t − 1) s ( s + 1)
Solution: (c) equal to [1996]
Input r (t) gives c (t) u (t). (a) 0.143 (b) 0.153
(c) 0.163 (d) 0.173
r(t ) r(t − l ) c(t − l )
e −s system Solution: (c)
Characteristic equation  = 1 + G ( s ) H ( s ) = 0.
Input is delayed by one second.  ∴   Output is
also delayed by one second, i.e., c (t − 1) u (t − 1). 1
1+ = 0.
Hence, the correct option is (c). s ( s + 1)
10. Introduction of integral action in the forward path s 2 + s + 1 = 0.
of a unity feedback system results in a [1997] s 2 + s + 1 = s 2 + 2ζωn s + ω 2 n .
(a) marginally stable system
ωn = 1 and ζ = 0.5.
(b) system with no steady state error
(c) system with increased stability margin Maximum overshoot
(d) system with better speed of response  − ζπ 
 
 1− ζ 2 
Solution: (b) Mp = e  
= 0.163.
Integral control action increases the type of the
system by one. If the type increases, the steady Hence, the correct option is (c).
state error decreases. 1 3. For a feedback control system of type 2, the steady
Hence, the correct option is (b). state error for a ramp input is [1996]

Chapter 04.indd 16 11/3/2015 3:38:48 PM


Chapter 4  Time Response Analysis | 5.17

(a) infinite (b) constant E(t ) S+1


(c) zero (d) indeterminate +
− S 2 + 5S + a
Solution: (c)
For a type 0 system error to a ramp input is ∞, for a 1
type 1 system error to a ramp input is finite and for S+4
a type 2 and higher type error to a ramp input is 0
Hence, the correct option is (c). (a) a = 0 (b) a = 2
14. The Laplace transformation of f (t) is F(s). Given (c) a ≥ 4 (d) for no value of a
ω Solution:(a)
F (s) = 2 ,  the final value of f (t) is [1995]
s + ω2
Error = Lt SE ( s ) = Lt e(t ).
(a) Inifinity (b) Zero s →0 t →∝

(c) One (d) None of these


C (s)
Solution: (d) TF  = .
R( s)
SF(s) poles are at s = ± jω, i.e., on the imaginary axis.
 ∴  System/function oscillates with a fixed ampli- 1
E (s) = R( s ) − C ( s ).
tude of frequency rad/sec, hence final value KH
can’t be determined. 1
E (s) = R( s ) − [TF ]R( s ).
Hence, the correct option is (d). KH
15. The steady state error due to a step input for type 1 1
system is . [1995] E ( s) = R( s )[1 − K H (TF )].
KH
Solution:
To a step input, steady state error of a type 0 system 1 1
K H = Lt H ( s ) = Lt
Where  = .
is finite, for a type 1 and higher type error is zero. s →0 s →0 ( s + 4) 4
16. The closed loop transfer function of a control sys- = Lt SE ( s ) = 0
Error 
C (s) 2( s − 1) s →0
tem is given by  = ,  for a unit
R( s ) ( s + 2)( s + 1)  4a 
step input the output is [1995]
= 4  = 0, gives a = 0.
 4a + 1 
(a) − 3e−2t + 4e−t − 1 (b) − 3e−2t − 4e−t + 1
(c) Zero (d) Infinity Hence, the correct option is (a).
Solution: (a)
R(s)
Two-marks Questions
2( s − 1)
C (s) = .
( s + 2)( s + 1) R(s)=
1 1. The open-loop transfer function of a DC motor is
s
ω (s) 10
C1 C C given as = . When connected in feed-
C (s) = + 2 + 3 . Va ( s ) 1 + 10 s
S S + 2 S +1 back as shown below, the approximate value of Ka
C1 = −1, C2 = −3, C3 = 4. that will reduce the time constant of the closed
−1 3 4 loop system by one hundred times as compared to
C (s) = − + .
s s + 2 s +1 that of the open-loop system is [2013]
Time response c(t ) = L−1[C ( s )].
R(s) Va (s ) 10 w(s)
 C (t ) = −1 − 3e −2t + 4e − t . + K2
− 1 + 10s
Hence, the correct option is (a).
17. For what values of a does the system shown in fig-
ure have a zero steady state error [i.e., lim E (t )] (a) 1 (b) 5
t →∞
for a step input? [1992] (c) 10 (d) 100

Chapter 04.indd 17 11/3/2015 3:38:51 PM


5.18 | Control Systems

Solution: (c) Solution: (a)


Given, open loop transfer function of a DC motor TF ⇒  Internal block
10 1
= .  The time constant form of a single or-
1 + 10s K ( s + 1) s 1
der transfer function is   ,  where  τ =  time = = A.
ks s ( s + 1) + ks
1+ sτ 1+
constant. By comparing two transfer functions, s ( s + 1)
τ = 10sec. The closed loop transfer function R (s ) + Y (s )
10 A

Ka ×
1 + 10 s = 10 K a
= .
1 + Ka ×
10 1 + 10 s + 10 K a
1 + 10 s Y (s) 1
= .
This transfer function in time constant form is R( s ) s 2 + (k + 1) s + 1
10 K a ωn2 = 1 2ξωn = k + 1
(1 + 10 K a ) k +1
=
  ξ = 2ω
.
10
1+ ×s n
1 + 10 K a If k increases, ξ increases and overshoot will de-
10  ξπ 
Time constant (τ 2 ) of closed loop is  τ 2 = . − 
1 + 10 K a creases M p = e
 1−ξ 2


 as natural frequency ωn will

1
τ 2  should be   time of  τ 1 .
100 not change.
τ1 Hence, the correct option is (a).
τ2 =
i.e.,  . 3. The response h(t) of a linear time invariant system
100
to an impulse δ (t), under initially relaxed condi-
10 10 tion is h(t) = e−t + e−2t. The response of this system
⇒ = .
1 + 10 K a 100 for a unit step input u (t) is [2011]
(a) u (t) + e−t + e−2t (b) (e−t + e−2t) u (t)
⇒ K a = 9.9 ≅ 10.
(c) (1.5 − e−t − 0.5e−2t) u (t) (d) e−t δ (t) + e−2t u (t)
∴ K a = 10. Solution: (c)
Hence, the correct option is (c). 1
h(t ) = e − t + e −2t ; R( s ) = .
s
2. A two-loop position control system is shown below.
1 1
TF = H ( s ) = + .
R(s) + + 1 Y(s) s +1 s + 2
− − s(s + 1) 2s + 3
C (s) = R( s)
( s + 1)( s + 2)
Ks (2 s + 3)
= .
Tacho-generator      s ( s + 1)( s + 2)
A B C
By partial fraction, = + + .
The gain k of the tacho-generator influences mainly s s +1 s + 2
the [2011] 3 −1
(a) peak overshoot −1
(b) natural frequency of oscillation C (s) = 2 + + 2 .
s s +1 s + 2
(c) phase shift of the closed loop transfer function
at very low frequencies (ω → 0) 3 1 
C (t ) =  − e − t − e −2t  4(t ).
(d) phase shift of the closed loop transfer function       2 2 
at very high frequencies (ω → ∞) Hence, the correct option is (c).

Chapter 04.indd 18 11/3/2015 3:38:55 PM


Chapter 4  Time Response Analysis | 5.19

4. The unit-step response of a unity feedback system with Solution: (a)


K 1
open loop transfer function G ( s ) =  is TF = .
( s + 1)( s + 2) s 2 + 3s + 2
shown in the figure. The value of K is [2009] Impulse input is applied at t = 1sec.

1
∴  Input 
= δ (t − 1).
L[δ (t − 1)] = e − s .
0.75
 1  −s
Resnose

0.5
Steady state/f  Final value = Lt s  2  e = 0.
s →0
 s + 3s + 1 
0.25 Hence, the correct option is (a).
6. The transfer function of a system is given as
0
0 1 2 3 4 100
.  The system is[2008]
Time (s) s 2 + 20 s + 100
(a) 0.5 (b) 2 (a) an over-damped system
(c) 4 (d) 6 (b) an under-damped system
Solution: (d) (c) a critically damped system
(d) an unstable system
K 1 Solution: (c)
G (s) = R( s) = .
( s + 1)( s + 2) s
100
TF = 2 .
Close loop transfer function s + 20 s + 100

G (s) K 2ξ un = 20  and  ωn2 = 100.
= . (1)
1 + G ( s ) ( s + 1)( s + 2) + K
20
From given graph steady state value = 0.75   ξ = 2 × 10 .
Thus, from Equation (1) steady state value to input
will be ξ = 1,    ωn = 10.

= lim s ⋅ (Cs ) Hence, critically damped system option (c).


s →0
Hence, the correct option is (c).
 K 
= lim s ⋅   R( s)
Common Data for Questions 7 and 8:
s →0
 ( s + 1)( s + 2) + K  The sate space equation of a system is described by
K x = Ax + Bu, y = Cx where x is state vector, u is input,
= . (2) 0 1  0
(2) + K y is output and A =   , B =   C = [1 0].
From Equations (1) and (2)  0 −2  1
7. The transfer function G(s) of this system will be
K
= 0.75 ⇒ K = 6.  [2008]
2+ K
s s +1
Hence, the correct option is (d). (a) (b)
( s + 2) s ( s − 2)
5. The transfer function of a linear time invariant
1 s 1
system is given as G ( s ) = 2 (c) (d)
.  The steady ( s − 2) s ( s + 2)
s + 3s + 2
state value of the output of the system for a unit Solution: (d)
impulse input applied at time instant t = 1 will be TF = C[ SI − A]−1 B + D
 [2008]
CAd j[ Si − A]
(a) 0 (b) 0.5 = B + D.
(c) 1 (d) 2 SI − A

Chapter 04.indd 19 11/3/2015 3:38:59 PM


5.20 | Control Systems

K1/s
 s 0  0 1   s −1 
  SI − A = 0 s  − 0 −2  = 0 s + 2  . Z w2
     
R(s) Kp s2 + 2xws + w2
 s + 2 1
Adj[ SI − A] =  .
 0 s  +1

SI − A = s 2 + 2 s. K1 /s
TF =
(−k pω )2
(− K1 /sω 2 )
 s + 2 1  0  1 1− −
[1 0]     [1 0]   s 2 + 2ξω s + ω 2 s 2 + 2ξω s + ω 2
 0 s  1  s .
TF = = K1 /s ( s 2 + 2ξω s + ω 2 )
s ( s + 2) s ( s + 2) = .
K
1 s 2 + 2ξω s + ω 2 + k pω 2 + 1 ω 2
TF = . s
s ( s + 2)
Steady state error
Hence, the correct option is (d). Z (∞) = lim SZ ( s )
s →0
8. A unity feedback is provided to the above system
G(s) to make it a closed loop system as shown in K1 /s ( s 2 + 2ξω s + ω 2 ).s
= lim s ⋅
figure. s →0 s ( s 2 + 2ξω s + ω 2 ) + k pω 2 s + K1ω 2
For a unit step input r (t), the steady state error in  K1 ( s 2 + 2ξω s + ω 2 ) 1
the output will be [2008] = lim s ⋅  2 
 s ( s + 2ξω s + ω ) + k pω s + K1ω  s
s →0 2 2 2

r (t) y(t)
Σ G(s) K1 (ω 2 )
= = 1.
0 + 0 + K1ω 2
Hence, the correct option is (a).
(a) 0 (b) l 1 0. R–L–C circuit shown in figure
(c) 2 (d) ∞
R = 10 W L = 1 mH
Solution: (a)
For type 1 system with unit step input the steady
state error is zero. +
ei Vo C = 10 µF
Hence, the correct option is (a).

9. Consider the feedback system shown below which
is subjected to a unit step input. The system is stable
and has following parameters kp = 4, k1= 10, ω = 500 For a step input ei, the overshoot in the output eo
and ξ = 0.7. The steady state value of z is [2007] will be [2007]
(a) 0, since, the system is not under damped
1 K1
(b) 5%
S (c) 16%
z
0
Kp
w2 (d) 48%
S S
s2 + 2xws + w2 Solution: (c)
 
1 1  ei ( s ) 
eo ( s ) = I (s) =  .
(a) 1 (b) 0.25 CS CS  R + LS + 1 
(c) 0.1 (d) 0  CS 
Solution: (a) e ( s) 1
TF = o = .
Z (s) ei ( s )  2 R 1 
TF = . LC  s + s +
R( s)  L LC 

Chapter 04.indd 20 11/3/2015 3:39:01 PM


Chapter 4  Time Response Analysis | 5.21

R 1 = T 1
Characteristic equation = s 2 + s+ . f 300
= Hz = 1.6 m sec (4)
L LC 2 (300)(2)
1
ωn2 = , L = 1 mH, C = 10 µF, R = 10 Ω, ωn = 104. T 1
LC =f 2=
KHz = 0.25 m sec (5)
2 (2)(2)103
R
2ξωn = . T  Settling time.
L ≥
2
R C 10 10 × 10−6 ∴  From Equations (2) and (4), it is clear that square

ξ= = .
2 L 2 10−3 wave of frequency f = 300 Hz can be used.
1
ξ= .
2
 −ξπ 
Peak overshoot = exp   = 16%.
 1− ξ 2 
 
T T
Hence, the correct option is (c).
2 2
11. R–L–C circuit shown in figure
R = 10 W L = 1 mH Hence, the correct option is (c).
1 2. When subjected to a unit step input, the closed
loop control system shown in the figure will have a
ci C = 10 µF
steady state error of [2005]

X(s )

2 Y(s )
If the above step response is to be observed on a + 3/s +
R(s ) − ( s + 2)
non-storage CRO, then it would be best have the ei
as a [2007]
(a) step function (a) −1.0 (b) −0.5
(b) square wave of 50 Hz (c) 0 (d) 0.5
(c) square wave of 300 Hz
(d) square wave of 2.0 KHz Solution: (c)
3 2 2
Solution: (c) ⋅ −
Y (s) s s + 2 s + 2
1 = .
ωn2 = . R( s)  −3 2 
LC 1−  ⋅
 s s + 2 
R C
ζ = . Y ( s) 6 − 2s
2 L = 2 .
R( s) s + 2s + 6
ωn = 104 , ζ = 0.5.
Final value to a step input = Lt SY ( s )
4 4 s →0
Settling time ts = = = 0.8 m sec (1)
ζωn (0.5)(104 )  6 − 2s  1
         = Lt s  2  = 1.
Frequency of the given step signal DC. s →0
 s + 2s + 6  s
1 Input = 1, Output = 1.
f = 0, T= =∞ (2) ∴  Error = 1 - 1 = 0.
0
T 1 Hence, the correct option is (c).
f = 50 Hz, = .
2 (50)(2) 1 3. The block diagram of a closed loop control sys-
T tem is given by figure. The values of K and P such
  = 0.01sec  (3) that the system has a damping ratio of 0.7 and an
2

Chapter 04.indd 21 11/3/2015 3:39:03 PM


5.22 | Control Systems

undamped natural frequency ωn of 5 rad/sec, are d 2 x 1 dx 1


respectively equal to [2004] + + x = 10 + 5e − 4t + 2e −5t ).  The natu-
dt 2 2 dt 18
ral time constants of the response of the system
R(s ) K C(s )
+ are [2003]
− s(s + 2 )
(a) 2s and 5s (b) 3s and 6s
(c) 4s and 5s (d) l/3s and l/6s
1 + sP Solution: (b)
d 2 x 1 dx 1
(a) 20 and 0.3 (b) 20 and 0.2 + + x = 10 + 5e − 4t + 2e −5t .
(c) 25 and 0.3 (d) 25 and 0.2 dt 2 2 dt 18
Solution: (d) Right hand side of the equation is the input (or)
Given data ζ = 0.7 and ωn = 5 rad/sec. excitation.
Taking the LT of the left hand side of the equation
k
G (s) = and H ( s ) = 1 + SP. gives
s ( s + 2) 1 1
s 2 x( s ) + sx( s ) + x( s ) = 0.
Characteristic equation = 1 + G ( s ) H ( s ) = 0. 2 18
k 1 1
1+ (1 + sp ) = 0. s 2 + s + = 0   Characteristic equation.
s ( s + 2) 2 18
s 2 + (kp + 2) s + k = 0.  1  1
 s +   s +  = 0.
ωn2 = k = 25.   3  6
2ζωn = (kp + 2). 1 1
Poles are − and − .
3 6
2ζω
ωn − 2 2(0.7)5 − 2
p= = = 0.2. 1 1
k 25 ∴   Time constants
= are 3=
sec and 6 sec.
1 1
k = 25.
3 6
p = 0.2. (3 sec, 6 sec)
Hence, the correct option is (d). Hence, the correct option is (b).
1 4. The unit impulse response of a second order under 1 6. The block diagram shown in figure given is a unity
damped system starting from rest is given by c (t) feedback closed loop control system. The steady
= 12.5e−6t sin 8t, t ≥ 0. The steady state value of the state error in the response of the above system to
unit step response of the system is equal to [2004] unit step input is [2003]
(a) 0 (b) 0.25 u (t ) + 3 15 y(t )
(c) 0.5 (d) 1.0 − ( s + 15 ) (s + 1)
Solution: (d)
IR c(t ) = 12.5 e −6t sin 8t u (t ).
(a) 25% (b) 0.75%
(12.5)(8) C (s) (c) 6% (d) 33%
TF = L[ IR ] = = .
( s + 6) + 8
2 2
R( s) Solution: (a)
Final value or steady state value to a step input 3 15
G (s) = . and H ( s ) = 1.
 (12.5)(8)  1 s + 15 s + 1
= Lt SC ( s ) = Lt S  2 
= 1.
 ( s + 6) + 8  s (3)(15)
s →0 s →0 2
K p = Lt G ( s ) = = 3.
s →0 (15)
Hence, the correct option is (d).
1 1
1 5. A control system with certain excitation is gov- Steady state error = =
erned by the following mathematical equation 1+ K p 1+ 3

Chapter 04.indd 22 11/3/2015 3:39:06 PM


Chapter 4  Time Response Analysis | 5.23

1 R(s ) + K C(s )
=
= 0.25 = 25%.
4 − s(s + 2 )
Hence, the correct option is (a).
17. The roots of the closed loop characteristic equa- 1 + as
tion of the system shown in figure is [2003]
u (t ) + 3 15 y(t ) (a) K = 4, a = 0.35 (b) K = 8, a = 0.455
− ( s + 15 ) (s + 1) (c) K = 16, a = 0.225 (d) K = 64, a = 0.9
Solution: (c)
Characteristics equation = 1 + G ( s ) H ( s ) = 0.
− l and − 15
(a) (b) 6 and 10
− 4 and − 15 (d)
(c) − 6 and − 10 K
1+ (1 + as ) = 0.
Solution: (d) s ( s + 2)
(3)(15) s 2 + 2 s + kas + k = 0
G ( s) = and H ( s ) = 1.
( s + 15)( s + 1) ⇒ s 2 + (ka + 2) s + k = 0
Characteristic Equation = 1 + G ( s ) H ( s ) ⇒ s 2 + 2ζωn s + ω 2 n = 0.
3(15)
    = 1 + ( s + 15)( s + 1) = 0. Comparing the like terms

s 2 + 16 s + 15 + 45 = 0. ωn2 = k = (4) 2 = 16.


s 2 + 16 s + 60 = 0. ∴ k = 16.
( s + 6)( s + 10) = 0. ka + 2 = 2ζωn = 2(0.7)(4).
Roots are − 6 and −10 2(0.7)( 4) − 2
a= = 0.2225.
Hence, the correct option is (d). 16
18. A unity feedback system has open loop transfer Hence, the correct option is (c).
25 2 0. The unit impulse response of a system is given as
function G ( s ) = . 
The peak overshoot in
s ( s + 6) c (t) = − 4e−t + 6e−2t. The step response of the same
the step-input response of the system is approxi- system for t ≥ 0 is equal to [1996]
mately equal to [2000] (a) −3e − 4e + l
−2t −t
(b) − 3e + 4e−t − 1
−2t

(a) 5% (b) 10% (c) − 3e − 4e − 1


−2t −t
(d) 3e−2t + 4e−t − 1
(c) 15% (d) 20% Solution: (b)
Solution: (b) IR = − 4e − t + 6e −2t .
25
Characteristic equation = 1 + = 0. −4 6
s ( s + 5) TF = L[ IR] = +
s +1 s + 2
s 2 + 6 s + 25 = s 2 + 2ζωn s + ω 2 n = 0. − 4( s + 2) + 6( s + 1)
= .
ωn = 5, 2ζωn = 6. ( s + 1)( s + 2)
6 C (s) 2( s − 1)
ζ = = 0.6. = .
2ωn R( s ) ( s + 1)( s + 2)
R(s)
Maximum or peak over shot = e − (ζπ / 1− ζ ) = 10%
2
2( s − 1)
Step response = C ( s ) =
Hence, the correct option is (b). ( s + 1)( s + 2) R(s)=
1
s
19. For the system shown in figure, with a damping
C (t ) = L−1[C ( s )] = −3e −2t + 4e − t − 1.
ratio ξ of 0.7 and an undamped natural frequency
ωn of 4 rad/sec, the values of K and a are [1996] Hence, the correct option is (b).

Chapter 04.indd 23 11/3/2015 3:39:09 PM


5.24 | Control Systems

21. A first order system and its response to a unit Solution:


step input are shown in figure below. The system 1
= 0=
From the figure Time constant  .2 .
parameters are [1991] (a)
a= a = 5.
b=
From the TF the final value c (∞)
c (t )
c(∞) = Lt sC ( s )
s →0
2.0
k k
c (t )
= Lt s ⋅ R( s) = .
s →0 s+a R(s)=
1 a
s

k
From the figure final value = c(∞) = 2 = .
0 0.2 t sec a
r(t ) K c(t ) k 2=
= a 10.
s+a

Chapter 04.indd 24 11/3/2015 3:39:10 PM


Chapter 4  Time Response Analysis | 5.25

Five-marks Questions  3   7 
− 2
1   5   = 1− 5 
1 
= 1 + 
5 3 3
1. Single input single output system with y as output s−  5 s− 
and u as input, is described by  5   5

d2y dy du 1 7 53 t 
+ 2 + 10 y = 5 − 3u. =  s (t ) − e .
dt 2 dt dt 5 5 
For the above system find an input u(t) with zero
initial condition, that produces the same output 2. Match the following [1995]
as with no input and with the initial conditions.
Root locations of the characteristic equations of
dy (0− )
= − 4, y (0− ) = 1.  [2002] second order systems.
dt
(A) jw (P) c(t )
Solution:
  d 2 y 2dy 5du s-Plane
+ + 10 y = − 3u; × × s
dt 2
dt dt 1.0

dy (0 ) t
= y1 (0− ) = 4 and y (0− ) = 1.
dt (B) jw (Q) c(t )
By Laplace transform [s2y (s) - sy (0-) - y1 (0)] + s-Plane
2[sy (s) - y (0-)] + 10y (s) = 0 as no input u = 0. ×
1.0
s
− − −
( s + 2 s + 10) y ( s ) = s y (0 ) + y (0 ) + 2 y (0 )
2 1
×
t
= s + (− 4) + 2(1) = s − 2.
(C) jw (R)
s−2 c(t)
  y(s) = (1)
s 2 + 2 s + 10 × 1.0
Now, s
t
From equation ×
s 2 y ( s ) + 2 y ( s ) + 10 y ( s ) = 5 u ( s ) − 3 u ( s ).
(D) jw (S) c(t)
y(s) 5s − 3
  = 2 (2)
u ( s ) s + 2 s + 10 s-Plane ×
As, 1.0
s
Both output must be equal thus Equations (1) and (2) t
×
s−2 5s − 3
= 2 u ( s ).
s + 2 s + 10 s + 2 s + 10
2
(E) jww (T) c(t)

  s-Plane 1.0
s−2 1s−2
u (s) = =   × × s
5s − 3 5  s − 3  t
 5
 3 3 
s − + − 2
1 5 5 Unit step responses of second order systems
=  
5 3  Solution:
  s −
 5  A - T,  B - S,  C - P,  D - R,  E - Q

Chapter 04.indd 25 11/3/2015 3:39:12 PM


Chapter 5
Root Locus Diagram
Im
One-mark Questions
2√3
1. The root locus of a unity feedback system is shown √3
in the figure Re
−3 −2 −1 1 2 3
jw −2√3
K=0 K=0 −√3

−2 −1 s
K K
(a) (b)
s 3
s 2
( s + 1)
The closed loop transfer function of the system is
 [2014-S1] K K
(c) 2 (d)
s ( s + 1) s ( s 2 − 1)
C (s) K
(a) =
R( s ) ( s + 1)( s + 2) Solution: (a)
All three poles are at origin and has three root
C (s)
(b) −K k
= ­locus coming out. Hence option (a)  .
R( s ) ( s + 1)( s + 2) + K s2
Hence, the correct option is (a).
C (s)
(c) K 3. Which of the following figure(s) represent valid root
=
R( s ) ( s + 1)( s + 2) − K loci in the s-plane for positive K ? Assume that the
system has transfer function with real coefficient.
C (s)
(d) K  [1992]
=
R( s ) ( s + 1)( s + 2) + K (a)  (b) 
Solution: ×
× × × ×
C (s) K ×
= .
R ( s ) ( s + 2)( s + 1) + k
(c)  (d) 
Hence, the correct option is (c). ×
2. Figure shows the root locus plot (location of poles × ×
not given) of a third order system whose open loop ×
transfer function is [2005]

Chapter 05.indd 26 11/9/2015 6:34:38 PM


Chapter 5  Root Locus Diagram  |  5.27

Solution: (a) (a) three roots with nearly equal real parts exist on
By using symmetry conditions and root locus dia- the left half of the s-plane
gram option (a) is correct answer. (b) one real root is found on the right half of the
Hence, the correct option is (a). s-plane
(c) the root loci cross the jω axis for a finite
4. A unity feedback system has an open-loop trans-
value of k; k ≠ 0
K ( s + a)
fer function of the form KG ( s ) = 2 ; b > a. (d) three real roots are found on the right half; of
s ( s + b) the s-plane
Which of [1991]
(a)   (b)  Solution: (a)
jw jw
 2
ks+ 
s s
G (s) = 2 3
.
× ×
−b −a 0 −b −a 0 s ( s + 2)

2
0−2−
Centroid = 3 = − 2.
(c)   (d)  s −1 3
jw jw
Angle of asymptote = ±90°.
s s
× × Number of asymptotes = 3 - 1 = 2.
−b −a 0 −b −a 0
As all three roots are left side of s-plane and have
equal real parts.

Solution: (a)
jw

× ×
−2 2

× ×× 3
−b −a s

Hence, the correct option is (a).


(P) Poles = 3 2. The characteristic equation of a closed-loop sys-
(Z) Zero = 1 tem is s(s + l)(s + 3) + k(s + 2) = 0, k > 0. Which of
Σ poles − Σ zero the following statements is true? [2010]
Centroid (σ ) =
P−Z (a) Its roots are always real
0−b+ a (b) It cannot have a breakaway point in the range
= .
   P − Z −1 < Re[s] <0
This gives negative value. (c) Two of its roots tend to infinity along the as-
ymptotes Re[s] = −1
Hence, the correct option is (a).
(d) It may have complex roots in the right half
plane
Two-marks Questions Solution: (c)

1. The open loop transfer function G(s) of a unity feed- ( s + 1)( s + 3) + k ( s + 2) = 0.


 2 k ( s + 2)
ks+ 
back control system is given as, G ( s ) = 2
3
.  1 + s ( s + 1)( s + 3) = 0.
s ( s + 2)
From the root locus, it can be inferred that when k k ( s + 2)
G (s) H (s) = .
tends to positive infinity [2011] s ( s + 1)( s + 3)

Chapter 05.indd 27 11/9/2015 6:34:40 PM


5.28 | Control Systems

Poles = 3. Solution: (b)


Zero = 1.
( s 2 − 4)( s + 1) + k ( s − 1) = 0.
Number of asymptote = 3 - 1 = 2, angle of asymp-
tote = ± 90°. k ( s − 1)
0 −1− 3 + 2 1+ = 0.
Centroid = = −1. ( s + 1)( s 2 − 4)
3 −1
The root locus will break somewhere between −1 Characteristics equation is given by 1 + G(s) H(s) = 0.
to 0 and tends to approach to infinity. k ( s − 1)
Compare with it G ( s ) H ( s ) = .
jw ( s + 1)( s 2 − 4)
Poles = 3.
Zero = 1.
Number of asymptotes = Poles – Zero = 3 – 1 = 2.
s −1 − 2 + 2 − 1
−3 −2 −1 Centroid = = −1.
3 −1
Angle of asymptotes = ± 90°.

jw
Hence, the correct option is (c).
3. A closed loop system has the characteristic func-
tion (s2 − 4)(s + l) + K(s − 1) = 0. Its root locus plot × × ×
−2 −1 +1 +2 s
against K is [2006]
(a) jw

Hence, the correct option is (b).


× × ×
−2 −1 +1 +2 s 4. A unity feedback system has an open loop transfer
K
function, G ( s ) = 2 . The root locus plot is [2002]
s
(b) jw (a)  jw  (b)  jw

s s
× × ×
−2 −1 +1 +2 s

(c) jw  (d)  jw
(c) jw

s s
× × ×
−2 −1 +1 +2 s

Solution: (b)
Poles = 2.
(d) jw
(2q + 1)180°
Angle of asymptotes =
P−Z
× × × 180
−2 −1 +1 +2 s =± = ± 90°.
2
Hence, the correct option is (b).

Chapter 05.indd 28 11/9/2015 6:34:41 PM


Chapter 5  Root Locus Diagram  |  5.29

(iv) Point of intersection with imaginary axis is


Five-marks Questions given by RH criteria.
Character equation 1 + G(s)H(s) = 0.
1. The open loop transfer function of a unity feed-
2( s + α ) s3 + 12s2 + 22s + 2a = 0.
back system is given by G ( s ) = .
s ( s + 2)( s + 10) s3 1 22
Sketch the root locus as a varies from 0 to ∞. Find
the angle and real axis intercept of the asymptotes, s 2
12 2α
breakaway points and the imaginary axis crossing 12(22) − 2α
points, if any. [2002] s1
12
Solution:   s0 2α
2( s + α )
(s) = . 12(22) − 2α
s ( s + 2)( s + 10) Thus, = 0.
12
Characteristic equation 1 + G ( s ) H ( s ) = 0. 2α = 12 × 22.
2( s + α ) α = 132.
1+ = 0.
s ( s + 2)( s + 10) 12 s 2 + 2α = 0 → Auxiliary equation.
∴ s ( s + 2)( s + 10) + 2( s + α ) = 0.
∴ 12 s 2 + 2(132) = 0.
∴ s ( s 2 + 12 s + 20 + 2) + 2α = 0.
∴ s ( s 2 + 12 s + 22) + 2α = 0. s = ± j 4.69.
2α jw
∴ 1+ = 0.
s ( s 2 + 12 s + 22)
j 4.69

∴ G (s) H (s) = .
s ( s + 12 s + 22)
2
s
−9.74 −4 −2.26
−12 ± 144 − 88 −12 ± 7.8
Pole → S = = . j 4.69
2 2
Poles = 3 (0, −2.26, −9.74).
−9.74 − 2.26 2. Given the characteristic equation s3 + 2s2 + Ks + K
(i)  Centroid = = −4 = 0. Sketch the root locus as K varies from zero to
3
infinity. Find the angle and real axis intercept of
(2q + 1)180° the asymptotes, break-away/break-in points, and
(ii)  Angle of asymptotes =
P−Z imaginary axis crossing points, if any. [2001]
θ = 60°, 180°, −60°
Solution:
(iii)  Break point
s 3 + 2 s 2 + ks + k = 0.
d  2 
 2  = 0. k ( s + 1)
ds  s ( s + 12 s + 22)  1+ = 0.

( s3 + 2s 2 )
( s 3 + 12 s 2 + 22 s )(0) − 2(3s 2 + 24 s + 22)
= 0. k ( s + 1)
( s 3 + 12 s + 22 s ) 2 G ( s) H (s) = .
s 2 ( s 2 + 2)
∴ 3s 2 + 24 s + 22 = 0. Poles = 3.
s = −1.06. Zero = 1.
s = −6.94. Real axis intercept = centroid, (σ)

Chapter 05.indd 29 11/9/2015 6:34:44 PM


5.30 | Control Systems

     0 + 0 − 2 +1 −1
   φ = cos (0.3).
1
= =− .
3 −1 2  ∴ φ = 72°.
(2q + 1)180° The k for ξ = 0.3 does not exist as 72° line does
Angle of asymptotes = . not interest the root locus diagram.
P−Z
q = 0; = 90°.
   q = 1; = 270°(−90°). 72°
No break away point. × ×
−5 −2 −1
jw

4. A unity feedback system has the forward loop


× ×
−2 −1 s K ( s + 2) 2
1 transfer function KG ( s ) = 2 . [1991]
s=−
2 s ( s − 1)
(a) Determine the range of K for stable operation
(b) Determine the imaginary axis crossover points
3. A unity feedback system has open-loop transfer (c) Without calculating the real axis break-away
K ( s + 5) points, sketch the form of root loci for the system
function G ( s ) = ; K ≥ 0.  [2000]
s ( s + 2) Solution:
(a) Draw a rough sketch of the root locus plot; (a) Characteristic equation is 1 + G(s)H(s) = 0
given that the complex roots of the character- K ( s + 2) 2
istic equation move along a circle Given H ( s )G ( s ) = 2 .
s ( s − 1)
(b) As K increases, does the system become less
stable? Justify your answer K ( s + 2) 2
(c) Find the value of K (if it exists) so that the 1+ = 0.
s 2 ( s − 1)
damping ζ of the complex closed loop poles
is 0.3 s 2 ( s − 1) + K ( s + 2) 2 = 0.
Solution: s 3 + (− s 2 ) + Ks 2 + 4 Ks + 4 K = 0.
K ( s + 5)
G ( s) = ; K ≥ 0. s 3 + ( K − 1) s 2 + 4 Ks + K = 0.
s ( s + 2)
(a) Root Locus s3 1 4K
As given complex root of the characteristic s2 K −1 4K
equation move along a circle. ( K − 1)4 K − 4 K
s1
K −1
s0 4K

K=0 K=0 K=0 As, → K - 1 > 0 ⇒ k > 1 and 4K > 0 ⇒ K > 0


× ×
−5 −2 0
( K − 1)4 K − 4 K
→ > 0.
K −1
(b) If K increases, the stability will increase be- ∴ ( K − 1) 4 K − 4 K > 0.
cause root locus is moving toward the left side. ∴ K − 1 − 1 > 0.
(c) ξ = 0.3.
∴ K − 2 > 0.
∴ cos φ = ξ . K > 2.
  
∴ φ = cos −1 (ξ ). Range will be for stability is K > 2.

Chapter 05.indd 30 11/9/2015 6:34:45 PM


Chapter 5  Root Locus Diagram  |  5.31

(b) Imaginary axis cross over points (c) jw


j 2√2
By using auxiliary equation (s2 row = 0)
∴ ( K − 1) s 2 + 4 K = 0 ( K = 2). × ×
s
−2 1
∴ s + 8 = 0.
2
−j2√2
s = ± j 2 2.

Chapter 05.indd 31 11/9/2015 6:34:45 PM


Chapter 6
Frequency Response
Analysis
39.8
One-mark Questions 20 log
10n
= −8 ⇒ log 39.8 − log 10n = 0.4

39.8
1. The Bode plot of a transfer function G(s) is shown ⇒ = 10−0.4
in the figure below [2013] 10n
10n = 100
40 n = 2.
32
Gain (dB)

20 Hence, the correct option is (b).


( s 2 + 9 ) ( s + 2)
0 2. A system with transfer function
1 ( s + 1) ( s + 3) ( s + 4)
−8 10 100
w (rad/s) is excited by sin ( ω  t). The steady-state output of
the system is zero at [2012]
The gain (20 log| G(s) |) is 32 dB and −8 dB at
(a) ω = 1 rad/s (b) ω = 2 rad/s
1 rad/s and 10 rad/s respectively. The phase is neg-
(c) ω = 3 rad/s (d) ω = 4 rad/s
ative for all ω. Then G(s) is
39.8 39.8 Solution: (c)
(a) (b)
s s 2 ( s 2 + 9)( s + 2)
G ( s) = .
32 32 ( s + 1)( s + 3)( s + 4)
(c) (d)
s s2 r (t ) = sin (ωt ).
Solution: (b) (−ω 2 + 9)( jω + 2)
G ( jω ) = .
k ( jω + 1)( jω + 3)( jω + 4)
G ( jω ) =  (Suppose).
( jω ) n
(9 − ω 2 ) ω 2 + 4
Gain = dB at w = 1 rad/sec. G ( jω ) = = 0.
(ω 2 + 1)(ω 2 + 9)(ω 2 + 16)
k
20 log n = 32 (9 − ω 2 ) ω 2 + 4 = 0
1
20 log k = 32 9 −ω2 = 0
k = 39.8. ω 2 = 9. ω = 3 rad/sec.
Gain = -8 dB at w = 10 rad/sec. Hence, the correct option is (c).

Chapter 06.indd 32 11/9/2015 5:15:26 PM


Chapter 6  Frequency Response Analysis | 5.33

3. The frequency response of a linear system G ( jω) ω  ω 


is provided in the tabular form below. ∠G ( jω ) = − tan −1 (ω ) − tan −1   − tan −1   .
2 3
| G( jω) | 1.3 1.2 1.0 0.8 0.5 0.3 ∠G ( jω ) ω =∞ = −90°° − 90° − 90° = −270°.
∠G( jω) −130° −140° −150° −160° −180° −200° ∠G ( jω ) ω =∞ ≠ − [ P − Z ] 90°.
The gain margin and phase margin of the system are   
[2011] ∴  Non-minimum phase function.
(a) 6 dB and 30° (b) 6 dB and −30° Hence, system is unstable and non-minimum phase
(c) −6 dB and 30° (d) −6 dB and −30° function.
Solution: (a) Hence, the correct option is (d).
At gain cross over frequency (wgc) magnitude 5. The polar plot of an open loop stable system is
G( jw) = j. shown below. The closed loop system is [2009]
G ( jω gc )= 1.
Phase G( jw) = ∠ G( jwgc) = -150°.

Imaginary
PM = 180° + φ
= 180° − 150°.
w=¥
PM = 30°. 1.42 Real
At phase cross over frequency ( wpc) ⋅ ∠ G( jw) = -180°
at wpc = | G( jw) | = 0.5.
1 1 w=¥
GM = 20 log = 2 log .
G ( jω ) 0.5
(a) always stable.
GM = 6 dB. (b) marginally stable.
Hence, the correct option is (a). (c) unstable with one pole on the RH s-plane.
(d) unstable with two poles on the RH s-plane.
4. An open loop system represented by the transfer
( s − 1) Solution: (d)
function G ( s ) = is [2011] So the given open loop system is stable
( s + 2)( s + 3)
(a) stable and of the minimum phase type. ∴  P = 0.
(b) stable and of the non-minimum phase type. w=0
(c) unstable and of the minimum phase type.
(d) unstable and of the non-minimum phase type.
Solution: (d)
w=¥
( s − 1) −1.42
G ( s) = .
( s + 2)( s + 3)
There is zero on the RHS of s-plane
Z =1 P=0 N = P−Z w=0

N = 0 −1 Number of encirclements = −2.


N = −1 N = −2.
Hence, system is unstable. N = P − Z.
( jω − 1) −2 = 0 − Z .
G ( jω ) = .
( jω + 2)( jω + 3) Z = 2.
− tan −1 (ω ) Z = Number of closed loop poles in RHS of s-plane.
∠G ( jω ) = .
ω  ω  Hence, the system is unstable with Z = 2.
tan −1   + tan −1  
2 3 Hence, the correct option is (d).

Chapter 06.indd 33 11/9/2015 5:15:27 PM


5.34 | Control Systems

6. A system with zero initial conditions has the I closed Put wpc = 0 in | G( jwpc) |.
s2 + 4
loop transfer function T ( s ) = . 1
( s + 1)( s + 4) G ( jω pc )== ∞.
0
The system output is zero at the frequency [2005] x = ∞.
(a) 0.5 rad/sec (b) 1 rad/sec
1 1
(c) 2 rad/sec (d) 4 rad/sec GM = = = 0.
x ∞
Solution: (c)
GM = 0.
s2 + 4 Hence, the correct option is (a).
T ( s) = .
( s + 1)( s + 4)
8. The Nyquist plot of loop transfer function G(s)H(s)
C (s) s2 + 4 of a closed loop control system passes through the
= .
R ( s ) ( s + 1)( s + 4) point (-1, j0) in the G(s)H(s) plane. The phase mar-
gin of the system is [2004]
Put s = jw.
(a) 0° (b) 45°
j 2ω 2 + 4 (c) 90° (d) 180°
T ( jω ) = .
( jω + 1)( jω + 4) Solution: (a)
4 −ω2 Img
T ( jω ) = .
(1 + jω )(4 + jω )
4 −ω2
T ( jω ) = . wg
(1 + ω 2 )(16 + ω 2 ) Real
(−1, 0) 1
4 −ω 2
= 0. 4 − ω 2 = 0.
(1 + ω )(16 + ω )
2 2

w2 = 4  w = 2 rad/sec.
Hence, the correct option is (c). wg is gain cross over frequency at which gain
| G( jw)H( jw) | becomes unity.
7. The gain margin of a unity feedback control system
( s +1) In this case, ∠G( jw)H( jw) = -180° at w = wg
with the open loop transfer function G ( s ) = is φ = −180°.
s2
[2005] So, phase margin = 180° + φ
1 = 180° − 180° = 0.
(a) 0 (b)
2 Hence, the correct option is (a).
(c) 2 (d) ∞
9. The polar plot of a type-1, 3-pole, open-loop system
Solution: (a) is shown in figure below. The closed-loop system is
( s + 1)  [2001]
G ( s) = .
s2
GH = plane
( jω + 1) 1+ ω2
 G ( jω ) = G ( jω )= .
( jω ) 2 ω2
−1.42 w=¥
∠G( jw) = -180° + tan–1 (w).
For wpc, ∠G( jwpc) = -180° w=0
−1
−180° = −180° + tan (ω pc ). (a) always stable.
−1
tan (ω pc ) = 0. (b) marginally stable.
(c) unstable with one pole on the right half s-plane.
ω pc = 0. (d) unstable with two poles on the right half s-plane.

Chapter 06.indd 34 11/9/2015 5:15:28 PM


Chapter 6  Frequency Response Analysis | 5.35

Solution: (d) There is pole at w = 25.


y = mx + c.
G-H plane
c = 20 log k.
From the bode plot
−1 s4 = − 40(log (0.1)) + 20 log k .
−1.42 w =¥
∴  k = 5.
s (1 + 0.2 s )
Now, TF = .
s (1 + 0.5s )(1 + 0.04 s )
2

w=0 50( s + 5)
TF = 2 .
By the Nyquist plot, number of encirclement of s ( s + 2)( s + 25)
(-1, 0) = -2 and number of poles on right side = 0. Hence, the correct option is (d).
P = 0.
11. The closed-loop transfer function of a control system
N = P − Z.
C (s) 1
−2 = 0 − Z . is given by = . For the input r (t) = sin t
R( s ) (1 + s )
∴ Z = 2. steady state value of c (t) is equal to [1996]
∴ System is unstable with two poles on the right 1
half s-plane. (a) cos t (b) 1
2
Hence, the correct option is (d). 1 1  π
(c) sin t (d) sin  t − 
10. The asymptotic approximation of the log-magni- 2 2  4
tude versus frequency plot of a minimum phase Solution: (d)
system with real poles and one zero is shown in
figure. Its transfer functions is [2001] C (s) 1
=
R( s ) (1 + s )
−40 dB/dec
54 dB
∴  r (t) = sin t.
−60 dB/dec So, w = 1.
−40 dB/dec
1 1 1
G ( jω ) = = = ∠ − 45°.
−60 dB/dec 1 + jω 1 + j 2
1 sin t
∴ C (t ) = R (t ) ∠ − 45° = ∠ − 45°.
0.1 2 5 25 rad/s 2 2
20( s + 5) 10( s + 5) 1
(a) (b) 2 C (t ) = sin(t − 45°).
s ( s + 2)( s + 25) ( s + 2) ( s + 25) 2
20( s + 5) 50( s + 5) Hence, the correct option is (d).
(c) 2 (d)
s ( s + 2)( s + 25) s ( s + 2)( s + 25)
2
12. A unity feedback system has the open loop trans-
Solution: (d) 1
fer function G ( s ) = .  The Nyquist
Starting slope -40 dB/dec. ( s − 1)( s + 2)( s + 3)
So, two poles at origin.  ∴ Type-2. plot of G encircle the origin [1992]
At w = 2, -20 dB/dec decrease in the slope. (a) never (b) once
(c) twice (d) thrice
∴  There is pole at w = 2.
At w = 5, +20 dB/dec. Solution: (b)
There is zero at w = 5. 1
G ( s) = .
At w = 25, -20 dB/dec. ( s − 1)( s + 2)( s + 3)

Chapter 06.indd 35 11/9/2015 5:15:30 PM


5.36 | Control Systems

N = P - Z. x = log ω c = 20 log k .
P = Number of open loop poles on RHS. ∴ 60 = −20 log (0.01) + 20 log k .
Z = Number of open loop zeros on RHS. 60 = 40 + 20 log k .
∴  Z = 0   P = 1. 60 − 40 = 20 log k .
N = 1 - 0. 10 = log k .
N = 1. ∴ k = 10.
Nyquist plot G(s) encircle the origin once. 10(1 + 10 s )
Hence, the correct option is (b). ∴ TF =
s (1 + 20 s )
13. The system having the Bode magnitude plot shown Hence, the correct option is (b).
in figure below has the transfer function  [1991]
14. Which of the following is the transfer function of
a system having the Nyquist plot shown in figure
below. [1991]
Im
60 −20 dB/dec

+Re
w 0′ +∞
0.01 0.05 0.1 1.0 100
−∞
0′

60( s + 0.01)( s + 0.1) 10(1 + 10 s )


(a) 2 (b)
s ( s + 0.05) 2 s (1 + 20 s )

3( s + 0.05) 5( s + 0.1) K K
(c) (d) (a) (b)
s ( s + 0.1)( s + 1) s ( s + .05) s ( s + 2) ( s + 5)
2
s ( s + 2)( s + 5)
2

K ( s + 1) K ( s + 1)( s + 3)
Solution: (b) (c) 2 (d)
s ( s + 2)( s + 5) s 2 ( s + 2)( s + 5)
Solution: (b)
Img

60 − 20 dB/dec

0′ +∞
w −∞ Real
0.01 0.05 0.1 1.0 100
0′

Starting slope is -20 dB/dec.


∴  There is one pole at origin.
∴ Type-1. As curve starts from -180° ⇒ type 2 system curve
ends at -360° difference between number of poles
Poles: 0, 0.05 [at w = 0.05 there is also decrease in 360°
slope]. and number of zeros = = 4.
90°
Zero: 0.1 [at w = 0.1 there is increase in slope] ∵  Only option (b) is satisfying these two properties.
   ∵ y = mx + c Hence, the correct option is (b).

Chapter 06.indd 36 11/9/2015 5:15:31 PM


Chapter 6  Frequency Response Analysis | 5.37

k = (10)0.9 = 7.94.
Two-marks Questions 1
a  
1. The Bode magnitude plot of the transfer function = 4 .
K (1 + 0.5s )(1 + as ) bk  1 
G ( s) = is shown below:   × 7 . 94
 s  s   12 
s 1 +  (1 + bs ) 1 + 
 8  36  a
= 0.3778.
Note that -6 dB/octave = -20 dB/decade. The bk
a
value of  is [2014-S1] 2. For the transfer function
bK
5( s + 4)
 G ( s) = . The values of the
s ( s + 0.25)( s 2 + 4 s + 25)
−6 dB/Octave 0 dB/Octave constant gain term and the highest corner frequ­
dB
6 dB/Octave ency of the Bode plot respectively are
−6 dB/Octave
0 dB/Octave  [2014-S2]
(a) 3.2, 5.0 (b) 16.0, 4.0
(c) 3.2, 4.0 (d) 16.0, 5.0
−12 dB/ Solution: 17
Octave
s ( s + 4)
0 G (s) = .
0.01 2 4 8 12 36 w(rad/s) s ( s + 0.25)( s 2 + 4 s + 25)
Solution:  5
5 × 4 1 + 
k (1 + 0.5s )(1 + as ) G ( s) =  4
G ( s) = .
 s  s   s    s 4s  
2
s 1 +  (1 + bs ) 1 +  
s 0.25  1 +     + + 1 
25
 8  36    0.25     25 25  
Pole addition (1 + as) slope = -20 dB/decade or 20(1 + 0.25s )
= -6 dB/octave. = .
s[0.25(1 + 4 s )][2 s (1 + 0.105 + 0.04 s 2 )]
Zero addition (1 + as) slope = +20 dB/decade or
3.2(1 + 0.25s )
= +6 dB/octave. G ( s) = .
1 1 s (1 + 4 s )(1 + 0.16 s + 0.04 s 2 )
=   a = rad/sec b rad/sec.
4 12 Value of constant gain term = 3.2.
Starting vertical line slope is -20 dB/dec.
Corner frequencies w = 4 rad/sec.
(0 − y )
−20 = . w = 0.25 rad/sec.
log(8) − log(0.01)
w = 5 rad/sec.
y
−20 = . Highest corner frequencies = 5 rad/sec.
log s + 2
Hence, the correct option is (a).
y = 58 dB.
1
y = mx + c. 3. The frequency response of G(s) =
[ s ( s + 1)( s + 2)]
58 = −20 log 0.01 + c. plotted in the complex G( jω) plane (for 0 < ω < ∞) is
58 = 40 + c. [2010]
c = 58 − 40 = 18. (a) −3 (b) w = 0 I
4 I m m
c = 20 log k .
18
= log k .
20
−3 Re
0.9 = log k.. w=0 4

Chapter 06.indd 37 11/9/2015 5:15:32 PM


5.38 | Control Systems

(c) Im (d) −1 Im Solution: (b)


w=0
6
80 dB −40 dB/dec
Re
−60 dB/dec
Re −40 dB/dec
−1
6 w=0
−60 dB/dec
Solution: (a)
0.1 2 5 25 rad/s
1
G (s) = .
s ( s + 1)( s + 2) At origin two poles present as -40 dB/dec slope
1 decrease.
G ( jω ) = .
jω ( jω + 1)( jω + 2) 80 = −40 log 2 + 1 + 20 log10 k .
40 = 20 log10 k .
Separate real and imaginary parts,
log10 k = 2.
− j (1 − jω )(2 − jω )
   G ( jω ) = . k = 100.
ω (ω 2 + 1)(ω 2 + 4)
At w = 2 rad/dec one pole present as -20 dB/dec
−3ω − j (2 − ω 2 ) slope decrease.
G ( jω ) = .
ω (ω 2 + 1)(ω 2 + 4) At w = 5 rad/dec one zero present as +20 dB/dec
−3ω (ω 2 − 2) slope increase.
G ( jω ) = + j .
ω (ω 2 + 1)(ω 2 + 4) ω (1 + ω 2 )(4 + ω 2 ) At w = 25 rad/dec one pole present as -20 dB/dec
Imaginary part = 0 when plot cuts to real axis. slope decrease.

3  s
100 1 + 
ω pc =
rad/sec.  5
4 TF = .
As type 1 and order 3 plot starts in 2nd quadrant 2 s  s 
s 1 +  1 + 
and terminate at 3rd quadrant.  2   25 
Hence, the correct option is (a). 1000( s + 5)
G ( s) = 2 .
s ( s + 2)( s + 25)
4. The asymptotic approximation of the log magni-
tude versus frequency plot of a system contain- Hence, the correct option is (b).
ing only real poles and zeros is shown. Its transfer
function is [2009] 5. The open loop transfer function of a unity feed-
(e −0.1s )
back system is given by G ( s ) = . The gain
s
−40 dB/dec margin of this system is [2009]
80 dB (a) 11.95 dB (b) 17.67 dB
−60 dB/dec
−40 dB/dec (c) 21.33 dB (d) 23.9 dB
−60 dB/dec Solution: (d)
e −0.15
G (s) = .
s
0.1 2 5 25 rad/s
Put s = jw.
10( s + 5) 1000( s + 5) e −0.1 jω
(a) (b) G ( jω ) = .
s ( s + 2)( s + 25) s ( s + 2)( s + 25)
2

100( s + 5) 80( s + 5) As, e − jωT = 1∠ − 57.3 ωT degree.
(c) (d)
s ( s + 2)( s + 25) s 2 ( s + 2)( s + 25)
So ∠G ( jω ) = −90° − 57.3 × 0.1 ω.

Chapter 06.indd 38 11/9/2015 5:15:34 PM


Chapter 6  Frequency Response Analysis | 5.39

∠G ( jω )ω pc = −180°. At next corner frequency one zero present, so +20


−180° = −90° − 57.3 × 0.1 ω. dB/dec slope add which results to 0 dB/dec slope.
90 Hence, 2 poles and 2 zeros contained by plot.
ω= . Hence, the correct option is (c).
5.73
ω = 15.7 rad/ssec. 7. If x = Re G( jω) and y = Im G( jω) then for ω → 0+,
1 1
G ( jω ) = . the Nyquist plot for G ( s ) = . [2007]
ω [ s ( s + 1)( s + 2)]
1 3
G ( jω pc ) = . (a) x = 0 (b) x = −
15.7 4
1 y
(c) x = y − (d) x=
1 6 3
GM = 20 log = 20 × log 15.7.
G ( jω pc ) Solution: (b)
GM = 23.91 dB. x = Re G( jw).
Hence, the correct option is (d). y = Im G( jw).
6. The asymptotic Bode magnitude plot of a mini- 1
G (s) = .
mum phase transfer function is shown in the figure: s ( s + 1)( s + 2)
 [2008]
Put s = jw.
|G( jw)| 1
(dB) 40 dB/decade G ( jω ) = .
jω ( jω + 1)( jω + 2)
20
Separate real and imaginary parts,
−20 dB/decade
− j (1 − jω )(2 − jω )
0 0.1
G ( jω ) = .
w(rad/s) ω (1 + ω 2 )(4 + ω 2 )
(log scale)
−20 3ω − j (2 − ω 2 )
G ( jω ) = − .
0 dB/decade
ω (1 + ω 2 )(4 + ω 2 )
This transfer function has −3 j (ω 2 − 2)
G ( jω ) = + .
(a) Three poles and one zero (1 + ω )(4 + ω ) ω (1 + ω 2 )(4 + ω 2 )
2 2

(b) Two poles and one zero


At w → 0+   ⇒  imaginary part = 0.
(c) Two poles and two zeros
−3
(d) One pole and two zeros So by equating imaginary part x = .
4
Solution: (c) Hence, the correct option is (b).
8. The Bode magnitude plot of
−40 dB/decade 104 (1 + jω )
H( jw) = is [2006]
20 (10 + jω )(100 + jω ) 2
−20 dB/decade (a) |H(jw) dB
0
40
−20
20
0 dB/decade
log(w)
0
At origin two poles are present as plot starts with −1 1 2 3
-40 dB/dec slope. −20

At next corner frequency one zero present, so +20 −40


dB/dec slope add and results to -20 dB/dec slope.

Chapter 06.indd 39 11/9/2015 5:15:35 PM


5.40 | Control Systems

(b) |H(jw) dB w = 10 rad/sec One pole present.


w = 100 rad/sec Two poles are present.
40 No pole/zero present at origin so start with 0 dB/dec.
20 20 log k
y= ⇒ y = −20 dB.
log(w) y
0
−1 1 2 3 At w = 1 rad/sec zero present so 20 dB/dec.
−20 Slope increase.
−40 At w = 10 rad/sec pole present so -20 dB/dec.
Slope decrease.
(c) |H(jw) dB log10 = 1.
At w = 100 rad/sec two poles present so -40 dB/dec
40
slope decrease. log 100 = 2.
20 40
log(w)
20
0
−1 1 2 3
1 2 3
−20

−40

(d) |H(jw) dB
Hence, the correct option is (a).
40 9. Consider the following Nyquist plots of loop
transfer functions over ω = 0 to ω = ∞. Which of
20 these plots represents a stable closed loop system?
log(w)  [2006]
0 Im Im
−1 1 2 3
−20 R w R
w=¥ w=¥
Re Re
−40 −1 −1
w
Solution: (a)
104 (1 + jω )
H ( jω ) = . (1)   (2)
(10 + jω )(100 + jω ) 2
Im Im
104 (1 + jω )
= 2
.
 jω   jω 
10 1 +
5
 1 +  w=¥
 10   100  −1
Re w=¥
Re
−1
1 (1 + jω ) w
H ( jω ) = 2
. w
10  jω   jω 
1 +  1 + 
 10   100  (3)   (4)
1 (a) (1) only (b) All, except (1)
k= .
10 (c) All, except (3) (d) (1) and (2) only
k = 0.1. Solution: (a)
Corner frequencies at w = 1 rad/sec one zero is Consider open loop poles in the RHS of s-plane
present. = P = O.

Chapter 06.indd 40 11/9/2015 5:15:36 PM


Chapter 6  Frequency Response Analysis | 5.41

1. N = -2.
N = P - Z.
Z = 2.
Hence, system is unstable.
−1 Hence, the correct option is (a).
10. In the GH(s) plane, the Nyquist plot of the loop trans-
π e −.25 s
fer function G ( s ) H ( s ) = passes through the
s
N = P - Z.
negative real axis at the point [2005]
O = O - Z. (a) ( − 0.25, j0) (b) ( − 0.5, j0)
Z = O. (c) ( −1, j0) (d) ( −2, j0)
Hence, system is stable.
Im
Solution: (b)
2.
π e −0.25 s
w = o− w G (s) H (s) = .
s
w = ¥+
As e −TS  (1 − TS ).
Re −1 w = ¥− π (1 − 0.25s )
We can write G ( s ) H ( s ) = .
s
w = o+
Put s = jw.
π (1 − 0.25 jω )
N = 2. G ( jω ) H ( jω ) = .

N = P - Z.
At phase angle −180° Nyquist plot cut to negative
Z = 2.
real axis.
Hence, system is unstable.
So ∠G ( jω ) H ( jω ) = −90° − 0.25 to -180° = -90°
3.
w = o−
- 0.25 tan –1 w.
w = 360° or w = 2π.
w = ¥+ π e − j 0.25×2π
G ( jω pc ) H ( jω pc ) =
−1 w = ¥− j 2π
1
= (cos 0.5π − j sin 0.5π )
w = o+ 2j
1 π π
N = -2. = − cos − j sin 
2 2 2
P - 2 = N.
O - Z = -2. 1
= − (0 − j ) = − 0.5 + j 0.
Z = 2. 2
Hence, system is unstable. Hence, the correct option is (b).
4. Im 11. If the compensated system shown in the figure has
w = o− a phase margin of 60° at the crossover frequency
of 1 rad/sec, then value of the gain K is [2005]
w = ¥+ R(s) + 1 Y(s)
K + 0.366s
−1 s(s + 1)
w= ¥−

(a) 0.366 (b) 0.732


w = o+
(c) 1.366 (d) 2.738

Chapter 06.indd 41 11/9/2015 5:15:37 PM


5.42 | Control Systems

Solution: (c) k
G (s) = .
R(s) + 1 Y(s)
s +1
K + 0.366s k
s(s + 1)
G ( jω ) = At ω = 1 rad/sec.
jω + 1
k 0°
Given PM = 60° at ω gc = 1 rad/sec. G ( jω ) = ; ∠G ( jω ) = −1
.
1+ ω 2 (t an 1)
k + 0.366 s k n
G ( s) =
       . So, G ( jω ) = ∠− .
s ( s + 1) 2 4
Unity feedback system k  π
So, output y (t ) = sin  t −  .
k + 0.366 s 2  4
G (s) H (s) =
So  .
s ( s + 1) Hence, the correct option is (a).
PM = 180 + φ . 13. The open loop transfer function of a unit feedback
φ = ∠G ( jω ) H ( jω )ω gc . as +1
control system is given as G ( s ) = 2 . The value
∠G ( jω ) H ( jω ) = 60 − 180°. s
of a to give a phase margin of 45° is equal to
∠G ( jω ) H ( jω ) = −120°. [2004]
0.366 ω (a) 0.141 (b) 0.441
∠G ( jω ) H ( jω ) = −90° − tan −1 ω + tan −1 .
k (c) 0.841 (d) 1.141
ω gc = 1 rad/sec phase angle will be
At  Solution: (b)
as +1
∠G ( jω ) H ( jω ) = −120° G (s) = 2 .
s
0.366 Given PM = 45°.
= −90° − tan −1 1 + tan −1
.
k PM = 180 + φ . So φ = −180 + 45°.
0.366 φ = −135°.
−120 = −90 − 45 + tan −1 .
k φ = ∠G ( jω )ωgc .
0.366
15 = tan −1 . To calculate wgc,
k a ( jω ) + 1
k = 1.366. G ( jω ) = .
( jω ) 2
Hence, the correct option is (c).
∠G ( jω )ω gc = −180° + tan −1 aω gc .
12. In the system shown in figure, the input x(t) = sin t.
In the steadystate, the response y (t) will be −135 = −180° + tan −1 aω gc .
[2004] tan −1 aω gc = 45°.
x(t ) K y(t ) 1
s+1 ω gc = .
a
K K    Gain = G ( jω gc ) = 1.
(a) sin(t − 45°) (b) sin(t + 45°)
2 2 1 + a 2ω 2 gc
= 1.
(c) K sin(t + 45o)
K sin(t − 45°) (d) ω 2 gc
Solution: (a) 1
1 + a2 ×
x(t ) = sin t ; ω = 1 rad/sec. a 2 = 1.
1
x(t ) K y (t ) a2
s+1 1
= 2.
From given block, a2

Chapter 06.indd 42 11/9/2015 5:15:39 PM


Chapter 6  Frequency Response Analysis | 5.43

So, a = 0.841. From plot magnitude = 20 log k.


Hence, the correct option is (b). ∴  Error in dB gain = 0.97 dB.
14. The asymptotic Bode plot of the transfer function  0.5a 
K ∠G ( jω ) ω = 0.5 a = − tan −1   = −26.56°.
. The error in phase angle and dB gain at a fre-  a 
s
1+  0.5a  φ − 0
a log10  = .
quency of a frequency of ω = 0.5a are respectively  0.1a  − 45°
 [2003] ∠G ( jω ) ω = 0.5 a = φ = −31.45°.
20 log K 20 dB/decode
Error in phase angle,
G dB
a = −26.56° − (−31.45°) = 5.89° ~ 5.7.
w
Hence, the correct option is (d).
0.1a 10a
w 15. The function corresponding to the Bode plot of
45°/decode
Ph° figure is [1999]

(a) 4.9°, 0.97 dB (b) 17°, 3 dBs A dB per


(c) 4.9°, 3 dB (d) 5.7°, 0.97 dB dB active
Solution: (d) 0 dB
1 f1
G ( s) = ω = 0.5 a. f
s
1+ jf 1
a A=
(a) A=
(b)
f1  jf1 
20 log K 20 log dB/dec 1 − 
 f 
G dB
1 1 + jf
w A=
(c) A=
(d)
 jf1  f1
1 + 
 f 
Ph
0.1a 10a
Solution: (d)
45°/decode

At s = jw. A dB per
k dB active
G ( jω ) = .
jω 0 dB
1+
a f1
k f
G ( jω ) = . Sterling slope 0 dB/dec at corner frequency f1 is
ω2
1+ present here.
a2
Hence, NF transfer function is
ω
∠G ( jω ) = − tan −1 . jf
a A = 1+ .
At w = 0.5a, f1
k k Hence, the correct option is (d).
G ( jω ) ω = 0.50 = = .
(0.5a ) 2
1.25 16. A unity feedback system with the open loop trans-
1+ 1
a2 fer function G ( s ) =  has gain mar-
s ( s + 2)( s + 4)
In dB = 20 log k − 20 log (1.25)1/ 2 gin of ... dB. [1997]
= 20 log k − 0.97 dB. Solution:

Chapter 06.indd 43 11/9/2015 5:15:40 PM


5.44 | Control Systems

1 kωn2
G (s) = . H (s) = .
s ( s + 2)( s + 4) s 2 + 2ξωn s + ωn2
1 k
G ( jω ) = . H (s) = .
jω ( jω + 2)( jω + 4) 2ξ s s 2
1+ +
ω ω ωn ωn2
∠G ( jω ) = −90° − tan −1 − tan −1 .    
2 4
∠G ( jω )ω gc = −180°. k
H ( jω ) = .
ω ω 2 jω ω 2
−180° = −90° − tan −1 − tan −1 . 1+ − 2
2 4 ωn ωn
ω ω k
+ H ( jω ) = at ω = 0.
∞ = 2 42 .
2
  ω  2   2ω  2
ω 1 −    + 
1−   ωn    ωn 
8  
ω2 f ( jω ) = 1.
So, 1 − = 0.
8
ω2 So k = 1.
1= .
8 Maximum overshoot H(p) = 2.5.
ω gc = 2 2 rad/sec. −πξ
1−ξ 2
1 2.5 = e .
G ( jω ) ω gc = .
ω 8 + 4 8 + 16 πξ
ln 2.5 = − .
1 1 1− ξ 2
G ( jω )ω gc = = .
2 2 12 24 48 ξ = 0.2.
1
GM = 20 log .
G ( jω )ω gc
GM = 33.600. Four-marks Questions
17. An under damped second order system having a trans-
kωn2 1. Match the polar plots for the following functions
fer function of the form M ( s ) = 2
s + 2ζωn s + ωn2 on the left hand side [1994]
has frequency response plot shown in figure below,  (P)  Im   (S)  Im
then the system gain K is and the damping
Re
factor is approximately  [1991] w=0 w=¥
Re
w=¥
2.5 w=0

 (Q)  Im  (T)  Im
w=0
1 1
Re
w w=0 w=¥
Re
w=¥
Solution:
 (R)  Im  (U)  Im
2.5
w=¥ −1 1
Re Re
1 w=0 w=0 w=∞
w

Chapter 06.indd 44 11/9/2015 5:15:42 PM


Chapter 6  Frequency Response Analysis | 5.45

from w = 0
s s2 + 1
(a) (b)
( s + 1)( s + 2) s3
w=1
s2 −1 1 w=¥
(c) 2 (d)
s +1 s 2 + 10
Solution:
s jω
(a) = s2 −1 −(ω 2 + 1)
( s + 1)( s + 2) ( jω + 1)( jω + 2) (c) 2 = .
s + 1 s = jω (1 − ω 2 )
ω  ω
= ∠90° −  tan −1 ω + tan −1  ω = 0 − 1.
(ω + 1)(ω + 4)
2 2
 2
ω > 1 − ve.
ω=0 0∠90°
ω = −1 − ∞.
… … ∠0°
ω > 1 + ve.
ω=∞ 0∠ − 90°
I mg
+∞.
G(s)H(s) Plane ω=∞ 1.
−1 1
w=0
1 w w=0 w =¥
w=¥ Real

1 1 1
(d) = = .
s 2 + 10 ( jω ) 2 + 10 10 − ω 2
s 2 + 1 ( jω ) 2 + 1 j(1 − ω 2 )
(b) 3 = = . ω = 00.1.
s ( jω )3 ω3
ω 2 < 10 + ve.
ω=0 j∞ 
 Positive imaginary axis. ω 2 = 10 ∞.
ω =1 j0 
ω 2 > 10 − ve.
ω = 1.0000001 − j∞  − ∞.
  Negative imaginary axis.
ω=∞ − j0  ω → ∞ − 0.
w=0 w = √10

w=¥ 0.1

Chapter 06.indd 45 11/9/2015 5:15:43 PM


5.46 | Control Systems

1 2
Five-marks Questions GM = =
G ( jω pc ) H ( jω pc ) 10
(or )

1. A unity feedback system has an open-loop transfer 2


GM = 20 log = −14 dB.
10000 10
function of G ( s ) = . [2001]
s ( s + 10) 2 (d) Since GM and PM both are negative, system is
(a) Determine the magnitude of G(jω) in dB at an unstable.
angular frequency of ω = 20 rad/sec. 2. Open-loop transfer function of a unity-feedback
(b) Determine the phase margin in degrees. system is
(c) Determine the gain margin in dB.
(d) Is the system stable or unstable. e − sτ D
G ( s ) = G1 ( s ) ⋅ e − sτ D = .
Solution: s ( s + 1)( s + 2)
104 Given: |G1( j ω)| ≈ 1 when ω = 0.466. [2000]
(a) G ( jω ) =
jω ( jω + 10) 2 (a) Determine the phase margin when τD = 0.
(b) Comment in one sentence on the effect of
104 dead-time on the stability of the system.
=
ω (ω 2 + 102 ) 2 ω = 20
(c) Determine the maximum value of dead-time
4 τD for the closed-loop system to be stable.
10
= Solution:
20(202 + 102 ) 1
= G1 ( s ) =
s ( s +1) ( s + 2 )
104 G ( s )τ D = 0
(a) .
= = 1.
20(500) =1
G1 ( s )ω =ω gc = 0.466 .
G ( jω ) ω = 20 = 1.
Ie gain cross over frequency ω gc = 0.466.
(b) PM = 180° + ∠G ( jω ) ω =ω ( Note ω gc = 20).
gc
1
 ω gc  PM = 180° + ∠
PM = 180° − 90° − 2 tan −1  . jω ( jω + 1)( jω + 2)
 10   ω gc ω gc 
PM = 180° − 90° − 2(63.43). = 180° −  90° + tan −1 + tan −1 
 1 2 
  PM = −36°.
= 180° − (90° + 25° + 13°) = 52°.
1
GM =
(c) PM = 52°.
G ( jω pc ) H ( jω pc )
(b) If dead time increases the stability of the system
104 decreases.
∠ = −180°
jω pc ( jω pc + 10) 2 (c) For just stability PM = 0°, GM = 0 dB.
 ω pc  PM = 0° = 180° + ∠G ( s ) ω =ω
−  90° + 2 tan −1  = −180°. gc
 10 
− jωτ D
ω pc = 10. e
=π +∠ = 0.
jω ( jω + 1)( jω + 2) ω =ω
G ( jω pc ) H ( jω pc ) = 10 gc

ω pc
104  π ω gc ω gc 
= π −  ω gcτ D + + tan −1 + tan −1 
ω pc (ω 2 pc + 102 )  2 1 2  ω = 0.466
gc

104 ω gcτ D = 0.3.


=
10(102 + 102 ) 0.3
4
τD = = 0.64.
10 10 0.466
= = .
(200)(10) 2 τ D = 0.64.

Chapter 06_Five Marks.indd 46 11/3/2015 3:16:02 PM


Chapter 6  Frequency Response Analysis | 5.47

For τ D = 0.64 system is just stable. Slope between (4 and 8) is −40 dB/dec.
τ D < 0.64 Stable. 0− y −y
−40 = ⇒ −40 = .
τ D > 0.64 Unstable. log 4 − log 8 0.602 − 0.903
y = −12 dB.
3. The asymptotic magnitude Bode plot of a system
is given in figure. Find the transfer function of the Slope between 8 and w3 is −20 dB/dec.
system analytically. It is known that the system is
minimal phase system. [1998] −12 − (−21) 9
−20 = ⇒ − 20 = .
Mag log 8 − log ω3 0.903 − log ω3
dB ω3 = 22.547 rad/sec.

20 dB/decade
By starting line
y = mx + c.
|G| −40 dB/decade 36 dB = −20 log (0.503) + C.
dB 36 dB
2 4 w3 36 = −20 × (−0.298) + 20 log K .
log w
0 dB w1 K = 31.54.
 s
−21 dB 31.54 1 + 
−40 dB/decade ∴ TF =  8 .
 s  s 
Solution: s 1 +  1 + 
The slope in between (w1 and 4) is − 40 dB/sec.  0.5   22.5 
0 − 36 8 2 ( s + 4)
−40 = . TF = .
   log 4 − log ω1 s (ss + 0.25)( s + 8 2 )
−36
−40 = .
0.602 − log ω1
24.08 − 40 log ω1 = 36.
− 40 log ω1 = 11.92.

∴ ω1 = 0.503 rad/sec.

Chapter 06_Five Marks.indd 47 11/3/2015 3:16:04 PM


Chapter 7
Controllers (or)
Compensators
 ∴   Improve in damping, so overshoot also de-
One-mark Questions creases.
Hence, the correct option is (b).
1. A lead compensator used for a closed loop control-
3. Introduction of integral action in the forward path
 s
K 1 +  of a unity feedback system results in a [1997]
 a (a) marginally stability
ler has the following transfer function .
 s (b) system with no steady state error
 1 + 
 a (c) system with increased stability margin
For such a lead compensator [2003] (d) system with better speed of response
(a) a < b (b) b<a Solution: (b)
(c) a > Kb (d) a < Kb
Integral action in the forward path of a unity feed-
Solution: (a) back system results in a system with no steady
For lead compensator, P > Z . state error.
Hence, the correct option is (b).
 s
k 1 +  4. The pole–zero configuration of a phase lead com-
a
TF =  . pensator is given by [1994]
 s
 1 + 
 b (a) jw (b) jw
Zeros = −a.
Poles = −b.
s s
∴ − b > −a.
∴ b > a.
Hence, the correct option is (a). (c) jw (d) jw
2. The phase lead compensations used to [1998]
(a) increase rise time and decrease overshoot s s
(b) decrease both rise time and overshoot
(c) increase both rise time and overshoot
(d) decrease rise time and increase overshoot
Solution: (b)
Solution: (b)
Lead compensator P > Z .
Lead compensator increase in bandwidth. Hence,
decrease in rise time. Hence, the correct option is (b).

Chapter 07.indd 48 11/12/2015 10:19:14 AM


Chapter 7  Controllers (or) Compensators  |  5.49

 ω2   1 1   ω ω   2ω 
Two-marks Questions 1 +  −  −  −   = 0.
 2 1 2   1 2  2 
Common Data for Questions 1 and 2: 1 ω2 ω2 ω2 1
+ = .    = ⇒ ω 2 = 2.
The transfer function of a compensator is given as 2 4 2 4 2
s+a 
Gc ( S ) = . ∴ ω = 2 rad/sec.
s+b
Hence, the correct option is (a).
1. GC(s) is a lead compensator if [2012]
(a) a = l, b = 2 (b) a = 3, b = 2 3. The transfer functions of two compensators are
(c) a = - 3, b = - l (d) a = 3, b = l given below:
10( s + 1) s + 10
Solution: (a) C1 = , C2 = .  Which one of the
( s + 10) 10( s + 1)
s+a
 Gc ( s ) = . following statements is correct? [2008]
s+b (a)  C1 is lead compensator and C2 is a lag compen-
jω + a sator
 Gc ( jω ) = .
jω + b (b) C1 is a lag compensator and C2 is a lead com-
pensator
ω  ω  (c) Both C1 and C2 are lead compensator
∠Gc ( jω ) = tan −1   − tan −1  
 a b (d) Both C1 and C2 are lag compensator
ω ω  Solution: (a)
 − 
= tan  a b2  .
−1
C1 =
10( s + 1)
C2 =
s + 10
.
1 + ω  ( s + 10) 10( s + 1)
 ab  Zero at s = −1   Zero at  s = −10.
For Gc ( s ) lead compensator, ∠G ( jω ) > 0. Pole at s = −10   Pole at s = −1.
ω ω
> .
a b
b > a. −10 −1 −10 −1
Hence, the correct option is (a).
|P |>| Z | s-plane |P |>| Z |
2. The phase of the above lead compensator is maxi-
C1 is lead compensator C2 is lag compensator
mum at [2012]
(a) 2 rad/s (b) 3 rad/s Hence, the correct option is (a).
(c) 6 rad/s (d) 1/ 3 rad/s 900
4. The system   is to be such that its gain
Solution: (a) s ( s + 1)( s + 9)
For phase to maximum, crossover frequency becomes same as its uncompen-
sated phase crossover frequency and provides a 45°

= 0. phase margin. To achieve this, one may use [2007]
dω (a) a lag compensator that provides an attenuation
ω ω of 20 dB and a phase lag of 45° at the frequen-

φ = a b2 . cy of 3 3 rad/s.
ω (b) a lead compensator that provides an amplifica-
1+
ab tion of 20 dB and a phase lead of 45° at the
frequency of 3 rad/s.
 ω 2   1 1   ω ω   2ω  
1 +   −  −  −     (c) a lag-lead compensator that provides an ampli-
 ab   a b   a b   ab   = 0. fication of 20 dB and a phase lag of 45° at the
2
 ω2  frequency of 3 rad/s.
1 +
 ab  (d) a lag-lead compensator that provides an at-
 
tenuation of 20 dB and phase lead of 45o at the
From above question a b 2.
= 1,= frequency of 3 rad/s.

Chapter 07.indd 49 11/12/2015 10:19:19 AM


5.50 | Control Systems

Solution: (d) T ( jω ) in dB = 20 log (100) − 20 log ω − 20 log 1 + ω 2


900
T ( s) = . 2
s ( s + 1)( s + 9) ω 
− 20 log 1 +  
900 9
T ( jω ) = .
jω ( jω + 1)( jω + 9) = 40 − 20 log ω − 20 log 1 + ω 2
ω  2
∠T ( jω ) = −90° − tan −1 ω − tan −1   . ω 
9 − 20 log 1 +   .
9
Phase crossover frequency of uncompensated sys- Gain of compensated system
tem (ω pc ) at this ∠T ( jω ) = −180°. T ( jω ) com = x + T ( jω ) dB .
 ω pc  Gain of compensated system = 0 at  (ω gc ) 2 .
−180° = −90° − tan −1 ω pc − tan −1  .
 9  0 = x + 40 − 20 log (ω gc ) 2 − 20 log 1 + (log c ) 22
 ω  2
 ω pc1 + pc1   ω gc 
− 20 log 1 +   .
90° = tan −1  9 .
 9 
  ω pc  
1 − (ω pc )   0 = x + 40 − 20 log 3 − 20 log 1 + 32
  9  
2
 10ω pc1  3
  − 20 log 1 +   .
9 9
90° = tan −1  .
 ω pc1 
2
1
1 −  x = 20 log 3 + 20 log 1 + 32 + 20 log 1 + − 40.
 9  9
(ω pc1 ) 2 x = −20 dB.
1− = 0.
9 So, the compensator provides  20 dB  attenuation.
 ω pc1 = 3 rad/sec. Hence, the correct option is (d).
(0.5s + 1)
As given in question, 5. D( s ) = .  Maximum phase lead of the
(0.05s + 1)
Gain crossover frequency  (ω gc2 ) =  Phase crossover
compensator is [2000]
frequency (ω pc ).
(a) 52 deg at 4 rad/sec
ω gc 2 = ω pc1 = 3 rad/sec. (b) 52 deg at 10 rad/sec
∴  PM = 180° + ∠T ( jω )ω = ωgc 2 . (c) 55 deg at 12 rad/sec
(d) None of the answers is correct
45° = 180° + ∠T ( jω )ω = ωgc . Solution: (d)
∠T ( jω )ω = ω gc 2 = 180° − 45°. (0.5s + 1)
(s) = .
∠T ( jω )ω = ω gc 2 = 135°. (0.05s + 1)
1 + aTs
∴  Compensator provides leads of 45° of the fre- Phase lead compensator TF = .
1 + Ts
quency of 3 rad/sec. T = 0.05  and  a = 10.
Gain of uncompensated system,  a −1  −1  10 − 1 
φm = sin −1   = sin  .
100  a +1  10 + 1 
T ( jω ) = . 9
ω 
2
φm = sin −1   .
ω 1+ ω 1+  
2
 11 
9
φm = 55°.
100
T ( jω ) = . 1 1
2 ∴ ωm = = = 6.32 rad/sec.
ω  T a 0.05 × 10
ω 1+ ω2 1+  
9 Hence, the correct option is (d).

Chapter 07.indd 50 11/12/2015 10:19:23 AM


Chapter 8
State Space Analysis
2. For a system with the transfer function
One-mark Questions 3( s − 2)
H (s) = 3 ,  the matrix A in the a state
s + 4s 2 − 2s + 1
1. The state transition matrix for the system space form  X = AX + BU  is equal to [2006]
 x1  1 0   x1  1
 =    +   u is [2014-S2] 1 0 0 0 1 0 
 x2  1 1   x2  1 
(a) 0 1 0
 (b)
 
  0 0 1 
 et 0  et 0  −1 2 −4   −1 2 − 4 
(a) (b)    
 t  2 t t
t e e 
t
e e
0 1 0 1 0 0 
e e te  t t    
t
0  3 −2 1 
(c) (d)
0 0 1 
(c)
 t t
(d)
 t   1 −2 4   −1 2 − 4 
te e 0 e     
Solution: (c) Solution: (b)
 x1  1 0   x1  1 The state space representation is phase variable
 x  = 1 1   x  + 1 u. form (controllable canonical form (CCF))
 L   2  
Y ( s) 3( s − 2)
∴ TF = = .
As, e At = L−1[( SI − A) −1 ]. U ( s) s3 + 4s 2 − 2s + 1
SSR in controllable canonical form is
s −1 0 
SI − A =  .
 −1 s − 1 0 1 0  0
  
X (t ) =  0 0 1  x(t ) + 0  U (t ).
adj( SI − A)
( SI − A) −1 =  −1 2 − 4  3 
| SI − A |
Y (t ) = [−2 1]x(t ).
s −1 0   1 
 1 6  0 1 0 
s − 1  ( s − 1)
= = . ∴ A =  0 0 1  .
( s − 1) 2  1 1    
 ( s − 1) 2  −1 2 − 4 
 s − 1 
Hence, the correct option is (b).
 et 0
e At =  t . 3. A second order system starts with an initial condition
te et 
 2
of   without any external input The state transition
Hence, the correct option is (c).  3

Chapter 08.indd 51 11/12/2015 10:24:54 AM


5.52 | Control Systems

 e −2t 0  Solution: (a)


matrix for the system is given by  − t  . The
 0 e   s + 3 −1 
SI − A =  .
state of the system at the end of 1 second is given by  0 s + 2 
[2003]
 1 1 
 0.271  0.135   s + 3 ( s + 2)( s + 3) 
(a)
  (b)
  [ SI − A]−1 =  .
 1.100   0.368   1 
 0 s+2 
 0.271   0.135 
(c)
  (d)   e −3t e −2t − e −3t 
 0.736   1.100  e At = L−1[( SI − A) − 1] =  .
 0 e −2t 
Solution: (a)
X (t ) = e At X (0).
2 e −2t 0
X (0) =   e At =  . e −3t e −2t − e −3t  10 
  3  e−t  X (t ) =   .
 0 e −2t   −10 
 0
X (t ) = e At X (0)
State  10e −3t −10e −2t + 10e −3t 
X (t ) =  .
 −10 e −2t 
e −2t 0  2
=    20e −3t −10e −2t 
 0 e − t  3  x(t ) =  .
 −10 e −2t 
 2 e −2t  0.271
=  −t  =  .  20e −3t −10e −2t  0 
  3 e  t =1 1.100  xss = Lt X (t ) = Lt   =  .
t →∞ t →∞ −10e −2t  0 

Hence, the correct option is (a).
Hence, the correct option is (a).
4. The state transition matrix for the system  X = AX
with initial state X(0) is [2002] 0 1 0
(a) (sI − A) −1 6. Given the matrix A =  0 0 1  .  Its eigen

(b) eAt X(0)  −6 −11 −6 
(c) laplace inverse of [(sI − A) −1] values are _________. [1995]
(d) laplace inverse of [(sI − A)−1 X(0)] Solution:
Solution: (c)  ⎮SI - A⎮ = 0. Characteristic equation s3 + 6s2 + 11s
e At =  State transition matrix. + 6 = 0 roots are s = - 1, - 2, -3 (Eigen values).
e At = L−1[( SI − A) −1 ]  irrespective of  X (0). 7. A system is described by the state equation
X = AX + BU. The output is given by Y = CX.
Hence, the correct option is (c).
 −4 −1 1
5. Given the homogeneous state-space equation Where A =   B =   C = [1 0] .
 3 −1 1
 −3 1 
X =   x the steady state value of Transfer function G(s) of the system is [1995]
 0 −2 
s 1
xss = lim x(t ), given the initial state value of x(0) (a) 2 (b)
t →∞ s + 5s + 7 s + 5s + 7
2

= [10 − 10]T, is [2001] s 1


(c) 2 (d)
0  −3 s + 3s + 2 s + 3s + 2
2

xss =  
xss =   (b)
(a)
0  −2  Solution: (a)

 −10  ∞  1 0   − 4 −1 ( s + 4) 1 
[ SI − A] = s   −  = .
xss = 
(c)  xss =  
(d)
 0 1   3 −1  −3 s + 1
 10  ∞   

Chapter 08.indd 52 11/12/2015 10:24:58 AM


Chapter 8  State Space Analysis  |  5.53

 s + 1 −1  10. Consider a second order system whose state space


 3 representation is of the form X = AX + BU. If x1(t)
s + 4 
( SI − A) −1 =  . = x2(t) then system is [1993]
( s + 1)( s + 4) + 3 (a) controllable (b) uncontrollable
Y (s) (c) observable (d) unstable
TF = = C[ SI − A]−1 B + D
U (s) Solution: (b)
 s + 1 −1  1      x1 (t ) = x2 (t ).
[1 0] 
 3 s + 4  1 ( s + 1) − 1 dx1 (t ) dx2 (t )
= = 2 . Therefore, = .
( s + 1)( s + 4) + 3 s + 5s + 7 dt dt
Y ( s) s Let the state equation be
∴ TF = = 2 .
U ( s ) s + 5s + 7  d x1 (t ) 
 dt   a b   x1 (t )   e 
Hence, the correct option is (a).  =  +   u (t ).

 a 1  d x2 (t )   c d   x2 (t )   f 
8. The eigen-values of the matrix   are[1994]  dt 
 a 1
Controllability matrix = [ B AB].
(a) (a + l), 0 (b) a, 0
(c) (a − l), 0 (d) 0, 0  a b  e   ae + bf 
AB =    =  .
Solution: (a)  c d   f  ce + df 
Characteristic equation, Since, x1 (t ) = x2 (t )
 s − a −1  a + b = c + d  and  e = f .
[ SI − A] =   = 0.
 −a s − 1
e (ae + bf )
( s − a )( s − 1) − a = 0. | [B AB] | = = 0.
f (ce + df )
s 2 − (a + 1) s + a − a = 0.
Hence, uncontrollable.
s = a + 1, s = 0.
Hence, the correct option is (b).
Hence, the correct option is (a).
11. The transfer function for the state variable representa-
9. The matrix of any state space equations for the tion X = AX + BU, Y = CX + DU, is given by [1993]
C (s)
transfer function of the system, shown below (a) D + C(sI − A) −1 B (b) B(sI − A) −1 C + D
R( s) (c) D(sI − A) B + C (d)
−1
C(sI − A) −1 D + B
in. figure is [1994]
Solution: (a)
R(s) 1 C(s)
3 + −
S X = AX ( s ) + BU ( s ) (1)
  Y = CX ( s ) + DU ( s ) (2)
Taking LT of Equation (1)
 −1 0   −1 0  sX ( s ) = AX ( s ) + BU ( s ).
(a)   (b)  
 0 1  0 1 sX ( s ) − AX ( s ) = BU ( s ).
(c) [−1] (d) [3]
X ( s )[ sI − A] = BU ( s ).
Solution: (c) −1
   X ( s ) = [ sI − A] BU ( s ) (3)
X = AX ( s ) + BU ( s ).
Substituting (3) in (2)
C (s) 3
TF = = .
R( s) s + 1 Y ( s ) = C[ SI − A]−1 BU ( s ) + DU ( s ).
Pole is at s = - 1 ∴ a = [- 1]1 × 1. Y (s)
∴ TF = = C[ SI − A]−1 B + D.
Eigen value of A = Pole of the system. U (s)
Hence, the correct option is (c). Hence, the correct option is (a).

Chapter 08.indd 53 11/12/2015 10:25:02 AM


5.54 | Control Systems

 −1 1  0   1 
Two-marks Questions PQ =     =  .
 0 −3 1   −3
1. A system matrix is given as follows 0 1 
M =  = −1.
0 1 −1 1 −3
  Hence, controllable.
A =  −6 −11 6  .   The absolute value of the
 −6 −11 5  C  P
For observability M = [C T AT C T ] =   =  .
ratio of the maximum [2014-S1]  AC   PR 
Solution:  −1 1 
   PR =   [0 1] = [0 −3].
0 1 −1  0 −3

A =  −6 −11 6  . 0 1 
 −6 −11 5  M =  = 0.
  0 −3
⇒   Characteristic equation  SI − A = 0. Not observable.
Hence, the correct option is (c).
s −1 1 
6 s + 11 − 6  = 0. Common Data for Questions 3 and 4:
  
The state variable formulation of a system is
6 11 s − 5  x   −2 0   x1  1
given as  1  =     +    u, x1(0) = 0,
s[( s + 11)( s − 5) + 66] + [6( s − 5) + 36]  x2   0 −1  x2  1
+ [66 − 6( s + 11)] = 0.  x1 
x2(0) = 0 and  y = [1 0]   . 
s[ s + 65 − 55 + 66] + 6 s + 6 + (− 6 s ) = 0.  x2 
2

s 3 + 6 s 2 + 115 + 6 s + 6 + (− 6 s ) = 0. 3. The system is [2013]


(a) controllable but not observable
s 3 + 6 s 2 + 115 + 6 = 0.
(b) not controllable but observable
s = −1, − 2, − 3. (c) both controllable and observable
(d) both not controllable and not observable
max
=
Ratio = 3. Solution: (a)
min
dX  −2 0  1
2. The second order dynamic system = PX + Qu , A=  B =   C = [1 0].
dt  0 −1 1
y = R X has the matrices P, Q, and R as follows: For controllability M = [ B AB ].
 −1 1  0  −2 0  1  −2 
P=  Q = 1  R = [0 1].  The system has    AB =  0 −1 1 =  −1 .
 0 − 3        
the following controllability and observability    M = [ B AB ].
properties: [2014-S2] 1 −2 
(a) Controllable and observable =  = −1 + 2 = 1  Controllable.
(b) Not controllable but observable 1 −1
(c) Controllable but not observable C 
For observability M = [C T AT C T ] =  .
(d) Not controllable and not observable  AC 
Solution: (c)  −2 0 
AC =   [1 0] = [−2 0].
 0 −1
 −1 1  0 
P=  Q=  R = [0 1].  1 0
 0 −3 1   M =   = 0.  Not observable.
 −2 0 
= [=
For controllability M B AB ] [Q PQ]. Hence, the correct option is (a).

Chapter 08.indd 54 11/12/2015 10:25:07 AM


Chapter 8  State Space Analysis  |  5.55

4. The response y (t) to a unit step input is [2013]  x1 


1 1 −2t (b) 1 1  
(a) − e 1 − e −2t − e − t y = (1 0 0)  x2  .
2 2 2 2 x 
 3
(c) e −e
−2t −t
(d) 1 − e −t
As that given system is controllable, i.e.,
Solution: (a)     M = [0 AB A2 B ].
TF = C ( SI − A) −1 B + D. 0 a1 0  0   0 
SI − A = 
s + 2 0 
.   AB =  0 0 a2  0  =  a2  .
 0 s + 1  a3 0 0  1   0 
s + 1 0 
0 a1 0   0   a1a2 
 0 s + 2 
( SI − A) −1 =  .  A B =  0
2
0 a2   a2  =  0  .
( s + 1)( s + 2) 
 a3 0 0   0   0 
s + 1 0  1
[1 0] 
 0 s + 2  1 0 0 a1a2 
TF =
( s + 1)( s + 2)
. M = 0 a2 0  = a1a2 (0 − a2 ),
1 0 0 
1
[ s + 1 0]  
1 = s +1 1 i.e., a1 ≠ 0, a2 ≠ 0 and a3 = 0.
TF = = .
( s + 1)( s + 2) ( s + 1)( s + 2) s + 2 Hence, the correct option is (d).
 −1 2 
1 6. The system X = AX + BU with A =  ,
Given as u ( s ) = .  0 2
s 0
B =   is [2010]
1 1 1 
TF ⇒ y(s) = . (a) stable and controllable
s+2 s
(b) stable but uncontrollable
1
y (t ) = (1 − e −2t ). (c) unstable but controllable
2 (d) unstable and uncontrollable
Hence, the correct option is (a). Solution: (c)
5. The state variable description of an LTI system is  −1 2 
 x1   0 a1 0   x1   0  A= .
 0 2
given by  x2  =  0 0 a2   x2  +  0  u,
 x   a 0 0   x   1  s + 1 −2
 3  3  3    SI − A = =0
0 s−2
 x1  = ( s + 1)( s − 2) = 0.
 
 y = (1 0 0)  x2  ,  where y is the output and u is Roots are −1, 2, unstable system.
x 
 3 Controllability matrix  M = [ B AB ].
the input. The system is controllable for [2012]  −1 2  0   2 
(a) a1 ≠ 0, a2 = 0, a3≠ 0 (b) a1 = 0, a2 ≠ 0, a3 ≠ 0 [ AB ] =     =  .
(c) a1 = 0, a2 ≠ 0, a3 = 0 (d) a1 ≠ 0, a2 ≠ 0, a3 = 0  0 2  1   2 
Solution: (d) 0 2 
M = [ B AB] =  .
1 2 
 x1   0 a1 0   x1  0 
 x  =  0 | M | ≠ 0.
 2  0 a2   x2  + 0  u3 .
∴  Controllable.
 x3   a3 0 0   x3  1 
Hence, the correct option is (c).

Chapter 08.indd 55 11/12/2015 10:25:11 AM


5.56 | Control Systems

Common Data for Questions 7 and 8: Solution: (b)


A system is described by the following state and e At = L−1[( SI − A) −1 ]
output equations:
 1 1 
dx1 (t ) s +3 ( s + 3)( s + 2) 
= −3 x1 (t ) + x2 (t ) + 2u (t ). = L−1  .
dt  1 
dx2 (t )  0 s+2 
= −2 x2 (t ) + u (t ).
dt
e −3t e −2t − e −3t 
y (t) = x1(t) when u (t) is the input and y (t) is the
 e At =  .
 0 e −2t 
output.
7. The system transfer function is [2009] Hence, the correct option is (b).
Common Data for Questions 9 and 10:
s+2 s+3
(a) 2 (b) The sate space equation of a system is described by
s + 5s − 6 s + 5s + 6
2

X = AX + BU , Y = Cx  where X is state vector, U


2s + 5 2s − 5 is input, Y is output and
(c) 2 (d)
s + 5s + 6 s + 5s − 6
2
0 1  0
Solution: (c) A=  B =   C = [1 0]
 0 −2  1
  TF = C[ SI − A]−1 B + D, 9. The transfer function G(s) of this system will be
 [2008]
 −3 1  2
Where, A =   , B = 1  , C = [1 0]. s s +1
 0 −2    (a) (b)
( s + 2) s ( s − 2)
 s + 3 −1  s 1
SI − A =  . (c) (d)
 0 s + 2  ( s − 2) s ( s + 2)
s + 2 1 
 0 Solution: (d)
s + 3
( SI − A) −1 =  . Y ( s)
( s + 3)( s + 2) TF = = C[ SI − A]−1 B + D.
U (s)
s + 2 1  2
[1 0] 
 0 s + 3 1  SI − A = 
 s −1 
( SI − A) −1 = . .
( s + 3)( s + 2) 0 s + 2 
2( s + 2) + 1  s + 2 1
TF = .  0
( s + 3)( s + 2) s 
( SI − A) −1 =  .
2s + 5 s ( s + 2)
  TF = 2 .
s + 5s + 6  s + 2 1  0 
[1 0] 
Hence, the correct option is (c).  0 s  1 
TF = +0
8. The state-transition matrix of the above system is s ( s + 2)
 [2009] 1 1
= 2 = .
 e −3t e −2t − e −3t  s + 2 s s ( s + 2)
 e −3t 0 
(a)
 −2t    (b)   
e + e
−3t
e −2t   0 e −2t  Hence, the correct option is (d).
1 0. A unity feedback is provided to the above system
 e −3t e −2t + e −3t   e −3t e −2t − e −3t 
(c)
    (d)    G(s) to make it a closed loop system as shown in
 0 e −2t   0 e −2t  figure. [2008]

Chapter 08.indd 56 11/12/2015 10:25:15 AM


Chapter 8  State Space Analysis  |  5.57

r (t ) + y (t) 12. The state transition equation [2005]


S G(s)
− t − e  −t
 t − e−t 
X (t ) =  − t     (b) X (t ) =  −3t 
(a)
 e   3e 
For a unit step input r (t), the steady state error in
the input will be  t − e −3t   t − e −3t 
(c)
X (t ) =  −3t     (d) X (t ) =  − t 
(a) 0 (b) l  3e   e 
(c) 2 (d) ∞
Solution: (c)
Solution: (a) t
1 X (t ) = e At X (0) + ∫ e A( t −τ ) BU (τ ) dτ .
G (s) = H ( s ) = 1.
s ( s + 2) 0

1 U (τ ) =  Unit step input.


Error to step Input = .
1+ K p  1 
1 (1 − e −3t )   −1
x(t ) =  3
K p = Lt G ( s ) H ( s ) = ∞.   3 
s →0
0 e −3t 
1
ess = = 0. t  1 
1+ ∞  1 (1 − e −3( t −τ ) )  1 
+∫ 3 (1)dτ
ess = 0.   0

0 0 e −3( t −τ )   
Hence, the correct option is (a).
 −e −3t  t 1   −e −3t  t 
Common Data for Questions 11 and 12: =  −3t  + ∫   dτ =  −3t  +   .
A state variable system 3e  0 0  3e  0 
 x1 (t )  t − e 
−3t
0 1  1
 X (t ) =   X (t ) +   u (t ) with the initial x(t ) =   =  .
 x2 (t )  3e
−3t
 0 −3  0 
condition X(0) = [−1 3]T and the unit step input Hence, the correct option is (c).
u (t) has
13. The state variable description of a linear autonomous
11. The state transition matrix [2005] system is, X = AX, where X is the two dimensional
 1 −3t   1 −t  state vector and A is the system matrix given by 
(a)  1 3 (1 − e )     (b)  1 (e − e −3t )   0 2
3 A=
     .  The roots of the characteristic equation
0 e −3t  0 e−t  2 0
are [2004]
 1 −t −3t 
(a) − 2 and + 2 (b) − j 2 and + j 2
 1 3 (e − e )   1 (1 − e − t ) 
(c)    (d)
  (c) − 2 and − 2 (d) + 2 and + 2
e −3t 
0  0 e −3t  Solution: (a)
Solution: (a) 0 2
A= .
 s −1  2 0
SI − A =  .
 0 s + 3 SI − A = 0.  Characteristic equation.
1 1 
( SI − A) −1 =  s s ( s + 3)  .  s −2 
  [ SI − A] =  .
 0 s + 3   −2 s 
 1  | SI − A | = s 2 − 4 = 0.
At −1  1 (1 − e −3t ) 
−1
e = L [( SI − A) ] = 3 . s = ± 2.
 
0 e −3t  ∴   Roots are 2 and −2.
Hence, the correct option is (a). Hence, the correct option is (a).

Chapter 08.indd 57 11/12/2015 10:25:19 AM


5.58 | Control Systems

14. The following equation defines a separately exited  2 3 1 


DC motor in the form of a differential equation 15. For the system X =   X + 0  u , which of
 0 5   
d 2ω B d ω K 2 K the following statements is true? [2002]
+ + ω= Va .
dt 2 J dt L J LJ (a) The system is controllable but unstable
The above equation may be organized in the state (b) The system is uncontrollable and unstable
space form as follows (c) The system is controllable and stable
 d 2ω  (d) The system is uncontrollable and stable
 2   dω 
 dt  = P  dt  + QVa , Solution: (b)
 dω   
   ω  s − 2 −3
  SI − A = = 0.
 dt  0 s −5
Where the P matrix is given by [2003]
( s − 2)( s − 5) + 3 = 0.
 B K2   K2 B
− − −
 (b) −  s − 2 s − 5s + 10 + 3 = 0.
2

(a)
 J LJ   LJ J
 1 0   0 1  s 2 − 7 s + 13 = 0 Characteristic equation.
  
Roots are right side of s-plane.
 0 1   1 0 
∴   System is unstable.
(c)  K 2 B
 (d)
 
− B − K
2
− −   Controllability matrix M = [ B
   AB ].
 LJ J  J LJ 
Solution: (a)  2 3 1   2 
[ AB ] =     =  .
dω  0 5  0   0 
Let x1 = .
dt 1 2 
M = [B AB] =  .
x2 = ω. 0 0 
d 2ω K B dω k 2 M = 0.
X 1 = 2 = Va − − ω.
dt Lj J dt LJ
∴   Not controllable.
K B k2
X 1 = Va − x1 − x2 . Hence, the correct option is (b).
LJ j Lj
dω 2 0 1
X 2 = = x1 . 16. For the system X =   X + 1 u; y = [4 0] X ,
dt  0 4  
∴   State space representation is with u as unit impulse and with zero initial state,
the output, y, becomes [2002]
 B K2   K  (a) 2e2t (b) 4e2t
 X 1   − −   x1 
  = J L J    +  L J  Va . (c) 2e4t (d) 4e4t
 X 2    x2   0 
 1 0    Solution: (b)
0 
 d 2ω  Input is given as impulse with  x(0) =   .
 dt 2   − B K 2   dω   K  0 
−  
 = J 
LJ   dt  +  L J  Va .
 dω   1
t

 dt  
0  ω  0  X (t ) = e At X (0) + ∫ e A( t −τ ) BU (τ )dτ .
0

 B K2  adj( SI − A)
− − e At = L−1[( SI − A) −1 ] = .
∴  Matrix P =  J 
LJ  . SI − A

 1 0  s − 2 0 
SI − A =  .
Hence, the correct option is (a).  0 s − 4 

Chapter 08.indd 58 11/12/2015 10:25:24 AM


Chapter 8  State Space Analysis  |  5.59

s − 4 0   1  e 2t  2t
 0 y (t ) = [4 0]x(t ) = [ 4 0]  4t  = 4 e .
2   s − 2
0 
s − e 2t 0 e
 
e At =  = = .
( s − 2)( s − 4)  1  0 e 4t  Hence, the correct option is (b).
0
 s − 4 
t
e 2 ( t −τ ) 0  1
∴ x(t ) = e At x(0) + ∫  4 ( t −τ )   
δ (t )dt
0  0 e  1
t
e 2 ( t −τ )  e 2t 
= ∫  4 ( t −τ )  δ (t )dt =  4t 
0 e  e 

Chapter 08.indd 59 11/12/2015 10:25:24 AM


5.60 | Control Systems

Thus,
Five-marks Questions
 x1  1 e − t   x1  0 
 x  =     +   u (t ).
1. Obtain a state variable representation of the sys-  2  0 −1   x2  1 
tem governed by the differential equation:
 x1 
d 2 y dy
2 + − 2 y = u (t )e − t , with the choice of state    y = [1 0]  x   (As x1 = y).
dt dt  2
 dy 
variables as x1 = y, x2 =  − y  et. Also find x2(t), Now,
 dt 
Input is step and x2(0) = 0.
given that u(t) is a unit step function and x2(0) = 0.
 [2002] x2 = u (t ) − x2 .
Solution:
 5 x2 ( s ) − x2 (0) = 1 − x2 ( s ).
 dy  s
x1 = y and x2 =  − y  et . (1)
 dx  x2 ( s ) =
1
⇒ x2 (t ) = 1 − e − t .
s ( s + 1)
d y dy 2
−t
As  2 + − 2 y = u (t )e . 2. Consider the state equation X (t ) = Ax(t ). Given:
dt dt
⇒ x1 = y . (2) e − t + te − t te − t   [2000]
e At =  −t −t −t 
 − te e − te 
dy
And x2 e − t = − y. (a) Find a set of states x1(1) and x2(1) such that
dx x1 (2) = 2.
(b) Show that
x2 e − t + y = y. (3)
1  s + 2 1
(sI − A) −1 = F(s) =  ;
∆  −1 s 
From Equation (2)
⇒ ∴ x1 = x2 e − t + y.    D = (s + 1)2.

x1 = x2 e − t + x1  (Given y = x1). (c) From F(s), find the matrix A.


Solution:
Differentiating Eq. (3)
x (t ) = Ax(t ).
(a)
yet + 
x2 =  yet − ye
 t − yet.
 x (t )  e − t + te − t +e−t   x1 (0) 
y + y − 2 y = u (t )e − t.
Given  e At =  1  =   .
 x2 (t )   −te e − te − t   x2 (0) 
−t −t

y = u (t )e − t + 2 y − y .

x1 (t ) = (e − t + te − t ) x1 (0) + te − t x2 (0).
x2 = (u (t )e − t − y + 2 y )et + ye
 t − ye
 + t − yet
−t −t −t
= u (t )e − t et − ye
 t + 2 yet + ye
 t − ye
 + t − yet   x2 (t ) = −te x1 (0) + (e − te ) x2 (0).
   
= u (t ) + 2 yet − yet − ye  t. Now,

 t t
  x2 = u (t ) − ye + ye .
 x1 (1) = (e −1 + e −1 ) x1 (0) + e −1 x2 (0)

= 0.74 x1 (0) + 0.32 x2 (0).
Now,
x2 = u (t ) − [ x2 e − t + x1 ]et + yet x2 (2) = (−e −2 + 2e −2 ) x1 (0) + 2e −2 x2 (0) = 2
t t = 0.40 x1 (0) + 0.28 x2 (0) = 2.
  = u (t ) − x2 + (− x1e ) + ye .
x2 = u (t ) − x2 − x1et + x1et x2 (2) = −e −2 x1 (0) + (e −1 − e −1 ) x2 (0) = 2.
= u (t ) − x2 . x2 (1) = 0.38 x1 (0).

Chapter 08.indd 60 11/2/2015 6:02:43 PM


Chapter 8  State Space Analysis  |  5.61

Now, x1(1) + x2(1)  s − a −b 


= .
  = 0.74 x1 (0) + 0.37 x2 (0) + 0.38 x1 (0).  −c s − d 

L[e At ] = [ SI − A]−1
(b)  s −1   s − a −b 
Compare with   and  .
 1 1 1  1 s + 2   −c s − d 
 s + 1 + ( s + 1) 2 ( s + 1) 2 
= . 0 1
 −1 1 1  ∴ A= .
 ( s + 1) 2 −  −1 −2 
 s + 1 ( s + 1) 2 
3. The state-space representation of a system is given
1  s + 2 1  X   −5 1   x1 
φ (s) = .
( s + 1) 2  −1 s  by  1  = 
    . Find the Laplace trans-
 X 2   −6 0   x2 
0  form of the state transistion matrix. Find also the
Input is given as impulse with x(0) =   . value of x1 at t = 1 if x1(0) = 1 and x2 (0) = 0.
0  [1998]
t
x(t ) = e At x(0) + ∫ e A( t −τ ) BU (τ ) dτ . Solution:
0
 −5 1 
At −1−1
A=
   e = L [( SI − A) ] .
 −6 0 
adj( SI − A)
= .
| SI − A | The state transition matrix = e At = L−1[( SI − A) −1 ].

s − 2 0   s + 5 −1
SI − A =  . SI − A = 
4 
.
 0 s −    6 s 
s − 4 0   1  s 1  s 1 
 0 s − 2   s − 2
0   −6 s + 5  −6 s + 5
e At =  =  [ SI − A]−1 =  =  
( s − 2)( s − 4)  1  s ( s + 5) + 6 ( s + 2)( s + 3)
0
 s − 4 
    
 s 1 
e 2t 0  ( s + 2)( s + 3)
= . ( s + 2)( s + 3) 
0 e 4t  = .
 −6 s+5 
t
e 2 ( t −τ )  ( s + 2)( s + 3) ( s + 2)( s + 3) 
0  1 
∴ x(t ) = e At x(0) + ∫    δ (t ) dt
4 ( t −τ )
0  0 e  1
 −2e −2t + 3e −3t e −2t − e −3t 
t
e 2 ( t −τ )  e 2t  L−1[( SI − A) −1 ] =  −2 t −3t −2 t −3t 
.
= ∫  4 ( t −τ )  δ (t )dt =  4t  .   6(−e + e ) 3e − 2e 
0 e  e 

And, y (t ) = [4 0] x(t ) x(t ) = e At x(0)

e 2t 
= [4 0]  4t  = 4e 2t .  −2e −2t + 3e −3t e −2t − e −3t  1 
e  =   .
6(−e −2t + e −3t ) 3e −2t − 2e −3t  0 
   
1  s + 2 1
φ ( s ) = [ SI − A] =
(c) −1
 .  −2e −2t + 3e −3t 
( s + 2) 2  −1 s  x(t ) =  .
−2 t −3t 
6(−e + e ) 
 s 0 a b 
SI − A =  − 
0 s   c d  x1 (t ) = 0.12.  At t = 1 sec.

Chapter 08.indd 61 11/2/2015 6:02:49 PM


5.62 | Control Systems

4. Determine the transfer function of the system hav-  −1 0 


a21 = (−1) 2 +1  = (−1)(−1)( s + 14).
14 
ing the following state variable representation:
 44 s +
 [1997]
s 0 
 0 1 0  0  a22 = (−1) 2 + 2   = s ( s + 14).
X =  0 0 1  x + 1  u.  40 s + 14 
 −40 −44 −14  0   s −1
a23 = (−1) 2 + 3   = (−1)[44 s + 40].
 40 44 
Y = [0 1 0]x.

 −1 0 
Solution: a31 = (−1) 4   = 1.
 s −1
 0 1 0  0  s 0 
x =  0 0 1  x + 1  u. a32 = (−1)3+ 2   = s.
0 −1
 −40 −44 −14  0 
 s −1
a33 = (−1)3+ 3   = s2 .
y = [0 10]x 0 s 

T
 s −1 0   s 2 + 14 s + 44 −40 −40 s 
  
−1  .  s + 14 s + 14 s −(44 s + 40) 
2
⇒ SI − A =  0 s
 40 44 s + 14   1 s s2 
  [ SI − A]−1 =   .
SI − A
 a11 a12 a13 
( SI − A) −1 =  a21 a22 a23  Cofactor matrix. C ( SI − A) B + D
TF =
 a31 a32 a33  SI − A

s −1   s 2 + 14 s + 44 s + 14 1  0 
a11 = (−1)1+1   = s + 14 s + 44.
2
 
 44 s + 14  [0 10]  −40 s + 14 s
2
s  1 

 −40 s −(44 s + 40) s 2  0 
0 −1  = 
a12 = (−1)1+ 2  = −40.
 40 s + 14  | SI − A |

0 s 0
a13 = (−1)1+ 3   = −40 s. = [−40 s 2 + 14 s s ] 1  .
 40 44 
 0 

s 2 + 14 s
TF = .
s 2 + 14 s 2 + 44 s + 40

Chapter 08.indd 62 11/2/2015 6:02:53 PM


Unit 6
Electrical and Electronics
Measurements
Chapter 1:  Fundamentals and Error Analysis 6.3
Chapter 2:  Indicating Instruments 6.5
Chapter 3:  Measurement of Voltage and Current 6.7
Chapter 4:  Measurement of Resistance 6.15
Chapter 5:  Bridge Measurement, Measurement of Induction and Capacitance 6.17
Chapter 6:  Extension of Instrument Ranges 6.21
Chapter 7:  Potentiometer 6.22
Chapter 8:  Measurement of Power and Energy 6.23
Chapter 9:  Measurement of Phase and Frequency 6.32
Chapter 10:  Potentiometric Recorders and Q-Meter 6.34
Chapter 11:  Oscilloscope 6.36
Chapter 12:  Electronic Multimeters and Digital Voltmeters 6.41
Chapter 13:  Instrument Transformer 6.43

Chapter 01.indd 1 11/9/2015 5:50:33 PM


Chapter 01.indd 2
Exam Analysis
Exam Year 91 92 93 94 95 96 97 98 99 00 01 02 03 04 05 06 07 08 09 10 11 12 13 14
1 Mark Questions 1 2 2 3 5 5 3 1 1 1 1 3 3 2 3 2 1 2 1 2 3 4 2 4
2 Marks Questions – – – 1 – – – 1 3 1 2 – 8 7 5 5 1 2 2 1 1 1 1 4
3 Marks Questions – – – – – 1 – – – – – – – – – – – – – – – – – –
5 Marks Questions – – – – – – – 1 1 1 – – – – – – – – – – – – – –
Total Marks 1 2 2 3 5 6 3 3 5 3 3 3 11 9 8 7 2 4 3 3 4 5 3 8
Fundamentals and Error Analysis – – 1 1 1 – – – 1 – – – – – – 1 – – – – – – – 1
Indicating Instruments – – – – – – – – – – – – – 1 – – – – – 1 – – – 1
Measurement of Voltage and 1 1 – 1 2 2 – 1 2 – – – 4 1 2 1 – – 1 – – 2 2 3
Current
Measurement of Resistance – – – – 1 – – – – – 1 – – – 1 – – – – – – – – –
Bridge Measurement, Measurement – – – – – – – – – – – 1 1 1 – 1 – 1 – 1 1 1 1 –
of Induction and Capacitance
Extension of Instrument Ranges – – – – – – – – – – 1 – – – 1 – – – – – – – – –
Potentiometer – 1 – 1 – – 2 – – – – – – – – – – – – – – – – –
Measurement of Power and Energy – – 1 – – 2 1 1 1 2 1 1 2 3 1 2 1 – 1 1 1 1 – 2
Measurement of Phase and – – – – – 1 – – – – – – – – – – – 1 – – – 1 – –
Frequency
Potentiometric Recorders and – – – – – – – 1 1 – – – 1 – 1 – – – – – – – – –
Q-Meter
Oscilloscope – – – 1 1 1 – – – – – 1 1 1 1 1 1 1 1 – 1 – – 1
Electronic Multimeters and Digital – – – – – – – – – – – – 1 – 1 – – 1 – – 1 – – –
Voltmeters
Instrument Transformer – – – – – – – – – 1 – – 1 2 – 1 – – – – – – – –

11/9/2015 5:50:33 PM
Chapter 1
Fundamentals and Error
Analysis
Resistance;  R = 120 Ω.
One-mark Questions To find: Change in resistance; ∆R = ?
∆R/R
1. Fringing in capacitive type transducer can be mini- Gf = ⇒ ∆R = G f × ε × R
mized by providing a . [1995] ε
Solution: = 2 × 10−5 × 120 Ω
Fringe effect is when lines of flux leap outside the = 2.4 × 10−3 Ω
edges of the capacitor plates. Due to this, capaci-  2.5 × 10−3 Ω.
tive transducers many a times show non linear be- Hence, the correct option is (c).
havior which can be eliminated using guard rings.
2. A precise measurement guarantees accuracy of the
measured quantity. [1994]
Two-marks Questions
Solution: 1. Suppose that resistor R1 and R2 are connected in
False parallel to give an equivalent resistor R. If resistors
Precision is necessary but not sufficient condition R1 and R2 have tolerance of 1% each, the equivalent
for accuracy. Accuracy means conformity to truth resistor R for resistors R1 = 300 Ω and R2 = 200 Ω
while precision refers to consistency or repeatabil- will have tolerance of [2014-S2]
ity of results. (a) 0.5% (b) 1%
3. A metal strain gauge has gauge factor of two, (c) 1.2% (d) 2%
it’s nominal resistance is 120 W. It undergoes a Solution: (b)
strain of 10 -5, the value of change of resistance in Given, Resistor R1 = 300 Ω.
response to the strain is [1993]
Tolerance of R1 , ∆R1 = 1%
(a) 240 W (b) 2 × 10−5 Ω
(c) 2.5 × 10−3 Ω (d) 1.2 × 10−3 Ω Resistance R2 = 200 Ω.
Tolerance of R2 ; ∆R2 = 1%
Solution: (c)
Gauge factor; Gf = 2. R1 R2
∵ R1 R2 .  Hence, Requ =
∆l R1 + R2
ε=
Stress;  = 10−5 ,
l ( R1 ± ∆R1 )( R2 ± ∆R2 )
=
Where, l =  length;  ∆l =  change in length. ( R1 ± ∆R1 ) + ( R2 ± ∆R2 )

Chapter 01.indd 3 11/9/2015 5:50:36 PM


6.4 | Electrical and Electronics Measurements

R1 R2 ± (∆R1 + ∆R2 ) 1
= y = 20; ∆y =
For  × 100 = 5 % rdg.
( R1 ± ∆R1 ) + ( R2 ± ∆R2 ) 20
(300 × 200) ± (1% + 1%) ∴ ∆w = ± (∆x + ∆y + ∆z )
= = ±(0.5 + 5 + 1.5)% rdg = ± 7% rdg.
(300 ± 1% of 300)(200 ± 1% of 200)
60000 ± 2% Hence, the correct option is (d).
=
(300 ± 3) + (200 ± 2) 3. Two 100 μA full scale PMMC meters are employed
60000 ± 2% to construct a 10 V and a 100 V full scale voltmeter.
= These meters will have figures of merit (sensitivities)
500 ± 5
as [1999]
60000 ± 2% (a) 10 kΩ/V and 10 kΩ/V
= = 120 ± 1%.
500 ± 1% (b) 100 kΩ/V and 10 kΩ/V
Hence, the correct option is (b). (c) 10 kΩ/V and 100 kΩ/V
2. A variable w is related to three other variables x, y, z (d) 10 kΩ/V and 1 kΩ/V
as w = xy/z. The variables are measured with meters Solution: (a)
of accuracy ± 0.5% reading, ±1% of full scale value Maximum current that can be measured; I = 100
and ±1.5% reading the actual readings of the three μA = 100 × 10-6 A.
meters are 80, 20 and 50 with 100 being the full Full scale voltage of 1st voltmeter = Va (say) = 10 V.
scale value for all three. The maximum uncertainty
Full scale voltage of 2nd voltmeter = Vb(say) = 100 V.
in the measurement of w will be [2006]
(a) ± 0.5% rdg (b) ± 5.5% rdg To find: Sensitivities/Figures of merit for VA and VB,
i.e., SVA = ? and SVB = ?.
(c) ± 6.7% rdg (d) ± 7.0% rdg
Now, we know, Sensitivity; Resistance R
Solution: (d) = S =
voltage V
Given variables w, x, y, z are related by w = xy/z. 1
 = [From Ohm’s law V = IR]
Also, Δ x = ±0.5% rdg; Δ is used to represent I
uncertainty in the measurement of the variable. 1
= A−1 = 10 × 103 Ω /V = 10 kΩ /V.
∆ z = ±1.5% rdg. 100 × 10−6
  
∆y = ± 1% full scale ∴ SVA = SVB = 10 kΩ /V.

= ± 1% ∫ 100 = 1. Hence, the correct option is (a).

Chapter 01.indd 4 11/9/2015 5:50:37 PM


Chapter 2
Indicating Instruments

0.2
One-mark Questions R= .
4
1. An ammeter has a current range of 0 - 5 A, and its R = 0.05 Ω.
internal resistance is 0.2 W. In order to change the Hence, the correct option is (d).
range to 0 - 25 A we need to add a resistance of
2. A galvanometer with a full scale current of 10 mA
[2010]
has a resistance of 1000 W. The multiplying power
(a) 0.8 W in series with the meter.
(the ratio of measured current to galvanometer
(b) 1.0 W in series with the meter.
current) of a 100 W shunt with this galvanometer
(c) 0.04 W in parallel with the meter.
is [2004]
(d) 0.05 W in parallel with the meter.
(a) 110 (b) 100
Solution: (d) (c) 11 (d) 10
Range of current ammeter = (0 - 5 A).
Solution: (c)
Internal resistance; r = 0.2 W. For a galvanometer,
To change to new range, i.e., 0 - 25 A. Full scale current; If = 10 mA.
Resistance required to add R = ? Resistance of galvanometer; Rg = 1000 W.
To increase current range, resistance should be Resistance shunt with Rg; Rsh = 100 W.
lowered; thereby we need to connect a shunt resist-
To find: multiplying factor;
ance across the given ammeter.
Measured current; I (say) I
0.5 A 0.2 Ω m= = .
Galvanometer current; I f If
I Im r

I A If Rg B
I − Im R

From the figure,


I − If Rsh
R
Im = (I − Im ) .
r From the diagram,
I r
−1 = . I f Rg = ( I − I f ) Rsh .
Im R
25 0.2 I − If Rg
−1 = . = .
5 R If Rsh

Chapter 02.indd 5 11/2/2015 2:02:05 PM


6.6 | Electrical and Electronics Measurements

I Rg Similarly, for ammeter Y, from the figure, at full


⇒ −1 = . scale
If Rsh
I Im 2 r2
I Rg
⇒ = + 1 = m.
If Rsh
Rg 1000
∴ m = 1+ = 1+ = 11.
Rsh 100 I − Im2 R2

Hence, the correct option is (c). I m 2 = 250 mA = 0.25 A, I = 15 A, r = 1.5 Ω.
Im2 r 0.25 × 1.5
R2 = = = 0.015 Ω.
Two-marks Questions I − I m 2 15 − 0.25
When X and Y are connected in parallel. Equiva-
1. Two ammeters X and Y have resistances of 1.2 W lent resistance of ammeter X along with shunt
and 1.5 W respectively and they give full-scale
deflection with 150 mA and 250 mA respectively. r1 R1 12 × 0.12
Rx = = = 0.0118 Ω.
The ranges have been extended by connecting r1 + R1 0.12 + 12
shunts so as give full scale deflection with 15 A.
The ammeters along with shunts are connected in Similarly, for Y ammeter
parallel and then placed in a circuit in which cur-
r2 R2 15 × 0.15
rent flowing is 15 A. The current in amperes indi- Ry = = = 0.0245 Ω.
cated in ammeter X is . [2014-S2] r2 + R2 15 + 0.15
Solution: Using current division,
For ammeter X, from the figure, at full scale,
Ry 0.0245
I r1 Ix = I = 15 × = 10.1 A.
Rx + Ry (0.0118 + 0.0245)
Im1
I = 15 A Ix r1

I − Im1 R1
Ammeter X Iy
R1 r2
I m1 = 150 mA = 0.150 A.
I = 15 A.
r1 = 1.2 Ω. R2

We know, Ix Rx = 0.0118 Ω

  I m1r1 = ( I − I m1 ) R1 (1) 15 A

I m1r 0.150 × 1.2


R1 = = = 0.012 Ω.
( I − I m1 ) 15 − 0.15 Iy Ry = 0.0245 Ω

Chapter 02.indd 6 11/2/2015 2:02:07 PM


Chapter 3
Measurement of V
  oltage
and Current
Solution: (a)
One-mark Questions •• The diode given is ideal diode and the PMMC
voltmeter has finite resistance.
1. The saw-tooth voltage waveform shown in the figure •• During positive half cycle of input, the diode
is fed to a moving iron voltmeter. Its reading would gets forward biased whereas during negative half
be close to _________ [2014-S2] cycle of input, the diode gets reverse biased.
100 V
•• PMMC voltmeter reads average value of voltage
across 10 kW resistor (which is half wave recti-
fied sinusoidal voltage).
PMMC voltmeter reading = VDC
= I DC × 100 kΩ
t
20 ms 40 ms = 0.318 × I m × 100 kΩ
Solution:
14.14
Moving iron voltmeter reads rms value, when AC = 0.318 × × 100 kΩ
is given rms value of saw tooth wave is; 101 kΩ
Vm 100 = 4.46 V.
V=
rms = = 57.7 V.
3 3 Hence, the correct option is (a).
2. The input impedance of the permanent magnet mov- 3. Three moving iron type voltmeters are connected
ing coil ( PMMC ) voltmeter is infinite. Assuming as shown below. Voltmeters readings are V, V1 and
that the diode shown in figure below is ideal, the V2, as indicated. The correct relation among the
reading of the voltmeter in volts is [2013] voltmeter reading is [2013]
1 kΩ −j1 Ω j2 Ω

14.14 sin −
∼ (314 t) V 100 Voltmeter V V1 V2 I
kΩ +

V1 V2
V=
(a) V = V1 + V2
(b) +
(a) 4.46 (b) 3.15 2 2
(c) 2.23 (d) 0 V = V1V2 (d)
(c) V = V1-V2

Chapter 03.indd 7 11/2/2015 2:17:15 PM


6.8 | Electrical and Electronics Measurements

Solution: (d) 5. A PMMC voltmeter is connected across a series


V2 combination of a DC voltage source V1 = 2 V and
an AC voltage source V2 (t ) = 3 sin(4t )V .  The meter
reads [2005]
I
2V +

V1
So, V = V2 - V1. 3 sin 4t ∼
Hence, the correct option is (d).
4. A periodic voltage waveform observed on an oscil-
loscope across a load is shown. A permanent mag- (a) 2 V (b) 5 V
net moving coil (PMMC) meter connected across
 3 17
(c)  2 +  V (d) V
the same load reads [2012]  2  2
V(t )
Solution: (a)
PMMC meter reads only DC.
10 V Hence, the correct option is (a).
6. A Manganin swamping resistance is connected in
5V series with a moving coil ammeter consisting of a
milli-ammeter and a suitable shunt in order to
time (ms) [2003]
0 10 12 20
(a) Minimize the effect of temperature variation
−5 V
(b) Obtain large deflecting torque
(a) 4 V (b) 5 V (c) Reduce the size of the meter
(c) 8 V (d) 10 V (d) Minimize the effect of stray magnetic fields
Solution: (a) Solution: (a)
V(t ) This swamping resistance is made up of constant
temperature coefficient materials like manganin
10 V and constantan for reducing the temperature errors.
5V Hence, the correct option is (a).
7. The effect of stray magnetic fields on the actuating
time (ms) torque of a portable instrument is maximum when
0 10 12 20
the operating field of the instrument and the stray
−5 V fields are [2003]
PMMC measures average value: (a) Perpendicular (b) Parallel
(c) Inclined at 60° (d) Inclined at 30°
1  
10 12 20
Average value =  ∫ (1) t dt − ∫ 5 dt + ∫ 5 dt  Solution: (b)
20  0 10 12  The effect of stray magnetic fields on the actuating
torque of a portable instrument is maximum when
1  t2 
10

=  (1) − 5t |12
10 +5t |12 
20 the operating field of the instrument and the stray
20  2 0  fields are parallel.

1 1  Hence, the correct option is (b).
= [100 − 0] − 5[12 − 10] + 5[20 − 12]
20  2  8. A DC voltmeter has a sensitivity of 1000 W/volt.
When it measures half full scale in 100 V range,
1 80
= [50 − 10 + 5 × 8] = = 4 V. the current through the voltmeter is [1998]
20 20 (a) 100 mA (b) 1 mA
Hence, the correct option is (a). (c) 0.5 mA (d) 50 mA

Chapter 03.indd 8 11/2/2015 2:17:16 PM


  oltage and Current  |  6.9
Chapter 3  Measurement of V

Solution: (c) Full Scale Accuracy


500 Instrument Type
Value (A) % of FS
100 V
0.5 mA 1 mA 1
M1 3 digit 20 ± 0.10
2
dual slope
M2 PMMC 10 ± 0.20
S DC = 1000 Ω /V I mfsd = 11000
/ = 1 mA.
M3 Electro-dynamic 5 ± 0.50
At half full scale, the reading will be 0.5 mA. M4 Moving-iron 1 ±1.00
(OR)
A current of 1 A is to be measured. To obtain mini-
mum error in the reading, one should select meter
DC (a) M1 (b) M2
50 V meter (c) M3 (d) M4
Solution: (d)
0.10
Rv
M 1 → Error = 20 × ⇒ ± 0.02.
100
Rv 0.10
S= . M 2 → Error = 10 × ⇒ ± 0.02 .
V fsd 100
Ω 0. 50
Rv = 1000 × 100 V ⇒ 100 kΩ. M 3 → Error = 5 × ⇒ ± 0.021.
V 100
50 1. 00
∴ Im = ⇒ 0.5 mA.
100 × 103 M 4 → Error = 1× ⇒ ± 0.01.
100
Hence, the correct option is (c). M 4 has less error on full scale.
9. An advantage of a permanent magnet moving coil
Hence, the correct option is (d).
instrument is that it is [1996]
(a) Free from friction error 1 2. Two 100 V full scale PMMC type DC voltmeter
(b) Has high (torque/weight of the moving parts) ratio having figure of merits (FOM) of 10 kW/V and
(c) Has low (torque/weight of the moving parts) ratio 20 kW/V are connected in series. The series com-
(d) Can be used on both AC and DC bination can be used to measure a maximum DC
voltage of __________. [1995]
Solution: (b)
The friction error of the PMMC instrument is less, Solution:
this error can be decided by torque/weight ratio. 0.05 mA
V1 V2
The friction error is less only when ratio is high.
100 V 100 V
Hence, the correct option is (b).
10. The scale of a voltmeter is uniform. Its type is Voltmeter V1:        Voltmeter V2:
 [1996] S DC1 = 10 kΩ/ V.      S DC 2 = 20 kΩ/ V.
(a) Moving iron 1 1
(b) Induction I fsd1
= = 0.1 mA.   I=
fsd 2 = 0.05 mA.
S DC1 S DC 2
(c) Moving coil permanent magnet
(d) Moving coil dynamometer The maximum allowable current in this combina-
Solution: (c) tion is 0.05 mA, since both are connected in series.
Hence, the correct option is (c). Maximum DC voltage can be measured as 0.05 mA
11. Four ammeters M1, M2, M3 and M4 with the follow- (10 kΩ/V × 100 + 20 Ω/V × 100)
ing specifications are available. [1995] = 3000 × 0.05 = 150 V.

Chapter 03.indd 9 11/2/2015 2:17:19 PM


6.10 | Electrical and Electronics Measurements

13. A 0 - 10 mA PMMC ammeter reads 4 mA in a Solution:


circuit. Its bottom control spring snaps suddenly.
 12 A + 5 A 
The meter will now read nearly [1994] VDC = I DC × 10 Ω =   × 10 = 85 V.
(a) 10 mA (b) 8 mA  2 
(c) 2 mA (d) Zero
Solution: (d) Two-marks Questions
Springs are in series with the moving coil, i.e., cur-
rent is passing through the spring and to the mov- 1. The reading of voltmeter (rms) in volts, for the cir-
ing coil. If spring snaps suddenly, current passing cuit shown in the figure is . [2014-S1]
to the coil is zero and meter reading will be zero.
R = 0.5 Ω
Hence, the correct option is (d).
14. An unshielded moving iron voltmeter is used to meas-
ure the voltage in an AC circuit. If a stray DC magnetic
field having a component along the axis of the meter 1j Ω 1

j
coil appears, the meter reading would be [1992]
100 sin (wt ) V
(a) Unaffected
(b) Decreased 1
Ω 1j Ω
j
(c) Increased
(d) Either decreased or increased depending on
the direction of the DC field
Solution: (d)
An unshielded moving iron voltmeter is used to Solution:
measure the voltage in an AC circuit. If a stray DC 0.5 Ω
magnetic field having a component along the axis
of the meter coil appears, the meter reading would +
100∠ 100∠
be either decreased or increased depending on the
direction of the DC field. jl −jl

Hence, the correct option is (d). 100∠0 V V


+ −
15. The current i(t) is passing through 10 W resistor −jl jl
as shown in Figure A and as a waveform as shown
in Figure B. Then the reading of the DC voltmeter −
connected across 10 W resistor is [1991]
i(t) 10 Ω

1
Z eq = 0.5 + [0] = Ω.
2
V
100∠0
DC voltmeter Iγ = = 200 ∠0 A.
Y2
Figure (A)
i(t) Current division rule (See figure)
So, KVL
12 A −V − j (100 ∠0) − j (100 ∠0) = 0.
5A V = − j 200 ∠0.
V = −[200∠90].
t
0 T 2T 3T
Vmax 200
Vrms = = = 141.42 V.
Figure (B) 2 2

Chapter 03.indd 10 11/2/2015 2:17:20 PM


  oltage and Current  |  6.11
Chapter 3  Measurement of V

2. The DC current flowing in a circuit is measured 80 kΩ


by two ammeters, one PMMC and another electro- I1
dynameter type, connected in series. The PMMC
meter contains 100 turns in the coil, the flux den- V x
sity in the air gap is 0.2 Wb/m2, and the area of the
coil is 80 mm2. The electrodynamometer ammeter
has a change in mutual inductance with respect
Total resistance ( R2 ) = 80 kΩ + x.
to deflection of 0.5 mH/deg. The spring constant
of both meters are equal. The value of current, in Voltmeter reading (V2 ) = 352 V.
which the deflection of the two meters are same,
is _________ [2014-S1] V2 ∝ I1 x.
Solution: 1
I2 ∝ .
Deflection of two meters are same θ PMMC = θ EMMC . R2
Spring constants of two meters are same
V2 I 2 R1
TCPMMC = TCEMMC . = = .
V1 I1 R2
Now, θ PMMC = θ EMMC .
352 20 + x
BINA I 2 d m = .
= . 440 80 + x
KC K C dθ ⇒ 352(80 + x) = (20 + x)(440).
BNA 0.2 × 100 × 80 × 10−6
I= = = 3.2 mA. ⇒ 440 x − 352 x = 352 × 80 − 440 × 20.
 dN  0.5
  ⇒ 88 x = 28160 − 8800.
 dθ 
3. An analog voltmeter uses external multiplier set- x = 220 kΩ
Ω.
tings. With a multiplier setting of 20 kW, it reads For multiplier setting = 40 kW.
440 V and with a multiplier setting of 80 kW, it
reads 352 V. For a multiplier setting of 40 kW, the Total resistance ( R3 ) = 40 + 220b = 260 kΩ.
voltmeter reads [2012]
V3 I 3 R1
(a) 371 V (b) 383 V = = .
(c) 394 V (d) 406 V V1 I1 R3
Solution: (d) V3 20 + 220
= ⇒ V3 = 406 V.
For multiplier setting = 20 kW. 440 40 + 220
Meter resistance = x.
Hence, the correct option is (d).
Total resistance = R1 = 20 kΩ + x.
4. An average-reading digital multimeter reads 10 V
Voltmeter reading  (V1 ) = 440 V. when fed with a triangular wave, symmetric about
20 kΩ the time axis. For the same input an rms reading
meter will read. [2009]
I1
20 10
(a) (b)
V x 3 3
(c) 20 3 (d) 10 3
Solution: (a)
V1 ∝ I1 x.
=Vrms ?=Vavg 10.
1
I1 ∝ .
R1 For triangular waveform
For multiplier setting = 80 kW. Vm
Vavg = .
Meter resistance = x. 2

Chapter 03.indd 11 11/2/2015 2:17:22 PM


6.12 | Electrical and Electronics Measurements

Vm 7. A moving coil of a meter has 100 turns, and a


10 = ⇒ Vm = 30. length and depth of 10 mm and 20 mm r­ espectively.
2
Vm 20 It is positioned in a uniform radial flux density of
Vrms = = V. 200 mT. The coil carries a current of 50 mA. The
3 3 torque on the coil is [2004]
Hence, the correct option is (a). (a) 200 μNm (b) 100 μNm
5. A current of −8 + 6 2 (sin ωt + 30°) A is passed (c) 2 μNm (d) 1 μNm
though three meters. They are a center zero PMMC Solution: (a)
meter, true rms meter and moving iron instrument.
L = 10 mm, I = 50 mA, B = 200 mT, D = 20 mm.
The respective reading (in A) will be [2006]
(a) 8, 6, 10 (b) 8, 6, 8 A = L × D.
(c) - 8, 10, 10 (d) - 8, 2, 2 T =?
Solution: (c) T = BINA Nm
PMMC → PMMC reads.
Only DC value, = 200 × 10−3 × 50 × 10−3 × 100 × 10 × 10−3 × 20 × 10−3
i.e., -8 V (because centre zero meter) and true rms = 200 µNm.
meter and moving iron instruments reads rms values. Hence, the correct option is (a).
2
6 2 8. The inductance of a certain moving-iron ammeter
∴ I rms = (−8) 2 +   = 100 = 10. θ2
 2  is expressed as L = 10 + 3θ − µH, where θ is the
4
The readings of the meters one -80, 10, 10. deflection in radians from the zero position. The
Hence, the correct option is (c). control spring torque is 25 × 10−6 Nm/radian.  The
deflection of the pointer in radian when the meter
6. A 1000 V DC supply has two 1-crore cables as its pos- carries a current of 5 A, is [2003]
itive and negative leads; their insulation resistances (a) 2.4 (b) 2.0
to earth are 4 MW and 6 MW respectively, as shown (c) 1.2 (d) 1.0
in the figure. A voltmeter with resistance 50 kW
is used to measure the insulation of the cable. Solution: (c)
When connected between the positive core and L = 10 + 3θ − θ 2/4 µH.
earth, then voltmeter reads [2005]
K C = 25 × 10−6 Nm/radian , I = 5 A θ = ?
1000 V
4 MΩ 6 MΩ
In moving iron meter deflection angle,
1 2 dL
θ= I
V 2KC dθ
1  25   θ 1 θ 
=  −6  
30 −  × 10−6 =  3 −  .
(a) 8 V (b) 16 V 2  25 × 10   2 2 2
(c) 24 V (d) 40 V ⇒ θ = 1.2 rad.
Solution: (a)
4 × 0.05 Hence, the correct option is (c).
Reff = 4 MΩ /50 kΩ = ⇒ 49.38 kΩ.
4.05 9. An AC voltmeter uses the circuit shown below,
From voltage division rule where the PMMC meter has an internal resistance
 49.38 kΩ  of 100 W and requires a DC current of 1 mA for
Voltmeter reading  = 1000   full scale deflection. Assuming the diodes to be
 49 .38 kΩ + 6000 kΩ  ideal, the value of Rs to obtain full scale deflection
      = 8.16 V. with 100 V (AC rms) applied to the input terminal
Hence, the correct option is (a). would be [2003]

Chapter 03.indd 12 11/2/2015 2:17:24 PM


  oltage and Current  |  6.13
Chapter 3  Measurement of V

Rs (a) 5 A ± 1%
(b) ( 2 + 3 2 ) A ± 1%
∼ +
PMMC (c) 3 A ± 1.7%
AC Input + − meter
∼ (d) 2 A ± 0.5%
Solution: (c)
(a) 80 kW (b) 89 kW Thermo couple type meter reads rms value of current
(c) 89.9 kW (d) 90 kW 2 2
 2 2 2
Solution: (c) I rms = (2) + 
2
 +  
 2  2 
RS = RV − 2 Rd − Rm
100 V = 9 ⇒ 3 A.
= 0.9 × − 0 − 100 Ω = 89.9 kΩ.
1 mA For Class 1 meter, error is 1%.
Hence, the correct option is (c). 1
So error for given 5 A is 5 × = 0.05.
10. A current of ⎣2 + 2 sin(314t + 30) + 2 2 cos 100
(952t + 45)⎦ is measured with a thermocouple 0.05
Then % error for 3 A = × 100 = 1.67%.
type, 5 A full scale, class 1 meter. The meter read- 3
ing would lie in the range, [1999] Hence, the correct option is (c).

Chapter 03.indd 13 11/2/2015 2:17:25 PM


6.14 | Electrical and Electronics Measurements

Circuit is half wave rectifier circuit A1 meter reads


Five-marks Questions V
rms value of current I rms = rms .
R
1. For the given circuit the internal resistances of the For half wave rectifier
ammeter as well as that of the current coil of the
Vm Vrms 2 230 × 2
wattmeter are zero, and the voltmeters have a very Vrms = = = .
large figure of merit. A1 is a 6 A full scale moving 2 2 2
iron type meter. A2 is a commercial full wave recti- 230 2
I rms = ⇒ 113.53 A.
fier type meter of full scale 5 A. V1 is a 500 V meter 2 × 46
is a peak response type. V2 is 300 V PMMC type
A2 meter reads average value. But indicates rms
meter and W is an electro dynamometer type 5 A,
value of current, i.e., I rms = 3.53 A.
230 V wattmeter. Determine the reading of A1, A2,
V1, V2 and W. [1999] Vm 23 × 2
A2 meter reads I avg = = ⇒ 2.25 A.
π × R π × 46
A1 W A2
M L
V1 → reads peak value.
C V R
230 V Vm = Vrms × 2 = 230 × 2 = 325.27 V.
V 50 Hz V1 V2 46
V2 → reads average value.

Vm 2
Vaverage = = 230 × = 103.53 V.
Solution: π π
Vrms = 230 V, f = 50 Hz and R = 46 Ω. Wattmeter reads average power
A1 → 6 A   Full scale moving iron type meter. W = 103.53 × 2.25 = 232.95 W.
A2 → 5 A   Full scale full wave rectifier type meter.
V1 → 500 V   Peak response type meter.
V2 → 300 V   PMMC type meter.

Chapter 03.indd 14 11/2/2015 2:17:28 PM


Chapter 4
Measurement
of Resistance
Solution: (d)
One-mark Questions IL RL IR
A
1. A Kelvin double bridge is best suited for the meas- 0.01 Ω
urement of [1995] V 2A
IV
(a) Inductance (b) Capacitance
180 V, 2000 Ω
(c) Low resistance (d) High resistance
Given, voltmeter reading; VR = 180 V.
Solution: (c)
Kelvin’s double bridge takes into account the meas- Ammeter reading; IR = 2 A.
urement of winding resistances of machines and Voltmeter resistance; rv = 2000 Ω.
contact and earth conductor resistances which are Ammeter resistance; ra = 0.01 Ω.
negligible in case we are measuring medium or high 180
resistances and comparable only to low resistance Measured value of resistance; R = = 90 Ω.
2
Hence, the correct option is (c). VR 180
From the figure, I=V = = 0.09.
rv 2000
Two-marks Questions True current; IL = IR - IV = (2 - 0.09) A = 1.91 A.
180
1. The set-up in the figure is used to measure resist- 180 V = 1.91 RL ⇒ RL = .
1.91
ance R. The ammeter and voltmeter resistance are
RL = 94.24 Ω.
0.01 W and 2000 W, respectively. Their readings
are 2 A and 180 V, respectively, giving a measured RL − R
% error = × 100
resistance of 90 W. The percentage error in the R
measurement is [2005] 94.24 − 90
= × 100 = 4.71%.
RL 90
A
Hence, the correct option is (d).
V 2. Resistance R1 and R2 have, respectively, nominal
values of 10 W and 5 W, and tolerances of ± 5%
(a) 2.25% (b) 2.35% and ± 10%. The range of values for the parallel
(c) 4.54% (d) 4.71% combination of R1 and R2 is [2001]

Chapter 04.indd 15 10/30/2015 2:45:14 PM


6.16 | Electrical and Electronics Measurements

(a) 3.077 – 3.636 W (b) 2.805 – 3.371 W For the resistances in parallel,


(c) 3.237 – 3.678 W (d) 3.192 – 3.435 W R1 R2
Equivalent resistance; Req = .
Solution: (a) R1 + R2
For lower range,
Given resistance R1 = 10 W.
9.5 × 4.5
Tolerance of  R1 ; ∆R1 = ± 5% = ±(0.05 × 10) = ± 0.5. Req = Ω = 3.05 Ω.
9.5 + 4.5
Range of R1 → (10.5 − 9.5) Ω. For higher range,
Resistance R2 = 5 W. 10.5 × 5.5
Req = Ω = 3.60 Ω.
Tolerance of R2 ; ∆R2 = ± 10% = ±(0.1× 5) = ± 0.5. 10.5 + 5.5
Range of R2 → (5.5 − 4.5) Ω. Hence, the correct option is (a).

Chapter 04.indd 16 10/30/2015 2:45:15 PM


Chapter 5
Bridge Measurement,
Measurement of Induction
and Capacitance
Solution: (c)
One-mark Questions The above bridge is Maxwell inductance–capaci-
tance bridge. It is suitable for the measurement of
1. The bridge method commonly used for finding Low Q Inductor.
mutual inductance is [2012] Hence, the correct option is (c).
(a) Heaviside Campbell bridge
(b) Schering bridge 3. Kelvin double bridge is best suited for the meas-
(c) De Sauty bridge urement of [2002]
(d) Wien bridge (a) resistances of very low value
(b) low value capacitance
Solution: (a) (c) resistance of very high value
The bridge method commonly used for finding (d) high value capacitance
mutual inductance is Heaviside Campbell bridge.
Solution: (a)
Hence, the correct option is (a). Kelvin double bridge is best suited for the meas-
2. The bridge circuit shown in the fig below is used for urement of resistances of very low value.
the measurement of an unknown element Zx. The Hence, the correct option is (a).
bridge circuit is best suited when Zx is a [2011]

C1
R2 Two-marks Questions
+ R1
Vs ∼ D 1. A strain gauge forms one arm of the bridge shown in
− the figure below and has a nominal resistance with-
R4
Zx out any load as Rs = 300 W. Other bridge resistances
are R1 = R2 = R3 = 300 W. The maximum permissible
current through the strain gauge is 20 mA. During
(a) low resistance (b) high resistance certain measurement when the bridge is excited by
(c) low Q Inductor (d) lossy Capacitor. maximum permissible voltage and the strain gauge

Chapter 05.indd 17 11/2/2015 2:24:15 PM


6.18 | Electrical and Electronics Measurements

resistance is increased by 1% over the nominal value, R2 R3


(a)
= R = L C4 R2 R3
the output voltage V0 in mV is [2013] R4

(b) R2 R3
Rs R1 = L = R C4 R2 R3
R4
V0
+ − + R4 1
V1

(c)
= R = L
R2 R3 C4 R2 R3
R3 R2 R4 1
= L =
(d) R
R2 R3 C4 R2 R3
(a) 56.02 (b) 40.83 Solution: (a)
(c) 29.85 (d) 10.02 At balance Z1 Z4 = Z2 Z3.
Solution: (c)
 j 
20 mA ( R + jω L)  R4 −  = R 2 R3 .
IT  ω c4 
300
300 V0  −j 
V1
+ − +  ω c × R4 
− ( R + jω L)  4  = R2 R3 .
R − j 
300 300  4 ωc 
 4 
 − jR4 
∴ IT = 40 mA. ( R + jω L)   = R2 R3 .
 ω R4 C4 − j 
Vi = IT × RT = 40 mA × 300 Ω = 12 V.
 R4 
( R + jω L)   = R2 R3 .
 1 + jωC4 R4 
303 Ω 300 Ω RR4 + jω LR4 = R2 R3 + jωC4 R4 R3 .
V0
− + Compare real and imaginary terms.
12 V B A
RR4 = R2 R3 .
300 Ω 300 Ω
R2 R3
R= .
R4
V0 = VA − VB LR4 = C4 R4 R2 R3 .
300 300 L = C4 R2 R3 .
= 12 × − 12 ×
300 + 303 300 + 300 Hence, the correct option is (a).
= 0.02985 V = 29.85 mV.
3. A bridge circuit is shown in the figure below.
Hence, the correct option is (c). Which one of the sequences given below is most
2. The Maxwell’s bridge shown in the figure is balanced. suitable for balancing the bridge? [2008]
The parameters of the inductive coil are [2010]
R1
R3
R + jwL R3 Jx1

R4
R2
R4
R2
−j −jX 4
wc 4

∼ ∼

Chapter 05.indd 18 11/2/2015 2:24:17 PM


Chapter 5  Bridge Measurement, Measurement of Induction and Capacitance  |  6.19

(a) First adjust R4, and then adjust R1 L1 = R2 R3C4 − R4 (ω 2 R4 C4 L1 )C4 .


(b) First adjust R2, and then adjust R3 L1 = R2 R3C4 (W R4 C4 ) 2 L1 .
(c) First adjust R2, and then adjust R4
(d) First adjust R4, and then adjust R2 L1 (1 + ω R4 C4 ) 2 = R2 R3C4 .
R2 R3C4
Solution: (c) L1 = .
1 + (ω R4 C4 ) 2
X 1 = ω L.
R4
1 C4
X4 = .
ω C4
1
Q= (on Z4).
R1 ω R4 C4
R3
jwl 1
At balance condition Q of Z4,
Is balanced against Q of Z1,
R4 ω L1 R RC
R2 Q= (on Z1)  L1 = 2 3 4 .
−jX 4 R1 1
1+ 2
Q

Similarly substituting L1 into R′1s equation,

Z1 ( s ) = R1 + SL1 .  
R R C 
Z 2 = R2 . R1 = ω R4 C4  2 3 4  .
2

Z 3 = R3 .  1+ 1 
 Q 2 
1
Z 4 ( s ) = R4 + . R R (ω R4 C4 ) 2
SC4 R1 = 2 3 .
R4 1 + 1
Assuming that the given Hay’s bridge is in balance. Q2
Z1 ( s ) Z 4 ( s ) = Z 2 Z 3 . 1
RR Q2 RR 1
 1  R1 = 2 3⋅ = 2 3⋅ 2 .
( R1 + SL1 )  R4 +  = R2 R3 . R4 1 + 1 R4 Q + 1
 SC4  Q2
( R1 + SL1 )( SR4 C4 +11) = SR2 R3C4 . RR 1
R1 = 2 3⋅ .
S 2 R4 C4 L1 + S ( L1 + R4 R1C4 ) + R1 = SR2 R3C4 . R4 1 + Q 2
R1 − W 2 R4 C4 L1 + jω ( L1 + R4 R1C4 ) = jω R2 R3C4 . R3 R4 C4
∴ L1 = .
1
Balancing real and imaginary parts, 1+ 2
Q
∴ R1 − ω 2 R4 C4 L1 = 0. First adjust R2 for resistive balance, and then adjust
L1 + R4 R1C4 = R2 R3C4 . R4 for inductive balance (without disturbing resist-
R1 = ω 2 R4 C4 L1 . ance balance).
L1 = R2 R3C4 − R4 R1C4 . Hence, the correct option is (c).
4. R1 and R2 are the opposite arms of R3 and R4 of a
R1 depends on L1.
Wheatstone bridge. The source voltage is applied
L1 depends on R1. across R1 and R3 under balanced conditions, which
Break this dependency one of the following is true? [2006]

Chapter 05.indd 19 11/2/2015 2:24:19 PM


6.20 | Electrical and Electronics Measurements

R3 R4 RR 6. The items in List-I represent the various types of


R1 =
(a) R1 = 2 3
(b)
2 R4 measurements to be made with a reasonable accu-
RR racy using a suitable bridge. The items in List-II
R1 = 2 4 (d)
(c) R1 = R2 + R3 + R4 represent the various bridges available for this
R3 purpose. Select the correct choice of the item in
Solution: (b) List-II for the corresponding item in List-I from
The source voltage is applied across R1 and R3 the following. [2003]
under balanced conditions List-I
R2 R3 A. Resistance in the milli Ohm range
R1 = . B. Low values of capacitance
R4
C.  Comparison of resistances which are nearly
Hence, the correct option is (b). equal
5. A moving iron ammeter produces a full scale D. Inductance of a coil with a large time-constant
torque of 240 μNm with a deflection of 120° at a List-II
current of 10 A. The rate of change of self induct- 1. Wheatstone bridge 2.  Kelvin double bridge
ance (μH/radian) of the instrument at full scale is 3. Schering bridge 4.  Wien’s bridge
 [2004] 5.  Hay’s bridge 6.  Carey Foster bridge
(a) 2.0 μH/radian (b) 4.8 μH/radian Codes:
(c) 12.0 μH/radian (d) 114.6 μH/radian A B C D
Solution: (b) (a) 2 3 6 5
(b) 2 6 4 5
Td = 240 µNm θ = 120°. (c) 2 3 5 4
dL (d) 1 3 2 6
I = 10 A =?
dθ Solution: (a)
1 2 dL Resistance in the milli Ohm range is measured by
Td = I .
2 dθ Kelvin double bridge.
dL 2T Low values of capacitance is measured by S ­ chering
⇒ = 2d .
dθ I bridge.
dL 2 × 240 × 10−6 Comparison of resistances which are nearly equal
= . is done using Carey Foster bridge.
dθ 100
dL 4.8 µH Inductance of a coil with a large time-constant is
∴ = . measured using Hay’s bridge.
dθ rad
Hence, the correct option is (a).
Hence, the correct option is (b).

Chapter 05.indd 20 11/2/2015 2:24:20 PM


Chapter 6
Extension of Instrument
Ranges
2. A 100 μA ammeter has an internal resistance of
Two-marks Questions 100 W. For extending its range to measure 500 μA,
the shunt required is of resistance (in W) [2001]
1. A DC ammeter has a resistance of 0.1 Ω and its cur- (a) 20.0 (b) 22.22
rent range is 0 - 100 A. If the range is to be extended (c) 25.0 (d) 50.0
to 0 - 500 A, then meter requires the following shunt Solution: (c)
resistance [2005] Given,
(a) 0.010 Ω (b) 0.011 Ω
(c) 0.025 Ω (d) 1.0 Ω Initial full scale of ammeter = 100 μA, Im = 100
× 10−6 A = 10−4 A.
Solution: (c)
Given, for a DC ammeter; initial full scale current Internal resistance r = 100 W.
Im = 100 A. After range extension, final full scale of ammeter
Internal resistance = 0.1 Ω. = 500 μA.
Final full scale current = 500 A = I. ⇒  I = 500 × 10-6 A = 5 × 10−4 A.
To find: required shunt resistance; R = ? To find shunt resistance required = ? = R (say).

I r I m r = ( I − I m ) R.
−1 = .
Im R Imr r r 100
R= = = = = 25 Ω.
500 0.1 0.1 0.1 I − Im  I  5 −1 4
−1 = ⇒ R= = .   − 1
100 R 5 −1 4  Im 
⇒ R = 0.025 Ω.
Hence, the correct option is (c).
Hence, the correct option is (c).

Chapter 06.indd 21 10/30/2015 3:04:54 PM


Chapter 7
Potentiometer
(c) stray thermal emf’s
One-mark Questions (d) erroneous standardization
Solution: (c)
1. A potentiometer is basically [1997] The major disadvantage of a potentiometer is that
(a) deflection type instrument extraneous or stray emfs picked up from stray
(b) null type instrument fields or couplings between portions of the poten-
(c) deflection as well as null type instrument tiometer seriously affect the result. This is elimi-
(d) a digital instrument nated by taking a second reading.
Solution: (b) Hence, the correct option is (c).
A potentiometer makes use of a balance or null
condition, no current flows, hence no power is
consumed in the circuit containing unknown emf
when the instrument is balanced. Two-marks Questions
Hence, the correct option is (b).
1. A DC potentiometer is designed to measure up to
2. A transfer instrument employed in the standardiza- about 2 V with a slide wire of 800 mm. A standard
tion of a polar type AC potentiometer is . cell of emf 1.18 V obtains balance at 600 mm. A
[1997] test cell is seen to obtain balance at 680 mm. The
(a) an electrostatic instrument emf of the test cell is [1994]
(b) a thermal instrument (a) 1.00 V (b) 1.34 V
(c) a dynamo meter instrument (c) 1.50 V (d) 1.70 V
(d) a moving coil instrument
Solution: (b)
Solution: (c) Standard cell emf; E = 1.18 V.
For a polar type AC potentiometer, a precision
Initial balance position; L1 = 600 mm = 600 × 10-3 m.
type electrodynamometer ammeter is required for
standardization purposes as a transfer instrument.
E 1.18
Hence, the correct option is (c). Working current; I = = = 1.967 A.
L1 600 × 10−3
3. In DC potentiometer measurements, a second
New balance position; L2 = 680 mm = 680 × 10-3 m.
reading is often taken after reversing the polarities
of the DC supply and the unknown voltage, and Emf of the test cell; V = IL2
−3
the average of the two readings is taken. This is,   =1.967 × 680 ×10 =1.34 V.
with a view to eliminate the effects of [1992] Hence, the correct option is (b).
(a) ripples in the DC supply
(b) stray magnetic fields

Chapter 07.indd 22 10/30/2015 3:13:50 PM


Chapter 8
Measurement of Power
and Energy
Solution:
One-mark Questions
W1 = 100 W.
1. Power consumed by a balanced 3-phase, 3-wire load
is measured by the two wattmeter method. The first W2 = 250 W = cos φ2 .
wattmeter reads twice that of the second. Then the    W − W1  
load impedance angle in radians is [2014-S1] Pf = cos  tan −1  3  2  
π    W2 + W1  
(a) π (b)
2 8    250 − 100  
π π = cos  tan −1  3   
(c) (d)    250 + 100  
6 3
Solution: (c)    150  
= cos  tan −1  3    = 0.802.
   350  
  W − W2  
φ = tan −1  3  1 
  W1 + W2   3. For the circuit shown in the figure, the volt-
  2 W2 − W2   age and current expressions are v (t) = E1 sin(ωt)
= tan −1  3   + E3 sin (3ωt) and i (t) = I1 sin (ωt - ϕ1) + I3 sin
  2 W2 + W2   (3ωt - ϕ3) + I5 sin (5ω t).
  W    1  The average power measured by the wattmeter is
= tan −1  3  2   = tan −1  .
  3W2    3  [2012]
π
φ= . i(t)
6
Hence, the correct option is (c). L
+ Wattmeter O
2. While measuring power of a three-phase balanced
v(t) A
load by the two-wattmeter method, the readings
− D
are 100 W and 250 W. The power factor of the load
is . [2014-S2]

Chapter 08.indd 23 11/2/2015 2:32:16 PM


6.24 | Electrical and Electronics Measurements

1 Current coil
(a) E1 I1 cos φ1
2
Potential coil Z1
1
(b) [ E1 I1 cos φ1 + E1 I 3 cos φ3 + E1 I 5 ] ∼
2 Wattmeter
Z2
1
(c) [ E1 I1 cos φ1 + E3 I 3 cos φ3 ]
2
1 (a) zero always
(d) [ E1 I1 cos φ1 + E3 I1 cos φ1 ] (b) total power consumed by Z1 and Z2
2
(c) power consumed by Z1
Solution: (c) (d) power consumed by Z2
Solution: (d)
Pave = P1 + P2 + P5 . The wattmeter reads power consumed by Z2.
E1 I1 E1 I1 Hence, the correct option is (d).
P1 ⇒ V1 I1 cos φ1 = ⋅ cos φ1 = cos φ1 .
2 2 2 6. A sampling wattmeter (that computes power from
E3 I3 simultaneously sampled values of voltage and cur-
P2 ⇒ V2 I 2 cos φ2 = ⋅ cos φ3 .
rent) is used to measure the average power of a load.
2 2
The peak to peak voltage of the square wave is 10 V
P5 = 0 as 5th harmonic does not contribute power
 and the current is triangular wave of 5 A p-p as
as it is not present in V (t). shown in the figure. The period is 20 ms. The read-
ing in w will be [2006]
1 EI
∴ Pave = [ E1 I1 cos φ1 + E3 I 3 cos φ3 ] = 3 3 cos φ3 .
2 2
Hence, the correct option is (c).
4. Consider the following statements:
(i) The compensating coil of a low power factor
wattmeter compensates the effect of the im- (a) 0 W (b) 25 W
pedance of the current coil. (c) 50 W (d) 100 W
(ii) The compensating coil of a low power factor Solution: (a)
wattmeter compensates the effect of the im- The positive power and negative power in earth
pedance of the voltage coil circuit. [2011] cycle gets cancelled, so Net power = 0 W.
(a) (i) is true but (ii) is false
Hence, the correct option is (a).
(b) (i) is false but (ii) is true
(c) Both (i) and (ii) are true 7. The circuit in figure is used to measure the power
(d) Both (i) and (ii) are false consumed by the load. The current coil and the volt-
age coil of the wattmeter have 0.02 W and 1000 W
Solution: (b) resistance respectively. The measured power com-
(ii) True (i) False, pared to the load power will be [2004]
That is, the compensating coil is designed to com- 0.02 Ω
pensate the pressure coil current existing in the 20 A
current coil.
∴   Compensates the effect of the impedance of U.P.F
 200 V LOAD
the voltage coil (or) potential coil circuit. 1000 Ω

Hence, the correct option is (b).


5. A wattmeter is connected as shown in the figure (a) 0.4% Less (b) 0.2% Less
the wattmeter reads [2010] (c) 0.2% More (d) 0.4% More

Chapter 08.indd 24 11/2/2015 2:32:17 PM


Chapter 8  Measurement of Power and Energy  |  6.25

Solution: (c) Wattmeter W1 reading is VLLL cos(30 - ∅)


Measured power = True power + error. = 100 × 11.547 cos 90° = 0.
True power = VI cos ϕ Wattmeter W2 reading is VLLL cos(30 + ∅)
     = 200 × 20 × 1  ⇒ 4000. = 100 × 11.547 cos 30° = 1000.
Error is due to only load side connected coil, i.e., Hence, the correct option is (c).
current coil.
9. The minimum number of wattmeters(s) required to
I 2R = 20 2 × (0.02)  ⇒ 400 × 0.02  ⇒  8 W. measure 3-phase, 3-wire balanced or unbalanced
Measured power power is [2001]
(a) 1 (b) 2
Pm = 4000 + 8 ⇒ 4008 W.
(c) 3 (d) 4
4008 − 4000
% error = × 100 = 0.2% more. Solution: (b)
4000
The minimum number of wattmeters(s) required to
Hence, the correct option is (c). measure 3-phase, 3-wire balanced or unbalanced
8. The line to line input voltage to the 3 phase, 50 Hz, power is 2.
AC circuit shown in figure is 100 V rms Assuming Hence, the correct option is (b).
that the phase sequence is RYB the wattmeter 10. Electrodynamic type wattmeters have large errors
would read. [2002] while measuring power in AC circuits at low power
W1 factor conditions, since the voltage across and the
current though the [1999]
R (a) current coil are not in phase.
5∠60° Ω
(b) current coil are not in quadrature.
5∠60° Ω 5∠60° Ω (c) pressure coil are not in phase.
(d) pressure coil are not in quadrature.
Y
Solution: (c)
 Electrodynamic type wattmeters have large er-
B rors while measuring power in AC circuits at low
W2 power factor conditions, since the voltage across
and the current though the pressure coil are not in
(a) W1 = 886 W and W2 = 896 W phase.
(b) W1 = 500 W and W2 = 500 W Hence, the correct option is (c).
(c) W1 = 0 W and W2 = 1000 W
11. A dynamometer type wattmeter responds to the
(d) W1 = 250 W and W2 = 750 W
 [1997]
Solution: (c) (a) average value of active power
VL = 100 V. (b) average value of reactive power
(c) peak value of active power
100
V ph = . (d) peak value of reactive power
3
Solution: (a)
In star connected load, A dynamometer type wattmeter responds to the av-
erage value of active power.
V ph ∠0
I ph = I L = Hence, the correct option is (a).
Z ∠φ
12. A water boiler at home is switched on to the AC
100 mains supplying power at 230 V/50 Hz. The fre-
= 3 ∠ − 60° = 11.547 ∠ − 60°. quency of instantaneous power consumed by the
5 boiler is [1996]

Chapter 08.indd 25 11/2/2015 2:32:18 PM


6.26 | Electrical and Electronics Measurements

(a) 0 Hz (b) 50 Hz ∴   Quantity is proportional to reactive power.


(c) 100 Hz (d) 150 Hz Hence, the correct option is (c).
Solution: (c)
The instantaneous power in AC circuit is double
the supply frequency. Two-marks Questions
P = (1 - cos 2ω t) /2,
1. The figure shows a three phase delta connected
i.e., 50 × 2 = 100 Hz.
load supplied from a 400 V, 50 Hz, 3ϕ balanced
Hence, the correct option is (c). source. The pressure coil and current coil of a watt-
13. The moving coil in a dynamometer wattmeter is meter are connected to the load as shown. With the
connected [1996] coil polarities suitably selected to ensure a positive
(a) in series with the fixed coil deflection. The wattmeter reading will be [2009]
(b) across the supply
(c) in series with the load R a
(d) across the load Z2 = 100 Ω
3f Z1 = 100 Ω
Solution: (b) supply
400 V c.c
The moving coil in a dynamometer wattmeter is
50 Hz Y
connected across the supply. c
b
Hence, the correct option is (b). B p.c
14. VRN , VYN and VBN are the instantaneous line to neu-
tral voltages and iR, iY and iB are instantaneous (a) 0 (b) 1600 W
line currents in a balanced three-phase circuit, the (c) 800 W (d) 400 W
computation, VRN (iY - iB) - (VYN - VBN)iR will yield a Solution: (c)
quantity proportional to [1993]
(a) the active power
(b) the power factor VB (line Voltage)
I BR = I L =
(c) the reactive power Z2
(d) the complex power 400 ∠120°
= ⇒ 4 ∠120°.
Solution: (c) 100
VRN
IL
IR

q 120°
reff
120°
IB IY − IB VYN − VBN
q VYB

IY Voltage across pressure coil = VBY = 400 ∠-120°.


VBN VYN
Wattmeter reading  = I L × VYB cos(∠I L and VYB )
By using the vector diagram VRN (iY - iB) = VphIL
= 4 × 400 cos(240°)
cos(90 + θ) and (VYN - VBN)iR = VLIph cos(90 - θ).
 −1 
Therefore, VRN (iY - iB) - (VYN - VBN)iR = 1600   = 800 W.
 2 
= V ph I L cos(90 + θ ) − VL I ph cos(90 − θ )
Hence, the correct option is (c).
= 3V ph I ph cos(90 + θ ) − 3 V ph I ph cos(90 − θ ) 2. A 3 V DC supply with an internal resistance of
= 3V ph I ph [cos(90 + θ ) − cos(90 − θ )] 2 Ω supplies a passive non-linear resistance char-
acterized by the relation VNL = I 2NL. The power dis-
= 2 3V ph I ph sin θ . sipated in the non-linear resistance is [2007]

Chapter 08.indd 26 11/2/2015 2:32:19 PM


Chapter 8  Measurement of Power and Energy  |  6.27

(a) 1.0 W (b) 1.5 W Now heater connected to peak to peak of 400 V
(c) 2.5 W (d) 3.0 W square wave.
Solution: (a)
From the circuit diagram V 2  2002 
∴ P= =  = 1.739 kW.
2Ω R  23 

Hence, the correct option is (b).


3V I VNL = I 2NL
4. Two wattmeters, which are connected to measure
the total power on a three-phase system supply-
ing a balanced load reads 10.5 kW and -2.5 kW
respectively. The total power and power factor
respectively are [2005]
3 = 2 I + I NL
2
.
(a) 13.0 kW, 0.334
To satisfy the above equation, the current must be (b) 13.0 kW, 0.684
one ampere. (c) 8.0 kW. 0.52
(d) 8.0 kW, 0.334
So I = 1 A.
Solution: (d)
∴ VNL = 1 V.

Power dissipated in that resistance is I × VNL ⇒ 1  3 (W1 − W2 ) 


× 1 ⇒ 1 W. φ = tan −1   .
 W +W
Hence, the correct option is (a).  1 2 
3. An energy meter connected to an immersion heater  3 (10.5 + 2.5) 
φ = tan −1   = 70.44.
(resistive) operating on an AC 230 V, 50 Hz, AC sin-  10.5 − 2.5 
gle phase source reads 2.3 units (kWh) in 1 hour.
The heater is removed from the supply and now
connected to a 400 V peak to peak square wave Power factor cosϕ = 0.3347.
source of 150 Hz. The power in kW dissipated by Total power = W1 + W2 = 10.5 - 2.5 = 8.0 kW.
the heater will be [2006] Hence, the correct option is (d).
(a) 3.478 (b) 1.739
(c) 1.540 (d) 0.870 5. A single-phase load is connected between R and Y
terminals of a 415 V, symmetrical, 3-phase, 4-wire
Solution: (b)
system with phase sequence RYB. A wattmeter is
connected in the system as shown in figure. The
200
power factor of the load is 0.8 lagging. The watt-
meter will read [2004]

W
R
200
100 Ω
Z
0.8 pf lag
2.3 units for one hour at 230 V.
Y
Energy consumed = kWh.
B
V2 N
2.3 = × t.
R
2302 -795 W
(a) (b) -597 W
2.3 = × 1 hr ⇒ R = 23 Ω.
R +597 W
(c) (d) +795 W

Chapter 08.indd 27 11/2/2015 2:32:20 PM


6.28 | Electrical and Electronics Measurements

Solution: (b) sequence is RYB. What will be the reading of this


wattmeter if its pressure coil alone is reconnected
VRY between the B and Y phases, all other connections
VRN remaining as before? [2003]
(a) 400.0 (b) 519.6
IR
30° (c) 300.0 (d) 692.8
−V YN q Solution: (b)
W
R
120°

VBN VYN
Y
W = I RVBN cos (∠I R and VBN ).
B
Given VRY = 415 V.
  cos φ = 0.8 Pf lag ⇒ φ = 36.86. W
R
And VRY = 415 ∠30°.
∴ θ = 30° − 36.96 ⇒ − 6.86°, Y
415 ∠30°
i.e.,   I R = ⇒ 4.15 ∠ − 6.86.
100 ∠36.96
B
415 W = 400 W.
∴ W = 4.15 × cos(120 + 6.86)
3 W = V ph I ph cos φ .
⇒ − 596.46 W. V ph I ph = 400/0.8.
Hence, the correct option is (b).
This type of connection gives reactive power.
6. A DC A-h meter is rated for 15 A, 250 V. The meter
400
constant is 14.4 A-sec/rev. The meter constant at W = 3V p I p sin φ = 3 × × 0.6 = 519.6.
rated voltage may be expressed as [2004] 0.8
(a) 3750 rev/kWh (b) 3600 rev/kWh Hence, the correct option is (b).
(c) 1000 rev/kWh (d) 960 rev/kWh 8. The voltage-flux adjustment of a certain 1-phase
Solution: (c) 220 V induction watt-hour meter is altered so that
Meter constant is 14.4 A-sec/rev the phase angle between the applied voltage and
14.4 A − h  A − h  250 × 14.4 the flux due to it 85° (instead of this 90°). The
  = ⋅ ⋅V  = = 1. errors introduced in the reading of this meter when
3600 rev  rev  3600
the current is 5 A at power factors of unity and 0.5
Wh = 1 rev. lagging are respectively [2003]
kWh (a) 3.8 mW, 77.4 mW
= 1000.
rev (b) -3.8 mW, -77.4 mW
∴   Meter constant  = 1000 rev . (c) -4.2 W, -85.1 W
kWh (d) 4.2 W, 85.1 W
Hence, the correct option is (c). Solution: (c)
7. A wattmeter reads 400 W when its current coil is V = 220 V I = 5 A.
connected in the R phase and its pressure coil is
connected between this phase and the neutral of a Td = VL I L sin(∆ − φ ).
symmetrical 3-phase system supplying a balanced At unity power factor ϕ = 0 and given Δ = 85° but
star connected 0.8 pf inductive load. The phase true value of Δ = 90°.

Chapter 08.indd 28 11/2/2015 2:32:22 PM


Chapter 8  Measurement of Power and Energy  |  6.29

Td Power (measure power) 9. The two wattmeter method is used to measure


∴ Power = 220 × 5 × sin 85°  ⇒  1095.81 W.
 active power on a three phase, three wire system.
If the phase voltage is unbalanced, then the power
True value of power = VLIL sin90° reading is [2000]
= 220 × 5  ⇒  1100 W.
         (a) affected by both negative sequence and zero
Error = 1095.81 - 1100  ⇒  -4.19 W. sequence voltages
(b) affected by negative sequence voltage but not
At power factor 0.5 → ϕ = 60°. by zero sequence voltages
Power = VLIL sin(85 - 60°) (c) affected by zero sequence voltages but not by
   = 220 × 5 × sin 25  ⇒ 464.80. negative sequence voltages
(d)  not affected by negative or zero sequence
True value = VLIL sin(90 - 60) = 550 W. ­voltages
Error = 464.80 - 550  ⇒  - 85.2 W. Solution: (d)
Hence, the correct option is (c). Hence, the correct option is (d).

Chapter 08.indd 29 11/2/2015 2:32:22 PM


6.30 | Electrical and Electronics Measurements

From Equation. (1) ⇒ w = 3Vphase I phase sin φ


Five-marks Questions
Wattmeter reading is proportional to reactive power.
1. A 3∅ load operates with balanced voltages applied 2. A symmetrical 400 V, 3ϕ supply is connected to
to its terminals and draws balanced currents. The the network shown in figure. The phase sequence
potential coil of a moving coil wattmeter is con- is RYB. Find the reading on the wattmeter [1998]
nected from R to Y terminals of the load. The cur-
R
rent coil of the meter is connected in series with R
R1
phase B. By appropriate derivation, show that the X1
quantity indicated by this wattmeter is proportional
to the reactive power drawn by the load [2000] YS
X2
Solution: B
Load is balanced load and draws balanced current.
Let us consider load is star connected one. Y
IR
R R R1 = 30 W,  x1 = 50 W and x2 = 40 W
Z Solution:
R
Z R
W Z R1
X1
IB y
B
B YS
X2
y B
IY

The reading of the wattmeter W = IBVRY cos(angle Y


between IB and VRY) and VRY = VR - VY .
VRY ∠0°
VRY → Line voltage, VR and VY are phase voltages.
IR
w = I BVRY cos(60° + 30° − φ )
11.56
= I BVRY cos(90° − φ ) = I BVRY sin φ . (1) VBR ∠−60°

90°
VRY
IY 113.13
VR
IB
IR
-VY f VYB ∠−120°
30° VBR ∠−240°
60°
f f VL = 400 V, R1 = 30 W, X1 = 50 W and X2 = 40 W and
VY
sequence is RYB.
IB
VB
IY So line voltages are VRY ∠0°, VYB ∠- 120°, VBR ∠- 240°.
Wattmeter reading is w = IRVYB cos(angle between IR
Given load is balanced
and VYB ).
V= V= VBR = VLine .
  RY YB
IR current is not obtained directly, because the
V=R V=
Y VB = VPhase . given diagram is not exactly in the form of either
VL star or delta.
And, VPhase = .
3 So we have to find out that value based on IY and IB .

Chapter 08.indd 30 11/2/2015 2:32:23 PM


Chapter 8  Measurement of Power and Energy  |  6.31

I R + IY + I B = 0. Note: for finding current IB, don’t take line voltage


VBR. Because we have to follow the sequence
I R = ( IY + I B ) → (1).
VRY 100 ∠0° I R = −(8∠90° + 8∠ − 113.13) = 3.20∠ − 11.56.
IY = = ⇒ 80 A.
impedance − j 50 Wattmeter reading
VRB 400∠ − 60° w = I RVYB cos(∠I R and VYB )
IB = = = 8∠ − 113.13.
30 + 40 j 30 + 40 j = 3.20 × 400 cos(120 − 11.56) = 404.87 W.

Chapter 08.indd 31 11/2/2015 2:32:23 PM


Chapter 9
Measurement of Phase
and Frequency
Solution: (a)
One-mark Questions For an induction type ammeter,
Rise in temperature; ∆θ ∝ I 2 R.
1. A two-phase load draws the following phase cur-
I is current; R is resistance.
rents: i1 (t) = Im sin(ωt - ϕ1), i2 (t) = Im cos(ωt -  ϕ2).
These currents are balanced if ϕ1 is equal to [2012] Δθ = KI 2R (1)
(a) -ϕ2 (b) ϕ2 ∴ Emf; E = aΔθ + bΔθ 2.
π  π   a∆θ (2)
(c)  − φ2  (d)  + φ2    b << a,

2  2 
Solution: (b) Where a and b are constants.
Given,  i1 (t ) = I m sin(ωt − φ1 ). Deflecting torque; τ = K1E
 i2 (t ) = I m cos(ωt − φ2 ). = K1a∆θ = K1a ( KI 2 R) = K 2 I 2 .
  τ ∝ I 2.
f1
90 − f 1 Hence, the correct option is (a).
f2
i1
Two-marks Questions
i2 1. The AC bridge shown in the figure is used to meas-
For i1 and i2 to be balanced, angle between i1 and i2 ure the impedance Z. If the bridge is balanced for
should be 90°. oscillator frequency f = 2 kHz, then the impedance
Z will be [2008]
∴ φ2 + 90° − φ1 = 90°.
B
φ1 = φ2 . 0.398 μF
500 Ω
Hence, the correct option is (b). 300 Ω

2. For a given frequency, the deflecting torque of ∼ Oscillator A D C


an induction ammeter is directly proportional to
15.91 mH
 [1996] Z
(a) Current2 (b) Current3 300 Ω
(c) Current (d) Current D

Chapter 09.indd 32 11/13/2015 11:31:41 AM


Chapter 9  Measurement of Phase and Frequency  |  6.33

(a) (260 + j 0) W (b) (0 + j 200) W Z Z


For balanced bridge  AB = BC  where ZAB, ZBC,
(c) (260 - j 200) W (d) (260 + j 200) W Z AD Z CD
Solution: (a) ZCD and ZAD are respectively impedances of the
Given, frequency; f = 2 kHz. branches AB, BC, CD and AD.
 1 
B  300 + 
500 jωC 
0.398 μF =
Oscilloscope ∼ 500 Ω (300 + jω L) Z
300 Ω
1  1 
A D C
⇒ Z=  300 +  (300 + 2π f L)
500  2π f × C 
15.91 mH
1 9 × 104 × C × L
Z = × = 259.94 Ω.
300 Ω 500 jωC
D Hence, the correct option is (a).

Chapter 09.indd 33 11/13/2015 11:31:42 AM


Chapter 10
Potentiometric Recorders
and Q-Meter
1 1
One-mark Questions ωC1 R
=
ω (Cx + C2 ) R
= 120.

1. The Q-meter works on the principle of [2005] C1 = C2 + C x .


(a) a mutual inductance (b) Self inductance Cx = C1 − C2 = 300 pF − 200 pF.
(c) series resonance (d) Parallel resonance Cx = 100 pF
F.
Solution: (c) Hence, the correct option is (a).
Q meter works on the principle of series resonance.
Hence, the correct option is (c).
Two-marks Questions
2. A reading of 120 is obtained when a standard
inductor was connected in the circuit of a Q-meter 1. The voltage phasor of a circuit is 10 ∠15° V and
and the variable capacitor is adjusted to a value of the current phasor is 2 ∠45° A . The active and the
300 pF. A lossless capacitor of unknown value C, is reactive powers in the circuit are [1999]
then connected and the same reading was obtained (a) 10 W and 17.32 V Ar.
when the variable capacitor is readjusted to a value (b) 5 W and 8.66 V Ar.
of 200 pF. The value of Cx in pF is [2003] (c) 20 W and 60 V Ar.
(a) 100 (b) 200 (d) 20 2 W and 10 2 V Ar.
(c) 300 (d) 500
Solution: (a)
Solution: (a) Given, Voltage phasor; V = 10∠15°.
Given, reading of Q meter; Q = 120.
Current phasor; I = 2∠-45°.
Initial capacitance; C1 = 300 pF.
Final capacitance of variable resistor; C2 = 200 pF. V

Connected capacitor to final capacitance; Cx = ?.


1 15°
Q= , ref
ωCR 45°
where, w = Frequency,
C = Capacitance,
R = Resistance.
\ Same Q was obtained for both connections. I

Chapter 10.indd 34 11/2/2015 2:40:25 PM


Chapter 10  Potentiometric Recorders and Q-Meter  |  6.35

Angle between V and I, (a) 1 V (b) 150 V


ϕ = 15° + 45°. (c) 120 V (d) 147 V
 ϕ = 60°. Solution: (c)
\  Active power = VI cos ϕ Total resistance; R = 2 MW + 6 MW + 1.2 MW
+ (600 kW + 120 kW + 60 kW + 20 W)
= 10 × 2 cos 60°
= 9.2 MΩ + 800 kΩ
1
= 10 × 2 × = 10 W. = 9.2 MΩ + (0.8 × 103 ) kΩ
2
= 9.2 MΩ + 0.8 MΩ = 10 MΩ.
Reactive power = VI sin ϕ
  = 10 × 2 sin 60° = 17.32 VAR. For switch at B, Voltage at M; Vm = 12 V.
Hence, the correct option is (a). R
Full scale voltage;VFS = Vm × m
2. The figure shows input attenuator of a multimeter. R
The meter reads full-scale with 12 V at M with the 2 MΩ
range switch at position B obtain full-scale deflec- = 12 × = 2.4 V.
10 MΩ
tion with the range switch position at D? [1998]
DC probe For switch at D,
M A
2 kΩ VFS = 2.4 V.
6 MΩ Rm
VFS = Vm × .
R
B
(120 kΩ + 60 kΩ + 20 kΩ)
12 MΩ
2.4 = Vm × .
10 MΩ
C Range 0.2 MΩ
2.4 = Vm .
10 MΩ
600 kΩ
Vm = 120 V.
D
Hence, the correct option is (c).
120 kΩ

60 kΩ

20 kΩ

Chapter 10.indd 35 11/2/2015 2:40:26 PM


Chapter 11
Oscilloscope
\ Frequency of Mux control signal is
One-mark Questions 1 1
f
= = f sweep .
2T 2
1. In an oscilloscope screen, linear sweep is applied
at the [2014-S1] \  Half the frequency on sweep signal.
(a) vertical axis Hence, the correct option is (d).
(b) horizontal axis 3. The two inputs of a CRO are fed with two station-
(c) origin ary periodic 3 signals. In the X-Y mode, the screen
(d) both horizontal and vertical axis shows a figure which changes from ellipse to circle
Solution: (b) and back to ellipse with its major axis changing
Horizontal axis. orientation slowly and repeatedly. The following
Hence, the correct option is (b). inference can be made from this. [2009]
(a) the signals are not sinusoidal
2. A dual trace oscilloscope is set to operate in the
(b) the amplitudes of the signals are very close but
­alternate. The control input of the multiplexer used in
not equal
the y-circuit is fed with a signal having a frequency
(c) the signals are sinusoidal with their frequen-
equal to [2011]
cies very close but not equal
(a) the highest frequency that the multiplexer can
(d) there is a constant but small phase difference
operate properly
between the signals
(b) twice the frequency of the time base (sweep)
oscillator Solution: (d)
(c) the frequency of the time base (sweep) oscillator
0° < f < 90° f = 90°
(d) half the frequency of the time base (sweep) os-
cillator
Solution: (d)
Time base-sweep   
Scope Scope Scope
CH-1 CH-2 CH-1 90° < f < 180°

t
T T
Max
Control 1 1
Because of phase difference only figures changes
(Selection)
from ellipse to circle and back to ellipse.
2T (2 input 1 output Mux) Hence, the correct option is (d).

Chapter 11.indd 36 11/2/2015 2:56:08 PM


Chapter 11  Oscilloscope  |  6.37

4. The probes of a non-isolated, two-channel oscillo- Time 1/1 kHz


→ =
scope are clipped to points A, B, and C in the circuit div 4 div
of the adjacent figure. Vin is a square wave of a suita- 1 ms ms
ble low frequency. The display on ch1 and ch2 are as = = 0.25 .
shown on the right. Then the ‘signal’ and ‘ground’ 4 div div
probes S1, G1 and S2, G2 of ch1 and ch2 respectively •  Channel-2 display:
are connected to points. [2007]
A B
ch 1 3 div
R GND12
L 8 div
V in ch 2
C
(a) A, B, C, A (b) A, B, C, B volt
→ V pp = 3 div × 2.5 = 7.5 v.
(c) C, B, A, B (d) B, A, B, C div
Solution: (b) ms
→ T = 8 div × 0.25 = 2 ms.
Here B is a common point. div
S1 → A S 2 → C. Hence, the correct option is (c).
G1 → B G2 → B. 6. Two in phase 50 Hz sinusoidal wave form of unit
Hence, the correct option is (b). amplitude are fed into channel-1 and channel-2
respectively of and oscilloscope. Assuming that
5. The time/div and voltage/div axes of an oscillo-
the voltage scale, time scale and other settings are
scope have been erased. A student connects a 1 kHz,
exactly the same for both the channels, what would
5 V p-p square wave calibration pulse to channel-1
be observed if the oscilloscope is operated in x-y
of the scope and observes the screen to be as shown
mode? [2002]
in the upper trace of the figure. An unknown sig-
(a) A circle of unit radius
nal is connected to channel-2 (lower trace) of the
(b) An ellipse
scope. If the time/div and V/div on both channels
(c) A parabola
are the same, the amplitude (p-p) and period of the
(d) A straight line inclined at 45° with respect to
unknown signal are respectively [2006]
the x-axis
Solution: (d)

Vy

(a) 5 V, 1 ms (b) 5 V, 2 ms Vx
(c) 7.5 V, 2 ms (d) 10 V, 1 ms
Solution: (c)
•  Channel-1 display:
→  Known signal (Vpp = 5 V,  f = 1 kHz).
Hence, the correct option is (d).
2 div 7. A certain oscilloscope with 4 cm by 4 cm screen has
its own sweep output fed to its input. If the x and y
4 div sensitivities are same, the oscilloscope will display a
[1995]
V 5v V (a) Triangular wave (b) Diagonal line
→ = = 2.5 .
div 2 div div (c) Sine wave (d) Circle

Chapter 11.indd 37 11/2/2015 2:56:09 PM


6.38 | Electrical and Electronics Measurements

Solution: (b) Y
Vy
X

Vx

(a) q(w2t) =  A sin w2t, w2 = 2w1


ω
(b) q (ω2 t ) = A sin ω2 t , ω2 = 1
2
Hence, the correct option is (b).
(c) q(w2t) =  A cos w2t, w2 = 2w1
8. A Lissajous pattern, as shown in figure below, is ω
observed on the screen of a CRO when voltages (d) q (ω2 t ) = A cos ω2 t , ω2 = 1
2
of frequencies fx and fy are applied to the x and y Solution: (d)
plates respectively. fx : fy is then equal to [1994]
Horizontal line

Vertical line

(a) 3 : 2 (b) 1 : 2 fy ny


= .
(c) 2 : 3 (d) 2 : 1 fx nx
Solution: (c) 1
f y =   fx .
2
ny = 2
In order to get the clock wise rotation, the signal
applied to the y-plate should be a cosine wave form.
nx = 3
Hence, the correct option is (d).
2. The simultaneous application of signals x(t) and y(t)
to the horizontal and vertical plates, respectively,
of an oscilloscope, produces a vertical figure-of-8
fy nx ­display. If P and Q are constants, and x(t) = P sin
= , (4t + 30), then y(t) is equal to [2005]
fx ny
(a) Q sin(4t - 30) (b) Q sin(2t + 15)
(c) Q sin(8t + 60) (d) Q sin(4t + 30)
Where, nx = number of horizontal cuts.
Solution: (b)
ny = number of vertical cuts.
Horizontal line
\  fx : fy = 2 : 3.

Hence, the correct option is (c).

Two-marks Questions Vertical line


x(t ) − p sin(4t + 30) y (t ) = ?
1. Two sinusoidal signal p(w1t) = A sin wt and q(w2t) f y ny 1
are applied to X and Y inputs of a dual channel = = .
CRO, the Lissajous figure displayed on the screen f x nx 2
is shown below. The signal q(w2t) will be repre- 1
fy = ( f x ).
sented as [2008] 2

Chapter 11.indd 38 11/2/2015 2:56:10 PM


Chapter 11  Oscilloscope  |  6.39

The frequency of ‘Q sin(2t + 15)’ is half of x(t). Vy = Vym sin (wt + Φ) are given to its X and Y plates
Hence, the correct option is (b). respectively and Φ is changed. Choose the correct
value of Φ from List-I to match with the corre-
3. A CRO probe has an impedance of 500 kW in par-
sponding figure of List-II. [2003]
allel with a capacitance of 10 pF. The probe is used
to measure the voltage between P and Q as shown List-I
in figure. The measured voltage will be [2004] π
(a) Φ = 0 (b) Φ=
2
100 kΩ π π
P (c) π < Φ < 3 (d) Φ=3
2 2
TO CRO List-II
10 V rms 100 kΩ
∼ Through 1  4 
100 kHz
probe

(a) 3.53 V (b) 4.37 V 2  5 


(c) 4.54 V (d) 5.00 V
Solution: (a)

100 kΩ
P 3  6 

10 V rms 100 kΩ
∼ 159 kΩ 500 kΩ
100 kHz

Q  Codes:
    A     B   C      D
C = 10 pF. (a) 1    3     6     5
(b) 2    6     4     5
1 1 (c) 2    3     5     4
XC = =
ω c 2π × 100 × 103 × 10 × 10−12 (d) 1    5     6     4
= 159 kΩ. Solution: (d)
100 kΩ X C Z = 159 kΩ 500 kΩ 100 kΩ Hence, the correct option is (d).
= 54.676 kΩ.

100 kΩ
P
Three-marks Questions
1. In an oscilloscope the input to the horizontal plates
10 V ∼ 54.676 kΩ is a 100 Hz voltage signal. The Lissajous patterns
(A), (B) and (C) will be generated when different
frequency voltage signals are applied to vertical
Q plates. Match each Lissajous pattern to the corre-
sponding frequency by [1996]
54.676
∴ V pq = 10 × = 3.53 V. (a) (b)
54.67 + 100
Hence, the correct option is (a).
4. List-I represents the figures obtained on a CRO
screen when the voltage signals Vx = Vxm sin wt and

Chapter 11.indd 39 11/2/2015 2:56:11 PM


6.40 | Electrical and Electronics Measurements

(c) n 
 × fx =   × 100 ⇒ 150 Hz
fy =  x 3
(b) n
 y  2
n 
(c) fy =  x  × fx =  3  × 100 ⇒ 300 Hz
n 
 y 1
(P) fy = 50 (S) fy = 150 Where, nx = number of horizontal peaks,
(Q) fy = 66.66 (T) fy = 200 ny = Number of vertical peaks.
(R) fy = 125 (U) fy = 300
Hence, the correct option is (a-P, b-S, c-U).
Solution: (a-P, b-S, c-U)
Given fx = 100 Hz.
fy n y
(a) =
fx nx
1 100
f y =   × fx = ⇒ 50 Hz
2 2

Chapter 11.indd 40 11/2/2015 2:56:12 PM


Chapter 12
Electronic Multimeters
and Digital Voltmeters
\  It’s bit size is 8
One-mark Questions Hence, the correct option is (b).
1. Two 8-bit ADCs, one of single slope integrating
type and other of successive approximation type.
Take TA and TB times to connect 5 V analog input Two-marks Questions
signal to equivalent digital output. If the input
analog signal is reduced to 2.5 V the approximate 1
time taken by the two ADCs will respectively be 1. A 4 digit DMM has the error specification as
2
[2008] 0.2% of reading + 10 counts. If a DC voltage of
TA
(a) TA, TB (b) , TB 100 V is read on its 200 V full scale. The maxi-
2
mum error that can be expressed in the reading
T TA TB is __________ [2011]
(c) TA , B (d) ,
2 2 2 (a) ±0.1% (b) ±0.2%
Solution: (a) (c) ±0.3% (d) ±0.4%
The time taken by the both ADCs will remain Solution: (c)
same, since it is single slope, i.e., TA, TB.
0.2
Hence, the correct option is (a). Error = × 100 V + 10 × 000.01 V
100
2. A digital-to-analog converter with a full-scale out-
= 0.2 V + 0.1 V = 0.3 V.
put voltage of 3.5 V has resolution close to 14 mV.  
It’s bit size is ________ [2005] 0.3 V
(a) 4 (b) 8 % Error = ± × 100 = ± 0.3%.
100 V
(c) 16 (d) 32
Hence, the correct option is (c).
Solution: (b)
full scale 2. The simplified block diagram of a 10-bit A/D con-
Resolution = analog value of LSB = .
2n verter of dual slope integrator type is shown in
3.5 v figure. The 10-bit counter at the output is clocked
14 mv = . by a 1 mHz clock. Assuming negligible timing
2n
overhead for the control logic, the maximum fre-
2 n = 250. quency of the analog signal that can be converted
28 ≈ 256. using this A/D converter is approximately. [2003]

Chapter 12.indd 41 11/13/2015 12:24:18 PM


6.42 | Electrical and Electronics Measurements

Intregrator, 
The maximum frequency can be obtained at
Comparator and T1 - T2 = 0.
Control logic
Therefore, T1 = 2NTc where, Tc is clock period.
1
f max =
10-5 V
Digital
T1
Counter
click output 1 f
= N
= Nc
(2 Tc ) 2
(a) 2 kHz (b) 1 kHz 1 mHz
(c) 500 Hz (d) 250 Hz = ≈ 1 kHz.
210
Solution: (b)
The dual slope integrated type voltmeter output Hence, the correct option is (b).
wave form.
T1 T2

Chapter 12.indd 42 11/13/2015 12:24:19 PM


Chapter 13
Instrument Transformer
∴   Ratio error
One-mark Questions Nominal ratio − Transformation ratio
= × 1000.
Transformation ratio
1. Instrument transformers are known to introduce
magnitude and phase errors in measurements. Kn − R
− 0.5 = × 100.
These are primarily due to [2000] R
(a) improper connections on the primary side − 0.5 R = 100 K n − 100 R.
(b) measurement errors inherent in the meter con- [ K n = 200]
nected to the transfer secondary
R = 201 = 200 + 1.
(c) open and short circuit parameters of the instru- ↓
ment transformer It is reduced by one turn.
(d) none
R = 200 + I 0 cos α .
Solution: (c)
Hence, the correct option is (c). 200 = 199 + 1.
Kn − R
% Error = × 100 = 0 %.
Two-marks Questions R
Hence, the correct option is (a).
1. A 200/1 current transformer (CT) is wound with 2. A 50 Hz, bar primary CT has a secondary with 500
200 turns on the secondary on a toroidal core. turns. The secondary supplies 5 A current into a
When it carries a current of 160 A on the primary, purely resistance burden of 1 W. The magnetizing
the ratio and phase errors of the CT are found to be amphere turns is 200. The phase angle between the
-50% and 30 minutes respectively. If the number of primary and secondary current is. [2004]
secondary turns is reduced by 1 the new ratio error (a) 4.6° (b) 85.4°
(%) and phase error (min) will be respectively. (c) 94.6° (d) 175.4°
 [2006]
Solution: (a)
(a) 0.0, 30 (b) - 0.5, 35
(c) -1.0, 30 (d) -1.0, 25 N1 = 1  N2 = 500  Is = 5 A  Load R2 = 1 W.
Solution: (a) Magnetizing amphere turns = 200 AT.
I 0 cos(α + δ ) = Magnetizing amphere turns 200
R = n+ , I0 = = 200 A.
IS Primary turns 1
where, R = Transformation ratio. Transformation ratio,
    n = Turns ratio. N 2 500
R = 200 + I 0 cos(α ) [assume δ = 0]. =n = = 500.
N 1 1

Chapter 13.indd 43 11/2/2015 3:01:37 PM


6.44 | Electrical and Electronics Measurements

The phase angle between primary and secondary Solution: (b)


current is 500 A
Primary
180  I 0 cos (α + δ )  180  200 × cos(0) 
θ=  =  . 5A
π  nI S  π  500 × 5  v
Io
≈ 4.6° ∵ (α + δ )  is very small. I21
Hence, the correct option is (a).
Io
3. The core flux in the CT of above problem under the
given operating condition is [2004] Iu
(a) 0 (b) 45.0 μWb
(c) 22.5 mWb (d) 100.0 mWb. Primary winding current
   Transformation ratio  = .
Solution: (b) Secondary winding current
Secondary induced emf, 500
Nominal ratio = ⇒ 100.
ES = 4.44 f φm N 2 . 5
ES IS × R Given magnetization required = 250 AT
φm = = Primary turns = 1.
4.44 × f × N 2 4.44 × 50 × 500
5 ×1 250 AT
= = 45 µWb. ⇒ Io = ⇒ 250 A.
4.44 × 50 × 500 T
Hence, the correct option is (b). I 500
K= 1 = ⇒ 100.
4. A 500 A/5 A, 50 Hz current transformer has a bar I2 5
primary. The secondary burden is a pure resistance I 2′ = KI 2 ⇒ 100(5) ⇒ 500 A.
of 1 W and it draws a current of 5 A. If the mag-
∴ I p = 5002 + 2502 = 559.069.
netic core requires 250 AT for magnetization, the
percentage ratio error is [2003] Ip 559.069
∴ Ratio = = = 111.8.
(a) 10.56 (b) -10.56 Is 5
(c) 11.80 (d) -11.80
Ratio Error
Nominal ratio − Transformation ratio
= × 1000
Transformation ratio
100 − 111.8
= × 100 ⇒ − 10.5 %.
111.8
Hence, the correct option is (b).

Chapter 13.indd 44 11/2/2015 3:01:38 PM


Unit 7
Digital Electronics and
Microprocessors
Chapter 1:  Number Systems and Code Conversions 7.3
Chapter 2:  Boolean Algebra 7.4
Chapter 3:  Logic Gates 7.5
Chapter 4:  Combinational Digital Circuits 7.9
Chapter 5:  Sequential Digital Circuits 7.14
Chapter 6:  Semiconductor Memories 7.20
Chapter 7:  Logic Gate Families 7.21
Chapter 8:  A/D and D/A Converters 7.23
Chapter 9:  Microprocessors 7.26

Chapter 01.indd 1 11/9/2015 6:13:58 PM


Chapter 01.indd 2
Exam Analysis
Exam Year 92 93 94 95 96 97 98 99 00 01 02 03 04 05 06 07 08 09 10 11 12 13 14
1 Mark Questions 1 2 2 2 1 1 2 – 1 3 2 1 2 1 – – 1 2 – 1 3 1 2
2 Marks Questions – – 1 – – 1 1 4 3 1 1 4 2 5 4 5 2 1 4 2 1 1 5
3 Marks Questions – – – – 1 – – – – – – – – – – – – – – – – – –
5 Marks Questions – – – – – 1 2 – – 1 1 – 1 – – – – – – – – – –
Total Marks 1 2 3 2 2 3 5 4 4 5 4 5 5 6 4 5 3 3 4 3 4 2 7
Number Systems and Code Conversions – – – – – – – – – – – – – – – 1 – – – – – – 1
Boolean Algebra – – – – – – – – – – – 1 1 – – – – – – – 1 – –
Logic Gates – – – – 1 – – 2 – 1 1 – 2 1 – 1 – 1 2 – – 1 2
Combinational Digital Circuits – – – – – 1 2 1 1 1 – 1 – – 1 – 1 – – 1 1 – –
Sequential Digital Circuits – – – 1 – – 1 – 2 1 2 1 1 2 – – – – – 1 2 1 2
Semiconductor Memories – – – – – – – – – – – – – – – – – 1 – – – – –
Logic Gate Families – – – – – – 2 – – – – – – 1 1 – – – 1 – – – –
A/D and D/A Converters – 1 1 – – – – 1 – 1 – 1 1 1 1 – – – – – – – –
Microprocessors 1 1 2 1 1 2 – – 1 1 2 1 – 1 1 3 2 1 1 1 – – 2

11/9/2015 6:13:58 PM
Chapter 1
Number Systems
and Code Conversions

One-mark Question Two-marks Question


1. Which of the following is an invalid state in an 1. The octal equivalent of the HEX number AB.CD is
8-4-2-1 binary-coded decimal counter? [2014-S2]  [2007]
(a) 1000 (b) 1001 (a) 253.314 (b) 253.632
(c) 0011 (d) 1100 (c) 526.314 (d) 526.632
Solution: (d) Solution: (b)
A BCD counter can count from 0000(0)d to
=
1001(9)d. Hence, 1100(12)d is not a valid state. 
   .110
( AB.CD)16 (10101011
=  01101
 
 ) (253.632) .
2 8

Hence, the correct option is (d). Hence, the correct option is (b).

Chapter 01.indd 3 11/9/2015 6:13:59 PM


Chapter 2
Boolean Algebra
Solution: (a)
One-mark Question
Y = ( ABC + D)( AD + BC )
1. In the sum of products function f (X, Y, Z) = Σ (2, = ABC ⋅ AD + ABC ⋅ BC + D ⋅ AD + D ⋅ BC
3, 4, 5), the prime implicants are [2012] = ABCD + 0 + AD + BCD (∵ X ⋅ X = X and XY = 0)
XY , X Y
(a) = AD( BC + 1) + BCD (∵ X + 1 = X )
XY , X Y Z , X Y Z
(b) = AD + BCD.
XY Z , XYZ , X Y
(c)
Hence, the correct option is (a).
(d) XY Z , XYZ , X Y Z , X YZ
2. The Boolean expression XY Z + XYZ + XY Z
Solution: (a)
+ X YZ + XYZ can be simplified to [2003]
 f ( X , Y , Z ) = Σ(2, 3, 4, 5) Because SOP form is
X Z + YZ + XZ
X Z + X Z + YZ (b)
(a)
mentioned, the K map for f is
XY + Y Z + X Z
XZ + YZ + Y Z (d)
(c)
YZ
X 00 01 11 10
Solution: (b)
0 1 1
f = x ′y + xy ′
1 1 1 Let f = XY Z + X YZ + XY Z + X YZ + XYZ .
Constructing the 3 variable K-map for the ex-
An implicant of a function is a product term that
pression,
is included in the function. A prime implicant of
a function is an implicant, i.e., not included in any we get, f = Σm(1, 2, 5, 6, 7)
other implicant of the function.
YZ
Thus, prime implicants of f are X′Y and XY′. X 00 01 11 10
Hence, the correct option is (a). 0 1 1
f = yz + yz + xy
1 1 1 1

Two-marks Questions or
1. The simplified form of the Boolean expression
X 00 01 11 10
Y = ( ABC + D) ( AD + BC ) can be written as
0 1 1
 [2004] f = yz + yz + xz
1 1 1 1
AD + BCD (b)
(a) AD + BCD
(c) AD + BC D
( A + D) + BC D) (d) Hence, the correct option is (b).

Chapter 02.indd 4 11/2/2015 3:20:59 PM


Chapter 3
Logic Gates
3. The digital circuit using two inverters shown in
One-mark Questions figure will act as [2004]

1. A bulb in a staircase has two switches, one switch


being at the ground floor and the other one at the
first floor. The bulb can be turned ON and also can
be turned OFF by any one of the switches irrespec-
tive of the state of the other switch. The logic of (a) a bistable multivibrator
switching of the bulb resembles [2013] (b) an astable multivibrator
(a) an AND gate (b) an OR gate (c) a monostable multivibrator
(c) an XOR gate (d) a NAND gate (d) an oscillator
Solution: (c) Solution: (a)
Let A be the condition of switch on ground floor (0 X
for off, 1 for ON) and B be the condition of switch X′ X
on first floor, and X  be the condition of bulb, then
A B X
0 0 0
0 1 1 The output of the circuit varies between X and X ,
thus it is a bistable multivibrator.
1 0 1
Hence, the correct option is (a).
1 1 0
4. The output of a logic gate is ‘1’ when all its inputs
This is equivalent to an XOR gate.
are at logic ‘0’. The gate is either [2001]
Hence, the correct option is (c). (a) a NAND or an EX-OR gate
2. The complete set of only those logic gates desig- (b) a NOR or an EX-OR gate
nated as universal gates is [2009] (c) an AND or an EX-NOR gate
(a) NOT, OR and AND gates (d) a NOR or an EX-NOR gate
(b) XNOR, NOR and NAND gates Solution: (d)
(c) NOR and NAND gates Output of NOR gate when all inputs are 0 = 1.
(d) XOR, NOR and NAND gates
Output of NAND gate when all inputs are 0 = 1.
Solution: (c)
Output of EXOR gate when all inputs are 0 = 0.
All gates can be derived from a combination of
NAND and NOR gates. Hence, these are desig- Output of EX-NOR gate when all inputs are 0 = 1.
nated as universal gates. Thus, from given options, answer is option (d).
Hence, the correct option is (c). Hence, the correct option is (d).

Chapter 03.indd 5 11/9/2015 6:37:53 PM


7.6 | Digital Electronics and Microprocessors

5. The Boolean expression for the output of the logic (c)


A
circuit shown in figure is [1996]
C
A
B
Y
C B

Y = AB + AB + C
(a) Y = AB + AB + C (b)
Solution: (c)
(c) Y = AB + AB + C (d)
Y = AB + AB + C
Solution: (a) AB
C 00 01 11 10
(XOR) 0 1 1
A A + B = AB + AB
B 1 1 1
Y = (AB + AB) • C
C
(NAND) F = AC + BC.

Y = ( AB + AB ) ⋅ C A
A AC
= AB + AB + C (using De Morgan’s Law )
= AB + AB + C (∵ AB + AB = A ⊕ B = A  B C AC + BC
C
= A(XNOR ) B). BC
B
Hence, the correct option is (a).
Hence, the correct option is (c).
Two-marks Questions 2. The SOP (sum of products O form of a Boolean
function is Σ(0, 1, 3, 7, 11), where inputs are A, B,
1. Which of the following logic circuits is a reali- C, D (A is MSB, and D is LSB). The equivalent
zation of the function F whose Karnaugh map is minimized expression of the function is [2014-S2]
shown in figure [2014-S1]
( B + C )( A + C )( A + B)(C + D)
(a)
AB
00 01 11 10 ( B + C )( A + C )( A + C )(C + D)
(b)
0 1 1
C ( B + C )( A + C )( A + C )(C + D)
(c)
1 1 1
( B + C )( A + B)( A + B)(C + D)
(d)
(a)
A Solution: (a)
F ( A, B, C , D) = Σm(0, 1, 3, 7, 11)
B = π m(2, 4, 5, 6, 8, 9, 10, 12, 13, 14, 15).
C

(b) CD
A 00 01 11 10
AB
00 0

B 01 0 0 0
C 11 0 0 0 0
(c) 10 0 0 0
A

C ⇒ F = ( B + C )( A + C )( A + B )(C + D).

B Hence, the correct option is (a).

Chapter 03.indd 6 11/9/2015 6:37:56 PM


Chapter 3  Logic Gates  |  7.7

 Common Data for Questions 3 and 4: Solution: (d)


3. A minimized form of the function F is [2010] Consider option (d).
YZ
X 00 01 11 10 X
X XY
0 1 1 1 0
1 0 0 1 0 Y
Y f = XY + YZ

F = X Y + YZ (b)
(a) F = XY + YZ Z YZ
(c) F = XY + Y Z (d)
F = XY + YZ
Solution: (a) Hence, the correct option is (d).
5. A, B, C and D are input bits, and Y is the output bit
YZ
X 00 01 11 10 in the XOR gate circuit of the figure below. Which
0 1 1 1 0 of the following statements about the sum S of
1 0 0 1 0 A, B, C, D and Y is correct? [2007]

F = XY + YZ A
XOR
B
Hence, the correct option is (a).
4. Which of the following circuits is a realization of XOR Y
the above function F? [2010]
C
XOR
(a) D
X
(a) S is always either zero or odd
F (b) S is always either zero or even
Y
(c) S = 1 only if the sum of A, B, C and D is given
Z (d) S = 1 only if the sum of A, B, C and D is odd
Solution: (b)

(b)
X
A B C D Y S
0 0 0 0 0 0
F 0 0 1 1 0 2
Y 1 1 1 1 0 4
Z
0 1 1 1 1 4
0 1 0 1 0 2
(c)
X 0 1 0 0 1 2

F (A = B = C = D) (Option ‘c’ violated).


Y (A = B, C = D) (Option ‘a’ violated).
Z
(A = B = C = D).
(A ≠ B, C= D).
(d) (A ≠ B, C ≠ D).
X
(A ≠ B, C = D) (Option ‘d’ violated).
Y F Hence, the correct option is (b).
6. In the figure, as long as X1 = 1 and X2 = 1, the out-
Z
put Q remains [2005]

Chapter 03.indd 7 11/10/2015 6:30:10 PM


7.8 | Digital Electronics and Microprocessors

1
Q P + Q + R + S (b)
(a) P+Q+R+S
1 Q
(c) ( P + Q) ( R + S ) (d) (P + Q) (R + S)
Q
Solution: (b)
(a) at 1 (b) at 0 The circuit reduces to:
(c) at its initial value (d) unstable
P P •Q PQ
Solution: (d)
Q
1 PQ • RS = P + Q + R + S
Q
1 Q R •S R •S
R
Q S

The output is oscillating between Q and Q, hence Hence, the correct option is (b).
it is unstable.
9. The logic function f = ( x ⋅ y ) + ( x ⋅ y ) is the same
Hence, the correct option is (d). as [1999]
7. A digital circuit which compares two numbers
f = ( x + y ) ( x + y )  (b)  f = ( x + y ) ( x + y )
(a) 
A3 A2 A1 Ao, B3 B2 B1 B0 is shown in figure. To get
output Y = 0, choose one pair of correct input (c)  f = ( x ⋅ y ) ⋅ ( x ⋅ y )    (d)  None of (a), (b), (c)
numbers [2004] Solution: (d)
B3 A3 B2 A2 B1 A1 B0 A0 f = ( x ⋅ y ) + ( x ⋅ y)
= ( x ⋅ y ) ⋅ ( x ⋅ y) (using De Morgan’s Law )
= ( x ⋅ y) ⋅ ( x ⋅ y )
= ( x y ) ⋅ ( xy )
= x y + xy
= xy + x y
Y = x ⊕ y.

(a) 1010, 1010 (b) 0101, 0101 Hence, the correct option is (d).
(c) 0010, 0010 (d) 0010, 1011 1 0. For a flip-flop formed from two NAND gates as
Solution: (d) shown in figure, the unusable state corresponds to
Output of NOR gate = 0 when at least one input is 1. [1999]
Output of XOR gate = 1 when bits are different. X Q
In option (d), A0 ⊕ B0 = 1 which serves as input to
NOR gate giving y = 0.
Hence, the correct option is (d).
Y
8. For the circuit shown in figure the Boolean expres-
sion for the output Y in terms of inputs P, Q, R (a) X = 0, Y = 0 (b) X = 0, Y = l
and S is [2002] (c) X = 1, Y = 0 (d) X = l, Y = ∞
P Solution: (a)
Q Assuming Q = 0 and Q = 1 in the previous state,
R
Y with  X = 0, Y = 0 ⇒ Q = 1, Q = 1  which gives
invalid result.
S
Hence, the correct option is (a).

Chapter 03.indd 8 11/9/2015 6:37:59 PM


Chapter 4
Combinational Digital
Circuits
(a) 1 (b) 0
One-mark Questions (c) X (d) X
Solution: (a)
1. The output Y of a 2-bit comparator is logic 1 when-
Output of XOR gate = 0 when its inputs are equal,
ever the 2-bit input A is greater than the 2-bit input
otherwise output is 1.
B. The number of combinations for which the out-
put is logic 1, is [2012] Hence, the correct option is (a).
(a) 4 (b) 6 3. Figure shows a 4 to 1 MUX to be used to imple-
(c) 8 (d) 10 ment the sum S of a 1-bit full adder with input bits
Solution: (b) P and Q and the carry input Cjn. Which of the fol-
A A= lowing combinations of inputs to I0, I1, I2 and I3 of
= 1 A0 , B B1 B0
Let
the MUX will realize the sum S ? [2003]
A1 A0 B1 B0 Output
0 1 0 0 1 I0
1 0 0 0 1
I1
1 0 0 1 1 4×1
S
MUX F
1 1 0 0 1 I2

1 1 0 1 1 I3 S1 S0
1 1 1 0 1

\  For 6 combinations, output is logic 1. P Q


Hence, the correct option is (b).
2. The output Y of the logic circuit given below is
I=
(a) I= Cin ; I= I= C in
 [2011] 0 1 2 3

I=
(b) 0 I=
1 C in ; I=
2 I=
3 Cin
X
Y I=
(c) 0 I=
3 Cin ; I=1 I=
2 C in
I=
(d) 0 I=
3 C in ; I=
1 I=
2 Cin

Chapter 04.indd 9 11/9/2015 6:39:17 PM


7.10 |  Digital Electronics and Microprocessors

Solution: (c) Solution: (b)


P Q Cin S ( s1 ) ( s0 )
0 0 0 0 x y f
0 0 1 1 0 0 0
0 1 0 1 0 1 1
1 0 1
0 1 1 0
1 1 1
1 0 0 1
⇒ f = x+ y
1 0 1 0
1 1 0 0 Hence, the correct option is (b).

1 1 1 1
Two-marks Questions
S = Σm(1, 2, 4, 7).
1. A 3 line to 8 line decoder, with active low outputs,
The corresponding values of Cin are 1, 0, 0, 1 re- is used to implement a 3-variable Boolean func-
spectively, i.e., when tion as shown in the figure: The simplified form
P = 0,  Q = 0,  Cin = 1. of Boolean function F(A, B, C) implemented in
P = 0,  Q = 1,  Cin = 0. ‘Product of Sum’ form will be [2008]
P = 1,  Q = 0,  Cin = 0.
0
P = 1,  Q = 1,  Cin = 1. Z A0 1
\  MUX inputs are Y A1
2
3L × 8L 3
Decoder 4
Cin I0 X A2
5
Cin I1 6
S 7
Cin I2

Cin I3   (a)  ( x + z ) ⋅ ( x + y + z ) ⋅ ( y + z )
  (b)  ( x + z ) ⋅ ( x + y + z ) ⋅ ( y + z )
P Q   (c)  ( x + y + z ) ⋅ ( x + y + z ) ⋅ ( x + y + z ) ⋅ ( x + y + z )
  (d)  ( x + y + z ) ⋅ ( x + y + z ) ⋅ ( x + y + z ) ⋅ ( x + y + z )
Hence, the correct option is (c).
4. The output f of the 4 to l MUX shown in figure is Solution: (a)
 [2001] X Y Z Output line Output (F )
VCC 3 0 0 0 0 0
2 f 0 0 1 1 1
MUX
1 0 1 0 2 0
0 S1 S0 0 1 1 3 1
1 0 0 4 0
1 0 1 5 1
x y
1 1 0 6 1
(a) x+y
xy + x (b) 1 1 1 7 0
(c) x + y (d) xy + x ⇒ F = π m(0, 2, 4, 7)

Chapter 04.indd 10 11/9/2015 6:39:19 PM


Chapter 4  Combinational Digital Circuits  |  7.11

In POS form, F = ( X + Y + Z ) ⋅ ( X + Y + Z ) 3. The minimal product-of-sums function described


⋅ ( X + Y + Z ) ⋅ ( X + Y + Z ). by the K-map given in figure is [2000]
Simplifying further using K-maps, AB
YZ C 00 01 11 10
X 00 01 11 10
0 1 1 f 0
0 0 0
1 0 0 f 0
1 0 0

F = (Y + Z ) ⋅ ( X + Z ) ⋅ ( X + Y + Z ).
(a) AC (b) A+C
Hence, the correct option is (a). (c) A + C (d) AC
2. A 4 × 1 MUX is used to implement a 3- input Boolean Solution: (b)
function as shown in figure. The Boolean function F
(A, B, C) implemented is [2006] AB
C 00 01 11 10
A I0
0 1 1 0
I1
1 0 0 0
‘1’ I2 F(A, B, C)
‘0’ I3
S1 S0
F = C + A = A + C.

In case of POS sum, we group all zeroes.


B C
(a) F(A, B, C ) = ∑ (l, 2, 4, 6) Hence, the correct option is (b).
(b) F(A, B, C ) = ∑ (l, 2, 6) 4. The logic function F = AC + ABD + ACD is to be
(c) F(A, B, C ) = ∑ (2, 4, 5, 6) realized using an 8 to 1 multiplexer shown in the
(d) F(A, B, C ) = ∑ (l, 5, 6) figure, using A, C and D as control inputs.
Solution: (a) [1999]
The truth table for the given function f (A, B, C ) is
B C F
0
0 0 A 1
0 1 A 2
3 8×1
1 0 1 F
4 MUX
1 1 0 5
6
Expanding the truth table,
7
A B C F D C A

0 0 0 0
0 0 1 1 LSB
0 1 0 1
(a) Indicate the inputs to be applied at the termi-
0 1 1 0
nals 0 to7.
1 0 0 1 (b) Can the function be realized using a 4 to 1
1 0 1 0 multiplexer?
1 1 0 1 State YES or NO.
1 1 1 0
⇒ F ( A, B, C ) = Σm(1, 2, 4, 6)
Hence, the correct option is (a).

Chapter 04.indd 11 11/9/2015 6:39:20 PM


7.12 |  Digital Electronics and Microprocessors

Solution: (b) Implementation using 4 × 1 MUX is not pos-


(a) D C A B Y = AC + ABD + ACD sible because for same logic on select lines,
0 0 0 × 0 more than one input is possible to be MUXed.
0 0 1 × 0 Hence, the correct option is No.
0 1 0 × 0 5. Match the following [1998]
0 1 1 × 1  Logic Function
1 0 0 × 0 (A)  X + Y (P) Sum
1 0 1 × ABD = B (∵ A = 1, D = 1) (B)  XY (Q) NAND
(C)  X Y (R) Carry
1 1 0 × 0
(S) NOR
1 1 1 × B Solution:
X + Y = X ⋅ Y = NAND gate logic.
(∵ For B = 0, Y = 0 with carry = 1. For B = 1, Y = 1
with carry = 1) ∴ ( A) ↔ (Q).
XY = Logic for carry of half adder.
0 I0 ∴ ( B) ↔ ( R).
0 I1
X Y = X + Y = NOR gate logic.
0 I2
1 I3 8 × 1 ∴ (C ) ↔ ( S ).
F
0 I4 MUX
B I5
0 I6
B I7

D C A

Chapter 04.indd 12 11/9/2015 6:39:22 PM


Chapter 4  Combinational Digital Circuits  |  7.13

2. A 3-input 2-output priority encoder has the follow-


Five-marks Questions ing truth table where X indicates don’t care con-
ditions. Realize the logic using NAND gates and
1. In a digital combinatorial circuit with 4 inputs (A, inverters. [1997]
B, C, D), it is required to obtain an output of logi-
cal 1 only for the input combination (A = l; B = C W2 W1 W0 Y1 Y0
= D = 0). It is known that the following combina- 0 0 0 0 0
tions of input are forbidden: 0 0 1 0 0
ABCD = 1010, 1011, 1100, 1101, 1110, 1111. 0 1 × 1 0
Evaluate the logical expression for the out-
put and realize the same with two input NAND 1 × × 1 1
gates. Assume that complements of inputs are not
­available. [1998] Solution:
Solution: w2 w1 w0 y1 y0
A B C D f 0 0 0 0 0
0 0 0 0 0 0 0 1 0 0
0 0 0 1 0 0 1 0 1 0
0 0 1 0 0 0 1 1 1 0
0 0 1 1 0 1 0 0 1 1
0 1 0 0 0 1 0 1 1 1
0 1 0 1 0 1 1 0 1 1
0 1 1 0 0 1 1 1 1 1
0 1 1 1 0 K-map for y0
1 0 0 0 1
W1
1 0 0 1 0 W2 W0
00 01 11 10
1 0 1 0 ×
0
1 0 1 1 ×
1 1 1 1 1
1 1 0 0 ×
1 1 0 1 × ⇒ y0 = w2 .
1 1 1 0 × K-map for y1
1 1 1 1 × W1
W2 W0
K-map implementation: 00 01 11 10
CD 0 1 1
AB 00 01 11 10
1 1 1 1 1
00
01
⇒ y1 = w2 + w1 = w1 ⋅ w2 .
11
10 1 Implementation using NAND logic gates:
⇒ f = AD. W2 Y0
W2
NAND gate implementation:
W1.W2
A A•D AD Y1
W1
D
D W1

Chapter 04.indd 13 11/9/2015 6:39:23 PM


Chapter 5
Sequential Digital Circuits

One-mark Questions
D Q
1. A cascade of three identical modulo 5 counters has
an overall modulus of [2014-S1]
CLK Q
(a) 5 (b) 25 10 kHz
(c) 125 (d) 625
Solution: (c) (a) 10 kHz (b) 2.5 kHz
3 Modulo 5 counters in cascade will divide the (c) 20 kHz (d) 5 kHz
1 1 1 1 Solution: (d)
frequency by × × = . Thus, the counters
5 5 5 125 Input frequency
have an overall modulus of 125.
Hence, the correct option is (c).
2. Consider the given circuit. In this circuit, the race
around [2012] Output frequency
A

CLK f
frequency =
2

B Qt D Qt + 1
(a) does not occur 0 1 1 1
(b) occurs when CLK = 0 Next cycle → 1 0 0 0
(c) occurs when CLK = I and A = B = 1 0 1 1 1
(d) occurs when CLK = 1 and A = B = 0
Solution: (a) And so on.
The circuit given is that of an S-R flip flop. S-R flip This forms a modulo-2 counter, and therefore, out-
flop has an invalid state when S = 1, R = 1 but no 1
put frequency = (input frequency) = 5 kHz.
race around. 2
Hence, the correct option is (a). Hence, the correct option is (d).
3. The frequency of the clock signal applied to the 4. For a J-K flip-flop its J input is tied to its own Q out-
rising edge triggered D flip-flop shown in figure put and its K input is connected to its own Q output. If
is 10 kHz. The frequency of the signal available at the flip-flop is fed with a clock of frequency 1 mHz,
Q is [2002] its Q output frequency will be ______ [1995]

Chapter 05.indd 14 11/9/2015 6:40:52 PM


Chapter 5  Sequential Digital Circuits  |  7.15

Solution: Solution: (b)

J
J K Qn Qn T = ( J + Qn )( K + Qn ) Qn + 1
Q
Cu 0 0 0 1 0 0
K Q  Qn
0 0 1 0 0 1
0 1 0 1 0 0
Qt −1 J K Qt 0
0 1 1 0 1 0
0 0 0 0
1 0 0 1 1 1
0 1 1 1 1
1 0 1 0 0 1
1 1 1 0
1 1 0 1 1 1
0 0 0 0  Qn
1 1 1 0 1 0
This forms a modulo-2 counter and therefore, out-
1 Hence, correct implementation is as in (b)
put frequency = (Input frequency) KQn
2
J 00 01 11 10
1
= = (1 kHz) 500 Hz. 0 0 0 1 0
  2
1 1 0 1 1

Two-marks Questions T = JQ n + KQn = ( J + Qn )( K + Q n )


Hence, the correct option is (b).
1. A JK flip flop can be implemented by T flip-flops. 2. The clock frequency applied to the digital circuit
Identify the correct implementation. [2014-S2] shown in the figure below is 1 kHz. If the initial
(a) state of the output Q of the flip-flop is ‘0’, then
the frequency of the output waveform Q in kHz is
J Qn  [2013]
T
T flip- (a) 0.25 (b) 0.5
K CLK
flop (c) 1 (d) 2
Qn Solution: (b)
(b) X = (Q ⊗ Q) ⋅ (Q  Q) = 1.
J Qn
T Thus, T = X = 1, i.e., T flip-flop will work in toggle
CLK
T flip- mode always.
K flop 1
Output frequency = (input frequency)
Qn 2
1
(c) = = (1 kHz) 0.5 kHz.
  2
J Qn Hence, the correct option is (b).
T
CLK T flip- 3. The state transition diagram for the logic circuit
K flop shown is [2012]
Qn
2-1 MUX
(d) D Q X1
J Qn
Y
T
T flip- CLK Q X0
CLK Select
K flop
Qn
A

Chapter 05.indd 15 11/9/2015 6:40:53 PM


7.16 | Digital Electronics and Microprocessors

 (a)  A=1 A=0  (b) A = 0 A=0 After 3 clock pulses, state = 10.
A=1 A=1 Hence, the correct option is (c).
Q=0 Q=1 Q=0 Q=1 5. The digital circuit shown in the figure works as a
A=0 A=1
 [2005]
 (c)  A=0 A=1  (d) A = 1 A=1
A=0 A=0 D Q
X

Q=0 A=1 Q=1 Q=0 A=0 Q=1


CLK Q

(a) JK flip-flop
Solution: (d)
(b) Clocked RS flip-flop
D = AX 0 + AX 1 (c) T flip-flop
Present state Input Next state (d) Ring counter
= AQ + AQ
Solution: (c)
(Q) (A) (Q)
0 0 1 1 Q(t + 1) = D = X ⊕ Q(t )
0 1 0 0 ∴ Q(t + 1) = X ⊕ Q(t )
1 0 0 0 X Q(t) Q(t + 1)
1 1 1 1 0 0 0 
 Q(t ) = Q(t + 1) when x = 0
The state transition diagram is as: 0 1 1 
A=1 A=1
1 0 1 
 Q(t ) = Q(t + 1) when x = 1
A=0 1 1 0 
Q=0 Q=1
This is the truth table of a T flip-flop with T input
A=0 being equal to X.
Hence, the correct option is (d). Hence, the correct option is (c).
4. A two-bit counter circuit is shown below 6. Select the circuit which will produce the given out-
put Q for the input signals X1 and X2 given in the
JA QA TB QB QB figure. [2005]
X1
t
KA QA QB
X2 t
t
Q
CLK

It the state QAQB of the counter at the clock time tn (a) 
X1   (b) X 1
is ‘10’ then the state QAQB of the counter at tn + 3 Q Q
(after three clock cycles) will be [2011]
(a) 00 (b) 01
(c) 10 (d) 11
Solution: (c) X2 X2

Clock JA KA TB QA QB (c) 
X1   (d) X 1
Q Q
Initially = QB = QB = QA 1 0
1st 1 0 1 1 1
2nd 0 1 1 0 0
X2 X2
3rd 1 0 0 1 0

Chapter 05.indd 16 11/9/2015 6:40:54 PM


Chapter 5  Sequential Digital Circuits  |  7.17

Solution: (b) (a) Equal to 2N− 2


X1 X2 Qa Qb Qc Qd (b) Equal to 2N− 1
(c) Proportional to Va
0 0 0 0 0
(d) Inversely proportional to Va
0 1 1 0 0
Solution: (c)
1 0 0 1 1
Analog i/p (Va ) Switch control
1 1 − 1 0 Control
R
(c ) = ( d ) − X
+ +
− Binary
From the timing diagrams, Qb represents the out- Integrator Comparator CLK counter
put diagram.
CLK Binary output
Hence, the correct option is (b).
7. The shift register shown in figure is initially The output X of comparator gets ended with clock
loaded with the bit pattern 1010. Sequential acts. signal (+ve or −ve edge) and binary output is thus
Subsequently the shift register is clocked, and with proportional to X, which is proportional to Va.
each clock pulse the pattern gets shifted by one Hence, the correct option is (c).
bit position to the right. With each shift, the bit at 9. The counter shown in figure is initially in state Q2 = 0,
the serial input is pushed to the left most position Q1 = l, Q0 = 0. With reference to the CLK input,
(MSB). After how may clock pulses will the content draw waveforms for Q2, Q1, Q0 and P for the next
of the shift register become 1010 again? [2003] three CLK cycles. [2000]
Clock 1 0 1 0 T0 Q0 T1 Q1 T2 Q2
Serial input
Q2

Sd Q 0 Sd Q 1 Sd Q 2
P

(a) 3 (b) 7
(c) 11 (d) 15
Solution: (b)
Solution:
CLK A B C D Serial input = B ⊕ (C ⊕ D)
0 1 0 1 0 1 CLK plus Q0 Q1 Q2 P
1 1 1 0 1 0 Initial 0 1 0 1
2 0 1 1 0 0 1st 1 1 0 1
3 0 0 1 1 0 2nd 0 0 1 1
4 0 0 0 1 1 3rd 1 1 1 0
5 1 0 0 0 0
6 0 1 0 0 1 CLK
7 1 0 1 0 (−ve edge
triggered)
1010 repeats after 7 clock cycles. Q0
Hence, the correct option is (b). Q1
8. A dual-slope analog-to-digital converter uses an
N-bit counter. When the input signal Va is being inte- Q2
grated, the counter is allowed to count up to a value:
P
[2000]

Chapter 05.indd 17 11/9/2015 6:40:54 PM


7.18 | Digital Electronics and Microprocessors

Solution:
Five-marks Questions (a)
CLK  Q2  Q1   Q0   A
1. The digital circuit shown in figure generates a 0   0  0  0  1
modified clock pulse at the output. Choose the 1   0  0  1  1
correct output waveform from the options given 2   0  1  0  1
below. [2004] 3   0  1  1  1
PRE = 1 4   1  0  0  1
1
5   0  0  0  0 ← Q0, Q1, Q2 are
J Q 6   0  0  1  1 cleared as A = 0 when
CLK O/P 7   0  1  0  1 Q2 = 1 and Q0 = 1
8   0  1  1  1
K Q
1 9   1  0  0  1
CLR = 1 10   0  0  0  1
(b) Modulo number of counter = 5, because it re-
Solution: sets after 5 clock pulses.
Q2 Q1 Q0

CLK (c) 6 = (1 1 0) 2
For modulo 6 counter, Q1 and Q2 should be con-
J = K = 1,
Q which nected to the input of NAND gate.
means, 3. For the ring counter shown in figure, find the steady
Q toggles
state sequence if the initial state of the counter is
1110 (i.e., Q3Q2Q1Q0 = 10). Determine the MOD
CLK and Q number of the counter. [2001]
O/P

2. The ripple counter shown in figure is made up of


negative edge triggered J-K flip-flops. The signal
levels at J and K inputs of all the flip-flops are D Q3 D Q2 D Q1 D Q0
maintained at logic 1. Assume that all outputs are
cleared just prior to applying theclock signal.
CLK1 Q2 Q1 Q0 A
CLK

J Q0 J Q1 J Q2 Solution:
CLK1 CLK CLK CLK CLK D = Q1 + Q3 Q2 Q1 Q0
K K K 0 - 1 1 1 0 ← initial state
CLR CLR CLR 1 0 0 1 1 1
A 2 0 1 0 1 1
3 0 1 0 0 1
(a) Create a table of Qo, Q1, Q2 and A in the format 4 1 1 0 0 0
given below for 10 successive input cycles of 5 0 0 1 0 0
the clock CLK1. 6 0 0 0 1 0
(b) Determine the module number of the counter. 7 0 0 0 0 1
(c) Modify the circuit of figure to create a a mod-
ulo-6 counter using the same components used Same state repeats after 4 clock pulses.
in the figure. [2002] \  Modules = 4

Chapter 05.indd 18 11/9/2015 6:40:55 PM


Chapter 5  Sequential Digital Circuits  |  7.19

4. (a) Construct the truth table for the circuit given (b)
in figure. Q1, Q2 and Q3 are outputs and the clock 1 2 3 4 5 6
pulses are the inputs. Unused J, K inputs are
assumed to be at logic 1. All flip-flops are reset at CLK
power ON. [1998]
Q1

Q2
J1 Q 1 J2 Q 2 J3 Q 3
Q3
K1 Q 1 K2 Q 2 K3 Q 3
CLK
(c) The circuit performs the function of a MOD-5
hybrid counter.
(b) Sketch the output waveforms at Q1 Q2 and Q3
(c) What function does this circuit perform.
Solution:
(a)
CLK Q1 Q2 Q3 J1K1 J2K2 J3K3 Q1 Q2 Q3
1 0 0 0 1 1 1 1 0 1 1 0 0
2 1 0 0 1 1 1 1 0 1 0 1 0
3 0 0 0 1 1 1 1 0 1 1 1 0
4 1 1 0 1 1 1 1 1 1 0 0 1
5 0 0 1 0 1 1 1 0 1 0 0 0

unused

Chapter 05.indd 19 11/9/2015 6:40:55 PM


Chapter 6
Semiconductor Memories
(a) (v), (ii), (iii), (iv), (i)
One-mark Question (b) (v), (ii), (iii), (i), (iv)
(c) (ii), (i), (iii), (iv), (v)
1. The increasing order of speed of data access for (d) (v), (ii), (i), (iii), (iv)
the following devices is [2009] Solution: (b)
(i) Cache memory Increasing order of speed is
(ii) CDROM
(iii) Dynamic RAM Magnetic Tape < CDROM < DRAM < Cache
(iv) Processor registers < Processor Registers.
(v) Magnetic tape Hence, the correct option is (b).

Chapter 06.indd 20 11/2/2015 4:07:32 PM


Chapter 7
Logic Gate Families
Solution: (a)
One-mark Questions VCC

1. If X1 and X2 are the inputs to the circuit shown in


the figure, the output Q is [2005] P PQ
Q

VDD
R
S RS

Output = PQ ⋅ RS.
Q
Hence, the correct option is (a).
Q1 Q2 Q3
X1 X2 3. In standard TTL gates, the totem pole output stage
is primarily used to [1998]
(a) increase the noise margin of the gate
(b) decrease the output switching delay
X1 ⋅ X 2
X 1 + X 2 (b)
(a) (c) facilitate a wired OR logic connection
(d) increase the output impedance of the circuit
X1 ⋅ X 2
X 1 ⋅ X 2 (d)
(c)
Solution: (b)
Solution: (d) Totem pole output stage is used to decrease the
Q1 forms a NOT gate with input X1. output switching delay.
Q2 and Q3 form a NOR gate with input X 1  (output Hence, the correct option is (b).
of Q1) and X2.

Q = X 1 + X 2 = X 1 ⋅ X 2 (Using DeMongan’s Law). Two-marks Questions


Hence, the correct option is (d).
1. The TTL circuit shown in the figure is fed with
2. The open collector outputs of two 2-inputs NAND
the waveform X (also shown). All gates have equal
gates are connected to a common pull up resistor.
propagation delay of 10 ns. The output Y of the
If the input to the gates are P, Q and R, S respec-
circuit is [2010]
tively, the output is equal to [1998]
X 100 ns
PQ ⋅ RS (b)
(a) PQ + RS
(c)
PQ + RS (d) PQRS 0 t

Chapter 07.indd 21 11/9/2015 6:42:03 PM


7.22 | Digital Electronics and Microprocessors

X +5 V
Y

4 kΩ 1.6 kΩ

(a) Y (b) Y
1 1 V1 Q2
Q1
0 t 0 t

(c) Y (d) Y
1 1 (a)
Q1 ON and Q2 OFF
0 t 0 t (b)
Q1 reverse ON and Q2 OFF
(c)
Q1 reverse ON and Q2 ON
(d)
Q1 OFF and Q2 reverse ON
Solution: (a)
X Solution: (c)
100 ns VBE = 0.7 V.
t 2′ Vi = 3 V.
t (ns)
t1 20 ns t 1′ t2
A 20 ns
5V

t (ns) 4 kΩ 1.6 kΩ
Y 0(XOR)1 0(XOR)1 X Z
=1 B
1(XOR)1 Q2
3V
E Q1 Y B
t (ns)
t1 t 1′ t2 t 2′ E
20 ns 20 ns
X = 3.7 V = VX
X Y

10 ms 10 ms A =
Voltage at point X 3=
.7 V VX .
delay delay
V=
C V=
Y 0.7 V.
Hence, the correct option is (a).
2. A TTL NOT gate circuit is shown in figure. Since VX > VY, Q1 is reverse ON and Q2 is ON
Assuming Vbe= 0.7 V of both the transistors, if
Vi = 3.0 V, then the states of the two transistors will (∵ VZ > VY ) .
be [2006] Hence, the correct option is (c).

Chapter 07.indd 22 11/9/2015 6:42:04 PM


Chapter 8
A/D and D/A Converters
3. The number of comparisons carried out in a 4-bit
One-mark Questions flash-type A/D converter is [1994]
(a) 16 (b) 15
1. The voltage comparator shown in figure can be (c) 4 (d) 3
used in the analog-to-digital conversion as [2004] Solution: (b)
Number of comparisons in flash type A/DC = 2N - 1,
V1 + V0 N = number of bits
V2 −
 = 24 - 1 = 15.
Hence, the correct option is (b).
(a) a 1-bit quantizer (b) A 2-bit ­quantizer
4. A 10 bit A/D converter is used to digitize an analog
(c) a 4-bit quantizer (d) An 8-bit quantizer
signal in the 0 to 5 V range. The maximum peak to
Solution: (a) peak ripple voltage that can be allowed in the DC
V1 V2 supply voltage is [1993]
V0=V1 − V2
(a) nearly 100 mV (b) nearly 50 mV
0 0 0 (c) nearly 25 mV (d) nearly 5.0 mV
0 1 1 Solution: (d)
1 0 1 Number of bits, N = 10.
1 1 0 Range of input voltage = 5 V.
Range 5 5
   V0 Can either be 0 or 1. Hence, it is one bit quantizer.    ∴ Resolution = = 10 = ≈ 5 mV
2N 2 1024
Hence, the correct option is (a).    = peak to peak ripple.
2. Among the following four, the slowest ADC Hence, the correct option is (d).
(analog-to-digital converter) is [2001]
(a) parallel-comparator (i.e., flash) type
(b) successive approximation type Two-marks Questions
(c) integrating type
(d) counting type 1. A student has made a M − 3 bit binary down coun-
Solution: (c) ter and connected to the R − 2R ladder type DAC
ADCs arranged in increasing order of speed: [Gain = (−l kΩ/2R) as shown in figure to generate
a staircase waveform. The output achieved is dif-
Integrating type < counting type < SAR < flash ADC.
ferent as shown in figure. What could be the pos-
Hence, the correct option is (c). sible cause of this error? [2006]

Chapter 08.indd 23 11/9/2015 6:42:52 PM


7.24 | Digital Electronics and Microprocessors

R R R 1 kΩ
Full scale output voltage
Resolution = .
2N
2R 2R 2R 2R +12 V
− 3.5 V
V0  ⇒  14 mV =
+ 2N
−12 V
Counter 10 kΩ 3.5
1 kHz  ⇒         2 N = × 103 = 250
Clock 14
N ≈ 8.
7
6 Hence, the correct option is (b).
5
4 3. The simplified block diagram of a 10-bit A/D con-
3 verter of dual slope integrator type is shown in the
2
1 figure. The 10-bit counter at the output is clocked
0 by a 1 mHz clock. Assuming negligible timing over-
0 1 2 3 4 5 6 7 ms head for the control logic, the maximum frequency

(a) The resistance values are incorrect of the analog signal that can be converted using this

(b) The counter is not working properly A/D converter is approximately. Input sample to be

(c) The connection from the counter to DAC is not converter [2003]
proper
(d) The R and 2R resistances are interchanged Input sample to
be converted Integrator, 10-bit
Solution: (c) Comparator Counter
Required output Output generated and Control
Reference Logic Clock
= 7 111= 111 7 DC output Digital
output
= 6 110= 011 3
5 = 101 101 = 5 (a) 2 kHz (b) 1 kHz
=    4 100 = 001 1 (c) 500 Hz (d) 250 Hz
= 3 011= 110 6 Solution: (b)
= 2 010= 010 2 Clocking frequency = 1 mHz.
= 1 001= 100 4 ∴  Sample time for each conversion = 1 ms.
= 0 000= 000 0 Max conversion time = (2N - 1) × 1 μ sec
The output generated is the reverse of output re- = (210 - 1) × 1 μs
quired, i.e., MSB and LSB are interchanged in out- ≈ 1024 μs ≈ 1 ms.
put generated. This could have happened because Thus, sampling period TS = 1 ms.
connection of counter to DAC is reversed. 1
Sampling frequency F=
S = 1 kHz.
Hence, the correct option is (c). TS
2. A digital-to-analog converter with a full-scale Hence, the correct option is (b).
output voltage of 3.5 V has a resolution close to
1
14 mV. Its bit size is [2005] 4. For a dual ADC type 3 digit DVM, the reference
(a) 4 (b) 8 2
voltage is 100 mV and the first integration time is set
(c) 16 (d) 32
to 300 ms. For some input voltage, the ­‘deintegration’
Solution: (b) period is 370.2 ms. The DVM will indicate.
V0 Full scale output voltage = 3.5 V. [1999]
Resolution = 14 mV. (a) 123.4 (b) 199.9
N = Number of bits. (c) 100.0 (d) 1.414

Chapter 08.indd 24 11/9/2015 6:42:53 PM


Chapter 8  A/D and D/A Converters  |  7.25

Solution: (a) Vin T1 = Vref T2 .


Vref = 100 mV.
100 × 10−3 × 370.2 × 10−3
T1 = First integration time = 300 ms. ⇒ Vin = = 123.4 mV.
300 × 10−3
T2 = Deintegration time = 370.2 ms.
Hence, the correct option is (a).
Vin = DVM output display voltage.

Chapter 08.indd 25 11/9/2015 6:42:54 PM


Chapter 9
Microprocessors
Solution: (a)
One-mark Questions A12 to A15 - used for chip selection.
A0 to A11 - used for address line selection.
1. An input device is interfaced with Intel 8085A
A15 A14 A13 A12 A11 … A0
microprocessor as memory mapped I/O. The
1 1 1 0 0 … 0 = E000 H
address of the device is 2500 H. In order to input
data from the device to accumulator, the sequence …
of instructions will be [2008] 1 1 1 0 1 … 1 = EFFF H
(a) LXI H, 2500 H Hence, the correct option is (a).
(b) LXI H, 2500 H MOV A, M MOV M, A 3. When a program is being executed in an 8085 micro-
(c) LHLD 2500 H processor, its Program Counter contains [2002]
(d) LHLD 2500 H MOV A, M MOV M, A (a) the number of instructions in the current pro-
Solution: (a) gram that have already been executed.
In memory mapped I/O, I/O devices are treated as (b) the total number of instructions in the program
memory locations. being executed.
∴  Code will be (c) the memory address of the instruction that is
being currently executed.
∴ LXI H, 2500 H : HL Points to memory loca- (d) the memory address of the instruction that is
tion 2500 H. to be executed next.
MOV A, M : Data at (address pointed by HL) memory Solution: (d)
is moved into A. The PC contains the memory address of instructions
Hence, the correct option is (a). that is to be executed next.
2. The logic circuit used to generate the active low chip Hence, the correct option is (d).
select (CS) by an 8085 microprocessor to address a 4. Which one of the following is not a vectored
peripheral is shown in figure. The peripheral will interrupt? [2000]
respond to addresses in the range. [2002] (a) TRAP (b) INTR
(c) RST 7.5 (d) RST 3
A15 Solution: (b)
A14
CS
TRAP, RST 7.5, RST 6.5, RST 5.5 are automati-
A13 cally vectored to specific memory locations. RST
A12
instructions like RST 0 to RST 7 get translated into
addresses when written in a program.
(a) E000 - EFFF (b) 000E - FFFE INTR is not a vectored interrupt.
(c) 1000 - FFFF (d) 0001 - FFF1 Hence, the correct option is (b).

Chapter 09.indd 26 11/9/2015 6:43:54 PM


Chapter 9  Microprocessors  |  7.27

5. In a microprocessor, the address of the next Solution: (b)


instruction to be executed, is stored in [1997] The decreasing order of priority of external hard-
(a) stack pointer. ware interrupts is
(b) address latch. TRAP > RST 7.5 > RST 6.5 > RST 5.5 > INT R.
(c) program counter.
∴ From the options, A must use RST 7.5, B must
(d) general purpose register.
use RST 6.5 and C must use RST 5.5.
Solution: (c)
Hence, the correct option is (b).
In the microprocessor, the address of next instruction
to be executed is stored in the program counter. 9. If the HLT instruction of a 8085 microprocessor is
executed, [1992]
Hence, the correct option is (c).
(a) the microprocessor is disconnected from the
6. In an 8085 microprocessor, after the execution of system bus till the Reset is pressed
XRA A instruction [1995] (b) the microprocessor enters into a Halt state and
(a) the carry flag is set. the buses are tri-stated
(b) the accumulator contain FF H. (c) the microprocessor halts execution of the pro-
(c) the Zero flag is set. gram and returns to monitor
(d) the accumulator contents are shifted left by (d) the microprocessor reloads the program from
one bit. the locations 0024 and 0025 H
Solution: (c) Solution: (b)
A ⊕ A = 0. When HLT instruction is executed, MPU finishes
executing the current instruction and halts any fur-
Thus, XRA A instruction causes accumulator to
ther execution. The MPU enters Halt Acknowledge
clear and zero flag is set.
machine cycle and wait states are inserted in every
Hence, the correct option is (c). clock cycle. The address and data bus are placed in
7. The contents of the accumulator in an 8085 micro- high impedance state.
processor are altered after the execution of the Hence, the correct option is (b).
instruction [1994]
(a) CMP C (b) CPI 3A
(c) ANI 5C (d) ORA A
Solution: (c)
Two-marks Questions
Compare instructions do not alter the accumulator
1. An output device is interfaced with 8 bit micropro-
content and ORA A gives the same output as A. AND
cessor 8085 A. The interfacing circuit is shown in
instruction changes the content of accumulator.
figure [2014-S1]
Hence, the correct option is (c).
AB
8. Three devices A, B and C have to be connected to 8
a 8085 microprocessor. Device A has highest pri- BDB
3L × 8L Decoder 8
ority and device C has the lowest priority. In this A15
I2 0
context which of the following is correct assign- A14
I1 1 Output port
ment of interrupt inputs? [1993]
A13 2
(a)  A uses TRAP, B uses RST 5.5 and C uses RST I0 8
A12 3
6.5 4
A11 E1
(b) A uses RST 7.5, B uses RST 6.5 and C uses 5
Output device
RST 5.5 E2 6
(c)  A uses RST 5.5, B uses RST 6.5 and C uses E3 7
RST 7.5 10
(d) A uses RST 5.5, B uses RST 6.5 and C uses WR
M
TRAP BCB

Chapter 09.indd 27 11/9/2015 6:43:54 PM


7.28 |  Digital Electronics and Microprocessors

The interfacing circuit makes use of 3 line to 8 3. A portion of the main program to call a subroutine
line decoder having 3 enable lines E1 , E 2 , E3 . The SUB in an 8085 environment is given below:
address of the device is LXI  :DDISP
(a) 50 H (b) 5000 H   LP     :CALL SUB
(c) A0 H (d) A000 H
It is desired that control be returned to LP + DISP
Solution: (b) + 3 when the RET instruction is executed in the
subroutine. The set of instructions that precede the
A15 A14 A13 A12 A11 A10 ........ A0 RET instruction in the subroutine are [2011]
0
1 
0 10 0..........0 (a) POP D (b) POP H
To select 2 nd output from decoder ( D2 ) To enable decoder DAD H DAD D
PUSH D INX H
Hence, address of device is 5000 H.
INX H
Hence, the correct option is (b). INX H
2. In an 8085 microprocessor, the following program PUSH H
is executed [2014-S2] (c) POP H (d) XTHL
Address location Instruction DAD D INX D
2000 H XRA A PUSH H INX D
2001 H MVIB, 04 H INX D
2003 H MVI A, 03 H XTHL
2005 H RAR Solution: (b)
2006 H DCR B LP + 1 is stored on stack.
2007 H JNZ 2005 ∴ In SUB, POP H  ;(HL) ← (LP) + 1.
200 AH HLT
DAD D; (HL) ← (DE) + (HL) i.e., (HL) ← (LP)
At the end of program, register A contains + 1 + (DISP).
(a) 60 H (b) 30 H
(c) 06 H (d) 03 H INX H 

Solution: (a) INX H  (HL) ← (HL) + 3
XRA A ;(A) ← 00 H (CY = 0). 
INX H 
MVI B, 04 H ;(B) ← 04 H.
(PUSH H; ((SP)) ← (HL).
MVI A, 03 H ;(A ) ← 03 H = 0000 0011.
RET  ;The subroutine will return to LP + DISP + 3.
Hence, the correct option is (b).
RAR
; (A) = 0000 0001 ,CH = 1 1st exceution 4. When a ‘CALL addr’ instruction is executed, the
DCR of loop CPU carries out the following sequential operations
; (B) = 03 H internally. [2010]
JNZ
Note:
(R) means content of register R.
In 2nd execution, (A) = 1000 0000, CY = 1. (R) means content of memory locating pointed by R.
(B) = 02 H. PC means Program Counter.
3rd execution (A) = 1100 0000, CY = 0. SP means Stack Pointer.
(B) = 01 H. (a) (SP) incremented. (b) (PC) ← (Addr).
4th execution (A) = 0110 0000, CY = 0. (PC) ← (Addr). ((SP)) ← (PC).
(B) = 00 H ← JNZ Condition fails here. ((SP)) ← (PC). (SP) incremented.
(c) (PC) ← (Addr). (d) ((SP)) ← (PC).
Hence, value in (A) = 60 H. (SP) incremented. (SP) incremented.
Hence, the correct option is (a). ((SP)) ← (PC). (PC) ← (Addr).

Chapter 09.indd 28 11/9/2015 6:43:54 PM


Chapter 9  Microprocessors  |  7.29

Solution: (d) respectively. When the following sequence of in-


When CALL instruction is processed, address of structions are executed,
next instruction to CALL instruction is stored in 2100 H:        DAD SP
stack, and then branching to address/label men- 2101 H:        PCHL
tioned after CALL is executed,
The contents of (SP) and (PC) at the end of execu-
i.e., ((SP)) ← (PC). tion will be
SP is incremented. (a) (PC) = 2102 H, (SP) = 2700 H
(PC) ← (Addr). (b) (PC) = 2700 H, (SP) = 2700 H
(c) (PC) = 2800 H, (SP) = 26FEH
Hence, the correct option is (d).
(d) (PC) = 2A02 H, (SP) = 2702 H
5. In 8085 microprocessor, the contents of the Ac­­
Solution: (b)
cumulator, after the following instructions are
executed will become [2009] (SP) = 2700 H.
XRA A (PC) = 2100 H.
MVI B F0 H (HL) = 0000 H.
SUB B ( HL ) ( SP )
2100 : DAD SP ;(HL) ← 0000 + 2700 = 2700 H.
(a) 01 H (b) OF
2101: PCHL ;PC ← 2700.
(c) F0 H (d) 10 H
(PC) = 2700 H.
Solution: (d)
XRA A ;(A) ← 00 H. (SP) = 2700 H.
MVI B, F0 H ;(B) ← F0 H. Hence, the correct option is (b).
SUB B ;(A) = (A) − (B) 7. In an 8085 A microprocessor based system, it is
desired to increment the contents of memory loca-
= 00 H − F0 H
tion whose address is available in (D, E) register
= (A) + 2st complement of B pair and store the result in same location. The
= 00000000 + 2 s comp (11110000) sequence of instructions is [2007]
= 0000 0000 + (00001111 + 1) (a) XCHG, INR M (b) XCHG, INX H
(c) INX D, XCHG (d) INR M, XCHG
= 0000 0000 + (00010000)
Solution: (a)
= 00010000 = 10 H. D, E register stores the memory location.
Hence, the correct option is (d). XCHG instruction will shift contents of DE into
6. The contents (in Hexadecimal) of some of the HL so that HL can point to the memory location.
memory locations in an 8085 A based system are Then INR M will increment the contents.
given below [2008] Hence, the correct option is (a).
Adress Contents 8. Which one of the following statements regarding
the INT (interrupt) and the BRQ (but request) pins
... ...
in a CPU is true? [2007]
26FE 00 (a) The BRQ pin is sampled after every instruc-
26FF 01 tion cycle, but the INT is sampled after every
machine cycle
2700 02
(b) Both INT and BRQ are sampled after every
2701 03 machine cycle
2702 04 (c) The INT pin is sampled after every instruc-
tion cycle, but the BRQ is sampled after every
... ...
machine cycle

The contents of stack pointer (SP), program coun- (d) Both INT and BRQ are sampled after every
ter (PC) and (HL) are 2700 H, 2100 H and 0000 H instruction cycle

Chapter 09.indd 29 11/9/2015 6:43:55 PM


7.30 |  Digital Electronics and Microprocessors

Solution: (a) MVI L, 255D


BRQ introduction is sampled after every instruc- LOOP: DCR L
tion INT is sampled after every machine cycle. JNZ LOOP
Hence, the correct option is (a). DCR H
JNZ LOOP
Common Data for Questions 9 and 10:
The associated figure shows the two types of rotate  How many times DCR L instruction will be
right instruction Rl, R2 available in a microproces- executed?
sor where Register is an 8-bit register and C is the (a) 255 (b) 510
carry bit. The rotate left instructions L1 and L2 are (c) 65025 (d) 65279
similar except that C now links the most significant Solution: (d)
bit of Register instead of the least significant one. The inner loop will be executed for 255 times for
value of H = 255 (   L = 254 to 0).

Register C The outer loop is executed for 255 times.
R 1:
When H = 254, DCR L is executed 256 times
(   L = 255 to 0).

Register C
R 2: When H = 253, DCR L is executed 256 times
(   L = 256 to 0).

9. Suppose Register contains the 2’s complement When H = 1, DCR L is executed 256 times.
number 11010110. If this number is divided by 2 When H = 0, condition fails, and DCR L is not
the answer should be [2007] executed then.
(a) 01101011 (b) 10010101 ∴ Number of times DCR L is executed = 255
(c) 11101001 (d) 11101011 + 254 × 256 = 65279.
Solution: (d) Hence, the correct option is (d).
Number contained in register is (in 2s complement
12. The 8085 assembly language instruction that stores
form) = 11010110.
the content of H and L registers into the memory lo-
When number is divided by 2, it shifts right by one cations 2050 H and 2051 H, respectively, is [2005]
location and the sign bit is retained (Arithmetic (a) SPHL 2050 H (b) SPHL 2051 H
right shift), (c) SHLD 2050 H (d) STAX 2050 H
i.e., number becomes 11101011. Solution: (c)
Hence, the correct option is (d). SHLD 2050 H will store the content of memory
10. Such a division can be correctly performed by the location 2050 in H register and 2051 in L register.
following set of operations [2007] Hence, the correct option is (c).
(a) L2, R2, Rl (b) L2, Rl, R2 13. The following program is written for an 8085
(c) R2, Ll, Rl (d) Rl, L2, R2 microprocessor to add two bytes located at memory
Solution: (a) addresses 1 FFE and 1 FFF [2003]
CY LXI H,1FFE
Reg: 1101 0110 0 MOV B, M
INR L
L2 : 1010 1101 1
MOV A, M
R2 : 1101 0110 1
ADD B
R1 : 1110 1011 0 INR L
MOV M, A
Hence, the correct option is (a).
XRA A
1 1. A software delay subroutine is written as given
On completion of the execution of the program,
below: [2006]
the result of addition is found.
DELAY: MVI H, 255D (a) In the register A

Chapter 09.indd 30 11/9/2015 6:43:55 PM


Chapter 9  Microprocessors  |  7.31

(b) At the memory address 1000 RAM


(c) At the memory address 1F00 A0 − A10
CS
(d) At the memory address 2000
A11
Solution: (c)
LXI H, IFFE ;[HL] ← 1FFE
MOV B, M ;[B] = [1FFE]
A15
INR L ;[L] ← FF
MOV A, M ;[A] = [1FFF]
ADD B ;[A] ← [A] + [B] Solution:
Both the inputs of NAND gate = 1 so that its output
INR L ;[L] = 00
= c and cs = 0 so that chip can be selected.
MOV M, A ;[1F00] ← [A]
∴  A15 to A11 = 1.
XRA A ;[A] = 00
The answer is F800 H to FFFFH.
Result of addition is found at address 1F00 H. 16. The stack pointer of a microprocessor is at A001 H.
Hence, the correct option is (c). At the end of execution of following instructions,
1 4. An Intel 8085 processor is executing the program the value of stack pointer is _____. [1994]
given below. [2001]  PUSH PSW
MVI A, 10 H  XTHL
MV1B, 10 H  PUSH D
BACK:    NOP   JMP FC70 H
ADDB
RLC Solution:
JNC BACK Initially, [SP] = A001 H.
There are 2 PUSH statements, and therefore, SP is
HLT The number of times that the operation NOP
decremented by 4.
will be executed is equal to
(a) 1 (b) 2 A001 H - 4 = 9FFD H.
(c) 3 (d) 4 The answer is 9FFD.
Solution: (c)
No. of
execution Three-marks Questions
of loop A B
0th 00010000 00010000 1. In a 8085 microprocessor, the following instruc-
ADD B: 0010 0000 (CY = 0) tions may result in change of accumulator contents
1st and change in status flags. Choose the correct
RLC: 0010 0000 (CY = 0)
ADD B: 0101 0000 (CY = 0) match for each instruction. [1996]
2nd      Contents of 
RLC: 1010 0000 (CY = 0)
ADD B: 1011 0000 (CY = 0)  ACC Cy flag   AC flag
3rd
RLC: 0110 0001 (CY = 0) (a) ANAr  (P)  Unchanged may by  SET  Unchanged
JNC fails after 3rd execution when CY = 1. (b) XRAr  (Q)  Unchanged SET   SET
(c) CMPr  (R)  Unchanged SET   RESET
∴  Number of executions of NOP instruction = 3.
       (S) May change      RESET RESET
Hence, the correct option is (c).        (T) May change      RESET SET
1 5. The range of addresses for which the memory chip
shown in figure, will be selected is … to … [1997]

Chapter 09.indd 31 11/9/2015 6:43:55 PM


7.32 |  Digital Electronics and Microprocessors

Solution: (c) → (P) (c) CMPr - Compare with ACC.


(a) ANAr - Logical AND with Accumulator (ACC). - Content of ACC is unchanged.
- Content of ACC may change. - CY may be set depending on whether
- CY is reset. (R) > (A).
- AC is set. Hence, the correct option is (c) → (P).
Answer is (a) → (T).
(b) XRA r - EXOR with ACC.
- Content of ACC may change.
- CY and AC are reset.
Answer is (b) → (S).

Chapter 09.indd 32 11/9/2015 6:43:55 PM


Unit 8
Analog Electronics

Chapter 1:  Diode Circuit 8.3


Chapter 2:  BJT and FET Biasing 8.13
Chapter 3:  Small Signal Modeling and Analysis 8.23
Chapter 4:  Frequency Response 8.28
Chapter 5:  Operational Amplifiers and Applications 8.29
Chapter 6:  Feedback Amplifiers and Oscillators 8.48
Chapter 7:  Function Generator and 555 Timer 8.54

Chapter 01.indd 1 11/9/2015 5:55:47 PM


Chapter 01.indd 2
Exam Analysis
Exam Year 91 92 93 94 95 96 97 98 99 00 01 02 03 04 05 06 07 08 09 10 11 12 13 14
1 Mark Questions 1 3 1 4 1 4 1 1 3 2 2 2 2 4 2 2 2 1 2 3 2 1 2 5
2 Marks Questions 2 2 – – – – – 2 – 2 3 2 5 5 5 3 2 4 3 1 2 2 4 4
5 Marks Questions 2 3 – 1 – – 1 1 2 1 3 2 – – – – – – – – – – – –
Total Marks 5 8 1 5 1 4 2 4 5 5 8 6 7 9 7 5 4 5 5 4 4 3 6 9
Diode Circuit 1 1 – – – – – – 2 1 – 3 1 2 1 2 – 2 1 1 1 1 2 3
BJT and FET Biasing 1 – – 1 – 1 – 2 1 1 1 1 3 2 3 1 1 1 – 1 1 1 – 2
Small Signal Modeling and Analysis 1 1 – – – 1 – – 1 1 2 – – 1 – – – – – – – – – –
Frequency Response – 1 – – – – – – – – – – – – 1 – – – – – – – – –
Operational Amplifiers and Applications 2 4 – 2 1 2 1 2 1 2 4 1 2 3 2 2 2 2 3 1 2 – 3 2
Feedback Amplifiers and Oscillators – 1 1 2 – – 1 – – – 1 1 – 1 – – – – – 1 – 1 1 2
Function Generator and 555 Timer – – – – – – – – – – – – 1 – – – 1 – 1 – – – – –

11/9/2015 5:55:48 PM
Chapter 1
Diode Circuit
Solution:
One-mark Questions We know that,
1. The figure shows the circuit of a rectifier fed from 1 1
 Pz = Vz I z ⇒ Iz = =
a 230 V (rms), 50 Hz sinusoidal voltage source. 5 × 4 20
If we want to replace the current source with a
resistor so that the rms value of the current sup- (Vo − Vz )
  = I z + I L   (Applying KCL)
plied by the voltage source remains unchanged, the Rs
value of resistance (in ohms) is (Assume
diodes to be ideal). [2014-S1] 20 − 5 1
⇒ =   (Neglecting I L  0)
Rs 20

∼ 10 A ⇒ Rs = 15 × 20 ⇒ Rs = 300 Ω.
230 V,
50 Hz 3. The i-v characteristics of the diode in the circuit
given below are [2012]
Solution:
Vrms = 230 V  and  I rms = 10 A
V  v − 0.7
  R = rms = 230 ⇒ R = 23 Ω.  A, v ≥ 0.7 V
I rms 10 i =  500
0 A, v < 0.7 V
2. The sinusoidal ac source in the figure has an rms
20
value of V. Considering all possible values of
2 1 kΩ
RL , the minimum value of Rs in Ω to avoid burn-
i
out of the Zener diode is . [2014-S2] +
+
Rs v
10 V
− −

20 R1
V ∼ 5V
√2
1 The current in the circuit is
W
4
(a) 10 mA (b) 9.3 mA
(c)
6.67 mA (d)
6.2 mA

Chapter 01.indd 3 11/9/2015 5:55:49 PM


8.4 | Analog Electronics

Solution: (d) 15
1 kΩ 10

i 5

Vs (Volts)
+
+ v 0

10 V −
−5

−10

Applying KVL in the loop, 10 = 103 i + V −15


0 100 200 300 400
 V − 0.7  Time (ms)
⇒ 10 = 103   +V .
 500 
1.5
⇒ 10 = 2 V − 1.4 + V .
1
⇒ 3 V = 11.4.

Current (mA)
0.5
⇒ V = 3.8.
0
∴ i = (10 − 3.8) × 10 −3 = 6.2 mA.
−0.5
Hence, the correct option is (d).
−1
4. Assuming that the diodes are ideal in the given
circuit, the voltage Vo is [2010] −1.5
0 100 200 300 400
Time (ms)
D1 10 kΩ D 2
The element connected between a and b could be
10 kΩ
15 V (a)
a b
10 V Vo

10 kΩ
(b)
a b

(c)
a b
(a)
4 V (b)
5V
(c) 7.5 V (d) 12.12 V (d)
Solution: (b)
a b
Analyzing the circuit one can observe that D1 is
ON and D2 is OFF.
Solution: (a)
10 In positive half cycle:
∴ Vo = 10 × = 5 V.
(10 + 10) Diode D → Forward Biased.
Hence, the correct option is (b).
∴ Vo′ = Vi ′.
5. The following circuit has a voltage source Vs as
shown in the graph. The current through the circuit In negative half cycle:
is also shown [2009] Diode D → Reverse Biased.
a b ∴ Vo′ = 0.
Hence, the correct option is (a).
+
Vs

R 10 kΩ 6. The equivalent circuits of a diode, during forward
and reverse biased conditions are shown in figure.
 [2008]

Chapter 01.indd 4 11/9/2015 5:55:50 PM


Chapter 1  Diode Circuit  |  8.5

(i) 0⋅7 V Rf Analyzing the circuit,


+ − + −
10
− + Vp = × 10 sin ωt   (Voltage Divide Rule).
(10 + 10)

(ii) 10 kΩ ⇒ V p = 5sin ωt.

This means that voltage across the diode will be


∼ Vo 10 kΩ less than or equal to zero. So it will always be
10 sin wt
Reverse Biased.
10
5V I ∴ Vo = × 10 sin ωt = 5 sin ωt .
(10 + 10)
If such diodes are used in the clipper circuit of
Hence, the correct option is (a).
figure given above, the output voltage (Vo ) of the
circuit will be 7. What are the states of the three ideal diodes of the
circuit shown in figure? [2006]
(a) 1Ω
+5 V
1Ω
wt
0 p 2p D2
−5 V 10 V 5A
D1 1Ω
D3

(b)
+5.7 V
wt
0 p 2p D1 − ON, D2 − OFF, D3 − OFF
(a)
−10 V
D1 − OFF, D2 − ON, D3 − OFF
(b)
(c) D1 − ON, D2 − OFF, D3 − ON
(c)
+5.7 V

wt D1 − OFF, D2 − ON, D3 − ON
(d)
0 p 2p
−10 V Solution: (a)
Analyzing the circuit, one can observe that D1 and
D2 → Forward Biased and D3 → Reverse Biased.
(d)
10 V But no current flows through D2 because current
wt
get shortest i.e., least resistance path through D1.
0 p 2p
Hence, the correct option is (a).
− 5.7 V
8. Assume that D1 and D2 in figure are ideal diodes
Solution: (a) the value of current I s is [2005]

10 kΩ p
D1 2 kΩ
10 sin wt ∼ V o 10 kΩ 1 mA
(DC) 1
5V D2 2 kΩ

Chapter 01.indd 5 11/9/2015 5:55:51 PM


8.6 | Analog Electronics

(a)
0 mA (b)
0.5 mA (a) 
3.3 V in the positive half cycle and 1.4 V in
(c)
1 mA (d)
2 mA the negative half cycle.
(b) 4 V in the positive half cycle and 5 V in the
Solution: (a)
negative half cycle.
Current will pass through the path offering least
resistance. (c) 
3.3 V in the both positive and negative half
cycle.
 ∴ D1 → Forward Biased and
(d) 4 V in the both positive and negative half
 D2 → Reverse Biased. cycle.
 ⇒ I = 0 mA. Solution: (b)
Hence, the correct option is (a). For positive half cycle 4 mA:
9. The current through the Zener diode in the given
circuit is [2004] 1 kΩ

2.2 kΩ
+ 3.3 V
+ 1 kΩ Vo
10 V
− 0.7 V
VZ = 3.3 V R L 3.5 V
10 V R Z = 100 Ω IZ

− So, Vo = 4 V.

For negative half cycle 2 mA:


(a)
33 mA (b)
3.3 mA
1 kΩ
2 mA (d)
(c) 0 mA
Solution: (c)
Vo = 3.5 V and VZ = 3.3 V. − 1 kΩ Vo
10 V
Zener diode offers dynamic resistance such that +
0C
RZ = 0.1 kΩ .
∴ VZ + RZ I Z = 3.5   (Using KVL)
1
(3.5 − 3.3) Vo = 10 × = 5 V.
IZ = = 2 mA. (1 + 1)
RZ
Hence, the correct option is (b).
Hence, the correct option is (c). 1 1. The forward resistance of the diode shown in
10. The cut-in voltage of both Zener diode Dz and D figure is 5 Ω and the remaining parameters are
shown in figure is 0.7 V, while breakdown voltage same as those of ideal diode. The DC component
of the Zener is 3.3 V and reverse break down of D of the source current is [2002]
is 50 V. The other parameters can be assumed to
D
be the same as those of an ideal diode. The values
of the peak output voltage (Vo ) are [2002]
Vi Vi = Vm sin wt 45 Ω
1 kΩ
w = 314 rad/sec

V Vm
1 kΩ (a) m (b)
∼ Vo 50 π 50π 2
10 sin wt t V 2Vm
w = 314 rad/sec (c) m (d)
100 π 2 50π

Chapter 01.indd 6 11/9/2015 5:55:54 PM


Chapter 1  Diode Circuit  |  8.7

Solution: (a) Solution: (b)


For the output to be clipped (Ideal/diodes) Vo is
Vm V Vav V
Vaverage = , I av = av = = av −1 V and −2 V.
π R (5 + 45) 50
Vm  10  Vi
∴ I av = . Vo = Vi  = .
50π  10 + 10  2

Hence, the correct option is (a). Vi = 2Vo .
1 2. The mobility of an electron in a conductor is Vi = −2 V to − 4 V.
expressed in terms of [1999]
(Or)
(a) cm /V − sec (b)
2
cm/V − sec
When Vi < −2 V , D1 − OFF, D2 − ON.
(c) cm 2 /sec
cm 2 /V (d)
Solution: (a)
Mobility 10 kΩ
+
Drift Velocity cm/sec
(µ ) = = = cm 2 /V − sec.
Electric Field V/cm
∼ Vi 10 kΩ Vo
Hence, the correct option is (a). 2V
13. As temperature is increased, the voltage across a
diode carrying a constant current [1999] −
(a) increases
(b) decreases But to get Vo = −2 V, the required value of input
(c) remains constant
voltage is − 4 V.
(d) may increase or decrease depending upon the
doping levels in the junction ∴  When Vi < − 4 V, Vo = −2 V.
Solution: (b)
dv ∆ When Vi > −1 V, D1 − ON, D2 − OFF.
We know that = −2.5 mV°/ °C
dt
\  As temperature ↑, diode voltage will decrease. 10 kΩ
Hence, the correct option is (b). +

Two-marks Questions ∼ Vi
1V
10 kΩ Vo

1. Assuming the diodes to be ideal in the figure, for −


the output to be clipped, the input voltage Vi must
be outside the range [2014-S2]
But to get Vo = −1 V, the required value of input
10 kΩ
voltage is −1 V.

∴  When V > −2 V, V = −1 V.
i o
10 kΩ Vo
Vi ∼ When −2 V ≤ Vi ≤ − 4 V, Vo = Vi .
1V 2V
Hence, the correct option is (b).
2. A voltage 1000 sin ωt , V is applied YZ. Assuming
(a) −2 V to −4 V
−1 V to −2 V (b) ideal diodes, the voltages measured across WX in
V is [2013]
+1 V to −2 V (d)
(c) +2 V to −4 V

Chapter 01.indd 7 11/9/2015 5:55:57 PM


8.8 | Analog Electronics

Solution: (b)
Is I Load
1 kΩ
100 Ω
W Y X
10 V Vz = 5 V RL
Z

+ 1 kΩ −
10 − 5
(sin ωt + |sin ωt |) Is = = 50 mA.
(a) sin wt       (b)  100
(c) (sin ωt − |sin ωt |)   (d)  0 for all 2t I L = 50 mA − 10 mA = 40 mA.
2
5V
Solution: (d) RL = = 125 Ω .
40 mA
For positive half cycle:
Pmin = 50 mA × 5 V = 250 mW.
1 kΩ Hence, the correct option is (b).
W Y X
Z 4. A clipper circuit is shown below. [2011]

1 kΩ 1 kΩ
D
∼ Vz = 10 V Vo
5V
Vwx = 0.
For negative half cycle: Assuming forward voltage drops of the diodes to
be 0.7 V, the input-output transfer characteristics
1 kΩ of the circuit is
W Y X (a) Vo
Z
D

1 kΩ 4.3 V

Short Circuit Condition Vi


4.3 V
Vwx = 0. (b) Vo

Hence, the correct option is (d). 10 V


3. In the circuit shown below, the knee current of 4.3 V
the ideal Zener diode is 10 mA. To maintain 5 V
across RL, the minimum value of RL in Ω and the Vi
4.3 V 10 V
minimum power rating of the Zener diode in mW (c)
respectively are [2013] 5.7 V

100 Ω
−0.7 V
ILoad Vi
0.7 V 5.7 V
10 V
(d) Vo
RL
Vz = 5 V
10 V

−5.7 V
(a) 125 and 125 (b) 125 and 250 Vi
10 V
−5.7 V
(c) 250 and 125 (d) 250 and 250

Chapter 01.indd 8 11/9/2015 5:55:58 PM


Chapter 1  Diode Circuit  |  8.9

Solution: (c) ∴ VC2 = −10 V.


For positive voltage the waveform clips at
Hence, the correct option is (d).
+(5 V + 0.7 V ) = 5.7 V.
6. Assuming the diodes D1 and D2 of the circuit
For negative voltage at −0.7 V, the Zener diode
shown in the figure to be ideal ones, the transfer
conducts and clips out.
characteristics of the circuit coil be [2006]
Vo

5.7 D1

0.7 Vi Vo RL = ∞
Vi
5.7
0.7
10 V 5V

Hence, the correct option is (c). (a)  Vo (b)  Vo


5. In the voltage doubler circuit shown in figure, the
switch ‘S’ is closed at t = 0. Assuming diodes D1 10 5
and D2 to be ideal, load resistance to be infinite
and initial capacitor voltages to be zero, the steady
state voltage across capacitors C1 and C2 .  [2008] Vi Vi
10 5

t = 0 VC1 D2
(c)  Vo (d)  Vo
+ −
S + 10
C1 10
∼ D1
C2 VC2 R load
5 sin wt −
5

Vi Vi
=
(a) =
VC1 10 V, VC 2 5 V 5 10 10

VC1 = 10 V, VC 2 = −5 V
(b) Solution: (a)
= VC1 5=
(c) V, VC 2 10 V When, Vi < 10 V  ⇒  D1, D2 are OFF.

VC1 = 5 V, VC 2 = −10 V
(d)  ∴ Vo = 10 V.
Solution: (d)  Vi < 10 V  ⇒  D1 is ON and D2 is OFF.
VC 1
t=0 + − D2  ∴ Vo = Vin .
S
C1
5 sin wt ∼ + Hence, the correct option is (a).
D1 VC2 RL
C2
IC 1 − 7. Assuming that the diodes are ideal in figure the
IC2
current in the diode D1 is [2004]
C1 Charges through D1 upto Vmax(5 V) with shown

polarities. 1 kΩ 1 kΩ

∴ VC1 = 5 V. D2
5V D1
Now diode D1 will be Reverse Biased and D2 will 8V
be Forward Biased ⇒ C2 will charge in reverse di-
rection through D2 upto 2Vmax.

Chapter 01.indd 9 11/9/2015 5:56:00 PM


8.10 | Analog Electronics

(a)
8 mA (b)
5 mA −3
∴ Vo = × 10 − 4 = −7 V.
0 mA (d)
(c) −3 mA 10
Solution: (c) Hence, the correct option is (d).
Analyzing the circuit, if D1 is ON, Kirchhoff’s Law 9. In the single phase diode bridge rectifier shown in
is not satisfied. figure, the load resistor is R = 50 Ω.
∴   D1 is OFF and D2 is ON. The source voltage is V = 200 sin ωt, where ω =
∴ I = 0 mA. 2π × 50 rad/sec. The powder dissipated in the load
resistor R is [2002]
Hence, the correct option is (c).
8. A voltage signal 10 sin ωt is applied to the circuit
with ideal diodes as shown in figure. The maxi-
V ∼ A
mum and minimum values of the output waveform B
R
of the circuit are respectively [2003]
10 kΩ
+ D

D1 D2 3200 W 400 W
(a) (b)
∼ Vi 4V Vo π π
4V
400 W (d)
(c) 800 W
10 kΩ
Solution: (b)

2
Vm Vrms V 2 ( 200) 2
+10 V and −10 V
(a) Vrms = ; P= = m = = 400 W.
2 R 2 R 2 × 50
+ 4 V and − 4 V
(b)
+7 V and − 4 V
(c) Hence, the correct option is (b).
+ 4 V and −7 V
(d) 1 0. A diode whose terminal characteristics are related
as iD = I S ev vr , where IS is the reverse satura-
Solution: (d)
tion current and VT is thermal voltage (= 25 mV)
10 kΩ
is biased at iD = 2 mA. Its dynamic resistance is
. [2000]
D1
Vin ± I D2 (a) 25 Ω
(b) 12.5 Ω
4V (c) 50 (d)
Ω 100 Ω
4V Solution: (b)
10 kΩ
We know that
VT 25 mV
rD = = = 12.5 Ω.
For +ve cycle, when Vin < − 4 V(Vin > 4 V ) D2 is ID 2 mA
ON, D1 is OFF and Vout = 4 V.   When Vin < 4 V,
Hence, the correct option is (b).
then Vout = Vin , because D2 also becomes OFF.
1 1. Figure shows as electronic voltage regulator the
For − ve cycle, when Vin < − 4 V Zener diode may be assumed to require a mini-
D1 → ON and D2 → OFF mum current of 25 mA for satisfactory operations.
The value of R required for satisfactory voltage
Vin + 4 regulation of the circuit is [1991]
∴ I= , but Vin = − ve.
20 kΩ R

Vin + 4 −3
∴ I =−
 =   (As for minimum output
20 kΩ 10 10 V 100 Ω
20 V
Vin = −10 V).

Chapter 01.indd 10 11/9/2015 5:56:03 PM


Chapter 1  Diode Circuit  |  8.11

Solution: Also 20 − 10 = 125 × 10 −3 R.


Current in the 100 Ω resistance
(10 × 1000)
10 V  ⇒ R =
I100 = = 100 mA; I 2 = 25 mA. 125
100 Ω
  = 80 Ω (Voltage across the Resistance R).
Current in R = (100 + 25) mA = 125 mA.

Chapter 01.indd 11 11/9/2015 5:56:03 PM


8.12 | Analog electronics

Apply KVL to loop (2)


Five-marks Questions sin t − i2 − (i1 + i2 ) = 0
1. In the circuit shown in figure calculate and sketch i1 + 2i2 = sin t (2)
the waveform of current i over one period of the i/p By solving (1) and (2) we get
voltages. Assume the diodes to be ideal. [1992] 2i1 + i2 = cos t
2i1 + 4i2 = 2 sin t
D1 1Ω 1Ω D2
− − −
i
+ + − 3i2 = cos t − 2 sin t
∼ cos t 1Ω sin t ∼
− −
1
0.8
i − i1 + i 2 0.6
0.4
Solution: 0.2
Assume that the diodes are ON state and 0
p p 3p p 5p 6p 7p 8p
−0.2
4 2 4 4 4 4 4
1 1 −0.4
−0.6
i2 −0.8
∼ cos t i1 1 ∼ sin t −1

 2 sin t − cos t  and i =  2 cos t − sin t  .


Apply KVL to loop (1) i2 =   1  
 3   3 
cos t − i1 − (i1 + i2 ) = 0
1
2i1 + i2 = cos t (1)
i = i1 + i2 = [cos t + sin t ].
3

Chapter 01.indd 12 11/3/2015 3:28:24 PM


Chapter 2
BJT and FET Biasing
Z i Vi hie
One-mark Questions Vb = Vi
Rs

Rs
.

1. The magnitude of the mid-band voltage gain of the Vo


Av = .
circuit shown in figure is (assuming hfe of the tran- Vi
sistor to be 100). [2014-S1] −106 I b −106 I b
Av = = = −100.
+V b R R
Vb s I b hie s
hie hie
10 kΩ
C ∴ Av = 100.
Vo
C 10 kΩ
hfe = 100 Hence, the correct option is (d).
2. The transistor in the given circuit should always be
Vi ∼
=
in active region. Take VCE ( sat ) 0=
.2 V, VBE 0.7 V.
1 kΩ C
The maximum value of RC in W which can be used,
is .
[2014-S2]
(a) 1 (b) 10
(c) 20 (d) 100
Solution: (d) RC

Ic +
RS = 2 kΩ 5V
b = 100
+
Ib
Ib +
Ic 5V
∼ Vi Vb hic hfe
Vo

Zi
− Solution:
 5 − 0.7 
Ib =  3 
.
 2 × 10 
Vo = −104 I C = −104 h fe I b = 10−6 I b .
We can say,
 Z  IC
Vb = Vi  . I B = I B min and IB < .
 Z i + Rs  β min

Chapter 02.indd 13 11/12/2015 10:43:27 AM


8.14 | Analog Electronics

4.3 4.8 4.8 × 2 × 103 Therefore the power dissipated in the transistor is
So, < ∴ RC < .
2 × 10 3
RC × 100 4.3 × 100 PD = VCE × I C = 4.1× 0.5893 = 2.416 W.
RC < 22.32 Ω.
Hence, the correct option is (c).
Here, we find out that maximum value of RC which 4. The common emitter forward current gain of the
can be used is 22 to 23 W. transistor shown is βF = 100. The transistor is oper-
3. The three terminal linear voltage regulator is con- ating in
nected to a 10 W load resistor as shown in the figure. [2007]
If Vin is 10 V, what is the power dissipated in the
transistor +10 V
[2007]

+10 V 1 kΩ

1 kΩ
V in RL = 10 Ω
6.6 V
Zener diode
0 270 kΩ
1 kΩ

(a) 0.6 W (b) 4.2 W


(c) 2.4 W (d) 5.4 W
(a) saturation region
Solution: (c)
(b) cut-off region
IC IE (c) reverse active region
E
+ (d) forward active region

IB 0.7 V Solution:
IL
+ + KVL is used from 10 V to ground through BE ter-
Vm II 1 kΩ
10 Ω
mination.
6.6 V
103 (1 + β )1µ + 270 × 103 I n = 9.3.

9.3
IB = = 25.06 µA.
So, we can say, (270 + 101) × 103
VE = 6.6 − 0.7 = 5.9 V. I C = 2.506 mA.
VE I E = 2.5318 mA.
IL = = 0.59 A.
10
Apply KVL from 10 V to ground,
Using KVL, Vm − VCE − VE = 0.
VCE = 10 − 103 ( I C + I E )
VCE = 10 − 5.9 = 4.1 V.
= 10 − 103 × 5.0378 × 10−3 = 4.962 V.
V −V 6.6 − 5.9
I1 = X 3 E = = 0.7 mA.
10 103 Hence, the correct option is (d).
I L = I E + I1 5. Assume that n-channel MOSFET shown in figure
I E = I L − I1 = 0.5893 A. is ideal and its threshold voltage is 1 V, the voltage
Vab between nodes a and b is
Assume that I E ≈ I C = 0.5893. [2005]

Chapter 02.indd 14 11/12/2015 10:43:30 AM


Chapter 2  BJT and FET Biasing  |  8.15

1 kΩ 1 kΩ Solution: (c)
a
Both transistor are perfectly matched, hence
D
10 V G 2 kΩ Vab VBE2 = VBE1 .
S
2V
b I C1 VBE − VBE2 
Therefore, = exp  1  =1
(a) 5 V (b) 2 V I C2  VT 
(c) 1 V (d) 0 V Also β is same
Solution: (d)
MOSFET is n-channel gate through source is so IR 3V
connected that MOSFET will be in enhance mode
and so conductivity of the channel will be in- 1 kΩ I = IC 2
creased very much and efficiency β terminal act as
2IB
short circuited. So Vab = 0 V. b = 1000
b = 1000
Hence, the correct option is (d).
6. A bipolar junction transistor (BJT) is used as a
power control switch by biasing it in the cutoff −5 V
region (OFF state) or in the saturation region (ON
state). In the ON state, for the BJT [2004] + 5 − 0.7
IR = = 4.3 mA.
(a) both Base-Emitter and Base-Collector junc- 1 kΩ
tion are reverse biased. Writing KCL at node B,
(b) the B-E junction is R.B. and Base-collector
junction is F.B. I C + 2 I B − I R = 0.
(c) the B-E junction is F.B. and Base-collector IC = β Ib .
junction is R.B.
 β 
(d) both B-E and B-C junctions are F.B. ∴ IC =   I R = I R (Because β is very large).
 β +2
Solution: (d)
There would be JE JC .
Hence I=
C1 I=
C2 I R = 4.3 mA.
R.B. → Cut OFF (OFF).
F.B. → Inverse active → Attenuator. Hence, the correct option is (c).
F.B.-R.B. → Active → Amplifier.
8. In the circuit of figure shown, assume that the tran-
F.B.-F.B. → Saturation → ON.
sistor has h fe 99
= = and VBE 0.7 V. The value
Hence, the correct option is (d). of collector current IC of the transistor is approxi-
7. Two perfectly matched Si transistors are connected mately [2003]
as shown in figure. The value of the current I is
3.3 kΩ
[2004]
Ic
+3 V 33 kΩ
12 V
1 kΩ I
4V 3.3 kΩ

b = 1000
+ b = 1000
0.7 V  3.3   3.3 
 3.3  mA  
(a)     (b)   (3.3 + 0.33)  mA
−5 V  

(a) 0 mA (b) 2.3 mA  3.3   3.3 


(c) mA
(c) 4.3 mA (d) 7.3 mA  33  mA     (d)  (33 + 33) 

Chapter 02.indd 15 11/12/2015 10:43:32 AM


8.16 | Analog Electronics

Solution: (b) 11. The depletion region (or) space charge region (or)
Using KVL in Base-emitter loop transition region in a semiconductor p-n junction
diode has [1996]
3.3 kΩ
(a) electrons and holes.
33 kΩ (b) positive ions and electron.
12 V (c) positive and negative ions.
(d) negative ions and holes.
3.3 kΩ
4V Solution: (c)
As there is recombining of e − and hole on both the
sides of the junction, diffusion of majority carriers
4 − 33 × I B − 0.7 − 3.3 × 100 I B = 0( I C = β I B ). takes place due to concentration gradient on P or
4 − 0.7 N side. Consequently, depletion region is formed.
∴ IB = .
(33 + 330) So, we can conclude as there are no mobile charge
3.3 × 100 3.3 carries (e − and hole), there are only immobile ions
∴ IC = = . (positive and negative) present.
33 + 330 (0.33 + 3.3)
Hence, the correct option is (c).
 3.3  12. In the transistor circuit shown in figure, collector
So, IC of the transistor is  + 0.33 mA.
 3.3  to ground voltage is +20 V. Which of the following
is the probable cause of error? [1994]
Hence, the correct option is (b).
+20 V
9. The enhancement type n-channel MOSFET is rep-
resented by the symbol [1999] 1 kΩ
47 kΩ
(a) (b) +10 V

(c) (d)
(a) Collector Emitter terminals shorted
(b) Emitter to ground connection open
(c) 10 kW Resistor open
Solution: (a) (d) Collector base terminals shorted
The enhancement type n-channel MOSFET is rep- Solution: (b)
resented by symbol.  20 − 10 kΩ × I C = 20
⇒ I C = 0 which gives I= E I=
C I B = 0.
That is only possible when emitter connection is
open. In other cases there will be base current.
Hence, the correct option is (a). Hence, the correct option is (b).
13. Figure shown below shows a common emitter
10. One of the application of current motor is [1998]
amplifier. The quiescent collector voltage of the
(a) output current limiting.
circuit is approximately. [1991]
(b) obtaining a very high current gain.
(c) current feedback. 20 V
(d) temperature Stabilized biasing. 10 kΩ
10 kΩ Vo
Solution: (d)
b = 100
Current mirror has several applications. One of the
applications of current mirror is temperature stabi- 5 kΩ 10 kΩ
lized biasing.
Hence, the correct option is (d).

Chapter 02.indd 16 11/12/2015 10:43:34 AM


Chapter 2  BJT and FET Biasing  |  8.17

20 (a) Av ≈ 100
Av ≈ 200 (b)
(a) V (b)
10 V
3
(c) Av ≈ 20 (d)
Av ≈ 10
(c)
14 V (d)
20 V
Solution (b)
Solution: (c)
13.7 V
20 V

12 kΩ
10 kΩ
R TH Vc
C
Vo
V TH 100 kΩ
10 kΩ C

10 kΩ
b = 100

  5
VTH = × 20 V = 6.67.
15
10 × 5
RTH = = 3.33 Ω. Equivalent AC model will be taking hie = 1 kΩ
10 + 5
h fe = β = 100.
VTH − I b RTH − 0.67 − ( β + 1) I b × 10 kΩ = 0.
6 = I b (3.33 + 101× 10) kΩ. 10 kΩ ib
i
I b = 5.921 µA.

12 kΩ
Vi ∼ 100 kΩ
V in hie hfe ib
I c = β I b = 0.5921 mA. 1 − Av
Vc = 20 − 10 × I c
100 kΩ ≅ 100 kΩ as A.I ↑
= 14.078 V ≈ 14 V.
Hence, the correct option is (c). Vo −h fe ib × (100 || 12) kΩ
AV = = .
Vin hie × ib
−100 × 10.71 kΩ
Two-marks Questions AV = = −1071.42.
1 kΩ

1. The voltage gain Av of the circuit shown in the fig-  


ure below, is [2012]  
4  hie × ib 
Vi = hie ib + 10 i = 10 ib + 10 ib +
4 3
  100 kΩ  
13.7 V   
  1 − AV  
12 kΩ Vi = 103 ib + ib [104 + (1072.42) × 100]
C Vo −h fe ib × 10.7142 × 103
Vo AVS = = .
100 kΩ V ib [103 + 104 + 102 × 1072.42]
C
−100 × 10.7142 × 103
10 kΩ
b = 100 AVS = .
Vi ∼ (103 + 104 + 102 × 1072.42)
AVS = − 9.06.
AVS = 9.06  10.

Chapter 02.indd 17 11/12/2015 10:43:37 AM


8.18 | Analog Electronics

+10 V
100 kΩ
≅ 100 kΩ as AV ↑
 1 
1 − 
 AV 
10 kΩ 50 kΩ
Hence, the correct option is (d).
2. The transistor is used in the circuit shown below
has a β of 30 and ICBO is negligible.
Vo
100 Ω
15 kΩ 2.2 kΩ

1 kΩ
(a) 4.65 V (b) 5 V
D (c) 6.3 V (d) 7.32 V
Vz = 5 V
Solution: (a)
Using KVL  ⇒
−12 V
10 = 10 kΩ I b + 0.7 + (100)( I b )100.
VBE = 0.75 V 9.3
Ib = = 0.465 mA.
VCE (sat ) = 0.2 V 20 kΩ
Vo = 100 × 0.465 × 100 = 4.65 V.
If the forward voltage drop of diode is 0.7 V. Then
Hence, the correct option is (a).
the current through collector will be
4. Two perfectly matched silicon transistors are con-
 [2011]
nected as shown in the figure. Assuming the b of
(a) 168 mA (b) 108 mA the transistors to be very high and forward voltage
(c) 20.54 mA (d) 5.36 mA drop to be 0.7 V, the value of current I is[2008]
+5 V
Solution: (d)
1 kΩ D
Suppose that transistor operated in active region I
then, using KVL to base-emitter loop
S − 103 I B − 0.7 − 0.7 + 12 = 0. Q1 Q2
I B = 15.6 mA and I C = 0.468 A.
Now using KVL to collector-emitter loop, −5 V
(a) 0 mA (b) 3.6 mA
0 − 2.2 × 103 I C − VCE + 12 = 0.
(c) 4.3 mA (d) 5.7 mA
VCE = 2200 I C − 12 = 1017.6 V. Solution: (b)
Since, both transistors are perfectly matched,
Since, 0 < VCE < VCC not satisfying this condition, So, VBE1 = VBE 2
Transistor operating in saturation region;
VBE 1 = VBE 2
V + (sat ) = 0.2 V. 1 kΩ IR P
+5 V
Using KVL, 0 − 2.2 I C − 0.2 + 12 = 0.
IC IC
I C = 5.36 mA
Hence, the correct option is (d).
Q1 Q2
3. The transistor circuit shown uses an Si transistor
with VBE = 0.7 V, I C ≈ I E and a DC current gain
−5 V
of 100. The value of Vo is
[2010]

Chapter 02.indd 18 11/12/2015 10:43:40 AM


Chapter 2  BJT and FET Biasing  |  8.19

I c1 V − V  12 − 0.2
= exp  BE1 BE 2  = e0 = 1. But I c( sat ) = as I c ( sat ) > I c ( active ) .
Ic2  VT  2.2 kΩ
Since, b for both are same, therefore Hence, BJT is in active region.
I=
b1 I=
b2 Ib Hence, the correct option is (b).
Applying KVL to loop as shown 6. The common Emitter amplifier shown in the ­figure
is biased using a 1 mA ideal current source. The
0 − 0.7 − 0.7 − (−5)
IR = = 3.6 mA. approximate base current value is .
1 kΩ [2005]
By KCL, VCC = 5 V
I
I R = Ic + 2Ib = Ic + 2 c
1β R C = 1 kΩ
β V out
∴ Ic = × IR
β +2
 ≈ I R ( β is very large) b = 100

Hence, the correct option is (b). +


∼ 1 mA
5. Consider the circuit shown in figure. If the b of the V in
transistor is 30 and ICBO is 20 nA and the input volt-
age is 5 V then the transistor would be operating in
 [2006]
10 mA
0 mA (b)
(a)
+12 V
100 mA (d)
(c) 1000 m A
2.2 kΩ
Solution (b)
Ic
15 kΩ β= .
Vi Ib
Q
1× 10−3
100 kΩ Ib = = 10 µA
100
−12 V Hence, the correct option is (b).
Common Data Questions for 7 and 8:
(a) saturation region (b) active region Assume that the threshold voltage of the n-channel
(c) break down region (d) cut-off region MOSFET shown in figure is 0.75 V. The output
Solution: (b) characteristics of the MOSFET are also shown
Assume BJT is an active Region and we neglect ICBO
VCC = 5 V
−12 × 15 5 × 100
VTH = + = 2.78 V.
115 5 R = 10 kΩ

15 × 100 V out
RTH = (15 || 100) = = 13 kΩ.
115
Ic = β Ib +
V in = 2 mV ∼
2.78 − 0.7
But, I b = = 0.16 mA. 2V
13
 I c = 4.8 mA

Chapter 02.indd 19 11/12/2015 10:43:42 AM


8.20 | Analog Electronics

IDS = 4(mA) is increased to 4 kW, the drain current ID will


become [2003]
4 VGS = 4 V
10 V
3 VGS = 3 V
VGS = 2 V
2 ID RD
VGS = 1 V
1

0
VDS (V)

7. The Transconductance of the MOSFET is [2005]


(a) 0.75 ms (b) 1 ms
(c) 2 ms (d) 10 ms
Solution: (b) (a) 2.8 mA (b) 2.0 mA
(c) 1.4 mA (d) 1.0 mA
∆I DC (4 − 3)mA Solution: (c)
gm = = = 1 ms
∆VGS (4 − 3)V
I D = K (VGS − VTH ) 2
Hence, the correct option is (b). 4 = K (6 − 2) 2 .
8. The voltage gain of the amplifier is [2005] 1
(a) +5 (b) −7.5 ∴ K = mA/V 2
4
(c) +10 (d) −10 VGS = 10 − 4 × 1 = 6 V.
Solution: (d)
When RD is increased to 4 kW
+
VGS = 10 − 4 I D .
gmVGS 1
I D = [10 − 4 I D − 2]2 .
V in rd R 4
V out
4 I D = 16 I D2 + 64 − 64 I D .
I=o
I 16 I D2 − 68 I D + 64 = 0.
− I D = 2.84 mA, 1.4 mA.

Since, rd >> R
For MOSFET to be on, VGS must be greater that Vt
All current will pass through R. and this is possible only if
Vout = − g mVGS ⋅ R I D = 1.4 mA
−3 −3 −3
= −1× 10 × 2 × 10 × 10 × 10 = −20 mV.
If I D = 2.84 mA
Vout 20 mV Then VGS become -ve and less than Vt so transis-
Voltage gain = = = −10.
Vin 2 mV tor will be off for this value which is not possible
since VDG = 0 ≥ −Vt .
Hence, the correct option is (d).
9. For the n-channel enhancement MOSFET shown Hence, the correct option is (c).
in figure, the threshold voltage VTH = 2 V. The 10. In the circuit shown, the current gain b of the ideal
drain current ID of the MOSFET is 4 mA when transistor is 10. The operating point of the transis-
the drain resistance is 1 kW. If the value of RD tor (VCE, IC) is [2003]

Chapter 02.indd 20 11/12/2015 10:43:45 AM


Chapter 2  BJT and FET Biasing  |  8.21

10 Ω 12. In the circuit of figure, the value of the base cur-


rent IB will be [2000]
IC

5V
0.5 A
40 V
5 kΩ
15 V

b = 80
+
(40 V, 4 A) (b)
(a) (0 V, 4 A) IB 0.7 V −
(c)
(40 V, 5 A) (d)
(15 V, 4 A)
6.3 kΩ
Solution: (c) RE
We know that I B = 0.5 A.
−10 V
Suppose transistor is in active I C = β I B = 5 A.
Then VCE = 40 − 16 × 5 = −10 V. 18.2 m A
0 m A (b)
(a)
VCE  is negative, Q is in saturation.
As  40 mA
26.7 m A (d)
(c)
∴ VCE = 0 V.
Solution: (b)
So, 40 − 10 I C − VCE = 0.
We will apply Kirchhoff Voltage Law in the box
I C = 4 A. loop

Hence, the correct option is (b).
0 + 0.7 + I E RE − 10 = 0.
1 1. An n-channel JFET having a pinch-off voltage (VP)
of -5 V shows a transconductance (gm) of 1 mA/V I E RE = 9.3.
when the applied gate to source voltage (VGS) is 9.3
IE = ⇒ 1.4762 mA.
-3 V. Its maximum transconductance (in mA/V) is 6.3 × 103
 [2001]
(a) 1.5 (b) 2.0 We know,
(c) 2.5 (d) 3.0
 I 
Solution: (c) I B =  E  = 18.22 µA.
1 + β 
−2 I DSS  VGS 
gm = 1 − 
VP  VP  Hence, the correct option is (b).
−2 I DSS  −3  13. An NPN Si transistor is meant for low-current
mA V = 1 − −5 
−5   audio amplification. Match its following charac-
teristics against their values. [1998]
2  3
= × 1 − I DSS .
5  5  Characteristics Values
2
5 (A) VEB max (P) 0.7 V
I DSS =   mA.
2 (B) V (Q) 0.2 V
CB max

−2 I DSS −2 × 25 (C) VCE max (R) 6 V


( g m ) max = = mA/V
VP 4 × (−5) (S) 50 V
mA Solution:
( g m ) max = 2.5 .
V (A) → (P) (B) → (Q)
Hence, the correct option is (c). (C) → (R)

Chapter 02.indd 21 11/12/2015 10:43:48 AM


8.22 | Analog Electronics

I E = 1 mA
Five-marks Questions
 β 
IC =   I = 0.99 mA.
1. For the circuit shown in figure, I E = 1 mA, β = 99 1+ β 
and VBE = 0.7 V determine [2002] I
I B = C = 10 µA.
(a) current through R1 and RC β
(b) the output voltage Vo
(c) the value of RE At Base
15 V
VB − VBE − I E × 103 = 0.
1 kΩ (IC + 1) VB = 0.7 + 1 = 1.7 V.
VB
I1 = = 100 µA.
RF 17 kΩ
Vo
IC
I
Then I = I1 + I B = 110 mA
IB
VB (a) Current R1 is 110 mA
I1
IE Current at RC is IC + I = 0.99 mA + 110 µA
= 1.1 mA
17 kΩ R1 1 kΩ IE
(b) O/P voltage Vo = 15 − 103 ( I C + I )
= 15 − 103 (1.1× 10−3 )
= 13.9 V.
Solution:
15 V V − VB  13.9 − 1.7
(c) We know, RF =  O = −6
.
 I  110 × 10
RC = 1 kΩ RF = 110.91 kΩ.
RF
Vo
IC

IB

17 kΩ R1 1 kΩ

Chapter 02.indd 22 11/12/2015 10:43:50 AM


Chapter 3
Small Signal Modeling
and Analysis
resistor is bypassed by the capacitor Ce to when it
One-mark Questions is not bypassed. (Assuming of simplified approxi-
mate h-parameter model for transistor) is [1996]
1. In the single stage transistor amplifier circuit VCC
shown in figure, the capacitor CE is removed. RC
C1 R1 Vo
Then the AC small signal midband voltage gain of Vi Cc
the amplifier [2001]
R2
Re
V CC Ce

R1 RC

C1 Vo (a) 1 (b) h fe
C2
+
(1+ h fe ) Re (1 + h fe ) Re
(c) 1+
(d)
Vi R2 RE
hie hie
CE
Solution: (d)

−h fe RC
When Ce is unbypassed, AV 1 = .
(a) increases (b) decreases hie
(c) is unaffected (d) drops to zero −h fe RC
When Ce is bypassed, AV 2 = .
Solution: (b) hie + (1 + h fe ) Re
AV 1  1 + h fe  AV 2 hie + (1 + h fe ) Re  1 + h fe 
= 1+   Re . = = 1+   Re .
AV 2  hie  AV 2 hie  hie 

AV 1 Hence, the correct option is (d).
> 1.
AV 2

⇒ AV 2 > AV 1 . Two-marks Questions

Hence, the correct option is (b). 1. The transconductance g m of the transistor shown
2. In the transistor amplifier shown in figure, the in figure is 10 ms. The value of input resistance Rin
ratio of small signal voltage gain when the emitter is [2004]

Chapter 03.indd 23 11/3/2015 10:05:03 AM


8.24 | Analog Electronics

VCC Solution: (d)


R1
RC RB = = 5 kΩ.
10 kΩ R2
Vo
C=∞
VS C=∞ We know
b = 50
g m rπ = β .
1 kΩ

10 kΩ C=∞ β
rπ = .
gm

50
rπ = = 5 kΩ.
(a) 10 kW (b) 8.3 kW 10 × 10−3
(c) 5 kW (d) 2.5 kW Rin = RB  γ = 2.5 kΩ.

Hence, the correct option is (d).

Chapter 03.indd 24 11/3/2015 10:05:04 AM


Chapter 3  Small Signal Modeling and Analysis  |  8.25

(b) The small signal model by simplified model is


Five-marks Questions
R S ( b + 1)

1. The transistor in the amplifier circuit shown in re


 kT 
figure is biased at IE = mA. Use vT  =  = 26 mv,
IE
E
 q 
β 0 = 200, rb = 0 and r0 ⇒ ∞.  [2001] RE
CE

V CC Re

R C = 1 kΩ  R 
Re = RE ||  re + s 
Vo  1+ β 
25 kΩ
Vi 100 × 150.37
= = 60.06 Ω.
250.37
100 kΩ RE CE
R
re + S = 26 + 124.37 = 150.37.
β
1
(a) Determine the AC small signal midband volt- fL = .
2π Re CE
v 
age gain  o  of the circuit. 1
 v1  CE = = 265 µF.
2π Re f L
(b) Determine the required value of CE for the
circuit to have a lower cutoff frequency of 2. A current amplifier has an input resistance of
10 Hz . 10 kΩ, an output resistance of 10 kΩ, and a current
Solution: gain of 1000. It is feed by a current source having
(a) The small signal model is a source resistance of 10 kΩ and its output con-
nected to a 10 kΩ load resistance. Find the ­voltage
IC gain and the power gain [2000]
25 kΩ
+ + Solution:
bre
IL
IL
Ib h fe I b + +
Vi R C = 1 kΩ V o
Ro
IS RS Vi Ri Ai = I i = 10 kΩ Vo RL
− −
I L = −I C = −h fe I b − −

RS = 10 kΩ; RL = 10 Ω; A1 = 1000.

26 mv Vo
Vo = −h fe I b ⋅ Rc re = . Voltage gain =
IE VS

Vo = I b (25 × 103 + β re ) re = 26 Ω. I L RL 999 × 10
= = = 0.998.
I i ( RS + Ri ) (104 + 10)
= I b ( 27.6 ) × 103 β re = 2.6 kΩ.  
  Power gain = Voltage gain × Current gain
Vo −h fe Rc −100 × 1000  Ro   104 
= = . IL =   A1 I i =  4 1000 I i .
Vi ( Rs + β re ) 27.6 × 103  Ro + RL   10 + 10 

⇒ Av = −3.623. I L = 999 I i .

Chapter 03.indd 25 11/3/2015 3:46:24 PM


8.26 | Analog Electronics

IL IL II Z b = β re + (1 + β ) Re .
AIS = = × .
I S Ii I S VT 26 × 10−3
re = = = 26 Ω.
 RS  IE 10−3
II = IS  . Z b = 100 × 26 + 101× 100.
 RS + Ri 
Z b = 12.7 kΩ.
I I  RS 
= . −100 × 103
I S  RS + Ri  AV = = −7.87.
12.7 × 103
3. For the small BJT amplifier shown in given figure,
determine at 1 kHZ the following [1999] (c) 
When transistor is operated in saturation
region, maximum swing of collector is ob-
5V tained.
Using KVL
420 kΩ R C (1 kΩ) S − I C × 103 − VCE (sat ) − 100 I E = 0.
RB
Vo I C  I E , VCE (sat ) = 0.2 V.
Vi 25 µF
+
25 µF 0⋅7 V 4.8 = I C [103 + 102 ].

R e = 100 kΩ 4.8
IC = = 4.3636 mA.
1100

If VCE (sat ) = 0,
(a) quiescent collector current I CQ
Vo 5
(b) small signal voltage gain IC
= = 4.545 mA.
Vr 1100
(c) maximum possible swing of the collector current. 4. For the JFET amplifier shown in figure μ = 100,
Solution: rd = 50 kΩ.
(a) 5V (i) Draw the AC equivalent circuit
(ii) Find the voltage gain of the amp [1992]
1 kΩ V DD
420 kΩ IB IC

+ 100 kΩ
R1
C Vo
IE
0⋅7 V
100 Ω Vi ∼ R2

By using Kirchhoff Voltage Law,


Solution: 
5 − 420 × 103 I − 0.7 − 100 I = 0.
B E G
4.3 4.3
IB = = = 10 µA V
g m gs
(420 × 10 + 101× 100) 430 ⋅ 100
3
rd
∼ Vt Vgs 100 kΩ Vo
⇒ I CQ = I C ≈ 1 mA.

− β RC
AV =
(b)
Zb Vgs = VE S

Chapter 03.indd 26 11/3/2015 3:46:27 PM


Chapter 3  Small Signal Modeling and Analysis  |  8.27

Vgs = Vi . V 
(c) evaluate the small signal gain  o  of the
µ ­amplifier.  Vt 
= 2 m. rd =
gm
Id = −IL . Solution:
50 × 100 (a) We use Thévenin’s Equivalent Circuit
R ′ = rd 100 = = 33.33 kΩ.
150
V CC
Vo = − I d ⋅ R ′ − g mVgs R ′.
  IC
RL
Vo Vo
= = Voltage gain = − g m R ′ R TH
Vgs Vt

= −2 × 10−3 × 33.33 × 103 = −66.667. IB IE
VTH Re
5. Figure shows a common emitter amplifier [1991]
VCC
RL
R1 Vo
Rs
R1 R2
Vi RTH = R1  R2 = .
Cs R1 + R2
R2
Re
 R2 
VTH = VCC  .
   R1 + R2 
Ic = h fe I b Vbe = (1 + h fe ) Re ⋅ I b .
(b)
Ib
B C Vo = I L RL
Vo −h fe R1
= = −h fe I b RL .
Vbe (1 + h fe ) Re
E
We know,
(a) simplify the circuit by applying Thévenin’s
theorem to biasing network R1 , R2 at the base  RTH 
Vbe = Vi  .
of the transistor.
   RTH + RS 
(b) assuming CS to be a short for frequency range
considered. Draw the small signal AC model V  −h fe RL   RTH 
of the circuit obtained in (a) by using the sim- (c) o =   .
Vi  (1 + h fe ) Re   RTH + RS 
ple model for the transistor shown in figure.

Chapter 03.indd 27 11/3/2015 3:46:29 PM


Chapter 4
Frequency Response

One-mark Questions Two-marks Questions


1. In an RC-coupled common Emitter amplifier which 1. The typical frequency response of a two-stage direct
of the following is true?[1992] coupled voltage amplifier is as shown in figure. [2005]
(a)
(a) Coupling Capacitance affects the high fre-
|Gain|
quency (h f ) response and bypass capacitance
Frequency
affects Low frequency ( I f ) response.
(b)
(b) Both coupling and by pass capacitances affect |Gain|
the If response only.
Frequency
(c) Both coupling and by pass capacitances affect (c)
the hf response only. |Gain|
(d) Coupling Capacitance affects the If response Frequency
and the bypass capacitance affects the hf (d)
­response. |Gain|
Solution: (d) Frequency
In the case of RC-coupled common Emitter am- Solution: (b)
plifier, coupling capacitance affects the If re- For a two stage direct coupled voltage amplifier
sponse and the bypass capacitance affects the hf provides a gain at zero frequency. And the direct
response. coupled (or) DC coupled amplifier provides a gain
Hence, the correct option is (d). at low frequency.
Hence, the correct option is (b).

Chapter 04.indd 28 11/3/2015 11:36:40 AM


Chapter 5
Operational Amplifiers
and Applications
R
One-mark Questions +12 V
R −12 V

1. In the circuit shown below what is the output volt- Vi
+

Vo
age (Vout ) in Volts if a silicon transistor Q and an −12 V +
R
ideal OP-AMP are used? [2013] R −12 V

Q R R

1 kΩ

+ V out

5V ± (a)  +12 V       (b)  Vo


Vo
+12 V

(a) −15 (b) −0.7 −6 +6


Vi
(c) +0.7 (d) +15 −6 +6
Vi
Solution: (b)
Using the concept of virtual ground, V = 0.
−12 V
Vout = −0.7 V. −12 V

Q
(c)  +12 V  (d)  Vo

+12 V

V out −6 V 6V Vi

−6 V 6V Vi
Hence, the correct option is (b).
2. For the circuit shown below, the correct transfer −12 V
characteristics is [2011]
−12 V

Chapter 05.indd 29 11/12/2015 10:44:18 AM


8.30 | Analog Electronics

Solution: (d) 4. Given that the OP-AMP is ideal, the O/P voltage
First section is differential amplifier having gain Vo is [2010]
of 1 2R
Vo
+10 V
+12 V R

Vo
+
12 −10 V
Vi
−12 +2 V

−12 V (a) 4 V (b) 6 V


(c) 7.5 V (d) 12.12 V
Solution: (b)
Output is
 2R 
V = 1+ V = (1 + 2)(2) = 6 V.

 R f + R1  Rg Rf o
 R 
Vo =  V − V.
 R + R  R 2 R 1 Hence, the correct option is (b).
 g 2  1 1
5. The nature of feedback in the OP-AMP circuit
Vo = −Vi .
shown is [2009]
Second stage-Schmidt trigger +6 V
1 kΩ 2 kΩ
− −
+ Vo + V out
V in ∼
−6 V
Rf
Rg
(a) Current-Current feedback
(b) Voltage-Voltage feedback
Rg Vo (c) Current-Voltage feedback
Vt = ± Vo = = ±6 V. (d) Voltage-Current feedback
R f + Rg 2
Solution: (b)
Hence, the correct option is (d). Voltage-series feedback arrangement or Voltage-
3. A low pass filter with a cutoff frequency as 30 Hz Voltage feedback.
is cascaded with a high pass filter with a cutoff Hence, the correct option is (b).
frequency of 20 Hz. The resultant system of filters 6. The circuit shown in figure is [2007]
coil function as [2011]
R1
(a) all pass filter (b) an all-stop filter +
V +
(c) a band stop filter (d) a band pass ­filter R2 −
LOAD
Solution: (d)
r
High pass filter
Low pass
filter rV
(a) a voltage source with voltage
R1 || R2
r || R2
(b) a voltage source with voltage V
R1
20 Hz 30 Hz r || R2 V
(c) a current source with current ⋅
Pass Band R1 + R2 r
It is a band pass filter. R2 V
(d) a current source with current ⋅
Hence, the correct option is (d). R1 + R2 r

Chapter 05.indd 30 11/12/2015 10:44:20 AM


Chapter 5  Operational Amplifiers and Applications  |  8.31

Solution: (d) 8. For the circuit shown in figure with an ideal


It behaves as current source because the output OP-AMP, the maximum phase shift of the output
current ( I o ) depends upon (Vin ) and resistance Vo with reference to the input Vin is [2003]
only.
 V × R2  R
 
R +R R
Where, I o =  1 2  −
r V in Vo
+
R2 V R
= ⋅ . C
R1 + R2 R
Hence, the correct option is (d).
(a) 0 (b) −90
7. For given sinusoidal input voltage, the voltage (c) +90 (d) ±180
waveform at point P of the clamper circuit shown
Solution: (d)
in figure will be [2006]
1
Vin ×
V in jCω
V( + ) =
1
R+
jCω
Vin
=
1 + jRCω
Where, V( − ) = V( + ) (For ideal OP-AMP)
+12 V
C
− Vin − V( − ) V( − ) − Vo

RL = .

+ V in
+ P R1 R1
−12 V
Vo − 2V( − ) − 2Vin + Vin = 2V( + ) − Vin

 2 
  (a)    (b)  = − 1 Vin
 1 + jω RC 
 1 − jω RC 
=  Vin
 1 + jω RC 
  (c)    (d)  V 
∠  o  = −2 tan −1 ω RC.
12 V
+0.7 V  Vin 
−0.7 V For −90° ≤ θ ≤ 90°, phase shift maximum occur is
−12 V
(±180°)
Solution: (d) Hence, the correct option is (d).
For −ve half of the input +ve terminal of OP- 9. The feedback factor for the circuit shown in figure
AMP is at higher potential than −ve terminal and is [2000]
output goes to +Vsat but due to this high potential 100 kΩ
diode gets on and restricts the output to 0.7 V −
− Vo
only. And for +ve half of the input +ve terminal +
VS
of OP-AMP is at lower potential than −ve termi- +
nals potential and output goes to −Vsat and remains 90 kΩ 1 kΩ
at −Vsat . 10 kΩ

Hence, the correct option is (d).

Chapter 05.indd 31 11/12/2015 10:44:24 AM


8.32 | Analog Electronics

9 9 12. Let the magnitude of the gain in the inverting


(a) (b)
100 10 OP-AMP amplifier circuit shown be x with switch
1 1 S1 open. When the switch S1 is closed, the magni-
(c) (d)
9 10 tude of gain becomes. [1996]
Solution: (d)
I 2 = 0, I1 = I . S1
R
V+ = 10 kΩ × I .
R R
Vi −
Vo = (90 + 10) kΩ × I = 100 kΩ × I . Vo
+
V+ 10 kΩ × I 1
β= = = .
Vo 100 kΩ × I 10
x
Hence, the correct option is (d). (a) (b) −x
2
10. The circuit shown in figure uses an ideal OP-AMP (c) −2x
2x (d)
working with +5 V and −5 V power supplies. The
output voltage Vo is equal to [2000] Solution: (a)
−2 R
= −2. X =
1 kΩ R
+5 V −R
X′= = −1.
−   R
Vo
1 mA a + Va = 0 X′ 1
−5 V = .
X 2

1
X′= X.
(a) +5 V (b) −5 V 2
(c) +1V (d) −1V
Hence, the correct option is (a).
Solution: (d)
13. A non inverting op-AMP amplifier is shown in fig-
1 kΩ ure. The o/p voltage is [1996]


Vo 2R
1 mA + R

R Vo
−2 +

0 − Vo = 1 mA × 1 kΩ. 2 + sin 100t


R
Vo = −1 V.
Hence, the correct option is (d). 3
(a) sin 100t (b) 3 sin 100t
11. A major advantage of active filters is that they can 2
be realized without using [1997] (c) 2 sin 100t (d) None of these
(a) OP-AMPS (b) inductors
(c) resistors (d) capacitors Solution: (a)
Solution: (b) −2 − V+ 2 + sin 100t − V+
A major advantage of active filters is that they can + = 0.
R R
be realized without using inductors. 1
Hence, the correct option is (b). V+ = sin 100t.
2

Chapter 05.indd 32 11/12/2015 10:44:27 AM


Chapter 5  Operational Amplifiers and Applications  |  8.33

0 − V+ V+ − Vo (c) The open-loop gain of the OP-AMP is infinity


= .
R 2R (d) CMRR is infinity.
3 Solutions: (c) and (d)
Vo = 3V+ = sin 100t. The two input terminals of the OP-AMP are at the
2
same potential because the open-loop gain of the
Hence, the correct option is (a). OP-AMP is infinity and CMRR is infinity.
14. The common mode voltage of a unity gain (voltage
Hence, the correct options are (c) and (d).
follower) OP-AMP buffer in terms of its output
voltage Vo is __________. [1995]
Solution: Two-marks Questions
1. In the figure shown assume the OP-AMP to be ideal.

Vo Which of the alternatives gives the correct Bode
Vi + VO (ω )
b plots for the transfer function ? [2014-S1]
Vr (ω )
Vi = Vo . +Vcc
1 kΩ
1 Vi
+
Vcm = Va + Vb = (Vi + Vo ) = Vi = Vo .
2 1 µF Vo
15. Given figure, shows a non-inverting OP-AMP −
summer with V1 = 2 V and V2 = −1 V. The output −Vcc
voltage Vo =  [1994]
Ro
2R   (a)  V0(w)
20 log
R V1(w)

R Vo f
2V + 0 10 102 103 w 0 10 102 103 w

−1 V −10 −p/4
R
−20 −p/2
R −30

  (b)  V0(w)
20 log
Solution: V1(w) f
2 − V (−1 − V ) V p/2
+ = p/4
R R R 0 10 102 103 w 0
1
⇒ V = V. −10 −p/4 10 102 103 w
3 −20 −p/2
0 − V V − Vo −30
=
R 2R
Vo = 3 V = 1 V.   (c)  V0(w)
20 log
V1(w)
16. An ideal OP-AMP is used to make an inverting
amplifier. The two input terminals of the OP-AMP
are at the same potential because [1992] 0 10 102 103 w 0 10 102 103 w
(a) The two input terminals are directly shorted
internally −10 −p/4
(b) The input impedance of the OP-AMP is −20 −p/2
infinity −30

Chapter 05.indd 33 11/12/2015 10:44:29 AM


8.34 | Analog Electronics

V0(w) 2. Given the OP-AMPs in the figure of ideal, the out-


 (d) 20 log
V1(w) put voltage Vo is [2014-S1]
f
p/2 V2 + R
p/4 −
R R
0 0 −
−10 10 102 103 w 10 102 103 w 2R Vo
−p/4 +
−20 −p/2 R R
−30 − R
V1 +

Solution: (a)
  Vo ( s ) = Va ( s ). V1 − V2 (b)
(a) 2(V1 − V2 )

Vb ( s )  Va ( s )  Vo ( s ). V − V2
(c) 1
V1 + V2
(d)
2
 1  Solution: (b)
 
Vb ( s ) = Vi ( s )  SC  .  V −V 
  1
R+   1 =  1 2 .
 SC   2R 
 1  V1′ − V1 = IR.
Vo ( s )  
=  RC  .  V1′ = V1 + IR.
Vi ( s )  1 
S +  V2 − V2′ = IR.
 RC 
V ′ = V2 − IR.
Put S = jω.   2
 Vo = (V1′ − V2′) = (V1 − V2 ) + 2 IR.
1
TF = Hence, the correct option is (b).
1 + jω CR
3. In the circuit shown below the OP-AMPs are ideal.
1
= . Then Vout in Volts is [2013]
1 + (ωCR) ∠ tan −1 (ωCR)
2

1 kΩ 1 kΩ
−2 V
1 +15 V
H (ω ) = . +15 V
1 + (ωCR) 2 +
+
− Vout

H (ω ) dB = −10 log(1 + (ωCR) ). 2
−15 V
1 kΩ
−15 V
When ω = 0, | H (ω )| = 0. 1 kΩ
+1 V 1 kΩ
ω = 10 , 3
H (ω ) = −3d B .
ω = 104, H (ω ) = −10d B .
(a) 4 (b) 6
P = − tan −1 (ωCR). (c) 8 (d) 10

When ω = 0, φ = 0. Solution: (c)


Part 1
π
ω = 103 , φ = − . V1 + 2 V1 − VX
4 + = 0.
π 1 kΩ 1 kΩ
ω = ∞, φ = − .
2 2V1 + 2 = VX .
Hence, the correct option is (a). VX = 4 ∴ V1 = 1 V.

Chapter 05.indd 34 11/12/2015 10:44:34 AM


Chapter 5  Operational Amplifiers and Applications  |  8.35

Part 2 +6 V
1 kΩ
Vx V − Vout Vin −
+ x = 0.
1 kΩ 1 kΩ Vout
+
Vout = 2VX = 8 V. 2 kΩ
−3 V
Hence, the correct option is (c).
1 kΩ
4. In the circuit shown below, Q1 has negligible
collector-to-emitter saturation voltage and the
diode drops negligible voltage across it under for-
(a) 6
ward bias. If VCC is +5 V, X and Y are digital signals
with 0 V as logic 0 and VCC as logic l, then the v
Boolean expression for Z is [2013]
t3 t6
0
+Vcc t
−3
R1 (b) 6
Z
R2 v
X Q1 t3 t6
0
t
Y −3
(c) 6
v
(a) XY (b) XY t6
0
t2 t4 t
(c) XY (d) XY −3
Solution: (b) (d) 6
X = 0 ⇒ Z = 0.
v
X Y Z
0 t2 t4 t6
0 0 0
t
0 1 1 −3
1 0 0
Solution: (d)
1 1 0
Vsat = 6 V, − Vsat = −3 V.
Here Z = X ⋅ Y .
 R2  1
Hence, the correct option is (b). VUTP = +Vsat   = 6   = 2 V.
5. An ideal OP-AMP circuit and its input waveform
 R1 + R2  3
are shown in the figures. The O/P w/f of this circuit Vo = +6 V.
will be [2009] If VUTP > Vi ⇒ Vo = +6 V.
VUTP < Vi ⇒ Vo = −3 V.
3
′ = −3   = −1 V.
1
2 VLTP
1
t4 t5
3
t6
V 0 At t < t4 , Vo = −3 V.
t1 t2 t3 t
−1
−2 When VLTP > Vi ⇒ Vo = +6 V.
−3
Hence, the correct option is (d).

Chapter 05.indd 35 11/12/2015 10:44:37 AM


8.36 | Analog Electronics

6. The following circuit has R = 10 kΩ, C = 10 μF. P


The input voltage is a sinusoidal at 50 HZ with and Vo
RMS values of 10 V. Under ideal conditions, the 0
Vin Vo
current Is from the source is [2009]
R
10 kΩ Q
Vs = 10 V rms, 50 Hz

is
+
a
∼ OP-AMP 7. If R= R= RA and R= R= RB , the circuit acts
b − 1 2 3 4
as a [2008]
10 kΩ (a) all pass filter (b) band pass
R (c) high pass filter (d) low pass
10 µF C Vb = Va = Vs Solution:
Suppose Z = R2 || 1 || SC.
(a) 10π mA leading by 90°
(b) 20π mA leading by 90°  1 
R2  
(c) 10 mA leading by 90°  =  SC  = R2
.
(d) 10π mA lagging by 90° 1 1 + SCR2
R2 +
Solution: (d) SC
Apply KCL at b RA
 Z = .
Vb − 0 Vb − Vo 1 + SCRA
+ = 0.
 1  R Apply KCL at b
 
 CS  I1 = I 2 .
 Vo 
Vb =  .
Vi − Vb Vb Vi
 1 + SCR  = ⇒ Vb = .
R3 R4 2
Apply KCL at a
Vb − Vb − Vb SCR KCL at a, I = I f .
Is = .
  R Vi − Va Va − Vo
= .
 I s = − jω CVb = − jω CVs . RA Z
I s = WC VS
 SCRA 
Vi
= 2π f × C × Vs Vo =
 .
 1 + SCRA 
2

= 2π × 50 × 10 × 10−6 × 10 = 10π × 10−3 HIGH PASS FILTER
= 10 mA lagging by 90°. Hence, the correct option is (c).
Hence, the correct option is (d).
8. The block diagram of two types of half wave recti-
Common Data for Questions 7 and 8: fiers are shown in figure. The transfer character-
A general filter circuit is shown in figure istics of the rectifiers are also shown within the
R2 block. [2008]

C Vo
R1
Vi Vin Vo

R3 Vo 0
+ Vin

R4 It is desired to make full wave rectifier using two


half wave rectifiers. The resultant circuit will be

Chapter 05.indd 36 11/12/2015 10:44:41 AM


Chapter 5  Operational Amplifiers and Applications  |  8.37

(a) R (a) It makes a transition from -5 V to +5 V at


t = 12.98 μs
R (b) It makes a transition from -5 V to +5 V at
Vin P −
+ Vo t = 2.57 μs
R
Q (c) It makes a transition from +5 V to -5 V at
t = 12.98 μs
(b) R (d) It makes a transition from +5 V to -5 V at
t = 2.57 μs
R Solution: (b)
Vin P −
R + Vo
Q Vc = 20(1 − e − t /τ ).
At inv. terminal of OP-AMP

 100  10
Vx = Vo   = Vo .
(c)  110  11
R 10
R
20(1 − e −1/τ ) ⇒ × 5.
Q 11
+
Vo
− P Vin 5
R 1 − e − t /τ = .
22
5 17
R e1/τ = 1 − = .
(d) R R 22 22
22
τ = Rcet /τ =
R − 17
+ Vo
Vin P
= 10−5 × 0.257829 = 2.578 µs.
R
Q Hence, the correct option is (b).
Solution: (b) 10. A relaxation oscillator is made using OP-AMP
Vo = −Vi  for Vi > 0. as shown in figure. The supply Voltage of the
Vo = 0  for Vi < 0. OP-AMP are ±12. The voltage waveform at point
p will be [2006]
From transfer characteristics of rectifier Q is
R1
Vo = 0  for Vi > 0.
Vo = −Vi  for Vi < 0. R2

Hence, the correct option is (b).


C −
9. The switch S in the circuit of the figure is initially
+
closed. It is opened at time t = 0. You may neglect 2 kΩ
the Zener diode forward voltage drops. What is the
behavior of Vout for t > 0? [2007] P 10 kΩ
+10 V +10 V 10 kΩ

1 kΩ

Vout
+
+10 kΩ 5.0 V
(a)
S 0.01 −10 V
6
Vo
100 kΩ
−10
−10 V

Chapter 05.indd 37 11/12/2015 10:44:44 AM


8.38 | Analog Electronics

(b) (d) Vout


10 t

−6
Solution: (c)
(c) OP-AMP has high open-loop gain and is forced to
6
operate in saturation region for given input wave-
form of OP-AMP
−10 Vo = AOL (0 − Vi ).

(d) Vo = −V .
10
Hence, the correct option is (c).
−6 12. Consider the inverting amplifier, using an ideal
operational amplifier shown in figure. The designer
Solution: (c) wishes to realize the input resistance seen by the
small signal source to be as large as possible, while
 10  keeping the voltage gain between −10 and −25.
V p = Vo   → D1 − ON = + 6 V.
 20  The upper limit on RF is 1 MΩ. The value of R1
 10  should be [2005]
D2 - ON  then  V p = Vo   = −10 V. RF
 12 
Hence, the correct option is (c).
11. In the given figure, if the input is a sinusoidal sig- Vin −
R1 + Vout
nal, the output will appear as shown [2005]

Vi (a) Infinity (b) 1 MΩ


t
(c) 100 kΩ (d) 40 kΩ
Solution: (c)
− RF
+V Gain of inverting AMP is A = .
R R1

RF = 1MΩ.
+ A → −10 and − 25. Then R1 = 105 = 100 kΩ
+ 
RL Vout when A = −10.
R −V
−106
R1 = = 40 kΩ when A = −25.
−25
∴ R1 = 100 kΩ.
Hence, the correct option is (c).
Vout
(a)
1 3. In the active filter circuit shown in figure, if Q = 1,
a pair of poles will be realized with two equal to
t
[2004]
R 1 = 200 kΩ
(b)
Vout

t 1 nF 1 nF
R2

Vout
(c)

t
+

Chapter 05.indd 38 11/12/2015 10:44:47 AM


Chapter 5  Operational Amplifiers and Applications  |  8.39

(a) 1000 rad/sec (b) 100 rad/sec +5 V


(c) 10 rad/sec (d) 1 rad/sec
Solution: (a)
200 kΩ
 1 1  1 1
S +S +
2
+ + V
T (S ) =  C1 C2  R1 C1C2 R1 R2 .
Q1 Q2
 1 1 1  1 V=0
S +S
2
+ + +
 C1 R1 C1 R2 C2 R1  C1C2 R1 R2
And,
ω0
S2 + S ⋅ + ω02 1 kΩ
Q
I = 1.25 mA
 1 1  1 1
= S +S +
2
 + .
C
 1 C R
2  1 C1 2 R2 R1
C
(a) 1.25 V (b) 1.35 V
(c) 2.5 V (d) 5.0 V
1
Then ω0 = . Solution: (b)
C1C2 R1 R2 When Vi = 0
−1 Q1(Cutoff) − Off and Q2 − ON (sat)
 CC RR  1 1 
Q= 1 2 1 2  +  . VCE (sat ) = 0.1 V.
 R1  C1 C2   Applying KVL from 5 V to ground through tran-
sistor Q2 C-E terminals
Q 1,=
Given= C1 C2 = 1 nF and R1 = 200 kΩ.
5 − I c × 200 = Vo .
During −ve half cycle of i/p (Vi < 0) OP-AMP sat- 5 − I c × 200 − VCE (sat ) − 1.25 × 10−3 × 103 = 0.
urated to Vo = +V .
Vo − 0.1 − 1.25 = 0.
Vo = 1.35 V.
1 R2 × 2 × 10−13
= (2 × 10 ).
9
Hence, the correct option is (b).
Q 2 × 105
 V 
R2 × 4.472 × 10−3 = 1. 15. The input resistance RIN  = x  of the circuit in
the figure is  Ix  [2004]
R2 = (223.606) 2 .
R 1 = 10 kΩ R 2 = 100 kΩ
R2 = 50 kΩ.
1
ω0 = . − Vo
C1C2 R1 R2 +
Vx
1
= . Ix
−9 −9 R 3 = 1 MΩ
10 × 10 × 200 × 103 × 50 × 103
ω0 = 10, 000 rad / s. -100 kΩ
(a) 100 kΩ (b)
(c) 1 MΩ (d) -1 MΩ
Hence, the correct option is (a).
Solution: (b)
1 4. In the Schmitt trigger circuit shown in figure, if
VCE ( sat ) = 0.1 V, the output logic low level (VOL) is Gain of OP-AMP circuit is Vo = 1 + 100 
[2004] Vx  10 

Chapter 05.indd 39 11/12/2015 10:44:50 AM


8.40 | Analog Electronics

Vo 100 Ω
= 11. Vi −
Vx +
10 kΩ
Vo
Vx =
.
11 Vi = 10 sin wt
3 kΩ
Using KCL at non-inverting terminal

Vx −11VX 2V
Ix =
106
(a) 5 V in the positive slope only
Vx (b) 5 V in the negative slope only
Ix ⇒ ⇒ − 100 kΩ
Ix (c) 5 V in the positive and negative slopes
Rin = −100 kΩ. (d) 3 V in the positive and negative slopes
Solution: (a)
Hence, the correct option is (b).
When Vo = +15 V
1 6. Assuming the operational amplifier to be ideal the
Vo 3
gain V for the circuit shown in figure [2003] VLTP = Vref + (−Vo − Vref )   .
i    13 
10 kΩ 10 kΩ
VUTP = 22++(15
 3 
(−17
− 2)) ..
1 kΩ   13
13 
1 kΩ When VUTP= >−1
VLTP V.i 923
3 V.
Vi −
+
Vo Vo = +15 V.
When VUTP < Vi ⇒ Vo = −15 V.
(a) −1 (b) −20 At Vo = −15 V
(c) −100 (d) −120 3
VLTP = Vref + (−Vo − Vref )   .
Solution: (d)  13 
Apply KVL at a 3
= 2 + (−17)   .
Vi Vx  13 
  I = I1 ⇒ =
10 10 × 10
3 3
VLTP = −1 . 923
3 V.
Vx = −10 Vi .  ∴ Output will change from +15 V to -15 V when
instantaneous value of input sine wave is 5 V
Using KVL at b
in positive slope only.
Vx Vx − Va Hence, the correct option is (a).
I1 = I 2 + I 3 ⇒ + .
103 10 × 103 18. An OP-AMP having a slew rate of 62.8 V/μs is
− Vx = 10 Vx + Vx − Vo . connected in a voltage follower configuration. If

Vo = 12 Vx = −120 Vi . the maximum amplitude of the input sinusoidal is
V 10 V. Then the minimum frequency at which the
⇒ Gain = o = −120. slew rate limited distortion would set in at the out-
Vi
put is [2001]
Hence, the correct option is (d). (a) 1 MHz (b) 6.28 MHz
17. The output voltage (Vo) of the Schmitt trigger (c) 10 MHz (d) 62.8 MHz
shown in figure swings between +15 V and +15 V. Solution: (a)
Assume that the operational amplifier is ideal. The Vo = Vi = Vm sin ωt
output will change from +15 V to -15 V when
the instantaneous value of the input sine wave is dVo
= +Vmω cos ωt
 [2002] dt

Chapter 05.indd 40 11/12/2015 10:44:54 AM


Chapter 5  Operational Amplifiers and Applications  |  8.41

 dVo  −Vsat V 
 dt  = SR = Vmax = 2π fVm = −Vsat +  sat + Vsat  eT2 /τ .
  max 2  2 

2π fVm Vsat 3Vsat −T2 /τ
SR = V/µs = e .
106 2 2
SR × 106 62.8 × 106 eT2 /τ = 3.
f = = .
2π Vm 2π × 10 T = 2τ ln 3.
f = 106 Hz = 1 MHz. Hence, the correct option is (b).
Hence, the correct option is (a). 20. Match the following [1998]
19. For the oscillator circuit shown in figure, the List-I (Circuit)
expression for the time period of oscillations can
be given by (where τ = RC) [2001] (A) Vcc

C

Vo −
+ Vo
+
R
∼ Vi

(B)
(a) τ ln 3 (b) 2τ ln 3
(c) τ ln 2 (d) 2τ ln 2 −
Vo
Solution: (b) +
Suppose Vo = +Vsat . FiF Vi

R  Vsat
 Vx = Vo  =+ .
 2R  2
As Vo = +Vsat is given to capacitor ‘C’ charges (C)
­towards Vsat through ‘R’ with τ = RC. −
Vo
For T1 +
∼ Vi
Vo = V1 + (Vi − V f )e − t /τ .
V
V f = Vsat , V1 = − sat , τ = RC.
2 List-II (Functions)
−V (P) High-pass filter
At t = T1 , Vc = sat
2 (Q) Amplifier
Vsat  −V  (R) Comparator
= Vsat +  sat − Vsat  e −T1 /τ . (S) Low-pass Filter
2  2 
−Vsat 3 Solution:
= − Vsat e −T1 /τ . (A) → (R) (A) → Comparator
2 2
T1 /τ (B) → (S) (B) → LPF
e = 3 ⇒ T1 = τ ln 3.
(C) → (P) (C) → HPF (2nd order)
For T1
+V 21. In the following circuit the output V follows an
V f = −Vsat , Vi = sat , τ = RC.
2 d 2V dV
equation of the form + a⋅ + bV = f (t ).
−V dt 2
dt
At t = T2 , Ve = sat . Find a, b and f (t ).  [1992]
2

Chapter 05.indd 41 11/12/2015 10:44:58 AM


8.42 | Analog Electronics

C dV1 dVa Va
C
= + .
R dt dt RC
− R
− 1  V2 + et 
RC ∫  2
+ V V1 = Va +   dt.
+

R
C R  V + et  1 et
R
V1 =  2 +
 2  2 RC
∫ V2 dt +
2 RC (1)

R Apply KCL at a
+
et V1  dV 
R = −C  − 2  .
R  dt 
Solution: dV
V1 = − RC 2 (2)
Using Kirchhoff current law at ‘b’ dt
dV
R V2 = − RC
− (3) dt
V1
V2 + Put (3) in (2), we get
R
Vd ≈ 0 d 2V
R Va ≈ Vb V1 = R 2 C 2 . (4)
dt
et
RHS in Equation. (1) is
Vd ≈ 0.
V2 + et 1 et
Va ≈ Vb . V1 =
2
+
2 RC ∫ V2 dt +
2 RC
.
V2 − Vb Vb − et et  1  RC dV 1 dv
= ⇒ 2Vb = V2 + et .
R R V1 = 1 + −
2  RC  2 dt 2 RC
+ ∫ − RC dt.
dt
1
Vb = (V2 + et ). d 2
V RC dV 1 e t
 1 
2 ∴ R 2C 2 2 + + V = 1 + .
dt 2 dt 2 2  RC 
1
Va = (V2 + et ). dV 1 dV 1 et  1 
2 + + V= 1+
2 2  .
dt 2 RC dt 2 RC 2 R C  RC 
Using KCL at a
0 − Va d Compare Equation (5) with the
= +C (Va − V1 ). 1
R dt ∴ a= .
Va dV dV 2 RC
− = a− . 1
RC dt dt b= .
2 RC
dV1 dVa Va
⇒ = + . et  1 
dt dt RC f (t ) = 1 + .
2R 2C 2  RC 
dV1 d  V2 + et  V2 + et 22. The circuit shown in figure is excited by the input
=  +
dt dt  2  2 RC wave form shown. Sketch the wave form of the
output. Assume all the components are ideal.
1 dV2 1 t V et
= + e + 2 + . [1992]
2 dt 2 2 RC 2 RC
V
dV1 1  V2 dV2  1 
=  + + et  1 +  . 1
dt 2 RC dt  RC 
0 t sec
We integrate on both sides, 1 2 3

Chapter 05.indd 42 11/12/2015 10:45:00 AM


Chapter 5  Operational Amplifiers and Applications  |  8.43


V1′ V2 V3 −Vo
Vo + + = .
Vi + VA D 10 5 4 20
1C D2
−Vo 2V1′ + 4V2 + 5V3
= .
20 20
Solution: ∴ Vo = −(2V1′ + 4V2 + 5V3 ).
Suppose capacitor is initially relaxed
Vo = −(−2 ∫ V1dt + 4V2 + 5V3 ).
VO = 0 = VC .
24. The output of the filter in above is given to the cir-
VA = AOL (Vi − 0).
cuit shown in figure. The gain vs frequency char-
VA = AOL (Vi − 0). acteristics of the o/p (Vo) will be
When Vi > 0, D1 − F.B. and replaced by S.C. for RA / 2
voltage follower circuit,
VO= V=
i VC . Vin C Vo

Output becomes maximum at t = 0.5 s


VA = AOL (Vi − 1). (a)
Gain  (b) Gain
But when Vi < 1
VA is −ve.
C holds previous value w w
V=O V=
C 1. Gain
(c)  (d) Gain
We find this is repeated for every cycle diode D2 is
on only when Vi > 1 V.
w w
So, D2 remains in reverse bias only.
Solution: (d)
23. With the ideal operational amplifiers, the circuit in
 
figure simulates the equation [1991]  
V S
Vo′ = i  
100 µF 20 kΩ 2 S+ 1 
10 kΩ  RAC 
− 10 kΩ  
V1 −  1 
+ Vo
+  SC 
5 kΩ Vo = V1′ 
4 kΩ
 RA + 1 
4 kΩ  2 SC 
−15 V V3

  
Vi  S  2

Solution: =   
I = If . 2 S+ 1   2 
 RAC   RAC  S + 
    RAC  
V1 dV ′
= −100 × 10−6 1 .
10 × 10 3
dt   2 
dV1′ −V1 V  S  R 
= = −V1 . = i  AC

dt 1 2 S + 1  S + 2 
 RAC   RAC 
By integrating  ⇒ Vi t = − ∫ V1dt.
Transfer Function of BPF
We apply KCL at a ′ Hence, the correct option is (d).

Chapter 05.indd 43 11/12/2015 10:45:05 AM


8.44 | Analog Electronics

1 kΩ 2 kΩ 4 kΩ 8 kΩ
Five Marks Questions 1V
− −
+ +
1. A simple active filter as shown in figure. Assume −1 V
ideal OP-AMP. Derive the transfer function Vo /V1 of 3 kΩ
the circuit and state the type of filter (i.e., high-pass, 1 kΩ
low-pass, band-pass or band-reject). Determine the
required values of R1, R2 and c in order for the filter Solution:
to have a Cutoff (or) 3-dB frequency of 1 kHz, a Apply KCL at a
high frequency input resistance of 100 kΩ and a
high frequency gain magnitude of 10. [2001] I3 = I 4 .
I − Va Va − V1
= .
R2 103 2 × 103
Vi R1 C 2.2Va = Va − V1 .

+
Vo V1 = 3Va − 2.
Apply KCL at b
I1 + I 2 = 0.
Solution: Vb Vb − Vo
Vd  0. + = 0.
103 3 × 103
Va  0.
3Vb + Vb − Vo = 0.
Then I = If  .
Vo
Vi −Vo −Vo R2 R2 . Vb =
= ⇒ = = . 4
Z R2 Vi Z  1 + SCR2 
 SC  Using KCL at inverting terminal of OP-AMP 2.
  I5 = I6 .
  V1 + 1 −1 − Vo
Vo − R2  S  = .
=  . 4 × 103 8 × 103
V1 R1  S + 1 
 2V1 + 2 = −1 − Vo .
R1C 
Vo = −2V1 − 3
(i) R1 = 100 kΩ.
= −2(3Va − 2) − 3 = −6Va + 4 − 3 = −6Vb + 1.
R
(ii) 2 = 10 ⇒ R2 = 1000 kΩ. V 
R1 Vo = −6  o  + 1.
 4 
 ∴ R2 = 1 MΩ.
2.5Vo = 1.
(iii) 3 − dB cut-off frequency is 1
Vo = .
1   2.5
fc = .
2π R1C ∴ Vo = 0.4 V.
1 1
C= = = 1.5912 nF. 3. The input voltage V1 in the given circuit is a 1 kHz
2π R1 f1 2π 105 × 103 since wave of 1 V amplitude. Assume ideal OP-AMP
2. For the OP-AMP circuit shown in figure. Determine with ±15 V DC Supply. Sketch on a single diagram
the output voltage Vo. Assume that the OP-AMPs the waveform of Vi, V1 and Vo shown, indicating the
are ideal [2001] peak values of V1 and average value of Vo. [1999]

Chapter 05.indd 44 11/2/2015 6:26:47 PM


Chapter 5  Operational Amplifiers and Applications  |  8.45

10 kΩ Solution:
5 kΩ (i) Current entering into OP-AMP terminal is zero.
1 kΩ 10 kΩ
∴ Vo = Vs = Va = Vo .

Vi − Vo
+ Vo
+ 100 µF Vo = Vs ⇒ = 1.
Vs
V1
=Vo 10
= V, Vs 10 V.
Solution: (ii) Vo = Vs and Is = 0
During +ve cycle of I/P diode reverse biased, Vs
D − OFF → OC Z in = = Ri = 100 kΩ.
Is
V1 5. Figure below shows an OP-AMP amplifier. Find
the output voltage in steady state condition where
− (i) switch is open (ii) S is closed [1994]
+ Va = 0
0.1 µF
V1 = 0
S
During -ve cycle of I/P diode is ON → SC
1 kΩ 1 kΩ

10 kΩ 1 kΩ If

1 kΩ a
V1 − 5 kΩ
+
+ Vi = 1 V

Solution:
5 × 10 (i) Vd  0 ⇒ Va  0.
= 3.33 kΩ.
15
Apply KCL at a
 3.33  V1 − Va Va − Vo
V1 = −Vi   = −3.33Vi . = ⇒ − Vo = 2 V
 1  103 2 × 103
Vi = 3.33 V. Vo = −2 V.
Vo = Vi OP-AMP 2 is voltage follower circuit. (ii) When S is closed. The capacitor changes to the
Vm 3.33 peak value. In steady state C acts as OC Then
Vdc = = = 1.061 V. again Vo = −2 V.
π π
4. In the circuit of figure RS = 2 kΩ, RL = 5 kΩ. For 6. For the circuit shown in figure [1992]
the OP-AMP A = 105, Ri = 100 kΩ, Ro = kΩ. For
V R1
Vo = 10 V. Calculate Vs and o and estimate the
VS
input resistance of the circuit. [1998] R1

Rs Vi Vo
+ Vo +
− C
Vs RL R

Chapter 05.indd 45 11/2/2015 6:26:50 PM


8.46 | Analog Electronics

Vo 1 kΩ
(i)  Calculate the transfer function . Vz = 5 V
V1 1 kΩ
(ii) Plot the amplitude and phase response as a
function for R = R1. Vi −

Solution: +
1 kΩ
(i) Apply KCL at b
  (i)  V1 ≤ 0
 Vi − Vb  Vb
 = . (ii)  0 < V1 < 5 V
   1  R (iii)  V1 > 5 V
 SC 
(b) Sketch the gain (Vo Vs Vi ) characteristics of the
(Vi − Vb ) SCR = Vb .
above circuit and label the salient features.
 1 + SCR  Solution: (a)
  Vi = Vb  .
 SCR  (i)  When Vi ≤ 0, Zener diode is forward biased
 SCR  and is replaced by SC. Then equivalent circuit
 ⇒ Vb = Vi  .
 1 + SCR  1 kΩ
 Apply KCL at a 1 kΩ
 V − Va   Va − Vo  V1
I2 −
Vo
  i = . 1 kΩ +
 R1   R1  b
I1
Vo = 2Vo − Vi
   SCR  Va  0.
= 2Vi   − V1 .
 1 + SCR   Vb − Va  0.
V SCR − 1  Vb = Va = 0.
    o =  .
V1  SCR + 1 
 Apply KCL at a
Vo (ωCR ) 2 + 1 I = I1 + I 2
(ii) = = 1.
Vi (ωCR ) 2 + 1
Vo −Vi −Vi
 Phase plot is ∠θ = π − 2 tan (ωCR). −1  103 = 103 + 103 .
Vo = −2Vi .
Vo
  =1 ⇒ V
Vi   o = −2
Vi
(ii) With 0 < V1 < 5 V, Zener reverse biased and
current through it is zero. Then equivalent cir-
cuit is

f 1 kΩ

7. In the figure shown, assume that the Zener diode 1 kΩ


Vi −
and operational amplifier to be ideal [1991] Vo
+
(a) 
Draw the equivalent circuit and evaluate the
gain (Vo Vs Vi ) of the circuit for

Chapter 05.indd 46 11/2/2015 6:26:53 PM


Chapter 5  Operational Amplifiers and Applications  |  8.47

(iii) When V1 > 5 V, Zener operation in breakdown Apply KCL at a


region and is replaced by zener voltage. I1 + I 2 = I f .
Vi  Vi − 5  −Vo
1 kΩ + = .
5V 1 kΩ If 103  103  102
− 2Vi − 5 = −Vo .
a Vo
Vi
1 kΩ +
Vo = 5 − 2Vi .
I1
Hence, the correct option is (a).

Chapter 05.indd 47 11/2/2015 6:26:54 PM


Chapter 6
Feedback Amplifiers
and Oscillators
 Z2 
One-mark Questions Vb ( s ) = Vo ( s )  .
 Z1 + Z 2 
1. In the Wien Bridge oscillator circuit shown in 1  1 
­figure, the bridge is balanced when [2014-S1] Z1 = R1 + , Z 2 = R2||  .
SC1  SC2 
C1  1+
+ SC1 R1   R2 
Z1 =   , Z2 =  .
R1 +Vcc  SC1   1 + SC2 R2 
+ Vb ( s ) Z2
− =β = .
−Vcc
R3 Vo ( s ) Z1 + Z 2
R2
C2 R2 R4 β=
1 + SC2 R2
 1 + SC1 R1  R2
 +
R R 1  SC1  1 + SC2 R2
(a) 3 = 1 , ω = SC1 R2
R4 R2 R1C1 R2 C2 =
1 + SC2 R2 + SC1 R1 + S 2 C1C2 R1 R2 + SC1 R2
R C 1 1
(b) 2 = 2 , ω = β=
R1 C1 R1C1 R2 C2  R1 C2  1
R3 R1 C2 1 + +  + SC2 R1 +
1  R C1  SC1 R2
(c) = + ,ω= 2
R4 R2 C1 R1C1 R2 C2 Put S = jω
R R C 1 1
(d) 3 + 1 = 2 , ω = β= .
R4 R2 C1 R1C1 R2 C2  R1 C2   1 
1 + +  + j  ωC2 R1 − 
Solution: (c)  R C1   ω C1 R2 
2

Vd  0. To get undamped oscillations


Vb  Va . 1
ωC2 R1 = .
Vb ( s )  Va ( s ). ωC1 R2

Chapter 06.indd 48 11/12/2015 2:16:20 PM


Chapter 6  Feedback Amplifiers and Oscillators  |  8.49

1 9/100 in the feedback path. The overall system


ω2 = . gain is approximately. [2010]
R1 R2 C1C2
1 +
ω= . 100 ± 10%
R1 R2 C1C2 −

For balanced condition 9/100
 R  1
Aβ = 1, A = 1 + 3 , A= . 10 ± 2%
β (a) 10 ± 1% (b)
 R4 
R R C (c) 10 ± 5% (d)
10 ± 10%
1+ 3 = 1+ 1 + 2 . Solution: (a)
R4 R2 C1
R3 R1 C2
= + . dA
R4 R2 C1 = 10%
A
Hence, the correct option is (c). 9
β= , A = 100
2. In the feedback network shown below, if the feed- 100
back factor k is increased, then the [2013] dAf 1 dA
= ×
Af 1+ β A A
+ + A0 + V out A
−V in V r − − Af =
1+ β A
1
= = 10 = 1%.
+ + 9
− k −
1+ × 10%
100
Vf = kV out
Hence, the correct option is (a).
4. The feedback used in the circuit shown figure can
(a) input impedance increases and output ­impedance
be described as [2004]
decreases
(b) input impedance increases and output imped- Vcc
ance also increases
(c) input impedance decreases and output i­ mpedance RC
also decreases RF C=a
Vo
(d) input impedance decreases and output i­ mpedance C
decreases
Solution: (a) RS RB RL
RC Ce
The given configuration is a voltage series feed-
back configuration. So, input impedance increases.

Rif = Ri (1 + Ao k ) (a) Shunt-Series feedback


And Then output impedance decreases. (b) Shunt-Shunt feedback
Ro (c) Series-Shunt feedback
Rof = (d) Series-Series feedback
1 + Ao k
Solution: (b)
Hence, the correct option is (a). Equivalent can be drawn out with input voltage
3. As shown in the figure, a negative feedback sys- comparison and current feedback. It is shunt-shunt
tem has an amplifier of gain 100 with ±10% toler- feedback.
ance in the forward path and an attenuator of value Hence, the correct option is (b).

Chapter 06.indd 49 11/12/2015 2:16:22 PM


8.50 | Analog Electronics

5. An OP-AMP has an open-loop gain of 105 and List-II


an open loop upper cutoff frequency of 10 Hz . If P. All DC voltages normal, Vo increases marginally
this OP-AMP is connected as an amplifier with a Q. Collector of TRZ at VCC , Vo = 0
closed loop gain of 100, then the new upper cut-off R. All DC voltages normal, gain of 2nd stage in-
­frequency is [2001] crease, Vo decrease
(a) 10 Hz (b) 100 Hz S. All DC voltage normal, Vo = 0
10 kHz (d)
(c) 100 kHz T. All DC voltages normal, overall gain of the
Solution: (c) amplifier increases, Vo increase
U. No change
AOL = 105. Solution:
f = 10 Hz. A→S
B→R
AOL
ACL = . C →T
1 + β AOL
D→Q
105
(1 + β AOL ) = 2
= 103. 8. A Wien bridge oscillator is shown in figure. Which
10
of the following statements are true if f is the
f 2′ = f 2 (1 + β AOL ) = 10 × 103 Hz = 10 kHz. ­frequency of oscillation? [1993]
Hence, the correct option is (c).
6. A practical R.C. sinusoidal oscillator is built using
R R1 R2
a positive feedback amplifier with a closed loop
gain slightly less than unity. (TRUE/FALSE) −
C Vo
 [1994] +
Solution:  False
R C
7. Given figure shows a two stage small signal tran-
sistor feedback amplifier. Match the defective  
component (listed on the left hand side below) with
its probable effect on the circuit (listed below). 1
 [1994] (a) For R = 1 kΩ, C = F, f = 1 kHz.

Vcc
1
R C1 R C2 (b) For R = 3 kΩ, C = F, f = 3 kHz.
R1 R1 18π
RS C 1 C2 C5
TR1 TR 2 Vo (c) The gain of the OP-AMP stage should be less
∼ V
than two for proper operation
S R2 R2
R C1 R C2 C3
(d) The gain of the OP-AMP stage should be three
C4 for proper operation.
RF Solution: (a), (b) and (d)
1
R = 1 kΩ, C = F, f = 1 kHz.
List-I 2π
A. Capacitor C1 is open 1
R = 3 kΩ, C = F, f = 3 kHz.
B. Capacitor C3 is open 18π
The gain of OP-AMP should be there for proper
C. Capacitor C4 is open ­operation.
D. RC2 is shorted Hence, the correct options are (a), (b) and (d).

Chapter 06.indd 50 11/12/2015 2:16:24 PM


Chapter 6  Feedback Amplifiers and Oscillators  |  8.51

9. In a common Emitter amplifier, the unbypassed 1


emitter resistance provides [1992] (a) Low pass filter with f 3dB = RAD/s
( R1 + R2 )C
(a) Voltage-Shunt feedback
(b) Current-Series feedback 1
(b) High pass filter with f 3dB = RAD/s
(c) Negative-Voltage feedback R1C
(d) Positive-Current feedback 1
(c) Low pass filter with f 3dB = RAD/s
Solution: (b) R1C
 The unbypassed emitter resistance provides 1
­current-series feedback. (d) High pass filter with f 3dB = RAD/s
( R1 + R2 )C
Hence, the correct option is (b). Solution: (b)
1 1 + SCR1
Two-marks Questions Z = R1 + = .
SC SC
V (s) R
1. An oscillator circuit using ideal OP-AMP and H (s) = o =− 2
diode is shown in the figure. [2014-S2] V1 ( s ) Z
 
R  
+5 V SCR2 R S
− =− =− 2  .
C + Vo 1 + SCR1 R1  S + 1 
−5 V 
3 kΩ R1C 

R 
1 kΩ
1 kΩ H ( jω) increases from 0 at ω = 0 to  2 
Given circuit is high pass filter with  R1 
1
The time duration for +ve part of the cycle Δt1 f 3dB = Hz.
∆t1 − ∆t2 2π R1C
and for –ve part is Δt2. The value of e will be RC
Hence, the correct option is (b).
.
Solution: Vc = V f + (Vi − V f )e − t /τ
When OP-AMP is operated in the saturation region
V f = 5 V, Vi = −2.5 V at t = ∆t1
Vo = 5 V → D1 − ON, D2 − OFF
1 Vc = 1.25 V, τ = RC.
Vb = Vo   = 1.25 V.
4
1.25 = 5 + (−2.5 − 5)e − ∆t / RC .
When op-amp is operated in −ve saturation region
Vo = −5 V → D1 − OFF, D2 − ON. 1
e − ∆t1 / RC = = 0.5.
1 2
Vb = (−5)   = −2.5 V.
2 ∆t2 :
Δt1:
Voltage across capacitor V f = −5 V, Vt = 1.25 V at t = ∆t2 .
2. The circuit is a  [2012] Vc = −2.5 V, τ = RC.
R2

− 2.5 = −5 + (6.25)e − ∆t2 / RC .


+5 V
C R1 +
+ − Output e ∆t2 / RC = 2.5.
Input + −

− e ∆t2 − ∆t1 / RC = 1.25.
−5 V

Chapter 06.indd 51 11/12/2015 2:16:27 PM


8.52 | Analog Electronics

Vy ( S ) = I 2 ( S ) R
Five-marks Questions
Vy ( S )
 Vy  = [(2 + 3SCR + S 2 C 2 R 2 − 1)].
1. Determine the transfer function   for the RC SCR
network shown in Figure 1.  Vx 
Vy ( S )
R C I 2 (S ) − .
R
Vx C R Vy Vy ( S ) SCR
= .
Vx ( S ) S C R + 3SCR + 1
2 2 2

Figure 1
Vy ( S ) 1
= .
RF Vx ( S )  1 
1 kΩ
 SCR + 3 + SCR 

Vo
+ Vt
β= .
Vo
C R
R C
1
β= .
1
SCR + 3 +
Figure 2 SCR

The network is used as a feedback circuit in an 1


= .
oscillator circuit shown in Figure 2 to generate  1 
3 + j  ωCR − 
sinusoidal oscillations. Assuming that the opera-  ωCR 
tional amplifier is ideal, determine the value of
RF for generating these oscillations. Also deter- 1
ωCR − = 0.
mine the oscillation frequency if {R} = 10 kΩ and ωCR
C = 100 PF.  [2002]
1 1
Solution: fo = = .
2π RC 2π × 10 × 103 × 100 × 10−12
Apply KVL to loop (1)
= 159.12 kHz.
 1  1
VX ( S ) = I1 ( S )  R +
SC  − I 2 ( S ) ⋅ SC (1)
 
1
Apply KVL to loop (2) Then β =
3
1  2 
− I1 ( S ) + I 2 (S )  R + = 0.
SC  SC  To get sustained oscillation AB = 1
1  2 + SCR 
I1 ( S ) = I 2 (S )  . (2) 1
SC  SCR  ⇒ A= =3
β
I1 ( S ) = I 2 ( S )[2 + SCR ].
RF
= 2.
Substitute (2) in (1) 103
 1 + SCR  1 ⇒ RF = 2 kΩ
 − I 2 (S ) ⋅
Vx ( S ) = I 2 ( S )(2 + SCR) 
 SC  SC
2. Determine the frequency of oscillation of the cir-
I2 ( S)
= [(1 + SCR)(2 + SCR) − 1]. cuit shown in figure. Assume the OP-AMP to be
SC ideal [1997]

Chapter 06.indd 52 11/12/2015 2:17:28 PM


Chapter 6  Feedback Amplifiers and Oscillators  |  8.53

R 2 = 1 kΩ R 1 = 2 kΩ Solution:
 Z2 
Vb = Vo  .

 Z1 + Z 2 
+
R = 1 kΩ  1 
R 
SC  R
C = 4.7 µF Z2 =  = .
R+
1 1 + SCR
R = 1 kΩ C = 4.7 µF SC
1  1 + SCR 
Z1 = R + = .
SC  SC 
 RSC 
Solution: Vb = Vo  .
 (1 + SCR ) + SCR 
2
1
fo =
2π RC  SCR 
Vb = V f = Vo  2 2 2 
.
 1 + 3SCR + S C R 
Given that R = 1 kΩ
Vf 1
C = 4.7 µF. =β = .
Vo  1 
1  3 + SCR + 
fo =  SCR 
2π × 10 × 4.7 × 10− 6
3
Put S = jω
f o = 33.8627 Hz.
1
β= .
3. Show that the ckt given in fig. will work as an  1 
3 +  ω RC − 
1  ω RC 
oscillator at f = .
2π RC 1 1
ω RC − =0 ⇒ f = .
If R f = 2 R2 ω RC 2π RC
1
β= .
3
R2 R1
− Aβ = 1.
+ Vo
Amin = 3.
R1
R C
1+ = 3.
C R R2
R1
= 2.
R2

⇒ R1 = 2 R2 .

Chapter 06.indd 53 11/12/2015 2:17:29 PM


Chapter 7
Function Generator
and 555 Timer

1 TON1
Two-marks Questions (c) f = ,D=
TON1 TON1 + TON2
1. The truth of a monoshot shown in the figure is
given in the table below: 1 TON1
f =
(d) ,D=
X Y Q Q TON1 TON1 + TON2
0
1 Solution: (a)
Monostable 1 is a −ve edge triggered and mon-
R C
ostable 2 is a +ve edge triggered. The waveforms
at Q1 , Q2 and Q2 are as follows
X Q
T ON = 0.7 RC
Y Q

Two monoshots, one positive edge triggered and


Q1 ⇒
other negative edge triggered, are connected as
shown in the figure. The pulse widths of the two QP ⇒
monoshot options, Q1 and Q2 are TON1 and TON 2
Q2 ⇒
respectively.
R C R C

X Q1 X Q2 1 TON1
T ON = 0.7 RC
f = ; D= × 100.
Y T ON = 0.7 RC TON1 + TON 2 TON1 + TON 2
+5 V Q2
Q1
Hence, the correct option is (a).
The frequency and the duty cycle of the signal at
2. IC 555 in the adjacent figure is configured as an
Q1 will respectively be [2008]
Astable multivibrator. It is enabled to oscillator at
1 TON1 t = 0 by applying a high input to pin 4. The pin
(a) f = ,D= description is 1 and 8-supply; 2-trigger; 4-reset;
TON1 + TON2 TON1 + TON2
6-threshold; 7-discharge. The wave form appear-
1 TON2 ing across the capacitor starting from t = 0 as
f =
(b) ,D=
TON1 + TON2 TON1 + TON2 observed on the storage CRO is [2007]

Chapter 07.indd 54 11/12/2015 10:45:31 AM


Chapter 7  Function Generator and 555 Timer  |  8.55

+
for a frequency of 10 kHz and a duty cycle of 0.75
RA for the output waveform are [2003]
10 kΩ 8
7 Vcc
RB IC 555 3
10 kΩ
2, 6 RA
4 TH
C 555
1 V out
RB Timer
TR
R1
 (a)   (b)  C

RA = 3.62 kΩ, RB = 3.62 kΩ


(a)

 (c)   (d)  RA = 3.62 kΩ, RB = 7.25 kΩ


(b)
RA = 7.25 kΩ, RB = 3.62 kΩ
(c)
RA = 7.25 kΩ, RB = 7.25 kΩ
(d)
Solution: (c)
TON
Solution: (a) Duty cycle = α = .
An astable multi-vibrator is providing pulses as T
−4
 TON = α T = 0.75 × 10 = 75 µs.
given below:

75 × 10− 6
= RA + RB
But in this case initial voltage at capacitor is zero 0.7 × 10−8
so it starts from zero. Also, charging time will be ∴ RA + RB = 10.714 kΩ
larger (normally) than discharging time but it is ∴ TOFF = 0.7 CRB
made equal by using a diode.
25 × 10− 6
Hence, the correct option is (a). ∴ RB =
0.7 × 10−8
3. The circuit of fig shows a 555 timer IC connected
RB = 3.57 kΩ, RA = 7.14 kΩ.
as an Astable multivibrator. The value of capacitor
‘C’ is 10 nF . The value of the resistors RA and RB Hence, the correct option is (c).

Chapter 07.indd 55 11/12/2015 10:45:34 AM


This page is intentionally left blank.

Chapter 07.indd 56 11/12/2015 10:45:34 AM


Unit 9
Power Electronics
and Drives
Chapter 1:  Basics and Power Semiconductor Devices 9.3
Chapter 2:  AC–DC Converters 9.12
Chapter 3:  DC–DC Converters 9.25
Chapter 4:  DC–AC Converters 9.35
Chapter 5:  AC–AC Converters 9.42
Chapter 6:  Fundamentals of Drives 9.45

Chapter 01.indd 1 11/12/2015 10:49:52 AM


Chapter 01.indd 2
Exam Analysis
Exam Year 92 93 94 95 96 97 98 99 00 01 02 03 04 05 06 07 08 09 10 11 12 13 14
1 Mark Questions 1 3 7 7 5 2 3 2 3 1 2 4 4 3 2 4 2 1 3 2 2 – 2
2 Marks Questions – – – – – – 2 – 2 2 4 3 6 5 8 7 8 4 1 3 2 6 6
5 Marks Questions – – – 1 1 2 2 1 2 2 1 – – – – – – – – – – – –
Total Marks 1 3 7 8 6 4 7 3 7 5 7 7 10 8 10 11 10 5 4 5 4 6 8
Basics and Power Semiconductor Devices – 1 2 1 3 1 3 – – – – 2 3 3 1 2 – 2 1 1 1 1 2
AC–DC Converters 1 – 2 2 1 2 2 2 2 2 3 2 2 2 4 4 4 1 1 2 1 – 2
DC–DC Converters – 1 1 2 1 – – – 1 1 3 1 2 1 3 1 1 1 2 1 1 1 2
DC–AC Converters – 1 1 3 – 1 – 1 2 1 1 1 – 1 1 3 2 9 – 1 1 3 1
AC–AC Converters – – 1 – – – – – 1 – – – 1 – – – 1 – – – – – 1
Fundamentals of Drives – – – – 1 – 2 – 1 1 – 1 2 1 1 1 2 – – – – 1 –

11/12/2015 10:49:52 AM
Chapter 1
Basics and Power
Semiconductor Devices
2. The MOSFET switch in its on-state may be con-
One-mark Questions sidered equivalent to [2014, 1998]
(a) resistor (b) inductor
1. Figure shows four electronic switches (i), (ii), (iii) (c) capacitor (d) battery
and (iv). Which of the switches can block voltages
Solution: (a)
of either polarity (applied between terminals a and
b) when the active device is in the OFF state? MOSFET in its on state gives a variable resistance
[2014-S1] depending upon the magnitude of gate voltage.
a a a a Therefore, it can be treated as a resistor.
Hence, the correct option is (a).
3. The typical ratio of latching current to holding
current in a 20 A thyristor is [2012]
(a) 5.0 (b) 2.0
(c) 1.0 (d) 0.5
Solution: (b)
Typical ratio of latching current to holding current
lies in the range of 2.5 to 3. Therefore, only 2.0 is
b   b  b  b the nearest option.
(i)   (ii)   (iii) (iv)
Hence, the correct option is (b).
(a) (i), (ii) and (iii) (b) (ii), (iii) and (iv)
(c) (ii) and (iii) (d) (i) and (iv) 4. Circuit turn-off time of an SCR is defined as the
time [2011]
Solution: (c)
Thyristor can block both forward and reverse volt- (a) taken by the SCR turn off
age in the off state. (b) required for the SCR current to become zero
(c) for which the SCR is reverse biased by the
Now, for the circuit given below, transistor will
commutation circuit
block the forward voltage in the off state whereas
diode will block the reverse voltage. (d) for which the SCR is reverse biased to reduce
its current below the holding current
Therefore, options (ii) and (iii) satisfy the given
condition. Solution: (c)
Hence, the correct option is (c).
Hence, the correct option is (c).

Chapter 01.indd 3 11/12/2015 10:49:52 AM


9.4 | Power Electronics and Drives

5. Figure shows a composite switch consisting of (c) a rectangular hyperbola


a power transistor (BJT) in series with a diode. (d) an exponentially decaying function
Assume that the transistor switch and the diode
Solution: (a)
are ideal. The I–V characteristic of the composite
The conduction loss in a MOSFET is due to the
switch is [2010]
variable resistance path during the on-state.
+ V −
∴ Ploss = I ds2 ( Rchannel + Rdevice )
I
= I ds2 RON .

∴  Conduction loss versus device current graph


(a) I (b) I will be a parabola.
Hence, the correct option is (a).
V
8. A bipolar junction transistor (BJT) is used as a
V
power control switch by biasing it in cut-off region
(OFF state) or in the saturation region (ON state).
In the ON state, for the BJT, [2004]
(c) I (d) I (a) both the base–emitter and base–collector junc-
tions are reverse biased
(b) the base–emitter junction is reverse biased, and
V V the base–collector junction is forward biased
(c) the base–emitter junction is forward biased, and
the base–collector junction is reverse biased
(d) both the base–emitter and base–collector junc-
Solution: (c) tions are forward biased.
Hence, the correct option is (c). Solution: (d)
RL
6. An SCR is considered to be a semi controlled
C IC
device because [2009]
+
(a) it can be turned OFF but not ON with a gate B VCE VCC
pulse IB


(b) it conducts only during one half-cycle of an E
alternating current wave
(c) it can be turned ON but not OFF with a gate Cut off region: Both base–emitter and base–collec-
pulse tor junctions are reverse biased.
(d) it can be turned ON only during one half-cycle Saturation region: Both base–emitter and base–
of an alternating voltage wave collector junctions are forward biased.
Solution: (c) Hence, the correct option is (d).
Application of negative gate pulse to a conduct- 9. The triggering circuit of a thyristor is shown in fig-
ing thyristor cannot turn-off the device. Therefore, ure. The thyristor requires a gate current of 10 mA,
only the turn-on of the SCR can be controlled us- for guaranteed turn-on. The value of R required for
ing a gate pulse. Hence, it is called a semi con- the thyristor to turn on reliably under all conditions
trolled switch. of Vb variation is [2004]
Hence, the correct option is (c). Load
7. The conduction loss versus device current charac- 100 V
teristic of a power MOSFET is best approximated R
by [2005]
(a) a parabola Vb = 12 ± 4 V
(b) a straight line

Chapter 01.indd 4 11/12/2015 10:49:53 AM


Chapter 1  Basics and Power Semiconductor Devices  |  9.5

(a) 10000 Ω (b) 1600 Ω switching device in the ON and OFF states through
(c) 1200 Ω (d) 800 Ω appropriate control. The ON and OFF states of the
Solution: (d) switch are given on the Vds – Is plane by [2003]
Thyristor requires a gate current of 10 mA to turn D
on. Thyristor will turn on under any voltage condi-
tions only when minimum gate voltage can pro-
duce the necessary gate current. VDS
G
∴ Vb min = 12 − 4 = 8 V. S
IS
Applying KVL in the gate circuit, we get,Vb min = IgR.
(a) IS (c) IS
Vb min
⇒ R=
Ig
VDS VDS
8
=
10 × 10−3
= 800 Ω. (c) IS (d) IS

Hence, the correct option is (d).


VDS VDS
1 0. Figure shows a thyristor with the standard termina-
tions of anode (A), cathode (K), gate (G) and the
different junctions named J1, J2 and J3. When the
thyristor is turned on and conducting [2003] Solution: (b)

A D

P J3 VDS
G
n J2 S
G IS
P J1

The above circuit can block forward voltage when


K in off state. It can conduct when a positive gate
voltage is applied. In the reverse direction, the de-
(a) J1 and J2 are forward biased and J3 is reverse
vice conducts and is unable to block reverse volt-
biased
age due to the presence of diode.
(b) J1 and J3 are forward biased and J2 is reverse
biased Hence, the correct option is (b).
(c) J1 is forward biased and J2 and J3 are reverse 12. The uncontrolled electronic switch employed in
biased power electronic converters is [1998]
(d) J1, J2 and J3 are all forward biased (a) Thyristor   (b)  Bipolar junction transistor
Solution: (d) (c) Diode      (d) MOSFET
When an SCR is in the on-state or in conduction Solution: (c)
state, all the junctions are forward biased. Amongst the given devices, diode is the only
Hence, the correct option is (d). device that can be switched on by just applying
11. Figure shows a MOSFET with an integral body forward voltage across it. Therefore, diode is an
diode. It is employed as a power switching device uncontrolled switch.
in the ON and OFF states through as a power Hence, the correct option is (c).

Chapter 01.indd 5 11/12/2015 10:49:53 AM


9.6 | Power Electronics and Drives

13. In a commutation circuit employed to turn off an Solution: (d)


SCR, satisfactory turn-off is obtained when TRIAC can be operated in all 4 modes of opera-
[1998] tion, i.e., forward blocking, forward conduction,
(a) circuit turn-off time < device turn-off time reverse blocking and reverse conduction.
(b) circuit turn-off time > device turn-off time This property is suitable only for application to
(c) circuit time constant > device turn-off time cycloconverters.
(d) circuit time constant < device turn-off time
Hence, the correct option is (d).
Solution: (b)
17. Which of the following does not cause permanent
 For a commutation circuit employed to turn off
damage of an SCR? [1996]
an SCR, the device can regain its forward voltage
(a) High current
blocking capability only when all the minority carri-
(b) High rate of rise of current
ers are swept out of the junction. This happens only
(c) High temperature rise
when circuit turn-off time > Device turn-off time.
(d) High rate of rise of voltage
Hence, the correct option is (b).
Solution: (d)
14. If a diode is connected in anti-parallel with a thy-
High rate of rise of voltage can lead to production
ristor, then [1997]
of charging current that arises due to the presence
(a)  both turn-off power loss and turn-off time
of stray junction capacitance. It is mathematically
decreases.
given by,
(b)  turn-off power loss decreases, but turn-off
time increases. dV
I c = CJ .
(c) turn-off power loss increases, but turn-off time dt
decreases. Therefore, this current can lead to false triggering
(d) None of me above. of the SCR. It cannot lead to damage of a SCR.
Solution: (b) Hence, the correct option is (d).
During turn-off period, diode will be conducting.
18. Figure shows two thyristors each rated 500 A (con-
Therefore, reverse voltage across the thyristor will
tinuous) sharing a load current. Current through
be the on state voltages drop of the diode (which is
thyristor y is 120 A. The current through thyristor
very less). Since, diode (in conduction) provides a
x will be nearly_______A. [1995]
low impedance path for current, therefore most of
the current conducts through diode due to which 0.06
recombination process in SCR will be slow. There-
x
fore, toff will be more. Since, voltage drop is very
less; therefore, power loss will also be very less. 0.05

Hence, the correct option is (b).


LOAD

y 120 A
1 5. Which semiconductor power device out of the fol- +
E
lowing is not a current triggered device? [1996] −
(a) Thyristor (b) GTO
(c) TRIAC (d) MOSFET
Solution: (d) Solution:
0.06
MOSFET gains the ability to conduct only when
appropriate voltage is applied to its gate terminal. x Ix
Therefore, MOSFET is a voltage controlled device. 0.05
Hence, the correct option is (d).
LOAD

y 120 A
16. The TRIAC can be used only in [1996]
(a) inverter (b) rectifier
(c) multiquadrant chopper (d)  cycloconverter

Chapter 01.indd 6 11/12/2015 10:49:54 AM


Chapter 1  Basics and Power Semiconductor Devices  |  9.7

Assuming there is no voltage drop across thyris- (c) average power dissipated in the device divided
tor during conduction, then voltage across the by the temperature difference from body to
two branches containing thyristor x and y must be ambient.
equal. (d) temperature difference from body to ambient
divided by average power dissipated in the
device.
⇒ Vx = Vy
Solution: (d)
⇒ I x × 0.06 + (0) = 120 × 0.05 + (0)
120 × 5
⇒ Ix = = 100 A
6

J C S A
P av Power flow
19. A switched mode power supply operating at 20 kHz TJ TC TS TA
to 100 kHz range uses as the main switching
­element [1994]
(a) Thysistor (b) MOSFET
(c) TRIAC (d) UJT
Solution: (b) J → Junction TJ → Junction temperature.
MOSFET can be used for applications requiring C → Case TC → Case temperature.
switching frequencies up to 1 mHz. S → Sink TS → Sink temperature.
Hence, the correct option is (b). A → Ambient TA → Ambient temperature.
20. A TRIAC can be triggered by a gate pulse of Thermal resistances between different layers:
polarity. [1994] θJC → Junction to case.
Solution: θCS → Case to sink.
TRIAC: θSA → Sink to ambient.
Thermal equivalent model of heat sink:

MT1 J qJC C qCS S qCS A


TJ TC TS TA

P av P av
G
MT2

Pav is analogous to current.


Therefore, a TRIAC can be trigerred by a gate
TJ − TA T − TA
pube of either positive or negative polarity. Pav ( w) = = J .
θ JC + θCS + θ SA θ JA
21. The thermal resistance between the body of a
power semiconductor device and the ambient is
Pav
expressed as [1993] ⇒ θ JA = ,
(a) voltage across the device divided by current TJ − TA
through the device.
(b) average power dissipated in the device divided where θJA = θJC + θCS + θSA.
by the square of the rms current in the device. Hence, the correct option is (d).

Chapter 01.indd 7 11/12/2015 10:49:55 AM


9.8 | Power Electronics and Drives

T L
Two-marks Questions +

1. The SCR in the circuit shown has a latching cur- 15 V C


rent of 40 mA. A gate pulse of 50 µs is applied to −
the SCR. The maximum value of R in Ω to ensure
successful firing of the SCR is [2014-S2] (a) 10 µs (b) 50 s
SCR (c) 100 µs (d) 200 µs
Solution: (c)
When the thyristor along with LC combination is
500 Ω
+
supplied from a DC supply, the nature of current
100 V R is sinusoidal. Since, thyristor conducts only in
one direction, conduction time is half of one full
200 mH cycle.

T = 2π LC .
Solution:
SCR T 100 × 100
⇒ t= = π LC = π = 100 µsec.
2 π2
Hence, the correct option is (c).
500 Ω R1
+ 3. Match the switch arrangements on the top row to
100 V R be steady-state V–I characteristics on the lower
row. The steady state operating points are shown
200 mH IL
by large black dots. [2009]

Current for above circuit is given by + − + −



V V (a)      (b)
i= (1 − e − t / ( L / R1 ) ) + DC .
R1 R

tw = Gate pulse width = 50 μs


Latching current = 40 mA = iL.
For successful firing circuit current must be equal
+ − + −
to the latching current.  
V V
∴ i = iL = (1 − e − t / ( L / R1 ) ) + .
R1 R (a) IS (b) IS

⇒ R = 6060.8 Ω.
VS VS
The answer is 6060.8 Ω.
2. Thyristor T in the figure is initially off and is
triggered with a single pulse of width 10 µs.
(b) IS (d) IS
 100   100 
It is given that L =   µH, C =   µF.
 π   π 
Assuming latching and holding currents of the VS VS
thyristor are both zero and the initial charge on C
is zero, T conducts for  [2013]

Chapter 01.indd 8 11/12/2015 10:49:57 AM


Chapter 1  Basics and Power Semiconductor Devices  |  9.9

(a) A - i, B - ii, C - iii, D - iv 5. The minimum approximate volt-second rating of


(b) A - ii, B - iv, C - i, D - iii the pulse transformer suitable for triggering the
(c) A - iv, B - iii, C - i, D - ii SCR should be: (Volt-second rating is the maxi-
(d) A - iv, B - iii, C - ii, D - i mum of product of the voltage and the width of the
Solution: (c) pulse that may be applied) [2007]
(a) 2000 µ V-s (b) 200 µ V-s
Hence, the correct option is (c).
(c) 20 µ V-s (d) 2 µ V-s
Common Data for Questions 4 and 5:
Solution: (a)
 A 1:1 pulse transformer (PT) is used to trigger the
Firstly, we must calculate the width of the gate
SCR in the adjacent figure. The SCR is rated at
pulse. This can be calculated by determining the
15 kV, 250 A with IL = 250 mA, In = 150 mA, and
circuit current and equating it to latching current.
IGmax = 150 mA with IL = 250 mA, IG m m = 100 mA.
The SCR is connected to an inductive load, where SCR current is given by,
L = 150 mH in series with a small resistance and
the supply voltage is 200 V DC. The forward drop V − Rt

i=  1 − e L
.
of all transistors/diodes and gate-cathode junction R 
during ON state is 1.0 V
200  −1×t

4. The resistance R should be [2007] iL = i = 1 − e 0.15  .
1  
1.0 Ω ⇒ t = 187µ sec.

∴  Voltage second rating of the pulse transformer



R S L
PT is given by
+10 V C
Vt = 10 × 187 µ sec
R = 1870 µ sec V
+ 200 V
≈ 2000 µV − sec.

Hence, the correct option is (a).


6. An SCR having a turn ON time of 5 µ sec, latching
current of 50 mA and holding current of 40 mA
(a) 4.7 kQ (b) 470 kQ is triggered by a short duration pulse and is used
(c) 47 Q (d) 4.7 Q in the circuit shown in figure The minimum pulse
width required to turn the SCR ON will be[2006]
Solution: (c)
We are given the maximum gate current Ig max
= 150 mA. The firing circuit should be such that
the gate current is always less than maximum gate 2Ω
current.
100 V 5 kΩ
Applying KVL equations in the gate circuit, we
get, 0.5 H

−10 + (150 × 10−3 ) R + 1 + 1 + 1 = 0.


⇒ R = 0.0467 × 103 Ω (a) 251 µ sec (b) 150 µ sec
= 46.7 Ω (c) 100 µ sec (d) 5 µ sec
≈ 47 Ω. Solution: (b)

Hence, the correct option is (c). Given: TON = 5 µ sec, I L = 50 mA, I H = 40 mA.

Chapter 01.indd 9 11/12/2015 1:26:26 PM


9.10 | Power Electronics and Drives

Energy loss = V ×  × I s × t1  + I  × V × t2 
1 1
i
2  2 
2Ω
VI
100 V 5 kΩ = (t1 + t2 ).
2
0.5 H Hence, the correct option is (a).
8. An electronics switch S is required to block volt-
age of either polarity during its OFF state as shown
VS  V in the figure (a). This switch is required to conduct
1 − e− R / L  + S
t
i= [Application of KVL] in only one direction its ON state as shown in the
R  R
figure (b). [2005]
100  100
1 − e −20 / 0.5  +
t
= . S
20   5000 I J′
±
±

−3 −40 t 1 (a)
50 × 10 = 5(1 − e )+ .
50 I i I′
⇒ t = 150 µsec.
(b)
Hence, the correct option is (b).
7. The figure shows the voltage across a power Which of the following are valid realizations of the
semiconductor device and the current through the switch S?
device during a switching transition. Is the transi- I I′
(1)
tion a turn ON transition or a turn OFF ­transition?
What is the energy lost during the transition?
 [2005] I
(2) I′

v,i
V
v
I
(3)
i

t I I′
t1 t2 (4)

VI
(a) Turn ON, (t1 + t2 )
2 (a) Only 1 (b) 1 and 2
(b) Turn OFF, VI (t1 + t2) (c) 1 and 3 (d) 3 and4
(c) Turn ON, VI (t1 + t2) Solution: (c)
The switch representation shown in diagram de-
VI
(d) Turn OFF, (t1 + t2 ) scribes forward and reverse voltage blocking and
2 forward conduction modes.
Solution: (a)
The energy lost can be determined by calculating
the energy loss during the transient period (i.e.,  
0 < i < I and 0 < v < v ). Since, during the tran-
sition period, voltage reduces to zero and current Above two circuit configurations exhibit the re-
increases to the rated value, the graph represents quired modes of operation.
Q turn on transition. Hence, the correct option is (c).

Chapter 01.indd 10 11/12/2015 1:26:28 PM


Chapter 1  Basics and Power Semiconductor Devices  |  9.11

9. A MOSFET rated for 15 A, carries a periodic cur- Solution: (c)


rent as shown in figure. The ON state resistance of
RON = 0.15 Ω.
the MOSFET is 0.15 Q. The average ON state loss
in the MOSFET is [2004] I = 15 A.

10 A Energy loss in total time period 1



Pavg = = ∫i R dt
2

Time period T 0

0 π 2π 3π
wt
1 π 2π

= × 0.5  ∫ (15) 2 dt + ∫ (0) 2 dt  = 7.5 W.
2π 0 π 
(a) 33.8 W (b) 15.0 W
(c) 7.5 W (d) 3.8 W Hence, the correct option is (c).

Chapter 01.indd 11 11/12/2015 10:50:00 AM


Chapter 2
AC–DC Converters
+
One-mark Questions
1. A half-controlled single-phase bridge rectifier is VDC
supplying an R-L load. It is operated at a firing
angle a and the load current is continuous. The −
fraction of cycle that the freewheeling diode con-
ducts is [2012] (a) 150 V (b) 210 V
1  α (c) 300 V (d) 100 p V
(a) (b) 1− 
2  π Solution: (a)
α α For α = 0°, the fully controlled thyristor converter
(c) (d) is active as a diode converter. The output voltage is
2π π
2V
given by Vo = m = 300 V.
Solution: (d) π
For α = 60°, the output voltage
2V
V = m cos α  = 300 cos 60° = 150 V.
Io o π
Hence, the correct option is (a).
3. A single-phase fully controlled thyristor bridge
a a AC–DC converter is operating at a firing angle of
25° and an overlap angle of 10° constant DC output
The freewheeling diodes conduct till the thyristors current of 20 A. The fundamental power factor (dis-
are not fired. So this happens in the interval 0 to α placement factor) at input AC mains is [2007]
and π to π + α. So total interval = 2α. (a) 0.78 (b) 0.827
2α α (c) 0.866 (d) 0.9
Ratio  = = .
2π π Solution: (c)
Hence, the correct option is (d).
Vs
2. The fully controlled thyristor converter in the fig-
ure is fed from a single-phase source. When the T1 wt
T3 Io
firing angle is 0°, the DC output voltage of the Vo
converter is 300 V. What will be the output ­voltage
T4 T2 wt
for a firing angle of 60°, assuming continuous
25°u 5° $
­conduction? [2010]  

Chapter 02.indd 12 11/9/2015 6:48:29 PM


Chapter 2  AC–DC Converters  |  9.13

The source current is shown in the figure. 6. A three-phase diode bridge rectifier is fed from a
The power factor 400 V rms, 50 Hz, three-phase AC source. If the
load is purely resistive, then peak instantaneous
4  3
= cos  + α  = cos(5 + 25°) = = 0.866. output voltage is equal to [2005]
2  2
(a) 400 V (b) 400 2 V
Hence, the correct option is (c).
2 400
4. A single phase full-wave half controlled bridge con- (c)
400 V (d) V
verter feeds an inductive load. The two SCRs in the 3 3
converter are connected to a common DC bus. The Solution: (b)
converter has to have a freewheeling diode [2007] The peak output voltage is the peak value of the
(a) because the converter inherently does not pro- input voltage, as that is what the output voltage.
vide for freewheeling.
Peak output voltage = 2 × 400 V.
(b) because the converter does not provide for free-
wheeling for high values of triggering angles. Hence, the correct option is (b).
(c) or else the freewheeling action of the converter 7. The circuit in figure shows a full-wave rectifier.
will cause shorting of the AC triggering angles. The input voltage is 230 V (rms) single-phase AC.
(d) or else if a gate pulse to one of the SCRs is The peak reverse voltage across the diodes Dl and
missed, it will subsequently cause a high load D2 is [2004]
current in the other SCR.
Solution: (a)
D1
Hence, the correct option is (a).
230 V
5. A single-phase half wave uncontrolled converter
50 Hz
circuit is shown in figure. A 2-winding transformer
AC
is used at the input for isolation. Assuming the load
D2
current to be constant and Vin = Vm sinw, the current
waveform through diode D2 will be [2006]
230 V/50-0-50 V
D1 iL L

(a)
100 2 V (b) 100 V
D2
V in V (c)
50 2 V (d) 50 V

Solution: (a)
The peak reverse voltage across the diode D1 and
(a)  wt  (b) 
p
wt D2 is 2Vm where Vm = Peak value of each secondary
0 p 2p 0 2p
voltage and the secondary of centre tapped.
(c)  wt  (d)  wt Transformer rms voltages are 50 - 0 - 50.
p 2p 0 p 2p
Solution: (d) Vao = 50(rms).
D2 will act as freewheeling diode and will conduct
in the negative half cycle of the source. Vao
= =
(max) 50 2 V Vm .
If source inductance is considered. PIV Vm 100 2 .
= 2=
If source inductance is not considered.
Hence, the correct option is (a).
(B) 8. The circuit in figure shows a 3-phase half-wave
p 2p
rectifier. The source is a symmetrical, 3-phase
four-wire system. The line-to-line voltage of the
(D)
p 2p source is 100 V. The supply frequency is 400 Hz.
Hence, the correct option is (d). The ripple frequency at the output is [2004]

Chapter 02.indd 13 11/9/2015 6:48:30 PM


9.14 | Power Electronics and Drives

R = Vml − Vml sin (120 + 30°) = Vml − 0.5Vml


∆v
Y = 0.5.
R Vml
B Hence, the correct option is (a).
10. A six pulse thyristor rectifier bridge is connected to
N
a balanced 50 Hz three phase AC source. Assuming
(a) 400 Hz (b) 800 Hz that the DC output current of the rectifier is constant.
(c) 1200 Hz (d) 2400 Hz The lowest frequency harmonic component in the
Solution: (c) AC source line current is [2002]
The ripple frequency (a) 100 Hz (b) 150 Hz
(c) 250 Hz (d) 300 Hz
Solution: (c)
Six pulse thyristor (or) 3ϕ full wave bridge rectifier.
rms value of nth harmonic line current is given by
4la nπ
is = cos .
2nπ 6
The half wave rectifier (3 - ϕ) is a 6-3 pulse converter. For fundamental current (n = 1)
The output voltage frequency = 3 × source frequency
6
           = 3 × 400 = 1200 Hz. ⇒ Is = Ia .
π
Hence, the correct option is (c). For 3rd harmonic current (n = 3)
9. A fully controlled natural commutated 3-phase ⇒  Is = 0.
bridge rectifier is operating with a firing angle For 5th harmonic current (n = 5)
a = 30°. The peak to peak voltage ripple expressed 6
⇒ Is = − Ia .
as a ratio of the peak output DC voltage at the out- 5π
put of the converter bridge is [2003] The lowest existing harmonic is 5th harmonic and
3 frequency= is 5 f 5= (50 Hz) 250 Hz.
(a) 0.5 (b)
2 Hence, the correct option is (c).
3 11. AC to DC circulating current dual converters are
1−
(c) 3 −1
(d)
2 operated with the following relationship between
Solution: (a) their triggering angles (a1 and a2). [2001]
The output line voltage waveform of 3ϕ full wave (a) a1 + a2 = 180° (b) a1 + a2 = 360°
bridge rectifier is (c) a1 − a2 = 180° (d) a1 + a2 = 90°
V0 Vcb Vab Solution: (a)
Since, in a dual converter, one converter is in rec-
tifier mode and the other converter is in inverter
mode with equal output voltage with 180° phase
difference α1 + α 2 = 180°.
30° 60° 90° 120° 150°
wt Hence, the correct option is (a).
12. A thyristorised three phase fully controlled converter
Vab = Vml sin (ωt + 30). feeds a DC load that draws a constant current. Then
Peak to peak ripple voltage the input AC line current to the converter has [2000]
(a) an rms value equal to the DC load current.
∆V = Vmaxi − Vmini = Vml − Vmn . (b) an average value equal to the DC load current.
∆v = Vml − Vml sin (ωt + 30°). (c) a peak value equal to the DC load current.
(d) a fundamental frequency component, whose
Consider Vab line voltage rms value is equal to the DC load current.

Chapter 02.indd 14 11/9/2015 6:48:31 PM


Chapter 2  AC–DC Converters  |  9.15

Solution: (c) ID
The circuit for a 3-phase rectifier is = [2 3 cos α ].
π
2I a
bn = −
Similarly,  3 sin α .
io π
T1 T3 T5

∼ I 
I5 So, fundamental components =  D 2 3 cos α  sin ωt
∼  π 
T6 T4 T2
I D 3 cos α
and rms value = ≠ ID .
π 2
The line current can be shown in the following figure. Hence, the correct option is (c).
13. Resonant converters are basically used to [1999]
ID
(a) generate large peaky voltage.
(b) reduce the switching losses.
p +a 5p + a 7p + a
(c) eliminate harmonics.
−Is
6 6 6 (d) convert a square wave into a sine wave.
(wt) Solution: (a)
Resonant converters are basically used to generate
11π large peaky voltage.
The rms value of line current  +α
6 Hence, the correct option is (a).
1 
5π / 6 + π
 14. In a dual converter, the circulating current [1997]
I 2rms =  ∫ I D d (ω L) + ∫ (− I D ) d (ωt ) 
2 2
(a) allows smooth reversal of load current, but in-
2π  π /6 +α  creases the response time.
1  2  5π π  2 11π 7π   (b) does not allow smooth reversal of load current,
=
2π ID  6 − 6  + ID  6 − 6  . but reduces the response time.
        
(c) allows smooth reversal of load current with
2 improved speed of response.
I rms =
  I D ≠ I D  (Load current). (d) flows only if there is no interconnecting inductor.
3
Average value of line current = 0. Solution: (c)
The circulating current brings both the converter
The max value of line current = ID load current. in the dual converter in continuous conduction, the
The fundamental component is calculated as below: time responds for changing from one quadrant of
operation to another.
 5π / 6 +α 11π
+α 
1 6 Hence, the correct option is (c).

a1 =  ∫ I D sin ωt + ∫ − I D sin ωt d (ωt )  15. In a 3-phase controlled bridge rectifier, with an
π π


+α  increase of overlap angle the output DC voltage
 6 6
 [1996]
1   π   5π 
=  I D cos  + α  − cos  + α  + I D (a) decreases
π    6   6  (b) increases
  11π   7π  (c) does not change
cos  6 + α  − cos  6 + α   (d) depends upon load inductance
    
Solution: (a)
1  3 sin α 3 The overlap angle is due to the presence of source
= ID  cos α − + cos α
π   2 α α inductances. There is a presence of short circuit,
where three diodes conduct simultaneously and
sin α 3 sin α 3 sin α  during that period output voltage is zero, so there
+ + cos α − + cos α + 
α α α α α  is a decrease in output voltage.

Chapter 02.indd 15 11/9/2015 6:48:33 PM


9.16 | Power Electronics and Drives

3Vm 3 So, Vm = 2 × 440 V.


Vo = − ω Ls I d
π π
  3 × 2 × 440
↵decrease in output voltage So, Vo = = 594.20 V.
π
Where Id is the ripple free output current, LS is the
source inductance. 18. Referring to the figure the type of load is [1994]
Hence, the correct option is (a).
Vo
16. A single phase diode bridge rectifier supplies
a highly inductive load. The load current can be
assumed to be ripple free. The AC supply side cur- ∼
a
rent waveforms will be  [1995]
(a) sinusoidal (b) constant DC Vo
(c) square (d) triangular
Solution: (c) (a) inductive load (b) resistive load
The 1 - ϕ diode bridge rectifier supplies a highly (c) DC motor (d) capacitive load
inductive load. The load current can be assumed Solution: (c)
to be ripple tree. The AC side supply waveform is
square.
Vo
The circuit shown is
Vi
L
∼ O
is
A
D1 D3 I0 D Vo

D4 D2
The load shown in the figure is DC motor. Because
there is a value of back emf as the output volt-
is age does not cross zero. Also the negative voltage
(voltage less than back emf) is due to the induct-
ance of the motor.
Hence, the correct option is (c).
The positive current when D1 and D2 conduct, the 19. The output voltage of a six-pulse double star recti-
negative current D3 and D4 conduct. fier is the same as that of a three-phase half wave
Hence, the correct option is (c). rectifier. (TRUE/FALSE)[1994]
17. A three-phase AC to DC diode bridge rectifier is Solution:
supplying from a three-phase, 440 V source. The False, the statement is false because output of 3
rectifier supplies a purely resistive load. The aver- phase half wave rectifier is 3 pulse and not 6 pulse.
age DC voltage across the load will be ______V 20. In the circuit shown in figure, L is large and the
 [1995] average value of i is 100 A. The thyristor is gated
Solution: in the ——— half cycle of e at a delay angle a
The average output voltage, for a 3 phase bridge equal to —— e(t) = 2 .200 sin 314t [1992]
rectifier,
0.01 Ω L
π/6
2 3Vm
Vo =
π ∫0 Vm cos ωtd (ωt ) = π . +
i
+
20 V
3 e(t ) ∼ −

Now 440 V is the rms value of voltage.

Chapter 02.indd 16 11/9/2015 6:48:34 PM


Chapter 2  AC–DC Converters  |  9.17

Solution: 1
The thyristor can be gated only in the ‘positive’ So, 325[1 + cos α ] = 70°.

half cycle due to the presence of freewheeling ⇒  α = 69.310°.
diode. Due to freewheeling the constant current
through the inductor will flow through diode and 2. A fully controlled converter bridge feeds a highly
thyristor will be reverse biased in the negative half inductive load with ripple free load current. The
cycle and cannot be triggered. input supply (Vs) to the bridge is a sinusoidal
source. Triggering angle of the bridge converter is
The voltage balance equation by KVL a = 30°. The input power factor of the bridge is
Ldi  [2014-S2]
e(t ) = Ro + + E.
dt
Taking average of each term,
is
1  
π 2π
1 +
 ∫ e(t )  = RI + o +
2π ∫o
E , where I = average Vs ∼ Load
2π α  −
value of current.
Ldi
 [∵   Also average integral of over a period = o].
dt
π Solution:
1 20(2π − 0)
× ∫ 2 × 200 sin 314t = 0.01× 100 + 0 + . 3
2π α 2π = cos 30° =
DPF  .
2
1 I
× 2 × 200 [cos α − cos π ] = 1 + 20. Net power factor = o1 cos α .
2π Io
⇒ cos α = − 0.5334. Now we will calculate Io1.
α = 122.24°. 30 210 360
1
A ∫0 ∫ ∫ −I
a1 = − Io cos ωt + Io cos ωt + o cos ωt
30 210

Two-marks Questions =
1
Io [sin 0 − sin 30°] + Io [sin 210 − sin 30]
π
1. The figure shown is the circuit diagram of a rectifier. + [sin(210) − sin 360°]Io
The load consists of resistance 10 W and inductance
  −1   −2
0.05 H connected in series. Assuming ideal thyristor = Io  −1 +    = Io and − 2.
and ideal diode, the thyristor firing angle (in degree)   π  π
needed to obtain an average load voltage of 70 V is
1 
30 210 360
 [2014-S1]
π  ∫0 ∫ ∫ −I
b1 =  − Io sin ωt + Io sin ωt + o sin ωt d (ωt ) 
30 210 
+ 1
= [ Io [cos 30 − 1] + Io [cos 30 − cos 210]
325 sin(314t ) V ∼ Load π
− + Io [cos 360 − cos 210]]
3.464
= Io .
Solution: π π
1
2π α∫
Average output volt = 325 sin(314)td (314t ) 4
I s1 = Io .
1 2
  = 2π × 325 [1 + cos α ].
\ 4 3
[   Because of the presence of freewheeling diode PF = × = 0.7794 log.
2π 2
the thyristor does not conduct beyond ω t = π].

Chapter 02.indd 17 11/9/2015 6:48:37 PM


9.18 | Power Electronics and Drives

Common Data for Questions 3 and 4: Solution: (b)


The input voltage given to a converter is Max Volt
Max current =
V1 = 100 2  sin (100 p t) V. Resistance
The current drawn by the converter is 400
     =   
= 40 A.
 π 10
i1 = 10  2  sin  100 π t −  + 5 2 Hence, the correct option is (b).
 3
π π 6. The kVA rating of the input transformer is [2011]
 
sin  300π t +  + 2 2 sin  500π t −  A. (a) 53.2 kVA (b) 46.0 kVA
 4  6 (c) 22.6 kVA (d) None
3. The input power factor of the converter is [2011] Solution: (c)
(a) 0.31 (b) 0.44
kVA rating = 3Vrms I rms .
(c) 0.5 (d) 0.71
Solution: (b) Vrms = 400 V.
I 2 2
The input PF = s1 cos φ . =I rms I= 40 = 32.65.
Is    o
3 3
Is = Net rms value  = 102 + 52 + 22 = 11.35. kVA rating  = 3 × 400 × 32.65
I s1 = 10 cos φ = cos 60°.
= 22.612 kVA.
    
PF = 0.4405.
Hence, the correct option is (c).
Hence, the correct option is (b).
7. The circuit shows an ideal diode connected to a
4. The active power drawn by the converter is [2011] pure inductor and is connected to a purely sinusoi-
(a) 181 W (b) 500 W dal 50 Hz voltage source. Under ideal condition
(c) 707 W (d) 887 W the current waveform through the inductor will
Solution: (b) look like [2009]
Active power input = Vrms ( I s1 ) cos φ
D
1
    = 100 × 10 × = 500 W. + −
2 +
Vs = 10 sin100pt ∼ L = (0.1/p)H
Hence, the correct option is (b). −
  Common Data for Questions 5 and 6:
A solar energy installation utilize a three-phase
1.5
bridge converter to feed energy into power system
through a transformer of 400 V/400 V, as shown
1
Current

below (a)
Fillter Choke 0.5

∼ 0
0 10 20 30 40 50
Battery
1.5
The energy is collected in a bank of 400 V battery
and is connected to converter through a large filter 1
Current

choke of resistance 10 W.
(b) 0.5
5. The maximum current through the battery will be
 [2011]
0
(a) 14 A (b) 40 A 0 10 20 30 40 50
(c) 80 A (d) 94 A Time(ms)

Chapter 02.indd 18 11/9/2015 6:48:39 PM


Chapter 2  AC–DC Converters  |  9.19

1.5 I
= o (1⋅ s ) = − 0.477 I o .
   π
1
Current

( I s1 ) m = an 2 + bn 2 .
0.5
(c) 1.340 I o
0 I s1 = = 0.82 I o .
0 10 20 30 40 50 2
Time(ms) I S = Io .
1.5
I s1
PF = cos 30° = 0.92 cos 30° = 0.78.
1 Io
Current

Hence, the correct option is (b).


(d) 0.5
9. A single-phase half controlled converter shown
0 in the figure is feeding power to highly inductive
0 10 20 30 40 50
Time(ms) load. The converter is operating at a firing angle of
60°. [2008]
Solution: (c)
Under ideal conditions the current waveform through
inductor will look like
Vo

Hence, the correct option is (c). If the firing pulses are suddenly removed, the
8. A single phase full bridge converter supplies a steady state voltage (Vo) waveform of the converter
load drawing constant and ripple free load current. will become
If the triggering angle is 30°, the input power fac- Vo
tor will be [2008]
(a)
(a) 0.65 (b) 0.78 0
2p
wt
p
(c) 0.85 (d) 0.866
Solution: (b) Vo
Let the constant load conductor be Io, then the
source current will have the waveform (b)
0 wt
p/3 p 4p/3 2p
Io
Vo

(c)
0 wt
30 210 330 p/3 p 4p/3 2p
30 210
330
Vo

1 
30 210 330
(d)
 ∫ − I o sin ωt + ∫ I o sin ωt + ∫ − I o sin ωt 
0 wt
So, a1 = p 2p
π 0 30 210 
Io Solution: (a)
= [3.33] = 1.06 Io .
π Vo

1 
30 210 330
b1 =  ∫ − I o cos ωt + ∫ I o cos ωt + ∫ − I o cos ωt 
π 0 30 210 
a

Chapter 02.indd 19 11/9/2015 6:48:40 PM


9.20 | Power Electronics and Drives

Due to inductive nature of load, the current will 1


never fall as a result the thyristor corresponding to =
[30] = 9.54.
π
the conducting diode will always conduct and the
 1 180 360

bridge will act as uncontrolled converter. bn =  ∫ 10 cos ωt + ∫ −10 cos ωt 
Hence, the correct option is (a). π
 60 240 
10. A single phase fully controlled converter bridge is 1
   = [−17.32] = −5.513 = 011.0.
used for electrical braking of a separately excited π
DC motor. The DC motor load is represented by an
equivalent circuit as shown in the figure. [2008] 011.01
=I s1 = 7.8 A.
Io 2
I s = 200 .


 200 
 − 7.8
2

∼ 230 V,  3 
50 Hz THD = = 30.9 %
    7.8
150 V
Hence, the correct option is (b).
1 2. A single-phase, 230 V, 50 Hz AC mains fed step
Assume that the load inductance is sufficient to down transformer (4:1) is supplying power to a
ensure continuous and ripple free load current. half-wave uncontrolled AC–DC converter used for
The firing angle of the bridge for a load current of charging a battery (12 V DC) with the series cur-
Io = 10 A will be rent limiting resistor being 19.04 W. The charging
(a) 44° (b) 51° current is [2007]
(c) 129° (d) 136° (a) 3.43 A (b) 1.65 A
Solution: (c) (c) 1.22 A (d) 1.0 A
Since, electrical braking is to be done, the motor Solution: (d)
has to be operated in generator mode 230
So the step down voltage= is  = 57.5 V.
4
−150 − 2 I = Vo . So, the angle at which diode starts conduction
2 Vm
−150 − 2 × 10 = cos α . 12
π ⇒ Q1 = sin −1 = 8.48°.
2 × 57.5
2 × 2 230
−130 = cos α = 129°.
π
Hence, the correct option is (c). 57.5 V ∼
1 1. A three phase fully controlled bridge converter is 12 V
feeding a load drawing a constant and ripple tree
load current of 10 A at a firing angle of 30°. The
approximate total harmonic distortion (% THD)
The conduction stops at = 180° - 8.48 = 171.51°.
and rms value of fundamental component of the
input current will respectively be [2008] 1
171.51
57.5 sin ωt − 12
(a) 31% and 6.8 A (b) 31% and 7.8 A Charging current =
2π ∫ 1904
(c) 66% and 6.8 A (d) 66 % and 7.8 A 8.48

Solution: (b) 1
= [57.5 × 2 (cos 8.48 − cos 171.51)
38.08π
1 
180 360°

∫ − 12(2.99 − 0.14)] = 1.05 A.


an =
π  60
10 sin ω t + ∫
240°
−10 sin ωt 
 Hence, the correct option is (d).

Chapter 02.indd 20 11/9/2015 6:48:42 PM


Chapter 2  AC–DC Converters  |  9.21

13. In the circuit of adjacent figure the diode connects 15. A solar cell of 350 V is feeding power to an AC
the AC source to a pure inductance L. [2007] supply of 440 V, 50 Hz through a 3-phase fully
D controlled bridge converter. A large inductance
is connected in the DC circuit to maintain the
AC ∼ DC current at 20 A. If the solar cell resistance of
Pure
L 0.5 W, then each thryristor will be reverse biased
for a period of [2006]
The diode conducts for (a) 125° (b) 120°
(a) 90° (b) 180° (c) 60° (d) 55°
(c) 270° (d) 360° Solution: (d)
Solution: (d) Since, the solar cell is feeding power to the supply,
The diode conducts at ω t = 0, the diode is forward the bridge is in inverter mode of operation
biased and diode starts conducting till ω t = π, the 3
So, −350° = VmL cos α .
diode is forward biased. π
Now after ω t = π, due to inductor stored energy,  VmL = 2 × 440 V.
diode will be forward biased. Angle α such that the
diode will be forward biased, till the volt-radian −350 × π
cos α = = 125°.
area under inductor voltage from 0 to p becomes 3 × 2 × 440
equal to p to a. So the reverse bias period = 180° - 125° = 55°.
For this to be true α = 2π. Hence, the correct option is (d).
VL
1 6. A single-phase bridge converter is used to charge
2p
a battery of 200 V having an internal resistance of
0 p
q 2 W as shown in figure. The SCRs are triggered by
a constant DC signal. If SCR 2 gets open circuited,
So diode conducts for 360°. what will be the average charging current? [2006]
200 V
Hence, the correct option is (d). Battery
1 4. A 3-phase fully controlled bridge converter with 230 V SCR 1 SCR 2
freewheeling diode is fed from 400 V, 50 Hz AC 50 Hz
source and is operating at a firing angle of 60°. The
load current is assumed constant at 10 A due to
high load inductance. The input displacement fac- SCR 3 SCR 4
tor (IDF) and the input power factor (IPF) of the
converter will be [2006] (a) 23.8 A (b) 15 A
(a) IDF = 0.867; IPF = 0.828 (c) 11.9 A (d) 3.54 A
(b) IDF = 0.867; IPF = 0.552 Solution: (c)
(c) IDF = 0.5; IPF = 0.478 When T2 gets open circuited, the bridge will b­ ehave
(d) IDF = 0.5; IPF = 0.318 as a half wave rectifier and since a constant DC
Solution: (c) signal is given to the thyristors in the gate, they act
Input displacement PF = cosα = cos 60° = 0.5. as diodes.
Load power = Vo Io = (3 V ml/π) cosα)Io So, the diodes start conduction at
= [(3 * 1.4142*400)/π] cos 60 * 10  E   200 
θ1 = sin −1   = sin −1 
  = 37° and stop
= 2700 W.  Vm   2 × 230 
Input power factor (IPF) = (power delivered)/(1.73 conduction at θ 2 = 143°.
* VS IS)
θ2
= 2700/1.73 * 400 * Io * (2/3)1/2 = 0.477 lag. 1 Vm sin ωt
Hence, the correct option is (c).
So, average current =
2π ∫
θ1 R
−E

Chapter 02.indd 21 11/9/2015 6:48:44 PM


9.22 | Power Electronics and Drives

1
143°
Vm sin ωt VDC VDC
=
4π ∫37° R − E
1 (c)   (d) 
= [Vm (cos 37° − cos 143°) − 200 (2.49 − 0.645)] t t

1
= [519.54 − 369] = 11.97 A.
4π Solution: (b)
Hence, the correct option is (c). In half controlled rectifiers due to the presence of
diode, the voltage can never be negative.
17. Consider a phase-controlled converter shown in
the figure. The thyristor is fired at an angle a in Hence, the correct option is (b).
every positive half cycle of the input voltage. If 19. A three phase thyristor bridge rectifier is used in an
the peak value of the instantaneous output voltage HVDC link. The firing angle a (as measured from
equals 230 V, the firing angle a is close to [2005] the point of natural commutation) is constrained to
lie between 5 and 30°. If the DC side current and
+
230 AC side voltage magnitude are constant, which of
(RMS) ∼ R Vo the following statements is true (neglect harmon-
50 Hz ics in the AC side current and commutation over-

lap in your analysis) [2002]
(a) 45° (b) 135°
(a)  reactive power absorbed by the rectifier is
(c) 90 (d) 83.6°
maximum when a = 5°.
Solution: (b) (b)  reactive power absorbed by the rectifier is
The maximum output voltage = 230 V. maximum when a = 30°.
Now, the maximum output voltage is less than max- (c)  reactive power absorbed by the rectifier is
imum source voltage ( 2 × 230 V). So this implies maximum when a = 15°.
that thyristor has to be fixed at α > 90° and the (d)  reactive power absorbed by the rectifier is
maximum voltage of output = voltage of source at maximum when a = 30°.
instant ωt = α . Solutions: (b) and (d)
So, 230 = 2 × 230 sin α . The reactive power absorbed by the converter will
1 be maximum for α = 30°,  because in that case
  α = ⇒ α = 135°. power factor will be minimum.
2
Hence, the correct options are (b) and (d).
Hence, the correct option is (b).
20. In the single phase diode bridge rectifier shown in
18. A phase-controlled half-controlled single-phase
fig, the load resistor is R = 50 W.
converter is shown in figure. The control angle
a = 30° [2003] The source voltage is V = 200 sin ω t.
Where w = 2π × 50 radians per second. The power
a = 30° dissipated in the load resistor R is [2002]
VDC
VDC I DC

∼ R
The output DC voltage wave shape will be as
shown in
VDC VDC

(a)   (b)  3200 400


(a) W (b) W
t t π π
(c) 400 W (d) 800 W

Chapter 02.indd 22 11/9/2015 6:48:45 PM


Chapter 2  AC–DC Converters  |  9.23

Solution: (c) ⇒  cos 120° = cos β.


V 2
Power dissipated = rms . ⇒  b = 240°[As β ≠ 120° and β lies in 3rd quad].
R
1
Hence, the correct option is (a).
  2π 
1 2
When the firing angle a of a single phase fully
22.
 1 
 2002 sin 2 ωt (d ωt )  .
2

∫
Vrms =   controlled rectifier feeding constant DC current
2π 1  into a load is 30°, the displacement power factor
  0
L
of the rectifier is [1998]
1
(a) l (b) 0.5
 1   2π  1 − cos ωt     2 1
V = × ∫ 
4
 4 10    (c) (d) 3
 2π  0 2     3 2
1 Solution: (d)
 1  1  2 200 The displacement power factor from the figure is
=  4 × 10 × 2 × 2π  = 2 .
4

 2π   3
cos α = cos 30° = .
2002 2
Power dissipated = = 400 W.
2 × 50 Vs, Is
Hence, the correct option is (c).
21. A half-wave thyristor converter supplies a purely
a
inductive load as shown in figure. If triggering
angle of the thyristor is 120° the extinction angle
will be [2001] Hence, the correct option is (d).
(a) 240° (b) 180° 2 3. A 3 phase, fully controlled, converter is feeding
(c) 200° (d) 120° power into a DC load at a constant current of 150 A.
The rms current through each thyristor of the con-
verter is [1998]
+ (a) 50 A (b) 100 A
∼ Vm sin wt L
_ 150 2 150
(c) (d)
3 3
Solution: (a) Solution: (d) 1/ 2
The integral of voltage across an inductor over a  1 5π / 6 +α

The thyristor rms current =  ∫ ID d (ωt ) 
2
period = 0.
 2π π /6 
Now from α = 120° to β ⇒ VL = Vm sinω t and 1
ω t < α and ω t < β ⇒ VL = 0.  1 2  2π   2 I D 150
β = ID   = = A.
 2π  3  3 3
So, ∫
α =120
Vm sin ωt = 0.
Hence, the correct option is (d).

Chapter 02.indd 23 11/9/2015 6:48:47 PM


9.24 | Power Electronics and Drives

2. An SCR is connected in series with a 300 V AC


Five-marks Questions supply and a 300 Ω load resistor. Calculate [1997]
(a) The reading of a moving coil ammeter con-
1. A three phase diode bridge is used to provide rec- nected in series with the load and
tified output from a 400 V, 50 Hz, 3-phase supply (b) The reading of a moving iron voltmeter con-
to an R-L load with 10 W resistance and 300 mH nected across the SCR for firing angle of 45°
inductance. Determine the [1999] Solution:
(a) DC level of the output voltage (a)
(b) rms value of the diode current
(c) rms value of the source current, and
(d) apparent power drawn from the mains +
300√2 × sin wt ∼
Solution: −
(a) DC level
3VmL 3 × 2 × VL 3 × 2 × 400  he moving coil ammeter measures the current
T
= = = = 540.18 V.
π π π through the resistance
180°
(b) Average value of output current 1 2 × 300
=
2π ∫ 300
sin ωt
Avg.Volt 540.18 45
=
  = = 54.018 A.
Resistance 10 2
= [cos 45° − cos 180°] = 0.3842 A.
54.018 2π
 The rms=
value  = 31.18 A. (b) Moving iron voltmeter measures the rms volt-
3
age across SCR; for 45°ω t < 150°
2 VSCR = 0.
(c) The rms value of source current = I o
3 For time period other than this
2 VSCR = Vi .
=I rms 54
=.18 44.23 A.
3 1
 1 2π + 45° 2
So, Vo =  ∫ × × ω t d ω t
2
(d) Apparent power drawn from mains ( 2 300 sin ) ( ) 
 2π π 
P = 3VL I rms = 3 × 400 × 44.23 = 30.649 kVA. = 221.558 V.

Chapter 02.indd 24 11/9/2015 6:48:49 PM


Chapter 3
DC–DC Converters
(a) step-down chopper (buck converter)
One-mark Questions (b) half-wave rectifier
(c) step-up chopper (boost converter)
1. A step-up chopper is used to feed a load at 400 V (d) full-wave rectifier
DC from a 250 V DC source. The inductor c­ urrent
Solution: (a)
is continuous. If the off time of the switch is
The circuit can be shown, we have two modes
20 µs, the switching frequency of the chopper in
kHz is . [2014-S2] A
+ +
Solution:
V out V out
= Vo 400= V Vs 250 V.
− −
V
NowVo = .
1− D Mode 1 Mode 2

250 So these two are the modes of step down (buck)
⇒ 400 = . converter.
    1− D
⇒ D = 0.375, Hence, the correct option is (a).
where D = Duty Ratio. 3. A step down chopper is operated in the continuous
conduction mode in steady state with a constant
ton ton duty ratio D. If Vo is the magnitude of the DC out-
Now D = = .
ton + toff ton + 20 put voltage and if Vs is the magnitude of the DC
⇒  ton = 12 μs. input voltage, the ratio Vo /Vs is given by [2002]
Net time period = (12 + 20) μs = 32 μs = T. (a) D (b) 1 − D
1 D
1 (c) (d)
Frequency= = 31.25 kHz. 1− D 1− D
T
Solution: (a)
2. The power electronic converter shown in the figure
Vo = DVs .
has a single-pole double-throw switch. The pole P
of the switch is connected alternately to throws A Vo
⇒ = D.
and B. The converter shown is a [2010] Vs
A Hence, the correct option is (a).
P L
+
4. In a thyristor DC chopper which type of commuta-
B tion results in best performance? [1996]
V in V out
(a) Voltage commutation
− (b) Current commutation

Chapter 03.indd 25 11/12/2015 10:48:55 AM


9.26 | Power Electronics and Drives

(c) Load commutation CVs


Load current= I= .
(d) Supply commutation tC
Solution: (b) CVs
Current rating of the thyristor is same as load c­ urrent. So, tC =
I
But in voltage commutated chopper it is more than
load current. 2 × 10−6 × 200
= = 40 µsec.
Hence, the correct option is (b). 10

5. A DC to DC transistor chopper supplied from a fixed 7. Match the following [1994]


voltage DC source feeds a fixed resistive-inductive List-I List-II
load and a freewheeling diode. The chopper operates
A. Communication P.  Resistive load
at 1kHz and 50% duty cycle. Without changing the
value of the average DC current through the load, B. V-Curves Q.  Inductive load
if it is desired to reduce the ripple constant of load C.  Freewheeling diode R.  Capacitive load
current, the control action needed will be [1995] D. Overlap S.  Inter pole
(a)  increase the chopper frequency keeping its T.  Source Inductance
duty cycle constant U.  Synchronous motor
(b) increase the chopper frequency and duty cycle
in equal ratio Solution:
(c) decrease only the chopper frequency A – S B–U
(d) decrease only the duty cycle C – Q D–T

Solution: (a) 8. A chopper operating at a fixed frequency is feed-


Vo DVs ing an R-L load. As the duty ratio of the chopper is
= I=
Average DC current o = . increased from 25% to 75%, the ripple in the load
R R
current[1993]
Vs (a) remains constant
Ripple = .
Gf s L (b) decreases, reaches a minimum at 50% duty ra-
So to maintain average DC current α should be con- tio and then increases
stant and to decrease ripple, f should be increased. (c) increases, reaches a maximum at 50% duty
­ratio and then decreases
Hence, the correct option is (a). (d) keeps on increasing as the duty ratio is i­ ncreased
6. Consider the chopper circuit in figure. The chop- Solution: (c)
per operates at 400 Hz and 50% duty cycle. The The ripple in load current (Δ I ) is given as
load current remains almost ripple free at 10 A.
Assuming the input voltage to be 200 V and the  −D
TR

TR
− (1− D )
TR

devices to be ideal, the turn off time available to Vs 1 − e L + e L − e L
,
∆I =
the thysistor TH M is µ s[1995] R −
TR 
 1− e L 
THM
where D = Duty ratio, T = Total time period.
2 μF L FD Differenting Δ I wrt D, and equating to zero,
+ O
200 V
− A  TR − D TR  TR −(1− D ) TR  
D  e L +− e L

d (∆I ) Vs  L  L  = 0
1 mH =  
dD R 1 − eTR / L
 
Solution:  
C = 2 μF. TR TR
−D − (1− D )
f = 400 Hz  a = 0.5, So, e L
=e L
.

Chapter 03.indd 26 11/12/2015 10:48:56 AM


Chapter 3  DC–DC Converters  |  9.27

Taking log on both sides switched at 250 kHz, with a duty ratio of 0.4. All
TR TR elements of the circuit are assumed to be ideal.
−D = −(1 − D) . 100 μH
L L
D = (1 − D).
+ 12 V
D = 0.5. − Q 470 μF 20 Ω
So, Δ I is max at D = 0.5.
So, on increasing duty from 25% to 75%, first the
ripple increases till 50% then decreases. 2. The average source current in Amps in steady-
Hence, the correct option is (c). state is  [2013]
3 5
(a) (b)
2 3
Two-marks Questions 5 15
(c) (d)
2 4
1. Figure (i) shows the circuit diagram of chopper.
Solution: (b)
The switch S in the circuit in figure (i) is switched 12
such that the voltage vD across the diagram as the The average output voltage = = 20.
1 − 0.4
wave shape as shown in figure (ii). The capacitance
C is large so that the voltage across it is constant. 20
= = 1 A.
The average output current
If switch S and diode are ideal, the peak to peak 20
­ripple (in A) in the inductor current is 1 1 10 5
The average source current = = = = .
[2014-S1] 1 − 0.4 0.6 6 3
S 1 mH Hence, the correct option is (b).
+ 3. The peak-to-peak source current ripple in Amps is
VD C Load [2013]
100 V
− (a) 0.96 (b) 0.144
(c) 0.192 (d) 0.288
Figure 1
Solution: (c)
VD
DT
1 VDC
100 V
∆ IL = ∆ Is =
L ∫V
O
s dt =
L
DT

12 12
0 t (ms) = × 0.4 ×
0.05 0.1 0.15 0.2 100 × 10−6 250 × 103
− p/2
= 0.192 A.
Figure 2
Hence, the correct option is (c).
Solution:
First we find out the average output voltage = VsD 4. In the circuit shown, an ideal switch S is operated
(as it is a buck converter) = 50 V and peak current at 100 kHz with a duty ratio of 50%. Given that
Δic is 1.6 A peak-to-peak and Io is 5 A DC, the peak
DT
(100 − 50) current in S is [2012]
= ∫
O
L
× dt

(100 − 50) S
L
= × 0.05 × 10−3 = 2.5 A. +
10−3 Io
ΔI c
+ R
Common Data for Questions 2 and 3: D C Vo
24 V

In the figure shown below, the chopper feeds a re-
sistive load from a battery source. MOSFET Q is −

Chapter 03.indd 27 11/12/2015 10:48:58 AM


9.28 | Power Electronics and Drives

(a) 6.6 A (b) 5.0 A (a) 5 cos(5 × 103 t) A (b) 5 sin(l04 t) A


(c) 5.8 A (d) 4.2 A (c) 10 cos(5 × 103 t) A (d) 10 sin(104 t) A
Solution: (c) Solution: (b)
∆i
The peak current in S = I o + c = 5.8 A. C 1
2 i = VC sin ωt ; ω = .
Hence, the correct option is (c). L LC
5. A voltage commutated chopper circuit, operated at 10 × 10−6
i = 100 ;
500 Hz, is shown below. 10−3
If the maximum value of load current is 10 A, then 1
the maximum current through the main (M) and ω= = 10−8 = 0.104
1× 10−3 × 10 × 10−6
auxiliary (A) thyristors will be [2011]
= 100 × 0.1sin(104 )t = 10 sin(104 t ).
M iM i L = 10 A
+ Hence, the correct option is (b).
0.1 μF A 7. The L-C circuit is used to commutate a thyristor,
iA
L which is initially carrying a current of 5 A as shown
O in the figure below. The values and initial condi-
200 V 1 mH A
D tions of L and C are the same. The switch is closed
at t = 0. If the forward drop is negligible, the time
taken for the device to turn off is [2010]

L
iM max = 12 A 
(a) and  iA max = 10 A i
iM max = 12 A 
(b) and  iA max = 2 A
C 100 V
iM max = 10 A 
(c) and  iA max = 12 A
iM max = 10 A 
(d) and  iA max = 8 A t=0

Solution: (a)
20 Ω
Maximum current through main thyristor 100 V 5A

10−7
= 10 + 200
10−3
(a) 52 µs (b) 156 µs
= 10 + 2 = 12 A. (c) 312 µs (d) 26 µs
Maximum current through Aux = Load current
Solution: (a)
= 10 A.
The devices will turn off when current through it
Hence, the correct option is (a). becomes zero.
Common Data for Questions 6 and 7: So, when current in resonant circuit = 5 A, then
The L-C circuit shown in the figure has an induct- switch turns off.
ance L = 1 mH and a Capacitance C = 10 µF. So, 5 = 10 sin 104 t.
6. The initial current through the inductor is zero,    104 t = π .
while the initial capacitor voltage is 100 V. The 6
switch is closed at t = 0. The current i′ through the   t = 52.3 m sec.
circuit is: [2010]
Hence, the correct option is (a).
L
8. In the chopper circuit shown, the main thyris-
i
− tor (TM ) is operated at a duty ratio of 0.8 which
C 100 V
+ is much larger than the commutation interval. If
t=0 the maximum allowable reapplied dv/dt on TM is

Chapter 03.indd 28 11/12/2015 1:27:34 PM


Chapter 3  DC–DC Converters  |  9.29

50 V/us, what should be the theoretical minimum Vs 20


The load voltage = = = 40 V.
value of C1? Assume current ripple through L0 to 1 − α 1 − 0.5
be negligible. [2009] Hence, the correct option is (c).
+ 10. The circuit in the figure is current commutated
TM DC–DC chopper where, THM is the main SCR and
L1 THAUX is the auxiliary SCR. The load current is
100 V constant at 10 A. THM is ON. THAUX is trigged at
C1 T A Lo t = 0. THM is turned OFF between. [2007]
− +

Do C 8Ω THAUX
o
− D1
L
O
(a) 0.2 µF (b) 0.02 µF + A
(c) 2 µF (d) 20 µF 230 V 25.28 μH D
10 μF
Solution: (a)
dv
α = 0.8 Vs = 100 V and = 50 V/msec. (a) 0 µs < t < 25 µ (b) 25 µs < t < 50 µs
dt
(c) 50 µs < t < 75 µs (d) 75 µs < t < 100 µs
Vo = α Vs = 0.8 × 100 ⇒ 80 V.
Vo 80 Solution: (c)
io = = = 10 A. THAUX conducts for time
R 8
During commutation of main SCR (TM) load c­ urrent π LC = π 10 × 10−6 × 25.28 × 10−6 = 50 msec.
will be flown through capacitor. After 50 ms duration a current pulse in opposition
dv will be injected.
∴ i=C So THM get off in the interval 50 ms < t < 75 ms.
dt
10 × 10−6 Hence, the correct option is (c).
⇒ C= ⇒ 0.2 µF.
50 Common Data for Questions 11 and 12:
Hence, the correct option is (a). A voltage commutated chopper operating at 1 kHz
9. In the circuit shown in the figure, the switch is is used to control the speed of DC motor as shown
operated at a duty cycle of 0.5. A large capacitor is in figure. The load current is assumed to be con-
connected across the load. The inductor current is stant at 10 A.
assumed to be continuous. [2008] M
+
IL = 4 A L ID D iL
1 μF A
L
S O V = 250 V 2 mH
20 V Vs
A
D −

11. The minimum time in µsec for which the SCR M


The average voltage across the load and the aver- should be ON is [2006]
age current through the diode will respectively be (a) 280 (b) 140
(a) 10 V, 2 A (b) 10 V, 8 A (c) 70 (d) 0
(c) 40 V, 2 A (d) 40 V, 8 A Solution: (b)
Solution: (c)
t tmin = π LmC = π 2 × 10−3 × 10−6 = 140 msec.
The average diode current = 4 × OFF = 4 × 0.5 = 2 A.
T Hence, the correct option is (b).

Chapter 03.indd 29 11/12/2015 10:49:00 AM


9.30 | Power Electronics and Drives

12. The average output voltage of the chopper will be 200 mH


[2006]
(a) 70 V (b) 47.5 V +
100 V
(c) 35 V (d) 0 V 5Ω
L −
Solution: (b)
VS
VO = [TON + 2tc ].
T (a) 10 A (b) 0.5 A
CVS (c) 0.125 A (d) 0.25 A
tc =
Io Solution: (a)
250  −4 10−6 × 250   −
∆RT

RT
− (1− D )
RT

= 140 × 10 + 2 ×  = 47.5 V ∆iL =
VS 1 − e L + e L − e L

10−3  10  R −
RT 
 1− e L 
Hence, the correct option is (b).
 
−3
 5  0.5×5×10
1 3. A voltage commutation circuit is shown in figure.  0.5× −3
×10−3  5 × 10−3 −
1 − e 200×10 
+e− − e 200×10−3

If the turn off time of the SCR is 50 µsec and as 100  200 × 10 −3

=
safety margin of 2 is considered, what will be the 5  5 × 10−3 
 1− e − 
approximate minimum value of capacitor required  200 × 10−3 
for proper commutation? [2006] = 0.125 A.
Hence, the correct option is (a).
50 Ω 50 Ω
1 5. Figure shows a chopper operating from a 100 V
DC input. The duty ratio of the main switch S is
100 V 0.8. The load is sufficiently inductive so that the
load current is ripple free. The average current
TH 1 TH 2 through the diode D under steady state is [2004]
S L
(a) 2.88 µF (b) l.44 µF +
(c) 0.91 µF (d) 0.72 µF 10 Ω
100 V D
Solution: (a)

Safety factor = 2, tc = 50 msec.
(a) 1.6 A (b) 6.4A
R1 = 50Ω.
(c) 8.0 A (d) 10.0 A
The voltage across thyristor 1 = VT 1
Solution: (a)
= Vs [1 − 2e − t / R1c ] VDC D 100
The average DC current = = × 0.8 = 8 A.

tc1 is the commutation time and indicates reverse R 10
voltage -Vs to zero Now this current flows through diode for the duty
ratio 0.2.
 − 1 
t

∴ 0 = Vs 1 − 2e  .
 R1c So, average DC current through diode = 8 × 0.2
= 1.6 A.
tc
=C = 2.88 mF. Hence, the correct option is (a).
R1In(2)
16. Figure shows a chopper. The device S1 is the main
Hence, the correct option is (a). switching device, S2 is the auxiliary commutation
1 4. The given figure shows a step-down chopper device. S1 is rated for 400 V, 60 A. S2 is rated for
switched at 1 kHz with a duty ratio D = 0.5. The 400 V, 30 A. The load current is 20 A. The main
peak-peak ripple in the load current is close to device operates with a duty ratio of 0.5. The peak
[2005] current through S1 is [2004]

Chapter 03.indd 30 11/12/2015 10:49:01 AM


Chapter 3  DC–DC Converters  |  9.31

S1 20 A 18. In the chopper circuit shown in figure the input DC


voltage has a constant value Vs. The output voltage
2 μF S2 D Vo is assumed ripple free. The switch S is operated
200

with a switching time period T and a duty ratio D.


What is the value of D at the boundary of continu-
200 μH
ous and discontinuous conduction of the induction
current iL? [2002]
(a) 10 A (b) 20 A
(c) 30 A (d) 40 A L

Solution: (d) S iL
The max current = Peak resonant current + Load + +
Vs R V
− C o
C −
current = VC + 20.
LM
VC = 200 V.
Vs 2L
2 × 10−6 D = 1−
(a) D=
(b)
Max current = 200 + 20 = 40 A. Vo RT
200 × 10−6
2L RT
Hence, the correct option is (d). D = 1−
(c) D=
(d)
RT L
17. A chopper is employed to charge a battery as
shown in figure. The charging current is 5 A. The Solution: (c)
duty ratio is 0.2. The chopper output voltage is iL
also shown in figure. The peak to peak ripple cur-
DiL
rent in the charging current is [2003]
L = 20 mH
DT T
Chopper

5A At the boundary of continuous and discontinuous


Vdc 12 V mode, the value of IL becomes zero.
∆I
So, average DC current = I L = L .
2
60 V DVDC t
V DC IL =  and  ∆iL = (VDC − Vo )
R L
1 tON [1 − D]DT
= VDC [1 − D] = VDC .
200 μs L L

1 ms VDC D[1 − D]T DVDC


So, = .
2L R
(a) 0.48 A (b) 1.2 A
(c) 2.4 A (d) 1 A 2L
D = 1− .
RT
Solution: (a)
Given the induction, Hence, the correct option is (c).

di 19. A step down chopper operates from a DC voltage


L = VDC − Vo .
dt source Vs feeds a DC motor armature with a back
tON
(VDC − Vo ) (60 − 12) × 200 × 10−3 emf Eb. From oscilloscope traces, it is found that
Now, ∆i = ∫ dt = the current increases for time tr, falls to zero over
0
L 20 × 10−3 time t0, and remains zero for time to in every chop-
= 0.48 A. ping cycle, then the average DC voltage across the
Hence, the correct option is (a). freewheeling diode is [2000]

Chapter 03.indd 31 11/12/2015 10:49:04 AM


9.32 | Power Electronics and Drives

Vt (Vs tr + Eb t f ) During tr < t < tq ⇒ the load current decreases, this


(a) s r (b)
(tr + t f + t0 ) (tr + t f + t0 ) implies the back emf is connected to the load in-
ductance and voltage across switch is zero as the
(Vs tr + Eb t0 ) Vs tr + Eb [t f + t0 ] complete back emf is dropped over load i­ nductance.
(c) (d)
(tr + t f + t0 ) (tr + t f + t0 ) During tq < 1 < t0 ⇒ the load current is zero; so
back emf appears across the switch.
Solution: (c)
During 0 < t < Vs ⇒ the load is connected to the Vs tr + Eb to
So, V0 = .
source and the voltage across switch is Vs and load t f + t r + to
current increases. Hence, the correct option is (c).

Chapter 03.indd 32 11/12/2015 10:49:05 AM


Chapter 3  DC–DC Converters  |  9.33

0 = -5 × 106 tOFF + 106 tON.


Five-marks Questions 5tON = tOFF
1. In figure, the ideal switch S is switched on and off And tON + tOFF = 100 ms.
with a switching frequency f = 10 kHz. The switch-  tON = 83.33 ms.
ing time period is T = ton + toff = 100 µs. The circuit is
Peak current = 106 × tON = 83.33 A.
operated in steady state at the boundary of continu-
ous and discontinuous conduction, so that the induc- 2. A voltage commutated thyristor chopper circuit is
tor current, is as shown in figure, find [2002] shown in figure. The chopper is operated at 500 Hz
(a) the on time tON of the switch with 50% duty ratio. The load takes a constant cur-
(b) the value of the peak current Ip rent of 20 A. [2001]
S
(a) Evaluate the circuit turn off time for the main
thyristor TH 1
(b) Calculate the value of inductor L, if the peak
+ i − current through the main thyristor TH , is lim-
100 μH 500 V
100 V − + ited to 180% of the load current.
(c) Calculate the maximum instantaneous output
voltage of the chopper
T H1 i L = 20 A
ON OFF ON
S
C
6 μF T H2
L
iP O
i + A
V DC L D1 D2
t − D
0 100 V
t ON t OFF

Solution:
T = tON + tOFF = 100 msec. Solution:
L = 100 mH. Given f = 500 Hz, δ = 50%, Io = 20 A.
KVL in first loop, C = 6 mF  Vs = 100 V.
di (a) Capacitor is in parallel with thyristor Th, when
100 = L . the auxiliary thyristor is turned ON. At this
dt
t i time the voltage across the capacitor -Vs, i.e.,
∫ 100 dt = ∫ L di. reverse voltage AC main thyristor. The circuit
o 0 turn off time is the time taken from voltage -Vs
100 to zero voltage of capacitor,
i= t = 10+6 t.
100 × 10−6 dv
i.e., i = C .
During tOFF , dt
di Vs
L = −500. Io = C .
dt Tc
tOFF
i
−500 Vs 6 × 10−6 × 100
∫ ds = ∫ dt. tC = C = ⇒ 30 msec.
io o
L Io 20

By using boundary conditions we get, (b) Peak current through main current

i = −5 × 106 t + io = −5 × 106 t + 106 tON . C


I p = Vs + 20.
t = tOFF , i = 0, L

Chapter 03.indd 33 11/12/2015 10:49:06 AM


9.34 | Power Electronics and Drives

Given Ip = 180% load current. drops from Imax to Imin in the off Period TOFF of the
same circuit. Both the rise and fall of the current
180 6 × 10−6 may be assumed to be approximately linear. What
× 20 = 100 + 20.
100 L is the average power loss in the machine armature?
⇒ L = 0.234 mH. [2000]
Solution:
(c) When main thyristor is off, the capacitor voltage I +I
and supply voltages are in series with the load. I o = max min .
2
 ∴ Maximum instantaneous output voltage = i
2Vs = 200 V. I max
3. A DC motor with armature resistance Ra is fed
from a step down chopper in the continuous mode, I min
and operates at some known speed and known t
T ON T
excitation current. The motor current rises from
Imin to Imax in the ON period TON of chopper; and Power loss = IO2Ra.

Chapter 03.indd 34 11/12/2015 10:49:07 AM


Chapter 4
DC–AC Converters
Solution: 
One-mark Questions The switch should be bipolar but conduct current
only in one direction.
1. A three-phase current source inverter used for the
speed control of an induction motor is to be real- 2. A 3-phase voltage source inverter is operated in
ized using MOSFET switches as shown below. 180° conduction mode. Which one of the follow-
Switches S1 to S6 are identical switches. [2011] ing statements is true? [2008]
I4
(a) Both pole-voltage and line-voltage will have
3rd harmonic components
A
(b) Pole-voltage will have 3rd harmonic compo-
S1 B S3 S5 nent but line-voltage will be free from 3rd
A IM ­harmonic
S4 S6 S2 (c) Line-voltage will have 3rd harmonic compo-
B
nent but pole-voltage will be free from 3rd
The proper configuration for realizing switches harmonic
S1 to S6 is (d) Both pole-voltage and line-voltage will be free
(a)  A  (b)  A
from 3rd harmonic components
Solution: (b)
Pole voltage will have third harmonics; line volt-
age will not have third harmonics.
Hence, the correct option is (b).
3. ‘Six MOSFETs connected in a bridge configura-
tion (having no other power device) MUST be
operated as Voltage Source Inverter (VSI)’. This
B B
statement is [2007]
(c)  A  (d)  A (a) True, because being majority carrier devices,
MOSFETs are voltage driven
(b) True, because MOSFETs have inherent anti-
parallel diodes
(c) False, because it can be operated both as Cur-
rent Source Inverter (CSI) or a VSI
(d) False, because MOSFETs can be operated as
excellent constant current sources in the satu-
B B ration region

Chapter 04.indd 35 11/9/2015 6:50:47 PM


9.36 | Power Electronics and Drives

Solution: (b) Solution: (d)


Six MOSFETs connected in a bridge configura- PWM switching helps in reducing the lower order
tion must be operated as VSI, as they have inherent harmonics and increase the higher order harmon-
anti-parallel diodes. ics which can be further eliminated using filter.
Hence, the correct option is (b). Hence, the correct option is (d).
4. A three-phase fully-controlled thyristor bridge con- 8. An inverter capable of supplying a balanced three-
verter is used as line commuted inverter to feed phase variable voltage variable frequency output
50 kW power 420 V DC to a three phase, 415 V (line), is feeding a three-phase induction motor rated for
50 Hz AC mains. Consider DC link current to be con- 50 Hz and 440 V. The stator winding resistance of
stant. The rms current of the thyristor is [2007] the motor is negligibly small during starting. The
(a) 119.05 A (b) 79.37 A current inrush can be avoided without sacrificing
(c) 68.73 A (d) 39.68 A the starting torque by suitably applying [1995]
(a) low voltage as rated frequency
Solution: (c)
(b) low voltage keeping the V/f ratio constant
Power 50, 000 (c) rated voltage at low frequency
=
The DC current = = 119.047 A.
Voltage 450 (d) rated voltage at rated frequency
1 Solution: (b)
= I=
rms current o 68.73 A. V
3 In induction motor, flux α . So to maintain the
Hence, the correct option is (c). f
V
flux, is kept constant and in order to reduce the
5. The output voltage waveform of a three phase f
square wave inverter contains [2005] inrush current, voltage is reduced.
(a) only even harmonics Hence, the correct option is (b).
(b) both odd and even harmonics
9. A single phase inverter with square wave output
(c) only odd harmonics
voltage will have in its output waveform a fifth
(d) only triple harmonics
harmonic component equal to percentage of the
Solution: (c) fundamental [1995]
The output voltage waveform in square wave in-
Solution:
verter contains only odd harmonics. V
Vn = rms value of nth harmonic = 1 ,
Hence, the correct option is (c). n
where V1 = rms of fundamental.
6. Triangular PWM control, when applied to a three
V 1
phase BJT based voltage source inverter, intro- So, n = s, s= = 20%.
duces [2000] V1 5
(a) low order harmonic voltages on the DC side. 10. A line-commutated inverter changes DC voltage
(b) very high order harmonic voltage on the DC side to AC voltage. (TRUE/FALSE) [1994]
(c) low order harmonic voltage on the AC side Solution: The given statement is true.
(d) very high order harmonic voltage on the AC side 11. When a line commutated converter operates in the
Solution: (d) inverter mode  [1993]
Triangular PWM introduces higher order harmon- (a) it draws both real and reactive power from the
ics in the AC side. AC supply
Hence, the correct option is (d). (b) it delivers both real and reactive power to the
AC supply
7. A PWM switching scheme is used with a three
(c) it delivers real power to the AC supply
phase inverter to[IES-2013 and 1999]
(d) it draws reactive power from the AC supply
(a) reduce the total harmonic distortion with mod-
est filtering Solution: (b)
(b) minimize the load on the DC side It delivers both real and reactive power to the AC
(c) increase the life of the batteries supply as in inverter mode power flows from DC to
(d) reduce low order harmonics and increase high AC side.
order harmonics Hence, the correct option is (b).

Chapter 04.indd 36 11/9/2015 6:50:48 PM


Chapter 4  DC–AC Converters  |  9.37

(a)
Ql, Q2 (b)
Q3, Q4
Two-marks Questions (c)
Dl, D2 (d)
D3, D4

1. In the figure shows one period of the output volt-


Solution: (d)
Since, Vo < 0 and io > 0.
age of an inverter, α should be chosen such that
60° < α < 90°. If rms value of the fundamental So, D4 and D3 must conduct.
component is 50 V, then α in degree is ________ Hence, the correct option is (d).
 [2014-S1] 3. Appropriate transition, i.e., zero voltage switching
100 V 100 V 100 V (ZVS)/zero current switching (ZCS) of the IGBTs
during turn on/turn off is [2013]
wt
(a)  ZVS during turn off   (b)  ZVS during turn on
0 180 − a 180 360 − a 360 (c)  ZCS during turn off   (d)  ZCS during turn on
Solution:
−100 V −100 V Out of syllabus.
Solution: Common Data for Questions 4 and 5:
1
2π In the 3-phase inverter circuit shown, the load
b1 =
π ∫ f (t ) sin ωt d (ωt ). is balanced and the gating scheme is 180° con-
0 duction mode. All the switching devices are
2VDC α π −α
ideal.
π  ∫0 ∫
=  sin ωt d (ωt ) − sin ωt d (ωt )
α
α

+ ∫ sin ωt d (ωt )  S1 S3 S5 Vph R = 20 Ω
π −α 
+
Vd
2VDC −
= [2 − 4 cos α ].
π S4 S6 S2 R R

 ⇒ cos α = 0.2223 ⇒ α = 77.15°.


Common Data for Questions 2 and 3: 3-phase inverter 3-phase balanced load
The voltage source inverter (VSI) shown in the fig-
ure below is switched to provide a 50 kHz square- 4. If the DC bus voltage Vd = 300 V, the power con-
wave AC output voltage (vo) across an R-L load. sumed by 3-phase load is [2012]
Reference polarity of vo and reference direction of (a) 1.5 kW (b) 2.0 kW
the output current io are indicated in the figure. It is (c) 2.5 kW (d) 3.0 kW
given that R = 3 Ω, L = 9.55 mH. Solution: (d)
3(V ph ) rms
2
  Power = .
Q1 Q3 Resistance
D1 D3 2 2
  V ph = 3 Vs = 3 × 300.
L
   
+ V i
+ 2
− d
−V R  2 
 × 300 
3
Power = 3 ×   = 3000 W = 3 kW.
Q4 Q2

D4 D2
20
Hence, the correct option is (d).
5. The rms value of load phase voltage is  [2012]
2. In the interval when vo < 0 and io > 0 the pair of (a) 106.1 V (b) 141.4 V
devices which conducts the load current is [2013] (c) 212.2 V (d) 282.8 V

Chapter 04.indd 37 11/9/2015 6:50:49 PM


9.38 | Power Electronics and Drives

Solution: (b)

2
V ph = × 300 = 141.4 V.
3
0.1 H
Hence, the correct option is (b). 200 V
io
6. The current source inverter shown in figure is
operated by alternately turning on thyristor pairs
(T1, T2) and (T3, T4). If the load is purely resistive,
the theoretical maximum output frequency obtain-
able will be [2009] The inverter is operated at 50 Hz in 180° square
wave mode. Assume that the load current does not
have any DC component. The peak value of the
inductor current io will be
T1 T3
0.1 µF (a) 6.37 A (b) 10 A
(c) 20 A (d) 40 A
+ −
D1 D3
Solution: (b)
The waveform for voltage and current is shown
10 Ω 10 A below.
D4 D2

+ −
0.1 µF
T4 T2

(a) 125 kHz (b) 250 kHz


(c) 500 kHz (d) 50 kHz T 1
2
V 100
200
Solution: (b) ipeak = ∫ dt = ∫ dt = 10 A.
The resistance and capacitance will be connected T L 1 0.1
4 200
in series when T1 T2 are turned on in the positive
have cycle. Hence, the correct option is (b).
8. A single-phase voltage source inverter is con-
0.1
Ceq = = 0.05 µF. trolled in a single pulse-width modulated mode
2 with a pulse width of 150° in each half cycle. Total
τ = RCeq = 10 × 0.05 µF = 0.5 µs. harmonic distortion is defined as
V 2rms − V12
Reasonable time period = 4 or 5 times time constant THD = × 100, where V1 is the rms
V1
≅ 4 × 0.5 = 2 µs. value of the fundamental component of the output
voltage. The THD of output AC voltage waveform
Complete time period = 2 + 2 = 4 µs. is [2007]
1 (a) 65.65% (b) 48.42%
Maximum frequency = = 250 kHZ. (c) 31.83% (d) 30.49%
4 × 10−6
Hence, the correct option is (b). Solution: (c)
1
7. A single phase voltage source inverter is feeding  1  5π   2 5
a purely inductive load as shown in the figure. Vrms =  × Vd2   × 2  = Vd = 0.9128.
 2π   
6 6
 [2008]

Chapter 04.indd 38 11/9/2015 6:50:51 PM


Chapter 4  DC–AC Converters  |  9.39

1 165 345
 Common Data for Questions 10 and 11:
 (V1 )peac =  π ∫ Vd sin ωt + ∫195 −Vd sin ωt  An inverter has a periodic output voltage with the
 15
output waveform as shown in figure
V 
=  d [cos 15 − cos 165] + [cos 345 − cos 195] 1
 π  a p 2p
= 1.22 Vd . a
−1
(V1 ) rms = 0.8696 Vd .
0.833Vd2 − 0.756 Vd2 10. When the conduction angle α = 120°, the rms
THD = = 31.8%. fundamental component of the output voltage
0.8696 Vd is [2003]

Hence, the correct option is (c). (a) 0.78 V (b) 1.10 V
9. A single-phase inverter is operated in PWM mode (c) 0.90 V (d) 1.27 V
generating a single pulse of width 2d in the centre Solution: (b)
of each half cycle as shown in figure. It is found Since, it is an odd function, we will only have sine
that the output voltage is free from 5th harmonic for components.
pulse width 144°. What will be percentage of 3rd
 150° 330

So, V1 =  1 sin ωt + −1 sin ωt 
harmonic present in the output voltage? [2006]
π ∫
 30°

210 
 V03 
 ? 1
 V01max  = [[cos 30° − cos 150°] + [cos 330° − cos 210°]]
π
= 1.102 V.
2d Hence, the correct option is (b).
V
1 1. With reference to the output waveform given in
3p/2
figure, the output of the converter will be free from
p/2 p 2p 5th harmonic when [2003]
(a) α = 72° (b) α = 36°
−V 2d
(c) α = 150° (d) α = 120°
Solution: (a)
(a) 0.0% (b) 19.6%  π

3π α
+ 
(c) 31.7% (d) 53.9% 1
2 2 3
 5α
Solution: (b)
V5 = 
π ∫ sin 5ωt + ∫ − sin 5ωt  = 4 sin
2
.
   π α

3π α
− 
Since, 5th harmonic is missing 2d = 72°. 2 2 2 2 
 1 125 306
 5α
So, V3 =  ∫ Vd sin 3ωt − ∫ Vd sin 3ωt  Now for Vs = 0, sin =0
 π 54 234  2

V or = 180°
= d [(cos 3 × 54 − cos 3 × 126) 2
3π or a = 72°.
+ cos(3 × 306) − cos(3 × 234)].
Hence, the correct option is (a).
4 Vs 12. Figure (a) shows an inverter circuit with a DC
Peak value of fundamental = sin 36°.
π source voltage Vs. The semiconductor switches of
V03 the inverter are operated in such a manner that the
=
 V 0=
.1959 19.6%. pole voltage V10 and V20 are as shown in figure (b).
01 What is the rms value of the pole-to-pole voltage
Hence, the correct option is (b). V12? [2002]

Chapter 04.indd 39 11/9/2015 6:50:53 PM


9.40 | Power Electronics and Drives

operated in square-wave mode with a frequency of


50 Hz. If the average load current is zero, what is
the time duration of conduction of each feedback
+ V1 V2 diode in a cycle? [2001]
Vs

T1 D1 D3
T3

+ L
Figure (a) −

V10 T4 D4 D2
T2

Vs

V20 q
p 2p (a) 5 m sec (b) 10 m sec
D
Vs
(c) 20 m sec (d) 2.5 m sec
Solution: (a)
q
T Each feedback diode conducts for one fourth of
1 1 
Figure (b) the total period =   = 5 m sec.
4  50 
Vφ φ
(a) s (b) VS Hence, the correct option is (a).
π 2 π
14. A three phase voltage source inverter supply a
φ VS purely inductive three phase load upon Fourier anal-
VS
(c) (d)
2π π ysis, the output voltage waveform is found to have
Solution: (b) an hth order harmonic of magnitude αh times that
of the fundamental frequency component (αh < 1),
V12 = V1 − V2 =
the load current would then have an hth order
­harmonic of magnitude [2000]
(a) zero
(b) αh times the fundamental frequency component
p+f (c)  hαh times the fundamental frequency component
f p 2p α
(d)  n times the fundamental frequency component
h
1
 1 2 φ Solution: (d)
Vrms =  Vs 2 (φ ) + Vs 2 (φ )  = Vs . αn
 2π  π  times the fundamental component as the reac-
h
Hence, the correct option is (b). tance for the nth order harmonic = hX1.
13. A single-phase full-bridge voltage source inverter α nV1 α n
feeds a purely induction load, as shown in figure, So, iL = = i1 .
hX 1 h
where T1, T2, T3, T4 are power transistors and D1,
D2, D3, D4 are feedback diodes. The inverter is Hence, the correct option is (d).

Chapter 04.indd 40 11/9/2015 6:50:55 PM


Chapter 4  DC–AC Converters  |  9.41

2. Match the following  [1995]


Five-marks Questions  (a) 
2E/3
1. A single-phase bridge inverter is fed from a 200 V
E/3
DC supply and is operated at 50 Hz. It is connected
to a load having a resistance of 20 W and an induct- 0 wt
ance of 0.2 H. Draw the load current waveform in
the steady indicating the peak values. [1997] -E/3

Solution: -2E/3

VS = 200 V.  (b) 
f = 50 Hz.
E
R = 20 Ω. 5p/3
0 wt
L = 0.2 mH. 2p/3 2p
-E p

VS
 (c) 
Em
t

0 wt
p 2p

a p+a
−VS
 (d) 
Vm

0 wt
p 2p p 4p 5p 2p
3 3 3 3

 (e) 
2E
 − RT

VS  1 − e 2 L  E
Io =  − RT .
R 2L 
1+ e  0 wt
T
RT 20 × 1
= ⇒ 1.
2 L 50 × 2 × 0.2 (P) Single phase fully controlled AC–DC converter
(Q) Voltage commutated DC–AC chopper (E = in-
200  1 − e −1  put DC voltage)
Io =   = 4.62 A.
20  1 + e +1  (R) Phase voltage of a star connected balanced
three phase load fed from a three-phase ­inverter
VS  − t 
R R
− t
with 180° conduction (input DC voltage = E)
I o (t ) = 1 − e L  − I o e L .
R  (S) Line voltage of a star stepped inverter with
­input DC voltage E
2  −R
t 
R
− t
(T) Three-phase diode bridge rectifier
0=  1 − e L
 − 4. 62 e L
.
20  
  Solution:
0.381× 0.2 A – R     B – S
t= = 3.812 m sec. C – P    D – T
  20
    E – Q

Chapter 04.indd 41 11/9/2015 6:50:57 PM


Chapter 5
AC–AC Converters
 ∴  Range of firing angle α for uncontrollable
One-mark Questions output voltage is 0 ≤ α ≤ 45°.
Hence, the correct option is (a).
1. In the single phase voltage controller circuit shown
in the figure, for what range of triggering angle (α), 2. Thyristor circuits that directly convert poly phase
the output voltage (V0) is not controllable? AC voltages from one frequency to another fre-
[2008] quency are called __________ [1994]
Solution:
AC voltage controller → variable output voltage.
+ 50 Ω
Cycloconverter → variable frequency output.
Vs ∼ Vo
− Thyristor circuits that directly convert polyphase
j5.0 Ω AC voltages from one frequency to another fre-
quency are called CYCLOCONVERTERS.
(a) 0° < α < 45° (b) 45° < α < 135°
(c) 90° < α < 180° (d) 135° < α < 180° Two-marks Questions
Solution: (a)
1. A single-phase SCR based AC regulator is feeding
power to a load consisting of 5 W resistance and
+ 50 Ω
16 mH inductance. The input supply is 230 V, 50 Hz
Vs ∼ Vo AC. The maximum firing angle at which the volt-
− age across the device becomes zero all throughout
j50 Ω
and the rms value of current through SCR, under
this operating condition, are [2014-S2]
Uncontrollable output voltage is obtained when (a) 30° and 46 A (b) 30° and 23 A
firing angle of the above circuit satisfies the condi- (c) 45° and 23 A (d) 45° and 32 A
tion shown below: Solution:
α < φ, R = 5 Ω L = 16 mH. Vsr = 230 V f = 50 Hz.
where α = Firing angle.
   ϕ = Circuit phase angle. ⇒ X L = 2π fL = 100π × 16 × 10−3 ≈ 5 Ω.

 ωL  −1  50  −1  ω L 
φ = tan −1   = tan   = 45°.  Load angle φ = tan   = 45°.
 R   50   R 

Chapter 05.indd 42 11/3/2015 12:49:23 PM


Chapter 5  AC–AC Converters  |  9.43

Now, voltage across the device becomes zero all (a) 3968 W (b) 5290 W
throughout when firing angle of the device is equal (c) 7935 W (d) 10580 W
to the load angle.
Solution: (d)
 ∴ α = 45°.
Vsr p
I Load (rms) = . a=
4
   Z 230√2 sin wt ∼ R = 10 Ω

Z = R + X = 5 + 5 = 5 2 Ω.
2 2
L
2 2

230 46 For α < 90°, the output voltage waveform will be


⇒ I Load (rms) = = A.
  5 2 2 of the form shown below.

Since, each SCR of the AC voltage controller circuit


conducts for only one half cycle in a complete time Vm
period, therefore, current through SCR is given by, p+a 2π wt
a p
1 1 46 23
I t1 = × I load (rms) = × = A.
    2 2 2 2
∴ I t1 (max) = 23 A. Vm = 230 2 V.
Vo2
Power dissipated in the circuit = .
2. The TRIAC circuit shown in figure controls the AC R
output power to the resistive load. The peak power When TRIAC is switched on, Vo = Vs.
dissipation in the load is [2004]

a=
p
4
   ⇒ Vo max = Vm = 230 2.
Vm2 (230 2 ) 2
∼ 230√2 sin wt R = 10 Ω ⇒ Pmax = = = 10580 W.
R 10
Hence, the correct option is (d).

Chapter 05.indd 43 11/3/2015 12:49:25 PM


9.44 | Power Electronics and Drives

⇒ At ωt = α , io = 0 A.
Five-marks Questions
Now, when thyristor starts conducting, circuit can
1. A single phase voltage source of magnitude Vs and be redrawn as shown below:
frequency W (rad/sec) is connected to an induct-
ance L through an anti-parallel back-to-back + +
pair of thyristors. The forward and reverse con- V m sin wt ∼ Vo
π
ducting thyrsitors are fired at an angle of α >  − −
2
from the positive going and negative going zero
crossings of the supply voltage respectively, in L dio
⇒ Vm sin ωt − = 0 [Apply KVL].
each cycle. Obtain an expression for the induc- dt
tor current in each cycle for a given value of α. dio Vm
The voltage drop across the tyristors when either ⇒ = sin ωt.
of them is in conduction, may be assumed to be dt L
negligible [2000] Vm
⇒ sin ωt dt.
dio =
Solution: L
Integrating both sides, we get
T1
Vm
+ + ∫ di o =
L ∫
sin ωt dt.

V m sinwt ∼ Vo Vm
T2 ⇒ io = − cos ωt + K .
− − ωL
The output voltage waveform for the above circuit 
K can be determined by applying initial condition, i.e.,
is shown below: At ωt = α, io = 0 A.
Vo
Vm
⇒ K= cos α .
wt ωL
Vm
for a ³ 90° ∴ io = [cosα − cos ωt ].
ωL
At the instant thyristor is fired, current through the
circuit is zero.

Chapter 05_Five Marks.indd 44 11/9/2015 6:52:44 PM


Chapter 6
Fundamentals of Drives
v
One-mark Questions rated V

1. The speed of a 3-phase, 440 V, 50 Hz induction 50


motor is to be controlled over a wide range from
f
zero speed to 1.5 time the rated speed using a
3-phase voltage source inverter. It is desired to Hence, the correct option is (b).
keep the flux in the machine constant torque
2. The speed torque regimes in a DC motor and the
region by controlling the terminal voltage as the
control methods suitable for the same are given
frequency changes. The inverter output voltage
respectively in List-II and List-I. [2003]
versus frequency characteristic should be
 [2006] List-I List-II
A. Field Control 1. Below base speed
(a) v (b) v B. Armature Control 2. Above base speed
3. Above base torque
4. Below base torque
(a) A-l, B-3 (b) A-2, B-l
f f (c) A-2, B-3 (d) A-l, A-4
50 50
Solution: (b)
(c) v (d) v Field control – Above base speed.
Armature control – Below base speed.
Hence, the correct option is (b).
3. A three phase semiconverter feeds the armature
f f of separately excited DC motor, supplying a non-
50 50
zero torque, for steady state operation, the motor
armature current is found to drop to zero at certain
Solution: (b) instances of time. At such instances, the voltage
If induction motor flux is given by the relationship assumes a value that is [2000]
V
φα . (a) equal is the instantaneous value of the AC
f phase voltage
Now, If f is constant, then above relation becomes (b) equal to the instantaneous value of the motor
Vα f. back emf
Therefore, voltage and frequency have a linear (c) arbitary
­relationship. (d) zero

Chapter 06.indd 45 11/9/2015 7:01:11 PM


9.46 | Power Electronics and Drives

Solution: (b) ⇒ I a = 0.5 × I rated = 10 A.


A DC motor can also be represented as a RLE load
Vt = E + I a Ra .
as shown below.
⇒ Vt = 130 + 10 (1).
= 140 V.
R
For step down chopper, Vt = dVs.

L 140
⇒ α= = 0.7.
200
E Hence, the correct option is (d).
2. A 220 V, 20 A, 1000 rpm, separately excited DC
Therefore, whenever load current is zero, the out-
motor has an armature resistance of 2.5 W. The
put voltage or load voltage becomes equal to E.
motor is controlled by a step down chopper with
Hence, the correct option is (b). a frequency of 1 kHz. The input DC voltage to the
chopper is 250 V. The duty cycle of the chopper for
Two-marks Questions the motor to operate at a speed of 600 rpm deliver-
ing the rated torque will be [2008]
1. The separately excited DC motor in the figure (a) 0.518 (b) 0.608
below has a rated armature current of 20 A and (c) 0.852 (d) 0.902
a rated armature voltage of 150 V. An ideal chop- Solution: (b)
per switching at 5 kHz is used to control the arma- Given: 220 V, 20 A, 1000 rpm.
ture voltage. If La = 0.1 mH, Ra = 1 W neglecting
Ra = 2.5 Ω.
armature reaction, the duty ratio of the chopper to
obtain 50% of the rated torque at the rated speed Step down chopper frequency = 1 kHz.
and the rated field current is [2013] Input DC voltage = 250 V.
Speed = 600 rpm.
Eb1 = 220 − 20 × 2.5 = 170 V (At rated operation).
At 600 rpm, back emf is given by,
+ La, R a
200 V
− N2
Eb 2 = × Eb1
N1
600
= × 170 = 102 V.
(a) 0.4 (b) 0.5 1000
(c) 0.6 (d) 0.7
∴ DVDC = 102 + 20 × 2.5,
Solution: (d)
where D is duty cycle.
Given: Ia = 20 A, V(Armature voltage) = 150 V.
⇒ D = 0.608.
Chopper switching frequency = 5 kHz.
Hence, the correct option is (b).
La = 0.1 mH Ra = 1 Ω.
3. A 220 V, 1400 rpm, 40 A separately excited DC
50% of rated torque operation at rated speed and motor has an armature resistance of 0.4 W. The
field current. motor is fed from a single phase circulating cur-
Vt = E + I a Ra . rent dual converter with an input AC line voltage
of 220 V (rms). The approximate firing angles of
150 = E + 20 (1). the dual converter for motoring operation at 50%
⇒ E = 130 V. of rated torque and 1000 rpm will be [2008]
(a) 43°, 137° (b) 43°, 47°
Now, τ = 0.5τ rated . (c) 39°, 141° (d) 39°, 51°

Chapter 06.indd 46 11/9/2015 7:01:13 PM


Chapter 6  Fundamentals of Drives  |  9.47

Solution: (c) If speed becomes half, then V=


t E= 220 V.
Given: DC motor: 220 V, 1400 rpm, 40 A.
P = 15 kW.
Ra = 0.4 Ω.
P = I o E.
Input AC line voltage = 220 V. ∴ I o = 34.09 A.
Motor operation at 50% of rated torque and 1000
rpm. Is(Source rms current)
We know that Eb = Vt − I a Ra . 2 2
= Io
= 34.09 × = 27.83 A.
= N1 1400
= rpm N 2 1000 rpm. 3 3
= I a1 40
= A I a 2 20 A. \  Supply power factor at half rated speed, is

given by
2Vm
Vt = cos α . 220 × 34.09
π cos φ = pf = = 0.354.
3 × 440 × 27.83
N2
Eb 2 = × Eb1 . Hence, the correct option is (a).
N1
5. An electric motor, developing a starting torque of
Eb1 = V − I a1 Ra 15 Nm, starts with a load torque of 7 Nm on its
= 220 − 40 × 0.4 shaft. If the acceleration at start is 2 rad/sec2, the
= 204 V. moment of inertia of the systems must be (neglect-
ing viscous and Coulomb friction) [2005]
1000 (a) 0.25 kg m2 (b) 0.25 Nm2
Eb2 = × 204 = 145.71 V.
1400 (c) 4 kg m2 (d) 4 Nm2
2Vm Solution: (c)
⇒ cos α1 = 145.71 + (20)(0.4) = 153.71. Given: Starting torque = 15 Nm.
π
⇒ cos α1 = 0.776. T (Load torque) = 7 Nm.
⇒ α1 = 39°. Acceleration at start = 2 rad/sec2.
In dual converter, ∴ Te = 15 Nm T = 7 Nm.

α1 + α 2 = 180°. dω
= 2 rad/sec 2 .
⇒ α 2 = 141°. dt
d ωm
Hence, the correct option is (c). ⇒ Te = T + J .
dt
4. A three-phase, 440 V, 50 Hz AC mains fed thy-
⇒ J (2) = 15 − 7
ristor bridge is feeding a 440 V DC, 15 kW,
1500 rpm separately excited DC motor with a rip- ⇒ J = 4 kg m 2
ple free continuous current in the DC link under
Hence, the correct option is (c).
all operating conditions, neglecting the losses,
the power factor of the AC mains at half the rated 6. A single-phase half-controlled rectifier is driving
speed is [2007] a separately excited DC motor. The DC motor has
(a) 0.354 (b) 0.372 a back emf constant of 0.25 V/rpm. The armature
(c) 0.90 (d) 0.955 current is 5 A without any ripple. The armature
resistance is 2 W. The converter is working from a
Solution: (a)
230 V, single phase AC source with a firing angle
Given: 3φ , 440 V, 50 Hz AC supply.
of 30°. Under this operating condition, the speed
DC motor: 440 V, 15 kW, 1500 rpm.
of the motor will be [2004]
If losses are neglected, then armature resistance (a) 339 rpm (b) 359 rpm
Ra = 0 and V=t E= 440 V. (c) 366 rpm (d) 386 rpm

Chapter 06.indd 47 11/9/2015 7:01:16 PM


9.48 | Power Electronics and Drives

Solution: (c)
Given  1− φ  half controlled rectifier DC motor back
emf= constant  0= .5 V/rpm K b . +20 Nm

Ia (Armature current) = 5 A.
torque
t
0.5 sec
Ra (Armature resistance) = 2 Ω.
Source: 1 - ϕ, 230 V.
+ 500 rpm
α = 30°.
speed t
For a separately excited DC motor Ea = KbN and −1500 rpm
Ea = V - IaRa
V
= m (1 + cos α ) − 5(2) (a) 0,048 kg m2 (b) 0.064 kg m2
    π
(c) 0.096 kg m2 (d) 0.128 kg m2
 Vm  Solution: (c)
∵ V0 = π (1 + cos α )for a 1 − φ semiconverter 
 
d ωm
230 2 Te = TL + J .
= (1 + cos 30°) − 10. dt
  π
Since, motor is operated at no load,
 ⇒  E = 183.20 V. \  TL = 0 Nm.
Now, Ea = KbN.
d ωm
E 183.2 ⇒ Te = J .
∴ N= a = = 366.4 rpm. dt
Kb 0.5
    2π [500 − (−1500)] 
∴ 40 = J  .
Hence, the correct option is (c).  60 0.5 
7. A variable speed drive rated for 1500 rpm, 40 Nm 40
is reversing under no load. Figure shows the revers- ⇒ J= = 0.096 kg m 2 .
  418.87
ing torque and the speed during the transient. The
moment of inertia of the drive is [2004] Hence, the correct option is (c).

Chapter 06.indd 48 11/9/2015 7:01:18 PM


Chapter 6  Fundamentals of Drives  |  9.49

(c) rms value of fundamental input current


Five-marks Questions
6 6
= Ia = × 56 = 43.663 A.
1. A separately excited DC motor is controlled by π π
varying its armature voltage using a single-phase 2. In a speed controlled DC motor drive, the load
full-converter bridge as shown in figure. The filed torque is 40 Nm. At time t = 0, the operation is
current is kept constant at the rated. The motor has under steady state and speed is 500 rpm under
an armature resistance of 0.2 W, and the motor this condition at t = 0, the generated torque is
voltage constant is 2.5 V/(rad/sec). The motor is instantly increased to 100 Nm, the inertia of
driving a mechanical load having a constant torque the drive is 0.01 Nm sec2/rad. The friction is
of 140 Nm. The triggering angle of the converter ­negligible [1998]
is 60°. The armature current can be assumed to be (a) write down the differential equation governing
continuous and ripple free [2001] the speed of the drive for t > 0
(a) calculate the motor armature current (b) evaluate the time taken for the speed to reach
(b) evaluate the motor speed in rad/sec 1000 rpm.
(c) calculate the rms value of the fundamental Solution:
component of the input current to the bridge Given: TL = 40 Nm.
Ia Speed = 500 rpm.
Steady state operation, J = 0.01 Nm sec2/rad.
Ra = 0.2 Ω At steady state Te = TL.
d ωm
⇒ 100 = J + 40.
250 V dt
(rms) ir constant
d ωm 60
⇒ = .
50 Hz dt 0.01
+ −
⇒ d ωm = 6000 dt.
Solution:
Given data: ra − 0.2 Ω Te = 140 Nm. Integrating on both sides, we get,
⇒  ωm = 6000t + A(1)
V V −s
Km = 2.5 (or ) . At  t = 0, N = 500 rpm.
rad/sec rad
α1 = 60°, Vs = 250 V. 2π × 500
ωm =
60
For full wave rectifier the average value of output 100π
2V = rad/s.
voltage Vo = m cos α , 6
π
100 π
(a) For separately excited DC motors Te = K m I a . ∴ ωm = 6000 t + . (2)
6
140 140
= Ia ⇒ Ia = = 56 A. Now, N = 1000 rpm,
Km 2.5
1000 × 2π 200π
(b) Motor back emf E = Kmωm. ωp = = .
60 6
Vo = Ea + I a ra .
200π 100π
2 × 250 2 = 600 t + .
⇒ cos 60°. 6 6
π
= 2.5 × ωm + 56 × 0.2. π
⇒ T= sec.
ωm = 40.53 rad/sec. 360

Chapter 06.indd 49 11/9/2015 7:01:21 PM


9.50 | Power Electronics and Drives

3. A DC motor driver from a fully controlled 3-phase Solution:


converter shown in figure draws a DC current of The conduction period of thyristor is 120°.
100 A with negligible ripple [1998] More: In which place the suffix a exists. (i.e., Vab,
(a) sketch the AC line current Is for one cycle Vac, Vca, Vba) in output waveform that time thyris-
(b) determine the 3rd and 5th harmonic compo- tor A is conducting with a period of 120° and the
nent of the line current as a percentage of fun- remaining time current through thyristor is zero.
damental current
From Vab → current is positive direction.
100 V Vba → current is in negative direction.
(b) The line current iA shown in below fig be ex-
iA pressed in Fourier series as

4I  nπ 
3-phase iAn = ∑
n =1, 315 nπ
cos 
 6
 sin(nωt − nα ).

input

VoVt

a
Vcb Vab Vac Vbc Vba Vca Vcb Vab Vac Vbc Vba Vca Vcb Vab

0 wt

ia

Ia
0 wt
a = 30°
iA

120°
1
6 Ia 4.3 − 4.5
0 wt
1.6 + 1.2 + p −Ia p
3
3
iTI 120

120° 120°
Ia Ia
0 wt
2p

rms value of the nth harmonic line current is given 2 2Ia π 6


by iS 1 = cos ⇒
30.
π 6 π
4I a nπ 2 2 I a nπ rms value of third harmonic current
iS = cos = ⋅ cos .
2nπ 6 nπ 6 2 2Ia  3π 
iS 3 = cos   ⇒ 0.
rms value of fundamental current π  6 

Chapter 06.indd 50 11/9/2015 7:01:22 PM


Chapter 6  Fundamentals of Drives  |  9.51

rms value of fifth harmonic current ⇒ K mωm = −10(1) + 220 = 210 V.


2 2I a − 6 210 × 60
iS 5 = cos(150°) = Ia . ⇒ Km = = 1.377 V − s/rad.
5π 5π 2π × 1500
From above third harmonic current as a percentage α1 = 30°, Te = 5 Nm.
of fundamental current = 0% and fifth harmonic
current as a percentage to fundament current, For a torque of 5 Nm, armature current
5
− 6 Ia =Ia = 3.74 A.
IS5 1.337
× 100 ⇒ 5π × 100 = −20%.
I S1 6 The equation for the operation of converter
Ia motor is
π
4. A 220 V, 1500 rpm, 10 A separately excited DC Vo = Vt = Ea + I aVa .
motor has an armature resistance of 1 W. It is fed
2Vm
from a single phase fully controlled bridge recti- cos α = K mωm + I a Ra .
fier with an AC source voltage of 230 V, 50 Hz. π
Assuming continuous load current, compute ⇒ ωm = 131.31 rad/s.
(a) motor speed at the firing angle of 30° and
∴ N = 1253.92 rpm.
torque of 5 Nm.
(b) developed torque at the firing angle of 45° and α = 45°, N = 1000 rpm, Te = ?
(b)
speed of 1000 rpm.  [1996]
Solution: 2 2 × 230
⇒ cos 45°
Given: 220 V, 1500 rpm, 10 A separately excited π
DC motor. 2π × 1000
= 1.337 × + I a × 1.
Ra = 1Ω. 60
1 - ϕ fully controlled rectifier (230 V, 50 Hz). 146.4 = 140.01 + I a × 1.
(a) Vt = Ea + IaRa. I a = 6.39 A.
 Now, Ea = K mωm . Te = K m I a = 1.337 × 6.39 = 8.543 Nm.

Chapter 06.indd 51 11/9/2015 7:01:24 PM


This page is intentionally left blank.

Chapter 06.indd 52 11/12/2015 11:22:34 AM


This page is intentionally left blank.

Chapter 06.indd 52 11/12/2015 11:22:34 AM


This page is intentionally left blank.

Chapter 06.indd 52 11/12/2015 11:22:34 AM

You might also like